Download as pdf or txt
Download as pdf or txt
You are on page 1of 793

VISIONIAS

www.visionias.in

Test Booklet Series

TEST BOOKLET

GENERAL STUDIES (P) 2024 – Test – 4126


C
Time Allowed: Two Hours Maximum Marks: 200

INSTRUCTIONS

1. IMMEDIATELY AFTER THE COMMENCEMENT OF THE EXAMINATION, YOU SHOULD CHECK THAT THIS BOOKLET
DOES NOT HAVE ANY UNPRINTED OR TURN OR MISSING PAGES OR ITEMS, ETC. IF SO, GET IT REPLACED BY A
COMPLETE TEST BOOKLET.

2. ENCODE CLEARLY THE TEST BOOKLET SERIES A, B, C OR D AS THE CASE MAY BE IN THE APPROPRIATE PLACE IN
THE ANSWER SHEET.

3. You have to enter your Roll Number on the Test Booklet in the Box
provided alongside. Do NOT write anything else on the Test Booklet.

4. This Test Booklet contains 100 items (Questions). Each item is printed in English. Each item comprises four
responses (answers). You will select the response which you want to mark on the Answer Sheet. In case you
feel that there is more than one correct response with you consider the best. In any case, choose ONLY ONE
response for each item.

5. You have to mark all your responses ONLY on the separate Answer Sheet provided. See direction in the
answers sheet.

6. All items carry equal marks. Attempt all items. Your total marks will depend only on the number of correct
responses marked by you in the answer sheet. For every incorrect response 1/3rdof the allotted marks will be
deducted.

7. Before you proceed to mark in the Answer sheet the response to various items in the Test booklet, you have to
fill in some particulars in the answer sheets as per instruction sent to you with your Admission Certificate.

8. After you have completed filling in all responses on the answer sheet and the examination has concluded, you
should hand over to Invigilator only the answer sheet. You are permitted to take away with you the Test
Booklet.

9. Sheet for rough work are appended in the Test Booklet at the end.

DO NOT OPEN THIS BOOKLET UNTIL YOU ARE ASKED TO DO SO


1 www.visionias.in ©Vision IAS

FREE BY KING R QUEEN P [ऋषभ राजपूत]


1. In the context of the basic philosophy of the 4. With reference to Ayushman Bharat-
constitution, which of the following is/are
Pradhan Mantri Jan Arogya Yojana
the functions of a constitution?
1. It provides a set of basic rules that allow (PMJAY), consider the following
for coordination among members of
society. statements:
2. It specifies who has powers to make
1. It is the world’s largest government-
decisions in a society.
3. It sets limits on what a government can funded healthcare program.
impose on its citizens.
Select the correct answer using code given 2. The beneficiaries under the scheme will
below.
be identified on the basis of the Socio-
(a) 1 only
(b) 1 and 2 only Economic Caste Census, 2011.
(c) 2 and 3 only
(d) 1, 2 and 3 3. National Health Authority (NHA) is the

nodal agency for its implementation.


2. 'The claims of the members of the Scheduled
Castes and the Scheduled Tribes shall be Which of the statements given above are
taken into consideration, consistently with
correct?
the maintenance of efficiency of
administration, in the making of (a) 1 and 2 only
appointments to services and posts in
connection with the affairs of the Union or a (b) 2 and 3 only
State'. This provision was provided in the
(c) 1 and 3 only
Indian Constitution under:
(a) Part IV (d) 1, 2 and 3
(b) Part III
(c) Part XVI
(d) Part XVII 5. Consider the following statements regarding

the recently released Global Economic


3. Which of the following are the features of
fundamental rights? Prospects report:
1. They strike a balance between individual
1. It is released by International Monetary
liberty and social control.
2. There are some fundamental rights that Fund.
are available against the arbitrary action
of private individuals. 2. It is released once a year.
3. There are some fundamental rights that
Which of the statements given above is/are
are available only to the citizens.
4. All the fundamental rights are self- correct?
executory.
Select the correct answer using the code (a) 1 only
given below.
(b) 2 only
(a) 1, 2 and 3 only
(b) 2 and 4 only (c) Both 1 and 2
(c) 1 and 3 only
(d) 1, 2, 3 and 4 (d) Neither 1 nor 2
2 www.visionias.in ©Vision IAS

FREE BY KING R QUEEN P [ऋषभ राजपूत]


6. Consider the following pairs: 8. Which one of the following

Schedule Provisions recommendations of the Swaran Singh

1. 4th Schedule : Allocation of seats in Committee is incorporated in the


the Rajya Sabha to the Constitution under Fundamental duties?
states and the Union (a) Empower Parliament to impose penalty
Territories or punishment for the non-compliance
2. 6th Schedule : Provisions relating to with any of the fundamental duties.
the administration and (b) Duty to defend the country and render
control of Scheduled
national service when called upon to do
Areas
so.
3. 10th Schedule : Provisions relating to
(c) Duty to pay taxes should be a
disqualification of the
Fundamental duty of the citizens.
members of the
(d) No law imposing penalty or punishment
Parliament
for the non-compliance with any of the
Which of the pairs given above is/are
fundamental duties shall be called in
correctly matched?
question in any court.
(a) 1 only

(b) 2 and 3 only


9. Which of the following is not a function of
(c) 1 and 3 only
the Central Bureau of Investigation (CBI)?
(d) 1, 2 and 3
(a) Investigating cases of corruption,

bribery, and misconduct of Central


7. Which of the following is/are Ozone
government employees.
Depleting Substances (ODS)?
(b) Probe terror attacks including bomb
1. Methyl Bromide
blasts, hijacking of aircraft and ships,
2. Hydrobromoflurocarbons (HBFCs)

3. Carbon tetrachloride attacks on nuclear installations, and use

4. Methyl chloroform of weapons of mass destruction.

Select the correct answer using the code (c) Investigating serious crimes, having

given below. national and international ramifications,

(a) 1, 2 and 3 committed by organized gangs of

(b) 2 and 4 only professional criminals.

(c) 4 only (d) Maintaining crime statistics and

(d) 1, 2, 3 and 4 disseminating criminal information.


3 www.visionias.in ©Vision IAS

FREE BY KING R QUEEN P [ऋषभ राजपूत]


10. With reference to the Universal Serial Bus 13. With reference to the seventh schedule of the
(USB) Type-C connector in news recently, Indian constitution, consider the following
consider the following statements: pairs:
Subject List
1. It is a double-sided connector with
1. Banking : Concurrent
reversibility.
2. Agriculture : State
2. It is not backward compatible with
3. Trade Unions : Union
previous USB Types. How many pairs given above are correctly
Which of the statements given above is/are matched?
correct? (a) None of the pairs
(a) 1 only (b) Only one pair
(b) 2 only (c) Only two pairs
(d) All three pairs
(c) Both 1 and 2
(d) Neither 1 nor 2
14. In which of the following judgments,
Supreme Court held that the harmony and
11. In the context of historical underpinnings of
balance between Fundamental rights and
the Indian constitution, which of the
Directive principles of State policy is part of
following Acts introduced ordinances for the basic structure?
first time? (a) Bhim Singhji Case
(a) Indian Councils Act of 1892 (b) Indira Sawhney Case
(b) Indian Councils Act of 1861 (c) Minerva Mills case

(c) Government of India Act of 1919 (d) Kihoto Hollohon Case

(d) Government of India Act of 1909


15. With reference to 'Bose-Einstein
Condensate', often termed as the fifth state
12. The directive principles of State policy laid
of matter, which of the following statements
down in Part IV of the Constitution cannot in
is/are correct?
any way override or abridge the fundamental 1. Its existence was posited by Albert
rights guaranteed by Part III. On the other Einstein and Indian physicist Jagadish
hand, they have to conform to and run as Chandra Bose.
subsidiaries to the fundamental rights laid 2. It is formed when atoms of certain
down in Part III. But, the Fundamental elements are heated to near critical
temperature.
Rights could be amended by the Parliament
3. It can be used in creation of atomic
by enacting constitutional amendment acts.
lasers and atomic clocks.
In which of the following cases did the
Select the correct answer using the code
Supreme Court hold this opinion? given below.
(a) Champakam Dorairajan case (1951) (a) 1 and 2 only
(b) Golaknath case (1967) (b) 3 only
(c) Keshavananda Bharti case (1973) (c) 2 and 3 only
(d) Minerva Mills case (1980) (d) 1, 2 and 3
4 www.visionias.in ©Vision IAS

FREE BY KING R QUEEN P [ऋषभ राजपूत]


16. Consider the following statements with 18. Which of the following has the power to
respect to freedom from taxation for the admit a new state into the Union of India?
promotion of a religion: (a) Parliament
1. State should not spend the money (b) President
collected by way of tax for the (c) Union Government
promotion of any particular religion. (d) Supreme Court
2. This provision prohibits the levy of a tax
and fee to control the secular 19. Which of the following are the advantages of
administration of religious institutions. parliamentary democracy?
Which of the statements given above is/are 1. Responsible Government
correct? 2. Wide Representation
(a) 1 only 3. Strict Separation of powers
(b) 2 only
4. Harmony Between Legislature and
(c) Both 1 and 2
Executive
(d) Neither 1 nor 2
Select the correct answer using the code

given below.
17. Which of the following statements is/are
(a) 1, 2 and 3 only
correct with regard to the consequences of
(b) 2, 3, and 4 only
the proclamation of a Financial Emergency?
(c) 1 and 4 only
1. Centre acquires full control over the
(d) 1, 2 and 4 only
states in financial matters.
2. President may reserve all money bills or
20. Which of the following provisions of the
other financial bills for consideration
constitution ensure political equality?
after they are passed by the legislature of
1. Elections based on adult suffrage.
the state.
2. Abolition of titles.
3. President may issue directions for the
reduction of salaries and allowances of 3. No ineligibility for inclusion in electoral

the judges of the Supreme Court and the rolls on grounds of religion, race, caste,

high court. or sex.

Select the correct answer using the code Select the correct answer using the code

given below. given below.

(a) 1 only (a) 1 and 3 only

(b) 1 and 2 only (b) 1 and 2 only


(c) 2 and 3 only (c) 1, 2 and 3
(d) 1, 2 and 3 (d) 3 only
5 www.visionias.in ©Vision IAS

FREE BY KING R QUEEN P [ऋषभ राजपूत]


21. Consider the following statements regarding 24. Which one of the following rights abolished
the amendment procedure of the constitution untouchability and prohibits its practice as a
of India:
1. A constitutional amendment bill cannot fundamental right?
be introduced by the nominated (a) Right to constitutional remedies
members of the Lok sabha.
(b) Right to equality
2. A constitutional amendment bill that
seeks to amend the federal provisions of (c) Right to freedom
the Constitution must be ratified by the (d) Right against exploitation
Legislatures of half of the States by a
special majority.
25. Consider the following statements regarding
Which of the statements given above is/are
correct? the State Human Rights Commission
(a) 1 only (SHRC):
(b) 2 only
1. The commission has no power to punish
(c) Both 1 and 2
(d) Neither 1 nor 2 the violators of human rights, nor to
award any relief including monetary
22. Consider the following statements with
relief to the victim.
respect to the Right to Equality:
1. Equality before the law is a positive 2. The commission submits its annual or
concept which means the absence of any special reports to the governor of the
special privileges in favour of any
state.
person.
2. Where equals and unequals are treated Which of the statements given above is/are
differently, Article 14 does not apply. correct?
Which of the statements given above is/are
(a) 1 only
correct?
(a) 1 only (b) 2 only
(b) 2 only (c) Both 1 and 2
(c) Both 1 and 2
(d) Neither 1 nor 2
(d) Neither 1 nor 2

23. Which of the following are the unitary 26. Which one of the following is the best
features of Indian Constitution? description of 'Tidal Disruption Event', that
1. Single constitution
was seen in the news recently?
2. Emergency Provisions
3. All-India Services (a) It is a lapse in time interval between two
4. Integrated Election Machinery diurnal tides.
5. Appointment of Governor
(b) It is a reversal in magnetism of Earth.
Select the correct answer using the code
given below. (c) It is a gravity anomaly on surface of
(a) 1, 2, 3 and 4 only moon.
(b) 2, 4 and 5 only
(c) 1, 3, 4 and 5 only (d) It is a destruction of a star by a black
(d) 1, 2, 3, 4 and 5 hole.

6 www.visionias.in ©Vision IAS

FREE BY KING R QUEEN P [ऋषभ राजपूत]


27. Articles 358 and 359 of the Indian 30. In the context of the Indian Constitution,
Constitution deal with which of the consider the following statements regarding
following situations?
fundamental duties:
(a) Proclamation of National Emergency
1. Fundamental Duties are confined to
under Article 352
(b) Financial emergency under Article 360 Indian citizens only.

(c) President's rule under Article 356 2. Constitution does not provide any legal
(d) President's rule under Article 365 sanctions against their violation.

Which of the statements given above is/are


28. Which of the following services, at present,
correct?
are regulated under Information Technology
(Intermediary Guidelines and Digital Media (a) 1 only

Ethics Code) Rules, 2021? (b) 2 only


1. Social media sites (c) Both 1 and 2
2. Over-the-top (OTT) streaming services
(d) Neither 1 nor 2
3. Online gaming services
4. Digital news sites
31. Which of the following are the Fundamental
Select the correct answer using the code
Duties under part IVA of the Constitution of
given below.
(a) 1 and 2 only India?

(b) 3 and 4 only 1. To develop the spirit of inquiry and


(c) 1, 2 and 4 only
reform.
(d) 1, 2, 3 and 4
2. To respect the National Flag and the

29. Consider the following statements: National Anthem.

1. In the USA, a citizen by birth, as well as 3. To strive towards excellence in all


a naturalized citizen, is eligible for the spheres of individual and collective
office of President.
activity.
2. In India, only a citizen by birth and not a
4. To cast a vote.
naturalized citizen is eligible for the
office of President. Select the correct answer using the code

Which of the statements given above given below.


is/are not correct? (a) 1 and 2 only
(a) 1 only
(b) 1, 2 and 3 only
(b) 2 only
(c) 1 and 4 only
(c) Both 1 and 2
(d) Neither 1 nor 2 (d) 2 and 4 only

7 www.visionias.in ©Vision IAS

FREE BY KING R QUEEN P [ऋषभ राजपूत]


32. Consider the following statements: 35. Which of the following Directive Principles
1. The proclamation of national emergency
of State Policy (DPSPs) were not added by
must be approved by both Houses of
Parliament within two months from the the 42nd Constitutional Amendment Act,
date of its issue.
1976?
2. If approved by both houses of
Parliament, the emergency continues for 1. To provide early childhood care and
six months and can be extended to an
education for all children until they
indefinite period with the approval of the
Parliament every six months. complete the age of six years.
Which of the statements given above is/are
2. To take steps to secure the participation
correct?
(a) 1 only of workers in the management of
(b) 2 only
industries.
(c) Both 1 and 2
(d) Neither 1 nor 2 3. To minimise inequalities in income,

status, facilities, and opportunities.


33. In the context of evolution of constitution of
India, which of the following are part of the Select the correct answer using the code
basic structure of the constitution?
given below.
1. Republic and democratic form of
government (a) 1 and 2 only
2. Presidential form of government
(b) 2 and 3 only
3. Secular character of the constitution
4. Separation of powers (c) 1 and 3 only
Select the correct answer using the code
(d) 1, 2 and 3
given below.
(a) 1, 2 and 3 only
(b) 1, 3 and 4 only 36. Consider the following statements regarding
(c) 2 and 4 only
(d) 1, 2, 3 and 4 the Government of India Act of 1935:

1. It provided for the establishment of an


34. Consider the following pairs:
Feature Borrowed from All-India Federation consisting of
1. Directive : Irish provinces and princely states as units.
principles Constitution
2. Fundamental : French 2. It introduced bicameralism in all the
rights Constitution provinces.
3. Joint sitting of : Australian
two houses of Constitution Which of the statements given above is/are
parliament correct?
Which of the pairs given above is/are
correctly matched? (a) 1 only
(a) 1 only (b) 2 only
(b) 2 and 3 only
(c) 1 and 3 only (c) Both 1 and 2
(d) 1, 2 and 3 (d) Neither 1 nor 2

8 www.visionias.in ©Vision IAS

FREE BY KING R QUEEN P [ऋषभ राजपूत]


37. With reference to 'constitutional torts' in 39. Consider the following provisions under the
Directive Principles of State Policy as
India, consider the following statements:
enshrined in the Indian Constitution:
1. In India, there is no legislation, which 1. Securing opportunities for the healthy
governs the liability of the State for the development of children
2. Organising agriculture on modern and
torts committed by its servants.
scientific lines
2. A court of law can award monetary 3. Prohibition of slaughter of cows
compensation to the victim in such 4. Promotion of Cooperative societies
Which of the directive principles given
cases.
above are based on Liberal-Intellectual
Which of the statements given above is/are Principles?
correct? (a) 1 and 2 only
(b) 2 only
(a) 1 only
(c) 2 and 3 only
(b) 2 only (d) 1 and 4 only
(c) Both 1 and 2
40. India was a pioneer when it comes to giving
(d) Neither 1 nor 2
voting rights to the people. In this context,
which of the following countries gave
38. In the context of various political systems universal adult franchises after India?
1. United States of America
prevailing in the world, which of the
2. Japan
following are the characteristics of a 3. France
democracy? Select the correct answer using the code
given below.
1. In a democracy, the final decision-
(a) 1 only
making power must rest with those (b) 1 and 3 only
elected by the people. (c) 2 and 3 only
(d) 1, 2 and 3
2. The ruling party in power must have a

fair chance of losing in an election. 41. Consider the following statements with
3. Each adult citizen must have one vote respect to Constituent Assembly:
1. Whenever the Constituent Assembly met
and each vote must have one value.
as the legislative body, it was chaired by
4. The Government is not limited by laws Dr. Rajendra Prasad.
or regulations. 2. The Constituent Assembly adopted the
national anthem and the national song in
Select the correct answer using the code
1947.
given below. Which of the statements given above is/are
correct?
(a) 1, 2 and 4 only
(a) 1 only
(b) 2, 3 and 4 only (b) 2 only
(c) 1, 2 and 3 only (c) Both 1 and 2
(d) Neither 1 nor 2
(d) 1, 3 and 4 only
9 www.visionias.in ©Vision IAS

FREE BY KING R QUEEN P [ऋषभ राजपूत]


42. Consider the following statements regarding 45. Which one of the following statements is
National Commission for Scheduled Tribes correct about the Right to Education?
(a) It was added after the passage of the
(NCST):
Right to Education (RTE) act.
1. The conditions of service and tenure of
(b) There was no provision in the
office of members of the commission are constitution for free and compulsory
determined by the Parliament. education for children before the 86th
2. The original constitution did not provide amendment.
(c) It includes that the state shall endeavour
for a separate National Commission for
to provide education for all children
Scheduled Tribes.
until they complete the age of six years.
Which of the statements given above is/are (d) This provision makes only elementary
correct? education and not higher education a
(a) 1 only fundamental right.

(b) 2 only
46. 'It is one of the world’s most active
(c) Both 1 and 2
volcanoes which has erupted again
(d) Neither 1 nor 2 recently. It is located near to world's largest
active volcano, Mauna Loa.'
43. Which one of the following designates a Which of the following volcanoes is being
bank as Domestic Systemically Important described in the passage given above?
(a) Mount Kilauea
Bank (D-SIB) in India?
(b) Mount Stromboli
(a) Ministry of Finance
(c) Mount Vesuvius
(b) Reserve Bank of India (d) Mount Etna
(c) Indian Banks' Association
(d) Securities and Exchange Board of India 47. With reference to the proclamation of
national emergency under Article 352
imposed till now, consider the following
44. Which of the following rights are a part of
statements:
the Right to Protection of Life and Personal
1. So far, a national emergency has been
Liberty? proclaimed two times.
1. Right not to be driven out of a state 2. The first proclamation of national
2. Right to travel abroad emergency was issued in October 1962
on account of Chinese aggression in the
3. Freedom of silence
NEFA (North-East Frontier Agency)
4. Right to the road in hilly areas
3. The second proclamation of national
Select the correct answer using the code emergency was made in 1975.
given below. Which of the statements given above is/are
(a) 3 and 4 only correct?

(b) 1 and 3 only (a) 1 only


(b) 2 and 3 only
(c) 1, 2 and 4 only
(c) 2 only
(d) 1, 2, 3 and 4 (d) 1 and 2 only
10 www.visionias.in ©Vision IAS

FREE BY KING R QUEEN P [ऋषभ राजपूत]


48. Recently, Global Risks Report 2023, an 51. The PRANA portal was continuously in the
annual report based on Global Risks news in recent times. Which of the following
Perception Survey was released by statements best explains its objective?
(a) World Bank (a) It is a portal for monitoring of
(b) World Economic Forum implementation of National Clean Air
(c) United Nations Office for Disaster Risk Programme.
Reduction (b) It will provide online education to far
(d) Germanwatch flung areas where physical infrastructure
of education is not present.
49. Consider the following statements with (c) It will provide the Gram Panchayats
respect to terms contained in the Preamble: with a single interface to prepare and
1. The term republic has been taken from implement their Gram Panchayat
German Constitution. Development Plan.
2. The ideals of justice in the preamble (d) It will ensure hassle-free disbursement
have been adopted from Soviet of pension to retired state government
Constitution. employees.
Which of the statements given above is/are
correct? 52. Under the Constitution of India, which of the
(a) 1 only following rights and privileges are conferred
(b) 2 only on the citizens of India and denied to aliens?
(c) Both 1 and 2 1. Right against discrimination on grounds
(d) Neither 1 nor 2 of religion, race, caste, sex, or place of
birth
50. Which of the following bodies/institutions 2. Right to freedom of speech and
is/are audited totally and directly by the expression, assembly, association,
CAG? movement, residence, and profession
1. Reserve Bank of India 3. Right to serve as a judge in the higher
2. Oil and Natural Gas Commission judiciary
3. Life Insurance Corporation of India 4. Right to contest for membership of the
4. State Bank of India Parliament and the state legislature
Select the correct answer using the code Select the correct answer using the code
given below. given below.
(a) 2 only (a) 2 and 3 only
(b) 2 and 3 only (b) 2 and 4 only
(c) 3 and 4 only (c) 1 and 3 only
(d) 1, 3 and 4 only (d) 1, 2, 3 and 4
11 www.visionias.in ©Vision IAS

FREE BY KING R QUEEN P [ऋषभ राजपूत]


53. On a particular day, 150 members are 56. Consider the following statements regarding
present in Rajya Sabha at the time of voting. the loss of citizenship under the Citizenship
In this scenario, a constitutional amendment
Act (1955):
bill, under Article 368 of the constitution
(a) can be passed with the support of only 1. When a person renounces his Indian
75 members. citizenship, every minor child of that
(b) cannot be passed with the given strength person also loses Indian citizenship.
of members present for voting.
2. When an Indian citizen voluntarily
(c) can be passed with a support from
minimum 100 members. acquires citizenship of another country,

(d) cannot be passed without support of his Indian citizenship automatically


atleast 123 members. terminates.
Which of the statements given above is/are
54. In the context Constituent Assembly of
correct?
India, consider the following pairs:
Committee Chairman (a) 1 only
1.Union Powers : Jawaharlal (b) 2 only
Committee Nehru
(c) Both 1 and 2
2. Steering Committee : Sardar Patel
(d) Neither 1 nor 2
3.Provincial : Dr. Rajendra
Constitution Prasad
Committee 57. Consider the following statements with
Which of the pairs given above is/are respect to the State Human Rights
correctly matched?
Commission:
(a) 1 only
1. It can inquire into violation of human
(b) 1 and 2 only
(c) 2 and 3 only rights only in respect of subjects
(d) 1, 2 and 3 mentioned in the State List and the
Concurrent List.
55. Consider the following statements with
2. If any case is already being inquired into
respect to Cultural and Educational rights
guaranteed by the Indian Constitution: by the National Human Rights
1. The scope of Article 29 is not restricted Commission, then the State Human
to minorities only.
Rights Commission does not inquire into
2. All minorities shall have the right to
that case.
establish and administer educational
institutions of their choice. Which of the statements given above is/are
Which of the statements given above is/are correct?
correct? (a) 1 only
(a) 1 only
(b) 2 only
(b) 2 only
(c) Both 1 and 2 (c) Both 1 and 2
(d) Neither 1 nor 2 (d) Neither 1 nor 2
12 www.visionias.in ©Vision IAS

FREE BY KING R QUEEN P [ऋषभ राजपूत]


58. Which of the following statements is/are 61. With reference to Bureau of Indian
correct regarding the Vice Chairperson of Standards (BIS), consider the following
NITI Aayog? statements:
1. He is appointed by the Prime Minister. 1. It is a statutory body established under
2. He enjoys the rank of a Minister of the Bureau of Indian Standards Act,
State. 1986.
3. He chairs the meetings of Regional 2. It is regulated under the Ministry of
Councils. Consumer Affairs, Food & Public
Select the correct answer using the code Distribution,
given below. Which of the statements given above is/are
(a) 2 only correct?
(b) 2 and 3 only (a) 1 only
(c) 1 only (b) 2 only
(d) 1, 2 and 3 (c) Both 1 and 2
(d) Neither 1 nor 2
59. Which of the following statements are
correct with respect to Presidential system of 62. With reference to the National Investigation
government? Agency (NIA) Amendment Act, 2019,
1. The President is the real head of the consider the following statements:
government. 1. The provisions of the Act extend to
2. The President is responsible to the scheduled offenses, committed in India
legislature. or abroad, that affect the interest of
3. The United States America follows the India.
system of presidential form of 2. It designates state high courts as special
government. courts for conducting the trial of
Select the correct answer using code given offenses under the NIA Act.
below. Which of the statements given above is/are
(a) 1 and 2 only correct?
(b) 2 and 3 only (a) 1 only
(c) 1 and 3 only (b) 2 only
(d) 1, 2 and 3 (c) Both 1 and 2
(d) Neither 1 nor 2
60. With reference to the production of millets
in India, consider the following statements: 63. Which of the following are the federal
1. They have low per-hectare yields at the features of the Indian Constitution?
national level compared to wheat and 1. Dual Polity
paddy. 2. Integrated Judiciary
2. Their production has decreased in India 3. Bicameralism
during the last two decades. 4. Written Constitution
Which of the statements given above is/are Select the correct answer using the code
correct? given below.
(a) 1 only (a) 1, 2 and 3 only
(b) 2 only (b) 2 and 4 only
(c) Both 1 and 2 (c) 1, 3 and 4 only
(d) Neither 1 nor 2 (d) 1 and 3 only
13 www.visionias.in ©Vision IAS

FREE BY KING R QUEEN P [ऋषभ राजपूत]


64. In the context of methods of election in 67. In the context of the defense sector, consider
India, consider the following statements the following statements regarding the Very
regarding the First Past The Post System: Short-Range Air Defense System
1. The Indian constitution has prescribed (VSHORADS) missile:
the First Past The Post System method 1. It is a portable surface-to-air missile.
of election to the Lok Sabha and the 2. It is being procured from Israel under the
Rajya Sabha. defense agreement.
2. In this system the candidate needs to get Which of the statements given above is/are
more than 50 percent to get elected. correct?
Which of the statements given above is/are (a) 1 only
correct? (b) 2 only
(a) 1 only (c) Both 1 and 2
(b) 2 only (d) Neither 1 nor 2
(c) Both 1 and 2
(d) Neither 1 nor 2 68. Consider the following statements with
respect to the National Commission for
65. Which of the following laws are enacted for Backward Classes (NCBC):
implementing the Directive Principles? 1. The National Commission for Backward
1. Payment of Bonus Act (1965) Classes (NCBC) is a constitutional body.
2. Contract Labour Regulation and 2. The Commission presents an annual
Abolition Act (1970) report to both Houses of Parliament.
3. Bonded Labour System Abolition Act Which of the statements given above is/are
(1976) correct?
4. Equal Remuneration Act (1976) (a) 1 only
Select the correct answer using the code (b) 2 only
given below. (c) Both 1 and 2
(a) 1 and 2 only (d) Neither 1 nor 2
(b) 2 and 3 only
(c) 1, 3 and 4 only
69. Consider the following statements with
(d) 1, 2, 3 and 4
regard to the extraordinary powers of the
President when the President’s Rule is
66. Which of the following are ex-officio imposed in a state:
members of The National Human Rights 1. He can take up the functions of the state
Commission? government and powers vested in the
1. Chairperson, National Commission for governor or any other executive
Backward Classes authority in the state.
2. Special officer for linguistic minorities.
2. He can declare that the powers of the
3. Chairperson, National Commission for
state legislature are to be exercised by
Protection of Child Rights.
the Parliament.
4. Chairperson, National Commission for
3. He can assume to himself the powers
Women.
vested in the concerned state high court
5. Chief Commissioner for Persons with
or suspend the provisions of the
Disabilities.
Constitution relating to it.
Select the correct answer using the code
Which of the statements given above is/are
given below.
correct?
(a) 1 and 2 only
(a) 1 only
(b) 2, 3, 4 and 5 only
(b) 1 and 2 only
(c) 1, 3 and 5 only
(c) 2 and 3 only
(d) 1, 3, 4 and 5 only
(d) 1, 2 and 3
14 www.visionias.in ©Vision IAS

FREE BY KING R QUEEN P [ऋषभ राजपूत]


70. With reference to the sovereign green bonds, 72. Consider the following statements with
consider the following statements: respect to the office of Lokayuktas:
1. The bonds will be issued in maturities of 1. The institution of Lokayukta was
five years and 10 years.
established for the first time in
2. They are eligible for trading in the
Maharashtra in 1971.
secondary market.
2. The Lokayukta of a state is always
3. As of now, NRIs are not allowed to
appointed by the chief justice of the high
invest in them.
court of the concerned state.
Which of the statements given above are
correct? 3. The recommendations made by the

(a) 1, 2 and 3 Lokayukta are only advisory and not

(b) 1 and 2 only binding on the state government.


(c) 2 and 3 only Which of the statements given above is/are
(d) 1 and 3 only correct?

(a) 2 and 3 only


71. With reference to the independence and
(b) 1 only
impartiality of the Election Commission,
(c) 1, 2 and 3
which of the following statements is/are
(d) 1 and 3 only
correct?
1. The Constitution has not prescribed the
73. Consider the following statements regarding
qualifications of the members of the
Election Commission. the District Disaster Management Authority

2. The Constitution has specified the term (DDMA):

of the members of the Election 1. The District Magistrate is the ex-officio


Commission. chairperson of the DDMA.
3. The Constitution has not debarred the 2. The chief executive officer of the
retiring election commissioners from
DDMA is appointed by the state
any further appointment by the
government.
government.
Which of the statements given above is/are
Select the correct answer using the code
correct?
given below.
(a) 1 only
(a) 1 and 2 only
(b) 2 only (b) 2 only

(c) 2 and 3 only (c) Both 1 and 2

(d) 1 and 3 only (d) Neither 1 nor 2


15 www.visionias.in ©Vision IAS

FREE BY KING R QUEEN P [ऋषभ राजपूत]


74. Which among the following constitutional 77. In the context of direct democracy, consider
amendments fall outside the scope of Article the following pairs:
368? Device Description
1. Abolition of Legislative Councils in
1. Referendum : Procedure whereby
States
proposed legislation
2. Elections to Parliament and State
is referred to the
Legislatures
3. Election of the President electorate for
Select the correct answer using the code settlement by their
given below. direct votes
(a) 1 only 2. Recall : Method of obtaining
(b) 1 and 2 only the opinion of
(c) 2 and 3 only
people on any issue
(d) 1 and 3 only
of public
importance
75. Consider the following statements regarding
the Constituent Assembly of India: 3. Plebiscite : Method by means of
1. Provinces were allotted seats in which the voters can
proportion to their respective remove a
populations. representative
2. The representatives of the princely states before the expiry of
were to be nominated by the heads of the his term
princely states.
Which of the pairs given above is/are
3. Elections to the Constituent Assembly
correctly matched?
were conducted after Independence.
Which of the statements given above is/are (a) 1 only
correct? (b) 2 and 3 only
(a) 1 only (c) 1 and 3 only
(b) 1 and 2 only (d) 1, 2 and 3
(c) 2 and 3 only
(d) 1 and 3 only
78. Consider the following statements :
1. Article 1 describes India as a federation
76. Which of the following languages are
of states.
mentioned in the eighth schedule of the
2. Currently, India has 28 states and 8
Constitution of India?
1. Bodo union territories.
2. English 3. The names of states and union territories
3. Nepali and their territorial extent are mentioned
4. Urdu in the third schedule of the Constitution.
5. Santhali Which of the statements given above is/are
Select the correct answer using the code
correct?
given below.
(a) 2 only
(a) 1, 2, 3 and 4 only
(b) 1 and 2 only
(b) 1, 3, 4 and 5 only
(c) 3, 4 and 5 only (c) 1 and 3 only
(d) 1, 2 and 5 only (d) 1, 2 and 3

16 www.visionias.in ©Vision IAS

FREE BY KING R QUEEN P [ऋषभ राजपूत]


79. With reference to 'grey hydrogen', consider 82. With reference to Advocate General for the
the following statements: states, consider the following statements:
1. It is generated from natural gas or
1. The term of office of the advocate
methane, through a process called
“steam reforming”. general is fixed by the Constitution.
2. It is called 'grey hydrogen due to the 2. He holds office at the pleasure of the
distinct grey smoke produced when governor of a state.
burnt in presence of oxygen.
3. He enjoys all the privileges and
Which of the statements given above is/are
correct? immunities that are available to a
(a) 1 only member of the state legislature.
(b) 2 only Which of the statements given above are
(c) Both 1 and 2
correct?
(d) Neither 1 nor 2
(a) 1 and 2 only
80. In the context of evolution of Indian (b) 2 and 3 only
constitution, which of the following Acts, for (c) 1 and 3 only
the first time, separated provincial budgets
(d) 1, 2 and 3
from the Central budget and authorized the
provincial legislatures to enact their
budgets? 83. Recently the Food Safety and Standards
(a) Indian Councils Act of 1909 Authority of India (FSSAI) was in the news.
(b) Government of India Act of 1919
FSSAI falls under which of the following
(c) Indian Councils Act of 1861
ministries?
(d) Government of India Act of 1935
(a) Ministry of Health & Family Welfare
81. Consider the following statements regarding (b) Ministry of Food Processing Industries
the Pradhan Mantri Garib Kalyan Anna
(c) Ministry of Commerce and Industry
Yojana (PMGKAY):
(d) It is an independent body and not part of
1. It is an integrated food security scheme
for providing free foodgrains to any ministry
Antodaya Ann Yojna (AAY) and
Primary Household (PHH) beneficiaries
84. In which of the following cases, the Supreme
under National Food Security Act
Court held that the doctrine of basic
(NFSA), 2013
2. The beneficiaries will also be getting structure would apply to constitutional
food grains at subsidized prices. amendments after April 24, 1973?
Which of the statements given above is/are
(a) S.R. Bommai Case (1994)
correct?
(a) 1 only (b) Waman Rao Case (1981)

(b) 2 only (c) Kuldip Nayar Case (2006)


(c) Both 1 and 2 (d) Indira Gandhi Nehru Case (1975)
(d) Neither 1 nor 2
17 www.visionias.in ©Vision IAS

FREE BY KING R QUEEN P [ऋषभ राजपूत]


85. With reference to the principle of 'collective 88. Consider the statements regarding the
responsibility' in India, consider the Digital India Land Records Modernisation
Programme (DILRMP):
following statements:
1. It is a central sector scheme under the
1. It is a judicial innovation and not
Ministry of Rural Development.
mentioned explicitly under the 2. It aims to achieve computerization of
constitutional provisions. land records.
2. It means that the Council of Ministers Which of the statements given above is/are
shall be collectively responsible to the correct?
(a) 1 only
Parliament.
(b) 2 only
Which of the statements given above is/are (c) Both 1 and 2
correct ? (d) Neither 1 nor 2
(a) 1 only
(b) 2 only 89. Consider the following statements regarding
(c) Both 1 and 2 the Central Vigilance Commission (CVC):
1. Its establishment was recommended by
(d) Neither 1 nor 2
the Rajamannar commission for the
prevention of corruption.
86. Which of the following countries have a 2. Vigilance Commissioners are not
federal system of government? eligible for further employment under
1. India the Central or state government.
2. United Kingdom 3. The salary, allowances, and other
conditions of service of the Central
3. Brazil
Vigilance Commissioner are similar to
4. Mexico those of the Chief Election
Select the correct answer using the code Commissioner.
given below. Which of the statements given above
(a) 1, 2 and 3 only is/are correct?
(a) 1 and 3 only
(b) 1, 3 and 4 only
(b) 2 only
(c) 2 and 4 only
(c) 2 and 3 only
(d) 1 and 3 only (d) 1 and 2 only

87. Consider the following statements regarding 90. With reference to the Goods and Services
the Drafting Committee of the Constitution : Tax Council, consider the following
1. It was constituted after independence. statements:
1. The Goods and Services Tax Council is
2. N. Gopalaswamy Ayyangar was the
a constitutional body.
chairman of this committee. 2. Union Revenue Secretary acts as the ex-
Which of the statements given above is/are officio Secretary to the Council.
correct? Which of the statements given above is/are
(a) 1 only correct?
(a) 1 only
(b) 2 only
(b) 2 only
(c) Both 1 and 2
(c) Both 1 and 2
(d) Neither 1 nor 2 (d) Neither 1 nor 2
18 www.visionias.in ©Vision IAS

FREE BY KING R QUEEN P [ऋषभ राजपूत]


91. Consider the following statements with 94. Consider the following statements with
reference to the National Commission for respect to the Preamble of the Constitution
Protection Of Child Rights (NCPCR): of India:
1. It is a statutory body under the 1. It was not a part of the constitution
adopted on 26th November 1949.
administrative control of the Ministry of
2. It specifies justice, liberty, equality, and
Women & Child Development.
fraternity as the objectives.
2. It inquires into violation of child rights 3. It was the first time recognized as part of
and takes suo-motu notice of matters the constitution in the Berubari Union
relating to deprivation and violation of case in 1960.
child rights. Which of the statements given above is/are
Which of the statements given above is/are correct?
correct? (a) 1 and 3 only
(a) 1 only (b) 2 only
(c) 2 and 3 only
(b) 2 only
(d) 1, 2 and 3
(c) Both 1 and 2
(d) Neither 1 nor 2
95. Recently, the Archeological Survey of India
discovered votive stupas at Nalanda
92. Consider the following statements regarding Mahavihara. In this context, consider the
the Central Information Commission: following statements regarding the Nalanda
1. The Central Information Commission is Mahavihara:
a statutory body, set up under the Right 1. It was visited by both Hiuen Tsang and
to Information Act in 2005. It-sing.
2. The Commission consists of a Chief 2. It was patronised by both Gupta and Pala
Information Commissioner and not more dynasties.
3. It is a UNESCO World Heritage site.
than ten Information Commissioners.
Which of the statements given above is/are
Which of the statements given above is/are
correct?
correct? (a) 1 and 2 only
(a) 1 only (b) 2 and 3 only
(b) 2 only (c) 3 only
(c) Both 1 and 2 (d) 1, 2 and 3
(d) Neither 1 nor 2
96. Article 51A of the Indian Constitution
93. Consider the following pairs: provides for the Fundamental Duties. In this
Provisions under DPSP Article context, which of the following acts have a
bearing on the implementation of some of
1. Promotion of cottage : Article 40
the Fundamental Duties?
industries
1. Prevention of Insults to National Honour
2. Prohibit the slaughter of : Article 47 Act (1971)
cows 2. Representation of People Act (1951)
3. Safeguard forests and : Article 3. Wildlife (Protection) Act of 1972
wildlife 48A 4. Unlawful Activities (Prevention) Act of
Which of the pairs given above is/are 1967
correctly matched? Select the correct answer using the code
(a) 1 only given below.
(a) 1 and 3 only
(b) 2 and 3 only
(b) 1, 2 and 3 only
(c) 3 only
(c) 1, 2 and 4 only
(d) 1, 2 and 3 (d) 1, 2, 3 and 4

19 www.visionias.in ©Vision IAS

FREE BY KING R QUEEN P [ऋषभ राजपूत]


97. With reference to Chief Election 99. The Directive Principles of State policy are
Commissioner, consider the following significant and useful because:
statements:
1. They serve as a common political
1. The appointment of the Chief Election
manifesto.
Commissioner and other election
commissioners shall be made by the 2. They facilitate stability and continuity in
President. domestic and foreign policies.
2. Chief Election Commissioner and the 3. They act as an ‘Instrument of
two other election commissioners have
Instructions’ to all authorities in the
equal powers and receive equal salaries,
Indian Union.
allowances, and other perquisites.
3. Chief Election Commissioner holds his Select the correct answer using the code
office till the pleasure of the president. given below.
Which of the statements given above is/are (a) 1 only
correct?
(b) 1 and 2 only
(a) 1 and 2 only
(c) 2 and 3 only
(b) 2 only
(c) 1 and 3 only (d) 1, 2 and 3
(d) 1, 2 and 3
100. The Directive Principles of State Policy are
98. Consider the following statements regarding enumerated in Part IV of the Constitution
the steps taken by the Centre to promote
from Articles 36 to 51. In this regard, the
app-based payment systems:
framers of the Constitution derived
1. UPI Lite is an 'on-device wallet' feature
by the National Payments Corporation of inspiration from which of the following?
India (NPCI) to allow users to make 1. The Irish Constitution
small-value payments of up to ₹1000 2. Instrument of Instructions enumerated in
offline.
the Government of India Act of 1935.
2. All Non-Resident Indians (NRIs) with
3. Gandhian ideology
NRE (non-resident external) accounts
can use the UPI platform from their 4. Freedom struggle of India
international mobile numbers. Select the correct answer using the code
Which of the statements given above is/are given below.
correct?
(a) 1, 2 and 3 only
(a) 1 only
(b) 2 and 4 only
(b) 2 only
(c) Both 1 and 2 (c) 1, 3 and 4 only
(d) Neither 1 nor 2 (d) 1, 2, 3 and 4

Copyright © by Vision IAS


All rights are reserved. No part of this document may be reproduced, stored in a retrieval system or
transmitted in any form or by any means, electronic, mechanical, photocopying, recording or otherwise,
without prior permission of Vision IAS.
20 www.visionias.in ©Vision IAS

FREE BY KING R QUEEN P [ऋषभ राजपूत]


VISIONIAS
www.visionias.in
ANSWERS & EXPLANATIONS
GENERAL STUDIES (P) TEST – 4126 (2024)

Q 1.D
• Constitution can be broadly defined as the basic principles and laws of a nation, state, or social group that
determine the powers and duties of the government and guarantee certain rights to the people in it.
• Basic functions of a constitution
o Any group will need some basic rules that are publicly promulgated and known to all members of that
group to achieve a minimal degree of coordination. But these rules must not only be known, they must
also be enforceable. If citizens have no assurance that others will follow these rules, they will
themselves have no reason to follow these rules
▪ Thus, It provides a set of basic rules that allow for minimal coordination amongst members
of a society.
o The constitution specifies who has the power to make decisions in a society. It decides how the
government will be constituted.
o Constitution also sets some limits on what a government can impose on its citizens. These limits
are fundamental in the sense that government may never trespass them.
o Constitution also enables the government to fulfil the aspirations of a society and create conditions for
a just society.
• Hence option (d) is the correct answer.

Q 2.C
• DIRECTIVES OUTSIDE PART IV: Apart from the Directives included in Part IV, there are some
other Directives contained in other parts of the Constitution. They are:
o Claims of SCs and STs to Services: The claims of the members of the Scheduled Castes and the
Scheduled Tribes shall be taken into consideration, consistently with the maintenance of
efficiency of administration, in the making of appointments to services and posts in connection
with the affairs of the Union or a State (Article 335 in Part XVI).
o Instruction in mother tongue: It shall be the endeavor of every state and every local authority within
the state to provide adequate facilities for instruction in the mother tongue at the primary stage of
education to children belonging to linguistic minority groups (Article 350-A in Part XVII).
o Development of the Hindi Language: It shall be the duty of the Union to promote the spread of the
Hindi language and to develop it so that it may serve as a medium of expression for all the elements
of the composite culture of India (Article 351 in Part XVII).
• The above Directives are also non-justiciable in nature. However, they are also given equal importance
and attention by the judiciary on the ground that all parts of the constitution must be read together.
• Hence, option (c) is the correct answer.

Q 3.A
• FEATURES OF FUNDAMENTAL RIGHTS
o The Fundamental Rights guaranteed by the Constitution are characterized by the following:
▪ Some of them are available only to citizens while others are available to all persons whether
citizens, foreigners, or legal persons like corporations or companies. Hence, the statement 3
is correct.
▪ They are not absolute but qualified. The state can impose reasonable restrictions on them.
However, whether such restrictions are reasonable or not is to be decided by the courts. Thus,
they strike a balance between the rights of the individual and those of the society as a whole,
between individual liberty and social control. Hence, statement 1 is correct.
1 www.visionias.in ©Vision IAS

FREE BY KING R QUEEN P [ऋषभ राजपूत]


▪ All of them are available against the arbitrary action of the state. However, some of them are
also available against the action of private individuals. Hence, statement 2 is correct.
▪ Some of them are negative in character, that is, place limitations on the authority of the State,
while others are positive in nature, conferring certain privileges on the persons.
▪ They are justiciable, allowing persons to move the courts for their enforcement, if and when they
are violated.
▪ They are defended and guaranteed by the Supreme Court. Hence, the aggrieved person can
directly go to the Supreme Court, not necessarily by way of appeal against the judgment of the
high courts.
▪ They are not sacrosanct or permanent. The Parliament can curtail or repeal them but only by a
constitutional amendment act and not by an ordinary act. Moreover, this can be done without
affecting the ‘basic structure’ of the Constitution.
▪ They can be suspended during the operation of a National Emergency except for the rights
guaranteed by Articles 20 and 21. Further, the six rights guaranteed by Article 19 can be
suspended only when an emergency is declared on the grounds of war or external aggression (i.e.,
external emergency) and not on the ground of armed rebellion (i.e., internal emergency).
▪ Their scope of operation is limited by Article 31A (saving of laws providing for the
acquisition of estates, etc.), Article 31B (validation of certain acts and regulations included
in the 9th Schedule), and Article 31C (saving of laws giving effect to certain directive
principles).
▪ Their application to the members of armed forces, para-military forces, police forces, intelligence
agencies, and analogous services can be restricted or abrogated by the Parliament (Article 33).
▪ Their application can be restricted while martial law is in force in any area. Martial law means
‘military rule’ imposed under abnormal circumstances to restore order (Article 34). It is different
from the imposition of a national emergency.
▪ Most of them are directly enforceable (self-executory) while a few of them can be enforced
on the basis of a law made for giving effect to them. Such a law can be made only by the
Parliament and not by state legislatures so that uniformity throughout the country is
maintained (Article 35). Hence, statement 4 is not correct.

Q 4.D
• With an objective to shift the focus of measuring the performance of hospitals from the volume of
services provided to the value of healthcare services, National Health Authority (NHA) is introducing
a new system to measure and grade hospital performance under the flagship scheme Ayushman
Bharat Pradhan Mantri Jan Arogya Yojana (AB PM-JAY).
• The new initiative will introduce the concept of ‘value-based care’, where payment will be outcome-
based and providers will be rewarded according to quality of treatment delivered.
• AB PMJAY is a centrally sponsored health assurance scheme. It provides a cover of up to Rs. 5 lakhs
per family per year for secondary and tertiary care hospitalization. This is the “world’s largest
government-funded healthcare program” targeting more than 50 crore beneficiaries. Hence,
statement 1 is correct.
• It provides cashless and paperless access to services for beneficiaries at point of service. Beneficiary
families have been identified from Socio-Economic Caste Census, 2011. Hence, statement 2 is
correct.
• National Health Authority (NHA) is the apex body responsible for implementing India’s flagship
public health insurance/assurance scheme called “Ayushman Bharat Pradhan Mantri Jan Arogya
Yojana” & has been entrusted with the role of designing strategy, building technological infrastructure,
and implementing of “National Digital Health Mission” to create a National Digital Health Eco-
system. Hence, statement 3 is correct.
• National Health Authority is the successor of the National Health Agency, which was functioning as
a registered society under the Societies Registration Act, 1860 since 23rd May 2018. Pursuant to
Cabinet decision for full functional autonomy, National Health Agency was reconstituted as the National
Health Authority.

Q 5.D
• Global Economic Prospects, a flagship report (issued twice a year) was recently released by World
Bank. Hence statements 1 and 2 are not correct.
• It examines global economic developments and prospects, with a special focus on emerging markets
and developing economies (EMDEs).
2 www.visionias.in ©Vision IAS

FREE BY KING R QUEEN P [ऋषभ राजपूत]


• Key Projections:
o Global GDP Growth: 1.7% in 2023 - third-weakest pace since 1993, overshadowed only by 2009 and
2020 global recessions.
o India is expected to grow at 6.6% in 2023.
o Reasons for the slowdown: Elevated Inflation, Higher interest rates, reduced investment and
disruptions caused by Russia-Ukraine war.
o Any additional adverse shocks could push the global economy into recession.
o Recession is a significant, pervasive, and persistent decline in economic activity – at least for two
consecutive quarters of negative GDP for most analyses.

Q 6.C
• Schedule 4 deals with the allocation of seats in the Rajya Sabha to the states and the union
territories. It covers articles 4 and 80. Hence pair 1 is correctly matched.
• Schedule 5 deals with provisions relating to the administration and control of scheduled areas and
scheduled tribes.
• Schedule 6 deals with provisions relating to the administration of tribal areas in the states of Assam,
Meghalaya, Tripura and Mizoram. Hence pair 2 is not correctly matched.
• 10th Schedule deals with the provisions relating to disqualification of the members of Parliament and
State Legislatures on the ground of defection. This schedule was added by the 52nd Amendment Act of
1985, also known as Anti-defection Law. Hence pair 3 is correctly matched.

Q 7.D
• Recently, a report titled “Scientific Assessment of Ozone Depletion: 2022” was prepared in
coordination with WMO, UNEP, National Oceanic and Atmospheric Administration (NOAA), NASA,
and European Commission.
• Key observations of the report:
o Actions taken under Montreal Protocol contribute to ozone recovery in the upper stratospheric layer.
o If current policies remain in place, the ozone layer is expected to recover to 1980 values by
around 2066 over the Antarctic, by 2045 over the Arctic, and by 2040 for the rest of the world.
• Ozone-depleting substances are man-made gases that destroy ozone once they reach the ozone
layer. The ozone layer sits in the upper atmosphere and reduces the amount of harmful ultraviolet
radiation that reaches Earth from the sun. Ultraviolet radiation can have detrimental effects on both
humans and the environment. For instance, inducing skin cancer and cataract distorts plant growth,
and damages the marine environment.
• Ozone-depleting substances include:
o chlorofluorocarbons (CFCs)
o hydrochlorofluorocarbons (HCFCs)
o hydrobromoflurocarbons (HBFCs). Hence option 2 is correct.
o halons
o methyl bromide. Hence option 1 is correct.
o carbon tetrachloride. Hence option 3 is correct.
o methyl chloroform. Hence option 4 is correct.
• They have been used as:
o refrigerants in commercial, home, and vehicle air conditioners and refrigerators
o foam blowing agents
o components in electrical equipment
o industrial solvents
o solvents for cleaning (including dry cleaning)
o aerosol spray propellants
o fumigants

Q 8.B
• Swaran Singh Committee was set up in 1976 by the Congress government to provide suggestions on the
inclusion of the Fundamental Duties in the Indian Constitution. The Congress Government at the Centre
accepted these recommendations and enacted the 42nd Constitutional Amendment Act in 1976. This
amendment added a new part, namely, Part IVA to the Constitution. This new part consists of only one
Article, that is, Article 51A.
• Though the Swaran Singh Committee suggested the incorporation of eight Fundamental Duties in the
Constitution, the 42nd Constitutional Amendment Act (1976) included ten Fundamental Duties.
3 www.visionias.in ©Vision IAS

FREE BY KING R QUEEN P [ऋषभ राजपूत]


Interestingly, certain recommendations of the Committee were not accepted by the Congress Party and
hence, not incorporated in the Constitution.
• Certain recommendations of the Swaran Singh Committee were not accepted and hence, not
incorporated into the Constitution. This include:
o The Parliament may provide for the imposition of such penalty or punishment as may be considered
appropriate for any non-compliance with or refusal to observe any of the duties. Hence statement (a)
is not correct.
o No law imposing such penalty or punishment shall be called in question in any court on the ground of
infringement of any of Fundamental Rights or on the ground of repugnancy to any other provision of
the Constitution. Hence statement (d) is not correct.
o Duty to pay taxes should also be a Fundamental Duty of the citizens. Hence statement (c) is not
correct.
• Duty to defend the country and render national service when called upon to do so is part of
Fundamental duties under article 51A (d) of the constitution. Hence, option (b) is the correct
answer.

Q 9.B
• The Central Bureau of Investigation (CBI) was set up in 1963 by a resolution of the Ministry of Home
Affairs. Later, it was transferred to the Ministry of Personnel and now it enjoys the status of an attached
office. The Special Police Establishment (which looked into vigilance cases) setup in 1941 was also
merged with the CBI. The establishment of the CBI was recommended by the Santhanam Committee on
Prevention of Corruption (1962–1964). The CBI is not a statutory body. It derives its powers from
the Delhi Special Police Establishment Act, of 1946.
• The functions of CBI are:
o Investigating cases of corruption, bribery, and misconduct of Central government employees.
Hence statement (a) is correct.
o Investigating cases relating to infringement of fiscal and economic laws, that is, breach of laws
concerning export and import control, customs and central excise, income tax, foreign exchange
regulations, and so on. However, such cases are taken up either in consultation with or at the request
of the department concerned.
o Investigating serious crimes, having national and international ramifications, committed by
organized gangs of professional criminals. Hence statement (c) is correct.
o Coordinating the activities of the anticorruption agencies and the various state police forces
o Taking up, on the request of a state government, any case of public importance for investigation.
o Maintaining crime statistics and disseminating criminal information. Hence statement (d) is
correct.
• Probing terror attacks including bomb blasts, hijacking of aircraft and ships, attacks on nuclear
installations, and use of weapons of mass destruction is the function of the National Investigation
Agency. Hence option (b) is not correct.

Q 10.C
• The Bureau of Indian Standards (BIS) under the Ministry of Consumer Affairs has introduced
standards for USB Type-C ports, plug, and cables used in electronic devices like mobile phones,
laptops, etc. It aims to provide common charging solutions for various electronic devices sold in the
country by March 2025.
• Need for Common Charger:
o To reduce the number of chargers per consumer, thus reducing e-waste in India
o Ease out the technological ‘lock-in’ effect, whereby a consumer becomes dependent on a single
manufacturer.
• USB i.e., Universal Serial Bus (an industry standard) is used for connecting a wide variety of devices
to a processor.
o USB Types: USB-A is a flat and rectangular shape; USB-B has multiple designs, but the common
shape is a square; and USB-C is a smaller rectangular with rounded corners.
o In the EU by 2024, all mobile phones, tablets, and cameras will be sold with a USB Type-C
charging port.
• USB Type-C uses a 24-pin double-sided connector allowing device manufacturers to enable Alternate
Mode, which unlocks the ability to use alternate data protocols for direct device-to-host transmission. This
allows you to output VGA, HDMI, DisplayPort, Thunderbolt, Ethernet, or other types of connections
from a single USB Type-C port with the help of compatible adapters. Hence, statement 1 is correct.
4 www.visionias.in ©Vision IAS

FREE BY KING R QUEEN P [ऋषभ राजपूत]


• The USB Type-C is a new reversible connector that allows you to connect the cable to your device in
any direction, unlike its predecessors.
• The physical USB-C connector isn’t backward compatible, but the underlying USB standard is. One
can’t plug older USB devices into a modern, tiny USB-C port, nor can one connect a USB-C connector to
an older, larger USB port. But that doesn’t mean one has to discard all your old peripherals. USB 3.1
is still backward-compatible with older versions of USB, so one just needs a physical adapter with a
USB-C connector on one end and a larger, older-style USB port on the other end. One can then plug
your older devices directly into a USB Type-C port. Hence, statement 2 is correct.
• It has the ability to use a single cable for data transfer, deliver video to a display, and power
delivery, which are the key features of USB Type-C.

Q 11.B
• After the great revolt of 1857, the British Government felt the necessity of seeking the cooperation of the
Indians in the administration of their country. In pursuance of this policy of association, three acts were
enacted by the British Parliament in 1861, 1892 and 1909. The Indian Councils Act of 1861 is an
important landmark in the constitutional and political history of India.
• The features of this Act were as follows
o It made the beginning of the representative institutions by associating Indians with the law-making
process. It, thus, provided that the Viceroy should nominate some Indians as non-official members
of his expanded council. In 1862, Lord Canning, the then Viceroy, nominated three Indians to his
legislative council–the Raja of Benaras, the Maharaja of Patiala and Sir Dinkar Rao.
o It initiated the process of decentralization by restoring the legislative powers to the Bombay and
Madras Presidencies.
o It also provided for the establishment of new legislative councils for Bengal, North-Western
Provinces and Punjab, which were established in 1862, 1886 and 1897, respectively.
o It empowered the Viceroy to make rules and orders for the more convenient transaction of business in
the council. It also gave recognition to the ‘portfolio’ system, introduced by Lord Canning in
1859. Under this, a member of the Viceroy’s council was made in-charge of one or more departments
of the Government and was authorised to issue final orders on behalf of the council on matters of his
department(s).
o It empowered the Viceroy to issue ordinances, without the concurrence of the legislative council,
during an emergency.
• Hence option (b) is the correct answer.

Q 12.A
• In the Champakam Dorairajan case (1951), the Supreme Court ruled that the directive principles of
State policy laid down in Part IV of the Constitution cannot in any way override or abridge the
fundamental rights guaranteed by Part III. On the other hand, they have to conform to and run as
subsidiaries to the fundamental rights laid down in Part III. But, it also held that Fundamental
Rights could be amended by the Parliament by enacting constitutional amendment acts.
• As a result, the Parliament made the First Amendment Act (1951), the Fourth Amendment Act (1955),
and the Seventeenth Amendment Act (1964) to implement some of the Directives.
• The above situation underwent a major change in 1967 following the Supreme Court’s judgement in
the Golaknath case (1967). In that case, the Supreme Court ruled that the Parliament cannot take away or
abridge any of the Fundamental Rights, which are ‘sacrosanct’ in nature. In other words, the Court held
that the Fundamental Rights cannot be amended for the implementation of the Directive Principles.
• In the Kesavananda Bharati case (1973), the Supreme Court declared the above second provision of
Article 31C as unconstitutional and invalid on the ground that judicial review is a basic feature of the
Constitution and hence, cannot be taken away.
• In the Minerva Mills case (1980), the Supreme Court also held that ‘the Indian Constitution is founded
on the bedrock of the balance between the Fundamental Rights and the Directive Principles. They together
constitute the core of the commitment to social revolution. They are like two wheels of a chariot, one no
less than the other. To give absolute primacy to one over the other is to disturb the harmony of the
Constitution. This harmony and balance between the two is an essential feature of the basic structure of
the Constitution.
• Hence, option (a) is the correct answer.

5 www.visionias.in ©Vision IAS

FREE BY KING R QUEEN P [ऋषभ राजपूत]


Q 13.B
• The Constitution divided the powers between the Centre and the states in terms of the Union List, State
List and Concurrent List in the Seventh Schedule.
o The Union List consists of 98 subjects (originally 97), the State List 59 subjects (originally 66) and
the Concurrent List 52 subjects (originally 47).
o Both the Centre and the states can make laws on the subjects of the concurrent list, but in case of a
conflict, the Central law prevails. The residuary subjects (ie, which are not mentioned in any of the
three lists) are given to the Centre.
• Important subjects as distributed in seventh schedule

• Hence option (b) is the correct answer.

Q 14.C
• In the Minerva Mills case (1980), the Supreme Court provided key clarifications on the interpretation of
the basic structure doctrine. The court ruled that the power of the parliament to amend the constitution is
limited by the constitution.
o Hence the parliament cannot exercise this limited power to grant itself unlimited power. In addition, a
majority of the court also held that the parliament's power to amend is not a power to destroy.
• The ruling struck down clause 4 and 5 of the Constitution (Forty-second Amendment) Act, 1976. Clause
4 gave precedence to directive principles of state policy over fundamental rights. Supreme court
struck down this provision and said that harmony and balance between fundamental rights
directive principles is part of basic structure of the constitution. Clause 5 gave unlimited power to
parliament to amend the constitution and this was also struck down by supreme court deeming it to be
destroying the basic structure of the constitution.
• Hence option (c) is the correct answer.

Q 15.B
• A Bose-Einstein condensate (BECs) is so named because its existence was posited almost a century ago
by Albert Einstein and Indian mathematician Satyendra Nath Bose. It is often termed as fifth state of
matter. Hence, statement 1 is not correct.
• It exists when atoms of certain elements are cooled to temperatures near absolute zero. At that point,
clusters of atoms begin functioning as a single quantum object with both wave and particle
properties. Hence, statement 2 is not correct.
• BECs have also been used to create atom lasers, atomic clocks and gravitational, rotational or
magnetic sensors with excellent sensitivity. Hence, statement 3 is correct.

6 www.visionias.in ©Vision IAS

FREE BY KING R QUEEN P [ऋषभ राजपूत]


Q 16.A
• Right to freedom of Religion (Articles 25 to 28)
o Freedom from Taxation for the Promotion of a Religion under Article 27 lays down that no
person shall be compelled to pay any taxes for the promotion or maintenance of any particular
religion or religious denomination.
o In other words, the State should not spend the public money collected by way of tax for the
promotion or maintenance of any particular religion. Hence, statement 1 is correct.
o This provision prohibits the State from favouring, patronising and supporting one religion over the
other. This means that taxes can be used for the promotion or maintenance of all religions.
o This provision prohibits only the levy of a tax and not a fee. This is because the purpose of a fee
is to control the secular administration of religious institutions and not to promote or maintain
the religion. Hence, statement 2 is not correct.
o Thus, a fee can be levied on pilgrims to provide them with some special service or safety measures.
Similarly, a fee can be levied on religious endowments for meeting the regulation expenditure.

Q 17.D
• Article 360 empowers the president to proclaim a FinancialEmergency if he is satisfied that a situation has
arisen due to which the financial stability or credit of India or any part of its territory is threatened.
• The consequences of the proclamation of a Financial Emergency are as follows:
o The executive authority of the Centre extends to the giving of
▪ directions to any state to observe such canons of financial propriety as may be specified in the
directions, and
▪ such other directions to any state as the President may deem necessary and adequate for the
purpose.
o Any such direction may include a provision requiring
▪ the reduction of salaries and allowances of all or any class of persons serving in the state; and
▪ the reservation of all money bills or other financial bills for the consideration of the
President after they are passed by the legislature of the state. Hence statement 2 is correct.
o The President may issue directions for the reduction of salaries and allowances of
▪ all or any class of persons serving the Union; and
▪ the judges of the Supreme Court and the high court. Hence statement 3 is correct.
o Thus, during the operation of a financial emergency, the Centre acquires full control over the states
in financial matters. Hence statement 1 is correct.
Q 18.A
• Article 3 authorizes the Parliament to:
o form a new state by separation of territory from any state or by uniting two or more states or parts of
states or by uniting any territory to a part of any state
o increase the area of any state
o diminish the area of any state
o alter the boundaries of any state
o alter the name of any state
• Admission of new states is not permitted under article 3 of the constitution. It is allowed under
Article 2 of the constitution which empowers the Parliament to ‘admit into the Union of India, or
establish, new states on such terms and conditions as it thinks fit’.
o Thus, Article 2 grants two powers to the Parliament:
▪ the power to admit into the Union of India new states. Hence option (a) is the correct
answer.
▪ the power to establish new states.
• The first refers to the admission of states which are already in existence, while the second refers to the
establishment of states which were not in existence before.

Q 19.D
• Modern democratic governments are classified into parliamentary and presidential on the basis of nature
of relations between the executive and the legislative organs of the government.
o The parliamentary system of government is one in which the executive is responsible to the legislature
for its policies and acts. The presidential system of government, on the other hand, is one in which the
executive is not responsible to the legislature for its policies and acts, and is constitutionally
independent of the legislature in respect of its term of office.
7 www.visionias.in ©Vision IAS

FREE BY KING R QUEEN P [ऋषभ राजपूत]


• Advantages of the Parliamentary System
o Harmony Between Legislature and Executive
▪ The greatest advantage of the parliamentary system is that it ensures harmonious relationship and
cooperation between the legislative and executive organs of the government. The executive is a
part of the legislature and both are interdependent at work.
o Responsible Government
▪ By its very nature, the parliamentary system establishes a responsible government. The ministers
are responsible to the Parliament for all their acts of omission and commission. The Parliament
exercises control over the ministers through various devices like question hour, discussions,
adjournment motion, no confidence motion.
o Ready Alternative Government
▪ In case the ruling party loses its majority, the Head of the State can invite the opposition party to
form the government. This means an alternative government can be formed without fresh
elections. Hence, Dr. Jennings says, ‘the leader of the opposition is the alternative prime
minister’.
o Wide Representation
▪ In a parliamentary system, the executive consists of a group of individuals (i.e., ministers who are
representatives of the people). Hence, it is possible to provide representation to all sections and
regions in the government. The prime minister while selecting his ministers can take this factor
into consideration.
o Prevents Despotism.
• Demerits of the parliamentary system
o Continuity of Policies
▪ The parliamentary system is not conducive to the formulation and implementation of long-term
policies. This is due to the uncertainty of the tenure of the government. A change in the ruling
party is usually followed by changes in the policies of the government.
o Unstable Government
o Dictatorship of the Cabinet
o Against strict separation of powers
o Government by Amateurs
• Hence option (d) is the correct answer.

Q 20.A
• Equality
o The term ‘equality’ means the absence of special privileges to any section of society and the provision
of adequate opportunities for all individuals without any discrimination. The Preamble secures all
citizens of India equality of status and opportunity. This provision embraces three dimensions of
equality–civic, political, and economic.
o There are some provisions in the Constitution that seek to achieve political equality.
▪ No person is to be declared ineligible for inclusion in electoral rolls on grounds of religion, race,
caste, or sex (Article 325). Hence, statement 3 is correct.
▪ Elections to the Lok Sabha and the state assemblies were to be on the basis of adult suffrage
(Article 326). Hence, statement 1 is correct.
o The following provisions of the chapter on Fundamental Rights ensure civic equality:
▪ Prohibition of discrimination on grounds of religion, race, caste, sex, or place of birth
(Article 15).
▪ Equality of opportunity in matters of public employment (Article 16).
▪ Abolition of untouchability (Article 17).
▪ Abolition of titles (Article 18). Hence, statement 2 is not correct.
Q 21.D
• The procedure for the amendment of the Constitution as laid down in Article 368 is as follows:
o An amendment of the Constitution can be initiated only by the introduction of a bill for the purpose in
either the House of Parliament not in the state legislatures.
o The bill can be introduced either by a minister or by a private member and does not require
prior permission of the president. A nominated member of the Lok sabha can be a minister or a
private member. Hence statement 1 is not correct.
o The bill must be passed in each House by a special majority, that is, a majority of the total
membership of the House and a majority of two-thirds of the members of the House present and
voting.
8 www.visionias.in ©Vision IAS

FREE BY KING R QUEEN P [ऋषभ राजपूत]


o Each House must pass the bill separately. In case of a disagreement between the two Houses, there is
no provision for holding a joint sitting of the two Houses for the purpose of deliberation and passage
of the bill.
o If the bill seeks to amend the federal provisions of the Constitution, it must also be ratified by
the legislatures of half of the states by a simple majority, that is, a majority of the members of
the House present and voting. Hence statement 2 is not correct.
o After duly passed by both Houses of Parliament and ratified by the state legislatures, where necessary,
the bill is presented to the president for assent.
o The president must give his assent to the bill. He can neither withhold his assent to the bill nor return
the bill for reconsideration of the Parliament.
o After the president’s assent, the bill becomes an Act (i.e., a constitutional amendment act) and the
Constitution stands amended in accordance with the terms of the Act.

Q 22.B
• RIGHT TO EQUALITY:
o Equality before Law and Equal Protection of Laws
▪ Article 14 says that the State shall not deny to any person equality before the law or equal
protection of the laws within the territory of India. This provision confers rights on all persons
whether citizens or foreigners.
▪ Moreover, the word ‘person’ includes legal persons, viz, statutory corporations, companies,
registered societies or any other type of legal person.
▪ The concept of ‘equality before the law’ (negative concept) is of British origin while the concept
of ‘equal protection of laws’ has been taken from the American Constitution. The first concept
connotes:
❖ the absence of any special privileges in favour of any person,
❖ the equal subjection of all persons to the ordinary law of the land administered by ordinary
law courts, and
❖ no person (whether rich or poor, high or low, official or non-official) is above the law.
▪ The second concept, on the other hand, connotes:
❖ the equality of treatment under equal circumstances, both in the privileges conferred and
liabilities imposed by the laws,
❖ the similar application of the same laws to all persons who are similarly situated, and
❖ the like should be treated alike without any discrimination.
▪ Thus, the former is a negative concept while the latter is a positive concept. However, both of
them aim at establishing equality of legal status, opportunity and justice. Hence, statement 1 is
not correct.
▪ The Supreme Court held that where equals and unequals are treated differently, Article 14 does
not apply. Hence, statement 2 is correct.
▪ While Article 14 forbids class legislation, it permits the reasonable classification of persons,
objects, and transactions by the law. But the classification should not be arbitrary, artificial, or
evasive. Rather, it should be based on an intelligible differential and substantial distinction.

Q 23.D
• Unitary state is a system of political organization in which most or all of the governing power resides in a
centralized government, in contrast to a federal state.
• Despite being a federation Indian constitution possesses many unitary features such as
o Single Constitution
▪ Usually, in a federation, the states have the right to frame their own Constitution separate from
that of the Centre. In India, on the contrary, no such power is given to the states. The Constitution
of India embodies not only the Constitution of the Centre but also those of the states. Both the
Centre and the states must operate within this single-frame.
o Emergency Provisions
▪ The Constitution stipulates three types of emergencies–national, state and financial. During an
emergency, the Central government becomes all powerful and the states go into the total control
of the Centre. It converts the federal structure into a unitary one without a formal amendment of
the Constitution. This kind of transformation is not found in any other federation.
o All-India Services
▪ In India, the Centre and the states have their separate public services. But, in addition, there are
all-India services (IAS, IPS, and IFS) which are common to both the Centre and the states. The
9 www.visionias.in ©Vision IAS

FREE BY KING R QUEEN P [ऋषभ राजपूत]


members of these services are recruited and trained by the Centre which also possess ultimate
control over them. Thus, these services violate the principle of federalism under the Constitution.
o Appointment of Governor
▪ The governor, who is the head of the state, is appointed by the President. He holds office during
the pleasure of the President. He also acts as an agent of the Centre. Through him, the Centre
exercises control over the states.
o Integrated Election Machinery
▪ The Election Commission conducts elections not only to the Central legislature but also to the
state legislatures. But, this body is constituted by the President and the states have no say in this
matter. The position is same with regard to the removal of its members as well
o Veto Over State Bills
o Parliament’s Authority Over State List
o Integrated Audit Machinery
o Integrated Judiciary
o No Equality of State Representation
• Hence option (d) is the correct answer.

Q 24.B
• Fundamental Rights Consists of :
• Right to equality (Articles 14–18)
o Equality before the law and equal protection of laws (Article 14).
o Prohibition of discrimination on grounds of religion, race, caste, sex or place of birth (Article 15).
o Equality of opportunity in matters of public employment (Article 16).
o Abolition of untouchability and prohibition of its practice (Article 17). Hence option (b) is the
correct answer.
o Abolition of titles except military and academic (Article 18).
• Right to freedom (Articles 19–22)
o Protection of six rights regarding freedom of:
▪ speech and expression,
▪ assembly,
▪ association,
▪ movement,
▪ residence, and
▪ profession (Article 19).
o Protection in respect of conviction for offences (Article 20).
o Protection of life and personal liberty (Article 21).
o Right to elementary education (Article 21A).
o Protection against arrest and detention in certain cases (Article 22).
• Right against exploitation (Articles 23–24)
o Prohibition of traffic in human beings and forced labour (Article 23).
o Prohibition of employment of children in factories, etc. (Article 24).
• Right to freedom of religion (Article 25–28)
o Freedom of conscience and free profession, practice and propagation of religion (Article 25).
o Freedom to manage religious affairs (Article 26).
o Freedom from payment of taxes for the promotion of any religion (Article 27).
o Freedom from attending religious instruction or worship in certain educational institutions (Article
28).
• Cultural and educational rights (Articles 29–30)
o Protection of language, script and culture of minorities (Article 29).
o Right of minorities to establish and administer educational institutions (Article 30).
• Right to constitutional remedies (Article 32)
o Right to move the Supreme Court for the enforcement of fundamental rights including the writs of (i)
habeas corpus, (ii) mandamus, (iii) prohibition, (iv) certiorari, and (v) quo warranto (Article 32).

Q 25.A
• The Protection of Human Rights Act of 1993 provides for the creation of not only the National Human
Rights Commission but also a State Human Rights Commission at the state level. A State Human
Rights Commission can inquire into violation of human rights only in respect of subjects mentioned in
the State List (List-II) and the Concurrent List (List-III) of the Seventh Schedule of the Constitution.
10 www.visionias.in ©Vision IAS

FREE BY KING R QUEEN P [ऋषभ राजपूत]


• The Commission may take any of the following steps during or upon the completion of an inquiry:
o it may recommend to the state government or authority to make payment of compensation or damages
to the victim;
o it may recommend to the state government or authority the initiation of proceedings for prosecution or
any other action against the guilty public servant;
o it may recommend to the state government or authority for the grant of immediate interim relief to the
victim;
o it may approach the Supreme Court or the state high court for the necessary directions, orders or writs
• From the above, It’s clear that the functions of the commission are mainly recommendatory in nature. It
has no power to punish the violators of human rights, nor to award any relief including monetary
relief to the victim. Notably, its recommendations are not binding on the state government or authority.
But, it should be informed about the action taken on its recommendations within one month. Hence,
statement 1 is correct.
• The Commission submits its annual or special reports to the state government. These reports are laid
before the state legislature, along with a memorandum of action taken on the recommendations of the
Commission and the reasons for non-acceptance of any of such recommendations. Hence, statement 2 is
not correct.

Q 26.D
• Recently, telescopes at National Aeronautics and Space Administration (NASA) observed a massive
black hole devouring a star. The incident was the fifth-closest example of a black hole destroying a star
and occurred 250 million light-years from the earth, in the centre of another galaxy.
• The astronomical phenomenon of the destruction of a star by a black hole is called a Tidal Disruption
Event (TDE).
• During a TDE, the tidal force of a black hole disrupts the star in vicinity. While about half of the star’s
debris continues on its original path, the other half is attracted by the black hole’s gravitational pull. The
gradual growth of this material bound to the black hole produces a short-lived flare of emission, known
as a tidal disruption event.
• Hence, option (d) is the correct answer.

Q 27.A
• Articles 358 and 359 describe the effect of a National Emergency (Article 352) on Fundamental
Rights. Article 358 deals with the suspension of the Fundamental Rights guaranteed by Article 19,
while Article 359 deals with the suspension of other Fundamental Rights (except those guaranteed by
Articles 20 and 21). Hence option (a) is the correct answer.
• The differences between Articles 358 and 359 can be summarised as follows:
o Article 358 is confined to Fundamental Rights under Article 19 only whereas Article 359
extends to all those Fundamental Rights whose enforcement is suspended by the Presidential
Order.
o Article 358 automatically suspends the fundamental rights under Article 19 as soon as the emergency
is declared. On the other hand, Article 359 does not automatically suspend any Fundamental Right. It
only empowers the president to suspend the enforcement of the specified Fundamental Rights.
o Article 358 operates only in case of External Emergency (that is, when the emergency is declared
on the grounds of war or external aggression) and not in the case of Internal Emergency (ie, when the
Emergency is declared on the ground of armed rebellion). Article 359, on the other hand, operates in
case of both External Emergency as well as Internal Emergencies.
o Article 358 suspends Fundamental Rights under Article 19 for the entire duration of the Emergency
while Article 359 suspends the enforcement of Fundamental Rights for a period specified by the
president which may either be the entire duration of the Emergency or a shorter period.
o Article 358 extends to the entire country whereas Article 359 may extend to the entire country
or a part of it.
o Article 358 suspends Article 19 completely while Article 359 does not empower the suspension of the
enforcement of Articles 20 and 21.
o Article 358 enables the State to make any law or take any executive action inconsistent with
Fundamental Rights under Article 19 while Article 359 enables the State to make any law or take any
executive action inconsistent with those Fundamental Rights whose enforcement is suspended by the
Presidential Order.

11 www.visionias.in ©Vision IAS

FREE BY KING R QUEEN P [ऋषभ राजपूत]


Q 28.C
• Information Technology (Intermediary Guidelines and Digital Media Ethics Code) Rules
2021 broadly deal with Social media sites, Digital news sites, and Over-the-top (OTT) streaming
platforms. Hence options 1, 2, and 4 are correct.
• It covers guidelines on Social Media Intermediaries, Grievance Appellate Committees,
ensuring online safety & dignity of users, and educating users about Privacy Policies. The Ministry of
Information and Broadcasting is responsible for the formulation of an oversight mechanism to ensure the
same.
• Recently, draft rules for online gaming, were released by the Ministry of Electronics and IT (MeitY).
Online games will have to register with a self-regulatory body, and only games that are cleared by the
body will be allowed to legally operate in India. Online gaming companies will not be allowed to
engage in betting on the outcome of games, as per the proposed rules. Hence option 3 is not correct.
• The proposed rules, aimed at safeguarding users against potential harm from skill-based games, have been
introduced as an amendment to the Information Technology (Intermediary Guidelines and Digital Media
Ethics Code) Rules, 2021.

Q 29.C
• One of the prima facie differences between the dual citizenship provisions is that the Indian constitution
actively prevents dual citizenship while the constitution of the United States does not mention any
restriction on it
• In India, both a citizen by birth as well as a naturalized citizen is eligible for the office of
President. Hence, statement 2 is not correct.
• while in the USA, only a citizen by birth and not a naturalized citizen is eligible for the office of
President. Hence, statement 1 is not correct.

Q 30.C
• The characteristics of the Fundamental Duties:
o Some of them are moral duties while others are civic duties. For instance, cherishing noble ideals of
freedom struggle is a moral precept and respecting the Constitution, National Flag and National
Anthem is a civic duty.
o They refer to such values which have been a part of the Indian tradition, mythology, religions and
practices. In other words, they essentially contain just a codification of tasks integral to the Indian
way of life.
o Unlike some of the Fundamental Rights which extend to all persons whether citizens or foreigners,
the Fundamental Duties are confined to citizens only and do not extend to foreigners. Hence
statement 1 is correct.
o Like the Directive Principles, the fundamental duties are also non-justiciable.
o The Constitution does not provide for their direct enforcement by the courts. Moreover, there is
no legal sanction against their violation. However, the Parliament is free to enforce them by
suitable legislation. Hence statement 2 is correct.
Q 31.B
• According to Article 51A, it shall be the duty of every citizen of India:
o to abide by the Constitution and respect its ideals and institutions, the National Flag, and the
National Anthem;
o to cherish and follow the noble ideals that inspired the national struggle for freedom;
o to uphold and protect the sovereignty, unity, and integrity of India;
o to defend the country and render national service when called upon to do so;
o to promote harmony and the spirit of common brotherhood amongst all the people of India
transcending religious, linguistic, and regional or sectional diversities and to renounce practices
derogatory to the dignity of women;
o to value and preserve the rich heritage of the country’s composite culture;
o to protect and improve the natural environment including forests, lakes, rivers, and wildlife, and to
have compassion for living creatures;
o to develop scientific temper, humanism, and the spirit of inquiry and reform;
o to safeguard public property and to abjure violence;
o to strive towards excellence in all spheres of individual and collective activity so that the nation
constantly rises to higher levels of endeavor and achievement, and
o to provide opportunities for education to his child or ward between the age of six and fourteen years.
This duty was added by the 86th Constitutional Amendment Act, of 2002.
12 www.visionias.in ©Vision IAS

FREE BY KING R QUEEN P [ऋषभ राजपूत]


• The list of duties does not cover important duties like casting vote, paying taxes, family planning,
and so on.
• Hence, option (b) is the correct answer.

Q 32.B
• The proclamation of national emergency (under Article 352) must be approved by both Houses of
Parliament within one month from the date of its issue. Originally, the period allowed for approval by the
Parliament was two months but was reduced by the 44th Amendment Act of 1978. Hence, statement 1
is not correct.
• However, if the proclamation of emergency is issued at a time when the Lok Sabha has been dissolved or
the dissolution of the Lok Sabha takes place during the period of one month without approving the
proclamation, then the proclamation survives until 30 days from the first sitting of the Lok Sabha after its
reconstitution, provided the Rajya Sabha has in the meantime approved it.
• If approved by both Houses of Parliament, the emergency continues for six months and can be extended
to an indefinite period with an approval of the Parliament every six months. This provision for
periodical parliamentary approval was also added by the 44th Amendment Act of 1978. Hence,
statement 2 is correct.

Q 33.B
• In the Kesavananda Bharati case(1973), the Supreme Court overruled its judgement in the Golak Nath
case (1967). It upheld the validity of the 24th Amendment Act (1971) and stated that Parliament is
empowered to abridge or take away any of the Fundamental Rights. At the same time, it laid down a new
doctrine of the ‘basic structure’ of the Constitution.
• It ruled that the constituent power of Parliament under Article 368 does not enable it to alter the ‘basic
structure’ of the Constitution. This means that the Parliament cannot abridge or take away a Fundamental
Right that forms a part of the ‘basic structure’ of the Constitution. The doctrine of basic structure of the
constitution was reaffirmed and applied by the Supreme Court in the Indira Nehru Gandhi case3a (1975).
• Elements of Basic Structure
o Supremacy of the Constitution
o Sovereign, democratic and republican nature of the Indian polity
o Secular character of the Constitution
o Separation of powers between the legislature, the executive and the judiciary
o Federal character of the Constitution
o Unity and integrity of the nation
o Welfare state (socio-economic justice)
o Judicial review
o Freedom and dignity of the individual
o Parliamentary system
o Rule of law
o Harmony and balance between Fundamental Rights and Directive Principles
o Principle of equality
o Free and fair elections
o Independence of Judiciary
o Limited power of Parliament to amend the Constitution
o Effective access to justice
o Principles (or essence) underlying fundamental rights
o Powers of the Supreme Court under Articles 32, 136, 141 and 142
o Powers of the High Courts under Articles 226 and 2277
• Presidential system of government is not part of the basic structure.
• Hence option (b) is the correct answer.

13 www.visionias.in ©Vision IAS

FREE BY KING R QUEEN P [ऋषभ राजपूत]


Q 34.C
• Indian constitution has borrowed many features from various constitutions of the world.

• Hence option (c) is the correct answer.

Q 35.C
• The 42nd Amendment Act of 1976 added four new Directive Principles to the original list. They require
the State:
o To secure opportunities for the healthy development of children (Article 39).
o To promote equal justice and to provide free legal aid to the poor (Article 39 A).
o To take steps to secure the participation of workers in the management of industries (Article 43
A).
o To protect and improve the environment and to safeguard forests and wildlife (Article 48 A).
• The 44th Amendment Act of 1978 added one more Directive Principle, which requires the State to
minimise inequalities in income, status, facilities, and opportunities (Article 38).
• The 86th Amendment Act of 2002 changed the subject matter of Article 45 and made elementary
education a fundamental right under Article 21A. The amended directive requires the State to provide
early childhood care and education for all children until they complete the age of six years.

14 www.visionias.in ©Vision IAS

FREE BY KING R QUEEN P [ऋषभ राजपूत]


• The 97th Amendment Act of 2011 added a new Directive Principle relating to cooperative societies. It
requires the state to promote voluntary formation, autonomous functioning, democratic control, and
professional management of cooperative societies (Article 43B).
• Hence, option (c) is the correct answer.

Q 36.A
• The Act marked a second milestone towards a completely responsible government in India. It was a
lengthy and detailed document having 321 Sections and 10 Schedules.
• The features of this Act were as follows:
o It provided for the establishment of an All-India Federation consisting of provinces and
princely states as units. The Act divided the powers between the Centre and units in terms of
three lists–Federal List (for Centre, with 59 items), Provincial List (for provinces, with 54 items) and
the Concurrent List (for both, with 36 items). Hence statement 1 is correct.
▪ Residuary powers were given to the Viceroy. However, the federation never came into being as
the princely states did not join it.
o It abolished dyarchy in the provinces and introduced ‘provincial autonomy’ in its place. The provinces
were allowed to act as autonomous units of administration in their defined spheres.
▪ Moreover, the Act introduced responsible Governments in provinces, that is, the Governor was
required to act with the advice of ministers responsible to the provincial legislature. This came
into effect in 1937 and was discontinued in 1939.
o It provided for the adoption of dyarchy at the Centre. Consequently, the federal subjects were divided
into reserved subjects and transferred subjects. However, this provision of the Act did not come into
operation at all.
o It introduced bicameralism in six out of eleven provinces. Thus, the legislatures of Bengal,
Bombay, Madras, Bihar, Assam and the United Provinces were made bicameral consisting of a
legislative council (upper house) and a legislative assembly (lower house). However, many
restrictions were placed on them. Hence statement 2 is not correct.
o It further extended the principle of communal representation by providing separate electorates for
depressed classes (Scheduled Castes), women and labour (workers).
o It abolished the Council of India, established by the Government of India Act of 1858. The secretary
of state for India was provided with a team of advisors.
o It extended franchise. About 10 per cent of the total population got the voting right.
o It provided for the establishment of a Reserve Bank of India to control the currency and credit of the
country.
o It provided for the establishment of not only a Federal Public Service Commission, but also a
Provincial Public Service Commission and Joint Public Service Commission for two or more
provinces.
o It provided for the establishment of a Federal Court, which was set up in 1937.

Q 37.C
• A ‘constitutional tort’ is a violation of one’s constitutional rights, particularly fundamental rights, by
an agent of the government, acting in his/her official capacity.
• In India, there is no legislation, which governs the liability of the State for the torts committed by its
servants. It is Article 300 of the Constitution of India, which enumerates the liability of the Union or
State in the tortious act of the Government. Hence, statement 1 is correct.
• A court of law can award monetary compensation to the victim in such a case. for eg: in Supreme
Court in State of Rajasthan V. Mst. Vidyawati Case, the Court ordered the defendant, i.e., the State of
Rajasthan to pay compensation of Rs. 15,000 to the plaintiff. Hence, statement 2 is correct.
• Recently, Supreme Court Justice Ramasubramanian said “no one can either be taxed or penalized for
holding an opinion which is not in conformity with the constitutional values. It is only when his opinion
gets translated into action and such action results in injury or harm or loss that an action in tort
will lie."

Q 38.C
• Democracy is a form of government in which the rulers are elected by the people. Features of democracy
often include
o freedom of assembly
o association, property rights
o freedom of religion and speech
15 www.visionias.in ©Vision IAS

FREE BY KING R QUEEN P [ऋषभ राजपूत]


o inclusiveness and equality
o citizenship
o consent of the governed
o voting rights
o freedom from unwarranted governmental deprivation of the right to life and liberty, and minority
rights.
• In some political systems, people may have elected their representatives to the national and provincial
assemblies but those elected representatives are not really the rulers. They cannot take the final decisions.
The power to take final decisions rests with other entities like army. This happens in many dictatorships
and monarchies.
o But in a democracy, the final decision-making power must rest with those elected by the people.
Hence statement 1 is correct.
• In China, elections are regularly held after every five years for electing the country’s parliament. But these
are not free and fair as the ruling communist party can never lose. But in a democracy, everyone should
have an equal opportunity to contest elections and be able to win them if desired by the people. Thus
ruling party in a democracy must not have any advantage over the opposition parties and must
have a fair chance of losing elections if desired by the people. Hence statement 2 is correct.
• In Saudi Arabia women do not have the right to vote. Estonia has made its citizenship rules in such a way
that people belonging to Russian minority find it difficult to get the right to vote.
• Democracy is based on a fundamental principle of political equality. But in a democracy political
equality is emphasized and each adult citizen must have one vote and each vote must have one
value. Hence statement 3 is correct.
• In a democracy, everyone should be equal in the eyes of law. People's rights must be protected and a
democratic government rules within limits set by constitutional law and citizens’ rights. Hence
statement 4 is not correct.

Q 39.B
• Liberal-Intellectual Principles The principles included in this category represent the ideology of
liberalism.
• They direct the state:
o To secure for all citizens a uniform civil code throughout the country (Article 44).
o To provide early childhood care and education for all children until they complete the age of six years
(Article 45).
o To organise agriculture and animal husbandry on modern and scientific lines (Article 48).
o To protect and improve the environment and to safeguard forests and wild life (Article 48 A).
o To protect monuments, places, and objects of artistic or historic interest which are declared to be of
national importance (Article 49).
o To separate the judiciary from the executive in the public services of the State (Article 50).
o To promote international peace and security and maintain just and honourable relations between
nations; to foster respect for international law and treaty obligations, and to encourage settlement of
international disputes by arbitration (Article 51).
• Prohibition of the slaughter of cows and the Promotion of Cooperative societies are based on
Gandhian ideology.
• Securing opportunities for the healthy development of children is based on an ideology of socialism.
• Hence, option (b) is the correct answer.

Q 40.A
• In the nineteenth century struggles for democracy often centered around political equality, freedom and
justice. One major demand was the right for every adult citizen to vote. Many European countries that
were becoming more democratic did not initially allow all people to vote.
o In some countries only people owning property had the right to vote. Often women did not have the
right to vote. In the United States of America, the blacks all over the country could not exercise the
right to vote until 1965.
• India granted Universal Adult Franchise to its citizens in 1950, when the Constitution of India came into
force declaring India a democratic republic. It is provided under the Article 326 of the Indian Constitution.

16 www.visionias.in ©Vision IAS

FREE BY KING R QUEEN P [ऋषभ राजपूत]


• Hence option (a) is the correct answer.

Q 41.D
• The Constituent Assembly held its first meeting on December 9, 1946. On December 13,
1946, Jawaharlal Nehru moved the historic ‘Objectives Resolution’ in the Assembly. It laid down the
fundamentals and philosophy of the constitutional structure. This Resolution was unanimously adopted by
the Assembly on January 22, 1947.
• After the acceptance of the Mountbatten Plan of June 3, 1947, for the partition of the country, the
representatives of most of the other princely states took their seats in the Assembly. The members of the
Muslim League from the Indian Dominion also entered the Assembly.
o The Assembly was made a fully sovereign body, which could frame any Constitution it pleased.
o Two separate functions were assigned to the Assembly, that is, making of the Constitution for free
India and enacting ordinary laws for the country. Whenever the Assembly met as the Constituent
body it was chaired by Dr. Rajendra Prasad and when it met as the legislative body , it
was chaired by G.V. Mavlankar. Hence statement 1 is not correct.
o The Muslim League members (hailing from the areas included in Pakistan) withdrew from the
Constituent Assembly for India. Consequently, the total strength of the Assembly came down to 299
as against 389 originally fixed in 1946 under the Cabinet Mission Plan.
• In addition to the making of the Constitution and enacting of ordinary laws, the Constituent Assembly
also performed the following functions:
o It ratified India’s membership of the Commonwealth in May 1949.
o It adopted the national flag on July 22, 1947.
o It adopted the national anthem on January 24, 1950.
o It adopted the national song on January 24, 1950. Hence statement 2 is not correct.
o It elected Dr. Rajendra Prasad as the first President of India on January 24, 1950.
• In all, the Constituent Assembly had 11 sessions over two years, 11 months and 18 days. On January 24,
1950, the Constituent Assembly held its final session. It, however, did not end, and continued as the
provisional parliament of India from January 26, 1950, till the formation of new Parliament after
the first general elections in 1951–52.
Q 42.B
• The National Commission for SCs and STs came into being consequent upon the passing of the 65th
Constitutional Amendment Act of 1990. The Commission was established under Article 338 of the
Constitution with the objective of monitoring all the safeguards provided for the SCs and STs under the
Constitution or other laws.
• In order to safeguard the interests of the STs more effectively, it was proposed to set up a separate
National Commission for STs by bifurcating the existing combined National Commission for SCs and
STs. This was done by passing the 89th Constitutional Amendment Act of 2003. This Act further
amended Article 338 and inserted a new Article 338-A in the Constitution. Hence, statement 2 is
correct.

17 www.visionias.in ©Vision IAS

FREE BY KING R QUEEN P [ऋषभ राजपूत]


• The separate National Commission for STs came into existence in 2004. It consists of a chairperson, a
vice-chairperson, and three other members. They are appointed by the President by warrant under his
hand and seal. Their conditions of service and tenure of office are also determined by the
President. Hence statement 1 is not correct.
• The Commission is vested with the power to regulate its own procedure.
• The Commission, while investigating any matter or inquiring into any complaint, has all the powers of a
civil court trying a suit and in particular in respect of the following matters:
o summoning and enforcing the attendance of any person from any part of India and examining him on
oath;
o requiring the discovery and production of any document;
o receiving evidence on affidavits;
o requisitioning any public record from any court or office;
o issuing summons for the examination of witnesses and documents; and
o any other matter which the President may determine.

Q 43.B
• Domestic Systemically Important Banks (D-SIB) means that the bank is too big to fail. According to
the RBI, some banks become systemically important due to their size, cross-jurisdictional activities,
complexity, and lack of substitute and interconnection. Banks whose assets exceed 2% of GDP are
considered part of this group.
• The D-SIB framework requires the Reserve Bank of India (RBI) to disclose the names of banks
designated as D-SIBs starting from 2015 and place these banks in appropriate buckets depending upon
their Systemic Importance Scores (SISs).
• Recently, the Reserve Bank of India declared that the State Bank of India, ICICI Bank, and HDFC
Bank will continue to be identified as Domestic Systemically Important Banks (D-SIBs) in India for year
2023.
• Hence, option (b) is the correct answer.

Q 44.C
• Article 21 declares that no person shall be deprived of his life or personal liberty except according to the
procedure established by law. This right is available to both citizens and non-citizens.
• The Supreme Court has reaffirmed its judgement in the Menaka case in the subsequent cases.
• It has declared the following rights as part of Article 21:
o Right to live with human dignity.
o Right to a decent environment including pollution-free water and air and protection against hazardous
industries.
o Right to livelihood.
o Right to privacy.
o Right to shelter.
o Right to health.
o Right to free education up to 14 years of age.
o Right to free legal aid.
o Right against solitary confinement.
o Right to a speedy trial.
o Right against handcuffing.
o Right against inhuman treatment.
o Right against delayed execution.
o Right to travel abroad. Hence, option 2 is correct.
o Right against bonded labour.
o Right against custodial harassment.
o Right to emergency medical aid.
o Right to timely medical treatment in government hospitals.
o Right not to be driven out of a state. Hence, option 1 is correct.
o Right to a fair trial.
o Right of the prisoner to have necessities of life.
o Right of women to be treated with decency and dignity.
o Right against public hanging.
o Right to the road in hilly areas. Hence, option 4 is correct.
o Right to information.
18 www.visionias.in ©Vision IAS

FREE BY KING R QUEEN P [ऋषभ राजपूत]


o Right to reputation.
o Right of appeal from a judgement of conviction
o Right to family pension
o Right to social and economic justice and empowerment
o Right against bar fetters
o Right to appropriate life insurance policy
o Right to sleep
o Right to freedom from noise pollution
o Right to sustainable development
o Right to opportunity.
• Freedom of Speech and Expression (Article 19(1)(a))
o It implies that every citizen has the right to express his views, opinions, belief and convictions freely
by word of mouth, writing, printing, picturing or in any other manner. The Supreme Court held that
the freedom of speech and expression includes the following:
▪ Right to propagate one’s views as well as the views of others. (b) Freedom of the press.
▪ Freedom of commercial advertisements.
▪ Right to telecast, the government has no monopoly on electronic media.
▪ Right against bundh called by a political party or organisation. (g) Right to know about
government activities.
▪ Freedom of silence. Hence, option 3 is not correct.
▪ Right against the imposition of pre-censorship on a newspaper. (j) Right to demonstration or
picketing but not right to strike.
o The State can impose reasonable restrictions on the exercise of the freedom of speech and expression
on the grounds of sovereignty and integrity of India, security of the state, friendly relations with
foreign states, public order, decency or morality, contempt of court, defamation, and incitement to an
offence.
Q 45.D
• Right to Education:
• Article 21 A declares that the State shall provide free and compulsory education to all children of the age
of six to fourteen years in such a manner as the State may determine. Thus, this provision makes only
elementary education a Fundamental Right and not higher or professional education. Hence, option
(d) is the correct answer.
• This provision was added by the 86th Constitutional Amendment Act of 2002. This amendment is a
major milestone in the country’s aim to achieve ‘Education for All’. The government described this step
as ‘the dawn of the second revolution in the chapter of citizens’ rights’.
• Even before this amendment, the Constitution contained a provision for free and compulsory
education for children under Article 45 in Part IV. Hence, option (b) is not correct.
• However, being a directive principle, it was not enforceable by the courts. Now, there is scope for
judicial intervention in this regard. This amendment changed the subject matter of Article 45 in
directive principles. It now reads–‘The state shall endeavour to provide early childhood care and
education for all children until they complete the age of six years.’ Hence, option (c) is not correct.
• It also added a new fundamental duty under Article 51A that reads–‘It shall be the duty of every citizen of
India to provide opportunities for education to his child or ward between the age of six and fourteen
years’.
• In 1993 itself, the Supreme Court recognised a Fundamental Right to primary education in the right to life
under Article 21. It held that every child or citizen of this country has a right to free education until he
completes the age of 14 years.
• Thereafter, his right to education is subject to the limits of the economic capacity and development of the
state. In this judgement, the Court overruled its earlier judgement (1992) which declared that there was a
fundamental right to education up to any level including professional education like medicine and
engineering.
• In pursuance of Article 21A, the Parliament enacted the Right of Children to Free and Compulsory
Education (RTE) Act, 2009. This Act seeks to provide that every child has a right to be provided
full time elementary education of satisfactory and equitable quality in a formal school which
satisfies certain essential norms and standards. Hence, option (a) is not correct.
• This legislation is anchored in the belief that the values of equality, social justice and democracy and the
creation of a just and humane society can be achieved only through the provision of inclusive elementary
education to all.

19 www.visionias.in ©Vision IAS

FREE BY KING R QUEEN P [ऋषभ राजपूत]


Q 46.A
• Hawaiian volcano kilauea has erupted again recently.
• Kilauea volcano is active shield volcano (erupts magma of low viscosity and hence less steep) located
on the Hawaiian island. Hawaii, USA is a group of 8 volcanic islands in the central Pacific Ocean
stretching in a crescent from Kure Island in west to Hawaii in east. Volcanoes make up 51% of Hawaii
Island landmass. Out of 8, five main volcanoes are considered active. Four of these active volcanoes are
located on Big Island. They include Kilauea, Mauna Loa, Mauna Kea, and Hualalai.
• Kilauea, also called Mount Kilauea, the world’s most active volcanic mass, located on the
southeastern part of the island of Hawaii, Hawaii state, U.S. The central feature of Hawaii Volcanoes
National Park, Kilauea (“Much Spreading” in Hawaiian), is an elongated dome built of lava eruptions
from a central crater and from lines of craters extending along east and southwest rifts, or fissures.
• Kilauea’s slopes merge with those of the nearby volcano Mauna Loa on the west and north. In a
series of eruptions that began in 1983 and continued into the early 21st century, Kilauea produced a
river of flowing lava that reached the sea 10 miles (16 km) south of the volcano.
• Hence, option (a) is the correct answer.
• Mount Stromboli is one of the four active volcanoes in Italy. It is one of the eight Aeolian Islands, a
volcanic arc north of Sicily.
• Mount Vesuvius is a stratovolcano located on the Gulf of Naples in Campania, Italy, about 9 km east of
Naples and a short distance from the shore. It is one of several volcanoes forming the Campanian volcanic
arc.
• Mount Etna, or simply Etna, is an active stratovolcano on the east coast of Sicily, Italy, in the
Metropolitan City of Catania, between the cities of Messina and Catania. It lies above the convergent plate
margin between the African Plate and the Eurasian Plate.

Q 47.C
• National Emergency has been proclaimed three times so far– in 1962, 1971, and 1975. Hence,
statement 1 is not correct.
• The First proclamation of National Emergency was issued in October 1962 on account of Chinese
aggression in the NEFA (North-East Frontier Agency–now Arunachal Pradesh) and was in force till
January 1968. Hence, a fresh proclamation was not needed at the time of the war against Pakistan in
1965. Hence, statement 2 is correct.
• The Second proclamation of national emergency was made in December 1971 in the wake of an attack
by Pakistan. Even when this Emergency was in operation. Hence, statement 3 is not correct.
• A Third proclamation of National Emergency was made in June 1975. Both the second and third
proclamations were revoked in March 1977.
• The first two proclamations (1962 and 1971) were made on the ground of ‘external aggression’,
while the third proclamation (1975) was made on the ground of ‘internal disturbance’, that is,
certain persons have been inciting the police and the armed forces against the discharge of their duties and
their normal functioning.

Q 48.B
• The World Economic Forum has recently published the Global Risks Report 2023.
• The report is published annually based on Global Risks Perception Survey.
• It highlights key risks across five categories: Economic, Environmental, Geopolitical, Societal, and
Technological. Global risk is defined as the possibility of the occurrence of an event or condition that, if it
occurs, would negatively impact a significant proportion of global GDP, population, or natural resources.
• Key findings:
o The top 5 risks for India are Digital inequality, Geopolitical contest for resources, Cost-of-living
crisis, Debt crises, Natural disasters, and Extreme weather events over the short and medium term.
o The cost of living dominates global risks in the next 2 years while climate action failure dominates the
next decade.
o The economic effects of COVID-19 and the Ukraine war resulted in rising inflation, rapid
normalization of monetary policies, low-growth, low-investment era.
o Technology such as AI, quantum computing and biotechnology, etc. will exacerbate inequalities and
the digital divide.
• Hence option (b) is the correct answer.

20 www.visionias.in ©Vision IAS

FREE BY KING R QUEEN P [ऋषभ राजपूत]


Q 49.B
• Preamble
o “We, THE PEOPLE OF INDIA, having solemnly resolved to constitute India into a SOVEREIGN
SOCIALIST SECULAR DEMOCRATIC REPUBLIC and to secure to all its citizens: JUSTICE,
Social, Economic and Political; LIBERTY of thought, expression, belief, faith and worship;
EQUALITY of status and of opportunity; and to promote among them all; FRATERNITY
assuring the dignity of the individual and the unity and integrity of the Nation; IN OUR
CONSTITUENT ASSEMBLY this twenty-sixth day of November 1949, do HEREBY ADOPT,
ENACT AND GIVE TO OURSELVES THIS CONSTITUTION”.
o Sources of the Constitution:
▪ Fundamental duties and the ideal of justice (social, economic, and political) in the Preamble
have been taken from Soviet Constitution (USSR, now Russia). Hence, statement 2 is
correct.
▪ Republic and the ideals of liberty, equality and fraternity in the Preamble have been taken
from French Constitution. Hence, statement 1 is not correct.

Q 50.A
• The role of CAG in the auditing of public corporations is limited. Broadly speaking, his relationship with
public corporations falls into the following three categories:
o Some corporations are audited totally and directly by the CAG, for example, Damodar Valley
Corporation, Oil and Natural Gas Commission, and others. Hence option 2 is correct.
o Some other corporations are audited by private professional auditors who are appointed by the Central
Government in consultation with the CAG. If necessary, the CAG can conduct a supplementary
audit. Examples are Central Warehousing Corporation, Industrial Finance Corporation, and others.
o Some other corporations are totally subjected to private audits. In other words, their audit is done
exclusively by private professional auditors and the CAG does not come into the picture at all. They
submit their annual reports and accounts directly to the Parliament. Examples of such corporations are
the Life Insurance Corporation of India, Reserve Bank of India, State Bank of India, Food
Corporation of India, and others. Hence options 1, 3, and 4 are not correct.
• The role of the CAG in the auditing of Government companies is also limited. They are audited by private
auditors who are appointed by the Government on the advice of the CAG. The CAG can also undertake a
supplementary audit or test audit of such Companies.

Q 51.A
• Four years since the introduction of the National Clean Air Programme (NCAP) — India’s first
national policy on curbing air pollution — air quality has improved in only 49 out of 131 cities in
FY 21-22, compared to the previous year, according to a report by the Centre for Research on
Energy and Clean Air (CREA). Less than 50% utilisation of total funds is released under NCAP.
• PRANA (Portal for Regulation of Air-pollution in Non-Attainment cities) is a portal for monitoring of
the implementation of the National Clean Air Programme (NCAP). Hence, option (a) is the correct
answer.
o It will support tracking of physical as well as financial status of city air action plan
implementation and disseminate information on air quality management efforts under NCAP to
public.
o It would also disseminate information on air quality to the public while also providing comprehensive
information related to NCAP programme details, progress, city action plans, implementation updates
by city/state/ national level agencies, air quality data, and trends, etc.
• About NCAP
• Launched in 2019, it is a time-bound, national level strategy from Central Government to tackle air
pollution problem across country in a comprehensive manner.
• It aims a 40% reduction in Particulate Matter (PM) concentrations by 2026 (base year 2017) in 131
cities (123 non-attainment cities and 42 million plus cities/urban agglomerations; 34 cities are common in
both categories).

Q 52.D
• The Constitution confers the following rights and privileges on the citizens of India (and denies the
same to aliens):
o Right against discrimination on grounds of religion, race, caste, sex or place of birth (Article
15). Hence option 1 is correct.
21 www.visionias.in ©Vision IAS

FREE BY KING R QUEEN P [ऋषभ राजपूत]


o Right to equality of opportunity in the matter of public employment (Article 16).
o Right to freedom of speech and expression, assembly, association, movement, residence, and
profession (Article 19). Hence option 2 is correct.
o Cultural and educational rights (Articles 29 and 30).
o Right to vote in elections to the Lok Sabha and state legislative assembly.
o Right to contest for membership of the Parliament and the state legislature.
o Eligibility to hold certain public offices, that is, President of India, Vice-President of India, judges of
the Supreme Court and the high courts, Governor of states, Attorney General of India, and
Advocate General of states. Hence options 3 and 4 are correct.

Q 53.D
• Amendment to the Constitution requires two different kinds of special majorities:
• in the first place, those voting in favor of the amendment bill should constitute at least half of the
total strength of that House.
• Secondly, the supporters of the amendment bill must also constitute two-thirds of those who actually
take part in voting.
• Both Houses of Parliament must pass the amendment bill separately in this same manner (there is no
provision for a joint session). For every amendment bill, this special majority is required.
• In the Rajya Sabha, there are 245 members. Therefore, any amendment must be supported by a
minimum of 123 members. Even if only 150 members are present at the time of voting, the
amendment bill must get the support of 123 out of them.
• Hence, option (d) is the correct answer.

Q 54.A
• The Constituent Assembly appointed a number of committees to deal with different tasks of constitution-
making. Out of these, eight were major committees and the others were minor committees.
o Union Powers Committee - Jawaharlal Nehru. Hence pair 1 is correctly matched.
o Union Constitution Committee - Jawaharlal Nehru
o Provincial Constitution Committee - Sardar Patel. Hence pair 3 is not correctly matched.
o Drafting Committee - Dr. B.R. Ambedkar
o Advisory Committee on Fundamental Rights, Minorities and Tribal and Excluded Areas - Sardar
Patel.
▪ This committee had the following five sub-committees:
▪ Fundamental Rights Sub-Committee - J.B. Kripalani
▪ Minorities Sub-Committee - H.C. Mukherjee
▪ North-East Frontier Tribal Areas and Assam Excluded & Partially Excluded Areas Sub-
Committee -Gopinath Bardoloi
▪ Excluded and Partially Excluded Areas (other than those in Assam) Sub-Committee - A.V.
Thakkar
▪ North-West Frontier Tribal Areas Sub-Committee8a
o Rules of Procedure Committee - Dr. Rajendra Prasad
o States Committee (Committee for Negotiating with States) - Jawaharlal Nehru.
o Steering Committee - Dr. Rajendra Prasad. Hence pair 2 is not correctly matched.
• Hence option (a) is the correct answer.

Q 55.C
• CULTURAL AND EDUCATIONAL RIGHTS
o Protection of Interests of Minorities
▪ Article 29 provides that any section of the citizens residing in any part of India having a distinct
language, script or culture of its own, shall have the right to conserve the same.
▪ Further, no citizen shall be denied admission into any educational institution maintained by the
State or receiving aid out of State funds on grounds only of religion, race, caste, or language.
▪ The first provision protects the right of a group while the second provision guarantees the right of
a citizen as an individual irrespective of the community to which he belongs.
▪ Article 29 grants protection to both religious minorities as well as linguistic minorities.
▪ However, the Supreme Court held that the scope of this article is not necessarily restricted
to minorities only, as it is commonly assumed to be. This is because of the use of the words
‘section of citizens in the Article that includes minorities as well as the majority. Hence,
statement 1 is correct.
22 www.visionias.in ©Vision IAS

FREE BY KING R QUEEN P [ऋषभ राजपूत]


▪ The Supreme Court also held that the right to conserve the language includes the right to agitate
for the protection of the language. Hence, the political speeches or promises made for the
conservation of the language of a section of the citizens do not amount to corrupt practice under
the Representation of the People Act, 1951.
o Right of Minorities to Establish and Administer Educational Institutions
▪ Article 30 grants the following rights to minorities, whether religious or linguistic:
❖ All minorities shall have the right to establish and administer educational institutions of
their choice. Hence, statement 2 is correct.
❖ The compensation amount fixed by the State for the compulsory acquisition of any property
of a minority educational institution shall not restrict or abrogate the right guaranteed to them.
This provision was added by the 44th Amendment Act of 1978 to protect the right of
minorities in this regard. The Act deleted the right to property as a Fundamental Right
(Article 31).
❖ In granting aid, the State shall not discriminate against any educational institution managed
by a minority.
❖ Thus, the protection under Article 30 is confined only to minorities (religious or linguistic)
and does not extend to any section of citizens (as under Article 29). However, the term
‘minority’ has not been defined anywhere in the Constitution. The right under Article 30 also
includes the right of a minority to impart education to its children in its own language.

Q 56.C
• The Citizenship Act (1955) prescribes three ways of losing citizenship whether acquired under the Act
or prior to it under the Constitution, viz, renunciation, termination, and deprivation:
o By Renunciation: Any citizen of India of full age and capacity can make a declaration renouncing his
Indian citizenship. Upon the registration of that declaration, that person ceases to be a citizen of India.
However, if such a declaration is made during a war in which India is engaged, its registration shall be
withheld by the Central Government. Further, when a person renounces his Indian citizenship, every
minor child of that person also loses Indian citizenship. Hence, statement 1 is correct.
o However, when such a child attains the age of eighteen, he may resume Indian citizenship.
o By Termination: When an Indian citizen voluntarily (consciously, knowingly and without duress,
undue influence, or compulsion) acquires citizenship of another country, his Indian citizenship
automatically terminates. Hence, statement 2 is correct. This provision, however, does not apply
during a war in which India is Engaged.
o By Deprivation: It is a compulsory termination of Indian citizenship by the Central government, if:
▪ the citizen has obtained citizenship by fraud:
▪ the citizen has shown disloyalty to the Constitution of India:
▪ the citizen has unlawfully traded or communicated with the enemy during a war;
▪ the citizen has, within five years after registration or naturalization, been imprisoned in any
country for two years; and
▪ the citizen has been ordinarily resident out of India for seven years continuously.

Q 57.C
• The Protection of Human Rights Act of 1993 provides for the creation of not only the National Human
Rights Commission but also a State Human Rights Commission at the state level. Accordingly, twenty-six
states have constituted the State Human Rights Commissions through Official Gazette Notifications.
• A State Human Rights Commission can inquire into violation of human rights only in respect of
subjects mentioned in the State List (List-II) and the Concurrent List (List-III) of the Seventh Schedule
of the Constitution. Hence, statement 1 is correct.
• However, if any such case is already being inquired into by the National Human Rights Commission or
any other Statutory Commission, then the State Human Rights, Commission does not inquire into that
case. Hence, statement 2 is correct.
• The central government may confer upon the State Human Rights Commissions the functions relating to
human rights being discharged by the union territories, except the union territory of Delhi. The
functions relating to human rights in the case of the union territory of Delhi are to be dealt with by the
National Human Rights Commission.

23 www.visionias.in ©Vision IAS

FREE BY KING R QUEEN P [ऋषभ राजपूत]


Q 58.C
• The composition of the NITI Aayog is as follows:
o Chairperson: The Prime Minister of India
o Governing Council: It comprises the Chief Ministers of all the States, Chief Ministers of Union
Territories with Legislatures (i.e., Delhi, Puducherry, and Jammu and Kashmir), and Lt.Governors of
other Union Territories.
o Regional Councils: These are formed to address specific issues and contingencies impacting more
than one state or region. These are formed for a specified tenure. These are convened by the Prime
Minister and comprised of the Chief Ministers of States and Lt. Governors of Union Territories in the
region. These are chaired by the Chairperson of the NITI Aayog or his nominee. Hence,
statement 3 is not correct.
o Special Invitees: Experts, specialists, and practitioners with relevant domain knowledge as special
invitees nominated by the Prime Minister.
• Full-time Organisational Framework: It comprises, in addition to the Prime Minister as the Chairperson:
o Vice-Chairperson: He is appointed by the Prime Minister. Hence, statement 1 is
correct. He enjoys the rank of a Cabinet
Minister. Hence, statement 2 is not correct.
o Members: Full-time. They enjoy the rank of a Minister of State.
o Part-time Members: Maximum of 2, from leading university research organizations and other relevant
institutions in an ex-officio capacity. Part-time members would be on a rotation.
o Ex-Officio Members: Maximum of 4 members of the Uni Council of Ministers to be nominated by
the Prime Minister.
o Chief Executive Officer: He is appointed by the Prime Minister for a fixed tenure, in the rank of
Secretary to the Government of India.
o Secretariat: As deemed necessary.

Q 59.C
• The presidential system elects a head of government independently of the legislature, while in contrast,
the head of government in a parliamentary system answers directly to the legislature.
• Features:
o Real head of the state: in this system the head of the state is the real executive head. Hence
statement 1 is correct.
o Separation of powers: the presidential form of government is based on the principle of separation of
power among the three organs of the government. The executive is not responsible to
legislature. The executive cannot dissolve the legislature. And the judiciary is independent from
executive and legislature.
o Principle of checks and balances: All the three organs of the government are separated from each
other but all are checking each other and restraining each other from any type of transgression of their
power and functions.
o Political homogeneity not necessary: it is not necessary that all cabinet should belong to same
political party.In a presidential system, the president is the head of state and head of government.
• In this system the office of president is very powerful, both in theory and practice. Countries with such a
system include the United States, Brazil and most Latin American countries. Hence statement 3 is
correct.
• In this system, president is usually elected directly by the people. It is based on the separation of powers
and president is elected independently of the legislature. Thus he is not accountable to the legislature.
Hence statement 2 is not correct.
Q 60.C
• Recent Context: The United Nations has declared 2023 as the International Year of Millets.
• Millet is a collective term referring to a number of small-seeded annual grasses that are cultivated as
grain crops, primarily on marginal lands in dry areas in temperate, subtropical, and tropical regions. Some
of the common millets available in India are Ragi (Finger millet), Jowar (Sorghum), Sama (Little millet),
Bajra (Pearl millet), and Variga (Proso millet).
• India is the largest producer of millet in the world accounting for 20% of global production and 80%
of Asia’s production.
• In India, the national average per-hectare yield is roughly 1 tonne for jowar, 1.5 tonnes for bajra and 1.7
tonnes for ragi, as against 3.5 tonnes for wheat and 4 tonnes for paddy. Hence, statement 1 is correct.
• Between 2003-04 and 2021-22, India’s millet output has actually fallen from 21.32 million tonnes (mt)
to 15.92 mt. Hence, statement 2 is correct.
24 www.visionias.in ©Vision IAS

FREE BY KING R QUEEN P [ऋषभ राजपूत]


Q 61.C
• Recently BIS celeberated its 75th foundation day. On this occassion, various Initiatives were
launched:
o Standards National Action Plan (SNAP) 2022- 27: SNAP to meet the emerging technologies and
concerns of sustainability and climate change. It will play an important role in steering the national
standardization efforts.
o Revised National Electrical Code of India 2023 or NEC 2023: Prepared by BIS, it provides
guidelines for regulating Electrical Installations practices across country. India’s first NEC was
formulated in 1985 (subsequently revised in 2011). Current revision includes latest International best
practices and addition of special locations like Hospitals, Community facilities, Hotels, etc.
o Revision Exercise of National Building Code of India (NBC 2016): BIS initiated exercise of
Revision of NBC to include Sustainable city planning norms; New and sustainable building materials,
etc.
o Standards Clubs in Schools: Through Standards Clubs, BIS aims to expose science students of class
9th and above to the concepts of Quality and Standardization through student centric activities.
o Training courses for national capacity building for judicious implementation of NBC 2016 and NEC
2023
• BIS was established under the BIS Act 1986, assuming the functions of the erstwhile Indian
Standards Institution (ISI) which was established in 1947. Presently BIS operates under the
framework of the BIS Act of 2016. Hence, statement 1 is correct.
o It is the national Standards Body of India working under the aegis of Ministry of Consumer Affairs,
Food & Public Distribution, Government of India. Hence, statement 2 is correct.
o BIS through its core activities of standardization and conformity assessment, has been benefiting the
economy by providing safe and reliable and quality goods; minimizing health hazards to consumers;
protecting the environment, promoting exports and imports substitute; controlling proliferation of
varieties etc

Q 62.A
• The various features or provisions of the National Investigation Agency (NIA) Amendment Act, 2019 are
as follows :
o It applied the provisions of the NIA Act also to persons who commit a scheduled offence beyond
India against Indian citizens or affect the interest of India. Hence, statement 1 is correct.
o It provided that the officers of the NIA shall have similar powers, duties, privies and liabilities being
exercised by the police officers in connection with the investigation of offences, not only in India but
also outside India.
o It empowered the central government, with respect to a scheduled offence committed outside India, to
direct the NIA to register the case and take up an investigation as if such an offence has taken place in
India.
o It provided that the central government and the state governments may designate Sessions
Courts as Special Courts for conducting the trial of offences under the NIA Act. Hence,
statement 2 is not correct.
o It inserted certain new offences in the Schedule of the NIA Act.
• The Act extends to the whole of India and it applies also
o to citizens of India outside India;
o to persons in the service of the Government wherever they may be;
o to persons on ships and aircraft registered in India wherever they may be; and
o to persons who commit a Scheduled Offence beyond India against an Indian citizen or affect the
interest of India.

Q 63.C
• Federalism is a system of government in which the power is divided between a central authority and
various constituent units of the country. Usually, a federation has two levels of government. One is the
government for the entire country that is usually responsible for a few subjects of common national
interest.
o The others are governments at the level of provinces or states that look after much of the day-to-day
administering of their state. Both these levels of governments enjoy their power independent of the
other.
• Federal features of Indian constitution
o Dual Polity
25 www.visionias.in ©Vision IAS

FREE BY KING R QUEEN P [ऋषभ राजपूत]


▪ Constitution establishes a dual polity consisting the Union at the Centre and the states at the
periphery. Each is endowed with sovereign powers to be exercised in the field assigned to them
respectively by the Constitution. The Union government deals with the matters of national
importance like defence, foreign affairs, currency, communication and so on. The state
governments, on the other hand, look after the matters of regional and local importance like
public order, agriculture, health, local government and so on.
o Written Constitution
▪ It specifies the structure, organisation, powers and functions of both the Central and state
governments and prescribes the limits within which they must operate. Thus, it avoids
misunderstandings and disagreements between the two.
o Division of Powers
▪ The Constitution divided the powers between the Centre and the states in terms of the Union List,
State List and Concurrent List in the Seventh Schedule.
o Supremacy of the Constitution
▪ The Constitution is the supreme (or the highest) law of the land. The laws enacted by the Centre
and the states must conform to its provisions.
o Rigid Constitution
▪ The division of powers established by the Constitution as well as the supremacy of the
Constitution can be maintained only if the method of its amendment is rigid. Hence, the
Constitution is rigid to the extent that those provisions which are concerned with the federal
structure.
o Independent Judiciary
▪ The Constitution establishes an independent judiciary headed by the Supreme Court for two
purposes: one, to protect the supremacy of the Constitution by exercising the power of judicial
review; and two, to settle the disputes between the Centre and the states or between the states.
o Bicameralism
▪ The Constitution provides for a bicameral legislature consisting of an Upper House (Rajya Sabha)
and a Lower House (Lok Sabha). The Rajya Sabha represents the states of Indian Federation,
while the Lok Sabha represents the people of India as a whole.
• Integrated Judiciary is a unitary feature of the Indian constitution.
o The Indian Constitution has established an integrated judicial system with the Supreme Court at the
top and the state high courts below it. This single system of courts enforces both the Central laws as
well as the state laws. In US, on the other hand, there is a double system of courts whereby the federal
laws are enforced by the federal judiciary and the state laws by the state judiciary.
• Hence option (c) is the correct answer.

Q 64.D
• First Past The Post System (FPTP) is a plurality voting method, a plurality meaning the largest part of the
whole, in contrast to majority, which generally means more than half of the whole. Under FPTP the
candidate with the highest number (but not necessarily a majority) of votes is elected.
• Though the Constitution has adopted the system of proportional representation in the case of Rajya
Sabha, it has not preferred the same system in the case of Lok Sabha. Instead, it has adopted the
First-past-the-post system (territorial representation) for the election of members to the Lok Sabha.
Hence statement 1 is not correct.
• Under this system, every member of the legislature represents a geographical area known as a
constituency. From each constituency, only one representative is elected. Hence such a constituency is
known as a single-member constituency. In this system, a candidate who secures majority of votes is
declared elected. There is no need to get 50 percent of the total votes to get elected. Hence statement
2 is not correct.
o This simple majority system of representation does not represent the whole electorate. In other words,
it does not secure due representation to minorities (small groups).
• The system of proportional representation aims at removing the defects of territorial representation. Under
this system, all sections of the people get representation in proportion to their number. Even the smallest
section of the population gets its due share of representation in the legislature.
o There are two kinds of proportional representation, namely, single transferable vote system and list
system. In India, the first kind is adopted for the election of members to the Rajya Sabha and state
legislative council and for electing the President and the Vice-President.

26 www.visionias.in ©Vision IAS

FREE BY KING R QUEEN P [ऋषभ राजपूत]


Q 65.D
• Since 1950, the successive governments at the Centre and in the states have made several laws for
implementing the Directive Principles such as:
• the Payment of Bonus Act (1965),
• the Contract Labour Regulation and Abolition Act (1970),
• the Child Labour Prohibition and Regulation Act (1986),
• the Bonded Labour System Abolition Act (1976), to protect the interests of the labour sections.
• In 2006, the government banned child labour. In 2016, the Child Labour Prohibition and Regulation Act
(1986) was renamed the Child and Adolescent Labour Prohibition and Regulation Act, 1986.
• The Maternity Benefit Act (1961) and the Equal Remuneration Act (1976) have been made to protect
the interests of women workers.
• Hence option (d) is the correct answer.

Q 66.D
• The NHRC is a multi-member body consisting of a chairperson and five members. The chairperson
should be a retired chief justice of India or a judge of the Supreme Court and members should be a serving
or retired judge of the Supreme Court, a serving or retired chief justice of a high court, and three persons
(out of which at least one should be a woman) having knowledge or practical experience with respect to
human rights.
• In addition to these full-time members, the commission also has seven ex-officio members–the
chairpersons of
o the National Commission for Minorities,
o the National Commission for SCs,
o the National Commission for STs,
o the National Commission for Women,
o the National Commission for BCs and
o the National Commission for Protection of Child Rights and
o the Chief Commissioner for Persons with Disabilities.
• Hence, options 1, 3, 4 and 5 are correct.

Q 67.A
• Recent Context: Defence Acquisition Council (DAC) accorded Acceptance of Necessity (AoN) to
procure VSHORAD (IR Homing) missile system.
• Very Short-Range Air Defense System (VSHORADS) is a Man Portable Air Defence System
(MANPAD) designed and developed indigenously by DRDO’s Research Centre Imarat (RCI),
Hyderabad in collaboration with other DRDO laboratories and Indian Industry Partners. Hence
statement 2 is not correct.
o It is meant to target low-altitude aerial threats at short ranges.
o These are lightweight and portable surface-to-air missiles. Hence statement 1 is correct.
o These can be fired by individuals or small groups to destroy aircraft or helicopters.
o Propelled by a dual-thrust solid motor, it incorporates many novel technologies including a
miniaturized Reaction Control System (RCS) and integrated avionics.

Q 68.A
• 102nd Amendment Act of 2018 conferred a constitutional status on the National Commission for
Backward Classes. For this purpose, the amendment inserted a new Article 338-B in the constitution.
Hence, the Commission ceased to be a statutory body and became a constitutional body. In other words,
the constitutional status of the new Commission is at par with the National Commission for
Scheduled Castes (NCSC) and the National Commission for Scheduled Tribes (NCST). Hence,
statement 1 is correct.
• The Commission presents an annual report to the President. It can also submit a report as and when it
thinks necessary. The President places all such reports before the Parliament, along with a memorandum
explaining the action taken on the recommendations made by the Commission. The memorandum should
also contain the reasons for the non-acceptance of any of such recommendations. Hence, statement 2 is
not correct.

27 www.visionias.in ©Vision IAS

FREE BY KING R QUEEN P [ऋषभ राजपूत]


Q 69.B
• An Emergency due to the failure of the constitutional machinery in the states (Article 356). This is
popularly known as ‘President’s Rule’. It is also known by two other names–‘State Emergency’ or
‘constitutional Emergency’. However, the Constitution does not use the word ‘emergency’ for this
situation.
• The President acquires the following extraordinary powers when the President’s Rule is imposed in a
state:
o He can take up the functions of the state government and powers vested in the governor or any
other executive authority in the state. Hence statement 1 is correct.
o He can declare that the powers of the state legislature are to be exercised by the Parliament.
Hence statement 2 is correct.
o He can take all other necessary steps including the suspension of the constitutional provisions relating
to anybody or authority in the state.
• It should be noted here that the President cannot assume to himself the powers vested in the
concerned state high court or suspend the provisions of the Constitution relating to it. In other words,
the constitutional position, status, powers, and functions of the concerned state high court remain the same
even during the President’s Rule. Hence, statement 3 is not correct.

Q 70.B
• The Sovereign Green Bonds (SGrBs) are bonds issued by any sovereign entity, inter-governmental
groups or alliances and corporates with the aim that proceeds of bonds are utilised for environmentally
sustainable projects. It will help in reducing the carbon intensity of the economy.
• SGrBs were announced in Union Budget 2022-23 and framework for SGrBs was issued (by Ministry of
Finance) in November 2022.
• RBI will issue 5-year and 10-year green bonds of Rs 4,000 crore each in current financial year.
Hence, statement 1 is correct.
• The only difference between Green Bonds and other ordinary government-issued bonds is that the funds
raised from investors are only used to support initiatives that have a good influence on the environment,
such as green construction and renewable energy. Primarily these aim to contribute to the planet and its
sustainability.
• The government has tried to make them attractive for institutional investors by giving sops like issuing
them through a uniform price auction, making them eligible for Repo as also SLR purposes, and
making them eligible for trading in the secondary market. Hence, statement 2 is correct.
• Even NRIs are allowed to invest in them. The risk is further reduced in these bonds as per the
framework announced earlier where the payments of principal and interest on these bonds are not
conditional on the performance of the eligible projects and the investors in these bonds do not bear any
project-related risks. Hence, statement 3 is not correct.
• Eligible projects under SGrBs framework include projects of renewable energy, energy efficiency,
clean transportation, climate change adaptation, sustainable water and waste management, pollution
prevention and control, green buildings, and biodiversity conservation.

Q 71.D
• Under Article 324 of the Constitution of India, the Election Commission of India is vested with the
power of superintendence, direction, and control of conducting the elections to the Lok Sabha and State
Legislative Assemblies.
• Though the constitution has sought to safeguard and ensure the independence and impartiality of the
Election Commission, some flaws can be noted, viz.,
o The Constitution has not prescribed the qualifications (legal, educational, administrative or
judicial) of the members of the Election Commission. Hence, statement 1 is correct.
o The Constitution has not specified the term of the members of the Election Commission. Hence,
statement 2 is not correct.
▪ Subject to the provisions of any law made by Parliament, the conditions of service and tenure of
office of the Election Commissioners and the Regional Commissioners shall be such as
the President may by rule determine. Thus, the constitution has not determined the tenure
of election commissioners.
o The Constitution has not debarred the retiring election commissioners from any further
appointment by the Government. Hence, statement 3 is correct.

28 www.visionias.in ©Vision IAS

FREE BY KING R QUEEN P [ऋषभ राजपूत]


Q 72.D
• The Lokayukta is the Indian Parliamentary Ombudsman, executed into power, through and for, each of
the State Governments of India. It is brought into effect in a state, after passing the Lokayukta Act in
the respective state Legislature and a person of reputable background is nominated to the post. The post
is created to quickly address grievances against the working integrity and efficiency of the government or
its administration (public servants). Once appointed, Lokayukta cannot be dismissed or transferred by
the government, and can only be removed by passing an impeachment motion by the state assembly,
making it a powerful deterrent against corruption and maladministration of the governing system.
• Even much before the enactment of the Lokpal and Lokayuktas Act (2013) itself, many states had
already set up the institution of Lokayuktas. It must be noted here that the institution of lokayukta
was established first in Maharashtra in 1971. Although Odisha had passed the Act in this regard in
1970, it came into force only in 1983. Hence, statement 1 is correct.
• The lokayukta and upalokayukta are appointed by the governor of the state. While appointing, the
governor in most of the states (varies from state to state) consults
o the chief justice of the state high court, and
o the leader of the Opposition in the state legislative assembly.
o the speaker of the legislative assembly (optional)
o In case the state has a legislative council, the chairman of the legislative council and the leader of
the opposition of the legislative council are also consulted
o Hence, statement 2 is not correct.
• The recommendations made by the Lokayukta are only advisory and not binding on the state
government. Hence, statement 3 is correct.
Q 73.C
• Every state government should establish a District Disaster Management Authority (DDMA) for
every district in the state.
• A DDMA consists of a chairperson and other members, not exceeding seven. The Collector (or District
Magistrate or Deputy Commissioner) of the district is the ex-officio chairperson of the DDMA.
Hence, statement 1 is correct.
• The chief executive officer of the DDMA, the superintendent of police, and the chief medical officer of
the district are the ex-officio members of the DDMA. The chief executive officer of the DDMA
is appointed by the state government. Hence, statement 2 is correct.

Q 74.B
• The constitution can be amended by a simple majority of the two Houses of Parliament outside the scope
of Article 368.
• These provisions include:
o Admission or establishment of new states.
o Formation of new states and alteration of areas, boundaries, or names of existing states.
o Abolition or creation of legislative councils in states.
o Second Schedule—emoluments, allowances, privileges, and so on of the president, the governors, the
Speakers, judges, etc.
o Quorum in Parliament.
o Salaries and allowances of the members of Parliament.
o Rules of procedure in Parliament.
o Privileges of the Parliament, its members, and its committees.
o Use of English language in Parliament.
o Number of puisne judges in the Supreme Court.
o Conferment of more jurisdiction on the Supreme Court.
o Use of official language.
o Citizenship—acquisition, and termination.
o Elections to Parliament and state legislatures.
o Delimitation of constituencies.
o Union territories.
o Fifth Schedule—administration of scheduled areas and scheduled tribes.
o Sixth Schedule—administration of tribal areas.
• Those provisions of the Constitution which are related to the federal structure of the polity can
be amended by a special majority of the Parliament and also with the consent of half of the state
legislatures by a simple majority. Like, Election of the President and its manner.
• Hence, option (b) is the correct answer.
29 www.visionias.in ©Vision IAS

FREE BY KING R QUEEN P [ऋषभ राजपूत]


Q 75.B
• It was in 1934 that the idea of a Constituent Assembly for India was put forward for the first time
by M.N. Roy, a pioneer of communist movement in India.
o In 1935, the Indian National Congress (INC), for the first time, officially demanded a Constituent
Assembly to frame the Constitution of India.
o In 1938, Jawaharlal Nehru, on behalf the INC declared that ‘the Constitution of free India must be
framed, without outside interference, by a Constituent Assembly elected on the basis of adult
franchise’.
• The Constituent Assembly was constituted in November 1946 under the scheme formulated by the
Cabinet Mission Plan.The features of the scheme were:
o The total strength of the Constituent Assembly was to be 389. Of these, 296 seats were to be allotted
to British India and 93 seats to the princely states.
o Out of 296 seats allotted to British India, 292 members were to be drawn from the eleven governors’
provinces and four from the four Chief Commissioners’ provinces, one from each.
o Each province and princely state (or group of states in case of small states) were to be allotted
seats in proportion to their respective population. Roughly, one seat was to be allotted for every
million population. Hence statement 1 is correct.
o Seats allocated to each British province were to be divided among the three principal communities–
Muslims, Sikhs and General (all except Muslims and Sikhs), in proportion to their population.
o The representatives of each community were to be elected by members of that community in the
provincial legislative assembly and voting was to be by the method of proportional representation by
means of a single transferable vote.
o The representatives of the princely states were to be nominated by the heads of the princely
states. Hence statement 2 is correct.
• It is, thus, clear that the Constituent Assembly was to be a partly elected and partly nominated
body. Moreover, the members were to be indirectly elected by the members of the provincial assemblies,
who themselves were elected on a limited franchise.
• The elections to the Constituent Assembly (for 296 seats allotted to the British Indian Provinces)
were held in July-August 1946. The Indian National Congress won 208 seats, the Muslim League 73
seats and the small groups and independents got the remaining 15 seats. However, the 93 seats allotted to
the princely states were not filled as they decided to stay away from the Constituent Assembly. Hence
statement 3 is not correct.

Q 76.B
• At present (2019), the Eighth Schedule of the Constitution specifies 22 languages (originally 14
languages). These are
• Assamese, Bengali, Bodo, Dogri (Dongri), Gujarati, Hindi, Kannada, Kashmiri, Konkani, Mathili
(Maithili), Malayalam, Manipuri, Marathi, Nepali, Odia , Punjabi, Sanskrit, Santhali, Sindhi, Tamil,
Telugu and Urdu. Sindhi was added by the 21st Amendment Act of 1967; Konkani, Manipuri and Nepali
were added by the 71st Amendment Act of 1992; and Bodo, Dongri, Maithili and Santhali were added by
the 92nd Amendment Act of 2003.
• English is not listed in eighth schedule.
• In terms of the Constitution provisions, there are two objectives behind the specification of the above
regional languages in the Eighth Schedule:
o the members of these languages are to be given representation in the Official Language Commission;
and
o the forms, style and expression of these languages are to be used for the enrichment of the Hindi
language
• Hence option (b) is the correct answer.

Q 77.A
• Democracy is of two types–direct and indirect. In direct democracy, the people exercise their supreme
power directly as is the case in Switzerland.
o There are four devices of direct democracy, namely, Referendum, Initiative, Recall and Plebiscite.
• In indirect democracy, on the other hand, the representatives elected by the people exercise the supreme
power and thus carry on the government and make the laws. This type of democracy, also known as
representative democracy.
• Referendum is a procedure whereby proposed legislation is referred to the electorate for settlement
by their direct votes. Hence pair 1 is correctly matched.
30 www.visionias.in ©Vision IAS

FREE BY KING R QUEEN P [ऋषभ राजपूत]


• Initiative is a method by means of which the people can propose a bill to the legislature for enactment.
• Recall is a method by means of which the voters can remove a representative or an officer before
the expiry of his term, when he fails to discharge his duties properly. Hence pair 2 is not correctly
matched.
• Plebiscite is a method of obtaining the opinion of people on any issue of public importance. It is
generally used to solve the territorial disputes. Hence pair 3 is not correctly matched.

Q 78.A
• Article 1 describes India, that is, Bharat as a ‘Union of States’ rather than a ‘Federation of States’.
This provision deals with two things: one, name of the country; and two, type of polity. Hence statement
1 is not correct.
• Secondly, the country is described as ‘Union’ although its Constitution is federal in structure. According
to Dr. B.R. Ambedkar, the phrase ‘Union of States’ has been preferred to ‘Federation of States’ for two
reasons: one, the Indian Federation is not the result of an agreement among the states like the American
Federation; and two, the states have no right to secede from the federation.
• According to Article 1, the territory of India can be classified into three categories:
1. Territories of the states
2. Union territories
3. Territories that may be acquired by the Government of India at any time.
• The names of states and union territories and their territorial extent are mentioned in the first schedule
of the Constitution. At present, there are 28 states and 8 union territories. Hence statement 2 is
correct and statement 3 is not correct.

Q 79.A
• Recently, Union Cabinet approved a 20,000 crore budget for National Green Hydrogen Mission.
• Grey hydrogen is the most common form of hydrogen and is generated from natural gas, or methane,
through a process called “steam reforming”. In this process, natural gas containing methane (CH4) can
be used to produce hydrogen with thermal processes, such as steam-methane reformation and partial
oxidation. Hence, statement 1 is correct.
• Hydrogen can be produced from a range of resources including fossil fuels, nuclear energy, biomass, and
renewable energy sources. Depending on production methods, hydrogen can be grey, blue, or green and
sometimes even pink, yellow or turquoise. They’re essentially color codes, used within the energy
industry to differentiate between the types of hydrogen. Depending on the type of production used,
different colors are assigned to the hydrogen.
• Hydrogen emits only water when burnt. It derives the grey hydrogen name because of the process of
production and not on basis of the by products after burning. Hence, statement 2 is not correct.
• Green hydrogen is the only type produced in a climate-neutral manner, meaning it could play a vital
role in global efforts to reduce emissions to net zero by 2050.

Q 80.B
• The Government of India Act of 1919 was enacted and came into force in 1921. This Act is also known as
Montagu- Chelmsford Reforms (Montagu was the Secretary of State for India and Lord Chelmsford was
the Viceroy of India).
• The features of this Act were as follows:
o It relaxed central control over the provinces by demarcating and separating the central and
provincial subjects. The central and provincial legislatures were authorized to make laws on their
respective list of subjects. However, the structure of government continued to be centralized and
unitary.
o It further divided the provincial subjects into two parts– transferred and reserved. The
transferred subjects were to be administered by the Governor with the aid of Ministers responsible to
the legislative council.
▪ The reserved subjects, on the other hand, were to be administered by the Governor and his
executive council without being responsible to the legislative council. This dual scheme of
governance was known as ‘dyarchy’.
o It required that the three of the six members of the Viceroy’s executive Council (other than the
Commander-in-Chief) were to be Indian.
o It extended the principle of communal representation by providing separate electorates for Sikhs,
Indian Christians, Anglo-Indians and Europeans.
o It granted franchise to a limited number of people on the basis of property, tax or education.
31 www.visionias.in ©Vision IAS

FREE BY KING R QUEEN P [ऋषभ राजपूत]


o It created a new office of the High Commissioner for India in London and transferred to him some of
the functions hitherto performed by the Secretary of State for India.
o It provided for the establishment of a public service commission. Hence, a Central Public Service
Commission was set up in 1926 for recruiting civil servants.
o It separated, for the first time, provincial budgets from the Central budget and authorized the
provincial legislatures to enact their budgets.
• Hence option (b) is the correct answer.

Q 81.A
• Centre has named its new integrated food security scheme for providing free foodgrains to
Antodaya Ann Yojna (AAY) and Primary Household (PHH) beneficiaries under National Food
Security Act (NFSA), 2013 as Pradhan Mantri Garib Kalyan Anna Yojana (PMGKAY). Hence
statement 1 is correct.
• It’ll strengthen provisions of NFSA, 2013 in terms of accessibility, affordability, and availability of food
grains for the poor. It will subsume two subsidy schemes of the Department of Food & Public Distribution
• Food Subsidy to FCI: It is in form of a consumer subsidy (Difference between Economic cost incurred
by FCI and realized Central Issue Price) and a buffer subsidy (Cost of holding and maintaining buffer
stock).
• Food Subsidy for decentralized procurement states dealing with procurement, allocation, and delivery
of free foodgrains to states under NFSA.
• Earlier, a scheme with a similar name (PMGKAY) was implemented during the Covid-19 pandemic.
• Difference between the two schemes
o Earlier, NFSA beneficiaries were getting foodgrains entitlement (35 kg per AAY Household and 5 kg
per person to PHH) at a subsidized rates (Rs 3/kg rice, Rs 2/kg wheat, and Rs 1/kg coarse grains).
o Now, the government has done away with subsided prices and providing food grains free of cost
for a year. However, an additional quantity of 5 kg of food grains will not be provided.

Q 82.B
• The Constitution (Article 165) has provided for the office of the advocate general for the states. He is
the highest law officer in the state.
• The term of office of the advocate general is not fixed by the Constitution. Further, the Constitution
does not contain the procedure and grounds for his removal. Hence, statement 1 is not correct.
• He holds office at the pleasure of the governor. This means that he may be removed by the governor at
any time. Hence, statement 2 is correct.
• In the performance of his official duties, the advocate general is entitled to appear before any court of law
within the state. Further, he has the right to speak and to take part in the proceedings of both the Houses of
the state legislature or any committee of the state legislature of which he may be named a member, but
without a right to vote. He enjoys all the privileges and immunities that are available to a member of
the state legislature. Hence, statement 3 is correct.

Q 83.A
• Recent Context: FSSAI Survey stated that 15% of dietary supplements were found to be unsafe for
consumption.
o unsupervised consumption of dietary supplements, warning poor-quality products can affect
cardiovascular health and kidneys.
• FSSAI is under Ministry of Health & Family Welfare. Hence option (a) is the correct answer.
• It is an autonomous statutory body established under Food Safety and Standards Act (FSSA), 2006.
• FSSA 2006 consolidates various acts & orders that had earlier handled food related issues in various
Ministries.
• It regulates and monitor, manufacture, processing, distribution of food while ensuring safe and wholesome
food to consumers.
• Headquarters - Delhi

Q 84.B
• In the Waman Rao case (1981), the Supreme Court adhered to the doctrine of the ‘basic structure’
and further clarified that it would apply to constitutional amendments enacted after April 24, 1973.
Hence option (b) is the correct answer.
• The court in Waman Rao vs Union of India ultimately ruled that the First Constitutional Amendment Act
of 1951, that introduced Articles 31A, 31B, as well as the Twenty-Fifth Amendment Act that introduced
32 www.visionias.in ©Vision IAS

FREE BY KING R QUEEN P [ऋषभ राजपूत]


Article 31C were constitutional, and did not damage any basic or essential features or the basic structure
of the Constitution.
• S.R. Bommai Case (1994):
o The verdict concluded that the power of the President to dismiss a State government is not absolute.
The verdict said the President should exercise the power only after his proclamation (imposing his/her
rule) is approved by both Houses of Parliament. Till then, the Court said, the President can only
suspend the Legislative Assembly by suspending the provisions of Constitution relating to the
Legislative Assembly.
• Kuldip Nayar Case (2006):
o The case of Kuldip Nayar v. Union of India (2006) deals with the amendments to the Representation
of People's Act, 1951 where, initially it required for the electors to have a “domicile” in the State
concerned for getting elected to the Council of States, which was subsequently deleted by the said
amendment in 2003.
• Indira Gandhi Nehru Case (1975):
o The doctrine of basic structure of the constitution was reaffirmed and applied by the Supreme Court in
the Indira Nehru Gandhi case (1975). In this case, the Supreme Court invalidated a provision of the
39th Amendment Act (1975) which kept the election disputes involving the Prime Minister and the
Speaker of Lok Sabha outside the jurisdiction of all courts.

Q 85.D
• Collective responsibility is the convention under which individual members of government are held
accountable for the acts and decisions of the government as a whole.
• Under the Constitution of India, Article 75(3) deals with Collective Responsibility at the Centre (The
Council of Ministers shall be collectively responsible to the House of the People), while Article
164(3) deals with state governments. Hence, statements 1 and 2 are not correct.
• Recently, Supreme Court stated, “a statement made by a minister, even if traceable to any affairs of the
state or for protecting the government, cannot be attributed vicariously to the government by invoking the
principle of collective responsibility". In other words, the flow of stream in collective responsibility is
from the Council of Ministers to the individual Ministers and not vice versa.

Q 86.B
• Federalism is a system of government in which the power is divided between a central authority and
various constituent units of the country. Usually, a federation has two levels of government. One is the
government for the entire country that is usually responsible for a few subjects of common national
interest.
o The others are governments at the level of provinces or states that look after much of the day-to-day
administering of their state. Both these levels of governments enjoy their power independent of the
other.
• Features of federalism
o There are two or more levels (or tiers) of government.
o Different tiers of government govern the same citizens, but each tier has its own jurisdiction in
specific matters of legislation, taxation and administration.
o The jurisdictions of the respective levels or tiers of government are specified in the constitution. So
the existence and authority of each tier of government is constitutionally guaranteed.
o The fundamental provisions of the constitution cannot be unilaterally changed by one level of
government. Such changes require the consent of both levels of government.
o Courts have the power to interpret the constitution and the powers of different levels of government.
The highest court acts as an umpire if disputes arise between different levels of government in the
exercise of their respective power.
o Sources of revenue for each level of government are clearly specified to ensure its financial
autonomy.

33 www.visionias.in ©Vision IAS

FREE BY KING R QUEEN P [ऋषभ राजपूत]


• Hence option (b) is the correct answer.

Q 87.A
• Among all the committees of the Constituent Assembly, the most important committee was the
Drafting Committee set up on August 29, 1947. It was this committee that was entrusted with the task
of preparing a draft of the new Constitution. Hence statement 1 is correct.
• It consisted of seven members. They were
o Dr. B.R. Ambedkar (Chairman). Hence statement 2 is not correct.
o N. Gopalaswamy Ayyangar
o Alladi Krishnaswamy Ayyar
o Dr. K.M. Munshi
o Syed Mohammad Saadullah
o N. Madhava Rau
o T.T. Krishnamachari
• The Drafting Committee, after taking into consideration the proposals of the various committees, prepared
the first draft of the Constitution of India, which was published in February, 1948. The people of India
were given eight months to discuss the draft and propose amendments.

Q 88.C
• Recent Context: The union government has informed that cadastral maps of 35% of over six lakh
villages in the country have so far been geo-referenced under the Digital India Land Records
Modernisation Programme (DILRMP).
o Cadastral Maps are a digital form of land records that show all boundaries of different parts of
land pieces based on their length, area, and direction.
• DILRMP is implemented by the Department of Land Resources under the Ministry of Rural
Development. It is a Central Sector scheme that has been extended to 2023-24, to complete its original
targets as well as expand its ambit with a slew of new schemes. Hence statement 1 is correct.
o Its three major components: Computerization of land record, Survey/re-survey, Computerization of
Registration. Hence statement 2 is correct.

Q 89.B
• The Central Vigilance Commission (CVC) is the main agency for preventing corruption in the Central
government. It was established in 1964 by an executive resolution of the Central government. Its
establishment was recommended by the Santhanam Committee on Prevention of Corruption (1962–
64). Hence, statement 1 is not correct.
• The CVC is a multi-member body consisting of a Central Vigilance Commissioner (chairperson) and
not more than two vigilance commissioners. They are appointed by the president by warrant under his
hand and seal on the recommendation of a three-member committee consisting of the prime minister as its
head, the Union minister of home affairs, and the Leader of the Opposition in the Lok Sabha. They hold
office for a term of four years or until they attain the age of sixty-five years, whichever is earlier. After
34 www.visionias.in ©Vision IAS

FREE BY KING R QUEEN P [ऋषभ राजपूत]


their tenure, they are not eligible for further employment under the Central or state government.
Hence, statement 2 is correct.
• The salary, allowances, and other conditions of service of the Central Vigilance Commissioner are
similar to those of the Chairman of UPSC and that of the vigilance commissioner are similar to those of
a member of UPSC. But they cannot be varied to his disadvantage after his appointment. Hence,
statement 3 is not correct.

Q 90.C
• The Constitution (One Hundred And First Amendment) Act, 2016” passed on the 8th of September,
2016. Since then the GST council and been notified bringing into existence the Constitutional body to
decide issues relating to GST, inserted a new Article 279-A in the Constitution. This Article empowered
the President to constitute a GST Council by order. Accordingly, the President issued the order in 2016
and constituted the Council. Hence statement 1 is correct.
• As per Article 279A of the amended Constitution, the GST Council which will be a joint forum of the
Centre and the States shall consist of the following members: -
o Union Finance Minister - Chairperson
o The Union Minister of State, in charge of Revenue of finance - Member
o The Minister In-charge of finance or taxation or any other Minister nominated by each State
Government - Members
• As per Article 279A (4), the Council will make recommendations to the Union and the States on
important issues related to GST, like the goods and services that may be subjected or exempted from
GST, model GST Laws, principles that govern Place of Supply, threshold limits, GST rates including the
floor rates with bands, special rates for raising additional resources during natural calamities/disasters,
special provisions for certain States, etc.
• The Secretariat of the Council is located in New Delhi. The Union Revenue Secretary acts as the ex-
officio Secretary to the Council. Hence statement 2 is correct.

Q 91.C
• The National Commission for Protection of Child Rights (NCPCR) has been constituted by the
Government of India, under the Commission for Protection of Child Rights (CPCR) Act,2005 to
exercise and performs the powers and functions assigned to it under CPCR Act,2005. The Commission
works under the aegis of the Ministry of Women and Child Development. Hence statement 1 is
correct.
• The commission consists of the following members namely: A chairperson who, is a person of eminence
and has done outstanding work for promoting the welfare of children; and Six members, out of which at
least two are women are appointed by the Central Government.
• The Functions of the National Commission for Protection of Child Rights as laid out in the Commissions
for Protection of Child Rights (CPCR) Act,2005 are as follows:
o Examine and review the safeguards provided by or under any law for the time being in force for the
protection of child rights and recommend measures for their effective implementation;
o Inquire into complaints and take suo-motu notice of matters relating to:
▪ Deprivation and violation of child rights; Hence statement 2 is correct.
▪ Non-implementation of laws providing for the protection and development of children;
▪ Non-compliance of policy decisions, guidelines or instructions aimed at mitigating hardships to
and ensuring the welfare of the children and provide relief to such children;
o Inspect or cause to be inspected any juveniles custodial home, or any other place of residence or
institution meant for children, under the control of the Central Government or any State Government
or any other authority, including any institution run by a social organization.
o Spread child rights literacy among various section of society and promote awareness of the safeguards
available for protection of these rights through publications, the media, seminar and other available
means;
• Hence, both statements 1 and 2 are correct.

Q 92.C
• The Central Information Commission is a statutory body, set up under the Right to Information Act in
2005 under the Government of India to act upon complaints from those individuals who have not been
able to submit information requests to a Central Public Information Officer or State Public Information
Officer due to either the officer not have been appointed, or because the respective Central Assistant

35 www.visionias.in ©Vision IAS

FREE BY KING R QUEEN P [ऋषभ राजपूत]


Public Information Officer or State Assistant Public Information Officer refused to receive the application
for information under the Right to Information Act. Hence statement 1 is correct.
• However, the newly amended act, Right to Information Act (Amendment), 2019 empowers the Central
Government to prescribe the term of office for the commissioners as it may deem fit to the government.
Before this amendment, their term was fixed for 5 years.
• The 2019 Amendment Act also states that the salary, allowances and other service conditions of the
Chief Information Commissioner and an Information Commissioner (of Centre as well as States)
shall be such as prescribed by the Central Government. Before this amendment, the salary, allowances
and other service conditions of the Chief Information Commissioner were similar to those of the Chief
Election Commissioner and that of an Information Commissioner were similar to those of an Election
Commissioner (State Election Commissioners in case of States).
• The Commission consists of a Chief Information Commissioner and not more than ten Information
Commissioners. The Commission, when constituted initially, had five commissioners including the Chief
Information Commissioner. Hence, statement 2 is correct.
• The President can remove the Chief Information Commissioner or any Information Commissioner from
office under the following circumstances:
o if he is adjudged insolvent; or
o if he has been convicted of an offence which (in the opinion of the President) involves moral
turpitude; or
o if he engages during his term of office in any paid employment outside the duties of his office; or
o if he is (in the opinion of the President) unfit to continue in office due to infirmity of mind or body; or
o if he has acquired such financial or other interest as is likely to affect prejudicially his official
functions.
• In addition to these, the President can also remove the Chief Information Commissioner or any
Information Commissioner on the ground of proven misbehaviour or incapacity. However, in these
cases, the President has to refer the matter to the Supreme Court for an enquiry. If the Supreme
Court, after the enquiry, upholds the cause of removal and advises so, then the President can remove him.
• The Commission submits an annual report to the Central Government on the implementation of the
provisions of this Act. The Central Government places this report before each House of Parliament.

Q 93.C
• Provisions under DPSP with relevant constitutional articles:
• To promote cottage industries on an individual or cooperation basis in rural areas (Article 43)
• To prohibit the consumption of intoxicating drinks and drugs which are injurious to health (Article 47).
• To prohibit the slaughter of cows, calves, and other milch and draught cattle and to improve their
breeds (Article 48).
• To organise agriculture and animal husbandry on modern and scientific lines (Article 48).
• To protect and improve the environment and to safeguard forests and wildlife (Article 48 A).
• Hence option (c) is the correct answer.

Q 94.B
• The American Constitution was the first, to begin with, a Preamble. Many countries, including India,
followed this practice. The term ‘Preamble’ refers to the introduction or preface to the Constitution. It
contains the summary or essence of the Constitution.
• The Preamble to the Indian Constitution is based on the ‘Objectives Resolution’, drafted and moved by
Pandit Nehru, and adopted by the Constituent Assembly. It has been amended by the 42nd Constitutional
Amendment Act (1976), which added three new words–Socialist, Secular and Integrity.
• Originally (1949), the Constitution contained a Preamble, 395 Articles (divided into 22 Parts) and 8
Schedules. Hence, statement 1 is not correct.
• The Preamble reveals four ingredients or components:
o Source of the authority of the Constitution: The Preamble states that the Constitution derives its
authority from the people of India.
o Nature of Indian State: It declares India to be of a sovereign, socialist, secular democratic, and
republican polity.
o Objectives of the Constitution: It specifies justice, liberty, equality, and fraternity as the
objectives. Hence, statement 2 is correct.
o Date of adoption of the Constitution: It stipulates November 26, 1949, as the date.
• In the Berubari Union case (1960), the Supreme Court said that the Preamble shows the general purposes
behind the several provisions in the Constitution and is thus a key to the minds of the makers of the
36 www.visionias.in ©Vision IAS

FREE BY KING R QUEEN P [ऋषभ राजपूत]


Constitution. Further, where the terms used in any article are ambiguous or capable of more than one
meaning, some assistance at interpretation may be taken from the objectives enshrined in the
Preamble. Despite this recognition of the significance of the Preamble, the Supreme Court
specifically opined that Preamble is not a part of the Constitution. Hence, statement 3 is not correct.
• The above opinion was reversed in the Keshavananda Bharati case in 1973; Supreme Court held that
Preamble is part of the constitution. This opinion was further clarified by the Supreme Court in the LIC
of India case (1995).

Q 95.D
• Recent Context: Votive stupas (offered in fulfillment of a vow) were discovered by the Archeological
Survey of India near Sarai Tila mound within the premises of Nalanda Mahavihara in Bihar.
• Stupa, Sanskrit for heap, is a mound-like (hemispherical) burial structure containing relics of Buddhist
monks. E.g. Sanchi Stupa built over Buddha relics.
• About Nalanda Mahavihara
o It is a UNESCO World Heritage Site with archaeological remains of a monastic and scholastic
institution from the 3rd century BCE to 13th century CE. Hence statement 3 is correct.
o Gautam Buddha as well as Mahavira stayed at Nalanda.
o Other famous scholars at Nalanda Nagarjuna, Dharampala, Dinnaga, Jinamitra Santaraksita etc.
o Imminent pilgrim monks like Hiuen Tsang and I-tsing also visited. Hence statement 1 is correct.
o It prospered under Gupta Dynasty, Harsha of Kannauj and Pala Dynasty. Hence statement 2 is
correct.
o It includes stupas, shrines, viharas (residential and educational buildings) and important art works in
stucco, stone and metal.

Q 96.D
• The Verma Committee on Fundamental Duties of the Citizens (1999) identified the existence of legal
provisions for the implementation of some of the Fundamental Duties. They are mentioned below:
o The Prevention of Insults to National Honour Act (1971) prevents disrespect to the Constitution
of India, the National Flag and the National Anthem.
o The various criminal laws in force provide punishments for encouraging enmity between different
sections of people on grounds of language, race, place of birth, religion and so on.
o The Protection of Civil Rights Act (1955) provides for punishments for offences related to caste
and religion.
o The Indian Penal Code (IPC) declares the imputations and assertions prejudicial to national
integration as punishable offences.
o The Unlawful Activities (Prevention) Act of 1967 provides for the declaration of a communal
organisation as an unlawful association.
o The Representation of People Act (1951) provides for the disqualification of members of the
Parliament or a state legislature for indulging in corrupt practice, that is, soliciting votes on the
ground of religion or promoting enmity between different sections of people on grounds of caste,
race, language, religion and so on.
o The Wildlife (Protection) Act of 1972 prohibits trade in rare and endangered species.
o The Forest (Conservation) Act of 1980 checks indiscriminate deforestation and diversion of forest
land for non-forest purposes.
• Hence, option (d) is the correct answer.

Q 97.A
• Article 324 of the Constitution has made the following provisions with regard to the composition of the
election commission:
o The Election Commission shall consist of the chief election commissioner and a number of other
election commissioners, if any, as the president may from time to time fix.
o The appointment of the chief election commissioner and other election commissioners shall be
made by the president. Hence statement 1 is correct.
o When any other election commissioner is so appointed, the chief election commissioner shall act as
the chairman of the election commission.
o The president may also appoint after consultation with the election commission such regional
commissioners as he may consider necessary to assist the election commission.
o The conditions of service and tenure of office of the election commissioners and the regional
commissioners shall be determined by the president.
37 www.visionias.in ©Vision IAS

FREE BY KING R QUEEN P [ऋषभ राजपूत]


• The chief election commissioner and the two other election commissioners have equal powers and
receive equal salaries, allowances, and other perquisites, which are similar to those of a judge of
the Supreme Court. Hence, statement 2 is correct.
• The chief election commissioner is provided with the security of tenure. He cannot be removed from his
office except in the same manner and on the same grounds as a judge of the Supreme Court. In other
words, he can be removed by the president on the basis of a resolution passed to that effect by both
Houses of Parliament with a special majority, either on the ground of proven misbehavior or incapacity.
• Thus, he does not hold his office till the pleasure of the president, though he is appointed by
him. Hence, statement 3 is not correct.

Q 98.D
• Centre has recently cleared Rs. 2600 crore incentive scheme to promote RUPAY, BHIM-UPI.
• The scheme aims to promote RuPay Debit Cards and low-value BHIM-UPI transactions (person-to-
merchant) in FY 2022-23.
• Features of scheme:
o Banks would be provided financial incentives for promoting Point of Sale (PoS) and e-commerce
transactions using RuPay and UPI.
o Promote UPI Lite and UPI 123PAY as economical and user-friendly digital payment solutions.
• UPI Lite is an 'on-device wallet' feature by the National Payments Corporation of India (NPCI) to
allow users to make small-value payments (upto ₹200) offline. Hence statement 1 is not correct.
• UPI 123PAY is an instant payment system for feature phone (non-smartphone) users.
• In addition, NPCI has allowed Non-Resident Indians (NRIs) with NRE (non-resident external) or
NRO (non-resident ordinary) accounts in 10 countries to use the UPI platform from their
international mobile numbers. Hence statement 2 is not correct.
• Member banks have to ensure such types of accounts are only allowed as per extant Foreign Exchange
Management Act regulations and RBI rules.
• 10 countries are Singapore, Australia, Canada, Hongkong, Oman, Qatar, USA, Saudi Arabia, UAE, and
United Kingdom. UPI works on the principle of SIM binding for security purposes and till now, it was
enabled only for Indian mobile numbers.

Q 99.D
• Directive Principles, although confer no legal rights and create no legal remedies, are significant and
useful in the following ways:
o They are like an ‘Instrument of Instructions’ or general recommendations addressed to all
authorities in the Indian Union. They remind them of the basic principles of the new social and
economic order, which the Constitution aims at building.
o They have served as useful beacon lights to the courts. They have helped the courts in exercising their
power of judicial review, that is, the power to determine the constitutional validity of a law.
o They form the dominating background to all State action, legislative or executive, and also a guide to
the courts in some respects.
o They amplify the Preamble, which solemnly resolves to secure to all citizens of India justice, liberty,
equality, and fraternity.
• The Directives also play the following roles:
o They facilitate stability and continuity in domestic and foreign policies in political, economic,
and social spheres in spite of the changes in the party in power.
o They are supplementary to the fundamental rights of the citizens. They are intended to fill in the
vacuum in Part III by providing for social and economic rights.
o Their implementation creates a favourable atmosphere for the full and proper enjoyment of the
fundamental rights by the citizens. Political democracy, without economic democracy, has no
meaning.
o They enable the opposition to exercise influence and control over the operations of the government.
The Opposition can blame the ruling party on the ground that its activities are opposed to the
Directives.
o They serve as a crucial test for the performance of the government. The people can examine the
policies and programmes of the government in light of these constitutional declarations.
o They serve as a common political manifesto. ‘A ruling party, irrespective of its political ideology,
has to recognize the fact that these principles are intended to be its guide, philosopher and friend in its
legislative and executive acts’
• Hence, option (d) is the correct answer.
38 www.visionias.in ©Vision IAS

FREE BY KING R QUEEN P [ऋषभ राजपूत]


Q 100.D
• The Directive Principles of State Policy are enumerated in Part IV of the Constitution from Articles 36 to
51. The framers of the Constitution borrowed this idea from the Irish Constitution of 1937.
• The provisions under Directive Principles derived their inspiration from many other sources such as
o ‘Instrument of Instructions’ enumerated in the Government of India Act of 1935.
o Gandhian ideology, was inspired from the program of reconstruction enunciated by Gandhi
during the national movement.
o Noble ideals inspired the national struggle for freedom.
• Hence, option (d) is the correct answer.

Copyright © by Vision IAS


All rights are reserved. No part of this document may be reproduced, stored in a retrieval system or
transmitted in any form or by any means, electronic, mechanical, photocopying, recording or otherwise,
without prior permission of Vision IAS.

39 www.visionias.in ©Vision IAS

FREE BY KING R QUEEN P [ऋषभ राजपूत]


VISIONIAS
www.visionias.in

Test Booklet Series

TEST BOOKLET

GENERAL STUDIES (P) 2024 – Test – 4127


C
Time Allowed: Two Hours Maximum Marks: 200

INSTRUCTIONS

1. IMMEDIATELY AFTER THE COMMENCEMENT OF THE EXAMINATION, YOU SHOULD CHECK THAT THIS BOOKLET
DOES NOT HAVE ANY UNPRINTED OR TURN OR MISSING PAGES OR ITEMS, ETC. IF SO, GET IT REPLACED BY A
COMPLETE TEST BOOKLET.

2. ENCODE CLEARLY THE TEST BOOKLET SERIES A, B, C OR D AS THE CASE MAY BE IN THE APPROPRIATE PLACE IN
THE ANSWER SHEET.

3. You have to enter your Roll Number on the Test Booklet in the Box
provided alongside. Do NOT write anything else on the Test Booklet.

4. This Test Booklet contains 100 items (Questions). Each item is printed in English. Each item comprises four
responses (answers). You will select the response which you want to mark on the Answer Sheet. In case you
feel that there is more than one correct response with you consider the best. In any case, choose ONLY ONE
response for each item.

5. You have to mark all your responses ONLY on the separate Answer Sheet provided. See direction in the
answers sheet.

6. All items carry equal marks. Attempt all items. Your total marks will depend only on the number of correct
responses marked by you in the answer sheet. For every incorrect response 1/3rdof the allotted marks will be
deducted.

7. Before you proceed to mark in the Answer sheet the response to various items in the Test booklet, you have to
fill in some particulars in the answer sheets as per instruction sent to you with your Admission Certificate.

8. After you have completed filling in all responses on the answer sheet and the examination has concluded, you
should hand over to Invigilator only the answer sheet. You are permitted to take away with you the Test
Booklet.

9. Sheet for rough work are appended in the Test Booklet at the end.

DO NOT OPEN THIS BOOKLET UNTIL YOU ARE ASKED TO DO SO


1 www.visionias.in ©Vision IAS

FREE BY KING R QUEEN P [ऋषभ राजपूत]


1. Consider the following statements with 4. With reference to the state council of
respect to the Supreme Court of India: ministers, consider the following statements:
1. The Supreme Court of India was 1. The Constitution does not specify the
inaugurated with the commencement of maximum strength of the state council of
the constitution. ministers.
2. After independence, Supreme Court
2. The advice tendered by the council of
replaced the British Privy Council as the
ministers to the Governor shall not be
highest court of appeal.
inquired into in any court.
Which of the statements given above is/are
Which of the statements given above is/are
correct?
correct?
(a) 1 only
(b) 2 only (a) 1 only
(c) Both 1 and 2 (b) 2 only
(d) Neither 1 nor 2 (c) Both 1 and 2
(d) Neither 1 nor 2
2. With reference to the role played by the
President of India in the Parliament, consider 5. Which of the following countries are
the following statements: bordering Lake Victoria?
1. He may nominate two members of the 1. Tanzania
Lok sabha having special knowledge in 2. Uganda
literature, science, art, and social service.
3. Rwanda
2. He lays the reports of the Comptroller
4. Kenya
and Auditor General before the
Select the correct answer using the code
Parliament.
given below.
Which of the statements given above is/are
(a) 1, 2 and 3 only
correct?
(a) 1 only (b) 2, 3 and 4 only
(b) 2 only (c) 1, 2 and 4 only
(c) Both 1 and 2 (d) 1, 3 and 4 only
(d) Neither 1 nor 2
6. In the context of the Indian Constitution,
3. Consider the following statements regarding which of the following contains provisions
International Health Regulations (IHR), related to local self-government in India?
2005: 1. Part IX
1. It is a framework to build the capability 2. Part III
to detect and report potential public 3. Article 324
health emergencies worldwide.
4. Part IV
2. It is legally binding on all WHO member
5. Article 280
states.
Select the correct answer using the code
Which of the statements given above is/are
given below.
correct?
(a) 1, 4 and 5 only
(a) 1 only
(b) 2 only (b) 2, 3 and 5 only
(c) Both 1 and 2 (c) 3, 4 and 5 only
(d) Neither 1 nor 2 (d) 1, 2, 3 and 4 only
2 www.visionias.in ©Vision IAS

FREE BY KING R QUEEN P [ऋषभ राजपूत]


7. Consider the following statement with 9. Which one of the following statements is/are
reference to None of the Above (NOTA): correct with reference to the speaker of the
1. The votes polled against the NOTA
state legislative assembly?
option are counted in total valid voters
1. The speaker once elected resigns from
polled by the contesting candidates for
the purpose of the return of security his party membership.
deposits to candidates. 2. He can be removed by a resolution
2. Even if the NOTA votes are more than
passed by a majority of all the then
the number of votes polled by any of the
members of the assembly.
candidates, the candidate who secures
the largest number of votes has to be 3. He resigns by writing to the Deputy
declared elected. Speaker.
Which of the statements given above is/are
Select the correct answer using the code
correct?
given below.
(a) 1 only
(b) 2 only (a) 1 and 2 only

(c) Both 1 and 2 (b) 2 and 3 only


(d) Neither 1 nor 2
(c) 3 only

(d) 1, 2 and 3
8. Which of the following have been explicitly
mentioned in the Constitution as a ground
for disqualification for being elected as a 10. Who among the following can enjoy the
member of parliament?
privileges of the state legislature?
1. Holds any office of profit under the
1. Governor
Union or state government.
2. Undischarged insolvent 2. Advocate-general of the state

3. Punished for preaching and practicing 3. State ministers


social crimes such as untouchability,
Select the correct answer using the code
dowry, and Sati.
given below.
Select the correct answer using the code
given below. (a) 1 and 2 only

(a) 1 and 3 only (b) 3 only


(b) 1 and 2 only
(c) 2 and 3 only
(c) 2 and 3 only
(d) 1, 2 and 3
(d) 1, 2 and 3
3 www.visionias.in ©Vision IAS

FREE BY KING R QUEEN P [ऋषभ राजपूत]


11. Consider the following statements with 13. Consider the following statements:
respect to the jurisdiction of the Supreme 1. The subordinate judiciary has a strictly
uniform organizational structure,
Court of India:
nomenclature, and jurisdiction
1. The supreme court has the discretion to throughout India.
grant special leave to appeal from a 2. The jurisdictions of Munsiff’s courts are
determined by High Courts.
judgment or order of any non-military
Which of the statements given above is/are
Indian court or tribunal.
correct?
2. The supreme court may refuse to tender (a) 1 only
its advice on any question of law or fact (b) 2 only
(c) Both 1 and 2
of public importance as referred by the
(d) Neither 1 nor 2
president.
3. Supreme court is not bound by its 14. Which of the following states are covered
under the Provisions of the Panchayats
previous decision and can depart from it
(Extension to the Scheduled Areas) Act?
in the interest of justice or community
1. Andhra Pradesh
welfare. 2. Maharashtra
Which of the statements given above are 3. Madhya Pradesh
4. Tamil Nadu
correct?
5. Karnataka
(a) 1 and 3 only Select the correct answer using the code
(b) 2 and 3 only given below.
(c) 1 and 2 only (a) 1, 2 and 4 only
(b) 3, 4 and 5 only
(d) 1, 2 and 3
(c) 1, 2 and 3 only
(d) 1, 2, 3 and 4 only
12. Consider the following statements regarding
15. Who among the following are appointed by
First Movers Coalition (FMC)
the President on the advise of the council of
1. It is a global initiative to decarbonize ministers?
seven industrial sectors. 1. Chairman and members of the Finance
2. India recently participated in the First commission
2. Chairman and members of the Election
Movers Coalition (FMC) Leadership
commission
Meeting of the World Economic Forum. 3. Attorney General of India and Solicitor
Which of the statements given above is/are general of India
Select the correct answer using the code
correct?
given below.
(a) 1 only
(a) 1 and 2 only
(b) 2 only (b) 2 and 3 only
(c) Both 1 and 2 (c) 1 and 3 only
(d) 1, 2 and 3
(d) Neither 1 nor 2
4 www.visionias.in ©Vision IAS

FREE BY KING R QUEEN P [ऋषभ राजपूत]


16. Consider the following statements with 18. Consider the following statements with
reference to the Leader of the Opposition reference to the composition of the Rajya
(LoP): Sabha:
1. The office of the LoP is a constitutional 1. All states are given equal representation
office.
in the Rajya Sabha.
2. The leader of the largest opposition
2. The Fourth Schedule of the Constitution
party having not less than one-tenth
deals with the allocation of seats in the
seats of the total strength of the House is
Rajya Sabha to the states and union
recognized as the LoP in that house.
territories.
3. The LoP is also entitled to the salary,
allowances, and other facilities Which of the statements given above is/are

equivalent to that of a cabinet minister. not correct?

Which of the statements given above is/are (a) 1 only

correct? (b) 2 only


(a) 2 and 3 only (c) Both 1 and 2
(b) 1 only (d) Neither 1 nor 2
(c) 3 only
(d) 1, 2 and 3
19. Who among the following can participate in

the election of the members of the legislative


17. Consider the following statements with
council of a state?
respect to the senior advocates in Supreme
1. Members of municipalities of the state
Court:
2. Graduates of three years standing and
1. These are advocates who are designated
as senior advocates by the supreme court residing within the state

of India or by any high court. 3. Teachers of three years standing in a

2. They cannot appear in the supreme court state not lower in standard than
without an advocate on record. secondary school
3. Only senior advocates are entitled to file 4. Members of the legislative assembly of
any matter or document before the the state
supreme court.
Select the correct answer using the code
Which of the statements given above are
given below.
correct?
(a) 1, 2 and 3 only
(a) 1 and 2 only
(b) 3 and 4 only
(b) 1, 2 and 3
(c) 1, 2, 3, and 4
(c) 1 and 3 only
(d) 2 and 3 only (d) 1 and 4 only
5 www.visionias.in ©Vision IAS

FREE BY KING R QUEEN P [ऋषभ राजपूत]


20. Consider the following statements regarding 22. With reference to the pardoning power of the
Metropolitan Planning Committee: President, consider the following statements:
1. The state legislature may make 1. Remission implies reducing the period
of sentence without changing its
provisions for the composition of
character.
Metropolitan Planning committees.
2. Commutation denotes the substitution of
2. Three-fourths of its members should be one form of punishment for a lighter
elected by the elected members of the form.
municipalities and chairpersons of the 3. Respite denotes awarding a lesser
panchayats in the metropolitan area from sentence in place of one originally
awarded due to some special reason.
amongst themselves.
Which of the statements given above are
3. The chairpersons of such committees
correct?
shall forward the development plan to (a) 1 and 2 only
the state government. (b) 2 and 3 only
Which of the statements given above is/are (c) 1 and 3 only
correct? (d) 1, 2 and 3
(a) 1 only
23. Which of the following are the constitutional
(b) 2 and 3 only
safeguards for the independence of the
(c) 1 and 3 only
Supreme Court?
(d) 1, 2 and 3 1. Power to punish for its contempt
2. Freedom to appoint its Staff
21. In which of the following cases the 3. Ban on practice after retirement
Governor can not make an ordinance 4. Its jurisdiction cannot be curtailed by the
Parliament
without the instructions from the President?
Select the correct answer using the code
1. If a bill containing the same provisions
given below.
would have required the sanction of the (a) 1 and 2 only
President for its introduction into the (b) 1 and 3 only
state legislature. (c) 2, 3 and 4 only
2. If he would have deemed it necessary to (d) 1, 2, 3 and 4
reserve a bill containing the same
24. Consider the following statements:
provisions for the consideration of the
1. Governor appoints the judges of the
President.
concerned state high court after
3. If an act of the state legislature consultation with the President.
containing the same provisions would 2. The appointment, posting and promotion
have been invalid without receiving the of district judges in a state are made by
President’s assent. the governor of the state in consultation
with the high court.
Select the correct answer using the code
Which of the statements given above is/are
given below.
correct?
(a) 1 and 2 only (a) 1 only
(b) 2 and 3 only (b) 2 only
(c) 1 and 3 only (c) Both 1 and 2
(d) 1, 2 and 3 (d) Neither 1 nor 2
6 www.visionias.in ©Vision IAS

FREE BY KING R QUEEN P [ऋषभ राजपूत]


25. With reference to the appointment of the 27. Which of the following are the compulsory
provisions under the 73rd amendment?
chief minister, consider the following
1. Three-tier system
statements: 2. Constitution of Gram Sabha
1. According to the Constitution of India, a 3. Reservation of seats for women
4. Granting financial powers to the
person must prove his majority in the panchayats
legislative assembly before he is 5. Representation of MPs in local bodies
Select the correct answer using the code
appointed as the Chief Minister.
given below.
2. A person who is not a member of the (a) 1, 2 and 3 only
state legislature can not be appointed as (b) 1, 2, 3 and 4 only
(c) 1, 3, 4 and 5 only
the Chief Minister. (d) 2, 4 and 5 only
Which of the statements given above is/are
28. With reference to Municipal Corporations,
correct?
consider the following statements:
(a) 1 only 1. The council of the municipal corporation
is a legislative wing of the corporation
(b) 2 only
consisting of both elected and nominated
(c) Both 1 and 2 members.
(d) Neither 1 nor 2 2. Mayor’s main function is to preside over
the meetings of the council.
3. The Municipal Commissioner is the
26. With reference to the consultative chief executive authority of the
corporation.
committees, consider the following
Which of the statements given above is/are
statements: correct?
1. These committees provide a forum for (a) 1 and 2 only
(b) 2 only
informal discussions between the (c) 1 and 3 only
ministers and the members of Parliament (d) 1, 2 and 3

on policies and programs of the


29. Consider the following statements with
government. regard to the Fast Track Courts in states
(FTCs):
2. These committees are constituted by the
1. FTCs are established by the governor in
Ministry of Parliamentary Affairs. consultation with respective high courts.
Which of the statements given above is/are 2. Judges of FTCs are appointed on an ad-
hoc basis.
correct?
Which of the statements given above is/are
(a) 1 only correct?
(a) 1 only
(b) 2 only
(b) 2 only
(c) Both 1 and 2 (c) Both 1 and 2
(d) Neither 1 nor 2 (d) Neither 1 nor 2
7 www.visionias.in ©Vision IAS

FREE BY KING R QUEEN P [ऋषभ राजपूत]


30. Without seeking the permission of the house, 33. With reference to the ordinance making
a member of a state legislature can be absent powers of the Governor, consider the
from all its meeting for a maximum period following statements:
of 1. It is co-extensive with the legislative
(a) Fifteen days
power of the state legislature.
(b) Thirty days
2. It is not a discretionary power.
(c) Forty days
3. An ordinance ceases to operate on the
(d) Sixty days
expiry of six months from the

31. Who among the following are included in reassembly of the state legislature.
the electoral college for the election of the Which of the statements given above are
President of India? correct?
1. The nominated members of both the (a) 1 and 2 only
Houses of Parliament (b) 2 and 3 only
2. The elected members of the legislative (c) 1 and 3 only
assemblies of the states
(d) 1, 2 and 3
3. The elected members of the legislative
assemblies of the Union Territories
34. On the recommendation of which of the
Select the correct answer using the code
following committees a permanent Inter-
given below.
(a) 1 and 2 only State Council was set up under Article 263?

(b) 2 and 3 only (a) First Administrative Reforms


(c) 1 and 3 only Commission (1966)
(d) 1, 2 and 3 (b) Rajamannar Committee (1969)
(c) Sarkaria Commission (1983)
32. Under which of the following special (d) National Commission to Review the
conditions, the Parliament can legislate on Working of the Constitution (2000)
subjects included in the State List?
1. In the national interest backed by a
35. Consider the following statements regarding
resolution of the Rajya Sabha
Choudhary Rehmat Ali:
2. When two or more states request the
1. He is credited with coining the “term”
parliament
3. To implement international treaties and Pakistan.

agreements. 2. The Muslim League’s Lahore session of


4. When a model code of conduct has been 1940 advocated his idea of creation of a
implemented due to state legislative 'Pakistan'.
elections Which of the statements given above is/are
Select the correct answer using the code correct?
given below. (a) 1 only
(a) 2 and 4 only
(b) 2 only
(b) 2, 3, and 4 only
(c) Both 1 and 2
(c) 1 and 3 only
(d) Neither 1 nor 2
(d) 1, 2, and 3 only
8 www.visionias.in ©Vision IAS

FREE BY KING R QUEEN P [ऋषभ राजपूत]


36. With reference to Gram Sabha, consider the 39. Which of the following countries has
following statements: recently genetically engineered a Chestnut
1. It is a village assembly consisting of all
tree to restore the forests in its region?
the eligible voters in the area of a
(a) United States
panchayat.
2. It may exercise powers and perform (b) Brazil
functions at the village level as the (c) China
legislature of a state determines. (d) India
Which of the statements given above is/are
correct?
40. Consider the following statements with
(a) 1 only
(b) 2 only respect to the writ jurisdiction of the High
(c) Both 1 and 2 Courts:
(d) Neither 1 nor 2 1. Unlike the supreme court, high courts
can issue writs for enforcement of even
37. Consider the following statements:
an ordinary legal right.
1. Transcriptome is a complete set of all
RNA molecules in a cell that carry 2. Writ jurisdiction is a part of the basic

genetic information from DNA. structure of the constitution.


2. RNA is a nucleic acid present in all Which of the statements given above is/are
living cells. correct?
Which of the statements given above is/are
(a) 1 only
correct?
(b) 2 only
(a) 1 only
(b) 2 only (c) Both 1 and 2
(c) Both 1 and 2 (d) Neither 1 nor 2
(d) Neither 1 nor 2

41. With reference to the pardoning power of the


38. Consider the following statements regarding
Governor given in Article 161, consider the
Paigah Tombs:
1. Paigah Tombs presents a beautiful blend following statements:
of Asaf Jah and Rajputani style of 1. He can pardon a death sentence.
architecture. 2. He can grant pardon to any person
2. Paigahs were the highest ranking nobles
convicted of any offense against any law
in the princely state of Hyderabad.
relating to a matter to which the
3. Built of red marbles, they captivate
visitors with intricately designed executive power of the State extends.
interiors. Which of the statements given above is/are
Which of the statements given above are correct?
correct?
(a) 1 only
(a) 1 and 2 only
(b) 2 only
(b) 2 and 3 only
(c) 1 and 3 only (c) Both 1 and 2
(d) 1, 2 and 3 (d) Neither 1 nor 2
9 www.visionias.in ©Vision IAS

FREE BY KING R QUEEN P [ऋषभ राजपूत]


42. Consider the following statements with 45. Consider the following statements regarding
respect to the judges of the Supreme Court the FireAId initiative:
of India: 1. It aims at using artificial intelligence to
1. A person who has been an advocate for
effectively manage wildfires.
over 10 years in any high court can be
2. It is launched by the World Economic
appointed as the judge of the Supreme
Court. Forum.
2. The Constitution has not fixed the tenure Which of the statements given above is/are
as well as the minimum age of a judge of correct?
the Supreme Court. (a) 1 only
Which of the statements given above is/are
(b) 2 only
correct?
(c) Both 1 and 2
(a) 1 only
(d) Neither 1 nor 2
(b) 2 only
(c) Both 1 and 2
(d) Neither 1 nor 2 46. Which of the following functions are
performed by the chief minister of a state?
43. Consider the following statements regarding 1. He advises the governor with regard to
the Hypersonic Technology Demonstrator
the appointment of the Solicitor General.
Vehicle (HSTDV):
2. He heads the Inter-State Council.
1. It is an unmanned demonstration aircraft
powered by ramjet engine. 3. He is the chairman of the State Planning
2. It uses atmospheric oxygen for Board.
propulsion. Select the correct answer using the code
Which of the statements given above is/are given below.
correct? (a) 1 and 2 only
(a) 1 only
(b) 1, 2 and 3
(b) 2 only
(c) 3 only
(c) Both 1 and 2
(d) Neither 1 nor 2 (d) None

44. Consider the following statements regarding 47. With reference to the evolution of Panchayat
the President of India: Raj in India, arrange the following
1. He is eligible for re-election for any
committees in chronological order:
number of terms.
1. Ashok Mehta Committee
2. He is entitled to such emoluments and
2. G. V. K Rao Committee
privileges as may be determined by the
constitution itself. 3. Thungon Committee
3. He is vested with qualified and pocket 4. L M Singhvi Committee
veto powers. Select the correct answer using the code
Which of the statements given above is/are given below.
correct?
(a) 2-1-3-4
(a) 1 only
(b) 4-3-2-1
(b) 1 and 3 only
(c) 2 and 3 only (c) 1-2-4-3
(d) 1, 2 and 3 (d) 3-2-1-4
10 www.visionias.in ©Vision IAS

FREE BY KING R QUEEN P [ऋषभ राजपूत]


48. Which of the statements given below is not 51. Consider the following statements regarding
correct regarding the state election Initiative on Critical and Emerging
commission? Technologies (iCET):
(a) It consists of a state election 1. The initiative has been launched by
commissioner to be appointed by the India and Russia to expand strategic
governor. partnership between the two nations.
(b) It prepares the electoral rolls for the 2. It is led by the Indian National Security
panchayat elections. Council Secretariat.
(c) The conditions of service of state Which of the statements given above is/are
election commissioner shall not be
correct?
varied to his disadvantage after his
(a) 1 only
appointment.
(b) 2 only
(d) It works under the overall supervision of
(c) Both 1 and 2
the Election Commission of India.
(d) Neither 1 nor 2

49. Consider the following statements regarding


52. According to the constitution of India, the
the duration of the panchayats:
expression ‘district judge’ includes which of
1. Panchayat at every level is provided
with a five-year term of office. the following judges?
2. A fresh election should be conducted 1. Additional district judge
every time a panchayat is prematurely 2. Joint district judge
dissolved. 3. Chief judge of a small cause court
3. A panchayat reconstituted after 4. Chief presidency magistrate
premature dissolution does not enjoy the 5. Sessions judge
full period of five years. Select the correct answer using the code
Which of the statements given above are given below.
correct? (a) 1, 2, 4 and 5 only
(a) 1 and 2 only (b) 3 and 4 only
(b) 2 and 3 only (c) 1, 2 and 5 only
(c) 1 and 3 only (d) 1, 2, 3, 4 and 5
(d) 1, 2 and 3
53. Consider the following statements with
50. Consider the following: reference to Gram Nyayalayas:
Assertion (A): The President of India is an 1. Gram Nyayalayas can try criminal cases
integral part of the Parliament. where the alleged offense attracts a
Reason (R): A bill passed by both Houses of maximum punishment of 3 years.
Parliament cannot become law without the
2. Gram Nyayalayas are strictly bound by
President’s assent.
the rules of evidence provided in the
In the context of the statements above, which
Indian Evidence Act, 1872.
of these is true?
Which of the statements given above is/are
(a) A and R both are true, and R is the
correct?
correct explanation for A
(a) 1 only
(b) A and R both are true, and R is the NOT
(b) 2 only
the correct explanation for A
(c) Both 1 and 2
(c) A is correct, R is incorrect
(d) A is incorrect, R is correct (d) Neither 1 nor 2
11 www.visionias.in ©Vision IAS

FREE BY KING R QUEEN P [ऋषभ राजपूत]


54. With reference to the Governor of a State, 57. Which of the following functions are
consider the following statements: provided to the panchayats under the
1. He is entitled to such emoluments, Eleventh Schedule of the Indian
allowances and privileges as may be
Constitution?
determined by the President.
1. Agriculture including agricultural
2. When he is the governor of two or more
extension
states, the emoluments and allowances
are shared by the states in such 2. Implementation of land reforms
proportion as determined by the 3. Fisheries
Parliament. 4. Small-scale industries
Which of the statements given above is/are 5. Technical training and vocational
correct? education
(a) 1 only Select the correct answer using the code
(b) 2 only
given below.
(c) Both 1 and 2
(a) 1, 3, and 4 only
(d) Neither 1 nor 2
(b) 2, 4 and 5 only
55. Consider the following statements: (c) 1, 2, 3 and 5 only
1. The Election Commission of India is (d) 1, 2, 3, 4 and 5
empowered to determine the manner in
which Lok Sabha seats are allotted to 58. Which of the following committees was
each state. appointed to give a report on state funding of
2. The 87th Amendment Act of 2003
elections?
provided for the delimitation of
(a) Tankha Committee
constituencies on the basis of the 1991
(b) Vohra Committee
census.
Which of the statements given above is/are (c) J. S. Verma Committee
correct? (d) Indrajit Gupta Committee
(a) 1 only
(b) 2 only 59. Consider the following statements regarding
(c) Both 1 and 2 Aditya L-1 mission:
(d) Neither 1 nor 2
1. It is the first observatory-class space-
based solar mission from India.
56. Consider the following statements:
2. The spacecraft will be placed in a halo
1. Every minister shall have the right to
speak and vote in any committee of the orbit around the first Lagrange point, L1,
state legislature. of the Sun-Earth system.
2. If any question arises whether a matter 3. It carries seven payloads to observe the
falls within the Governor’s discretion or photosphere, chromosphere, and the
not, decision of the Governor shall be corona.
final. Which of the statements given above are
Which of the statements given above is/are
correct?
correct?
(a) 1 and 2 only
(a) 1 only
(b) 2 only (b) 2 and 3 only
(c) Both 1 and 2 (c) 1 and 3 only
(d) Neither 1 nor 2 (d) 1, 2 and 3
12 www.visionias.in ©Vision IAS

FREE BY KING R QUEEN P [ऋषभ राजपूत]


60. Which of the following statements is not 63. Consider the following statements regarding
correct with respect to the Electoral Bond Ottanthullal:
Scheme? 1. It is an ancient martial art-form of
(a) The electoral bonds can be issued in the Kerala famous for its harmonious
denomination of ₹1,00,00,000.
synergy of art, science and medicine.
(b) The electoral bonds can be used for
2. It is believed to have been developed in
making donations to all registered
the forests by hunters observing the
political parties.
fighting techniques of animals.
(c) The information furnished by the doner
is treated confidential. Which of the statements given above is/are
(d) Foreign companies can use electoral correct?
bond for making donations to political (a) 1 only
parties. (b) 2 only
(c) Both 1 and 2
61. Consider the following statements regarding (d) Neither 1 nor 2
a protected area:
1. It is located in North Eastern part of
64. Consider the following statements regarding
Telangana.
notified area committees:
2. The reserve is nestled in the Sahyadri
1. They are statutory bodies created by an
Mountain Ranges along the banks of
Godavari. act of state legislature.
3. Geographically, the reserve is under the 2. They are an entirely nominated body.
Indo-Malayan region with tiger and 3. They are created for the administration
leopard as important species. of a fast-growing town due to
Which of the following sanctuaries is best industrialization.
described by the statements given above? Which of the statements given above is/are
(a) Amrabad Tiger Reserve correct?
(b) Kawal Tiger Reserve
(a) 1 only
(c) Tadoba Andhari Tiger Reserve
(b) 1 and 3 only
(d) Nagarjunsagar-Srisailam Tiger Reserve
(c) 3 only

62. Consider the following statements with (d) 2 and 3 only

regard to the Family Courts Act, of 1984:


1. It provides for the establishment of 65. Justice B. N. Srikrishna Committee was set
family courts by the State Governments up to.
in consultation with the High Courts. (a) review the scope of jurisdiction of the
2. No appeal shall lie to any court against Lokpal.
the award of the Family Court. (b) review the institutionalization of the
Which of the statements given above is/are
arbitration mechanism.
correct?
(c) review the scope of jurisdiction of the
(a) 1 only
Gram Nyayalayas.
(b) 2 only
(d) review the working of National Legal
(c) Both 1 and 2
(d) Neither 1 nor 2 Services Authority (NALSA).
13 www.visionias.in ©Vision IAS

FREE BY KING R QUEEN P [ऋषभ राजपूत]


66. This protected area is located in the Nicobar 69. Consider the following statements regarding
qualifications for a person to be chosen as a
Islands. It is India's nesting site for giant
member of the Parliament:
leatherback turtles. Recently, the entire 1. Candidates contesting an election to the
Wildlife Sanctuary was denotified to make Rajya Sabha from a particular state
should be an elector in that particular
way for International Container
state.
Transhipment Port. 2. A person of scheduled castes or
Which of the following protected areas is scheduled tribes can contest from a seat
not reserved for them.
being described in the passage given above? Which of the statements given above is/are
(a) Galathea Bay Wildlife Sanctuary correct?
(a) 1 only
(b) Barren Island Wildlife Sanctuary
(b) 2 only
(c) Landfall Island Wildlife Sanctuary (c) Both 1 and 2
(d) Cuthbert Bay Wildlife Sanctuary (d) Neither 1 nor 2

70. Which of the following are not a part of the


67. Who administers the oath of office and original jurisdiction of the supreme court?
1. Interstate water disputes
secrecy to the Council of Ministers?
2. Any dispute between two or more states
(a) President 3. A dispute arising out of any pre-
(b) Prime Minister constitution treaty
4. Recovery of damages by a state against
(c) Chief Justice of India
the centre
(d) Vice-President Select the correct answer using the code
given below.
(a) 1, 2 and 3 only
68. Which of the following are elected by a (b) 2 and 4 only
system of proportional representation by a (c) 1, 3 and 4 only
(d) 1, 2, 3 and 4
means of the single transferable vote?

1. Members of Lok Sabha 71. Consider the following statements regarding


2. Members of Rajya Sabha BharOS:
1. It is an indigenous mobile operating
3. Members of the state legislative council
system developed by IIT Madras.
4. Vice-President of India 2. BharOS is based on the Android Open
Source Project and includes all Google
Select the correct answer using the code
services.
given below. Which of the statements given above is/are
(a) 1 and 2 only correct?
(a) 1 only
(b) 1 and 3 only
(b) 2 only
(c) 2, 3 and 4 only (c) Both 1 and 2
(d) 1, 2, 3 and 4 (d) Neither 1 nor 2
14 www.visionias.in ©Vision IAS

FREE BY KING R QUEEN P [ऋषभ राजपूत]


72. Which of the following bills lapses on the 75. With reference to the Prime Minister,
dissolution of the Lok Sabha? consider the following statements:
1. A bill passed by the Lok Sabha but 1. His term of office is not fixed and he
pending in the Rajya Sabha holds office during the pleasure of the
2. A bill passed by both houses but pending
President.
the assent of the President
2. The salary and allowances are
3. A bill pending in the Rajya Sabha but
determined by the President.
not passed by the Lok Sabha
Select the correct answer using the code 3. The death of an incumbent Prime

given below. Minister automatically dissolves the


(a) 1 only council of ministers.
(b) 2 and 3 only Which of the statements given above are
(c) 3 only correct?
(d) 1 and 3 only (a) 1 and 2 only
(b) 2 and 3 only
73. Consider the following statements regarding
(c) 1 and 3 only
Basmati Rice:
(d) 1, 2 and 3
1. It is cultivated mostly in the Indian
subcontinent.
2. Recently the Food Safety and Standards 76. In the context of virology, which of the

Authority of India (FSSAI) has notified following best describes 'reassortment'?


standards for basmati rice. (a) It is the formation of a hybrid virus
Which of the statements given above is/are containing parts from the genomes of
correct? two distinct viruses.
(a) 1 only (b) It is the process in which a virus infects
(b) 2 only
and replicates within bacteria.
(c) Both 1 and 2
(c) It is the process in viruses by which an
(d) Neither 1 nor 2
enzyme makes a copy of DNA from
RNA.
74. Which of the following statements is correct
regarding the cantonment board? (d) It is the process of mutating the genetic

(a) It works under the administrative control code of a mammal in order to create a
of the state government. new species.
(b) The executive officer of the cantonment
board is appointed by the governor of 77. Who administers the oath of office to the
the concerned state. governor of a state?
(c) The President of the board is directly
(a) President
elected by the civilian population of the
(b) Chief Justice of India
cantonment area.
(c) Chief justice of the concerned state high
(d) The military officer commanding the
court
station presides over the meetings of the
cantonment board. (d) Vice-President
15 www.visionias.in ©Vision IAS

FREE BY KING R QUEEN P [ऋषभ राजपूत]


78. When does the Model Code of Conduct for 81. Consider the following statements regarding
guidance of candidates and political parties the recently established Grievance Appellate
comes into effect? Committees (GACs) under the IT Rules
(a) Immediately after the announcement of 2021:
the schedule of elections 1. It has been set up by the government to
(b) Immediately after the last date for filing look into users' complaints against large
nominations social media companies.
(c) Immediately after the last date for 2. It is a digital platform that will operate
withdrawal of nomination only online.
(d) 48 hours before the actual election starts Which of the statements given above is/are
correct?
79. Consider the following statements with (a) 1 only
regard to the Permanent Lok Adalats: (b) 2 only
1. Permanent Lok Adalats are established (c) Both 1 and 2
under the Legal Service Authorities Act (d) Neither 1 nor 2
of 1987.
2. The award of the Permanent Lok Adalat 82. Which of the following statements is correct
is final and binding upon the parties. regarding China+1 strategy, recently seen in
Which of the statements given above is/are the news?
correct? (a) It is a USA led strategic alliance to
(a) 1 only counter Chinese aggression in the Indo-
(b) 2 only Pacific region.
(c) Both 1 and 2 (b) It is strategic decision being taken by
(d) Neither 1 nor 2 MNCs to shift their supply chains
outside China.
80. Consider the following statements with (c) It a defence program that involves
reference to parliamentary proceedings in inclusion of one more member country
India: in the Quad alliance.
1. The Attorney General of India has the (d) It is a space cooperation program of
right to speak and take part in the China and Russia to set up a new
proceedings of either house or any joint International Space Station.
sitting of both houses.
2. A minister, who is not a member of 83. Mr. XYZ wants to become a district judge.
either house, cannot participate in the What is the minimum age prescribed in the
proceedings of any house. Constitution of India so that he becomes a
Which of the statements given above is/are District judge?
correct? (a) 21 years
(a) 1 only (b) 25 years
(b) 2 only (c) 35 years
(c) Both 1 and 2 (d) No Minimum age is prescribed in the
(d) Neither 1 nor 2 Constitution
16 www.visionias.in ©Vision IAS

FREE BY KING R QUEEN P [ऋषभ राजपूत]


84. Consider the following statements regarding 87. Consider the following statements:
Immune Imprinting: 1. The facility of the proxy vote is provided
1. It is the body's tendency to repeat its to the service voters belonging to the
immune response through previously Armed Forces.
encountered variant.
2. The facility of the postal ballot is
2. T cells lymphocytes play a central role
provided to certain classes of people
in this process.
notified by the Election Commission.
Which of the statements given above is/are
correct? Which of the statements given above is/are
(a) 1 only correct?
(b) 2 only (a) 1 only
(c) Both 1 and 2 (b) 2 only
(d) Neither 1 nor 2 (c) Both 1 and 2
(d) Neither 1 nor 2
85. With reference to the money bill in the state
legislature, consider the following
88. With reference to the Vice President,
statements:
consider the following statements:
1. When the governor reserves a money
1. The electoral college for his election
bill for the consideration of the
President, the governor's assent is no does not include the members of the
longer required. state legislative assemblies.
2. The governor cannot return a money bill 2. A formal impeachment is not required
for the reconsideration of the state for his removal.
legislature. Which of the statements given above is/are
Which of the statements given above is/are correct?
correct? (a) 1 only
(a) 1 only
(b) 2 only
(b) 2 only
(c) Both 1 and 2
(c) Both 1 and 2
(d) Neither 1 nor 2
(d) Neither 1 nor 2

86. Consider the following statements regarding 89. Consider the following statements with
zonal councils: reference to the two houses of the
1. These are advisory bodies that were Parliament:
created under the provisions of Article 1. The Constitution has fixed the term of
263 of the Indian Constitution. office of members of the Rajya Sabha.
2. The Union Home Minister acts as the
2. Only the speaker of the Lok Sabha in
chairman of all the zonal councils.
consultation with the central government
3. There are four zonal councils in total
can dissolve the Lok Sabha.
that covers all 28 states of India.
Which of the statements given above is/are
Which of the statements given above is/are
correct? correct?
(a) 1 and 2 only (a) 1 only
(b) 2 only (b) 2 only
(c) 3 only (c) Both 1 and 2
(d) 1, 2 and 3 (d) Neither 1 nor 2
17 www.visionias.in ©Vision IAS

FREE BY KING R QUEEN P [ऋषभ राजपूत]


90. Arrange the following officials from higher 93. Both houses of the parliament have separate
to lower rank as per Table of Precedence.
committees of this kind. This committee
1. Prime Minister
2. Former Presidents regulates the program and timetable of the
3. Governors of states within their House. It allocates time for the transaction of
respective states
legislative and other exercises brought
4. Cabinet Ministers of the Union
Select the correct answer using the code before the House by the government. The
given below. Lok Sabha committee consists of 15
(a) 2-1-4-3
(b) 1-2-3-4 members including the Speaker as its
(c) 1-3-2-4 chairman. In the Rajya Sabha, it has 11
(d) 3-1-4-2
members including the Chairman as its ex-

91. With reference to the Council of Ministers, officio chairman.


consider the following statements: Which of the following parliamentary
1. They are collectively responsible to the
committees has been described in the
Lok Sabha and the Rajya sabha.
2. There is no legal responsibility of a passage given above?
minister as per the constitution.
(a) Business Advisory Committee
3. It is the duty of every minister to stand
by cabinet decisions and support them (b) Rules Committee
both within and outside the Parliament. (c) General Purposes Committee
Which of the statements given above is/are
(d) Ethics Committee
correct?
(a) 3 only
(b) 2 and 3 only 94. With reference to the Council of Ministers,
(c) 1, 2 and 3
(d) 1 and 3 only consider the following statements:

1. A person who is not a member of either


92. Consider the following statements about the
House of Parliament can not be
Deputy Speaker of Lok Sabha:
1. Whenever he is appointed as a member appointed as a minister.
of a parliamentary committee, he 2. President can appoint only those persons
automatically becomes its chairman.
as ministers who are recommended by
2. The date of his election is fixed by the
President. the prime minister.
3. He is subordinate to the speaker of Lok
Which of the statements given above is/are
Sabha.
Which of the statements given above is/are correct?
correct? (a) 1 only
(a) 2 only
(b) 2 only
(b) 1 only
(c) 2 and 3 only (c) Both 1 and 2
(d) 1 and 3 only (d) Neither 1 nor 2
18 www.visionias.in ©Vision IAS

FREE BY KING R QUEEN P [ऋषभ राजपूत]


95. Consider the following statements: 97. In the context of Indian polity, consider the
following statements:
1. Anything that works as a superconductor
1. The President is the chief spokesman of
above 70 Kelvin (-203°C) is generally the Union government.
considered as a High-temperature 2. The President announces government
policies on the floor of the House.
superconductor (HTS). 3. The Prime Minister can be a member of
2. Solid mercury behaves as a any of the two Houses of parliament.
Which of the statements given above are not
superconductor below its threshold
correct?
temperature i.e. (around –270°C). (a) 1 and 2 only
(b) 2 and 3 only
Which of the statements given above is/are
(c) 1 and 3 only
correct? (d) 1, 2 and 3
(a) 1 only
98. Which of the following acts of parliament
(b) 2 only
provides for the preparation and revision of
(c) Both 1 and 2 electoral rolls for the purpose of elections to
the House of People?
(d) Neither 1 nor 2
(a) Representation of the People Act, 1950
(b) Representation of the People Act, 1951
(c) Parliament (Prevention of
96. Which of the following statements is/are
Disqualification) Act, 1959
correct regarding the appointment of the (d) Presidential and Vice-Presidential
Governor as provided in the Constitution of elections Act, 1952

India?
99. Consider the following statements regarding
1. He should be a non-resident of the state Mutual Legal Assistance Treaty (MLATs) in
criminal matters:
where he is appointed.
1. They are the bilateral treaties between
2. The president is required to consult the the countries that allow for exchange of
chief minister of the state concerned but evidence.
2. The Ministry of External Affairs is the
the advise is not binding on him. nodal Ministry for providing mutual
3. He should have completed 35 years of legal assistance in criminal law matters.
3. Criminal Procedure Code (CrPC) does
age.
not mention arrangements with the
Select the correct answer using the code Foreign Governments regarding service
of judicial processes in this regard.
given below.
Which of the statements given above is/are
(a) 1 and 2 only correct?
(b) 2 only (a) 1 and 2 only
(b) 2 and 3 only
(c) 3 only (c) 1 only
(d) 1 and 3 only (d) 1, 2 and 3
19 www.visionias.in ©Vision IAS

FREE BY KING R QUEEN P [ऋषभ राजपूत]


100. Consider the following statements regarding
World Economic Forum (WEF):
1. It is a non-governmental lobbying
organisation largely funded by global
enterprises.
2. It has been used for pivotal international
diplomacy and even avoiding wars.
Which of the statements given above is/are
correct?
(a) 1 only
(b) 2 only
(c) Both 1 and 2
(d) Neither 1 nor 2

Copyright © by Vision IAS


All rights are reserved. No part of this document may be reproduced, stored in a retrieval system or
transmitted in any form or by any means, electronic, mechanical, photocopying, recording or otherwise,
without prior permission of Vision IAS.
20 www.visionias.in ©Vision IAS

FREE BY KING R QUEEN P [ऋषभ राजपूत]


VISIONIAS
www.visionias.in
ANSWERS & EXPLANATIONS
GENERAL STUDIES (P) TEST – 4127 (2024)

Q 1.B
• Unlike the American Constitution, the Indian Constitution has established an integrated judicial system
with the Supreme Court at the top and the high courts below it. Under a high court (and below the state
level), there is a hierarchy of subordinate courts: district courts and other lower courts.
• This single system of courts, adopted from the Government of India Act of 1935, enforces both
central and state laws.
• In the USA, on the other hand, federal laws are enforced by the federal judiciary and state laws are
enforced by the state judiciary. There is thus a double system of courts in the USA–one for the centre and
the other for the states.
• To sum up, although a federal country like the USA, India has a unified judiciary and one system of
fundamental law and justice.
• The Supreme Court of India was inaugurated on January 28, 1950, and the date of the
commencement of the Indian Constitution is 26 January 1950. Hence, statement 1 is not correct.
• It succeeded the Federal Court of India, established under the Government of India Act of 1935.
However, the jurisdiction of the Supreme Court is greater than that of its predecessor. This is
because the Supreme Court has replaced the British Privy Council as the highest court of appeal.
Hence, statement 2 is correct.
• Articles 124 to 147 in Part V of the Constitution deal with the organisation, independence, jurisdiction,
powers, procedures and so on of the Supreme Court. The Parliament is also authorised to regulate them.

Q 2.B
• The President is an integral part of the Parliament of India, and enjoys the following legislative powers.
o He nominates 12 members of the Rajya Sabha from amongst persons having special knowledge
or practical experience in literature, science, art and social service. Hence, statement 1 is not
correct.
o Earlier, he could nominate two members to the Lok Sabha from the Anglo- Indian Community. But,
the Constitutional (104th Amendment) Act, 2019 effectively abolished the practice of the
President of India (Governor in the states) nominating two members of the Anglo-Indian
Community (one in the state) on the recommendation of the Prime Minister of India (CM in
states).
o He lays the reports of the Comptroller and Auditor General, Union Public Service Commission,
Finance Commission, and others, before the Parliament. Hence, statement 2 is correct.
o He can summon or prorogue the Parliament and dissolve the Lok Sabha. He can also summon a joint
sitting of both the Houses of Parliament, which is presided over by the Speaker of the Lok Sabha.
o He can address the Parliament at the commencement of the first session after each general election
and the first session of each year
o He can send messages to the Houses of Parliament, whether with respect to a bill pending in the
Parliament or otherwise.
Q 3.C
• While disease outbreaks and other acute public health risks are often unpredictable and require a range of
responses, the International Health Regulations (2005) (IHR) provide an overarching legal framework that
defines countries’ rights and obligations in handling public health events and emergencies that have the
potential to cross borders.
• The IHR are an instrument of international law that is legally binding on 196 countries, including
the 194 WHO Member States. Hence statement 2 is correct.
1 www.visionias.in ©Vision IAS

FREE BY KING R QUEEN P [ऋषभ राजपूत]


• The International Health Regulations (IHR) 2005 is an agreement of countries to build the
capability to detect and report potential public health emergencies worldwide. IHR require that all
countries have the ability to detect, assess, report, and respond to public health events. Hence statement 1
is correct.
• They create rights and obligations for countries, including the requirement to report public health events.
The Regulations also outline the criteria to determine whether or not a particular event constitutes a
“public health emergency of international concern”.

Q 4.B
• The Constitution does not specify the size of the state council of ministers or the ranking of
ministers. They are determined by the chief minister according to the exigencies of the time and
requirements of the situation.
• However, the 91st Amendment to the Constitution (2003) inserted a new article to Article 164 i.e.
Article 164(1A) which states that the total number of Ministers, including the Chief Minister, in the
Council of Ministers in a State shall not exceed fifteen percent of the total number of members of the
Legislative Assembly of that State; provided that the number of Ministers, including Chief Minister
in a State, shall not be less than twelve. Hence, statement 1 is not correct.
• Like at the Centre, in the states too, the council of ministers consists of three categories of ministers,
namely, cabinet ministers, ministers of state, and deputy ministers. The difference between them lies
in their respective ranks, emoluments, and political importance. At the top of all these ministers stands the
chief minister–the supreme governing authority in the state.
• The cabinet ministers head the important departments of the state government like home, education,
finance, agriculture, and so forth. They are members of the cabinet, attend its meetings and play an
important role in deciding policies. Thus, their responsibilities extend over the entire gamut of state
government.
• The ministers of state can either be given independent charge of departments or can be attached to
cabinet ministers. However, they are not members of the cabinet and do not attend the cabinet meetings
unless specially invited when something related to their departments is considered by the cabinet.
• Next in rank are the deputy ministers. They are not given independent charge of departments. They are
attached to the cabinet ministers and assist them in their administrative, political and parliamentary duties.
They are not members of the cabinet and do not attend cabinet meetings.
• At times, the council of ministers may also include a deputy chief minister. The deputy chief
ministers are appointed mostly for local political reasons.
• The advice tendered by Ministers to the President shall not be inquired into in any court. Hence,
statement 2 is correct.

Q 5.C
• Recently, the Centre for Science and Environment and National Environment Management Council
(NEMC), Tanzania have jointly released a report on managing the water quality of Lake Victoria.
• Lake Victoria, also called Victoria Nyanza, largest lake in Africa and the chief reservoir of the Nile. It is
the second-largest freshwater lake in the world after lake Superior. Lake Victoria has been suffering from
a variety of unsustainable human activities over the last five decades.
• Lake Victoria and its flora and fauna support the livelihoods of about 45 million people. The ecologically
unique water body is shared by three countries — Tanzania (51 per cent), Uganda (44 per cent) and
Kenya (5 per cent). Hence option (c) is the correct answer.

Q 6.A
• The term Panchayati Raj in India signifies the system of rural local self-government. It has been
established in all the states of India by the Acts of the state legislatures to build democracy at the grass
root level. It is entrusted with rural development. It was constitutionalized through the 73rd Constitutional
Amendment Act of 1992. This act has added a new Part-IX to the Constitution of India. This part is
entitled ‘The Panchayats’ and consists of provisions from Articles 243 to 243 O. In addition, the act
has also added a new Eleventh Schedule to the Constitution. Hence option 1 is correct.
• Article 40 of the constitution under Directive Principles of State Policy (part IV) states that the
State shall take steps to organize village panchayats and endow them with such powers and
authority as may be necessary to enable them to function as units of self-government. Hence option
4 is correct.
• Under Article 280, the Finance Commission is entrusted with the duty to give recommendations on
the measures needed to augment the consolidated fund of a state to supplement the resources of the
2 www.visionias.in ©Vision IAS

FREE BY KING R QUEEN P [ऋषभ राजपूत]


panchayats and the municipalities in the state on the basis of the recommendations made by the
state finance commission. Hence option 5 is correct.
• Under Article 324, the Election Commission of India has the responsibility of the superintendence,
direction and control of the preparation of the electoral rolls for, and the conduct of, all elections to
Parliament and to the Legislature of every State and of elections to the offices of President and Vice
President. For elections of the Panchayati Raj institutions, a separate State Election Commission has
to be set up. Hence option 3 is not correct.
• The Fundamental Rights are enshrined in Part III of the Constitution (Articles 12-35). They have no
express provisions regarding the Panchayati Raj institutions. Hence, option 2 is not correct.

Q 7.B
• According to the directions of the Supreme Court, the Election Commission made provision in the ballot
papers/EVMs for the None of the Above (NOTA) option so that the voters who come to the polling booth
and decide not to vote for any of the candidates in the fray, are able to exercise their right not to vote for
such candidates while maintaining the secrecy of their ballot. The provision for NOTA has been made
since the General Election to State Legislative Assemblies of Chhattisgarh, Madhya Pradesh, Mizoram,
NCT of Delhi and Rajasthan in 2013 and continued in the General Election to State Legislative
Assemblies of Andhra Pradesh, Arunachal Pradesh, Odisha and Sikkim in 2014 along with the General
Elections to the Sixteenth Lok Sabha (2014).
• The votes polled against the NOTA option are not taken into account for calculating the total valid
voters polled by the contesting candidates for the purpose of the return of security deposits to
candidates. Even if the number of electors opting for NOTA options is more than the number of
votes polled by any of the candidates, the candidate who secures the largest number of votes has to
be declared elected. Hence, statement 1 is not correct and statement 2 is correct.
• In 2001, the ECI sent a proposal to the Government to amend the law so as to provide for a neutral vote
provision for the electors who did not wish to vote for any of the candidates. In 2004, PUCL (People’s
Union for Civil Liberties) filed a petition seeking a direction to provide the necessary provision in ballot
papers and EVMs for the protection of the right to not vote for any candidate, secretly. The Supreme
Court in 2013 held that the ECI may provide for the None of the Above (NOTA) option on EVMs and
ballot papers.

Q 8.B
• Under the Constitution, a person shall be disqualified for being elected as a member of Parliament:
o if he holds any office of profit under the Union or state government (except that of a minister or any
other office exempted by Parliament). Hence option 1 is correct.
o if he is of unsound mind and stands so declared by a court.
o if he is an undischarged insolvent. Hence option 2 is correct.
o if he is not a citizen of India or has voluntarily acquired the citizenship of a foreign state or is under
any acknowledgement of allegiance to a foreign state; and
o if he is so disqualified under any law made by Parliament.
• The Parliament has laid down the following additional disqualifications in the Representation of
People Act (1951):
o He must not have been found guilty of certain election offences or corrupt practices in the elections.
o He must not have been convicted for any offence resulting in imprisonment for two or more years.
But, the detention of a person under a preventive detention law is not a disqualification.
o He must not have failed to lodge an account of his election expenses within the time.
o He must not have any interest in government contracts, works or services.
o He must not be a director or managing agent nor hold an office of profit in a corporation in which the
government has at least 25 per cent share.
o He must not have been dismissed from government service for corruption or disloyalty to the State.
o He must not have been convicted for promoting enmity between different groups or for the offence of
bribery.
o He must not have been punished for preaching and practising social crimes such as
untouchability, dowry and Sati. Hence, option 3 is not correct.

Q 9.B
• Speaker of the Assembly
o The Assembly elects the Speaker from amongst its members. Usually, the Speaker remains in office
during the life of the Assembly. However, he vacates his office earlier in the following cases:
3 www.visionias.in ©Vision IAS

FREE BY KING R QUEEN P [ऋषभ राजपूत]


▪ If he ceases to be a member of the Assembly.
▪ If he resigns by writing to the Deputy Speaker. Hence statement 3 is correct.
o If he is removed by a resolution passed by a majority of all the then members of the assembly.
Such a resolution can only be moved after giving 14 days advance notice. Hence statement 2 is
correct.
o The powers of the Speaker of the Legislative Assembly are similar to those of the Lok Sabha.
o The speaker after the election does not resign from party membership but he refrains from
active politics. Hence statement 1 is not correct.
o Powers and Function:
▪ The important function of the speakers is to preside over the sessions of the Legislative assembly
and to maintain order and discipline within the house.
▪ The Speaker does not take part in the debate and usually does not vote except to break a tie.
▪ When the assembly meets the speaker calls House to order to maintain discipline in the house.
▪ He may adjourn the session or sitting of the House.
▪ He may suspend or expel members of the House for unruly behavior.
▪ Within the House, the Speaker is the master who decides whether a bill is a money bill or not. The
Speaker’s decision cannot be challenged in the Court of Law. The Speaker of the Lok Sabha
presides over the joint sessions of the Parliament. The constitution does not provide for joint
sessions of the State Legislatures even where the State Legislatures are bicameral.
• Deputy Speaker of Assembly
o The Deputy Speaker is also directly elected by the Assembly from amongst its members. He is elected
after the election of the Speaker has taken place. Usually, he remains in office during the
o life of the Assembly. However, he vacates his office earlier in the following cases:
o If he ceases to be a member of the Assembly.
o If he resigns by writing to the Speaker.
o If he is removed by a resolution passed by a majority of all the then members of the assembly. Such a
resolution can only be moved after giving 14 days advance notice.

Q 10.C
• Privileges of the state legislature (Article:194)
o Privileges of a state legislature are a sum of special rights, immunities, and exemptions enjoyed by
the Houses of the state legislature, their committees, and their members. They are necessary in
order to secure the independence and effectiveness of their actions. Without these privileges, the
Houses can neither maintain their authority, dignity, and honor nor can protect their members from
any obstruction in the discharge of their legislative responsibilities.
o The Constitution has also extended the privileges of the state legislature to those persons who are
entitled to speak and take part in the proceedings of a House of the state legislature or any of its
committees. These include the advocate-general of the state and state ministers.
o It must be clarified here that the privileges of the state legislature do not extend to the
governor who is also an integral part of the state legislature. Hence option (c) is the correct answer.
o The privileges of a state legislature can be classified into two broad categories–those that are enjoyed
by each House of the state legislature collectively, and those that are enjoyed by the members
individually.
o Collective Privileges: The privileges belonging to each House of the state legislature collectively are:
▪ It has the right to publish its reports, debates, and proceedings and also the right to prohibit others
from publishing the same.
▪ It can exclude strangers from its proceedings and hold secret sittings to discuss some important
matters.
▪ It can make rules to regulate its own procedure and the conduct of its business and to adjudicate
upon such matters.
▪ It can punish members as well as outsiders for breach of its privileges or its contempt by
reprimand, admonition, or imprisonment (also suspension or expulsion, in the case of members).
▪ It has the right to receive immediate information on the arrest, detention, conviction,
imprisonment, and release of a member.
▪ It can institute inquiries and order the attendance of witnesses and send relevant papers and
records.
▪ The courts are prohibited to inquire into the proceedings of a House or its Committees.
▪ No person (either a member or outsider) can be arrested, and no legal process (civil or criminal)
can be served within the precincts of the House without the permission of the presiding officer.
4 www.visionias.in ©Vision IAS

FREE BY KING R QUEEN P [ऋषभ राजपूत]


o Individual Privileges: The privileges belonging to the members individually are:
▪ They cannot be arrested during the session of the state legislature and 40 days before the
beginning and 40 days after the end of such session. This privilege is available only in civil cases
and not in criminal cases or preventive detention cases.
▪ They have freedom of speech in the state legislature. No member is liable to any proceedings in
any court for anything said or any vote given by him in the state legislature or its committees.
This freedom is subject to the provisions of the Constitution and to the rules and standing orders
regulating the procedure of the state legislature.
▪ They are exempted from jury service. They can refuse to give evidence and appear as a witness in
a case pending in a court when the state legislature is in session.
Q 11.D
• JURISDICTION AND POWERS OF THE SUPREME COURT
• The Constitution has conferred very extensive jurisdiction and vast powers on the Supreme Court.
• It is not only a Federal Court like the American Supreme Court but also a final court of appeal like the
British House of Lords (the Upper House of the British Parliament).
• It is also the final interpreter and guardian of the Constitution and guarantor of the fundamental rights of
the citizens. Further, it has advisory and supervisory powers.
• The jurisdiction and powers of the Supreme Court can be classified into the following:
• Appellate Jurisdiction
o As mentioned earlier, the Supreme Court has not only succeeded the Federal Court of India but also
replaced the British Privy Council as the highest court of appeal.
o The Supreme Court is primarily a court of appeal and hears appeals against the judgements of the
lower courts. It enjoys a wide appellate jurisdiction which can be classified under four heads:
▪ Appeals in constitutional matters.
▪ Appeals in civil matters.
▪ Appeals in criminal matters.
✓ The Supreme Court hears appeals against the judgement in a criminal proceeding of a high
court if the high court
❖ has on appeal reversed an order of acquittal of an accused person and sentenced him to
death; or
❖ has taken before itself any case from any subordinate court and convicted the accused
person and sentenced him to death; or
❖ certifies that the case is a fit for appeal to the Supreme Court.
✓ In the first two cases, an appeal lies to the Supreme Court as a matter of right (ie,
without any certificate of the high court).
✓ But if the high court has reversed the order of conviction and has ordered the acquittal of the
accused, there is no right to appeal to the Supreme Court.
▪ Appeals by special leave.
✓ The Supreme Court is authorised to grant in its discretion special leave to appeal from
any judgement in any matter passed by any court or tribunal in the country (except
military tribunal and court-martial). Hence, statement 1 is correct.
✓ This provision contains four aspects as under:
❖ It is a discretionary power and hence, cannot be claimed as a matter of right.
❖ It can be granted in any judgement whether final or interlocutory.
❖ It may be related to any matter–constitutional, civil, criminal, income-tax, labour,
revenue, advocates, etc.
❖ It can be granted against any court or tribunal and not necessarily against a high court (of
course, except a military court).
✓ Thus, the scope of this provision is very wide and it vests the Supreme Court with a plenary
jurisdiction to hear appeals.
✓ On the exercise of this power, the Supreme Court itself held that ‘being an exceptional and
overriding power, it has to be exercised sparingly and with caution and only in special
extraordinary situations. Beyond that, it is not possible to fetter the exercise of this power by
any set formula or rule.
• Advisory Jurisdiction
o The Constitution (Article 143) authorises the president to seek the opinion of the Supreme Court in
two categories of matters:
▪ On any question of law or fact of public importance which has arisen or which is likely to
arise.
5 www.visionias.in ©Vision IAS

FREE BY KING R QUEEN P [ऋषभ राजपूत]


▪ On any dispute arising out of any pre-constitution treaty, agreement, covenant, engagement, sanad
or other similar instruments.
▪ In the first case, the Supreme Court may tender or may refuse to tender its opinion to the
president. Hence, statement 2 is correct.
▪ But, in the second case, the Supreme Court ‘must’ tender its opinion to the president. In both
cases, the opinion expressed by the Supreme Court is only advisory and not a judicial
pronouncement. Hence, it is not binding on the president; he may follow or may not follow the
opinion. However, it facilitates the government to have an authoritative legal opinion on a matter
to be punished for contempt not only of itself but also of high courts, subordinate courts and
tribunals functioning in the entire country.
• Power of Judicial Review
o Judicial review is the power of the Supreme Court to examine the constitutionality of legislative
enactments and executive orders of both the Central and state governments. On examination, if they
are found to be violative of the Constitution (ultra-vires), they can be declared illegal, unconstitutional
and invalid (null and void) by the Supreme Court. Consequently, they cannot be enforced by the
Government.
• Constitutional Interpretation
o The Supreme Court is the ultimate interpreter of the Constitution. It can give the final version of the
spirit and content of the provisions of the constitution and the verbiage used in the constitution.
o While interpreting the constitution, the Supreme Court is guided by a number of doctrines.
• Other Powers
o Besides the above, the Supreme Court has numerous other powers:
▪ It decides the disputes regarding the election of the president and the vice president. In this regard,
it has the original, exclusive and final authority.
▪ It enquires into the conduct and behaviour of the chairman and members of the Union Public
Service Commission on a reference made by the president. If it finds them guilty of misbehaviour,
it can recommend to the president for their removal. The advice tendered by the Supreme Court in
this regard is binding on the President.
▪ It has the power to review its own judgement or order. Thus, it is not bound by its previous
decision and can depart from it in the interest of justice or community welfare. Hence,
statement 3 is correct.
▪ In brief, the Supreme Court is a self-correcting agency. For example, in the Kesavananda Bharati
case (1973), the Supreme Court departed from its previous judgement in the Golak Nath case
(1967).
▪ It is authorised to withdraw the cases pending before the high courts and dispose of them by itself.
It can also transfer a case or appeal pending before one high court to another high court.
▪ Its law is binding on all courts in India. Its decree or order is enforceable throughout the country.
All authorities (civil and judicial) in the country should act in aid of the Supreme Court.
▪ It has the power of judicial superintendence and control over all the courts and tribunals
functioning in the entire territory of the country.
• The Supreme Court’s jurisdiction and powers with respect to matters in the Union list can be enlarged by
the Parliament.
• Further, its jurisdiction and powers with respect to other matters can be enlarged by a special agreement
between the Centre and the states.
Q 12.C
• The First Movers Coalition is a global initiative harnessing the purchasing power of companies to
decarbonize seven “hard to abate” industrial sectors that currently account for 30% of global
emissions: Aluminium, Aviation, Chemicals, Concrete, Shipping, Steel, and Trucking; along with
innovative Carbon Removal technologies. Hence statement 1 is correct.
• The 50+ companies who make up the Coalition seek to send a powerful market signal to
commercialize zero-carbon technologies. The First Movers Coalition’s unique approach assembles
ambitious corporate purchasing pledges across the heavy industry and long-distance transport sectors
responsible for a third of global emissions. For these sectors to decarbonize at the speed needed to keep
the planet on a 1.5-degree pathway, they require low-carbon technologies that are not yet competitive with
current carbon-intensive solutions but must reach commercial scale by 2030 to achieve net-zero emissions
globally by 2050.
• To jump-start the market, the coalition’s members commit in advance to purchasing a proportion of the
industrial materials and long-distance transportation they need from suppliers using near-zero or zero-
carbon solutions, despite the premium cost.
6 www.visionias.in ©Vision IAS

FREE BY KING R QUEEN P [ऋषभ राजपूत]


• India participated in the First Movers Coalition (FMC) Leadership Meeting of the World Economic
Forum which deliberated on the world need for clean energy technologies to confront the climate
crisis globally. Industry leaders appreciated India’s fight against COVID leveraging digital technologies
and also expressed their appreciation for the Government partnering with industry in its fight. Hence
statement 2 is correct.

Q 13.B
• The organizational structure, jurisdiction, and nomenclature of the subordinate judiciary are laid
down by the states. Hence, they differ slightly from state to state. Hence statement 1 is not correct.
• For example, at the lowest level, on the civil side, is the Court of Munsiff, and on the criminal side, is the
Court of Judicial Magistrate. The munsiff possesses limited jurisdiction and decides civil cases of small
pecuniary stake. The judicial magistrate tries criminal cases which are punishable with imprisonment for a
term of up to three years. In some metropolitan cities, there are city civil courts (chief judges) on the civil
side and the courts of metropolitan magistrates on the criminal side
• The Panchayat courts function in civil and criminal areas under various regional names like Nyaya
Panchayat, Panchayat Adalat, Gram Kutchery, etc.
• Munsiff’s courts are next-level civil courts, the jurisdictions of which are determined by High
Courts. Hence statement 2 is correct.
• Above Munsiffs are subordinate judges who have unlimited pecuniary jurisdiction and act as first appeals
from munsiffs.

Q 14.C
• Recently, Madhya Pradesh has notified its PESA Rules on the occasion of Janjatiya Gaurav Divas on 15th
November, 2022. At the State Level Janjatiya Gaurav Divas Sammelan at Shahdol in Madhya Pradesh,
Governor of Madhya Pradesh Shri Mangubhai Patel handed over the first copy of the Panchayats
(Extension to Scheduled Areas) Act (PESA Act) Manual to the President of India Smt. Droupadi Murmu.
• The provisions of Part IX of the constitution relating to the Panchayats are not applicable to the Fifth
Schedule areas. However, the Parliament may extend these provisions to such areas, subject to such
exceptions and modifications as it may specify. Under this provision, the Parliament enacted the
“Provisions of the Panchayats (Extension to the Scheduled Areas) Act”, 1996, popularly known as the
PESA Act or the Extension Act.
• Presently, 10 States viz. Andhra Pradesh, Chhattisgarh, Gujarat, Himachal Pradesh, Jharkhand,
Madhya Pradesh, Maharashtra, Odisha, Rajasthan and Telangana, have Fifth Schedule Areas in
their respective States.
• Out of the ten PESA States, eight States namely; Andhra Pradesh, Chhattisgarh, Gujarat,
Himachal Pradesh, Maharashtra, Madhya Pradesh, Rajasthan and Telangana have framed and
notified their State PESA Rules under their respective State Panchayati Raj Acts. Nine PESA
States, except Rajasthan, have incorporated the provisions of PESA 1996 in their respective State
Panchayati Raj Acts. The tenth State, Rajasthan, has notified “The Rajasthan Panchayat Raj
(Modification of Provisions in their Application to the Scheduled Areas) Act 1999”.
• Hence option (c) is the correct answer.

Q 15.A
• Prime Minister advises the president with regard to the appointment of important officials like
attorney general of India, Comptroller and Auditor General of India, chairman and members of the
UPSC, election commissioners, chairman and members of the finance commission and so on.
• Article 280 of the Constitution of India provides for a Finance Commission as a quasi judicial
body. It is constituted by the president of India every fifth year or at such earlier time as he
considers necessary. The Finance Commission consists of a chairman and four other members to be
appointed by the president. They hold office for such period as specified by the president in his
order. They are eligible for reappointment. Hence option 1 is correct.
• The Election Commission shall consist of the chief election commissioner and such number of other
election commissioners, if any, as the president may from time to time fix.The appointment of the chief
election commissioner and other election commissioners shall be made by the president. Hence
option 2 is correct.
• The Constitution (Article 76) has provided for the office of the Attorney General for India. He is the
highest law officer in the country. The Attorney General (AG) is appointed by the president. In
addition to the AG, there are other law officers of the Government of India. They are the solicitor
general of India and additional solicitor general of India. They assist the AG in the fulfilment of his
7 www.visionias.in ©Vision IAS

FREE BY KING R QUEEN P [ऋषभ राजपूत]


official responsibilities. It should be noted here that only the office of the AG is created by the
Constitution. In other words, Article 76 does not mention about the solicitor general and additional
solicitor general. Hence option 3 is not correct.
• The solicitor general of India is appointed to assist the attorney general along with four additional
solicitors general by the Appointments Committee of the Cabinet. The proposal for appointment of
Solicitor General, Additional Solicitor General is generally moved at the, level of Joint secretary/Law
Secretary in the Department of Legal Affairs and after obtaining the approval of the Minister of Law &
Justice, the proposal is sent to the Appointments Committee of the Cabinet for its approval.

Q 16.A
• In each House of Parliament, there is the ‘Leader of the Opposition’.
• The leader of the largest Opposition party having not less than one-tenth seats of the total strength
of the House is recognized as the leader of the Opposition in that House. Hence, statement 2 is correct.
• In a parliamentary system of government, the leader of the opposition has a significant role to play. His
main functions are to provide constructive criticism of the policies of the government and to provide an
alternative government. Therefore, the leader of the Opposition in the Lok Sabha and the Rajya Sabha
were accorded statutory recognition in 1977. Hence, statement 1 is not correct.
• They are also entitled to the salary, allowances, and other facilities equivalent to that of a cabinet
minister. Hence, statement 3 is correct.

Q 17.A
• SUPREME COURT ADVOCATES
o Three categories of Advocates are entitled to practice law before the Supreme Court. They are:
✓ Senior Advocates
❖ These are Advocates who are designated as Senior Advocates by the Supreme Court of
India or by any High Court. Hence, statement 1 is correct.
❖ The Court can designate any Advocate, with his consent, as Senior Advocate if in its opinion
by virtue of his ability, standing at the Bar or special knowledge or experience in law the said
Advocate is deserving of such distinction.
❖ A Senior Advocate is not entitled to appear without an Advocate-on-Record in the
Supreme Court or without a junior in any other court or tribunal in India. Hence,
statement 2 is correct.
❖ He is also not entitled to accept instructions to draw pleadings or affidavits, advise on
evidence or do any drafting work of an analogous kind in any court or tribunal in India or
undertake conveyancing work of any kind whatsoever but this prohibition shall not extend to
settling any such matter as aforesaid in consultation with a junior.
✓ Advocates-on-Record
❖ Only these advocates are entitled to file any matter or document before the Supreme
Court. They can also file an appearance or act for a party in the Supreme Court. Hence,
statement 3 is not correct.
✓ Other Advocates
❖ These are advocates whose names are entered on the roll of any State Bar Council maintained
under the Advocates Act, 1961 and they can appear and argue any matter on behalf of a party
in the Supreme Court but they are not entitled to file any document or matter before the
Court.

Q 18.A
• The maximum strength of the Rajya Sabha is fixed at 250, out of which, 238 are to be the representatives
of the states and union territories (elected indirectly) and 12 are nominated by the president.
• At present, the Rajya Sabha has 245 members. Of these, 229 members represent the states, 4 members
represent the union territories and 12 members are nominated by the president.
• The Fourth Schedule of the Constitution deals with the allocation of seats in the Rajya Sabha to the states
and union territories. Hence, statement 2 is correct.
o Representation of States: The representatives of states in the Rajya Sabha are elected by the elected
members of state legislative assemblies. The election is held in accordance with the system of
proportional representation by means of the single transferable vote. The seats are allotted to the
states in the Rajya Sabha on the basis of population. Hence, the number of representatives varies
from state to state. For example, Uttar Pradesh has 31 members while Tripura has 1 member only.
However, in the USA, all states are given equal representation in the Senate irrespective of their
8 www.visionias.in ©Vision IAS

FREE BY KING R QUEEN P [ऋषभ राजपूत]


population. The USA has 50 states and the Senate has 100 members–2 from each state. Hence,
statement 1 is not correct.
o Representation of Union Territories: The representatives of each union territory in the Rajya
Sabha are indirectly elected by members of an electoral college specially constituted for the
purpose. This election is also held in accordance with the system of proportional representation by
means of the single transferable vote. Out of the nine union territories, only three (Delhi, Puducherry,
and Jammu & Kashmir) have representation in Rajya Sabha. The populations of the other six union
territories are too small to have any representative in the Rajya Sabha.
o Nominated Members: The president nominates 12 members to the Rajya Sabha from people who
have special knowledge or practical experience in art, literature, science, and social service. The
rationale behind this principle of nomination is to provide eminent persons a place in the Rajya Sabha
without going through the process of election. It should be noted here that the American Senate has no
nominated Members.

Q 19.C
• Composition of Council (Article 171)
o Strength: Unlike the members of the legislative assembly, the members of the legislative council are
indirectly elected. The maximum strength of the council is fixed at one-third of the total strength of
the assembly and the minimum strength is fixed at 406. It means that the size of the council depends
on the size of the assembly of the concerned state.
o This is done to ensure the predominance of the directly elected House (assembly) in the legislative
affairs of the state. Though the Constitution has fixed the maximum and the minimum limits, the
actual strength of a Council is fixed by Parliament.
o Manner of Election: Of the total number of members of a legislative council:
▪ 1/3 are elected by the members of local bodies in the state like municipalities, district
boards, etc.,
▪ 1/12 are elected by graduates of three years standing and residing within the state,
▪ 1/12 are elected by teachers of three years standing in the state, not lower in standard than
secondary school,
▪ 1/3 are elected by the members of the legislative assembly of the state from amongst persons
who are not members of the assembly, Hence option (c) is the correct answer.
▪ and the remainder is nominated by the governor from persons who have a special knowledge or
practical experience of literature, science, art, cooperative movement, and social service.
▪ Thus, 5/6 of the total number of members of a legislative council are indirectly elected and 1/6 are
nominated by the governor. The members are elected in accordance with the system of
proportional representation by means of a single transferable vote. The bonafide or propriety of
the governor’s nomination, in any case, cannot be challenged in the courts.
o This scheme of the composition of a legislative council as laid down in the Constitution is tentative
and not final. The Parliament is authorized to modify or replace the same. However, it has not enacted
any such law so far.

Q 20.C
• The Metropolitan Planning Committee is established by Article 243ZE of the Constitution. Every
metropolitan area shall have a metropolitan planning committee to prepare a draft development plan. The
state legislature may make provisions with respect to the following:
o The composition of such committees;
o The manner of election of members of such committees;
o The representation in such committees of the Central government, state government and other
organizations;
o The functions of such committees in relation to planning and
o coordination for the metropolitan area; and
o The manner of election of chairpersons of such committees. Hence, statement 1 is correct.
• The act lays down that two-thirds of the members of a metropolitan planning committee should be
elected by the elected members of the municipalities and chairpersons of the panchayats in the
metropolitan area from amongst themselves. The representation of these members in the committee
should be in proportion to the ratio between the population of the municipalities and the panchayats in that
metropolitan area. Hence, statement 2 is not correct.
• The chairpersons of such committees shall forward the development plan to the state government.
Hence, statement 3 is correct.
9 www.visionias.in ©Vision IAS

FREE BY KING R QUEEN P [ऋषभ राजपूत]


• In preparing the draft development plan, a metropolitan planning committee shall have to regard to
o The plans prepared by the Municipalities and the Panchayats in the Metropolitan area;
o Matters of common interest between the Municipalities and Panchayats, including co-ordinated
spatial planning of the area sharing of water and other physical and natural resources, t integrated
development of infrastructure and environment conservation;
o The overall objectives and priorities set by the Government of India and the government of the state;
o The extent and nature of investments likely to be made in the Metropolitan area by agencies of the
Government of India and the Government of the State and other available resources whether financial
or otherwise; and
• Also, the Metropolitan Planning Committee shall consult such institutions and organizations as the
Governor may specify.

Q 21.D
• Governor can promulgate ordinances when the state legislature is not in session. These ordinances
must be approved by the state legislature within six weeks from its reassembly. He can also withdraw
an ordinance anytime. This is the most important legislative power of the governor.
• He cannot make an ordinance without the instructions from the President in three cases:
o If a bill containing the same provisions would have required the previous sanction of the President for
its introduction into the state legislature.
o If he would have deemed it necessary to reserve a bill containing the same provisions for the
consideration of the President.
o if an Act of the Legislature of the State containing the same provisions would under this Constitution
have been invalid unless, having been reserved for the consideration of the President, it had received
the assent of the President.
• Hence, option (d) is the correct answer.

Q 22.D
• Article 72 of the Constitution empowers the President to grant pardons to persons who have been
tried and convicted of any offence in all cases where the:
o Punishment or sentence is for an offence against a Union Law;
o Punishment or sentence is by a court martial (military court);
o Sentence is a sentence of death.
• The pardoning power of the President is independent of the Judiciary; it is an executive power. But,
the President while exercising this power, does not sit as a court of appeal.
• The pardoning power of the President includes the following:
o Pardon removes both the sentence and the conviction and completely absolves the convict from all
sentences, punishments and disqualifications.
o Commutation denotes the substitution of one form of punishment for a lighter form. For
example, a death sentence may be commuted to rigorous imprisonment, which in turn may be
commuted to a simple imprisonment. Hence statement 2 is correct.
o Remission implies reducing the period of sentence without changing its character. For example, a
sentence of rigorous imprisonment for two years may be remitted to rigorous imprisonment for one
year. Hence statement 1 is correct.
o Respite denotes awarding a lesser sentence in place of one originally awarded due to some
special fact, such as the physical disability of a convict or the pregnancy of a woman offender. Hence
statement 3 is correct.
o Reprieve implies a stay of the execution of a sentence (especially that of death) for a temporary
period. Its purpose is to enable the convict to have time to seek pardon or commutation from the
President.

Q 23.D
• INDEPENDENCE OF SUPREME COURT
• The Supreme Court has been assigned a very significant role in the Indian democratic political system. It
is a federal court, the highest court of appeal, the guarantor of the fundamental rights of the citizens and
the guardian of the Constitution.
• Therefore, its independence becomes very essential for the effective discharge of the duties assigned to it.
It should be free from the encroachments, pressures and interferences of the executive (council of
ministers) and the Legislature (Parliament). It should be allowed to do justice without fear or favour.

10 www.visionias.in ©Vision IAS

FREE BY KING R QUEEN P [ऋषभ राजपूत]


• The Constitution has made the following provisions to safeguard and ensure the independent and
impartial functioning of the Supreme Court:
o Mode of Appointment
▪ The judges of the Supreme Court are appointed by the President (which means the cabinet) in
consultation with the members of the judiciary itself (ie, judges of the Supreme Court and the
high courts). This provision curtails the absolute discretion of the executive as well as ensures that
the judicial appointments are not based on any political or practical considerations.
o Security of Tenure
▪ The judges of the Supreme Court are provided with the Security of Tenure. They can be removed
from office by the President only in the manner and on the grounds mentioned in the Constitution.
This means that they do not hold their office at the pleasure of the President, though they are
appointed by him. This is obvious from the fact that no judge of the Supreme Court has been
removed (or impeached) so far.
o Fixed Service Conditions
▪ The salaries, allowances, privileges, leave and pension of the judges of the Supreme Court are
determined from time to time by the Parliament. They cannot be changed to their disadvantage
after their appointment except during a financial emergency. Thus, the conditions of service of the
judges of the Supreme Court remain the same during their term of Office.
o Expenses Charged on Consolidated Fund
▪ The salaries, allowances and pensions of the judges and the staff as well as all the administrative
expenses of the Supreme Court are charged on the Consolidated Fund of India. Thus, they are
non-votable by the Parliament (though they can be discussed by it).
o The conduct of Judges cannot be Discussed
▪ The Constitution prohibits any discussion in Parliament or in a State Legislature with respect to
the conduct of the judges of the Supreme Court in the discharge of their duties, except when an
impeachment motion is under consideration of the Parliament.
o Ban on Practice after Retirement
▪ The retired judges of the Supreme Court are prohibited from pleading or acting in any Court or
before any authority within the territory of India. This ensures that they do not favour anyone in
the hope of future favour. Hence, option 3 is correct.
o Power to Punish for its Contempt
▪ The Supreme Court can punish any person for its contempt. Thus, its actions and decisions cannot
be criticised and opposed by anybody. This power is vested in the Supreme Court to maintain its
authority, dignity and honour. Hence, option 1 is correct.
o Freedom to Appoint its Staff
▪ The Chief Justice of India can appoint officers and servants of the Supreme Court without any
interference from the executive. He can also prescribe their conditions of service. Hence, option
2 is correct.
o Its Jurisdiction cannot be Curtailed
▪ The Parliament is not authorised to curtail the jurisdiction and powers of the Supreme Court. The
Constitution has guaranteed to the Supreme Court, the jurisdiction of various kinds. However, the
Parliament can extend the same. Hence, option 4 is correct.
o Separation from Executive
▪ The Constitution directs the State to take steps to separate the Judiciary from the Executive in
public services. This means that the executive authorities should not possess judicial powers.
Consequently, upon its implementation, the role of executive authorities in judicial administration
came to an end.

Q 24.B
• The judicial powers and functions of the governor are:
o He can grant pardons, reprives, respites and remissions of punishment or suspend, remit and commute
the sentence of any person convicted of any offence against any law relating to a matter to which the
executive power of the state extends.
o He is consulted by the president while appointing the judges of the concerned state high court.
o He makes appointments, postings and promotions of the district judges in consultation with the
state high court.
o He also appoints persons to the judicial service of the state (other than district judges) in consultation
with the state high court and the State Public Service Commission.

11 www.visionias.in ©Vision IAS

FREE BY KING R QUEEN P [ऋषभ राजपूत]


• Appointment of Judges of High Court
o The judges of a high court are appointed by the President. The chief justice is appointed by the
President after consultation with the chief justice of India and the governor of the state
concerned. Hence, statement 1 is not correct.
o For appointment of other judges, the chief justice of the concerned high court is also consulted. In
case of a common high court for two or more states, the governors of all the states concerned are
consulted by the president.
• The appointment, posting and promotion of district judges in a state are made by the governor of
the state in consultation with the high court. Hence, statement 2 is correct.

Q 25.D
• The Constitution does not contain any specific procedure for the selection and appointment of the
Chief Minister. Article 164 only says that the Chief Minister shall be appointed by the governor.
However, this does not imply that the governor is free to appoint any one as the Chief Minister. In
accordance with the conventions of the parliamentary system of government, the governor has to appoint
the leader of the majority party in the state legislative assembly as the Chief Minister.
• But, when no party has a clear majority in the assembly, then the governor may exercise his personal
discretion in the selection and appointment of the Chief Minister. In such a situation, the governor usually
appoints the leader of the largest party or coalition in the assembly as the Chief Minister and ask him to
seek a vote of confidence in the House within a month.
• The Constitution does not require that a person must prove his majority in the legislative assembly
before he is appointed as the Chief Minister. The governor may first appoint him as the Chief Minister
and then ask him to prove his majority in the legislative assembly within a reasonable period. This is what
has been done in a number of cases. Hence, statement 1 is not correct.
• A person who is not a member of the state legislature can be appointed as Chief Minister for six
months, within which time, he should be elected to the state legislature, failing which he ceases to be
the Chief Minister. Hence, statement 2 is not correct.

Q 26.C
• Consultative committees are attached to various ministries/departments of the Central Government. They
consist of members of both Houses of Parliament. The Minister / Minister of State in charge of the
Ministry concerned acts as the chairman of the consultative committee of that ministry.
• These committees provide a forum for informal discussions between the ministers and the members
of Parliament on policies and programs of the government and the manner of their implementation.
Hence, statement 1 is correct.
• These committees are constituted by the Ministry of Parliamentary Affairs. The guidelines regarding
the composition, functions, and procedures of these committees are formulated by this Ministry. The
Ministry also makes arrangements for holding their meetings both during the session and the inter-session
period of Parliament. Hence, statement 2 is correct.
• The membership of these committees is voluntary and is left to the choice of the members and the
leaders of their parties. The maximum membership of a committee is 30 and the minimum is 10. These
committees are normally constituted after the new Lok Sabha is constituted, after General Elections for
the Lok Sabha. In other words, these committees shall stand dissolved upon the dissolution of every Lok
Sabha and shall be reconstituted upon the constitution of each Lok Sabha.
• This committee regulates the program and timetable of the House. It allocates time for the transaction of
legislative and other business brought before the House by the government. The Lok Sabha committee
consists of 15 members including the Speaker as its chairman. In the Rajya Sabha, it has 11 members
including the Chairman as its ex-officio chairman.

Q 27.A
• The provisions of the 73rd Amendment Act can be grouped into two categories– compulsory and
voluntary. The compulsory (mandatory or obligatory) provisions of the act have to be included in the state
laws creating the new panchayati raj system. The voluntary provisions, on the other hand, may be
included at the discretion of the states. Thus the voluntary provisions of the act ensure the right of the
states to take local factors like geographical, politico-administrative and others, into consideration while
adopting the new panchayati raj system. Some of the compulsory provisions are given below.
• Compulsory Provisions:
o Organization of Gram Sabha in a village or group of villages.
o Establishment of panchayats at the village, intermediate and district levels.
12 www.visionias.in ©Vision IAS

FREE BY KING R QUEEN P [ऋषभ राजपूत]


o Direct elections to all seats in panchayats at the village, intermediate and district levels.
o Indirect elections to the post of chairperson of panchayats at the intermediate and district levels.
o Voting rights of the chairperson and other members of a panchayat elected directly or indirectly.
o 21 years to be the minimum age for contesting elections to panchayats.
o Reservation of seats (both members and chairpersons) for SCs and STs in panchayats at all three
levels.
o Reservation of one-third of seats (both members and chairpersons) for women in panchayats at
all three levels.
o Fixing tenure of five years for panchayats at all levels and holding fresh elections within six months in
the event of supersession of any panchayat.
o Establishment of a State Election Commission for conducting elections to the panchayats.
o Constitution of a State Finance Commission after every five years to review the financial position of
the panchayats.
• Voluntary Provisions:
o Endowing the Gram Sabha with powers and functions at the village level.
o Determining the manner of election of the chairperson of the village panchayat.
o Giving representation to the chairpersons of the village panchayats in the intermediate panchayats or
in the case of a state not having intermediate panchayats, in the district panchayats.
o Giving representation to the chairpersons of the intermediate panchayats in the district panchayats.
o Giving representation to members of the Parliament (both the Houses) and the state legislature
(both the Houses) in the panchayats at different levels falling within their constituencies.
o Providing reservation of seats (both members and chairpersons) for backward classes in
panchayats at any level.
o Granting powers and authority to the panchayats to enable them to function as institutions of self-
government (in brief, making them autonomous bodies).
o Devolution of powers and responsibilities upon panchayats to prepare plans for economic
development and social justice; and to perform some or all of the 29 functions listed in the
Eleventh Schedule of the Constitution.
o Granting financial powers to the panchayats, that is, authorizing them to levy, collect and
appropriate taxes, duties, tolls and fees.
o Assigning to a panchayat the taxes, duties, tolls and fees levied and collected by the state
government.
o Making the grants-in-aid to the panchayats from the consolidated fund of the state.
o Providing for the constitution of funds for crediting all money of the panchayats. Hence, option (a) is
the correct answer.

Q 28.D
• Municipal corporations are created for the administration of big cities like Delhi, Mumbai, Kolkata,
Hyderabad, Bangalore and others. They are established in the states by the acts of the concerned state
legislatures, and in the union territories by the acts of the Parliament of India. There may be one common
act for all municipal corporations in a state or a separate act for each municipal corporation.
• A municipal corporation has three authorities, namely, the council, the standing committees and the
commissioner.
• The Council is the deliberative and legislative wing of the corporation. It consists of the Councillors
directly elected by the people, as well as a few nominated persons having knowledge or experience
in municipal administration. In brief, the composition of the Council including the reservation of
seats for SCs, STs and women is governed by the 74th Constitutional Amendment Act, 1992 .
Hence, statement 1 is correct.
• The Council is headed by a Mayor. He is assisted by a Deputy Mayor. He is elected in a majority of the
states for a one-year renewable term. He is basically an ornamental figure and a formal head of the
corporation. His main function is to preside over the meetings of the Council. Hence statement 2 is
correct.
• The standing committees are created to facilitate the working of the council, which is too large in size.
They deal with public works, education, health, taxation, finance and so on. They take decisions in their
fields.
• The municipal commissioner is responsible for the implementation of the decisions taken by the
council and its standing committees. Thus, he is the chief executive authority of the corporation. He
is appointed by the state government and is generally a member of the IAS. Hence statement 3 is
correct.
13 www.visionias.in ©Vision IAS

FREE BY KING R QUEEN P [ऋषभ राजपूत]


Q 29.B
• Fast Track Courts (FTCs) are additional Session Courts set up for speeding up the trials of long
pending cases, particularly those involving under trials.
• Fast Track Courts were initially established for a period of five years (2000-2005).
• The 11th Finance Commission recommended for establishment of 1734 FTCs for expeditious disposal of
cases pending in lower courts.
• FTCs were established by state governments in consultation with respective high courts. Hence
statement 1 is not correct.
• Judges of these FTCs were appointed on an ad-hoc basis and they were selected by the High Court of
respective states. Hence statement 2 is correct. There are primarily three sources of recruitment:• by
promoting members from amongst the eligible judicial officers;• by appointing retired high court judges
and• recruited from amongst the member of the bar of the respected state.
• The cases are disposed of within a given time frame.
• In 2005, the Supreme Court directed the central government to continue with the FTC scheme,
which was extended until 2010-2011.
• Subsequently, the government discontinued the FTC scheme in March 2011 due to financial problems and
stopped financing FTCs. But as state governments enjoyed the liberty to continue if they want, some
states like Arunachal Pradesh, Assam, Maharashtra, Tamil Nadu, and Kerala decided to continue with
FTCs, while Haryana and Chhattisgarh discontinued.

Q 30.D
• Vacation of Seats
o In the following cases, a member of the state legislature vacates his seat:
o Double Membership: A person cannot be a member of both Houses of the state legislature at one and
the same time. If a person is elected to both Houses, his seat in one of the houses falls vacant as per
the provisions of a law made by the state legislature.
o Disqualification: If a member of the state legislature becomes subject to any of the disqualifications,
his seat becomes vacant.
o Resignation: A member may resign his seat by writing to the Chairman of the legislative council or
Speaker of the legislative assembly, as the case may be. The seat falls vacant when the resignation
is accepted.
o Absence: A House of the state legislature can declare the seat of a member vacant if he absences
himself from all its meeting for a period of sixty days without its permission. Hence option (d) is
the correct answer.
o Other Cases: A member has to vacate his seat in the either House of the state legislature,
▪ if his election is declared void by the court,
▪ if he is expelled by the House,
▪ if he is elected to the office of president or office of vice-president, and
▪ If he is appointed to the office of governor of a state.

Q 31.B
• The President is the head of the Indian State. He is the first citizen of India and acts as the symbol of
unity, integrity and solidarity of the nation.
• The President is elected not directly by the people but by members of electoral college consisting of:
o the elected members of both the Houses of Parliament;
o the elected members of the legislative assemblies of the states, and
o the elected members of the legislative assemblies of the Union Territories of Delhi and
Puducherry
• Thus, the nominated members of both of Houses of Parliament, the nominated members of the state
legislative assemblies, the members (both elected and nominated) of the state legislative councils (in case
of the bicameral legislature) and the nominated members of the Legislative Assemblies of Delhi and
Puducherry do not participate in the election of the President.
• Hence, option (b) is the correct answer.

Q 32.D
• Parliament's legislative power on subjects included in the State List
o Further, under special conditions, the Parliament can legislate on subjects included in the State List,
under some specific circumstances, which are as follows:

14 www.visionias.in ©Vision IAS

FREE BY KING R QUEEN P [ऋषभ राजपूत]


o In the National Interest (Art.249) - If the Council of States (Rajya Sabha) declares that it is
necessary for the Centre to legislate upon a subject in the State list, in the national interest, and
passes a resolution to this effect, with a majority of at least 2/3rd of members present and voting.
This resolution remains in force for a year and can be renewed any number of times, but for not
more than one year at a time. The laws so made do not have any effect six months after the
resolution has ceased to be in force. At the same time, the State can also legislate upon the same
subject, but in case of any inconsistency, laws of the Centre prevail. This particular feature makes
the entire legislative process federal in nature. Hence option 1 is correct.
o By Agreement between States (Art. 252) - When two or more State Legislatures pass a
resolution, requesting the Parliament to legislate upon a subject in the State List. The law passed
by Union Parliament shall be applicable only to the States, which demanded such legislation. Any
other State may later adopt it by passing a resolution to that effect. In this case, States cease to
have the power to legislate upon that subject and only the Parliament can amend or repeal such a
law. In past, laws have been made using this provision, some of them are The Wildlife
(Protection) Act of 1972, and the Urban Land (Ceiling and Regulation) Act. Hence option 2 is
correct.
o To Implement International Treaties and Agreements (Art. 253) - This provision enables the
central government to fulfil its international obligations. The Lokpal and the Lokayuktas Bill,
2011 was introduced in the Parliament through the provisions of this particular article. Hence,
option 3 is correct.
o Under Proclamation of National Emergency (Article 352) - During a national emergency, the
Parliament can legislate upon any subject in the State List. Such a law becomes inoperative on the
expiration of six months after the emergency has ceased to operate. However, at the same time,
the State can also legislate upon the same subject, but in case of any inconsistency, the laws of the
Centre prevail.
o Under Proclamation of President’s Rule (Art.356) - During President’s rule in a State, the
Parliament can make laws with respect to any subject in the State list, in relation to that state.
Such a law continues to be operative even after the President’s rule. But it can be repealed, altered
or re-enacted later by the State Legislature.
o The Centre cannot make a law on a subject in the state list when a model code of conduct has
been implemented due to state legislative elections. Hence option 4 is not correct.
Q 33.A
• The Governor can promulgate an ordinance only when the legislative assembly (in case of a unicameral
legislature) is not in session or (in case of a bi- cameral legislature) when both the Houses of the state
legislature are not in session or when either of the two Houses of the state legislature is not in session.
• His ordinance-making power is co-extensive with the legislative power of the state legislature. This
means that he can issue ordinances only on those subjects on which the state legislature can make
laws. Hence, statement 1 is correct.
• An ordinance issued by him is subject to the same limitations as an act of the state legislature. This means
that an ordinance issued by him will be invalid to the extent it makes any provision which the state
legislature cannot make.
• His ordinance-making power is not a discretionary power. This means that he can promulgate or
withdraw an ordinance only on the advice of the council headed by the chief minister. Hence, statement
2 is correct.
• An ordinance issued by him ceases to operate on the expiry of six weeks from the reassembly of the
state legislature. It may cease to operate even earlier than the prescribed six weeks, if a resolution
disapproving it is passed by the legislative assembly and is agreed to by the legislative council (in case of
a bicameral legislature). Hence, statement 3 is not correct.

Q 34.C
• Sarkaria Commission
o The Central government appointed a three-member Commission in 1983 on the Centre-State
relationship under the Chairmanship of R S Sarkaria, a retired Judge of the Supreme Court. The
Commission submitted its report in October 1987 with 247 recommendations. Its important
recommendations were:
▪ A permanent Inter-State Council called the Inter-Governmental Council should be set up
under Article 263. Hence, option (c) is the correct answer.
▪ Article 356 (President’s rule) should be used very sparingly, in extreme cases as a last resort when
all the available alternatives fail.
15 www.visionias.in ©Vision IAS

FREE BY KING R QUEEN P [ऋषभ राजपूत]


▪ The institution of All-India Services should be further strengthened and some more such services
should be created.
▪ The residuary power of taxation should continue to remain with the parliament, while the other
residuary powers should be placed in the concurrent list.
▪ When the President withholds his assent to the state bills, the reason should be communicated to
the state government.
▪ The Zonal Councils should be constituted afresh and reactivated to promote the spirit of
federalism.
▪ The Centre should have powers to deploy its armed forces, even without the consent of states.
However, it is desirable that the states should be consulted.
▪ The Centre should consult the states before making a law on a subject of the Concurrent List.
▪ The procedure of consulting the Chief Minister in the appointment of State Governor should be
prescribed in the Constitution itself.
▪ The Governor’s term of five years in a state should not be disrupted except for extremely
compelling reasons.

Q 35.A
• Choudhary Rehmat Ali can be credited with coining the “term” Pakistan, styling himself as the “Founder
of the Pakistan National Movement”. On January 28, 1933, he released a pamphlet titled “Now or Never:
Are we to live or perish forever”. In it he made a vehement “appeal on behalf of the thirty million
Muslims of PAKISTAN, who live in the five Northern Units of India, for the recognition of their national
status, as distinct from the other inhabitants of India, by the grant to Pakistan of a separate Federal
Constitution on religious, social and historical grounds.” Hence statement 1 is correct.
• In 1940, the famous Lahore Resolution was passed, advocating that the “geographical contiguous units” in
the Muslim-majority areas in India’s “North-Western and Eastern Zones of India, should be grouped to
constitute Independent States in which the constituent units shall be autonomous and sovereign.” While
this resolution did not mention “Pakistan,” Jinnah’s ideas echoed Rahmat Ali’s. Hence statement 2 is not
correct.
• Rehmat Ali was not a politician. Nor did he stay in the subcontinent for much of the 1930s and 1940s
when the struggle for Pakistan was taking shape. His contribution to Pakistan are solely limited to his
writings and ideas.
• Unlike Allama Iqbal, more popularly known as the philosopher behind Pakistan’s creation, Ali’s work
remained restricted to a far smaller audience.

Q 36.B
• The 73rd amendment gave constitutional status to the Panchayati Raj institutions. It brought in uniformity
with the three-tier Panchayati Raj structure. At the base is the ‘Gram Panchayat‘. A Gram Panchayat
covers a village or group of villages. The intermediary level is the Mandal (also referred to as Block or
Taluka). These bodies are called Mandal or Taluka Panchayats. The intermediary level body need not be
constituted in smaller States. At the apex is the Zilla Panchayat covering the entire rural area of the
District.
• The act provides for a Gram Sabha as the foundation of the Panchayati raj system. It is a body consisting
of persons registered in the electoral rolls of a village comprised within the area of Panchayat at the
village level. Thus, it is a village assembly consisting of all the registered voters in the area of a
panchayat. Hence statement 1 is not correct.
• It may exercise such powers and perform such functions at the village level as the legislature of a
state determines. Hence statement 2 is correct.

Q 37.C
• The human genome is made up of DNA (deoxyribonucleic acid), a long, winding molecule that contains
the instructions needed to build and maintain cells. These instructions are spelled out in the form of "base
pairs" of four different chemicals, organized into 20,000 to 25,000 genes. For the instructions to be carried
out, DNA must be "read" and transcribed - in other words, copied - into RNA (ribonucleic acid). These
gene readouts are called transcripts, and a transcriptome is a collection of all the gene readouts present in
a cell.
• The entire RNA landscape is known collectively as the transcriptome, and mapping this allows
researchers to better understand an individual’s gene expression. Hence statement 1 is correct.
• Ribonucleic acid (abbreviated RNA) is a nucleic acid present in all living cells that has structural
similarities to DNA. RNA’s most commonly recognized function is to translate DNA into
16 www.visionias.in ©Vision IAS

FREE BY KING R QUEEN P [ऋषभ राजपूत]


proteins, but scientists now understand that the vast majority of RNA does not make proteins, but instead
can play roles such as influencing cell structure or regulating genes. Hence statement 2 is correct.
• In a new paper published in the journal Nature Methods, researchers at the University of California, Santa
Cruz, have proposed a “pantranscriptome,” which combines a transcriptome and a pangenome -- a
reference that contains genetic material from a cohort of diverse individuals, rather than just a single
linear strand.
• The pantranscriptome-concept builds on the emerging concept of “pangenomics” in the genomics field.
Typically, when evaluating an individual’s genomic data for variation, scientists compare the individual’s
genome to that of a reference made up of a single, linear strand of DNA bases. Using a pangenome allows
researchers to compare an individual’s genome to that of a genetically diverse cohort of reference
sequences all at once, sourced from individuals representing a diversity of biogeographic ancestry. This
gives the scientists more points of comparison for which to better understand an individual’s genomic
variation.

Q 38.A
• Recently it was reported that the necropolis of noblemen dating from the Asaf Jahi era known as
Paigah Tombs Complex in Santosh Nagar is set to be restored with funding from the US
Ambassadors Fund for Cultural Preservation.
• Paigah Tombs or Maqhbara Shams al-Umara, are the tombs belonging to the nobility of the Paigah
family, who were fierce loyalists of the Nizams. The Paigah tombs are among the major wonders of
Hyderabad State which are known for its architectural excellence. These tombs are 200 years old and
represent the final resting places of several generations of the Paigah Nobles.
• One of the best examples of Indo-Saracenic architecture, the tombs present a beautiful blend of
Asaf Jah and Rajputani styles of architecture. Built of white marble, Paigah tombs captivate visitors
with intricately designed interiors. Paigahs who married daughters of the Nizams were the highest-
ranking nobles in the princely state of Hyderabad, after the family of Nizams. Paigahs were the only
noble family who was permitted by the Sultan to have their private army. Hence option (a) is the
correct answer.
• These tombs are magnificent structures, decorated in stucco work, and represent the Mughal, Greek,
Persian, Asaf Jahi, Rajasthani, and Decani styles of architecture. Stucco or render is a construction
material made of aggregates, a binder, and water. Stucco is applied wet and hardens to a very dense solid.
It is used as a decorative coating for walls and ceilings, exterior walls, and as a sculptural and artistic
material in architecture. The famous ‘Jali’ work makes it even more attractive. Each Tomb has been
designed differently, ensuing a unique style and design.

Q 39.A
• The Darling 58 is a genetically engineered American chestnut tree. The tree was created by American
Chestnut Research & Restoration Program at the State University of New York College of Environmental
Science and Forestry (United States of America), to restore the American chestnut to the forests of North
America. Hence option (a) is the correct answer.
• Native un-modified trees are killed from the ground up by the blight, and only the root system survives.
The roots then continue to send up shoots that are once again attacked by the blight and die back before
they reach maturity, repeating the cycle. Darling-58 trees survive to reach maturity, produce chestnuts,
and multiply to restore the American Chestnut tree to the forests of North America. These Darling-58
trees are attacked by chestnut blight but survive.
• More than a century after the American chestnut tree became functionally extinct, the United States is
weighing whether to allow a genetically engineered (GE) version to spread in the wild.

Q 40.C
• Writ Jurisdiction of High Courts
o Article 226 of the Constitution empowers a high court to issue writs including habeas corpus,
mandamus, certiorari, prohibition and quo warranto for the enforcement of the fundamental rights of
the citizens and for any other purpose.
o The phrase ‘for any other purpose’ refers to the enforcement of an ordinary legal right.
o The high court can issue writs to any person, authority and government not only within its
territorial jurisdiction but also outside its territorial jurisdiction if the cause of action arises
within its territorial jurisdiction.
o The writ jurisdiction of the high court (under Article 226) is not exclusive but concurrent with the writ
jurisdiction of the Supreme Court (under Article 32).
17 www.visionias.in ©Vision IAS

FREE BY KING R QUEEN P [ऋषभ राजपूत]


o It means, when the fundamental rights of a citizen are violated, the aggrieved party has the option of
moving either the high court or the Supreme Court directly.
o However, the writ jurisdiction of the high court is wider than that of the Supreme Court.
o This is because, the Supreme Court can issue writs only for the enforcement of fundamental
rights and not for any other purpose, that is, it does not extend to a case where the breach of an
ordinary legal right is alleged. Hence, statement 1 is correct.
o In the Chandra Kumar case (1997), the Supreme Court ruled that the writ jurisdiction of both
the high court and the Supreme Court constitute a part of the basic structure of the
Constitution. Hence, statement 2 is correct.
o Hence, it cannot be ousted or excluded even by way of an amendment to the Constitution.

Q 41.B
• Article 161 mentions the pardoning power of the Governor. It states that the Governor of a
State shall have the power to grant pardons, reprieves, respites, or remissions of punishment or to
suspend, remit or commute the sentence of any person convicted of any offense against any law
relating to a matter to which the executive power of the State extends. Hence, statement 2 is correct.
• He cannot pardon a death sentence. Even if a state law prescribes for death sentence, the power to grant
pardon lies with the President and not the governor. Hence, statement 1 is not correct.

Q 42.C
• Qualifications of Judges
o A person to be appointed as a judge of the Supreme Court should have the following qualifications:
▪ He should be a citizen of India.
▪ He should have been a judge of a High Court (or high courts in succession) for five years; or
▪ He should have been an advocate of a High Court (or High Courts in succession) for ten
years, or
▪ He should be a distinguished jurist in the opinion of the president.
▪ Hence, statement 1 is correct.
▪ From the above, it is clear that the Constitution has not prescribed a minimum age for
appointment as a judge of the Supreme Court.
o Salaries and Allowances
▪ The salaries, allowances, privileges, leave, and pension of the judges of the Supreme Court are
determined from time to time by the Parliament.
▪ They cannot be varied to their disadvantage after their appointment except during a financial
emergency. In 2018, the salary of the chief justice was increased from ₹1 lakh to ₹2.80 lakh per
month and that of a judge from ₹90,000 to ₹2.50 lakh per month.
▪ They are also paid a sumptuary allowance and provided with free accommodation and other
facilities like medical, car, telephone, etc.
▪ The retired chief justice and judges are entitled to 50 per cent of their last drawn salary as a
monthly pension.
o Tenure of Judges
▪ The Constitution has not fixed the tenure of a judge of the Supreme Court. Hence,
statement 2 is correct.
▪ However, it makes the following three provisions in this regard:
✓ He holds office until he attains the age of 65 years. Any question regarding his age is to be
determined by such authority and in such manner as provided by Parliament.
✓ He can resign his office by writing to the president.
✓ He can be removed from his office by the President on the recommendation of the
Parliament.

Q 43.B
• Recently, DRDO carried out the test of the Hypersonic Technology Demonstrator Vehicle off the coast of
Odisha. India's Defence Research and Development Organisation (DRDO) successfully tested an
indigenously-developed hypersonic technology demonstrator vehicle (HSTDV) powered by a
scramjet engine. Hence statement 1 is not correct.
• The air-breathing scramjet engine was successfully flight-tested at hypersonic speed within the
atmosphere. The test paves the way for development of many more critical technologies, materials and
hypersonic vehicles.

18 www.visionias.in ©Vision IAS

FREE BY KING R QUEEN P [ऋषभ राजपूत]


• Hypersonic Technology Demonstrator Vehicle (HSTDV) is an unmanned scramjet demonstration aircraft
for hypersonic speed flight. HSTDV is not a weapon itself but and is being developed as a carrier vehicle
for hypersonic and long-range cruise missiles. The HSTDV cruise vehicle is mounted on a solid rocket
motor, which takes it to a required altitude. Once it attains certain mach numbers for speed, the cruise
vehicle is ejected out of the launch vehicle.
• A scramjet engine is an improvement over the ramjet engine because the former operates efficiently at
hypersonic speeds and allows supersonic combustion. Ramjets, in contrast, operate well at supersonic
speeds around Mach 3 but their efficiency drops at hypersonic speeds.
• The HSTDV has an air-breathing scramjet engine which means that it uses atmospheric oxygen for
propulsion. Launch vehicles use combustion of propellants consisting of oxidiser and fuel for deriving
energy. Air breathing propulsion systems use atmospheric oxygen, which is available up to about 50 km
of earth's surface to burn the fuel stored on-board thereby making the system much lighter, more efficient
and cost effective. Air breathing propulsion is a solution for a powered long return cruise flight necessary
for reusable launch vehicles. Hence statement 2 is correct.

Q 44.A
• The Constitution lays down the following conditions of the President’s office:
o He should not be a member of either House of Parliament or a House of the state legislature. If any
such person is elected as President, he is deemed to have vacated his seat in that House on the date on
which he enters upon his office as President.
o He should not hold any other office of profit.
o He is entitled, without payment of rent, to the use of his official residence (the Rastrapathi Bhavan).
o He is entitled to such emoluments, allowances and privileges as may be determined by
Parliament. Hence, statement 2 is not correct.
• The President holds office for a term of five years from the date on which he enters upon his
office. The President can hold office beyond his term of five years until his successor assumes charge. He
is also eligible for re-election to that office. He may be elected for any number of terms. However, in
USA, a person cannot be elected to the office of the President more than twice. Hence, statement 1 is
correct.
• The veto power enjoyed by the executive in modern states can be classified into the following four types:
o Absolute veto, that is, withholding of assent to the bill passed by the legislature.
o Qualified veto, which can be overridden by the legislature with a higher majority.
o Suspensive veto, which can be overridden by the legislature with an ordinary majority.
o Pocket veto, that is, taking no action on the bill passed by the legislature.
• Of the above four, the President of India is vested with three– absolute veto, suspensive veto and
pocket veto. There is no qualified veto in the case of Indian President; it is possessed by the
American President. Hence, statement 3 is not correct.

Q 45.C
• FireAId initiative: Wildfire severity, spread and frequency have all increased as a result of climate
change. Wildfires pose a particular threat to forest ecosystems as well as high-risk rural areas.
• Critical services and resources such as health and safety, forestry, natural disaster and emergency relief
agencies, and rural planning are also overburdened, and existing hazard management systems would
likely struggle to respond to the increased number of fires. Their most important need is technological
innovation on two fronts: prediction and better firefighting, where AI can help to predict locations at risk,
and the best possible strategy to extinguish the fire.
• To address this global issue, the World Economic Forum's Artificial Intelligence and Machine
Learning Platform, in collaboration with the C4IR Network, Koç Holding and the Turkish Ministry of
Forestry (TMF), has launched a FireAId initiative to mitigate wildfire risks using AI systems.
• The Initiative includes:
o A dynamic wildfire risk map that ranks the likelihood of forest fires based on seasonal variables
o Resource allocation that is optimal based on many data sources and the wildfire risk map
o A first-response proposal using pre-optimized resources and maps in the event of a forest fire
• The FireAId initiative of using artificial intelligence to effectively manage wildfires had a successful pilot
run since it was launched in January 2022 by the World Economic Forum (WEF). Hence statements 1
and 2 are correct.

19 www.visionias.in ©Vision IAS

FREE BY KING R QUEEN P [ऋषभ राजपूत]


Q 46.C
• In the scheme of parliamentary system of government provided by the Constitution, the governor is the
nominal executive authority (de jure executive) and the Chief Minister is the real executive authority
(de facto executive). In other words, the governor is the head of the state while the Chief Minister is the
head of the government.
• The Chief Minister performs many functions. Some of them are listed below:
o He is the chairman of the State Planning Board. Hence, statement 3 is correct.
o He acts as a vice-chairman of the concerned zonal council by rotation, holding office for a period of
one year at a time.
o He is a member of the Inter-State Council and the Governing Council of NITI Aayog, both
headed by the prime minister. Hence, statement 2 is not correct.
o He is the chief spokesman of the state government.
o He is the crisis manager-in-chief at the political level during emergencies.
o As a leader of the state, he meets various sections of the people and receives memoranda from them
regarding their problems, and so on.
o He is the political head of the services.
o He advises the governor with regard to the appointment of important officials like advocate
general, chairman and members of the state public service commission, state election commissioner,
and so on.
• The Constitution (Article 76) has provided for the office of the Attorney General for India. He is the
highest law officer in the country. In addition to the AG, there are other law officers of the
Government of India. They are the solicitor general of India and additional solicitor general of
India.
o They assist the AG in the fulfilment of his official responsibilities. It should be noted here that only
the office of the AG is created by the Constitution.
o In other words, Article 76 does not mention about the solicitor general and additional solicitor
general. Hence, statement 1 is not correct.
• Recently, The NITI Aayog, the think-tank at the Central level, will help each state to set up similar
bodies, replacing their outdated state planning boards, for faster and inclusive economic growth, in
tandem with the national vision of becoming a developed nation by 2047.
o The move is in recognition of the fact that except for sectors like defence, railways and highways, the
national gross domestic product (GDP) growth is an aggregation of states’ rates of growth.
o The Niti Aayog and the proposed SITs will play a critical role in India achieving goals set for 2047,
the 100th year of independence.
o Four states — Karnataka, Uttar Pradesh, Madhya Pradesh and Assam — have already begun work in
this regard while Maharashtra, Odisha, Andhra Pradesh and Gujarat will likely commence work soon.

Q 47.C
• Though constitutional status was given to Panchayati Raj institutions by the 73rd amendment, the idea of
Panchayati Raj was discussed and considered important since the independence and constitutional
assembly debated the issue. Beginning with Balwantrai Mehta Committee, many committees were formed
to give shape to local governance in India.
• In December 1977, the Janata Government appointed a committee on Panchayati raj Institutions
under the chairmanship of Ashok Mehta. It submitted its report in August 1978 and made 132
recommendations to revive and strengthen the declining Panchayati raj system in the country. Due to the
collapse of the Janata Government before the completion of its term, no action could be taken on the
recommendations of the Ashok Mehta Committee at the central level. However, the three states of
Karnataka, West Bengal and Andhra Pradesh took steps to revitalize the Panchayati raj, keeping in view
some of the recommendations of the Ashok Mehta Committee.
• The Committee to review the existing Administrative Arrangements for Rural Development and
Poverty Alleviation Programmes under the chairmanship of G.V.K. Rao was appointed by the
Planning Commission in 1985. The Committee came to the conclusion that the developmental process
was gradually bureaucratised and divorced from the Panchayati Raj. This phenomenon of
bureaucratization of development administration as against the democratization weakened the Panchayati
Raj institutions resulting in what is aptly called as ‘grass without roots’.
• In 1986, the Rajiv Gandhi government appointed a committee to prepare a concept paper on
‘Revitalisation of Panchayati Raj Institutions for Democracy and Development’ under the
chairmanship of L.M. Singhvi. It recommended that the Panchayati Raj institutions should be given
constitutional status.
20 www.visionias.in ©Vision IAS

FREE BY KING R QUEEN P [ऋषभ राजपूत]


• In 1988, a sub-committee of the Consultative Committee of Parliament was constituted under the
chairmanship of P.K. Thungon to examine the political and administrative structure in the district for
the purpose of district planning. This committee suggested the strengthening of the Panchayati Raj
system.
• The Committee on Policy and Programmes was constituted in 1988 by the Congress party under
the chairmanship of V.N. Gadgil. This committee was asked to consider the question of “how best
Panchayati Raj institutions could be made effective”. Hence, option (c) is the correct answer.

Q 48.D
• The superintendence, direction and control of the preparation of electoral rolls and the conduct of
all elections to the panchayats shall be vested in the state election commission. It consists of a state
election commissioner to be appointed by the governor. His conditions of service and tenure of office
shall also be determined by the governor. He shall not be removed from the office except in the manner
and on the grounds prescribed for the removal of a judge of the state high court. His conditions of service
shall not be varied to his disadvantage after his appointment.
• The state legislature may make provisions with respect to all matters relating to elections to the
panchayats.
• The State government is required to appoint a State Election Commissioner who would be responsible for
conducting elections to the Panchayati Raj institutions. Earlier, this task was performed by the State
administration which was under the control of the State government. Now, the office of the State Election
Commissioner is autonomous like the Election Commissioner of India. However, the State Election
Commissioner is an independent officer and is not linked to nor is this officer under the control of
the Election Commission of India. Hence option (d) is the correct answer.

Q 49.C
• The 73rd Amendment Act provides for a five-year term of office to the panchayat at every level.
However, it can be dissolved before the completion of its term. Further, fresh elections to constitute a
panchayat shall be completed
o Before the expiry of its duration of five years; or
o In case of dissolution, before the expiry of a period of six months from the date of its dissolution.
Hence, statement 1 is correct.
• But, where the remainder of the period (for which the dissolved panchayat would have continued) is
less than six months, it shall not be necessary to hold any election for constituting the new
panchayat for such a period. Hence, statement 2 is not correct.
• Moreover, a panchayat constituted upon the dissolution of a panchayat before the expiration of its
duration shall continue only for the remainder of the period for which the dissolved panchayat would have
continued had it not been so dissolved. In other words, a panchayat reconstituted after premature
dissolution does not enjoy the full period of five years but remains in office only for the remainder
of the period. Hence, statement 3 is correct.

Q 50.A
• Under the Constitution, the Parliament of India consists of three parts viz, the President, the Council of
States, and the House of the People.
• The President of India is not a member of either House of Parliament and does not sit in the Parliament to
attend its meetings, he is an integral part of the Parliament. This is because a bill passed by both Houses of
Parliament cannot become law without the President’s assent. Hence, A and R both are true, and R is
the correct explanation for A.
• He also performs certain functions relating to the proceedings of the Parliament, for example, he
summons and prorogues both Houses, dissolves the Lok Sabha, addresses both Houses, issues ordinances
when they are not in session, and so on.

Q 51.B
• India and USA announced the initiative on Critical and Emerging Technology (iCET) in May 2022
to elevate and expand our strategic technology partnership and defense industrial cooperation between the
governments, businesses, and academic institutions of our two countries. Hence statement 1 is not
correct.
• Under iCET, the both countries have identified six areas of cooperation which would include co-
development and co-production. The six areas for cooperation are scientific research and development;

21 www.visionias.in ©Vision IAS

FREE BY KING R QUEEN P [ऋषभ राजपूत]


quantum and artificial intelligence, defense innovation, space, advanced telecom which would include
things like 6G and semiconductors.
• iCET initiative is led by the Indian National Security Council Secretariat and the US National Security
Council. Hence statement 2 is correct.
• It would forge closer linkages between government, academia and industry of the two countries. The
launch of the ambitious iCET dialogue is seen as “an alignment of strategic, commercial and scientific
approaches” in the field of technology.

Q 52.D
• Articles 233 to 237 in Part VI of the Constitution make the following provisions to regulate the
organization of subordinate courts and to ensure their independence from the executive.
• The expression ‘district judge’ includes the judge of a
o city civil court,
o additional district judge,
o joint district judge,
o assistant district judge,
o the chief judge of a small cause court,
o chief presidency magistrate,
o additional chief presidency magistrate,
o sessions judge,
o additional sessions judge and assistant sessions judge.
• Hence, option (d) is the correct answer.

Q 53.D
Jurisdiction of Gram Nyayalaya:
• The Gram Nyayalaya is a mobile court and exercises the powers of both Criminal and Civil Courts.
• The seat of the Gram Nyayalaya is located at the headquarters of the intermediate Panchayat, but they go
to villages, work there and dispose of the cases.
• The Gram Nyayalayas try criminal cases (where the alleged offence attracts a punishment of not
more than 2 years or when the value of the property involved in a criminal case is not more than 20000
rupees), civil suits(cases over the cattle trespassing act, minimum wages act, protection of women from
domestic violence act and property disputes, etc.), claims or disputes. Hence statement 1 is not correct.
• Gram Nyayalayas can follow special procedures in civil matters, in a manner it deems just and reasonable
in the interest of Justice. They, in the first instance, allow for conciliation of the dispute and settlement of
the same.
• The Gram Nyayalayas are not strictly bound by the rules of evidence provided in the Indian
Evidence Act, 1872 but are guided by the principles of natural justice and subject to any rule made by the
High Court. Hence statement 2 is not correct.

Q 54.D
• The Constitution lays down the following conditions for the the governor’s office:
o He should not be a member of either House of Parliament or a House of the state legislature. If any
such person is appointed as governor, he is deemed to have vacated his seat in that House on the date
on which he enters upon his office as the governor.
o He should not hold any other office of profit. He is entitled without payment of rent to the use of his
official residence (the Raj Bhavan).
o He is entitled to such emoluments, allowances and privileges as may be determined by
Parliament. Hence, statement 1 is not correct.
o When the same person is appointed as the governor of two or more states, the emoluments and
allowances payable to him are shared by the states in such proportion as determined by the
president. Hence, statement 2 is not correct.
o His emoluments and allowances cannot be diminished during his term of office.
Q 55.D
• After every census, a readjustment is to be made in
o the allocation of seats in the Lok Sabha to the states, and
o the division of each state into territorial constituencies.
• Parliament is empowered to determine the authority and the manner in which it is to be made. Hence
statement 1 is not correct.
22 www.visionias.in ©Vision IAS

FREE BY KING R QUEEN P [ऋषभ राजपूत]


• Accordingly, the Parliament enacted the Delimitation Commission Acts in 1952, 1962, 1972, and 2002 for
this purpose.
• The 42nd Amendment Act of 1976 froze the allocation of seats in the Lok Sabha to the states and the
division of each state into territorial constituencies till the year 2000 at the 1971 level. This ban on
readjustment was extended for another 25 years (ie, up to the year 2026) by the 84th Amendment Act of
2001, with the same objective of encouraging population limiting measures.
• The 84th Amendment Act of 2001 also empowered the government to undertake readjustment and
rationalization of territorial constituencies in the states on the basis of the population figures of the 1991
census.
• Later, the 87th Amendment Act of 2003 provided for the delimitation of constituencies on the basis of
the 2001 census and not the 1991 census. However, this can be done without altering the number of seats
allotted to each state in the Lok Sabha. Hence, statement 2 is not correct.

Q 56.B
• As the Constitution of India provides for a parliamentary system of government in the states on the Union
pattern, the council of ministers headed by the chief minister is the real executive authority in the
politico-administrative system of a state. The council of ministers in the states is constituted and
function in the same way as the council of ministers at the Centre.
• Article 163–Council of Ministers to aid and advise Governor:
o There shall be a Council of Ministers with the Chief Minister as the head to aid and advise the
Governor in the exercise of his functions, except in so far as he is required to exercise his functions in
his discretion.
o If any question arises whether a matter falls within the Governor’s discretion or not, decision of
the Governor shall be final, and the validity of anything done by the Governor shall not be called in
question on the ground that he ought or ought not to have acted in his discretion. Hence, statement 2
is correct.
o The advice tendered by Ministers to the Governor shall not be inquired into in any court.
• Article 177–Rights of Ministers as Respects the Houses
o Every minister shall have the right to speak and take part in the proceedings of the Assembly (and
also the Council where it exists) and any Committee of the State Legislature of which he may be
named a member. But he shall not be entitled to vote. Hence, statement 1 is not correct.

Q 57.D
• Twenty-nine subjects, which were earlier in the State list of subjects, are identified and listed in the
Eleventh Schedule of the Constitution. These subjects are to be transferred to the Panchayati Raj
institutions. These subjects were mostly linked to development and welfare functions at the local level.
The actual transfer of these functions depends upon the State legislation. Each State decides how many of
these twenty-nine subjects would be transferred to the local bodies.
• Under the Eleventh Schedule of the constitution, there are 29 functional items placed within the purview
of panchayats, Some of the important functions are mentioned below :
o Agriculture, including agricultural extension
o Land improvement, implementation of land reforms, land consolidation and soil conservation
o Minor irrigation, water management and watershed development
o Fisheries
o Social forestry and farm forestry
o Minor forest produce
o Small-scale industries, including food processing industries
o Khadi, village and cottage industries
o Rural housing
o Rural electrification, including the distribution of electricity
o Education, including primary and secondary schools
o Technical training and vocational education
o Health and sanitation including hospitals, primary health centers and dispensaries
o Women and child development.
• Hence, option (d) is the correct answer.

23 www.visionias.in ©Vision IAS

FREE BY KING R QUEEN P [ऋषभ राजपूत]


Q 58.D
• Various committees and commissions have examined our electoral system, election machinery as well as
election process and suggested reforms to improve the processes and system.
• The Vohra Committee Report on Criminalisation of Politics was constituted to identify the extent of
the politician-criminal nexus and recommend ways in which the menace can be combated. Chapter 4 of
the report of the National Commission to Review the Working of the Constitution cites the Vohra report
as follows: “The nexus between the criminal gangs, police, bureaucracy and politicians has come out
clearly in various parts of the country” and that “some political leaders become the leaders of these
gangs/armed senas and over the years get themselves elected to local bodies, State assemblies, and the
national parliament.
• The Indrajit Gupta Committee on State Funding of Elections, 1998, backed the idea of state funding
of elections on principle, stating that “The Committee sees full justification constitutional, legal as well
as on the ground of public interest, for grant of State subvention to political parties, so as to establish
such conditions where even the parties with modest financial resources may be able to compete with
those who have superior financial resources.” Hence, option (d) is the correct answer.
• Tankha Committee (Core Committee) was appointed in 2010 to look into the whole gamut of
election laws and electoral reforms.
• Justice J.S. Verma Committee was constituted to recommend amendments to the Criminal Law so
as to provide for quicker trial and enhanced punishment for criminals accused of committing sexual
assault against women. The Committee submitted its report on January 23, 2013.

Q 59.D
• Recently it was reported that the Indian Space Research Organisation (ISRO) is planning to launch the
Aditya-L1 mission by June or July this year.
• Aditya-L1 is the first observatory-class space-based solar mission from India. The spacecraft will be
placed in a halo orbit around the first Lagrange point, L1, of the Sun-Earth system. A satellite
around the L1 point has the major advantage of continuously viewing the Sun without
occultation/eclipses. Hence statement 1 and statement 2 are correct.
• This position provides a greater advantage of observing solar activities continuously. Aditya-L1 carries
seven payloads to observe the photosphere, chromosphere, and the outermost layers of the Sun (the
corona) using electromagnetic and particle detectors. Hence statement 3 is correct.
• Four payloads directly view the Sun from the unique vantage point of L1, and the remaining three
payloads carry out in-situ studies of particles and fields at the Lagrange point L1.
• Visible Emission Line Coronagraph (VELC) is the primary payload in this mission.

24 www.visionias.in ©Vision IAS

FREE BY KING R QUEEN P [ऋषभ राजपूत]


Q 60.B
• In 2018, the central government notified the Electoral Bond Scheme. This scheme was announced in the
2017 budget. It is touted as an alternative to cash donations made to political parties. It is aimed at
bringing clean money and substantial transparency into the system of political funding. The salient
features of the scheme are:
o The electoral bond means a bond issued in the nature of a promissory note which is a bearer banking
instrument and does not carry the name of the buyer or payee.
o The electoral bonds may be purchased by a citizen of India or entities incorporated or established in
India.
o The electoral bonds can be used for making donations to only those registered political parties
which have secured not less than one percent of the votes polled in the last general election to
the Lok Sabha or the State Legislative Assembly. Hence, option (b) is the correct answer.
o The electoral bonds can be encashed by an eligible political party only through a bank account with
the authorized bank.
o The electoral bonds are issued in the denomination of ₹1,000, ₹10,000, ₹1,00,000, ₹10,00,000 and
₹1,00,00,000.
o The information furnished by the buyer is treated as confidential by the authorized bank and is not to
be disclosed to any authority for any purposes, except when demanded by a competent court or upon
registration of a criminal case by any law enforcement agency.

Q 61.B
• The Kawal Tiger Reserve is located in the North Eastern part (Old Adilabad district) of Telangana
State along the banks of river Godavari, forming part of the Deccan peninsula-central highlands. The
reserve is nestled in the Sahyadri Mountain Ranges, and has diverse habitat comprising of dense
forests, grasslands, open areas, rivers, streams and water bodies.
• Geographically the reserve is situated in the southern-most tip of the Central Indian Tiger
Landscape, having linkages with the Tadoba-Andhari (Maharashtra) and Indravati (Chhattisgarh) tiger
reserves. Thus, the habitat has tremendous significance for tiger conservation in the region. It is also a
major catchment of river Godavari and local rivulets like Peddavagu and Kadam.
• Zoo-geographically, the reserve comes under the Indo-Malayan region, and the major wild animals
include: nilgai, chousinga, chinkara, black buck, sambar spotted deer, wild dog, wolf, jackal, fox,
tiger, leopard and the jungle cat.
• The reserve has a low tiger density at present but has a tremendous potential as a source area with stepped
up protection and habitat amelioration under Project Tiger.
• Hence option (b) is the correct answer.

Q 62.A
The salient features of the Family Courts Act, of 1984 are as follows:
• It provides for the establishment of Family Courts by the State Governments in consultation with
the High Courts. Hence statement 1 is correct.
• It makes it obligatory on the State Governments to set up a Family Court in every city or town with a
population exceeding one million.
• It enables the State Governments to set up Family Courts in other areas also if they deem it
necessary.
• It makes it obligatory on the part of the Family Court to endeavour, in the first instance to effect a
reconciliation or a settlement between the parties to a family dispute. During this stage, the proceedings
will be informal and rigid rules of procedure shall not apply.
• It provides for the association of social welfare agencies, counsellors, etc., during the conciliation stage
and also secures the service of medical and welfare experts.
• It provides that the parties to a dispute before a Family Court shall not be entitled, as of right, to be
represented by a legal practitioner. However, the court may, in the interest of justice, seek the assistance
of a legal expert as amicus curiae.
• It simplifies the rules of evidence and procedure so as to enable a Family Court to deal effectively with a
dispute.
• It provides for only one right of appeal which shall lie to the High Court. Hence statement 2 is not
correct.

25 www.visionias.in ©Vision IAS

FREE BY KING R QUEEN P [ऋषभ राजपूत]


Q 63.D
• Ottanthullal (or Thullal, in short) is a recite-and-dance art-form of Kerala that is famous for its
humour and social satire, and marked by its simplicity as opposed to more complex dance-forms
like Kathakali and Koodiyattam. Hence statement 1 is not correct.
• Ottan thullal is the combination of storytelling along with singing and dancing. It is an example of a
famous proverb, “Old is Gold,” as this performing art is almost 300 years old, created by poet Kunchan
Nambiar in the 18th century. The poet Kunchan Nambiar performed this art instead of another similar art
called “Chakyar Koothu” as a protest against society’s social, economic, and prejudicated behaviour. It is
also a colourful representation of social criticism through humour. The facial expression of the artist and
involvement of dance is the main attraction for the viewers.
• Ottan Thullal, paryan thullal, and seethagan thullal are the three forms of Thullals. Each Thulllal delivers
a different kind of performance. For example, Ottan Thullal’s presentation involves more body movement
and a different way of singing than the other two. Also, its costume is considered to be the most attractive
one. Mostly, the performances take place outside the temple during religious event. Hence statement 2 is
not correct.

Q 64.D
• A notified area committee is created for the administration of two types of areas–a fast developing
town due to industrialisation, and a town which does not yet fulfil all the conditions necessary for
the constitution of a municipality, but which otherwise is considered important by the state
government. Since it is established by notification in the government gazette, it is called as notified
area committee. Hence, statement 3 is correct.
• Though it functions within the framework of the State Municipal Act, only those provisions of the act
apply to it which are notified in the government gazette by which it is created. It may also be entrusted to
exercise powers under any other act. Its powers are almost equivalent to those of a municipality.
• But unlike the municipality, it is an entirely nominated body, that is, all the members of a notified
area committee including the chairman are nominated by the state government. Thus, it is neither
an elected body nor a statutory body. Hence, statement 1 is not correct and statement 2 is correct.

Q 65.B
• A High-Level Committee, under the Chairmanship of Justice B. N. Srikrishna, to review the
institutionalization of the arbitration mechanism recommended:
• Setting up an Autonomous Body, styled the Arbitration Promotion Council of India (APCI), having
representatives from all stakeholders for grading arbitral institutions in India.
• The APCI may recognize professional institutes providing for the accreditation of arbitrators.
• The APCI may also hold training workshops and interact with law firms and law schools to train
advocates with an interest in arbitration and with a goal to create a specialist arbitration bar comprising
advocates dedicated to the field.
• Creation of a specialist Arbitration Bench to deal with such Commercial disputes, in the domain of the
Courts.
• Changes have been suggested to make arbitration speedier and more efficacious and incorporate
international best practices.
• The Committee is also of the opinion that the National Litigation Policy (NLP) must promote
arbitration in Government Contracts.
• The committee recommended declaring the International Centre for Alternative Dispute Resolution
(ICADR) working under the aegis of the Ministry of Law and Justice, Department of Legal Affairs
as an Institution of national importance and takeover of the institution by a statute as revamped ICADR
has the potential be a globally competitive institution.
• Creation of the post of an ‘International Law Adviser’ (ILA). It shall advise the Government and
coordinate dispute resolution strategy for the Government in disputes arising out of its international law
obligations particularly disputes arising out of BITs.
• Hence, option (b) is the correct answer.

Q 66.A
• As part of the holistic development of the Great Nicobar Islands, the Ministry of Ports, Shipping, and
Waterways (MoPSW) has invited Expression of Interest for building an International Container
Transhipment Port at Galathea Bay.
• In 2021, the National Board for Wildlife (NBWL) has de-notified Galathea Bay Wildlife Sanctuary, an
important nesting site of leatherback sea turtles in the Andaman and Nicobar Islands.
26 www.visionias.in ©Vision IAS

FREE BY KING R QUEEN P [ऋषभ राजपूत]


• The International Union for Conservation of Nature lists leatherback turtles among the vulnerable species
as their numbers are decreasing. Extremely vulnerable to temperature extremes, the leatherback turtles are
the world’s largest turtles and the only species without scales and a hard shell. They are named after their
tough rubbery skin and have existed in their current form since the age of the dinosaurs. Leatherbacks are
migratory and some of them swim over 10,000 miles a year between nesting and foraging grounds.
• The National Board for Wildlife has suggested the implementation of a mitigation plan to facilitate the
nesting of turtles as well as connectivity between the Galathea river and the Bay of Bengal.
• Hence option (a) is the correct answer.

Q 67.A
• Article 74 deals with the status of the council of ministers while Article 75 deals with the
appointment, tenure, responsibility, qualification, oath and salaries and allowances of the ministers.
• Article 75 says
o The Prime Minister shall be appointed by the President and the other Ministers shall be appointed by
the President on the advice of the Prime Minister.
o The ministers shall hold office during the pleasure of the President.
o The council of ministers shall be collectively responsible to the Lok Sabha.
o The President shall administer the oaths of office and secrecy to a minister.
o The salaries and allowances of ministers shall be determined by the Parliament.
• Hence, option (a) is the correct answer.

Q 68.C
• The system of proportional representation aims at removing the defects of territorial representation. Under
this system, all sections of the people get representation in proportion to their number. Even the smallest
section of the population gets its due share of representation in the legislature.
• There are two kinds of proportional representation, namely, the single transferable vote system and the list
system. In India, the first kind is adopted for the election of members to the Rajya Sabha and state
legislative council and for electing the President and the Vice-President. Hence, option (c) is the
correct answer.
• Though some members of the Constituent Assembly had advocated the system of proportional
representation for the election of members to the Lok Sabha, the Constitution has not adopted the system
due to two reasons.
o Difficult for the voters to understand the system (which is complicated) due to the low literacy scale
in the country.
o Unsuitability to the parliamentary government due to the tendency of the system to multiply political
parties leading to instability in government.
• Additionally, the system of proportional representation has the following demerits:
o It is highly expensive.
o It does not give any scope for organizing by-elections.
o It eliminates intimate contact between voters and representatives.
o It promotes minority thinking and group interests.
o It increases the significance of party system and decreases that of voter.

Q 69.B
• The Constitution lays down the following qualifications for a person to be chosen as a member of the
Parliament:
o He must be a citizen of India.
o He must make and subscribe to an oath or affirmation before the person authorized by the election
commission for this purpose.
o He must be not less than 30 years of age in the case of the Rajya Sabha and not less than 25 years of
age in the case of the Lok Sabha.
o He must possess other qualifications prescribed by Parliament.
• The Parliament has laid down the following additional qualifications in the Representation of People
Act (1951).
o He must be registered as an elector for a parliamentary constituency. This is the same in the case of
both, the Rajya Sabha and the Lok Sabha. The requirement that a candidate contesting an election
to the Rajya Sabha from a particular state should be an elector in that particular state was
dispensed with in 2003. In 2006, the Supreme Court upheld the constitutional validity of this
change. Hence statement 1 is not correct.
27 www.visionias.in ©Vision IAS

FREE BY KING R QUEEN P [ऋषभ राजपूत]


o He must be a member of a scheduled caste or scheduled tribe in any state or union territory if he
wants to contest a seat reserved for them. However, a member of scheduled castes or scheduled
tribes can also contest a seat not reserved for them. Hence statement 2 is correct.

Q 70.C
• Original Jurisdiction
o As a federal court, the Supreme Court decides the disputes between different units of the Indian
Federation.
o More elaborately, any dispute:
▪ Between the Centre and one or more states; or
▪ Between the Centre and any state or states on one side and one or more other states on the
other side; or
▪ Between two or more states. Hence, option 2 is correct.
o In the above federal disputes, the Supreme Court has exclusive original jurisdiction. Exclusive
means, no other court can decide such disputes and original means, the power to hear such disputes in
the first instance, not by way of appeal.
o With regard to the exclusive original jurisdiction of the Supreme Court, two points should be noted.
▪ One, the dispute must involve a question (whether of law or fact) on which the existence or extent
of a legal right depends. Thus, questions of political nature are excluded from it.
▪ Two, any suit brought before the Supreme Court by a private citizen against the Centre or a state
cannot be entertained under this.
o Further, original jurisdiction of the Supreme Court does not extend to the following:
▪ A dispute arising out of any pre-Constitution treaty, agreement, covenant, engagement,
sanad or other similar instruments. Hence, option 3 is not correct.
▪ A dispute arising out of any treaty, agreement, etc., which specifically provides that the said
jurisdiction does not extend to such a dispute.
▪ Interstate water disputes. Hence option 1 is not correct.
▪ Matters referred to the Finance Commission.
▪ Adjustment of certain expenses and pensions between the Centre and the states.
▪ Ordinary dispute of Commercial nature between the Centre and the states.
▪ Recovery of damages by a state against the Centre. Hence, option 4 is not correct.
▪ In 1961, the first suit, under the original jurisdiction of the Supreme Court, was brought by West
Bengal against the Centre.
▪ The State Government challenged the Constitutional validity of the Coal Bearing Areas
(Acquisition and Development) Act, 1957, passed by the Parliament. However, the Supreme
Court dismissed the suit by upholding the validity of the Act.

Q 71.A
• Recently, the Union government conducted a successful test of 'BharOS', the recently-unveiled
indigenous mobile operating system developed by the Indian Institute of Technology, Madras (IIT).
Hence statement 1 is correct.
• BharOS is an Indian government-funded project to develop a free and open-source operating system (OS).
The project aims to reduce the dependence on foreign OS in smartphones and promote the use of locally
developed technology.
• BharOS is based on the Android Open Source Project (AOSP). This makes it somewhat similar to
Google’s Android operating system. iOS is Apple’s proprietary OS and BharOS has no similarity with it
per se. BharOS can run virtually on all Android apps.
• The big difference between Android and BharOS is that the latter does not come with any Google
services or apps. BharOS just does not have any pre-installed apps. This gives users the flexibility to
download any app of their choice. Hence statement 2 is not correct.
• The BharOS has been developed by JandK Operations Private Limited (JandKops), which has been
incubated by IIT Madras Pravartak Technologies Foundation, a Section 8 (Not for Profit) Company
established by IIT Madras. The Foundation is funded by the Department of Science and Technology
(DST), Government of India, under its National Mission on Interdisciplinary Cyber-Physical Systems
(NMICPS).

28 www.visionias.in ©Vision IAS

FREE BY KING R QUEEN P [ऋषभ राजपूत]


Q 72.A
• When the Lok Sabha is dissolved, all business including bills, motions, resolutions, notices, petitions and
so on pending before it or its committees lapse. They (to be pursued further) must be reintroduced in the
newly-constituted Lok Sabha. However, some pending bills and all pending assurances that are to be
examined by the Committee on Government Assurances do not lapse on the dissolution of the Lok Sabha.
• The position with respect to lapsing of bills is as follows:
o A bill pending in the Lok Sabha lapses (whether originating in the Lok Sabha or transmitted to it by
the Rajya Sabha).
o A bill passed by the Lok Sabha but pending in the Rajya Sabha lapses. Hence option 1 is correct.
o A bill not passed by the two Houses due to disagreement and if the president has notified the holding
of a joint sitting before the dissolution of Lok Sabha, does not lapse.
o A bill pending in the Rajya Sabha but not passed by the Lok Sabha does not lapse. Hence option 3 is
not correct.
o A bill passed by both Houses but pending assent of the President does not lapse. Hence option 2
is not correct.
o A bill passed by both Houses but returned by the president for reconsideration of Houses does not
lapse.
Q 73.C
• Basmati is a variety of long, slender-grained aromatic rice which is traditionally grown in India, Pakistan,
and Nepal. As of 2019, India accounted for 65% of the international trade in basmati rice, while Pakistan
accounted for the remaining 35%.
• Basmati rice is cultivated in the Himalayan foothills of the Indian subcontinent. The specific agro-
climatic conditions, processing techniques such as harvesting and ageing are said to make this rice
unique. In India, rice grown in specific parts of Punjab, Haryana, Himachal Pradesh, Delhi,
Uttarakhand, Uttar Pradesh and Jammu & Kashmir can be labelled as basmati. Hence statement 1
is correct.
• Basmati rice is just one of the thousands of rice varieties available in India. However, this fragrant rice has
invited the most controversy.
• In 2020, India’s application for a geographical indication tag recognised in the European Union market
was put on hold after Pakistan opposed the move.
• In a bid to promote the business around basmati rice, the Food Safety and Standards Authority of
India (FSSAI) notified standards for basmati rice January 12, 2023. They will be enforced from
August 1, 2023. Hence statement 2 is correct.
• FSSAI hopes that the standards would protect consumer interest and ensure the basmati rice sold in the
market has the characteristic fragrance identified with this variety and is free from artificial fragrances and
colouring.
• The authority has also set standards on parameters such as average size of grains and their elongation ratio
after cooking. It has set the maximum limits for moisture, amylose content, uric acid, damaged grains and
presence of non-basmati rice.
• The standards are applicable to brown basmati rice, milled basmati rice, parboiled brown basmati rice and
milled parboiled basmati rice.

Q 74.D
• A cantonment board is established for municipal administration for the civilian population in the
cantonment area. A cantonment area is a delimited area where the military forces and troops are
permanently stationed. It is set up under the provisions of the Cantonments Act of 2006–legislation
enacted by the Central government. It works under the administrative control of the defense ministry of
the Central government. Thus, unlike the above four types of urban local bodies, which are created and
administered by the state government, a cantonment board is created as well as administered by the
Central government.
• A cantonment board consists of partly elected and partly nominated members. The elected members hold
office for a term of five years while the nominated members (i.e., ex-officio members) continue so long as
they hold the office in that station. The military officer commanding the station is the ex-officio
president of the board and presides over its meetings. The vice-president of the board is elected by
the elected members from amongst themselves for a term of five years. Hence, option (d) is the
correct answer.
• The functions performed by a cantonment board are similar to those of a municipality. These are
statutorily categorized into obligatory functions and discretionary functions. The sources of income
include both, tax revenue and non-tax revenue.
29 www.visionias.in ©Vision IAS

FREE BY KING R QUEEN P [ऋषभ राजपूत]


• The executive officer of the cantonment board is appointed by the President of India. He implements
all the resolutions and decisions of the board and its committees. He belongs to the central cadre
established for the purpose.

Q 75.C
• In the scheme of parliamentary system of government provided by the constitution, the President is the
nominal executive authority (de jure executive) and Prime Minister is the real executive authority (de
facto executive). In other words, President is the head of the State while Prime Minister is the head of
the government.
• The salary and allowances of the Prime Minister are determined by the Parliament from time to
time. He gets the salary and allowances that are payable to a member of Parliament. Additionally, he gets
a sumptuary allowance, free accommodation, travelling allowance, medical facilities, etc. Hence,
statement 2 is not correct.
• The term of the Prime Minister is not fixed and he holds office during the pleasure of the president.
However, this does not mean that the president can dismiss the Prime Minister at any time. So long
as the Prime Minister enjoys the majority support in the Lok Sabha, he cannot be dismissed by the
President. However, if he loses the confidence of the Lok Sabha, he must resign or the President can
dismiss him. Hence, statement 1 is correct.
• Since the Prime Minister stands at the head of the council of ministers, the other ministers cannot function
when the Prime Minister resigns or dies. In other words, the resignation or death of an incumbent
Prime Minister automatically dissolves the council of ministers and thereby generates a vacuum.
The resignation or death of any other minister, on the other hand, merely creates a vacancy which
the Prime Minister may or may not like to fill. Hence, statement 3 is correct.

Q 76.A
• The ongoing bird flu outbreak in the US is now the longest and deadliest on record. It is caused by highly
pathogenic avian influenza (HPAI) A(H5N1).
• Highly Pathogenic Avian Influenza (HPAI) A(H5N1) virus occurs mainly in birds and is highly
contagious among them. H5N1 can infect number of mammals including humans. H5N1 has a
frighteningly high mortality rate.
• For any pathogen to have the potential to cause a human pandemic, it has to have three important
qualities. It must spread easily among humans, particularly through the air. It must cause human disease.
And it must be something that most of our immune systems haven’t encountered before — that is, it must
be novel.
• H5N1 doesn’t meet all of these criteria. H5N1 doesn’t easily bind to cells in airways of humans. Flu
viruses that can’t cause infection in humans’ airways are much harder to transmit among humans — and
therefore, they can’t cause a pandemic. Unless something changes.
• If there’s anything concerning about the current bird flu, it’s that it could mutate and evolve. Influenza
viruses are incredibly changeable. Like other viruses, H5N1 picks up small mutations as it replicates
within a host; over time, that can give the virus certain benefits. But influenza viruses can also undergo
much bigger and more consequential shifts through a process called reassortment.
• Reassortment is the formation of a hybrid virus containing parts from the genomes of two distinct
viruses in a mixed infection. In simple terms, Reassortment is like something out of science fiction:
When two influenza viruses infect the same cell in the same host, they can trade entire chunks of their
genomes with each other, yielding a variety of Franken-flus. Hence option (a) is the correct answer.
• It’s a red flag for researchers when avian flu spreads among animals that can also easily get sick with
other kinds of influenza. Pigs, for example, have flu receptors in their respiratory systems that both human
and bird viruses easily bind to, so they can get infected with both. Should these two viruses meet inside
these animals, they might swap parts, producing an avian flu that can more easily infect mammals.
Q 77.C
• Before entering upon his office, the governor has to make and subscribe to an oath or affirmation. In his
oath, the governor swears
o to faithfully execute the office;
o to preserve, protect and defend the Constitution and the law; and
o to devote himself to the service and well-being of the people of the state.
• The oath of office to the governor is administered by the chief justice of the concerned state high
court and in his absence, the senior-most judge of that court available. Every person discharging the
functions of the governor also undertakes the similar oath or affirmation.
• Hence, option (c) is the correct answer.
30 www.visionias.in ©Vision IAS

FREE BY KING R QUEEN P [ऋषभ राजपूत]


Q 78.A
• During the election campaign, the political parties and contesting candidates are expected to abide by a
Model Code of Conduct evolved by the Election Commission on the basis of a consensus among political
parties. It is intended to maintain the election campaign on healthy lines, avoid clashes and conflicts
between political parties or their supporters and to ensure peace and order during the campaign period and
thereafter, until the results are declared.
• The Model Code of Conduct for the guidance of political parties and candidates is a set of norms that has
been evolved with the consensus of political parties who have consented to abide by the principles
embodied in the said code and also binds them to respect and observe it in its letter and spirit. The model
code also prescribes guidelines for the ruling party either at the Centre or in the state to ensure that a level
field is maintained and that no cause is given for any complaint that the ruling party has used its official
position for the purposes of its election campaign
• The Commission normally announces the schedule of elections in a major press conference a few weeks
before the formal process is set in motion. The Model Code of Conduct for the guidance of candidates
and political parties comes immediately into effect after such an announcement.
• Hence, option (a) is the correct answer.

Q 79.C
• National Lok Adalat: National Level Lok Adalats are held at regular intervals on a single day
throughout the nation, in every one of the courts, from the Supreme Court to the Taluk Levels, wherein
the cases are disposed of in huge numbers.
• They are held every two months across the country to dispose of the pending cases. According to the
statistics of the Ministry of Law, more than 50 lakh cases are disposed of annually on average by these
courts.
• Permanent Lok Adalat: It was established according to Section 22 B of the Legal Service Authorities
Act, 1987. These are permanent bodies with a Chairman and two members giving an obligatory pre-
litigation system for conciliation and settlement of cases pertaining to public utility services. Hence
statement 1 is correct.
• In these courts, even if there is a failure in reaching a settlement, the Permanent Lok Adalat has the
jurisdiction to decide the matter, provided, the dispute does not relate to any offense.
• The award given by the Permanent Lok Adalat is the last and official for every one of the parties.
Hence statement 2 is correct.
• Portable Lok Adalats: These are mobile dispute settlement bodies and are set up in different parts of the
country to resolve matters by encouraging the resolution of disputes and easing the burden on the formal
judiciary.

Q 80.A
• In addition to the members of a House, every minister and the attorney general of India have the right
to speak and take part in the proceedings of either House, any joint sitting of both the Houses, and
any committee of Parliament of which he is a member, without being entitled to vote. Hence,
statement 1 is correct.
• There are two reasons underlying this constitutional provision:
o A minister can participate in the proceedings of a House, of which he is not a member of. In other
words, a minister belonging to the Lok Sabha can participate in the proceedings of the Rajya Sabha
and vice-versa.
o A minister, who is not a member of either House, can participate in the proceedings of both
Houses. It should be noted here that a person can remain a minister for six months, without being a
member of either House of Parliament. Hence, statement 2 is not correct.
Q 81.C
• Recently, the Centre established three Grievance Appellate Committees based on the recently amended
Information Technology (Intermediary Guidelines and Digital Media Ethics Code) Rules, 2021 (“IT Rules
2021”).
• The IT rules 2021 provide for creating avenues for grievance redressal apart from Courts and ensure that
the Constitutional rights of Indian citizens are not contravened by any Big-tech Platform.
• The Grievance Appellate Committees (GACs) set up by the government to look into users'
complaints against large social media companies such as Facebook, Twitter and Instagram will
commence operation from March 1, 2023. Hence statement 1 is correct.
• As per the notification, three Grievance Appellate Committees have been constituted with three members
each.
31 www.visionias.in ©Vision IAS

FREE BY KING R QUEEN P [ऋषभ राजपूत]


• Grievance Appellate Committee (GAC) is a critical piece of overall policy and legal framework to ensure
that Internet in India is Open, Safe & Trusted and Accountable. The need for GAC was created due to
large numbers of grievances being left unaddressed or unsatisfactorily addressed by Internet
Intermediaries. GAC is expected to create a culture of responsiveness amongst all Internet Platforms and
Intermediaries towards their consumers.
• The GAC will be a virtual Digital platform that will operate only online and digitally - wherein the
entire appeal process, from filing of appeal to the decision thereof, shall be conducted digitally. Hence
statement 2 is correct.
Q 82.B
• The China-U.S. trade war and the COVID-19 pandemic laid bare the need for companies to diversify
supply chains outside of China. This has given rise to the “China plus one” strategy, in which
multinational firms are moving to other countries, in addition to China. Some Asian countries have
put forward plans to attract overseas investment as companies look for another center of production or
distribution. These include Thailand, Malaysia, and Vietnam, which have introduced preferential policies
for overseas firms investing in the country. Hence option (b) is the correct answer.
• For now, the “China plus one” strategy is working for some companies. One key determinant of how
sustainable the trend will be is how quickly infrastructure can be built to accommodate more firms
moving into other nations. However, companies that do choose to diversify into Southeast Asian nations
will continue to be able to take advantage of the varied and easily accessible suppliers from other Asian
nations. In addition, the recently signed Regional Comprehensive Economic Partnership (RCEP) will
allow firms with supply chains distributed among several Asian nations to take advantage of common
rules of origin for the entire bloc. This will allow RCEP countries to use only a single certificate of origin.
• As a result, the “China plus one” strategy, with firms venturing into other Asian nations, has become a
popular trend that is likely to continue over the long run, even if some firms focus production more on
China in the short run. In particular, Thailand, Malaysia, and Vietnam will continue to appeal to
multinational firms, especially as these countries continue to build up infrastructure and production
capacity.

Q 83.D
• Appointment of District Judges (Art.233)
• The appointment, posting, and promotion of district judges in a state are made by the Governor of State,
in consultation with the High Court. The qualifications of a person for appointment to the post are:1. He
should not already be in the service of the central or state government.2. He should have been an advocate
or pleader for not less than seven years3. He should be recommended by the high court for appointment.
• From the above, it is clear that the Constitution has not prescribed a minimum age for appointment as
a district judge.
• Hence, option (d) is the correct answer.

Q 84.A
• Immune Imprinting: It is a tendency of the body to repeat its immune response based on the first variant
it encountered through infection or vaccination. Imprinting acts as a database for the immune system,
helping it put up a better response in order to repeat infections. The concept was first observed in
1947. Hence statement 1 is correct.
• Issues with Immune Imprinting
o After our body is exposed to a virus for the first time, it produces memory B cells which circulate in
the bloodstream and quickly produce antibodies. When a similar or variant of virus enters the body,
the immune system, rather than generating new B cells, activates memory B cells, which in turn
produce antibodies which bind to features found in both the old and new strains, known as cross-
reactive antibodies. Although the cross-reactive antibodies do offer some protection against the new
strain, they aren’t as effective as the ones produced by the B cells when the body first came across the
original virus. Hence statement 2 is not correct.
• Adaptive Immune System
o Adaptive immune responses are carried out by white blood cells also called lymphocytes. There are
two broad classes of such responses: antibody responses and cell-mediated immune responses, and
they are carried out by certain different classes of lymphocytes, called B cells and T cells, respectively
o B cells mature in the bone marrow (therefore the name “B cell”).
o Cells which eventually become T cells travel from the bone marrow to the thymus by way of our
bloodstream where they mature (hence the name “T cell”).The thymus is present just above the heart
behind the sternum, or breastbone.
32 www.visionias.in ©Vision IAS

FREE BY KING R QUEEN P [ऋषभ राजपूत]


Q 85.C
• Money bills can be introduced in the state legislature only with the Governor's prior
recommendation.
• Every money bill, after it is passed by the state legislature (unicameral or bicameral), is presented to the
governor for his assent. He has three alternatives:
o He may give his assent to the bill, the bill then becomes an act.
o He may withhold his assent to the bill, and the bill then ends and does not become an act.
o He may reserve the bill for the consideration of the president.
• Thus, the governor cannot return a money bill for reconsideration by the state legislature. Normally,
the governor gives his assent to a money bill as it is introduced in the state legislature with his previous
permission. Hence, statement 2 is correct.
• When the governor reserves a money bill for the consideration of the President, he will not have any
further role in the enactment of the bill. If the President gives his assent to the bill, it becomes an Act.
This means that the assent of the governor is no longer required. Hence, statement 1 is correct.

Q 86.B
• The Zonal Councils are the statutory (and not the constitutional) bodies. They are established by the
States Reorganisation Act of 1956. The act divided the country into five zones (Northern, Central,
Eastern, Western, and Southern) and provided a zonal council for each zone. However, the North-
Eastern Council was created by a separate Act of Parliament—the North-Eastern Council Act of 1971.
The Union Home Minister is the common chairman of all the zonal councils. Hence option (b) is the
correct answer.
• The Broad Objectives of Zonal Councils are:
o The zonal councils aim at promoting cooperation and coordination between states, union territories,
and the Centre.
o They discuss and make recommendations regarding matters like economic and social planning,
linguistic minorities, border disputes, interstate transport, and so on.
• They are only deliberative and advisory bodies. As per Section 17(1) of the States Re-organisation Act,
each Zonal Council shall meet at such time as the Chairman of the Council may decide. Punchhi
Commission recommended that the Zonal Councils should meet at least twice a year with an agenda
proposed by states concerned to maximize coordination and promote harmonization of policies and
actions having inter-state ramifications.
Q 87.C
• In 2003, the facility to opt to vote through proxy was provided to the service voters belonging to the
Armed Forces and members belonging to a Force to which provisions of the Army Act apply. Such
service voters who opt to vote through proxy have to appoint a proxy in a prescribed format and intimate
the Returning Officer of the constituency. Hence, statement 1 is correct.
• In 1999, a provision was made for voting by certain classes of persons through postal ballot. Thus,
any class of persons can be notified by the Election Commission, in consultation with the
government, and the persons belonging to such notified class can give their votes by postal ballot, and
not in any other manner, at elections in their constituency or constituencies. Hence, statement 2 is
correct.

Q 88.C
• The Vice-President occupies the second highest office in the country. He is accorded a rank next to the
President in the official warrant of precedence. This office is modelled on the lines of the American
Vice-President.
• The Vice-President, like the president, is elected not directly by the people but by the method of indirect
election. He is elected by the members of an electoral college consisting of the members of both
Houses of Parliament. Thus, this electoral college is different from the electoral college for the election
of the President in the following two respects:
o It consists of both elected and nominated members of the Parliament (in the case of president, only
elected members).
o It does not include the members of the state legislative assemblies (in the case of President, the
elected members of the state legislative assemblies are included). Hence, statement 1 is correct.
• The Vice-President holds office for a term of five years from the date on which he enters upon his office.
However, he can resign from his office at any time by addressing the resignation letter to the President.
He can also be removed from the office before completion of his term. A formal impeachment is not
required for his removal. Hence, statement 2 is correct.
33 www.visionias.in ©Vision IAS

FREE BY KING R QUEEN P [ऋषभ राजपूत]


o He can be removed by a resolution passed by a majority of all the then members of the Rajya
Sabha and agreed to by the Lok Sabha. This means that this resolution should be passed in the
Rajya Sabha by an effective majority and in the Lok Sabha by a simple majority. It must be
noted here that the effective majority in India is only a type of special majority and not a separate one.
Further, this resolution can be introduced only in the Rajya Sabha and not in the Lok Sabha.

Q 89.D
• The Rajya Sabha (first constituted in 1952) is a continuing chamber, that is, it is a permanent body and not
subject to dissolution. However, one-third of its members retire every second year. Their seats are filled
up by fresh elections and presidential nominations at the beginning of every third year. The retiring
members are eligible for re-election and renomination any number of times.
• The Constitution has not fixed the term of office of members of the Rajya Sabha and left it to the
Parliament. Accordingly, the Parliament in the Representation of the People Act (1951) provided that the
term of office of a member of the Rajya Sabha shall be six years. The act also empowered the president of
India to curtail the term of members chosen in the first Rajya Sabha. Hence, statement 1 is not correct.
• In the first batch, it was decided by lottery as to who should retire. Further, the act also authorized the
President to make provisions to govern the order of retirement of the members of the Rajya Sabha.
• Unlike the Rajya Sabha, the Lok Sabha is not a continuing chamber. Its normal term is five years from the
date of its first meeting after the general elections, after which it automatically dissolves. The Speaker
cannot dissolve the Lok Sabha. However, the President is authorized to dissolve the Lok Sabha at
any time even before the completion of five years and this cannot be challenged in a court of law.
Hence, statement 2 is not correct.
• Further, the term of the Lok Sabha can be extended during a period of national emergency be a law of
Parliament for one year at a time for any length of time. However, this extension cannot continue beyond
a period of six months after the emergency has ceased to operate.

Q 90.C
• The Table of Precedence is related to the rank and order of the officials of the Union and State
Governments. But, the order in this Table is meant for state and ceremonial occassions and has no
application in the day-to-day business of Government. The updated version of the Table, containing all
the amendments made therein so far (2019), is given below:
o President
o Vice-President
o Prime Minister
o Governors of states within their respective states
o Former presidents
o 5A. Deputy Prime Minister
o Chief Justice of India; Speaker of Lok Sabha
o Cabinet Ministers of the Union, Chief Ministers of States within their respective States Vice-
Chairperson, NITI Aayog Former Prime Ministers Leaders of Opposition in Rajya Sabha and Lok
Sabha 7A. Holders of Bharat Ratna decoration
o Ambassadors Extraordinary and Plenipotentiary and High Commissioners of Commonwealth
countries accredited to India, Chief Ministers of States outside their respective States Governors of
States outside their respective States
o Judges of Supreme Court 9A. Chairperson, Union Public Service Commission, Chief Election
Commissioner, Comptroller & Auditor General of India
o Deputy Chairman, Rajya Sabha, Deputy Chief Ministers of States Deputy Speaker, Lok Sabha,
Members of the NITI Aayog
• Hence, option (c) is the correct answer.

Q 91.B
• Article 75 clearly states that the council of ministers is collectively responsible to the Lok
Sabha. This means that all the ministers own joint responsibility to the Lok Sabha for all their acts of
omission and commission. They work as a team and swim or sink together. When the Lok Sabha passes a
no-confidence motion against the council of ministers, all the ministers have to resign including those
ministers who are from the Rajya Sabha. Hence, statement 1 is not correct.
• The principle of collective responsibility also means that the Cabinet decisions bind all cabinet
ministers (and other ministers) even if they differed in the cabinet meeting. It is the duty of every
minister to stand by cabinet decisions and support them both within and outside the Parliament. If
34 www.visionias.in ©Vision IAS

FREE BY KING R QUEEN P [ऋषभ राजपूत]


any minister disagrees with a cabinet decision and is not prepared to defend it, he must resign. Hence,
statement 3 is correct.
• Article 75 also contains the principle of individual responsibility. It states that the ministers hold
office during the pleasure of the president, which means that the President can remove a minister. even
at a time when the council of ministers enjoys the confidence of the Lok Sabha. However, the President
removes a minister only on the advice of the Prime Minister.
• In Britain, every order of the King for any public act is countersigned by a minister. If the order is in
violation of any law, the minister would be held responsible and would be liable in the court. The legally
accepted phrase in Britain is, “The king can do no wrong.” Hence, he cannot be sued in any court.
o In India, on the other hand, there is no provision in the Constitution for the system of legal
responsibility of a minister. It is not required that an order of the President for a public act
should be countersigned by a minister. Hence, statement 2 is correct.
Q 92.B
• The Deputy Speaker has one special privilege, that is, whenever he is appointed as a member of a
parliamentary committee, he automatically becomes its chairman. Like the Speaker, the Deputy
Speaker, while presiding over the House, cannot vote in the first instance; he can only exercise a casting
vote in the case of a tie. Further, when a resolution for the removal of the Deputy Speaker is under
consideration by the House, he cannot preside at the sitting of the House, though he may be
present. Hence, statement 1 is correct.
• Like the Speaker, the Deputy Speaker is also elected by the Lok Sabha itself from amongst its members.
He is elected after the election of the Speaker has taken place. The date of election of the Deputy
Speaker is fixed by the Speaker. Whenever the office of the Deputy Speaker falls vacant, the Lok Sabha
elects another member to fill the vacancy. Hence, statement 2 is not correct.
• The Deputy Speaker performs the duties of the Speaker’s office when it is vacant. He also acts as the
Speaker when the latter is absent from the sitting of the House. In both cases, he assumes all the powers of
the Speaker. He also presides over the joint sitting of both Houses of Parliament, in case the Speaker is
absent from such a sitting.
• It should be noted here that the Deputy Speaker is not subordinate to the Speaker. He is directly
responsible to the House. Hence, statement 3 is not correct.

Q 93.A
• Business Advisory Committee: This committee regulates the program and timetable of the House. It
allocates time for the transaction of legislative and other business brought before the House by the
government. The Lok Sabha committee consists of 15 members including the Speaker as its chairman. In
the Rajya Sabha, it has 11 members including the Chairman as its ex-officio chairman. Hence, option (a)
is the correct answer.
• Rules Committee: This committee considers the matters of procedure and conduct of business in the
House and recommends necessary amendments or additions to the rules of the House. The Lok Sabha
committee consists of 15 members including the Speaker as its ex-officio chairman. In the Rajya Sabha, it
consists of 16 members including the Chairman as its ex-officio chairman.
• General Purpose Committee: This committee considers and advises on matters concerning affairs of the
House, which do not fall within the jurisdiction of any other parliamentary committee. In each House, this
committee consists of the presiding officer (Speaker / Chairman) as its ex-officio chairman, Deputy
Speaker (Deputy Chairman in the case of Rajya Sabha), members of panel of chairpersons (panel of vice
chairpersons in the case of Rajya Sabha), chairpersons of all the departmental standing committees of the
House, leaders of recognised parties and groups in the House and such other members as nominated by
the presiding officer.
• Ethics Committee: This committee was constituted in Rajya Sabha in 1997 and in Lok Sabha in 2000. It
enforces the code of conduct of members of Parliament. It examines the cases of misconduct and
recommends appropriate action. Thus, it is engaged in maintaining discipline and decorum in Parliament.

Q 94.B
• The Prime Minister is appointed by the President, while the other ministers are appointed by the President
on the advice of the Prime Minister. This means that the President can appoint only those persons as
ministers who are recommended by the Prime minister. Hence, statement 2 is correct.
• Usually, the members of Parliament, either Lok Sabha or Rajya Sabha, are appointed as ministers. A
person who is not a member of either House of Parliament can also be appointed as a minister. But,
within six months, he must become a member (either by election or by nomination) of either House of
Parliament, otherwise, he ceases to be a minister. Hence, statement 1 is not correct.
35 www.visionias.in ©Vision IAS

FREE BY KING R QUEEN P [ऋषभ राजपूत]


Q 95.C
• Recently theorists have finally explained the superconductivity of mercury, the first superconductor
ever discovered—gaining insights that could be relevant to the search for room-temperature
superconductors.
• Superconductivity is a set of physical properties observed in certain materials where electrical resistance
vanishes and magnetic flux fields are expelled from the material.
• Superconductors are separated into two main categories: Low-temperature superconductors (LTS), also
known as conventional superconductors and High-temperature superconductors (HTS) also called
unconventional superconductors.
• LTS can be described by the Bardeen-Cooper-Schrieffer(BCS) theory to explain how the electrons form
Cooper pairs. HTS use other microscopic methods to achieve zero resistance.
• The origins of HTS are one of the major unsolved problems of modern-day physics. Most of the
historical research on superconductivity has been in the direction of LTS, because those superconductors
are much easier to discover and study, and almost all applications of superconductivity involve LTS. HTS,
in contrast, are an active and exciting area of modern-day research.
• Anything that works as a superconductor above 70 Kelvin (-203oC) is generally considered an
HTS. Hence statement 1 is correct.
• In 1911, Dutch physicist Heike Kamerlingh Onnes discovered superconductivity in mercury. He found
that at a very low temperature, called the threshold temperature (around –270°C), solid mercury offers no
resistance to the flow of electric current. Hence statement 2 is correct.

Q 96.C
• The Constitution lays down only two qualifications for the appointment of a person as a
governor. These are:
o He should be a citizen of India.
o He should have completed the age of 35 years. Hence statement 3 is correct.
• Additionally, two conventions have also developed in this regard over the years.
o First, he should be an outsider, that is, he should not belong to the state where he is appointed so that
he is free from local politics. Hence statement 1 is not correct.
o Second, while appointing the governor, the president is required to consult the chief minister of
the state concerned, so that the smooth functioning of the constitutional machinery in the state is
ensured. Hence statement 2 is not correct.
o However, both conventions have been violated in some cases.

Q 97.A
• The Prime Minister is the leader of the Lower House. In this capacity, he enjoys the following powers:
o He advises the President with regard to summoning and proroguing of the sessions of the Parliament.
o He can recommend dissolution of the Lok Sabha to President at any time.
o He announces government policies on the floor of the House. Hence, statement 2 is not correct.
• Constitutionally, the Prime Minister may be a member of any of the two Houses of parliament. For
example, three Prime Ministers, Indira Gandhi (1966), Deve Gowda (1996) and Manmohan Singh (2004),
were members of the Rajya Sabha. In United Kingdom, on the other hand, the Prime Minister should
definitely be a member of the Lower House (House of Commons). Hence, statement 3 is correct.
• The Prime Minister has various roles.
o He is the chairman of the NITI Ayog (which succeeded the planning commission), National
Integration Council, Inter State Council, National Water Resources Council and some other bodies.
o He plays a significant role in shaping the foreign policy of the country.
o He is the chief spokesman of the Union government. Hence, statement 1 is not correct.
o He is leader of the party in power.

Q 98.A
• Representation of the People Act, 1950 makes the following provisions relating to the elections:
o Allocation of seats in the House of the People, the State Legislative Assemblies and the State
Legislative Councils.
o Delimitation of Parliamentary, Assembly and Council Constituencies.
o Election officers like chief electoral officers, district election officers, electoral registration officers
and so on.
o Electoral rolls for Parliamentary, Assembly and Council constituencies.
o Manner of filling seats in the Council of States to be filled by representatives of union territories.
36 www.visionias.in ©Vision IAS

FREE BY KING R QUEEN P [ऋषभ राजपूत]


o Local authorities for purposes of elections to the State Legislative Councils.
o Barring the jurisdiction of civil courts. Hence, option (a) is the correct answer.
• Representation of the People Act, 1951 provides for the actual conduct of elections to the Houses of
Parliament and to the House or Houses of the Legislature of each State, the qualifications and
disqualifications for the membership of these Houses, the corrupt practices and other election offenses,
and the decision of election disputes.
• Parliament (Prevention of Disqualification) Act, declares that certain offices of profit under the
Government shall not disqualify the holders thereof for being chosen as (or for being) members of
Parliament.
• Presidential and Vice-Presidential elections Act, 1952 regulates certain matters relating to or connected
with elections to the offices of the President and Vice-President of India.

Q 99.C
• Mutual Legal Assistance Treaties (MLATs): The MLATs in criminal matters are the bilateral
treaties entered between the countries for providing international cooperation and assistance. These
agreements allow for the exchange of evidence and information in criminal and related matters between
the signing countries. Hence statement 1 is correct.
• Nodal Agency in India: The Ministry of Home Affairs is the nodal Ministry and the Central
authority for seeking and providing mutual legal assistance in criminal law matters. Hence
statement 2 is not correct. The Ministry of External Affairs may be involved in this process when such
requests are routed through diplomatic channels by these Ministries.
• Legal Basis:
• Section 105 of the Criminal Procedure Code (CrPC) speaks of reciprocal arrangements to be made
by the Central Government with the Foreign Governments with regard to the service of
summons/warrants/judicial processes. Hence statement 3 is not correct. India has entered into Mutual
Legal Assistance Treaties/Agreements with 42 countries (November 2019).

Q 100.C
• The World Economic Forum, mostly known for its annual summit in Davos, Switzerland, is a non-
governmental lobbying organisation. According to the WEF website, “The Forum strives in all its
efforts to demonstrate entrepreneurship in the global public interest while upholding the highest standards
of governance.” Klaus Schwab founded the WEF in 1971 and has been its chairperson since then. The
WEF was envisioned as a forum that would bring Schwab’s vision of capitalism to life by engaging with
foremost political and business leaders of the world.
• The WEF is largely funded by its partnering corporations. These are generally global enterprises
with annual turnover greater than $ 5 billion. Hence statement 1 is correct.
• The reason behind choosing Davos as location for annual meet
• Davos was the setting for Thomas Mann’s novel The Magic Mountain. The book is the story of a young
man who goes to Davos to stay at a sanatorium for three weeks and ends up spending seven years. A
TIME review of the novel stated: “In the sanatorium, a high and chilly retreat, the perspective of life
changes … With death for a background, massive and eccentric as the high Alps, the caperings of man
seem puny by comparison. The idiotic decadence, the absurdly microscopic preoccupations of humanity
are emphasized by their isolation.”
• WEF’s “magical meeting place” seeks to have a philosophical significance on world leaders. Relaxed in
its serene environs, it seeks to cut through the many distractions of global politics to succeed in its mission
to create a more prosperous global economy.
• Its salience goes far beyond discussions on economic issues. In the past, it has been used as a location
for pivotal international diplomacy as leaders are able to break tensions in the town. For instance,
in 1988 Greece and Turkey avoided war, when the two signed the now famous Davos Declaration.
Hence statement 2 is correct.

Copyright © by Vision IAS


All rights are reserved. No part of this document may be reproduced, stored in a retrieval system or
transmitted in any form or by any means, electronic, mechanical, photocopying, recording or otherwise,
without prior permission of Vision IAS.
37 www.visionias.in ©Vision IAS

FREE BY KING R QUEEN P [ऋषभ राजपूत]


VISIONIAS
www.visionias.in

Test Booklet Series

TEST BOOKLET

GENERAL STUDIES (P) 2024 – Test – 4128


C
Time Allowed: Two Hours Maximum Marks: 200

INSTRUCTIONS

1. IMMEDIATELY AFTER THE COMMENCEMENT OF THE EXAMINATION, YOU SHOULD CHECK THAT THIS BOOKLET
DOES NOT HAVE ANY UNPRINTED OR TURN OR MISSING PAGES OR ITEMS, ETC. IF SO, GET IT REPLACED BY A
COMPLETE TEST BOOKLET.

2. ENCODE CLEARLY THE TEST BOOKLET SERIES A, B, C OR D AS THE CASE MAY BE IN THE APPROPRIATE PLACE IN
THE ANSWER SHEET.

3. You have to enter your Roll Number on the Test Booklet in the Box
provided alongside. Do NOT write anything else on the Test Booklet.

4. This Test Booklet contains 100 items (Questions). Each item is printed in English. Each item comprises four
responses (answers). You will select the response which you want to mark on the Answer Sheet. In case you
feel that there is more than one correct response with you consider the best. In any case, choose ONLY ONE
response for each item.

5. You have to mark all your responses ONLY on the separate Answer Sheet provided. See direction in the
answers sheet.

6. All items carry equal marks. Attempt all items. Your total marks will depend only on the number of correct
responses marked by you in the answer sheet. For every incorrect response 1/3rdof the allotted marks will be
deducted.

7. Before you proceed to mark in the Answer sheet the response to various items in the Test booklet, you have to
fill in some particulars in the answer sheets as per instruction sent to you with your Admission Certificate.

8. After you have completed filling in all responses on the answer sheet and the examination has concluded, you
should hand over to Invigilator only the answer sheet. You are permitted to take away with you the Test
Booklet.

9. Sheet for rough work are appended in the Test Booklet at the end.

DO NOT OPEN THIS BOOKLET UNTIL YOU ARE ASKED TO DO SO


1 www.visionias.in ©Vision IAS

FREE BY KING R QUEEN P [ऋषभ राजपूत]


1. Which of the following factors affect 4. Which of the following conditions aids in
insolation at the surface of the earth? the growth of Tropical Evergreen forests in

1. Rotation of the earth on its axis India?


1. Topography of the western slope of the
2. The angle of inclination of the sun’s rays
Western Ghats
3. The length of the day
2. Warm and humid areas with an annual
4. The transparency of the atmosphere precipitation of over 200 cm
5. Configuration of land in terms of its 3. Mean annual temperature above 35
aspect degrees Celsius.
Select the correct answer using the code Select the correct answer using the code

given below. given below.


(a) 1 and 2 only
(a) 1, 2, 3 and 4 only
(b) 2 and 3 only
(b) 2, 4 and 5 only
(c) 1 and 3 only
(c) 1, 3 and 5 only
(d) 1, 2 and 3
(d) 1, 2, 3, 4 and 5
5. Consider the following statements:
2. Terracing is a method of soil conservation 1. During equinoxes, all parts of the world

used to have equal days and nights.


2. Equinoxes occur during the months of
(a) prevent gully erosion and control their
June and December every year.
formation
Which of the statements given above is/are
(b) protect cultivable lands from
correct?
encroachment by sand dunes (a) 1 only
(c) reduce the impact of wind and water on (b) 2 only
bare soil (c) Both 1 and 2

(d) carry surface runoff across land without (d) Neither 1 nor 2

causing erosion
6. With reference to the Mediterranean region,
which of the following is/are cold local
3. The red soil of India has been formed due to
winds?
the weathering of 1. Sirocco
(a) brown forest soil in areas of low rainfall. 2. Mistral
(b) sandstone and quartzite in areas of high 3. Bora

rainfall. Select the correct answer using the code


given below.
(c) crystalline igneous rocks in areas of low
(a) 1 only
rainfall.
(b) 2 and 3 only
(d) shale and limestone in areas of high
(c) 1 and 2 only
rainfall. (d) 3 only
2 www.visionias.in ©Vision IAS

FREE BY KING R QUEEN P [ऋषभ राजपूत]


7. Consider the following pairs: 10. Recently, the Ministry of Culture announced
Peak Location a year-long commemoration of the birth
1. Mahendragiri : Eastern Ghats anniversary of Santh Sevalal Maharaj. He is
a spiritual teacher/guru of which of the
2. Doddabetta Peak : Cardamom hills
following communities?
3. Anai Mudi : Nilgiri Hills
(a) Gujjar
4. Dhupgarh : Mahadeo Hills
(b) Hatti
Which of the pairs given above are correctly (c) Banjara
matched? (d) Kalbeliya
(a) 1 and 2 only
(b) 2 and 3 only 11. Arrange the following gases in decreasing
(c) 3 and 4 only order of their share in the atmosphere.
(d) 1 and 4 only 1. Carbon Dioxide
2. Oxygen
8. With reference to the alluvial soils of India, 3. Hydrogen
consider the following statements: 4. Argon
1. They are depositional soils, transported Select the correct answer using the code
and deposited by rivers and streams. given below.
2. They are generally rich in phosphorus (a) 2-1-4-3
but poor in potash. (b) 2-4-1-3
Which of the statements given above is/are (c) 4-2-1-3

correct? (d) 2-3-1-4

(a) 1 only
12. Gersoppa Falls in India is formed by which
(b) 2 only
one of the following rivers?
(c) Both 1 and 2
(a) Kaveri
(d) Neither 1 nor 2
(b) Sharavati
(c) Pennar
9. With reference to the distribution of
(d) Tungabhadra
volcanoes in the world, consider the
following statements:
13. With reference to data embassies, consider
1. Mt. Cameroon is an active volcano in
the following statements:
East African Rift Valley. 1. They are a set of servers that store one
2. Mt. Kilimanjaro is an extinct volcano in country’s data in another country.
West Asia. 2. It is a physical data center of trusted
3. Most of the volcanic activities take place nations that enjoy diplomatic immunity
below the sea. from local laws.
Which of the statements given above are not Which of the statements given above is/are
correct? correct?
(a) 1 and 2 only (a) 1 only
(b) 2 and 3 only (b) 2 only
(c) 1 and 3 only (c) Both 1 and 2
(d) 1, 2 and 3 (d) Neither 1 nor 2
3 www.visionias.in ©Vision IAS

FREE BY KING R QUEEN P [ऋषभ राजपूत]


14. In the context of the Indian Monsoon, which 17. Consider the following statements:
of the following best describes the reason for 1. The Chilika Lake is the largest
the break in monsoon over northern India? freshwater lake in India.
(a) The rain-bearing storms are not very
2. India’s only active volcano is found on
frequent along the monsoon trough over
Barren island in the Lakshadweep group
northern India.
of Islands.
(b) The rain-bearing monsoon winds blow
parallel to the west coast of India. Which of the statements given above is/are
(c) The rain-bearing monsoon winds blow correct?
parallel to the east coast of India. (a) 1 only
(d) The westerly jet stream withdraws from (b) 2 only
its position over the north Indian plain. (c) Both 1 and 2
(d) Neither 1 nor 2
15. This forest covers vast areas of the country,
where rainfall ranges between 70 -100 cm.
18. These are types of local storms, which are
In the higher rainfall regions of the
Peninsular plateau and the northern Indian dreaded evening thunderstorms in Bengal
plain, these forests have a parkland and Assam. These showers are useful for tea,
landscape with open stretches in which teak jute, and rice cultivation. In Assam, these
and other trees interspersed with patches of storms are known as “Bardoli Chheerha”.
grass are common. As the dry season begins, Which of the following local storm is
the trees shed their leaves entirely and the mentioned in the above passage?
forest appears like a vast grassland with
(a) Loos
naked trees. Tendu, palas, amaltas, bel,
(b) Blossom shower
khair, axlewood, etc. are the common trees
of these forests. (c) Norwesters
Which of the following types of forests is (d) Mango Shower
being described in the above given passage?
(a) Moist deciduous forest 19. With reference to the measurement of
(b) Dry deciduous forest earthquakes, consider the following
(c) Semi evergreen forest statements:
(d) Tropical thorn forest
1. The Richter scale is a magnitude scale
calculated by the logarithm of the
16. Consider the following statements with
amplitude of waves.
regard to the Paris Club:
1. Its main objective is to find sustainable 2. The Mercalli scale is an intensity scale
debt-relief solutions for countries that used to quantify the effect of an
cannot repay their bilateral loans. earthquake on human life.
2. India is a member of this club. Which of the statements given above is/are
Which of the statements given above is/are correct?
correct?
(a) 1 only
(a) 1 only
(b) 2 only
(b) 2 only
(c) Both 1 and 2
(c) Both 1 and 2
(d) Neither 1 nor 2 (d) Neither 1 nor 2

4 www.visionias.in ©Vision IAS

FREE BY KING R QUEEN P [ऋषभ राजपूत]


20. Which of the following are the 22. Consider the following statements regarding
the distribution and direction of monsoon
characteristics of weather in the Indian
rainfall across India:
subcontinent in the summer season? 1. Rainfall in the western Himalayas is
often caused by the joining of the Arabin
1. Intertropical Convergence Zone shifts to
Sea branch and the Bay of Bengal
20° N and 25° N. Branch.
2. Arakan valley has a significant role in
2. Withdrawal of Easterly jet streams
directing the Bay of Bengal branch
3. Onset of Westerly jet streams on the towards the Indian sub-continent.
Which of the statements given above is/are
peninsula
correct?
Select the correct answer using the code (a) 1 only
(b) 2 only
given below.
(c) Both 1 and 2
(a) 1 only (d) Neither 1 nor 2

(b) 2 and 3 only


23. In terms of percentage, arrange the share of
(c) 1 and 3 only the following sources of freshwater in
increasing order.
(d) 2 only
1. Ice Caps and Glaciers
2. Lakes
3. Atmosphere
21. Consider the following passage:
4. Streams and rivers
These volcanoes are characterized by Select the correct answer using the code
given below.
eruptions of cooler and more viscous lavas
(a) 3-4-2-1
than basalt. These volcanoes often result in (b) 4-3-2-1
(c) 2-4-3-1
explosive eruptions. Along with lava, large
(d) 4-3-1-2
quantities of pyroclastic material and ashes
24. With reference to River Cities Alliance
find their way to the ground. This material
(RCA), consider the following statements:
accumulates in the vicinity of the vent 1. The alliance is open to only those cities
which are situated across the river
openings leading to formation of layers, and
Ganga.
gives a distinct appearance. 2. It is a collaborative effort between the
National Mission for Clean Ganga
Which of the following types of volcano is
(NMCG) and the National Institute for
described in the above passage? Urban Affairs (NIUA).
Which of the statements given above is/are
(a) Shield Volcano
correct?
(b) Composite Volcano (a) 1 only
(b) 2 only
(c) Calderas
(c) Both 1 and 2
(d) Flood basalt Volcano (d) Neither 1 nor 2
5 www.visionias.in ©Vision IAS

FREE BY KING R QUEEN P [ऋषभ राजपूत]


25. Which of the following commonly found 28. Which of the following is/are cold ocean
plants in India have medicinal properties? currents?
1. Sarpagandha 1. Benguela current
2. Babool 2. Humboldt current
3. Arjun 3. Kuroshio current
4. Kachnar
Select the correct answer using the code
Select the correct answer using the code
given below.
given below.
(a) 1 only
(a) 1, 2 and 3 only
(b) 1 and 2 only
(b) 1 and 3 only
(c) 2 and 3 only
(c) 2 and 4 only
(d) 1, 2 and 3
(d) 1, 2, 3 and 4

26. Consider the following pairs: 29. Consider the following statements:

Relief Features Physiographic division 1. Mizoram is unique by the presence of

1. Karewas : Kashmir Himalayas the ‘Loktak’ lake at the center,

2. Barchans : Great Himalayan range surrounded by mountains from all sides.

3. Bugyals : Great Indian Desert 2. Manipur is also known as the ‘Molasses

Which of the pairs given above is/are basin’ made up of soft unconsolidated

correctly matched? deposits.


(a) 1, 2 and 3 Which of the statements given above is/are
(b) 2 and 3 only correct?
(c) 1 only (a) 1 only
(d) 1 and 2 only (b) 2 only
(c) Both 1 and 2
27. With reference to the earthquakes, consider (d) Neither 1 nor 2
the following statements:
1. Deep-focus earthquakes are highly
30. Consider the following mountain ranges in
destructive because of high wave
India:
amplitude at the surface.
1. Satpura Range
2. The Circum-pacific belt accounts for 65
2. Shiwalik Range
percent of the total earthquakes in the
3. Vindhya Range
world.
Which of the statements given above is/are What is the order of the above ranges from

correct? North to South direction?

(a) 1 only (a) 2-3-1


(b) 2 only (b) 2-1-3
(c) Both 1 and 2 (c) 3-2-1
(d) Neither 1 nor 2 (d) 1-2-3
6 www.visionias.in ©Vision IAS

FREE BY KING R QUEEN P [ऋषभ राजपूत]


31. With reference to soil erosion, consider the 34. Which of the following can be found in the
following statements: vegetation cover of the Himalayan ranges?
1. Gully erosions are commonly found on 1. Deciduous forests
dry and arid-leveled lands. 2. Montane Wet Temperate Forests
2. Sheet erosion is common on steep 3. Evergreen broad leaf trees
slopes. 4. Temperate grasslands
3. A region with a large number of deep 5. Alpine forests
gullies or ravines is called a badland. Select the correct answer using the code
Which of the statements given above is/are given below.
correct? (a) 1, 2, 3 and 5 only

(a) 2 only (b) 1, 2 and 4 only

(b) 3 only (c) 3 and 5 only


(d) 1, 2, 3, 4 and 5
(c) 1 and 3 only
(d) 1, 2 and 3
35. In the context of our solar system, arrange
the following planets in increasing order of
32. Consider the following statements with
their density.
respect to Koeppen’s scheme of
1. Mercury
classification of Indian climatic regions:
2. Earth
1. It classified India into ten major climatic
3. Mars
regions.
4. Jupiter
2. It classified the Coromandel coast of
Select the correct answer using the code
Tamil Nadu as a 'Monsoon with dry
given below.
summer' climatic region.
(a) 1-3-4-2
Which of the statements given above is/are
(b) 4-3-1-2
not correct?
(c) 4-1-2-3
(a) 1 only (d) 3-4-2-1
(b) 2 only
(c) Both 1 and 2 36. With reference to the Temperate continental
(d) Neither 1 nor 2 (Steppe) climate, consider the following
statements:
33. Consider the following statements regarding 1. It has less maritime influence.
aldermen: 2. It has the presence of deciduous trees
1. They are members of a city council or which shed their leaves in the cold and
municipal body. dry seasons.
2. They do not have the right to vote in 3. It has the presence of local winds like
meetings. Fohn and Chinook.
Which of the statements given above is/are Which of the statements given above are
correct? correct?
(a) 1 only (a) 1 and 2 only
(b) 2 only (b) 2 and 3 only
(c) Both 1 and 2 (c) 1 and 3 only
(d) Neither 1 nor 2 (d) 1, 2 and 3
7 www.visionias.in ©Vision IAS

FREE BY KING R QUEEN P [ऋषभ राजपूत]


37. Consider the following statements regarding 40. In which of the following types of erosion a
clouds: fairly uniform layer of soil is removed over
1. Cirrus clouds are formed at high an entire surface area?
altitudes and have a feathery appearance.
(a) Gully erosion
2. Cumulus clouds look like cotton wool
(b) Rill erosion
with a flat base and exist in patches.
(c) Sheet erosion
3. Stratus clouds are black or dark gray and
are formed at middle levels or very near (d) Bank erosion

to the surface.
Which of the statements given above is/are 41. Consider the following pairs:
correct? Rock Type
(a) 1 only 1. Gabbro : Metamorphic
(b) 1 and 2 only
2. Limestone : Sedimentary
(c) 2 and 3 only
3. Marble : Igneous Rock
(d) 1, 2 and 3
Which of the pairs given above is/are

38. Consider the following statements with correctly matched?

regard to the India State of Forest Report (a) 1 and 2 only

2021: (b) 2 only


1. The total forest cover in India is less (c) 1 and 3 only
than 25 per cent. (d) 3 only
2. In the total forest cover, the share of
open forests is more than dense forests.
42. Which of the following statements is/are the
3. Lakshadweep has zero per cent forest
reason for the small daily and annual range
area.
Which of the statements given above is/are of temperature in the southern part of India?

correct? 1. Southern part of India lies in the tropical

(a) 1 and 2 only zone.


(b) 2 only 2. Southern part of India has a long
(c) 1 and 3 only coastline.
(d) 1, 2 and 3
Select the correct answer using the code

given below.
39. What is 'aubrite' recently seen in the news?
(a) 1 only
(a) It is a rare earth metal.
(b) It is a meteorite. (b) 2 only

(c) It is a cryptocurrency. (c) Both 1 and 2

(d) It is a type of lab-grown diamond. (d) Neither 1 nor 2


8 www.visionias.in ©Vision IAS

FREE BY KING R QUEEN P [ऋषभ राजपूत]


43. Northern Ireland Protocol was recently seen 46. Arrange the following elements in increasing
in the news in the context of affairs of of their weight percentage in the earth's
United Kingdom and European Union. In crust:
this context, which of the regions given
1. Oxygen
below are part of the United Kingdom (UK)?
2. Silicon
1. Wales
2. Scotland 3. Aluminium
3. England 4. Iron
4. Northern Ireland Select the correct answer using the code
5. Republic of Ireland given below.
Select the correct answer using the code (a) 1-2-4-3
given below.
(b) 1-2-3-4
(a) 1, 2 and 3 only
(c) 4-3-2-1
(b) 1, 4 and 5 only
(c) 2, 3 and 5 only (d) 4-3-1-2
(d) 1, 2, 3 and 4
47. Arrange the following coasts of the western
44. With reference to the seismic zones of India, coastal plains from north to south.
consider the following statements:
1. Kathiawar coast
1. The Koyna region of Maharashtra falls
2. Konkan coast
in the Seismic Zone V.
3. Kachchh coast
2. The North-East of India falls in the
Seismic Zone IV. 4. Malabar coast
3. The Konkan coast of India falls in the Select the correct answer using the code
Seismic Zone II. given below.
Which of the statements given above is/are (a) 3-1-2-4
correct?
(b) 3-2-1-4
(a) 1 only
(c) 1-2-4-3
(b) 2 and 3 only
(c) 3 only (d) 1-4-2-3
(d) None
48. Consider the following statements regarding
45. Consider the following statements regarding Savanna type climate:
the mammatus clouds: 1. This type of climate is best developed in
1. These clouds are usually formed in
Sudan.
association with large cirrostratus
2. It is characterized by extreme diurnal
clouds.
2. It is a cellular pattern of pouches range of temperature.
hanging underneath the base of a cloud. Which of the statements given above is/are
Which of the statements given above is/are correct?
correct? (a) 1 only
(a) 1 only (b) 2 only
(b) 2 only
(c) Both 1 and 2
(c) Both 1 and 2
(d) Neither 1 nor 2
(d) Neither 1 nor 2
9 www.visionias.in ©Vision IAS

FREE BY KING R QUEEN P [ऋषभ राजपूत]


49. Consider the following term 'expunction', 52. Consider the following statements with
which is used in the day-to-day working of reference to the influence of climate on soil:
the parliament in India: 1. Soil in tundra regions has deeper profiles

1. It is the removal of certain words, compared to soils in tropical regions.

sentences, or portions of a speech from 2. Bacterial action in the soil is relatively


slow in tundra regions as compared to
the records of Parliament.
tropical regions.
2. The Presiding Officer of the House has
Which of the statements given above is/are
the discretion to expunge the word or
correct?
usage.
(a) 1 only
3. Rules of Procedure and Conduct of
(b) 2 only
Business in Parliament provide for
(c) Both 1 and 2
‘expunction’. (d) Neither 1 nor 2
Which of the statements given above is/are
correct? 53. Which of the following factors affect the
(a) 1 and 2 only temperature of a place?
(b) 3 only 1. Latitude
(c) 1 and 3 only 2. Altitude
(d) 1, 2 and 3 3. Continentality
4. Ocean currents
5. Slope
50. Consider the following statements regarding
Select the correct answer using the code
the earth's crust:
given below.
1. Oceanic crust is thicker when compared
(a) 1, 2, 3 and 4 only
to continental crust.
(b) 3, 4 and 5 only
2. Oceanic crust is made up of basaltic
(c) 1, 2 and 5 only
rock.
(d) 1, 2, 3, 4 and 5
Which of the statements given above is/are
correct? 54. Which of the following are the possible
(a) 1 only factors behind earthquakes?
(b) 2 only 1. Mining activity
(c) Both 1 and 2 2. Volcanoes
(d) Neither 1 nor 2 3. Tectonic movements
4. Reservoir-induced seismicity
51. Which of the following volcanoes is known Select the correct answer using the code

as the 'Lighthouse of the Mediterranean'? given below.


(a) 1, 2 and 3 only
(a) Mt. Chimborazo
(b) 1, 2, 3 and 4
(b) Mt. Stromboli
(c) 2 and 3 only
(c) Mt. Fuji
(d) 1 and 4 only
(d) Mt. Vesuvius
10 www.visionias.in ©Vision IAS

FREE BY KING R QUEEN P [ऋषभ राजपूत]


55. Consider the following statements with 58. Consider the following pairs:
respect to circuit breaker in the stock market: Craft State
1. It is an emergency-use regulatory
1. Longpi pottery : Sikkim
measure that temporarily halts trading on
an exchange. 2. Dokra craft : Chhattisgarh
2. It caps how much the value of a stock 3. Pattachitra paintings : Odisha
can fall in a single trading session.
Which of the pairs given above is/are
Which of the statements given above is/are
correctly matched?
correct?
(a) 1 only (a) 3 only
(b) 2 only (b) 1 and 2 only
(c) Both 1 and 2
(c) 2 and 3 only
(d) Neither 1 nor 2
(d) 1, 2 and 3

56. Consider the following statements regarding


MicroLED: 59. Which of the following is/are essential
1. They have brighter and better color reasons for the formation of major deserts of
reproduction than OLED.
the world?
2. The basis of microLED technology is
1. Presence of on-shore trade winds
sapphires which can shine on their own
forever. 2. Interior continental location
Which of the statements given above is/are 3. Rain-shadow position on the leeward
correct?
side of mountains
(a) 1 only
Select the correct answer using the code
(b) 2 only
(c) Both 1 and 2 given below.
(d) Neither 1 nor 2 (a) 1 and 2 only
(b) 3 only
57. With reference to El Nino, which of the
(c) 2 and 3 only
following statements are correct?
(d) 1, 2 and 3
1. It leads to an increase in the temperature
of the water on the Peruvian coast.
2. It results in an increase in the number of 60. The term ‘Bard’ recently seen in the news is
plankton in the Eastern Pacific ocean.
associated with which of the following?
3. It can delay the onset of southwest
(a) Google’s own conversational AI chatbot.
monsoons in India.
Select the correct answer using the code (b) 3-D printed drugs manufactured by
given below. Zydus Lifesciences Ltd.
(a) 1 and 2 only (c) World's most powerful supercomputer.
(b) 2 and 3 only
(d) World’s most advanced drone developed
(c) 1 and 3 only
by Lockheed Martin.
(d) 1, 2 and 3
11 www.visionias.in ©Vision IAS

FREE BY KING R QUEEN P [ऋषभ राजपूत]


61. Consider the following statements regarding 64. Consider the following statements regarding
the drainage patterns: the Himalayan Range:
1. The tributaries of a dendritic drainage 1. The part of the Himalayas lying between
river Indus and Satluj are known as
pattern join the main stream at nearly
Punjab Himalaya.
right angles.
2. The part of the Himalayas lying between
2. A radial pattern is found in areas with a
the Satluj and Kali rivers is known as
central uplift and outward-flowing Kumaon Himalayas.
streams. 3. The part of the Himalayas lying between
Which of the statements given above is/are the Tista and Dihang rivers is known as
correct? Assam Himalayas.
(a) 1 only Which of the statements given above are
(b) 2 only correct?
(a) 1 and 2 only
(c) Both 1 and 2
(b) 2 and 3 only
(d) Neither 1 nor 2
(c) 1 and 3 only
(d) 1, 2 and 3
62. Consider the following statements with
respect to Biodiversity Heritage Sites (BHS): 65. With reference to the Northern plains,
1. The State Governments are empowered consider the following statements:
to notify BHS under the Environment 1. The gently-sloping coarse alluvial zone
Protection Act, 1986. at the foothills of Shivalik where streams
disappear sediments is known as Bhabar.
2. Yaya Tso is Ladakh’s first biodiversity
2. The older alluvium with a higher sandy
heritage site.
loam component is known as Khadar.
Which of the statements given above is/are
3. The newer, younger deposits of the
correct? floodplains are called Bhangar.
(a) 1 only Which of the statements given above is/are
(b) 2 only correct?
(c) Both 1 and 2 (a) 1 only
(d) Neither 1 nor 2 (b) 1 and 2 only
(c) 2 and 3 only
(d) 1, 2 and 3
63. Which of the following explains the
excessive cold in northern India during the
66. In the context of wind as the dominant
months of December-January? external agent in forming landforms, which
1. Continentality of the following landforms is/are formed due
2. Snowfall in the Himalayas to erosional activity of wind erosion?
3. Impact of cold winds coming from the 1. Zeugen
Tibetan plateau 2. Mesa
Select the correct answer using the code 3. Barchan
Select the correct answer using the code
given below.
given below.
(a) 1 only
(a) 1 only
(b) 2 and 3 only (b) 1 and 2 only
(c) 1 and 2 only (c) 2 and 3 only
(d) 1, 2 and 3 (d) 1, 2 and 3
12 www.visionias.in ©Vision IAS

FREE BY KING R QUEEN P [ऋषभ राजपूत]


67. Consider the following peninsular rivers: 70. Which of the following rivers are tributaries
1. Subarnarekha of Brahmaputra?
2. Sharavati 1. Subansiri
3. Brahmani 2. Lohit
Which of the rivers given above flow from 3. Barak
east to west? 4. Dibang
(a) 1 and 2 only Select the correct answer using the code

(b) 2 only given below.

(c) 3 only (a) 1, 2 and 3 only

(d) None (b) 2 and 4 only


(c) 1, 2 and 4 only
(d) 3 and 4 only
68. In the context of landform development due
to water action, which of the landforms are
71. With reference to the tsunami, consider the
formed due to erosional action of water?
following statements:
1. Valleys
1. The wave height of a tsunami decreases
2. Plungepools
rapidly as it reaches shallow water from
3. Pointbars
the open ocean.
4. Alluvial fans
2. The speed of a tsunami increases as it
Select the correct answer using the code
reaches shallow water from the open
given below.
ocean.
(a) 1 and 2 only
Which of the statements given above is/are
(b) 1, 2 and 3 only
correct?
(c) 2, 3 and 4 only
(a) 1 only
(d) 1, 3 and 4 only
(b) 2 only
(c) Both 1 and 2
69. Consider the following statements regarding (d) Neither 1 nor 2
the virtual reality:
1. It is a 3-dimensional, digital 72. With reference to azonal soils, consider the
environment completely generated following pairs:
through computers. Soil Characteristics
2. Like augmented reality, one does not 1. Regosol : a regular supply of sediments
require any equipment or gear to view
2. Lithosol : eroded soil on the steep slopes
virtual reality images.
3. Alluvial : dry and loose dune sands
Which of the statements given above is/are
How many pairs given above are correctly
correct?
matched?
(a) 1 only
(a) Only one pair
(b) 2 only (b) Only two pairs
(c) Both 1 and 2 (c) All the three pairs
(d) Neither 1 nor 2 (d) None of the pairs
13 www.visionias.in ©Vision IAS

FREE BY KING R QUEEN P [ऋषभ राजपूत]


73. With reference to the Ganga river system, 76. Which of the following is/are the
consider the following pairs: characteristic/characteristics of regur or
Place of Confluence of Rivers black soils?
confluence 1. They are formed from volcanic lava.
1. Devprayag : Bhagirathi meets 2. They are poor in iron content.
Alaknanda 3. They are self-ploughing in nature.
2. Vishnuprayag : Dhauliganga joins Select the correct answer using the codes
Alaknanda given below.
3. Rudraprayag : Pindar joins Alaknanda (a) 1 only
(b) 2 only
Which of the pairs given above is/are
(c) 1 and 3 only
correctly matched?
(d) 1, 2 and 3
(a) 1 and 2 only
(b) 1 and 3 only
77. Which of the following statements regarding
(c) 1 only
the laterite soils of India are correct?
(d) 2 only
1. They are normally heavy and black in
colour.
74. Consider the following statements with
2. They are rich in organic matter and
respect to reverse flipping:
humus.
1. It is the process of shifting the domicile
3. They are well-developed in areas with
of an overseas entity back to India.
high temperature and high rainfall.
2. Companies reverse flip due to VISA
4. They are widely cut as bricks for use in
restrictions for employees in western
house construction.
countries.
Select the correct answer using the code
Which of the statements given above is/are
given below.
correct?
(a) 1, 2 and 3 only
(a) 1 only
(b) 2, 3 and 4 only
(b) 2 only
(c) 1 and 4 only
(c) Both 1 and 2
(d) 3 and 4 only
(d) neither 1 nor 2

78. With reference to the landforms associated


75. Consider the following passage:
with volcanic activities, consider the
"These are large dome-shaped intrusive
following statements:
bodies with a level base and connected by a
1. Sills are solidified horizontal lava layers
pipe-like conduit from below. It resembles
inside the earth.
the surface volcanic domes of composite
2. Dykes are vertical intrusions that
volcano, only these are located at deeper
develop into wall-like structure.
depths. It can be regarded as the localised
3. Batholiths are huge masses of igneous
source of lava that finds its way to the
rocks formed due to the solidification of
surface."
hot magma inside the earth.
Which of the following volcanic intrusive
Which of the statements given above is/are
landform is being described in the passage
correct?
given above?
(a) 1 and 2 only
(a) Batholith
(b) 3 only
(b) Laccolith
(c) 1 and 3 only
(c) Dyke
(d) 1, 2 and 3
(d) Sill
14 www.visionias.in ©Vision IAS

FREE BY KING R QUEEN P [ऋषभ राजपूत]


79. Consider the following statements regarding 82. Consider the following pairs:
Hot Wet Equatorial Climate: Tribes State
1. Irula : Telangana
1. The diurnal range of temperature is very
2. Toto : Arunachal Pradesh
small but the annual range of
3. Bondo : Odisha
temperature is high. Which of the pairs given above is/are
2. The double rainfall peaks coinciding correctly matched?
(a) 1 and 2 only
with the equinoxes are a characteristic
(b) 3 only
feature of equatorial climate. (c) 2 and 3 only
Which of the statements given above is/are (d) 1, 2 and 3

correct?
83. Consider the following statements with
(a) 1 only
regard to social forestry:
(b) 2 only 1. Urban forestry pertains to the raising and
(c) Both 1 and 2 management of trees on public lands
only.
(d) Neither 1 nor 2
2. Agro-forestry is the raising of trees and
agriculture crops on the same land
80. What is common between Baiga Pardhauni, inclusive of the waste patches.
3. Community forestry involves the raising
Gusadi and Harul?
of trees on public, privately owned or
(a) These are harvest festivals celebrated by community lands.
tribal communities. Which of the statements given above is/are
correct?
(b) These are GI tagged rice varieties.
(a) 1 and 2 only
(c) These are traditional saree varieties.
(b) 2 only
(d) These are tribal dances. (c) 3 only
(d) 2 and 3 only

81. With reference to the lava erupting from a


84. With reference to the Inter-Tropical
volcano, consider the following statements: Convergence Zone (ITCZ), consider the
1. Basic lava is highly fluid and poor in following statements:
1. It is an area where air tends to descend.
iron and magnesium.
2. In July, ITCZ is located around 20°S-
2. Acid lava is highly viscous with a high 25°S.
melting point. 3. Shift in ITCZ causes the trade winds of
the southern hemisphere to cross the
Which of the statements given above is/are
equator.
correct? Which of the statements given above is/are
(a) 1 only correct?
(b) 2 only (a) 1 and 2 only
(b) 2 and 3 only
(c) Both 1 and 2
(c) 3 only
(d) Neither 1 nor 2 (d) 1, 2 and 3
15 www.visionias.in ©Vision IAS

FREE BY KING R QUEEN P [ऋषभ राजपूत]


85. Consider the following pairs: 89. With reference to the Indus river, consider
Island Ocean the following statements:
1. Sardinia : Mediterranean 1. It originates from a glacier in the Kailash
2. Zanzibar : Atlantic Mountain range.
3. Vancouver : Pacific 2. The Satluj is the largest tributary of the
Which of the pairs given above is/are Indus.
correctly matched? 3. The Indus flows in India only through
(a) 1 only the Union Territory of Jammu and
(b) 2 and 3 only
Kashmir.
(c) 1 and 3 only
Which of the statements given above is/are
(d) 1, 2 and 3
correct?
(a) 1 and 2 only
86. In the context of landform formation,
(b) 2 and 3 only
Cirques, Aretes and Bergschrunds are:
(c) 1 only
(a) landforms of glaciation
(d) 1, 2 and 3
(b) landforms of wind erosion
(c) landforms of water erosion
(d) landforms of Karst topography 90. With reference to the arid soils in India,
consider the following statements:
87. Arrange the following Indian cities from 1. They are generally sandy in structure
north to south: and saline in nature.
1. Chennai 2. They lack moisture and contain little
2. Bengaluru humus and organic matter.
3. Bhopal Which of the statements given above is/are
4. Jaipur correct?
Select the correct answer using the code (a) 1 only
given below. (b) 2 only
(a) 3-2-4-1 (c) Both 1 and 2
(b) 3-4-2-1 (d) Neither 1 nor 2
(c) 4-3-2-1
(d) 4-3-1-2 91. Consider the following statements regarding
the phenomenon of ‘October heat’:
88. Consider the following statements regarding
1. It occurs during the retreating monsoon
earthquakes:
season.
1. All natural earthquakes take place in the
2. It is marked by clear skies and high
lithosphere.
temperatures.
2. P waves (Primary) are considered to be
3. Lack of moisture exacerbates the heat
the most damaging waves in an
during this season.
earthquake.
Which of the statements given above is/are Which of the statements given above are
correct? correct?
(a) 1 only (a) 1 and 2 only
(b) 2 only (b) 2 and 3 only
(c) Both 1 and 2 (c) 1 and 3 only
(d) Neither 1 nor 2 (d) 1, 2 and 3

16 www.visionias.in ©Vision IAS

FREE BY KING R QUEEN P [ऋषभ राजपूत]


92. In the context of plate tectonics, which of the 95. Which of the following statements best
following are classified as minor plates? describes the term 'follow on public offer',
1. Pacific plate recently seen in the news?
2. Cocos plate (a) A company issuing of additional new
3. Nazca plate shares to its investors.
Select the correct answer using the code (b) A private company going public by the
given below.
sale of its stocks to the general public.
(a) 1 only
(c) A company buying its own outstanding
(b) 2 and 3 only
shares to reduce the number of shares
(c) 3 only
available on the open market.
(d) 1, 2 and 3
(d) A process by which an underwriter

93. Consider the following: attempts to determine the price at which

Assertion (A): The Eastern coastal plain is an initial public offering (IPO) will be
broader and is an example of an emergent offered.
coast.
Reason (R): It has less number of ports and 96. Most of the major Peninsular rivers, except
harbors. Narmada and Tapi, flow from west to east
In the context of the statements given above, due to
which one of the following is correct? (a) the peninsula's tilt towards the southeast.
(a) Both A and R are true but, R is the (b) discontinuous mountain range in the
correct explanation of A. eastern region.
(b) Both A and R are true but, R is not the
(c) upheaval of the Himalayas.
correct explanation of A.
(d) the large catchment area of Peninsular
(c) A is true, but R is false.
rivers.
(d) A is false, but R is true.

97. Consider the following statements regarding


94. Consider the following statements regarding
the New START treaty often mentioned in
the British type or The Cool Temperate
Western Margin type climate : the news:

1. These are under the permanent influence 1. It is a nuclear arms reduction treaty
of westerlies all round year. between the United States and Iran.
2. It has adequate rainfall throughout the 2. Under it, deploying not more than 1000
year with a tendency towards a slight strategic nuclear warheads and a
summer maximum. maximum of 500 long-range missiles
3. There are four distinct seasons in this and bombers has been provided.
climate. Which of the statements given above is/are
Which of the statements given above is/are correct?
correct? (a) 1 only
(a) 1 only
(b) 2 only
(b) 2 and 3 only
(c) Both 1 and 2
(c) 1 and 3 only
(d) Neither 1 nor 2
(d) 1, 2 and 3
17 www.visionias.in ©Vision IAS

FREE BY KING R QUEEN P [ऋषभ राजपूत]


98. Arrange the following states/UTs of India in
descending order as per the area under
mangrove forests:
1. Andaman and Nicobar Islands
2. Maharashtra
3. West Bengal
4. Gujarat
Select the correct answer using the code
given below.
(a) 3-4-1-2
(b) 3-4-2-1
(c) 3-1-4-2
(d) 4-3-2-1

99. "A warm summer with off-shore trade


winds, a concentration of rainfall in winter
with onshore westerlies; bright, sunny
weather with hot dry summers and wet, mild
winters and prominence of local winds."
Which of the following types of climate is
best described in the above passage given
above?
(a) Monsoon
(b) China type
(c) Savannah
(d) Mediterranean

100. With reference to the earthquakes, consider


the following statements:
1. The point where the energy is released is
called the focus.
2. The point directly above the focus is
called the hypocentre.
3. Epicenter is the first point that
experiences earthquake waves.
Which of the statements given above are
correct?
(a) 1 and 2 only
(b) 2 and 3 only
(c) 1 and 3 only
(d) 1, 2 and 3

Copyright © by Vision IAS


All rights are reserved. No part of this document may be reproduced, stored in a retrieval system or
transmitted in any form or by any means, electronic, mechanical, photocopying, recording or otherwise,
without prior permission of Vision IAS.
18 www.visionias.in ©Vision IAS

FREE BY KING R QUEEN P [ऋषभ राजपूत]


VISIONIAS
www.visionias.in
ANSWERS & EXPLANATIONS
GENERAL STUDIES (P) TEST – 4128 (2024)

Q 1.D
• The earth's surface receives most of its energy in short wavelengths. The energy received by the earth is
known as incoming solar radiation which in short is termed as insolation.
• As the earth is a geoid resembling a sphere, the sun's rays fall obliquely at the top of the atmosphere and
the earth intercepts a very small portion of the sun's energy. On an average the earth receives 1.94 calories
per sq. cm per minute at the top of its atmosphere.
• The solar output received at the top of the atmosphere varies slightly in a year due to the variations in the
distance between the earth and the sun. During its revolution around the sun, the earth is farthest from the
sun (152 million km) on 4th July. This position of the earth is called aphelion. On 3rd January, the earth is
the nearest to the sun (147 million km). This position is called perihelion.
o Therefore, the annual insolation received by the earth on 3rd January is slightly more than the amount
received on 4th July.
• Variability of Insolation at the Surface of the Earth
o The amount and the intensity of insolation vary during a day, in a season and in a year. The factors
that cause these variations in insolation are :
✓ the rotation of earth on its axis
✓ the angle of inclination of the sun’s rays
✓ the length of the day
✓ the transparency of the atmosphere
✓ the configuration of land in terms of its aspect.
• The last two however, have less influence. The fact that the earth’s axis makes an angle of 66½ with the
plane of its orbit round the sun has a greater influence on the amount of insolation received at different
latitudes.
• The second factor that determines the amount of insolation received is the angle of inclination of the rays.
This depends on the latitude of a place. The higher the latitude the less is the angle they make with the
surface of the earth resulting in slant sun rays.
• The area covered by vertical rays is always less than the slant rays. If more area is covered, the energy
gets distributed and the net energy received per unit area decreases. Moreover, the slant rays are required
to pass through greater depth of the atmosphere resulting in more absorption, scattering and diffusion.
• Hence option (d) is the correct answer.
Q 2.A
• Soil erosion is essentially aggravated by faulty practices. Lands with a slope gradient of 15–25 percent
should not be used for cultivation. If at all the land is to be used for agriculture, terraces should carefully
be made.
• Efforts should be made to prevent gully erosion and control their formation. Finger gullies can be
eliminated by terracing. In bigger gullies, the erosive velocity of water may be reduced by constructing a
series of check dams. Special attention should be paid to control headward extension of gullies. This can
be done by gully plugging, terracing, or by planting cover vegetation.
o Terracing is a soil conservation practice applied to prevent rainfall runoff on sloping land from
accumulating and causing serious erosion. Terraces consist of ridges and channels constructed across-
the-slope.
• In arid and semi-arid areas, efforts should be made to protect cultivable lands from encroachment by sand
dunes through the development of shelter belts of trees and agroforestry. Lands not suitable for cultivation
should be converted into pastures for grazing.
• Hence option (a) is the correct answer.
1 www.visionias.in ©Vision IAS

FREE BY KING R QUEEN P [ऋषभ राजपूत]


Q 3.C
• Red soil develops on crystalline igneous rocks in areas of low rainfall in the eastern and southern
parts of the Deccan Plateau. Along the piedmont zone of the Western Ghat, a long stretch of area is
occupied by red loamy soil.
• Yellow and red soils are also found in parts of Odisha and Chhattisgarh and in the southern parts of the
middle Ganga plain. The soil develops a reddish colour due to a wide diffusion of iron in crystalline and
metamorphic rocks. It looks yellow when it occurs in a hydrated form.
• The fine-grained red and yellow soils are normally fertile, whereas the coarse-grained soils found in dry
upland areas are poor in fertility. They are generally poor in nitrogen, phosphorous and humus.
• Hence option (c) is the correct answer.

Q 4.A
• Tropical Evergreen forests are found in the western slope of the Western Ghats, hills of the
northeastern region and the Andaman and Nicobar Islands.
• They are found in warm and humid areas with an annual precipitation of over 200 cm and mean
annual temperature above 22 degree C.
• The Western Ghats in India provide favorable conditions for orographic rainfall. The warm and moist air
from the Arabian sea is blocked by the Western Ghats and gets lifted over mountain ranges. As the air
rises and cools, orographic clouds are formed resulting in precipitation.
o Due to orographic rainfall, the western side of the western ghats receives heavy rainfall, more than
250cm rainfall per year. and aids in growth of Tropical evergreen forests in India.
• These forests can be extremely dense and multi-layered, with a few areas being so dense that the forest
floor hardly receives any sunlight due to the engulfing leaves and large trees.
• The main vegetation found in these forests is Ebony, Mahogany, and Rosewood.
• The tropical evergreen forests are known as lungs of the Earth because of huge greenery and the amount
of oxygen they produce. The plants usually draw carbon dioxide and produce oxygen, hence are called as
the lungs of the Earth.
• Hence, option (a) is the correct answer.

Q 5.A
• In the course of a year, the earth’s revolution round the sun with its axis inclined at 66 1/2 degrees to the
plane of the ecliptic changes the apparent altitude of the midday sun. The sun is vertically overhead at
the equator on two days each year. These are usually 21 March and 21 September though the date
changes because a year is not exactly 365 days. Hence statement 2 is not correct.
• These days are termed equinoxes meanings equal nights because, on these two days, all parts of the
world have equal days and nights. Hence statement 1 is correct.
• After the March equinox the sun appears to move north and is vertically overhead at the Tropic of Cancer
(23 1/2 degrees N.) on about 21 June. This is known as the June or summer solstice, when the northern
hemisphere will have its longest day and shortest night. Hence statement 2 is not correct.
• By about 22 December, the sun will be overhead at the Tropic of Capricorn (23 1/2 degrees S.). This is
the winter solstice when the southern hemisphere will have its longest day and shortest night. Hence
statement 3 is correct.
• The Tropics thus mark the limits of the overhead sun, for beyond these, the sun is never overhead at any
time of the year. Such regions are marked by distinct seasonal changes spring, summer, autumn and
winter. Beyond the Arctic Circle (66 1/2 degrees N.) and the Antarctic Circle (66 1/2 degrees S.) where
darkness lasts for 6 months and daylight is continuous for the remaining half of the year, it is always cold;
for even during the short summer the sun is never high in the sky.
• Within the tropics, as the midday sun varies very little from its vertical position at noon daily, the four
seasons are almost indistinguishable. Days and nights are almost equal all year round.

Q 6.B
• Many local winds, some hot, others cold are common around the Mediterranean Sea. The causes are many
and varied. The topography of the region with the high Alps in the north, the Sahara desert in the south,
continental interiors in the east and the open Atlantic on the west give rise to great differences in
temperature, pressure and precipitation.
• The passing cyclones from the Atlantic, the anticyclones from the north, and the cold air masses from the
continental interiors are often interrupted or channelled by relief features, resulting in the birth of local
winds around the Mediterranean. These winds varying in strength, direction and duration affect the lives,
crops and activities of the people there.
2 www.visionias.in ©Vision IAS

FREE BY KING R QUEEN P [ऋषभ राजपूत]


• Sirocco.
o This is a hot dry dusty wind which originates in the Sahara Desert . Though it may occur at any
time of the year, it is most frequent in spring and normally lasts for only a few days. The Sirocco
blows outwards in a southerly direction from the desert interiors into the cooler Mediterranean Sea. It
is usually associated with depressions from the Atlantic passing from the coast eastwards inland.
• Mistral.
o Mistral is a cold wind from the north of Mediterranean sea , rushing down the Rhone valley in
violent gusts between 40 and 80 miles per hour. The velocity of the Mistral is intensified by the
funnelling effect in the valley between the Alps and the Central Massif, and in extreme cases trains
may be derailed and trees uprooted.
• A similar type of cold north-easterly wind experienced along the Adriatic coast is called the Bora.
Like the Mistral, it is caused by a difference in pressure between continental Europe and the
Mediterranean. This usually occurs in winter, when the atmospheric pressure over continental Europe is
higher than that of the Mediterranean Sea.
• In West Africa, the North- East Trades blow offshore from the Sahara Desert and reach the Guinea coast
as a dry, dust -laden wind, called locally the Harmattan meaning The doctor’.
o It is so dry that its relative humidity seldom exceeds 30 per cent. The doctor’ provides a welcome
relief from the damp air of the Guinea lands by increasing the rate of evaporation with resultant
cooling effects, but it is such a dry dusty wind that , besides ruining the crops, it also stirs up a thick
dusty haze and impedes inland river navigation.
• Hence option (b) is the correct answer.

Q 7.D
• The height of the Western Ghats progressively increases from north to south. The highest peaks include
the Anai Mudi (2,695 metres) and the Doda Betta (2,637 metres).
• Doddabetta is the highest mountain in the Nilgiri Mountains at 2,637 metres (8,652 feet). There is a
reserved forest area around the peak. It is 9 km from Ooty, on the Ooty-Kotagiri Road in the Nilgiris
District of Tamil Nadu, India. It is a popular tourist attraction with road access to the summit. It is the
fourth-highest peak in South India next to Anamudi, Mannamalai and Meesapulimala. Hence pair 2 is
not correctly matched.
• Anai Mudi is a mountain located in the Ernakulam district and Idukki district of the Indian state of
Kerala. It is the highest peak in the Western Ghats and in South India, at an elevation of 2,695 metres
(8,842 ft) and a topographic prominence of 2,479 metres (8,133 ft). It is located in the southern region of
Eravikulam National Park at the junction of the Cardamom Hills, the Anaimalai Hills and the Palani
Hills. Hence pair 3 is not correctly matched.
• Mahendragiri is a mountain in
the Rayagada block of the
district of Gajapati, Odisha,
India. It is situated amongst
the Eastern Ghats at an
elevation of 1,501 metres. It is
the second biodiversity
Heritage Site of Odisha. This is
considered one out of Seven
Kula Parvat of India. Hence
pair 1 is correctly matched.
• Mount Dhupgarh or
Dhoopgarh is the highest point
in the Mahadeo Hills, Madhya
Pradesh, India. Located in
Pachmarhi in the
Narmadapuram district, it has
an elevation of 1,352 metres.
The top of the hill is a popular
area to watch sunsets.
Pachmarhi Hill station is
located close to the
peak. Hence pair 4 is
correctly matched.
3 www.visionias.in ©Vision IAS

FREE BY KING R QUEEN P [ऋषभ राजपूत]


Q 8.A
• Alluvial soils are widespread in the northern plains and the river valleys. These soils cover about 40
percent of the total area of the country. They are depositional soils, transported and deposited by rivers
and streams. Hence statement 1 is correct.
• The alluvial soils vary in nature from sandy loam to clay. They are generally rich in potash but poor in
phosphorous. Hence statement 2 is not correct.
• In the Upper and Middle Ganga plain, two different types of alluvial soils have developed, viz.
Khadar and Bhangar. Khadar is the new alluvium and is deposited by floods annually, which enriches
the soil by depositing fine silts.
• Bhangar represents a system of older alluvium, deposited away from the flood plains. Both the Khadar
and Bhangar soils contain calcareous concretions (Kankars). These soils are more loamy and clayey in
the lower and middle Ganga plain and the Brahmaputra valley.

Q 9.A
• Most known volcanic activity and earthquakes occur along converging plate margins and mid-oceanic
ridges. Nearly 70 percent of earthquakes occur in the Circum-Pacific belt. Another 20 percent of
earthquakes take place in the Mediterranean-Himalayan belt including Asia Minor, the Himalayas,
and parts of north-west China.
• Only 10 percent to 20 percent of all volcanic activity is above the sea, and terrestrial volcanic
mountains are small when compared to their submarine counterparts. Hence statement 3 is correct.
• The circum-Pacific region popularly termed the ‘Pacific Ring of Fire’, has the greatest
concentration of active volcanoes. The volcanic belt and earthquake belt closely overlap along the
‘Pacific Ring of Fire’.
• Mount Cameroon
o It is an active volcano that is also known by many other names, and it is located near the Gulf of
Guinea, within the Republic of Cameroon.
o Mt. Cameroon is the highest peak in sub-Saharan western and central Africa and the westernmost
extension of a series of hills and mountains that form a natural boundary between northern
Cameroon and Nigeria. Hence statement 1 is not correct.
• In Africa, some volcanoes are found along the East African Rift Valley, e.g. Mt. Kilimanjaro and
Mt. Kenya. Hence statement 2 is not correct.
• Mount Kilimanjaro
o Located in Tanzania, Mount Kilimanjaro is Africa’s tallest mountain at about 5,895 meters (19,340
feet). It is the largest free-standing mountain rise in the world, meaning it is not part of a mountain
range.
o Also called a stratovolcano (a term for a very large volcano made of ash, lava, and rock),
Kilimanjaro is made up of three cones: Kibo, Mawenzi, and Shira. Kibo is the summit of the
mountain and the tallest of the three volcanic formations. While Mawenzi and Shira are extinct,
Kibo is dormant and could possibly erupt again. Scientists estimate that the last time it erupted
was 360,000 years ago.
• Mediterranean volcanism
o Volcanoes of the Mediterranean region are mainly associated with the Alpine folds, e.g. Vesuvius,
Stromboli (Light House of the Mediterranean) and those of the Aegean islands. A few continue into
Asia Minor (Mt. Ararat, Mt. Elbruz).
o Mt. Stromboli is a small island in the Tyrrhenian Sea, off the north coast of Sicily, containing one of
the three active volcanoes in Italy. It is one of the eight Aeolian Islands, a volcanic arc north of
Sicily.

Q 10.C
• For the first time ever, the Ministry of Culture has announced a year-long commemoration of the
birth anniversary of Banjara Dharmaguru Santh Sevalal Maharaj.
o Born in 1739 in Shivamogga district of Karnataka.
o He was a social reformer and spiritual teacher of Banjara Community. Hence option (c) is the correct
answer.
✓ Spread across India, Banjara Community is a nomadic community with nearly 10-12 crore
population. They are classified as SC, ST and OBC in different states.
✓ Having sound knowledge in Ayurveda and Naturopathy, he eradicated myths and superstitions
prevalent among forest dwellers and nomadic tribe.

4 www.visionias.in ©Vision IAS

FREE BY KING R QUEEN P [ऋषभ राजपूत]


Q 11.B
• Atmosphere is a mixture of different gases and it envelopes the earth all round. It contains life-giving
gases like oxygen for humans and animals and carbon dioxide for plants. The air is an integral part of the
earth’s mass and 99 per cent of the total mass of the atmosphere is confined to the height of 32 km from
the earth’s surface.

• As per NASA CO2 composition by volume at present is 0.407%.


• The concentration of carbon dioxide in Earth’s atmosphere is currently at nearly 412 parts per million
(ppm) and rising. This represents a 47 percent increase since the beginning of the Industrial Age, when the
concentration was near 280 ppm,
• Hence option (b) is the correct answer.

Q 12.B
• Gerosoppa Falls, also known as Jog Falls, is located in the Shimoga district of Karnataka. It is one
of the highest waterfalls in India, with a height of 253 metres (829 feet). It is a segmented waterfall that,
depending on rain and season, becomes a plunge waterfall.
• The waterfall is formed by the Sharavati River, which originates in the Western Ghats. It is an
important river in Karnataka, flowing towards the west. It flows through the dense forests of the Western
Ghats and drops down the rocky cliffs at Jog Falls. The water plunges into a deep green valley and creates
a spectacular view. Hence option (b) is the correct answer.
• Other important waterfalls in India-
Kunchikal Falls Varahi River
Dhuandhar Falls Narmada River
Shivanasamudra Falls Kaveri River
Dudhsagar Falls Mandovi River

Q 13.C
• Data embassies
o They are a set of servers that store one country’s data and are under that country’s jurisdiction while
being located in another country. Hence statement 1 is correct.
o It is a physical data center of trusted nations that enjoy diplomatic immunity from local laws. Hence
statement 2 is correct.
o The government may allow only non-personal datasets to be stored in data embassies.

Q 14.A
• During the southwest monsoon period after having rained for a few days, if rain fails to occur for one or
more weeks, it is known as a break in the monsoon. These dry spells are quite common during the rainy
season. These breaks in the different regions are due to different reasons:
o In northern India, rains are likely to fail if the rain-bearing storms are not very frequent along
the monsoon trough or the ITCZ over this region.
o Over the west coast, the dry spells are associated with days when winds blow parallel to the coast.
• Hence option (a) is the correct answer.

Q 15.B
• Dry deciduous forest covers vast areas of the country, where rainfall ranges between 70 -100 cm.
• On the wetter margins, it has a transition to the moist deciduous, while on the drier margins to thorn
forests.
• These forests are found in rainier areas of the Peninsula and the plains of Uttar Pradesh and Bihar.
5 www.visionias.in ©Vision IAS

FREE BY KING R QUEEN P [ऋषभ राजपूत]


• In the higher rainfall regions of the Peninsular plateau and the northern Indian plain, these forests
have a parkland landscape with open stretches in which teak and other trees interspersed with
patches of grass are common.
• As the dry season begins, the trees shed their leaves completely and the forest appears like a vast
grassland with naked trees all around.
• Tendu, palas, amaltas, bel, khair, axlewood, etc. are the common trees of these forests.
• In the western and southern part of Rajasthan, vegetation cover is very scanty due to low rainfall and
overgrazing.
• Hence, option (b) is the correct answer.

Q 16.A
• Recent context: Paris Club likely to provide financial assurances to IMF on Sri Lanka debt.
o The Paris Club is a group of mostly western creditor countries that grew from a 1956 meeting in
which Argentina agreed to meet its public creditors in Paris.
o Their objective is to find sustainable debt-relief solutions for countries that are unable to repay their
bilateral loans. Hence, statement 1 is correct.
o All 22 are members of the group called Organisation for Economic Co-operation and
Development (OECD).
o The members are: Australia, Austria, Belgium, Brazil, Canada, Denmark, Finland, France, Germany,
Ireland, Israel, Italy, Japan, Korea, Netherlands, Norway, Russian Federation, Spain, Sweden,
Switzerland, United Kingdom, United States of America.
o It operates on the principles of consensus and solidarity. Any agreement reached with the debtor
country will apply equally to all its Paris Club creditors.
o A debtor country that signs an agreement with its Paris Club creditors, should not then accept from its
non-Paris Club commercial and bilateral creditors such terms of treatment of its debt that are less
favourable to the debtor than those agreed with the Paris Club.
o India and China are not members. India acts as an ad-hoc participant. Hence, statement 2 is not
correct.

Q 17.D
• The Chilika Lake is the largest saltwater lake in India. It lies in the state of Orissa, to the south of the
Mahanadi delta. In 1981, Chilika Lake has designated the first Indian wetland of international importance
under the Ramsar Convention.
It is the largest wintering ground for migratory birds on the Indian sub-continent. The lake is home to a
number of threatened species of plants and animals. The lake is an ecosystem with large fishery resources.
It sustains more than 150,000 fishers–folk living in 132 villages on the shore and islands. Hence,
statement 1 is not correct.
• India’s only active volcano is found on Barren island in the Andaman and Nicobar group of
Islands. The first recorded eruption of the volcano dates back to 1787. Since then, the volcano has erupted
more than ten times, with the most recent one being in 2020. The island is a protected area under Barren
Island Wild Life Sanctuary. Hence, statement 2 is not correct.

Q 18.C
• The summer months are a period of excessive heat and falling air pressure in the northern half of the
country. Because of the heating of the subcontinent, the ITCZ moves northwards occupying a position
centered at 25°N in July.
• In the heart of the ITCZ in the northwest, the dry and hot winds known as ‘Loo’, blow in the afternoon,
and very often, they continue to well into midnight. Dust storms in the evening are very common during
May in Punjab, Haryana, Eastern Rajasthan, and Uttar Pradesh.
• Towards the end of summer, there are pre-monsoon showers which are common phenomena in Kerala and
coastal areas of Karnataka. Locally, they are known as mango showers since they help in the early
ripening of mangoes. These are called mango showers.
• Blossom showers help coffee flowers blossom in Kerala and nearby areas.
• Norwesters are dreaded evening thunderstorms in Bengal and Assam. Their notorious nature can be
understood from the local nomenclature of ‘Kalbaisakhi’, a calamity of the month of Baisakh. These
showers are useful for tea, jute, and rice cultivation. In Assam, these storms are known as “Bardoli
Chheerha”. Hence, option (c) is the correct answer.

6 www.visionias.in ©Vision IAS

FREE BY KING R QUEEN P [ऋषभ राजपूत]


Q 19.C
• Measurement of Earthquakes is done according to the magnitude or intensity of the shock.
• Magnitude Scale:
o Magnitude is the amount of energy released and is based on the direct measurement of the size of
seismic waves. The magnitude scale is known as the Richter Scale. Hence statement 1 is correct.
o The Richter magnitude scale was developed in 1935 by Charles F. Richter as a mathematical device to
compare the size of earthquakes. The magnitude of an earthquake is determined from the logarithm
of the amplitude of waves recorded by seismographs.
o Because of the logarithmic basis of the scale, each whole number increase in magnitude represents
a ten-fold increase in measured amplitude; as an estimate of energy, each whole number step in the
magnitude scale corresponds to the release of about 31 times more energy than the amount associated
with the preceding whole number value.
• Intensity Scale:
o The intensity of an earthquake is measured in terms of its effects on human life. The intensity of
an earthquake at a specific location depends on a number of factors the total amount of energy
released, the distance from the epicenter, the types of rocks, and the degree of consolidation.
o The Mercalli intensity scale is a scale used for measuring the intensity of an earthquake. The
scale quantifies the effects of an earthquake on the Earth's surface, humans, objects of nature, and
man-made structures on a scale of I through XII, with I denoting ‘not felt’, and XII ‘total
destruction’. Hence statement 2 is correct.
o Data is gathered from individuals who have experienced the quake, and an intensity value will be
given to their location.

Q 20.A
• As the summer sets in and the sun shift northwards, the wind circulation over the subcontinent undergoes
a complete reversal at both, the lower as well as the upper levels. By the middle of July, the low-pressure
belt nearer the surface, (termed as Inter Tropical Convergence Zone (ITCZ), shifts northwards,
roughly parallel to the Himalayas between 20° N and 25° N. Hence statement 1 is correct.
• By this time, the westerly jet stream withdraws from the Indian region. In fact, meteorologists have
found an interrelationship between the northward shift of the equatorial trough (ITCZ) and the withdrawal
of the westerly jet stream from over the North Indian Plain. Hence statement 2 is not correct.
• At the upper level, an easterly jet stream flows over the southern part of the Peninsula in June and
has a maximum speed of 90 km per hour. In August, it is confined to 15°N latitude, and in September
up to 22° N latitude. The easterlies normally do not extend to the north of 30° N latitude in the upper
atmosphere. Hence statement 3 is not correct.

Q 21.B
• A volcano is a place where gases, ashes and/or molten rock material – lava – escape to the ground. A
volcano is called an active volcano if the materials mentioned are being released or have been released out
in the recent past.
o The layer below the solid crust is mantle. It has higher density than that of the crust. The mantle
contains a weaker zone called asthenosphere. It is from this that the molten rock materials find their
way to the surface.
• Volcanoes are classified on the basis of nature of eruption and the form developed at the surface. Major
types of volcanoes are as follows
o Shield Volcanoes
✓ Barring the basalt flows, the shield volcanoes are the largest of all the volcanoes on the earth. The
Hawaiian volcanoes are the most famous examples. These volcanoes are mostly made up of
basalt, a type of lava that is very fluid when erupted.
✓ For this reason, these volcanoes are not steep. They become explosive if somehow water gets into
the vent; otherwise, they are characterised by low-explosivity. The upcoming lava moves in the
form of a fountain and throws out the cone at the top of the vent and develops into cinder cone.
o Composite Volcanoes
✓ These volcanoes are characterised by eruptions of cooler and more viscous lavas than basalt.
These volcanoes often result in explosive eruptions. Along with lava, large quantities of
pyroclastic material and ashes find their way to the ground. This material accumulates in
the vicinity of the vent openings leading to formation of layers, and this makes the mounts
appear as composite volcanoes.

7 www.visionias.in ©Vision IAS

FREE BY KING R QUEEN P [ऋषभ राजपूत]


▪ Composite volcanoes, also known as stratovolcanoes, are tall, steep-sided volcanoes formed
from alternating layers of lava flows, ash, and other volcanic debris. They are characterized
by their distinctive conical shape and their explosive eruptions.
▪ Some of the features of composite volcanoes are
➢ Steep-sided cone shape: Composite volcanoes have a tall, conical shape, with steep sides
that can reach up to 30 degrees.
➢ Crater: At the summit of the volcano, there is often a bowl-shaped depression called a
crater. This is the vent through which lava, ash, and other volcanic materials are ejected
during an eruption.
➢ Lava flow: Composite volcanoes are typically composed of both explosive eruptions and
effusive eruptions, which produce lava flows. The lava flows from these volcanoes can be
quite thick and viscous, meaning that they don't travel very far from the vent.
➢ Lahars: Composite volcanoes are prone to producing lahars, which are fast-moving
mudflows that are triggered by volcanic activity. Lahars can be highly destructive, as they
can carry large boulders, trees, and other debris, and can travel many kilometers from the
volcano.
o Caldera
✓ These are the most explosive of the earth’s volcanoes. They are usually so explosive that when
they erupt they tend to collapse on themselves rather than building any tall structure. The
collapsed depressions are called calderas. Their explosiveness indicates that the magma chamber
supplying the lava is not only huge but is also in close vicinity.
o Flood Basalt
✓ Provinces These volcanoes outpour highly fluid lava that flows for long distances. Some parts of
the world are covered by thousands of sq. km of thick basalt lava flows. There can be a series of
flows with some flows attaining thickness of more than 50 m. Individual flows may extend for
hundreds of km.
✓ The Deccan Traps from India, presently covering most of the Maharashtra plateau, are a much
larger flood basalt province. It is believed that initially the trap formations covered a much larger
area than the present.
o Mid-Ocean Ridge Volcanoes
✓ These volcanoes occur in the oceanic areas. There is a system of mid-ocean ridges more than
70,000 km long that stretches through all the ocean basins. The central portion of this ridge
experiences frequent eruptions.
• Hence option (b) is the correct answer.

Q 22.C
• As the monsoon winds approach the Indian subcontinent, their southwesterly direction is modified by the
relief and thermal low pressure over northwest India. The monsoon approaches the landmass in two
branches: the Arabian Sea branch and the Bay of Bengal branch.
• The monsoon winds originating over the Arabian Sea splits into three branches.
o Its one branch is obstructed by the Western Ghats. They bring heavy rainfall in the windward side of
the Sahyadris and western coastal plains.
o Another branch of the Arabian sea monsoon strikes the coast north of Mumbai. Moving along the
Narmada and Tapi river valleys, these winds cause rainfall in extensive areas of central India.
o A third branch of this monsoon wind strikes the Saurashtra Peninsula and the Kachchh. It then passes
over west Rajasthan and along the Aravalis, causing only a scanty rainfall. In Punjab and Haryana,
it too joins the Bay of Bengal branch. These two branches, reinforced by each other, cause rains
in the western Himalayas. Hence, statement 1 is correct.
• The Bay of Bengal branch strikes the coast of Myanmar and part of southeast Bangladesh. But the
Arakan Hills along the coast of Myanmar deflect a big portion of this branch toward the Indian
subcontinent. The monsoon, therefore, enters West Bengal and Bangladesh from the south and southeast
instead of from the south-westerly direction. Hence, statement 2 is correct.
Q 23.B
• The distribution of water on earth is quite uneven. Many locations have plenty of water while others have
very limited quantity. The hydrological cycle, is the circulation of water within the earth’s hydrosphere in
different forms i.e. the liquid, solid and the gaseous phases. It also refers to the continuous exchange of
water between the oceans, atmosphere, landsurface and subsurface and the organisms.
• About 71 per cent of the planetary water is found in the oceans. The remaining is held as freshwater in
glaciers and icecaps, groundwater sources, lakes, soil moisture, atmosphere, streams and within life.
8 www.visionias.in ©Vision IAS

FREE BY KING R QUEEN P [ऋषभ राजपूत]


Nearly 59 per cent of the water that falls on land returns to the atmosphere through evaporation from over
the oceans as well as from other places.
• Distribution of water resources

• Hence option (b) is the correct answer.

Q 24.B
• Recent Context: The Driving Holistic Action for Urban Rivers (DHARA) Conference was organized
recently by the National Mission for Clean Ganga (NMCG) and the National Institute of Urban Affairs
(NIUA) in Pune.
o DHARA is the annual meeting of the members of the River Cities Alliance (RCA).
o It provides a platform to co-learn and discuss solutions for managing local water resources.
o One of the thrust areas of the Urban 20 (U20) Initiative is to promulgate urban water security and
DHARA 2023 is in sync with U20 as healthy rivers are vital to enhancing overall water security.
• About River Cities Alliance (RCA)
o It is a collaborative effort between the National Mission for Clean Ganga (NMCG) and the National
Institute for Urban Affairs (NIUA). Hence statement 2 is correct.
o It is launched by the Ministry of Jal Shakti along with the Ministry of Housing and Urban Affairs.
o River Cities Alliance started with 30 cities in 2021 and currently has 95 cities as members across
India.
o The Alliance is open to all river cities of India. Any river city can join the Alliance at any time. Hence
statement 1 is not correct.
o The Alliance focuses on three broad themes Networking, Capacity Building, and Technical Support.
o It is a dedicated platform for river cities in India to ideate, discuss and exchange information for
sustainable management of Urban Rivers such as:
✓ Minimizing their water footprint,
✓ Reducing impacts on river and water bodies,
✓ Capitalizing on natural, intangible, architectural heritage and associated services and
✓ Develop self-sufficient, self-sustainable water resources through recycling, and reuse strategy.

Q 25.D
• India is known for its herbs and spices from ancient times. Some 2,000 plants have been described in
Ayurveda and at least 500 are in regular use. The World Conservation Union’s Red List has named
352 medicinal plants of which 52 are critically threatened and 49 endangered. The commonly used
plants in India are:
o Sarpagandha: Used to treat blood pressure; it is found only in India.
o Jamun: The juice from ripe fruit is used to prepare vinegar, which is carminative and diuretic, and
has digestive properties. The powder of the seed is used for controlling diabetes.
o Arjun: The fresh juice of leaves is a cure for earache. It is also used to regulate blood pressure.
o Babool: Leaves are used as a cure for eye sores. Its gum is used as a tonic.
o Neem: Has high antibiotic and antibacterial properties.
o Tulsi: Is used to cure cough and cold.
o Kachnar: It is used to cure asthma and ulcers. The buds and roots are good for digestive
problems.
• Hence, option (d) is the correct answer.

9 www.visionias.in ©Vision IAS

FREE BY KING R QUEEN P [ऋषभ राजपूत]


Q 26.C
• Karewas are the thick deposits of glacial clay and other materials embedded with moraines. The
Kashmir Himalayas are also famous for Karewa formations, which are useful for the cultivation of
Zafran, a local variety of saffron. Hence pair 1 is correctly matched.
• The extension of the Peninsular plateau can be seen as far as Jaisalmer in the West, where it has been
covered by longitudinal sand ridges and crescent-shaped sand dunes called barchans. To the northwest of
the Aravali hills lies the Great Indian desert. It is a land of undulating topography dotted with
longitudinal dunes and barchans. This region receives low rainfall below 150 mm per year; hence, it has
an arid climate with low vegetation cover. It is because of these characteristic features that this is also
known as Marusthali. Hence pair 2 is not correctly matched.
• Dehradun is the largest of all the duns with an approximate length of 35-45 km and a width of 22-25 km.
In the Great Himalayan range, the valleys are mostly inhabited by the Bhotias. These are nomadic
groups who migrate to ‘Bugyals’ (the summer grasslands in the higher reaches) during the summer
months and return to the valleys during winter. The famous ‘Valley of flowers’ is also situated in this
Region. Hence pair 3 is not correctly matched.

Q 27.B
• Earthquake focus depth is an important factor in shaping the characteristics of the waves and the damage
they inflict. The focal depth can be deep (from 300 to 700 km), intermediate (60 to 300 km) or shallow
(less than 60 km).
• Deep focus earthquakes are rarely destructive because the wave amplitude is greatly attenuated by
the time it reaches the surface. Hence statement 1 is not correct.
• Shallow focus earthquakes are more common and are extremely damaging because of their close
proximity to the surface.
• The main seismic belts are as under:
o Circum-Pacific Belt: The Belt includes the coastal margins of North America, South America
and East Asia. These are as represent the eastern and western margins of the Pacific Ocean
respectively, and account for about 65 per cent of the total earthquakes of the world. Hence
statement 2 is correct.
✓ The western marginal zones are represented by the Rockies and the Andes mountain chains.
These are also the zones of convergent plate boundaries where the Pacific oceanic plate is
subducted below the American plates.
✓ The eastern marginal zones are represented by the island arcs of Kamchatka, Sakhalin, Japan
and Philippines. The earthquakes are caused due to collision of the Pacific and the Asiatic
plates and the consequent volcanic activity. Japan records about 1500 seismic shocks every
year.
o Mid-Continental Belt: The Mid-Continental Belt includes the Alpine mountains and their off
shoots in Europe, Mediterranean Sea, northern Africa, eastern Africa and the Himalayas. The
Mid-Continental Belt extends through Sulaiman and Kirthar zones in the west, the Himalayas in the
north and Myanmar in the east. This belt represents the weaker zone of Fold Mountains. About 21 per
cent of the total seismic events are recorded in this belt.
o Mid-Atlantic Ridge Belt: The Mid-Atlantic Ridge Belt includes the Mid-Atlantic ridge and several
islands near the ridge. It records moderate earthquakes which are caused due to the moving of plates
in the opposite directions. Thus the seafloor spreading and the fissure type of volcanic eruptions cause
earthquakes of moderate intensity in this region.
Q 28.B
• Ocean currents are like river flow in oceans. They represent a regular volume of water in a definite path
and direction.
• The ocean currents may be classified based on their depth as surface currents and deep water currents :
o surface currents constitute about 10 per cent of all the water in the ocean, these waters are the upper
400 m of the ocean
o deep water currents make up the other 90 per cent of the ocean water.
• These waters move around the ocean basins due to variations in the density and gravity. Deep waters sink
into the deep ocean basins at high latitudes, where the temperatures are cold enough to cause the density
to increase.
• Ocean currents can also be classified based on temperature: as cold currents and warm currents:
o cold currents bring cold water into warm water areas. These currents are usually found on the west
coast of the continents in the low and middle latitudes (true in both hemispheres) and on the east coast
in the higher latitudes in the Northern Hemisphere
10 www.visionias.in ©Vision IAS

FREE BY KING R QUEEN P [ऋषभ राजपूत]


o warm currents bring warm water into cold water areas and are usually observed on the east coast of
continents in the low and middle latitudes (true in both hemispheres). In the northern hemisphere they
are found on the west coasts of continents in high latitudes.

• The Kuroshio Current, also known as the Black or Japan Current or the Black Stream, is a north-flowing,
warm ocean current on the west side of the North Pacific Ocean basin.
• Hence option (b) is the correct answer.

Q 29.D
• Most of the ranges in Northeast India are separated from each other by numerous small rivers. The Barak
is an important river in Manipur and Mizoram.
• The physiography of Manipur is unique by the presence of a large lake known as ‘Loktak’ lake at the
center, surrounded by mountains from all sides. Hence, statement 1 is not correct.
• Mizoram which is also known as the ‘Molassis basin’ is made up of soft unconsolidated deposits. Most
of the rivers in Nagaland form the tributary of the Brahmaputra. While two rivers of Mizoram and
Manipur are the tributaries of the Barak river, which in turn is the tributary of Meghna; the rivers in the
eastern part of Manipur are the tributaries of Chindwin, which in turn is a tributary of the Irrawaddy of
Myanmar. Hence, statement 2 is not correct.
• They are flanked by Nepal Himalayas in the west and Bhutan Himalayas in the east. It is relatively small
but is a most significant part of the Himalayas. Known for its fast-flowing rivers such as Tista, it is a
region of high mountain peaks like Kanchenjunga (Kanchengiri), and deep valleys.

Q 30.A
• Siwalik Range, also called Siwalik Hills or Outer Himalayas, Siwalik also spelled Shiwalik, is the sub-
Himalayan range of the northern Indian subcontinent. It extends west-northwestward for more than 1,000
miles (1,600 km) from the Tista River in Sikkim state, northeastern India, through Nepal, across
northwestern India, and into northern Pakistan. Though only 10 miles (16 km) wide in places, the range
has an average elevation of 3,000 to 4,000 feet (900 to 1,200 meters). It rises abruptly from the plain of
the Indus and Ganges (Ganga) rivers (south) and parallels the main range of the Himalayas (north), from
which it is separated by valleys. The Siwaliks are sometimes considered to include the southern foothills
of the Assam Himalayas, which extend eastward for 400 miles (640 km) across southern Bhutan to the
bend of the Brahmaputra River. The range proper, to which the name Siwalik (from Sanskrit, meaning
“Belonging to [the God] Shiva”) was formerly restricted, is the 200 miles (320 km) of foothills in India
extending from the Ganges River at Haridwar, Uttarakhand state, northwestward to the Beas River.

11 www.visionias.in ©Vision IAS

FREE BY KING R QUEEN P [ऋषभ राजपूत]


• Vindhya Range is a broken range of hills forming the southern escarpment of the central upland of India.
From Gujarat state on the west, it extends about 675 miles (1,086 km) across Madhya Pradesh state to
abut on the Ganges (Ganga) River valley near Varanasi, Uttar Pradesh. The mountains form the southern
edge of the Malwa Plateau and then divide into two branches: the Kaimur Range, running north of the Son
River into western Bihar state, and the southern branch, running between the upper reaches of the Son and
Narmada rivers to meet the Satpura Range in the Maikala Range (or Amarkantak Plateau).
• The Satpura Range is formed by a series of scarped plateaus on the south, generally at an elevation
varying between 600-900 m above the mean sea level. This forms the northernmost boundary of the
Deccan plateau. It is a classic example of the relict mountains which are highly denuded and form
discontinuous ranges.

• Hence option (a) is the correct answer.

Q 31.B
• Wind and water are powerful agents of soil erosion because of their ability to remove soil and transport it.
Wind erosion is significant in arid and semi-arid regions. In regions with heavy rainfall and steep slopes,
erosion by running water is more significant. Water erosion, which is more serious and occurs extensively
in different parts of India, takes place mainly in the form of sheet and gully erosion.
• Sheet erosion takes place on level lands after a heavy shower, and the soil removal is not easily
noticeable. But it is harmful since it removes the finer and more fertile top soil. Hence statement 2 is not
correct.
• Gully erosion is common on steep slopes. Gullies deepen with rainfall, cutting the agricultural lands into
small fragments and making them unfit for cultivation. Hence statement 1 is not correct.
• A region with a large number of deep gullies or ravines is called a badland. Ravines are widespread in the
Chambal basin. Besides this, they are also found in Tamil Nadu and West Bengal. Hence statement 3 is
correct.
12 www.visionias.in ©Vision IAS

FREE BY KING R QUEEN P [ऋषभ राजपूत]


Q 32.A
• A climatic region has a homogeneous climatic condition which is the result of a combination of factors.
Temperature and rainfall are two important elements that are considered to be decisive in all the schemes
of climatic classification. The classification of climate, however, is a complex exercise. There are
different schemes of classification of climate.
• Koeppen based his scheme of Climatic classification on monthly values of temperature and precipitation.
He identified five major climatic types, namely:
o Tropical climates, where the mean monthly temperature throughout the year is over 18°C.
o (Dry climates, where precipitation is very low in comparison to temperature, and hence, dry. If
dryness is less, it is semiarid (S); if it is more, the climate is arid(W).
o Warm temperate climates, where the mean temperature of the coldest month is between 18°C and
minus 3°C.
o Cool temperate climates, where the mean temperature of the warmest month is over 10°C, and the
mean temperature of the coldest month is under minus 3°C.
o Ice climates, where the mean temperature of the warmest month is under 10°C.
• Each type is further subdivided into sub-types on the basis of seasonal variations in the distributional
pattern of rainfall and temperature. Koeppen used letter symbols to denote climatic types. He used S for
semi-arid and W for arid and the following small letters to define sub-types: f (sufficient precipitation), m
(rain forest despite a dry monsoon season), w (dry season in winter), h (dry and hot), c (less than four
months with mean temperature over 10°C), and g (Gangetic plain).
• Koeppen divided India into nine climatic regions. Hence, statement 1 is not correct.
• The details of Indian climatic regions based on this classification are given in the below table.

• Hence, statement 2 is correct.

Q 33.C
• Recent Context: The Supreme Court said the Constitution does not allow nominated members
(aldermen) of a municipality the right to vote in meetings. Hence statement 2 is correct.
o "Alderman” refers to a member of a city council or municipal body, with exact responsibilities
depending on the location of its usage. It is derived from Old English. Hence statement 1 is correct.
• Historical Linkages
o It originally referred to elders of a clan or tribe, though soon it became a term for the king’s viceroys,
regardless of age.
o Soon, it denoted a more specific title – “chief magistrate of a county,” having both civic and military
duties.
o As time passed, it became particularly associated with guilds with chiefs/leaders being referred to as
alderman.
o In the 12th century CE, as guilds became increasingly associated with municipal governments, the
term came to be used for officers of municipal bodies. This is the sense in which it is used to date.
• The Scenario in Delhi
o As per the Delhi Municipal Corporation Act, of 1957, ten people, over the age of 25 can be nominated
to the corporation by the administrator (the Lieutenant Governor).
o These people are expected to have special knowledge or experience in municipal administration. They
are meant to assist the house in taking decisions of public importance.

Q 34.D
• The Himalayan ranges show a succession of vegetation from the tropical to the tundra, which changes
with altitude.
• Deciduous forests are found in the foothills of the Himalayas.
• It is succeeded by the wet temperate type of forests between an altitude of 1,000-2,000 m.
13 www.visionias.in ©Vision IAS

FREE BY KING R QUEEN P [ऋषभ राजपूत]


• In the higher hill ranges of northeastern India, hilly areas of West Bengal and Uttaranchal, evergreen
broad leaf trees such as oak and chestnut are predominant.
• Between 1,500-1,750 m, pine forests are also well-developed in this zone, with Chir Pine as a very useful
commercial tree.
• Deodar, a highly valued endemic species grows mainly in the western part of the Himalayan range.
• Deodar is a durable wood mainly used in construction activity.
• Similarly, the chinar and the walnut, which sustain the famous Kashmir handicrafts, belong to this zone.
• Blue pine and spruce appear at altitudes of 2,225-3,048 m.
• At many places in this zone, temperate grasslands are also found.
• But in the higher reaches there is a transition to Alpine forests and pastures.
• Silver firs, junipers, pines, birch and rhododendrons, etc. occur between 3,000-4,000 m.
• However, these pastures are used extensively for transhumance by tribes like the Gujjars, the Bakarwals,
the Bhotiyas and the Gaddis.
• The southern slopes of the Himalayas carry a thicker vegetation cover because of relatively higher
precipitation than the drier north-facing slopes.
• At higher altitudes, mosses and lichens form part of the tundra vegetation.
• Hence, option (d) is the correct answer.

Q 35.B
• Our Solar system consists of eight planets. Out of the eight planets, mercury, venus, earth and mars are
called as the inner planets as they lie between the sun and the belt of asteroids the other four planets are
called the outer planets.
• Alternatively, the first four are called Terrestrial, meaning earth-like as they are made up of rock and
metals, and have relatively high densities. The rest four are called Jovian or Gas Giant planets. Jovian
means jupiter-like. Most of them are much larger than the terrestrial planets and have thick atmosphere.
• The difference between terrestrial and jovian planets can be attributed to the following conditions:
o The terrestrial planets were formed in the close vicinity of the parent star where it was too warm for
gases to condense to solid particles. Jovian planets were formed at quite a distant location.
o The solar wind was most intense nearer the sun; so, it blew off lots of gas and dust from the terrestrial
planets. The solar winds were not all that intense to cause similar removal of gases from the Jovian
planets.
o The terrestrial planets are smaller and their lower gravity could not hold the escaping gases.

• Hence option (b) is the correct answer.

Q 36.C
• Temperate continental climates (Steppe) are located in the heart of continents meaning they have little
maritime influence. Their climate is thus continental with extremes of temperature - summers are very
warm and winters are very cold. Hence, statement 1 is correct.
• The presence of deciduous trees is a feature of Tropical grasslands (short trees and tall grasses),
whereas, in the steppes, trees are very scarce, because of the scanty rainfall, long droughts, and severe
winters. Tall, fresh, and nutritious prairie grass are found, thus, they are often referred to as 'Granaries of
the world'. Hence, statement 2 is not correct.
• Fohn (Switzerland) and Chinook (Canadian praries) are names of local winds that play an influential
role in the pastures of temperate grasslands. Hence, statement 3 is correct.

14 www.visionias.in ©Vision IAS

FREE BY KING R QUEEN P [ऋषभ राजपूत]


Q 37.B
• Cloud is a mass of minute water droplets or tiny crystals of ice formed by the condensation of the water
vapour in free air at considerable elevations. As the clouds are formed at some height over the surface of
the earth, they take various shapes. According to their height, expanse, density and transparency or
opaqueness clouds are grouped under four types :
o Cirrus
o Cumulus
o Stratus
o Nimbus
• Cirrus clouds are formed at high altitudes (8,000 - 12,000m). They are thin and detatched clouds having
a feathery appearance. They are always white in colour. Hence statement 1 is correct.
• Cumulus clouds look like cotton wool. They are generally formed at a height of 4,000 - 7,000 m. They
exist in patches and can be seen scattered here and there. They have a flat base. Hence statement 2 is
correct.
• Stratus clouds are layered clouds covering large portions of the sky. These clouds are generally
formed either due to loss of heat or the mixing of air masses with different temperatures.
• Nimbus clouds are black or dark gray. They form at middle levels or very near to the surface of the
earth. These are extremely dense and opaque to the rays of the sun. Hence statement 3 is not
correct.
o Sometimes, the clouds are so low that they seem to touch the ground. Nimbus clouds are shapeless
masses of thick vapour.
• A combination of these four basic types can give rise to the following types of clouds: high clouds –
cirrus, cirrostratus, cirrocumulus; middle clouds – altostratus and altocumulus; low clouds – stratocumulus
and nimbostratus and clouds with extensive vertical development – cumulus and cumulonimbus.
Q 38.C
• According to the India State of Forest Report 2021:
• The Total Forest and Tree cover is 24.62% of the geographical area of the country. Hence statement 1 is
correct.
o The Total Forest cover is 7,13,789 sq km which is 21.71% of the geographical area of the country.
o The Tree cover is 2.91% of the geographical area of the country.
• Forest cover: Includes all lands having trees of more than one hectare in an area with a tree canopy
density of more than 10%, irrespective of ownership, the legal status of the land, and species composition
of trees.
o Very Dense Forest: All lands with tree canopy density of 70% and above. The relative composition
of forest cover under this category is 3.04%.
o Moderately Dense Forest: All lands with tree canopy density of 40% and more but less than 70%.
The forest cover under this category is 9.33%.
o Open Forest: All lands with tree canopy density of 10% and more but less than 40 %. The forest
cover of 9.34% falls under this category. Hence statement 2 is not correct.
• Lakshadweep has zero per cent forest area; Andaman and Nicobar Islands have 86.93 per cent. Hence
statement 3 is correct.
• Most of the states with less than 10 per cent of the forest area lie in the north and northwestern part of the
country.
• These are Rajasthan, Gujarat, Punjab, Haryana and Delhi.

Q 39.B
• Recent context: A paper was published by the Indian Academy of Sciences peer-reviewed journal
Current Science on the meteorite that fell in Gujarat’s Banaskantha district on August 17 last year.
o It states that it was a rare specimen of an aubrite seen in India for the first time since 1852.
• Diyodar Meteorite streaked over India, breaking apart as it descended through the air to scatter over two
villages in Banaskantha, Gujarat. This is the second recorded crash of an aubrite in India. The last was on
December 2, 1852, in Basti, Uttar Pradesh.
• Worldwide, aubrites have crashed in at least 12 locations since 1836, including three in Africa and six in
the U.S.
• Aubrites: are a type of achondritic (or achondrite) stony meteorite, which means they do not contain
chondrules, or small spherical grains of mineral that are common in other types of meteorites. They are
named after the Aubres meteorite, which fell in France in 1836 and was the first known example of this
type of meteorite.
• Hence option (b) is the correct answer.
15 www.visionias.in ©Vision IAS

FREE BY KING R QUEEN P [ऋषभ राजपूत]


Q 40.C
• Soil erosion refers to the removal of soil at a greater rate than its replacement by natural agencies.
Topography, rainfall, wind, lack of vegetation cover, land use practices, etc. are the causes of soil erosion.
• Water erosion means that soil particles are detached either by splash erosion (caused by raindrops), or
by the effect of running water. It is mainly categorized into the following types:
o Sheet Erosion - This means when a fairly uniform layer of soil is removed over an entire surface
area.
o Rill Erosion - It occurs where water runs in very small channels over the soil surface, with the
abrading effect of transported soil particles causing incision of the channels into the soil surface.
o Gully Erosion - It occurs when rills flow together to make larger streams (gully formation).
o Bank Erosion - It is caused by water cutting into the banks of streams and rivers. It can cause large
floods and major destruction to property.
• Hence, option (c) is the correct answer.

Q 41.B
• There are many different kinds of rocks which are grouped under three families on the basis of their mode
of formation. They are:
o Igneous Rocks — solidified from magma and lava
o Sedimentary Rocks — the result of deposition of fragments of rocks by exogenous processes
o Metamorphic Rocks — formed out of existing rocks undergoing recrystallisation
• Igneous rocks are classified based on texture. Texture depends upon size and arrangement of grains or
other physical conditions of the materials. If molten material is cooled slowly at great depths, mineral
grains may be very large.
o Sudden cooling (at the surface) results in small and smooth grains. Intermediate conditions of cooling
would result in intermediate sizes of grains making up igneous rocks.
o Granite, gabbro, pegmatite, basalt, volcanic breccia and tuff are some of the examples of igneous
rocks.
• Depending upon the mode of formation, sedimentary rocks are classified into three major groups:
o mechanically formed — sandstone, conglomerate, limestone, shale, loess etc. are examples
o organically formed— geyserite, chalk, limestone, coal etc. are some examples
o chemically formed — chert, limestone, halite, potash etc. are some examples
• Metamorphic rocks are classified into two major groups — foliated rocks and non-foliated rocks.
Gneissoid, granite, syenite, slate, schist, marble, quartzite etc. are some examples of metamorphic rocks.
• Hence option (b) is the correct answer.

Q 42.C
• The northern part of India lies in the sub-tropical and temperate zone and the part lying south of the
Tropic of Cancer falls in the tropical zone. The tropical zone being nearer to the equator experiences high
temperatures throughout the year with small daily and annual ranges. The area north of the Tropic of
Cancer, being away from the equator, experiences an extreme climate with a high daily and annual range
of temperature. Hence, statement 1 is correct.
• With a long coastline, large coastal areas have an equable climate. Areas in the interior of India are far
away from the moderating influence of the sea. Such areas have extremes of climate. That is why the
people of Mumbai and the Konkan coast have hardly any idea of extremes of temperature and the
seasonal rhythm of weather. On the other hand, the seasonal contrasts in weather at places in the interior
of the country such as Delhi, Kanpur, and Amritsar affect the entire sphere of life. Hence, statement 2 is
correct.

Q 43.D
• Northern Ireland Protocol: The Northern Ireland Protocol is a post-BREXIT agreement that
created a trade border between Northern Ireland and the rest of the United Kingdom (UK).
o The protocol was an integral part of the 2019 BREXIT agreement signed between the UK and the
European Union (EU).
o Under the protocol:
✓ Northern Ireland remains in the EU single market.
✓ Trade-and-customs inspections of goods coming from Great Britain take place at Northern Ireland
ports along the Irish Sea.
• The United Kingdom (UK) is made up of England, Scotland, Wales, and Northern Ireland. Hence
option (d) is the correct answer.
16 www.visionias.in ©Vision IAS

FREE BY KING R QUEEN P [ऋषभ राजपूत]


o Apart from England, These countries have their own devolved governments, each with varying
powers.
o Ireland (also known as the Republic of Ireland) is a sovereign state which is a part of the
European Union (EU) and is not a part of the UK.
o During Brexit Voting, England and Wales voted in favor of exit from the EU. Scotland and Northern
Ireland voted in favor of staying in the EU.
o Geographically, the United Kingdom includes the island of Great Britain, the northeastern part of the
island of Ireland, and many smaller islands. Northern Ireland is the only part of the United Kingdom
that shares a land border with another sovereign state—the Republic of Ireland.

Q 44.D
• The Indian sub-continent is highly prone to multiple natural disasters including earthquakes, which
is one of the most destructive natural hazards with the potentiality of inflicting huge loss to lives and
property. Earthquakes pose a real threat to India with 59% of its geographical area vulnerable to seismic
disturbances of varying intensities including the capital city of the country.
• The varying geology at different locations in the country implies that the likelihood of damaging
earthquakes taking place at different locations is different. Thus, a seismic zone map is required so that
buildings and other structures located in different regions can be designed to withstand the different levels
of ground shaking. The current zone map divides India into four zones – II, III, IV and V.
• Regions that fall under the Earthquake (seismic) Zones in India
o Zone-V covers the entire of northeastern India, some parts of Jammu and Kashmir, some parts of
Ladakh, Himachal Pradesh, Uttarakhand, Rann of Kutch in Gujarat, some parts of North Bihar and
Andaman & Nicobar Islands. Hence statement 2 is not correct.
o Zone-IV covers the remaining parts of Jammu & Kashmir, Ladakh and Himachal Pradesh, Union
Territory of Delhi, Sikkim, northern parts of Uttar Pradesh, Bihar and West Bengal, parts of Gujarat,
and small portions of Maharashtra near the west coast and Rajasthan. The Koyna region of
Maharashtra is also in this zone. Hence statement 1 is not correct.
o Zone-III comprises of Kerala, Goa, Lakshadweep islands, remaining parts of Uttar Pradesh, Gujarat
and West Bengal, parts of Punjab, Rajasthan, Madhya Pradesh, Bihar, Jharkhand, Chhattisgarh,
Maharashtra, Odisha, Andhra Pradesh, Tamil Nadu and Karnataka. A large part of the
country stretches from the North including some parts of Rajasthan to the South through the
Konkan coast, and also the Eastern parts of the country. Hence statement 3 is not correct.
o Zone-II These two zones are contiguous, covering parts of Karnataka, Andhra Pradesh, Orissa,
Madhya Pradesh, and Rajasthan, known as low-risk earthquake zones.

Q 45.B
• Mammatus clouds
o Mammatus is a cellular pattern of pouches hanging underneath the base of a cloud, typically a
cumulonimbus raincloud, although they may be attached to other classes of parent clouds. Hence
statement 2 is correct.
17 www.visionias.in ©Vision IAS

FREE BY KING R QUEEN P [ऋषभ राजपूत]


o How are these formed?
✓ Mammatus clouds are usually formed in association with large cumulonimbus clouds. Typically,
turbulence within the cumulonimbus cloud will cause Mammatus to form. Hence statement 1 is
not correct.

Q 46.C
• The earth is composed of various kinds of elements. These elements are in solid form in the outer layer of
the earth and in hot and molten form in the interior. About 98 per cent of the total crust of the earth is
composed of eight elements like oxygen, silicon, aluminium, iron, calcium, sodium, potassium and
magnesium, and the rest is constituted by titanium, hydrogen, phosphorous, manganese, sulphur, carbon,
nickel and other elements.
• Major elements of earth's crust

• The elements in the earth’s crust are rarely found exclusively but are usually combined with other
elements to make various substances. These substances are recognised as minerals.
• Hence option (c) is the correct answer.

Q 47.A
• Extending from the Gujarat coast in the north to the Kerala coast in the south, the west coast may be
divided into the following divisions – the Kachchh and Kathiawar coast in Gujarat, Konkan coast in
Maharashtra, Goan coast and Malabar coast in Karnataka and Kerala respectively.
• The western coastal plains are narrow in the middle and get broader towards the north and south. The
rivers flowing through this coastal plain do not form any delta. The Malabar coast has got certain
distinguishing features in the form of ‘Kayals’ (backwaters), which are used for fishing, and inland
navigation and also due to its special attraction for tourists. Every year the famous Nehru Trophy
Vallamkali (boat race) is held in Punnamada Kayal in Kerala.

• Hence, option (a) is the correct answer.


18 www.visionias.in ©Vision IAS

FREE BY KING R QUEEN P [ऋषभ राजपूत]


Q 48.C
• The Savanna or Sudan Climate is a transitional type of climate found between the equatorial forests and
the trade wind hot deserts. It is confined within the tropics and is best developed in the Sudan where
the dry and wet seasons are most distinct , hence its name the Sudan Climate. Hence statement 1 is
correct.
• The Savanna type of climate is characterized by an alternate hot , rainy season and cool , dry season.
• Days are hot and during the hot season noon temperatures of over 100° F are quite frequent . When night
falls the clear sky which promotes intense heating during the day also causes radiation in the night .
Temperatures drop to well below 50°F and night frosts are not uncommon at this time of the year.
o This extreme diurnal range of temperature is another characteristic feature of the Sudan type
of climate. Hence statement 2 is correct.
• The prevailing winds of the region are the Trade Winds, which bring rain to the coastal districts. They are
strongest in the summer but are relatively dry by the time they reach the continental interiors or the
western coasts of the continents, so that grass and scattered short trees predominate.

Q 49.D
• Expunction is the removal of certain words, sentences, or portions of a speech from the records of
Parliament by the orders of the Speaker. Hence statement 1 is correct.
o Expunction is a fairly routine procedure in the Parliament and is carried out in accordance with laid
down rules.
o Deciding Authority - The Presiding Officer of the House (Speaker in Lok Sabha under Rule 380)
has the discretion to expunge the word or usage. The presiding officer decides which parts of the
proceedings will be expunged. Hence statement 2 is correct.
o Rule 380 of the Rules of Procedure and Conduct of Business in Lok Sabha provides for
‘expunction’. Hence statement 3 is correct.
o Unparliamentary Expressions - The words or expressions that would likely be considered rude or
offensive in most cultures and found ‘unparliamentary’ by Presiding officers over the years.
o ‘Unparliamentary Expressions’ is a bulky volume of books published by the Lok Sabha secretariat
containing unparliamentary expressions. It also contains content that would appear to be fairly
harmless and innocuous.
o Basis - The context in which a word or sentence is used is key to making the decision on whether to
expunge.
o Expunged portions of the proceedings cease to exist in the records of Parliament.
o They can no longer be reported by media houses, even though they may have been heard during the
live telecast of the proceedings.
o NOTE: Under Article 105(2) of the Constitution no Member of Parliament shall be liable to any
proceedings in any court in respect of anything said in Parliament or any committee thereof. However,
the MPs cannot use ‘defamatory or indecent or undignified or unparliamentary’ words inside the
House.

Q 50.B
• The Crust is the outermost solid part of the earth. It is brittle in nature. The thickness of the crust varies
under the oceanic and continental areas.
• Oceanic crust is thinner as compared to the continental crust. The mean thickness of oceanic crust
is 5 km whereas that of the continental is around 30 km. Hence statement 1 is not correct.
• The continental crust is thicker in the areas of major mountain systems. It is as much as 70 km thick in the
Himalayan region.
• It is made up of heavier rocks having density of 3 g/cm3. This type of rock found in the oceanic crust is
basaltic. The mean density of material in oceanic crust is 2.7g/cm3. Hence statement 2 is correct.

Q 51.B
• Stromboli is an island in the Tyrrhenian Sea, off the north coast of Sicily, containing Mount Stromboli,
one of the four active volcanoes in Italy. It is one of the eight Aeolian Islands, a volcanic arc north of
Sicily.
• The volcano has erupted many times and is constantly active with minor eruptions, often visible
from many points on the island and from the surrounding sea, giving rise to the island's nickname
"Lighthouse of the Mediterranean".
• Mount Stromboli has been in almost continuous eruption for the past 2,000–5,000 years, its last serious
one occurred in 1921. A pattern of eruption is maintained in which explosions occur at the summit
19 www.visionias.in ©Vision IAS

FREE BY KING R QUEEN P [ऋषभ राजपूत]


craters, with mild to moderate eruptions of incandescent volcanic bombs, a type of tephra, at
intervals ranging from minutes to hours. This pattern of Strombolian eruption, as it is known, is
also observed at other volcanoes worldwide.
• Eruptions from the summit craters typically result in a few short, mild, but energetic bursts, ranging up to
a few hundred meters in height, containing ash, incandescent lava fragments, and stone blocks.
Stromboli's activity is almost exclusively explosive, but lava flows do occur at times when volcanic
activity is high.
• Hence option (b) is the correct answer.
• Mt. Fuji: Japan’s Mt. Fuji is an active volcano about 100 kilometers southwest of Tokyo. Commonly
called “Fuji-san,” it’s the country’s tallest peak, at 3,776 meters.
• Mount Vesuvius is a somma-stratovolcano located on the Gulf of Naples in Campania, Italy, about 9 km
east of Naples and a short distance from the shore. It is one of several volcanoes forming the Campanian
volcanic arc.
• Chimborazo is a currently inactive stratovolcano in the Cordillera Occidental range of the Andes. Its last
known eruption is believed to have occurred around 550 A.D. Chimborazo's summit is the farthest point
on the Earth's surface from the Earth's center, given that it is located along the planet's equatorial bulge.

Q 52.B
• Climate plays a major influence in governing the rate and type of soil formation, particularly through
precipitation in terms of its intensity, frequency, and duration; and temperature in terms of seasonal and
diurnal variations.
• The soil of the hot tropical regions shows deeper profiles as compared to the soils of the cold tundra
regions. Although the leaf fall in tropical forests is great, much of this is consumed and translocated down
the soil profile. Hence, statement 1 is not correct.
• The effect of temperature is to influence the rate of chemical and biological reactions. In cool climates,
bacterial action is relatively slow while in the tropics, bacteria thrive. Hence, statement 2 is correct.

Q 53.D
• The Importance of Temperature
o Temperature influences the actual amount of water vapour present in the air and thus decides the
moisture-earning capacity of the air.
o It decides the rate of evaporation and condensation. and therefore governs the degree of stability of
the atmosphere.
o As relative humidity is directly related to the temperature of the air, it affects the nature and types of
cloud formation and precipitation.
• Factors Influencing Temperature
o Latitude
✓ Due to the earths inclination, the mid-day sun is almost overhead within the tropics but the sun's
rays reach the earth at an angle outside the tropics. Temperature thus diminishes from equatorial
regions to the poles.
o Altitude.
✓ Since the atmosphere is mainly heated by conduction from the earth, it can be expected that
places nearer to the earths surface are warmer than those higher up. Thus temperature decreases
with increasing height above sea level. This rate of decrease with altitude ( lapse rate ) is never
constant, varying from place to place and from season to season.
o Continentality
✓ Land surfaces are heated more quickly than water surfaces, because of the higher specific heat of
water. In other words, it requires only one-third as much energy' to raise the temperature of a
given volume of land by 1 °F as it does for an equal volume of water. This accounts for the
warmer summers, colder winters and greater range of temperature of continental interiors as
compared with maritime districts.
o Ocean currents and winds
✓ Both ocean currents and winds affect temperature by transporting their heat or coldness into
adjacent regions. Ocean currents like the Gulf Stream or the North Atlantic Drift warm the coastal
districts of western Europe keeping their ports ice-free.
o Slope, shelter and aspect.
✓ A steep slope experiences a more rapid change in temperature than a gentle one. Mountain ranges
that have an eastwest alignment like the Alps show a higher temperature on the south-facing
‘sunny slope’ than the northfacing ‘sheltered slope’.
20 www.visionias.in ©Vision IAS

FREE BY KING R QUEEN P [ऋषभ राजपूत]


o Natural vegetation and soil.
✓ There is a definite difference in temperature between forested regions and open ground. The thick
foliage of the Amazon jungle cuts off much of the in-coming insolation and in many places
sunlight never reaches the ground. It is, in fact, cool in the jungle and its shade temperature is a
few degrees lower than that of open spaces in corresponding latitudes.
• Hence option (d) is the correct answer.

Q 54.B
• An earthquake in simple words is shaking of the earth. It is caused due to release of energy, which
generates waves that travel in all directions.
• The release of energy occurs along a fault. A fault is a sharp break in the crustal rocks. Rocks along a fault
tend to move in opposite directions. As the overlying rock strata press them, the friction locks them
together. However, their tendency to move apart at some point of time overcomes the friction. As a result,
the blocks get deformed and eventually, they slide past one another abruptly. This causes dissipation of
energy, and the energy waves travel in all directions.
• Types of Earthquakes:
o Tectonic Earthquakes: These are generated due to sliding of rocks along a fault plane. This
movement causes imbalance in the crustal rocks which results in earthquakes of varying magnitude,
depending upon the nature of dislocation in the rock strata.
o Volcanic Earthquakes: Volcanic activity is considered to be one of the main causes of earthquakes.
In fact, volcanic activity and seismic events are so intimately related to each other that they become
cause and effect for each other. Each volcanic eruption is followed by an earthquake and many of
the severe earthquakes can cause volcanic eruptions.
o Collapse Earthquakes: In areas of intense mining activity, sometimes the roofs of underground
mines collapse causing minor tremors.
o Explosion Earthquakes: Ground shaking may also occur due to the explosion of chemical or
nuclear devices.
o Reservoir induced seismicity: The earthquakes that occur in the areas of large reservoirs are referred
to as reservoir induced earthquakes.
• Hence option (b) is the correct answer.

Q 55.C
• Recent Context: On the back of the Hindenburg revelations, many of Adani Group companies’
stocks have hit the lower circuits in subsequent trading sessions.
• The term "circuit breaker refers" to an emergency-use regulatory measure that temporarily halts
trading on an exchange. Hence statement 1 is correct.
• Circuit breakers function automatically by stopping trading when prices hit predefined levels in exchanges
around the world.
o Circuit breakers are triggered to stop the sell-off by stockholders. They curb panic-selling of stocks
and prevent markets from crashing.
• Effectively, circuit-breakers cap how much the value of a stock can fall in a single day/trading
session. Hence statement 2 is correct.
• These circuit breakers bring about a coordinated trading halt in all equity and equity derivative markets
nationwide, when triggered.
• Method - The Stock Exchange computes the Index circuit breaker limits on a daily basis based on the
previous day’s closing level of the index.
o This index-based market-wide circuit breaker system applies at all 3 stages of the index movement, at
10%, 15% and 20%.

Q 56.C
• Recent Context: Apple is reportedly working on a new display technology called micro LEDs, which is
considered the next big thing in the display industry.
o MicroLED is a self-illuminating diode that has brighter and better color reproduction than Organic
Light Emitting Diode (OLED) display technology. Hence statement 1 is correct.
o Principle: The basis of microLED technology is sapphires, which can shine on their own
forever. Hence statement 2 is correct.
o A microLED screen is filled with such small but strong light.
o The picture in a microLED screen is generated by several individual light-emitting diodes.
o Working: Each of these micro LEDs are semiconductor that receives electrical signals.
21 www.visionias.in ©Vision IAS

FREE BY KING R QUEEN P [ऋषभ राजपूत]


o Once these micro LEDs are gathered, they form a module. Several modules are then combined to
form screens.
o Advantages: MicroLED displays are brighter, have better color reproduction, and provide better
viewing angles.
o MicroLEDs are resolution-free, bezel-free, ratio-free, and even size-free.
o They have limitless scalability and the screen can be freely resized in any form for practical usage.
o In addition to being self-emissive, MicroLEDs also individually produce red, green, and blue colors
without needing the same backlighting or color filters as conventional displays.
• OLED Displays
o In this, each pixel has its own lighting, so this is an emissive technology.
o This allows the screen to have granular control over, which pixel is supposed to show more light and
which one is not.
o OLEDs typically have a better contrast ratio compared to LCDs.
Q 57.C
• EI-Nino is a complex weather system that appears once every three to seven years, bringing drought,
floods, and other weather extremes to different parts of the world.
• The system involves oceanic and atmospheric phenomena with the appearance of warm currents off the
coast of Peru in the Eastern Pacific and affects weather in many places including India. EI-Nino is merely
an extension of the warm equatorial current which gets replaced temporarily by the cold Peruvian current
or Humbolt current. This current increases the temperature of the water on the Peruvian coast by
10°C. Hence, statement 1 is correct.
• This results in:
o the distortion of equatorial atmospheric circulation
o irregularities in the evaporation of seawater
o reduction in the amount of plankton which further reduces the number of fish in the sea. Hence,
statement 2 is not correct.
• The word EI-Nino means ‘Child Christ’ because this current appears around Christmas in December.
December is a summer month in Peru (Southern Hemisphere).
• EI-Nino is used in India for forecasting long-range monsoon rainfall. In 1990-91, there was a wild EI-
Nino event and the onset of the southwest monsoon was delayed over most parts of the country ranging
from five to twelve days. Trade winds coming from South America normally blow westward towards Asia
during Southwest Monsoon. Warming of the Pacific Ocean results in the weakening of these winds.
Therefore, moisture and heat content get limited and result in the reduction and uneven distribution of
rainfall across the Indian subcontinent. Hence, statement 3 is correct.

Q 58.C
• Recent Context: Aadi Mahotsav, the mega National Tribal Festival at Major Dhyan Chand National
Stadium in Delhi.
o It is an annual initiative of the Tribal Cooperative Marketing Development Federation Limited
(TRIFED) under the Ministry of Tribal Affairs.
o Many tribal crafts, dances etc were displayed.
• Longpi (Nungbi) Pottery: Manipur. Hence pair 1 is not correctly matched.
o It is black earthenware crafted by Tangkhul tribe of Manipur who resides in Nungbi village.
o The technique of this art is said to be handed down from the Neolithic period.
o A unique feature of this craft is that it is crafted without a potter’s wheel.
o Clay and black rock are the two main ingredients used for this craft.
o Basically a male-oriented handicraft, this craft requires a high degree of skill and attention.
• Dhokra Craft: Chhattisgarh. Hence pair 2 is correctly matched.
o Exquisite dull gold figurines and objects de art are crafted in the Bastar & Raigarh region of
Chhattisgarh out of bell metal, brass, and bronze.
o The Ghadwas of Bastar and Jharas of Raigarh practice the dhokra art with lost wax technique or
hollow casting. It involves intricately patterning a clay core with wax ribbons and then coating it
carefully with a mix of clay and hay.
o The wax is subsequently melted off and the cavity formed is filled with molten metal. When this
solidifies, the craftsman reveals the beauty of his creation by cautiously breaking open the outer clay
shell.
• Pattachitra paintings: Odisha. Hence pair 3 is correctly matched.
o Pattachitra is a picture painted on a piece of cloth.
o This form of art is closely related to the cult of Shri Jagannath and the temple traditions in Puri.
22 www.visionias.in ©Vision IAS

FREE BY KING R QUEEN P [ऋषभ राजपूत]


o It is believed to have originated as early as the 12th century. Some of the popular themes represented
through this art form are Thia Badhia– a depiction of the temple of Jagannath; Krishna Lila – an
enactment of Jagannath as Lord Krishna displaying his powers as a child; Dasabatara Patti – the ten
incarnations of Lord Vishnu; Panchamukhi – depiction of Lord Ganesh as a five-headed deity.
o Most of the materials used in this painting are natural substances.
o A floral border is a must around the paintings, and so is the use of natural colors.
o The paintings are executed primarily in profile with elongated eyes, as well.
o With the use of such prominent solid shades, the paintings end up depicting stark emotional
expressions with great detail.
Q 59.C
• The major hot deserts of the world are located on the western coasts of the continents between
latitudes 15 and 30 degrees north and south.
• The aridity of the hot deserts is mainly due to the effects of off-shore Trade Winds, hence they are also
called Trade wind Deserts. Hence, option 1 is not correct.
• The mid-latitude deserts are mainly found on plateaux and at a considerable distance from the
sea (location in interior continental location). Hence, option 2 is correct.
• Deserts like the Patagonian desert are more due to their rain-shadow position on the leeward side of
the lofty Andes than to continentality. Hence, option 3 is correct.

Q 60.A
• Google's Bard
o Bard is Google’s own conversational articficial intelligence (AI) chatbot. Hence option (a) is the
correct answer.
o Bard is based on Google’s AI model, Language Model for Dialogue Application or Lambda.
o Google introduced LaMDA in 2021 as its generative language model for dialogue applications which
can ensure that the Google Assistant would be able to converse on any topic.
o Bard is an experimental conversational AI service that draws on information from the web to provide
fresh, high-quality responses.
o Bard will give in-depth, conversational and essay-style answers just like ChatGPT does right now.
o Technology - Bard is built on Transformer technology, which is also the backbone of ChatGPT and
other AI bots.
o Transformer technology is a neural network architecture, which is capable of making predictions
based on inputs. It is primarily used in natural language processing and computer vision technology.
• Voice Deepfakes
o A voice deepfake is one that closely mimics a real person’s voice.
o The voice can accurately replicate tonality, accents, cadence, and other unique characteristics of the
target person.
o Creating deepfakes needs high-end computers with powerful graphics cards, leveraging cloud
computing power.
o AI can use this data to render an authentic-sounding voice, which can then be used to say anything.
o Attackers are using such technology to defraud users, steal their identity, and to engage in various
other illegal activities like phone scams and posting fake videos on social media platforms.
o OpenAI’s Vall-e, My Own Voice, Resemble, Descript, ReSpeecher, and iSpeech are some of the
tools that can be used in voice cloning.
o Detecting voice deepfakes needs highly advanced technologies, software, and hardware to break
down speech patterns, background noise, and other elements.
Q 61.B
• The flow of water through well-defined channels is known as ‘drainage’ and the network of such channels
is called a ‘drainage system’. The drainage pattern of an area is determined by the geological time period,
nature and structure of rocks, topography, slope, amount of water flowing, and periodicity of the flow.
• Some of the important drainage patterns are:
• Dendritic: The drainage pattern resembling the branches of a tree is known as "dendritic," the examples
of which are the rivers of the northern plain. It develops where the river channel follows the slope of the
terrain. Hence statement 1 is not correct.
• Radial: When the rivers originate from a hill and flow in all directions, the drainage pattern is known as
"radial." The rivers originating from the Amarkantak range present a good example of it. Hence
statement 2 is correct.
• Trellis: When the primary tributaries of rivers flow parallel to each other and secondary tributaries join
them at right angles, the pattern is known as a "trellis." It develops where hard and soft rocks exist parallel
23 www.visionias.in ©Vision IAS

FREE BY KING R QUEEN P [ऋषभ राजपूत]


to each other. The right bank tributaries of the Brahmaputra River make a trellis pattern, while the left
bank tributaries exhibit a dendritic pattern.
• Centripetal: When the rivers discharge their waters from all directions into a lake or depression, the
pattern is known as "centripetal." It is the reverse of radial and occurs in areas of karst topography.

Q 62.B
• Biodiversity Heritage Sites are areas that are unique, ecologically fragile ecosystems having a rich
biodiversity.
o Under Biological Diversity Act (BDA) 2002, the State Governments are empowered to notify BHS,
in consultation with ‘local bodies’, of areas of biodiversity importance as Biodiversity Heritage
Sites. Hence, statement 1 is not correct.
o There are 36 BHS in India, Mahendragiri hill (Odisha) is the last BHS added in 2022.
• Yaya Tso recently was proposed to be made Ladakh's first biodiversity heritage site. Hence,
statement 2 is correct.
o Yaya Tso is a nesting habitat for a large number of birds and animals, such as the bar-headed goose,
black-necked crane, and brahminy duck, adding it also has the distinction of being one of the highest
breeding sites of the black-necked crane in India.

Q 63.C
• Usually, the cold weather season sets in by mid-November in northern India. December and January are
the coldest months in the northern plain. The mean daily temperature remains below 21°C over most parts
of northern India. The night temperature may be quite low, sometimes going below freezing points in
Punjab and Rajasthan.
• There are three main reasons for the excessive cold in north India during this season:
o States like Punjab, Haryana, and Rajasthan are far away from the moderating influence of sea
experience continental climate.
o The snowfall in the nearby Himalayan ranges creates a cold wave situation; and
o Around February, the cold winds coming from the Caspian Sea and Turkmenistan (not Tibetan
Plateau) bring cold waves along with frost and fog over the northwestern parts of India.
• The Tibetan plateau impacts the monsoon season in India by acting as a heat engine and initiating
monsoons. The vast Himalayan mountain range acts as a tall barrier, preventing cold, dry air in the
northern latitudes from entering the subcontinent. Excessive cold over North India in the month of
December-January is not impacted by the Tibetan plateau. Hence, option (c) is the correct answer.

Q 64.D
• Besides the longitudinal divisions, the Himalayas have been divided on the basis of regions from west to
east. These divisions have been demarcated by river valleys.
• For example, the part of the Himalayas lying between the Indus and Satluj has been traditionally known
as Punjab Himalaya but it is also known regionally as Kashmir and Himachal Himalaya from west to
east respectively. Hence, statement 1 is correct.
24 www.visionias.in ©Vision IAS

FREE BY KING R QUEEN P [ऋषभ राजपूत]


• The part of the Himalayas lying between the Satluj and Kali rivers is known as Kumaon Himalayas.
Hence, statement 2 is correct.
• The Kali and Tista rivers demarcate the Nepal Himalayas and the part lying between Tista and Dihang
rivers is known as Assam Himalayas. Hence, statement 3 is correct.

Q 65.A
• According to the variations in relief features, the Northern plains can be divided into four regions. The
rivers, after descending from the mountains deposit pebbles in a narrow belt of about 8 to 16 km in width
lying parallel to the slopes of the Shiwaliks. It is known as bhabar. All the streams disappear in this
bhabar belt. Hence, statement 1 is correct.
• South of this belt, the streams and rivers re-emerge and create a wet, swampy, and marshy region known
as terai. This was a thickly forested region full of wildlife. The forests have been cleared to create
agricultural land and to settle migrants from Pakistan after the partition.
• The largest part of the northern plain is formed of older alluvium. They lie above the flood plains of the
rivers and present a terrace-like feature. This part is known as bhangar. Hence, statement 2 is not
correct.
o The soil in this region contains calcareous deposits locally known as kankar. The newer, younger
deposits of the flood plains are called khadar. They are renewed almost every year and so are fertile,
thus, ideal for intensive Agriculture. Hence, statement 3 is not correct.
Q 66.B
• The wind though not the most effective agent of erosion , transportation and deposition, is more efficient
in arid than in humid regions. Since there is little vegetation or moisture to bind the loose surface
materials, the effects of wind erosion are almost unrestrained.
• Wind erosion is carried out in the following ways
o Deflation
o Abrasion
o Attrition
• Landforms of Wind Erosion in Deserts
o Rock pedestals or mushroom rocks.
✓ The sand blasting effect of winds against any projecting rock masses wears back the softer layers
so that an irregular edge is formed on the alternate bands of hard and soft rocks. Grooves and
hollows are cut in the rock surfaces, carving them into fantastic and grotesque looking pillars
called rock pedestals. This process of under cutting produces rocks of mushroom shape called
mushroom rocks or gour in the Sahara
o Zeugen.
✓ These are tabular masses which have a layer of soft rocks lying beneath a surface layer of
more resistant rocks. The sculpting effects of wind abrasion wear them into a weird-looking
‘ridge and furrow’ landscape.
✓ Mechanical weathering initiates their formation by opening up joints of the surface rocks. Wind
abrasion further ‘eats’ into the under lying softer layer so that deep furrows are developed.
✓ The hard rocks then stand above the furrows as ridges or zeugen and many even overhang. Such
tabular blocks of zeugen may stand 10 to 100 feet above the sunken furrows.
25 www.visionias.in ©Vision IAS

FREE BY KING R QUEEN P [ऋषभ राजपूत]


o Yardangs.
✓ Quite similar to the ‘ridge and furrow’ landscape of zeugen are the steep-sided yardangs. Instead
of lying in horizontal strata upon one another, the hard and soft rocks of yardangs are vertical
bands and are aligned in the direction of the prevailing winds.

o Mesas and buttes.


✓ It is a flat, table-like land mass with a very resistant horizontal top layer, and very steep
sides. The hard stratum on the surface resists denudation by both wind and water, and thus
protects the underlying layers of rocks from being eroded away. Mesas may be formed in
canyon regions e.g. Arizona, or on fault blocks e.g. the Table Mountain of Cape Town,
South Africa.
o Inselberg.
✓ This is a German word meaning island-mountain’. They are isolated residual hills rising abruptly
from the level ground. They arc characterised by their very steep slopes and rather rounded tops.
• Barchans are crescent or moon shaped dunes which occur individually or in groups They are live dunes
which advance steadily before winds that come from a particular prevailing direction . They are most
prevalent in the deserts of Turkestan and in the Sahara .
o Barchans are initiated probably by a chance accumulation of sand at an obstacle, such as a patch of
grass or a heap of rocks. They occur transversely to the wind , so that their horns thin out and become
lower in the direction of the wind due to the reduced frictional retardation of the winds around the
edges.
o The windward side is convex and gently-sloping while the leeward side, being sheltered , is concave
and sleep
o These are depositional landforms due to wind action.
• Hence option (b) is the correct answer.

Q 67.B
• The Western Ghats, which run close to the western coast, serve as a water divide between the major
Peninsular rivers, which discharge their water into the Bay of Bengal, and the small rivulets joining the
Arabian Sea.
o Most of the major Peninsular rivers except Narmada and Tapi flow from west to east.
• The rivers flowing towards the Arabian Sea have short courses. The Sharavati is one such river, which
originates in the Shimoga district of Karnataka and drains a catchment area of 2,209 sq. km. The
total length of the river is around 128 km, and it joins the Arabian Sea at Honnavar in Uttara
Kannada district.
o On its way, the Sharavati forms the Jog Falls, where the river falls from a height of 253 metres.
• The Narmada originates on the western flank of the Amarkantak plateau. Flowing in a rift valley
between the Satpura in the south and the Vindhyan range in the north, it forms a picturesque gorge in
marble rocks and the Dhuandhar waterfall near Jabalpur. After flowing a distance of about 1,312 km, it
meets the Arabian sea south of Bharuch, forming a broad 27 km long estuary.
• There are a number of small rivers that join the Bay of Bengal. The Subarnrekha, the Baitarni, the
Brahmani, the Vamsadhara, the Penner, the Palar, and the Vaigai are important east flowing
rivers.
• Hence option (b) is the correct answer.

Q 68.A
• In humid regions, which receive heavy rainfall running water is considered the most important of the
geomorphic agents in formation of landforms.
• Erosional Landforms
26 www.visionias.in ©Vision IAS

FREE BY KING R QUEEN P [ऋषभ राजपूत]


o Valleys
✓ Valleys start as small and narrow rills; the rills will gradually develop into long and wide
gullies; the gullies will further deepen, widen and lengthen to give rise to valleys.
✓ Depending upon dimensions and shape, many types of valleys like V-shaped valley, gorge,
canyon, etc. can be recognised. A gorge is a deep valley with very steep to straight sides and a
canyon is characterised by steep steplike side slopes and may be as deep as a gorge.
✓ A gorge is almost equal in width at its top as well as its bottom. In contrast, a canyon is wider at
its top than at its bottom. In fact, a canyon is a variant of gorge. Valley types depend upon the
type and structure of rocks in which they form.
o Potholes and Plunge Pools
✓ Over the rocky beds of hill-streams more or less circular depressions called potholes form because
of stream erosion aided by the abrasion of rock fragments. Once a small and shallow depression
forms, pebbles and boulders get collected in those depressions and get rotated by flowing water
and consequently the depressions grow in dimensions.
✓ A series of such depressions eventually join and the stream valley gets deepened. At the foot
of waterfalls also, large potholes, quite deep and wide, form because of the sheer impact of
water and rotation of boulders. Such large and deep holes at the base of waterfalls are called
plunge pools.
o Incised or Entrenched Meanders
✓ In streams that flow rapidly over steep gradients, normally erosion is concentrated on the bottom
of the stream channel. Also, in the case of steep gradient streams, lateral erosion on the sides of
the valleys is not much when compared to the streams flowing on low and gentle slopes.
✓ Because of active lateral erosion, streams flowing over gentle slopes, develop sinuous or
meandering courses. It is common to find meandering courses over floodplains and delta plains
where stream gradients are very gentle. But very deep and wide meanders can also be found cut in
hard rocks. Such meanders are called incised or entrenched meanders.
o River Terraces
✓ River terraces are surfaces marking old valley floor or floodplain levels. They may be bedrock
surfaces without any alluvial cover or alluvial terraces consisting of stream deposits.
✓ River terraces are basically products of erosion as they result due to vertical erosion by the stream
into its own depositional floodplain. There can be a number of such terraces at different heights
indicating former river bed levels. The river terraces may occur at the same elevation on either
side of the rivers in which case they are called paired terraces.
• Despositional landforms
o Alluvial Fans
✓ Alluvial fans are formed when streams flowing from higher levels break into foot slope plains of
low gradient. Normally very coarse load is carried by streams flowing over mountain slopes. This
load becomes too heavy for the streams to be carried over gentler gradients and gets dumped and
spread as a broad low to high cone shaped deposit called alluvial fan.
✓ Usually, the streams which flow over fans are not confined to their original channels for long and
shift their position across the fan forming many channels called distributaries. Alluvial fans in
humid areas show normally low cones with gentle slope from head to toe and they appear as high
cones with steep slope in arid and semi-arid climates.
o Deltas
✓ Deltas are like alluvial fans but develop at a different location. The load carried by the rivers is
dumped and spread into the sea. If this load is not carried away far into the sea or distributed
along the coast, it spreads and accumulates as a low cone.
✓ Unlike in alluvial fans, the deposits making up deltas are very well sorted with clear stratification.
The coarsest materials settle out first and the finer fractions like silts and clays are carried out into
the sea. As the delta grows, the river distributaries continue to increase in length and delta
continues to build up into the sea.
o Floodplains, Natural Levees and Point Bars
✓ Deposition develops a floodplain just as erosion makes valleys. Floodplain is thus a major
landform of river deposition.
✓ Natural levees and point bars are some of the important landforms found associated with
floodplains.
✓ Natural levees are found along the banks of large rivers. They are low, linear and parallel ridges
of coarse deposits along the banks of rivers, quite often cut into individual mounds.

27 www.visionias.in ©Vision IAS

FREE BY KING R QUEEN P [ऋषभ राजपूत]


✓ Point bars are also known as meander bars. They are found on the concave side of
meanders of large rivers and are sediments deposited in a linear fashion by flowing waters
along the bank. They are almost uniform in profile and in width and contain mixed sizes of
sediments.
• Hence option (a) is the correct answer.

Q 69.A
• Recent context: PlayStation VR2 has recently launched with dozens of games.
o The term virtual reality refers to a computer-generated, three-dimensional environment. It is a
fully digital experience that can either stimulate or differ completely from the real world. Hence
statement 1 is correct.
o In order to experience and interact with virtual reality, you’ll need the proper equipment, like a
pair of VR glasses or a headset. Hence statement 2 is not correct.
o Some examples of VR include experiences generated by gadgets such as Oculus Rift and Samsung
Gear VR. Hence, option (c) is the correct answer.
• Augmented Reality (AR) does not create a new reality, but it overlays digital images onto the real world
with the help of a device like a mobile phone or tablet.
o Examples include Instagram filters and Snapchat’s lenses.
• The distinctions between VR and AR come down to the devices they require and the experience
itself:
o AR uses a real-world setting to augment the user’s experience while VR is completely virtual;
o AR users can control their presence in the real world; VR users are controlled by the system;
o VR requires a headset device, but AR can be accessed with a smartphone;
o AR enhances both the virtual and real-world while VR only enhances a fictional reality.

Q 70.C
• The Brahmaputra has its origin in the Chemayungdung glacier of the Kailash range near the
Mansarovar lake. From here, it traverses eastward longitudinally for a distance of nearly 1,200 km in a
dry and flat region of southern Tibet, where it is known as the Tsangpo. The Rango Tsangpo is the major
right bank tributary of this river in Tibet. It emerges as a turbulent and dynamic river after carving out a
deep gorge in the Central Himalayas near Namcha Barwa (7,755 m).
• The river emerges from the foothills under the name of Siang or Dihang. It enters India west of Sadiya
town in Arunachal Pradesh. Flowing southwest, it receives its main left bank tributaries, viz., Dibang or
Sikang and Lohit; thereafter, it is known as the Brahmaputra.
• The Lohit River rises in the eastern Himalayas of Tibet and flows through Arunachal Pradesh
before joining the Brahmaputra in Assam, whereas the Dibang River rises in Arunachal Pradesh's
Mishmi Hills. It flows through the Dibang Valley and joins the Brahmaputra near Sadiya, in
Assam.
• The Brahmaputra receives numerous tributaries in its 750-kilometer journey through the Assam valley. Its
major left bank tributaries are the Burhi Dihing and Dhansari (South), whereas the important right bank
tributaries are the Subansiri, Kameng, Manas and Sankosh.
• The Subansiri, which has its origin in Tibet, is an antecedent river. It is the largest tributary of the
Brahmaputra. It flows through the states of Arunachal Pradesh and Assam before joining the
Brahmaputra.
• The Brahmaputra enters Bangladesh near Dhubri and flows southward. It finally merges with the river
Padma, which falls into the Bay of Bengal.
• On the other hand, the Barak River is not a tributary of the Brahmaputra. It originates in the
Manipur Hills and flows through the states of Manipur, Mizoram, and Assam before entering Bangladesh
and joining the Meghna River.
• Hence option (c) is the correct answer.
Q 71.D
• A tsunami is a series of enormous ocean waves caused by earthquakes, underwater landslides,
volcanic eruptions, or asteroids. As the tsunami attacks the coastline, the wave energy is compressed
into a much shorter distance creating destructive, life-threatening waves.
• In an open ocean, a tsunami is less than a few feet high at the surface, but its wave height increases
rapidly in shallow water. Tsunami wave energy extends from the surface to the bottom in the deepest
waters. Hence, statement 1 is not correct.
• In deep water, the tsunami moves at speeds of up to 800 km/h. When it approaches shallower coastal
areas, it slows down. Hence statement 2 is not correct.
28 www.visionias.in ©Vision IAS

FREE BY KING R QUEEN P [ऋषभ राजपूत]


Q 72.D
• Azonal soils are those that are immature or poorly developed. It lacks a B-horizon. Thus, A-horizon
likes immediately above the C-horizon of the weathered parent material. This may happen because of the
characteristics of the parent material or the nature of the terrain or simply the lack of time for
development.
• Azonal soils are subdivided into:
o Lithosol - erosion removes soil almost as fast, so they form on steep slopes. Hence pair 2 is not
correctly matched.
o Regosol - dry and loose dune sands. Hence pair 1 is not correctly matched.
o Alluvial soils - regular supply of sediments. Hence pair 3 is not correctly matched.

Q 73.A
• The Ganga is the most important river of India, both from the point of view of its basin and cultural
significance. It rises in the Gangotri glacier near Gaumukh (3,900 m) in the Uttarkashi district of
Uttarakhand. Here, it is known as the Bhagirathi. It cuts through the Central and the Lesser Himalayas in
narrow gorges. At Devprayag, the Bhagirathi meets the Alaknanda; hereafter, it is known as the Ganga.
Hence pair 1 is correctly matched.
• The Alaknanda has its source in the Satopanth glacier above Badrinath. The Alaknanda consists of the
Dhauli and the Vishnu Ganga, which meet at Joshimath or Vishnuprayag. Hence pair 2 is correctly
matched.
• The other tributaries of Alaknanda such as the Pindar joins it at Karnaprayag while Mandakini or Kali
Ganga meets it at Rudraprayag. Hence pair 3 is not correctly matched.
• The Ganga enters the plains at Haridwar. From there, it flows first to the south, then to the south-east and
east before splitting into two distributaries, namely the Bhagirathi and the Padma before emptying into the
Bay of Bengal.

Q 74.A
• Recent Context: According to Economic Survey 2023 there is a reverse flipping by start-ups
• Flipping is the process of transferring the entire ownership of an Indian company to an overseas entity. It
is generally accompanied by a transfer of all intellectual property and data owned by an Indian company.
• Reverse Flipping is the process of shifting the domicile of those companies back to India who
flipped earlier. Hence statement 1 is correct.
• Companies reverse flip because of easy access to capital from private equity and venture capital, changes
in rules regarding round-tripping, and the growing maturity of India's capital market. Hence statement 2
is not correct.
• Why do Companies flip?
o Flipping happens at the early stage of the startup, driven by commercial, taxation, and personal
preferences of founders and investors. Some companies decide to ‘flip’ because the major market of
their product is offshore.
29 www.visionias.in ©Vision IAS

FREE BY KING R QUEEN P [ऋषभ राजपूत]


o Sometimes, investor preferences like access to incubators drive the companies to ‘flip’ as they insist
on a particular domicile.
o For easy access to capital from private equity and venture capital, changes in rules regarding round-
tripping, and the growing maturity of India's capital market.

Q 75.B
• The lava that is released during volcanic eruptions on cooling develops into igneous rocks. The cooling
may take place either on reaching the surface or also while the lava is still in the crustal portion.
o Depending on the location of the cooling of the lava, igneous rocks are classified as volcanic rocks
(cooling at the surface) and plutonic rocks (cooling in the crust). The lava that cools within the crustal
portions assumes different forms. These forms are called intrusive forms. Ex. laccoliths, lapolith,
phacolith and sills.
• Laccoliths
o These are large dome-shaped intrusive bodies with a level base and connected by a pipe-like
conduit from below. It resembles the surface volcanic domes of composite volcanoes, only these
are located at deeper depths. It can be regarded as the localised source of lava that finds its way
to the surface. The Karnataka plateau is spotted with domal hills of granite rocks. Most of these,
now exfoliated, are examples of lacoliths or batholiths. Hence option (b) is the correct answer.

• Hence option (b) is the correct answer.

Q 76.C
• Black soil is formed due to the weathering and denudation of indigenous rocks (basalt) or the cooling and
solidification of lava after the volcanic eruption. Hence, statement 1 is correct.
• On account of its high iron content and humus, it is black in color. Hence, statement 2 is not correct.
• These soils are known for their ‘self-ploughing’ nature as they swell and become sticky when wet and
shrink when dried. So, during the dry season, these soils develop wide cracks. Hence, statement 3 is
correct.
• The black soil retains moisture for a very long time, which helps the crops, especially, the rain-fed ones,
to sustain even during the dry season.

Q 77.D
• The laterite soils develop in areas with high temperatures and high rainfall. These are the results of intense
leaching due to tropical rains. Hence statement 3 is correct.
• With rain, lime and silica are leached away, and soils rich in iron oxide and aluminium compounds are left
behind. Because of their high iron oxide content, nearly all laterites are rusty-red in color. Hence
statement 1 is not correct.
• The humus content of the soil is quickly removed by bacteria that thrive well at high temperatures. These
soils are poor in organic matter, nitrogen, phosphate, and calcium, while iron oxide and potash are in
excess. Hence statement 2 is not correct.
30 www.visionias.in ©Vision IAS

FREE BY KING R QUEEN P [ऋषभ राजपूत]


• Laterites are not suitable for cultivation; however, the application of manures and fertilisers is required to
make the soils fertile for cultivation. Red laterite soils in Tamil Nadu, Andhra Pradesh and Kerala are
more suitable for tree crops like cashewnut. Laterite soils are widely cut as bricks for use in house
construction. Hence statement 4 is correct.
• These soils have mainly developed in the higher areas of the Peninsular plateau. The laterite soils are
commonly found in Karnataka, Kerala, Tamil Nadu, Madhya Pradesh and the hilly areas of Odisha and
Assam.

Q 78.D
• Intrusive landforms are formed when magma cools within the crust. The intrusive activity of volcanoes
gives rise to various forms.
• Sills: These are solidified horizontal lava layers inside the earth. Hence statement 1 is correct.
o The near horizontal bodies of the intrusive igneous rocks are called sill or sheet, depending on the
thickness of the material. The thinner ones are called sheets while the thick horizontal deposits are
called sills. Example: Great whin sill of NE England.
• Dykes: When the lava makes its way through cracks and the fissures developed in the land, it
solidifies almost perpendicular to the ground. It gets cooled in the same position to develop a wall-
like structure. Such structures are called dykes. Hence statement 2 is correct.
o These are the most commonly found intrusive forms in the western Maharashtra area. These are
considered the feeders for the eruptions that led to the development of the Deccan traps. Cleveland
Dyke of Yorkshire, England.
• Batholiths:
o These are huge masses of igneous rocks, usually of granite. These rock masses formed due to the
cooling down and the solidification of hot magma inside the earth. Hence statement 3 is correct.
o They appear on the surface only after the denudation processes remove the overlying materials and
may be exposed on the surface after erosion.
o Example: Wicklow mountains of Ireland; the uplands of Brittany, France.
• Laccoliths:
o These are large dome-shaped intrusive bodies connected by a pipe-like conduit from below.
o These are basically intrusive counterparts of an exposed domelike batholith.
o Example: The laccoliths of Henry mountains in Utah, USA.
• Lapoliths:
o As and when the lava moves upwards, a portion of the same may tend to move in a horizontal
direction wherever it finds a weak plane. In case it develops into a saucer shape, concave to the sky
body, it is called Lopolith.
o Example: The Bushveld lopolith of Transvaal, South Africa
• Phacolith:
o A wavy mass of intrusive rocks, at times, is found at the base of synclines or at the top of anticlines in
the folded igneous country. Such wavy materials have a definite conduit to source beneath in the form
of magma chambers (subsequently developed as batholiths). These are called the Phacoliths.
o Example: Corndon hill in Shropshire, England.

Q 79.B
• The equatorial, hot wet climate is found between 5° and 10° north and south of the equator. Its greatest
extent is found in the lowlands ot the Amazon, the Congo. Malaysia and the East Indies.
• The most outstanding feature of the equatorial climate is its great uniformity of temperature throughout
the year. The mean monthly temperatures are always around 80°F with very little variation.
o There is no winter. Cloudiness and heavy precipitation help to moderate the daily temperature, so that
even at the equator itself, the climate is not unbearable.
• The diurnal range of temperature is small, and so is the annual range.
o Hence statement 1 is not correct.
• Precipitation is heavy, between 60 inches and 100 inches, and well distributed throughout the year. There
is no month without rain and distinct dry season like those of the Savanna or the tropical Monsoon
Climates, is absent. Instead, there are two periods of maximum rainfall.
o The double rainfall peaks coinciding with the equinoxes are a characteristic feature of
equatorial climates not found in any other type of climate. Hence statement 2 is correct.
• Due to the great heat in the equatorial belt, mornings are bright and sunny. There is much evaporation and
convectional air currents are set up, followed by heavy downpours of convectional rain in the afternoons
from the towering cumulonimbus clouds. Thunder and lightning often accompany the torrential showers
31 www.visionias.in ©Vision IAS

FREE BY KING R QUEEN P [ऋषभ राजपूत]


and the amount of rainfall recorded in one single afternoon may be as much as the deserts receive for the
entire year!
• The relative humidity is constantly high ( over 80 percent) and makes one feel "sticky and uncomfortable.
The monotonous climate, oppressive and enervating, taxes one's mental alertness and physical capability.

Q 80.D
• Recently at the Aadi Mahotsav, an annual initiative of the Tribal Cooperative Marketing
Development Federation Limited (TRIFED) under the Ministry of Tribal Affairs tribal dance
performances were held.
o Dance Performances:
✓ Bagurumba of Assam
✓ Panthi Dance of Chhattisgarh
✓ Gusadi of Telangana
✓ Baiga Pardhauni of Madhya Pradesh
✓ Tamang Selo of Sikkim
✓ Siddhi Dhamal of Gujarat
✓ Purulia Chhau of West Bengal
✓ Harul dance of Uttarakhand.
• Hence, option (d) is the correct answer.

Q 81.B
• A volcano is a vent or a fissure in the crust from which lava (molten rock), ash, gases, and rock fragments
erupt from a magma chamber below the surface. Volcanism is the phenomenon of the eruption of molten
rock, pyroclastics, and volcanic gases to the surface through a vent.
• Magma is composed of molten rock and is stored in the Earth’s crust. Lava is magma that reaches
the surface through a volcano vent.
• Acidic lava
o These lavas are highly viscous with a high melting point. Hence statement 2 is correct.
o They are light-colored, of low density and have a high percentage of silica.
o They flow slowly and seldom travel far before solidifying.
o The resultant volcanic cone is therefore stratified (hence the name stratovolcano) and steep-
sided.
o The rapid solidifying of lava in the vent obstructs the flow of the out-pouring lava, resulting in loud
explosions, and throwing out many volcanic bombs or pyroclasts.
• Basic lava
o These are the hottest lavas, about 1,000 °C, and are highly fluid.
o They are dark-colored basalt, rich in iron and magnesium but poor in silica. Hence statement 1
is not correct.
o They flow out of volcanic vents quietly and are not very explosive.
o Due to their high fluidity, they flow readily with a speed of 10 to 30 miles per hour.
o They affect extensive areas, spreading out as thin sheets over great distances before they solidify.
o The resultant volcano is gently sloping with a wide diameter and forms a flattened shield or dome.

Q 82.B
• Recent Context: The recognition for Vadivel Gopal and Masi Sadaiyan of the Irula community,
who are among this year’s Padma Shri recipients, has put the focus on the Irula Snake Catchers’
Industrial Cooperative Society, one of the major anti-snake venom (ASV) producers in the country.
o Irula community is a particularly vulnerable tribal group (PVTG).
o Irula (people of darkness) inhabit mostly in the northern Tamil Nadu districts and some parts of
Kerala and Karnataka. Hence pair 1 is not correctly matched.
o Researches have shown that the tribe have their origin from ethnic groups of and Australia.
o They speak Irula language that is closely related to Dravidian languages like Tamil and Kannada.
o Irulas has been traditionally catching snake and rat, but also work as labourers.
• Recent Context: Dhaniram has recently been awarded Padma Shri and has contributed for creating
the Toto language script and Toto alphabet.
o The Toto is a primitive and isolated tribal group of population of only 1,632 people.
o They are residing only in a small enclave called Toto Para in Alipurduar district in West Bengal.
Hence pair 2 is not correctly matched.

32 www.visionias.in ©Vision IAS

FREE BY KING R QUEEN P [ऋषभ राजपूत]


o Toto Para is located at the foot of the Himalayas just to the south of the borderline between Bhutan
and West Bengal (on the western bank of Torsa River).
o They speak the eponymous language called Toto language.
o Toto language belongs to Tibeto-Burman family of sub-Himalayan group, as classified by Hodgson
and Grierson.
o The Toto language does not have their own script.
o Dhaniram has created the Toto language script and Toto alphabet, for which he received ‘Padma
Shri’.
o At present, poems and novels are written in Toto.
• Recent Context: This year's budget launched PM-PVTG (Pradhan Mantri Particularly Vulnerable
Tribal Groups) programme.
o It is in line with the Odisha government’s own PVTG Empowerment and Livelihood Improvement
Programme (OPELIP), launched in 2015 for the upliftment of tribals.
o Of the 75 PVTGs identified in India, Odisha has 13, namely Birhor, Bondo, Chuktia Bhunjia,
Didayi, Juang, Kharia, Dongria Kondh, Lanjia Saora, Lodha, Mankidia, Paudi Bhuyan and
Saora. Hence pair 3 is correctly matched.

Q 83.B
• Social forestry means the management and protection of forests and afforestation on barren lands with
the purpose of helping in the environmental, social and rural development.
• The National Commission on Agriculture (1976) has classified social forestry into three categories.
• These are Urban forestry, Rural forestry and Farm forestry.
• Urban forestry pertains to the raising and management of trees on public and privately owned
lands in and around urban centres such as green belts, parks, roadside avenues, industrial and commercial
green belts, etc. Hence statement 1 is not correct.
• Rural forestry lays emphasis on promotion of agro-forestry and community-forestry.
• Agro-forestry is the raising of trees and agriculture crops on the same land inclusive of the waste
patches. Hence statement 2 is correct.
• It combines forestry with agriculture, thus, altering the simultaneous production of food, fodder, fuel,
timber and fruit.
• Community forestry involves the raising of trees on public or community land such as the village
pasture and temple land, roadside, canal bank, strips along railway lines, and schools etc. Hence
statement 3 is not correct.
o Community forestry programme aims at providing benefits to the community as a whole.
o Community forestry provides a means under which the people of landless classes can associate
themselves in treeraising and thus, get those benefits which otherwise are restricted for landowners.

Q 84.C
• The Inter Tropical Convergence Zone (ITCZ) is a low-pressure zone located at the equator where trade
winds converge, and so, it is a zone where air tends to ascend. Hence, statement 1 is not correct.
• In July, the ITCZ is located around 20°N-25°N latitudes (over the Gangetic plain), sometimes called the
monsoon trough. This monsoon trough encourages the development of thermal lows over north and
northwest India. Hence, statement 2 is not correct.
• Due to the shift of ITCZ, the trade winds of the southern hemisphere cross the equator between 40° and
60°E longitudes and start blowing from southwest to northeast due to the Coriolis force. It becomes a
southwest monsoon. In winter, the ITCZ moves southward, and so the reversal of winds from northeast to
south and southwest takes place. They are called northeast monsoons. Hence, statement 3 is correct.

Q 85.C
• Sardinia is the second-largest island in the Mediterranean Sea, after Sicily. It is located west of the
Italian Peninsula, north of Tunisia and immediately south of the French island of Corsica. Hence pair 1 is
correctly matched.
• Zanzibar Island, also known as Unguja, is by far the largest and most famous of the islands that make up
the Zanzibar Archipelago located in Indian ocean. Hence pair 2 is not correctly matched.
• Vancouver Island is an island lying off southwestern mainland British Columbia, Canada in Pacific
ocean. Hence pair 3 is correctly matched.
o With an area of 12,079 square miles (31,285 square km), it is the largest island on the Pacific coast of
North America.

33 www.visionias.in ©Vision IAS

FREE BY KING R QUEEN P [ऋषभ राजपूत]


o Vancouver Island is separated from mainland Canada by the straits of Georgia, Johnstone, and Queen
Charlotte and from the United States by Juan de Fuca Strait.
o The island, averaging 50 miles (80 km) in width and extending for 285 miles (460 km) along a
northwest–southeast axis paralleling the mainland, is actually the top of a partially submerged
mountain system.

Q 86.A
• Glaciation generally gives rise to erosional feature in the highlands and depositional features on the
lowlands, though these processes are not mutually exclusive because a glacier plays a combined role of
erosion, transportation and deposition throughout its course. A glacier erodes its valley by two processe
plucking and abrasion. By plucking the glacier freezes the joints and beds of the underlying rock: tears out
individual blocks and drags them away by abrasion, the glacier scratches, scrapes, polishes and scours the
valley floor with the debris frozen into it.
• Major glacial landforms are
• Cirque
o Cirques are the most common landforms in glaciated mountains. The cirques quite often are found at
the heads of glacial valleys. The accumulated ice cuts these cirques while moving down the mountain
tops. They are deep, long and wide troughs or basins with very steep concave to vertically drop high
walls at its head as well as sides.
• Horns and Serrated Ridges
o Horns form through head ward erosion of the cirque walls. If three or more radiating glaciers cut
headward until their cirques meet, high, sharp pointed and steep sided peaks called horns form. The
divides between cirque side walls or head walls get narrow because of progressive erosion and
turn into serrated or saw-toothed ridges sometimes referred to as aretes with very sharp crest and
a zig-zag outline.
• Bergschrund.
o At the head of a glacier, where it begins to leave the snowfield of a cirque, a deep vertical crack opens
up called a bergschrund (in German) or rimaye (in French). This happens in summer, although the ice
continues to move out of the cirque. There is no new snow to replace it. In some cases not one but
several such cracks occur.
• Hanging valleys.
o The main valley is eroded much more rapidly than the tributary valleys as it contains a much larger
glacier. After the ice has melted a tributary valley therefore hangs' above the main valley so that its
stream plunges down as a waterfall. Such tributary valleys are termed hanging valleys and may form a
natural head of water for generating hydro-electric power.
• Moraines
o Moraines are made up of the pieces of rock that are shattered by frost action, embedded in the glaciers
and brought down the valley. Those that fall on the sides of the glacier, mainly screes, form lateral
moraines. When two glaciers converge, their inside lateral moraines unite to form a medial moraine.
• Roche Moutonnee.
o This is a resistant residual rock hummock. The surface is striated by ice movement . Its upstream side
is smoothed by abrasion and its downstream side is roughened by plucking and is much steeper.
• Drumlins.
o These are swarms of oval, elongated “ whale-back' hummocks composed wholly of boulder clay, with
their elongation in the direction of the ice flow, that is on the downstream side. They are low' hills varying
from a few' yards to 400 feet in height and may be a mile or two long. They appear a little steeper at the
onset side and taper off at the leeward end.
• Hence option (a) is the correct answer.

Q 87.D
• Cities Coordinates
Jaipur : 26.9°N 75.8°E
Bhopal : 23°15′N 77°25′E
Chennai: 13°4′57″N 80°16′30″E
Bengaluru: 12°58′44″N 77°35′30″E

34 www.visionias.in ©Vision IAS

FREE BY KING R QUEEN P [ऋषभ राजपूत]


• Hence, option (d) is the correct answer.

Q 88.A
• Earthquake is a natural event. It is caused due to release of energy, which generates waves that travel in
all directions.
• The release of energy occurs along a fault. A fault is a sharp break in the crustal rocks. Rocks along a fault
tend to move in opposite directions. As the overlying rock strata press them, the friction locks them
together.
• However, their tendency to move apart at some point of time overcomes the friction. As a result, the
blocks get deformed and eventually, they slide past one another abruptly. This causes a release of energy,
and the energy waves travel in all directions.
• The point where the energy is released is called the focus of an earthquake, alternatively, it is called
the hypocentre. The energy waves travelling in different directions reach the surface. The point on the
surface, nearest to the focus, is called epicentre. It is the first one to experience the waves. It is a point
directly above the focus.
• All natural earthquakes take place in the lithosphere. Lithosphere refers to the portion of depth up to
200 km from the surface of the earth. Hence statement 1 is correct.
• Earthquake waves are basically of two types — body waves and surface waves. Body waves are generated
due to the release of energy at the focus and move in all directions travelling through the body of the
earth.
• The body waves interact with the surface rocks and generate new set of waves called surface waves.
These waves move along the surface. The velocity of waves changes as they travel through materials with
different densities. The denser the material, the higher is the velocity. Their direction also changes as they
reflect or refract when coming across materials with different densities.
35 www.visionias.in ©Vision IAS

FREE BY KING R QUEEN P [ऋषभ राजपूत]


o There are two types of body waves. They are called P and S-waves. P-waves move faster and are
the first to arrive at the surface. These are also called ‘primary waves’. The P-waves are similar
to sound waves. They travel through gaseous, liquid and solid materials.
o S-waves arrive at the surface with some time lag. These are called secondary waves. An important
fact about S-waves is that they can travel only through solid materials.
• The surface waves are the last to report on seismograph. These waves are more destructive. They
cause displacement of rocks, and hence, the collapse of structures occurs.
• Different types of earthquake waves travel in different manners. As they move or propagate, they cause
vibration in the body of the rocks through which they pass. P-waves vibrate parallel to the direction of the
wave.
o This exerts pressure on the material in the direction of the propagation. As a result, it creates density
differences in the material leading to stretching and squeezing of the material.
o Other three waves vibrate perpendicular to the direction of propagation. The direction of vibrations of
S-waves is perpendicular to the wave direction in the vertical plane. Hence, they create troughs and
crests in the material through which they pass.
o Surface waves are considered to be the most damaging waves. Hence statement 2 is not correct.

Q 89.C
• The Indus, also known as the Sindhu, is the westernmost of the Himalayan rivers in India. It
originates from a glacier near Bokhar Chu in the Tibetan region, at an altitude of 4,164 m in the
Kailash Mountain range. In Tibet, it is known as ‘Singi Khamban; or the Lion’s Mouth. Hence
statement 1 is correct.
• From Tibet, the river enters India through Union territory of Ladakh. From Ladakh, the river flows
through the Union Territories of Jammu and Kashmir. From here, the Indus river flows through the
regions of Baltistan and Gilgit. It cuts across the Ladakh range, forming a spectacular gorge near Gilgit in
Jammu and Kashmir. It then enters Pakistan near Chilas in the Dardistan region and flows through the
provinces of Punjab and Sindh before emptying into the Arabian Sea. Hence statement 3 is not correct.
• The Indus receives a number of tributaries, the most significant of which is the ‘Panjnad’. The Panjnad is
the name given to the five rivers of Punjab, namely the Satluj, the Beas, the Ravi, the Chenab and the
Jhelum.
• The Chenab is the largest tributary of the Indus. It is formed by two streams, the Chandra and the
Bhaga, which join at Tandi near Keylong in Himachal Pradesh. Hence, it is also known as Chandrabhaga.
The river flows for 1,180 km before entering into Pakistan. Hence statement 2 is not correct.

Q 90.C
• Arid soils range from red to brown in colour. They are generally sandy in structure and saline in nature.
o The sandy structure is due to the low levels of clay and organic matter in the soil, which results in
poor water holding capacity. The saline nature of the soil is due to the accumulation of salts in the
soil, which is a result of the high rates of evaporation and low rates of rainfall in these regions. In
some areas, the salt content is so high that common salt is obtained by evaporating the saline water.
• As a result, the soils are often alkaline in nature and require appropriate management practices such as
leaching, adding organic matter, and improving drainage to make them suitable for cultivation. Hence
statement 1 is correct.
• Due to the dry climate, high temperature and accelerated evaporation, they lack moisture and humus.
Nitrogen is insufficient and the phosphate content is normal.
• Lower horizons of the soil are occupied by ‘kankar’ layers because of the increasing calcium content
downwards. The ‘Kankar’ layer formation in the bottom horizons restricts the infiltration of water, and as
such, when irrigation is made available, the soil moisture is readily available for sustainable plant growth.
Arid soils are characteristically developed in western Rajasthan, which exhibits characteristic arid
topography. These soils are poor and contain little humus and organic matter. Hence statement 2 is
correct.
Q 91.A
• The months of October and November are known for retreating monsoons. By the end of September,
the southwest monsoon becomes weak as the low-pressure trough of the Ganga plain starts moving
southward in response to the southward march of the sun.
• The retreating southwest monsoon season is marked by clear skies and a rise in temperature. The
land is still moist. Owing to the conditions of high temperature and humidity, the weather becomes
rather oppressive. This is commonly known as the ‘October heat’. Hence, statements 1 and 2 are
correct but statement 3 is not correct.
36 www.visionias.in ©Vision IAS

FREE BY KING R QUEEN P [ऋषभ राजपूत]


Q 92.B
• Since the advent of the concept of sea floor spreading, the interest in the problem of distribution of oceans
and continents was revived. It was in 1967, McKenzie and Parker and also Morgan, independently
collected the available ideas and came out with another concept termed Plate Tectonics.
• A tectonic plate (also called lithospheric plate) is a massive, irregularly-shaped slab of solid rock,
generally composed of both continental and oceanic lithosphere. Plates move horizontally over the
asthenosphere as rigid units.
• The lithosphere includes the crust and top mantle with its thickness range varying between 5-100 km in
oceanic parts and about 200 km in the continental areas.
• A plate may be referred to as the continental plate or oceanic plate depending on which of the two occupy
a larger portion of the plate. Pacific plate is largely an oceanic plate whereas the Eurasian plate may be
called a continental plate.
• The theory of plate tectonics proposes that the earth’s lithosphere is divided into seven major and some
minor plates. Young Fold Mountain ridges, trenches, and/or faults surround these major plates. The major
plates are as follows
o Antarctica and the surrounding oceanic plate
o North American (with western Atlantic floor separated from the South American plate along the
Caribbean islands) plate
o South American (with western Atlantic floor separated from the North American plate along the
Caribbean islands) plate
o Pacific plate
o India-Australia-New Zealand plate
o Africa with the eastern Atlantic floor plate
o Eurasia and the adjacent oceanic plate.
• Some important minor plates are listed below:
o Cocos plate : Between Central America and Pacific plate
o Nazca plate : Between South America and Pacific plate
o Arabian plate : Mostly the Saudi Arabian landmass
o Philippine plate : Between the Asiatic and Pacific plate
o Caroline plate : Between the Philippine and Indian plate (North of New Guinea)
o Fuji plate : North-east of Australia

• Hence option (b) is the correct answer.

Q 93.A
• As compared to the western coastal plain, the eastern coastal plain is broader and is an example of
an emergent coast. There are well-developed deltas here, formed by the rivers flowing eastward into the
Bay of Bengal. These include the deltas of the Mahanadi, the Godavari, the Krishna, and the Kaveri.
• Because of its emergent nature, it has less number of ports and harbors. The continental shelf extends
up to 500 km into the sea, which makes it difficult the development of good ports and harbors.
• Hence, option (a) is the correct answer.
37 www.visionias.in ©Vision IAS

FREE BY KING R QUEEN P [ऋषभ राजपूत]


Q 94.C
• The cool temperate western margins are under the permanent influence of the Westerlies all round
the year. They are also regions of much cyclonic activity, typical of Britain, and are thus said to
experience the British type of climate. Hence statement 1 is correct.
• From Britain, the climatic belt stretches far inland into the lowlands of North-West Europe, including
such regions as northern and western France, Belgium, the Netherlands, Denmark, western Norway and
also north western Iberia .
o There is so much oceanic influence on both the temperature and the precipitation that the climate is
also referred to as the North-West European Maritime Climate.
• In North America, the high Rockies prevent the on-shore Westerlies from penetrating far inland and the
British type of climate is confined mainly to the coastlands of British Columbia.
o In the southern hemisphere, the climate is experienced in southern Chile, Tasmania and most parts of
New Zealand, particularly in South Island.
• The mean annual temperatures are usually between 40°F. and 60°F. The climate is ideal for maximum
comfort and mental alertness. People can work for long hours without feeling drowsy and lethargic as
they do in the tropics.
• The British type of climate has adequate rainfall throughout the year with a tendency towards a
slight winter or autumn maximum from cyclonic sources. Since the rain-bearing winds come from the
west, the western margins have the heaviest rainfall. The amount decreases eastwards with increasing
distance from the sea. Hence statement 2 is not correct.
• There are four distinct seasons in the British climate type. Light snowfalls can be expected in the
winter months normally only of short duration because of the comparatively mild weather. Hence
statement 3 is correct.
o Spring is the driest and the most refreshing season when people emerge from the depressing winter to
see everything becoming green again.
o This is followed by the long, sunny summer.
o With the roar of gusty winds and the fall of ‘golden’ leaves, autumn is ushered in and the cycle
repeats itself.
• This type of climate with its four distinct seasons is something that is conspicuously absent in the tropics.

Q 95.A
• Recent Context: Adani Enterprises calls off Follow on Public Offer (FPO), shares worth Rs 20,000
crore to be returned to investors
• Follow on Public Offer is an additional issue of a company, which is already listed on an exchange.
o Under FPO, the company issues new shares to the investors or the existing shareholders, usually the
promoters. Hence option (a) is the correct answer.
o A company uses FPO after it has gone through the process of an IPO (Initial Public Offering) to
diversify their equity base.
o An FPO is an additional issue whereas an IPO is an initial or first issue.
o Reasons to go for FPO
✓ To make more of its shares available to the public
✓ To raise capital to expand equity base
✓ To pay off debt.
• Initial public offering is the process by which a private company can go public by the sale of its
stocks to the general public. Companies can raise equity capital with the help of an IPO by issuing new
shares to the public or the existing shareholders can sell their shares to the public without raising any fresh
capital. Hence option (b) is not correct.
• Book building is the process by which an underwriter attempts to determine the price at which an
initial public offering (IPO) will be offered. SEBI guidelines define Book Building as "a process
undertaken by which a demand for the securities proposed to be issued by a body corporate is elicited and
built-up and the price for such securities is assessed for the determination of the quantum of such
securities to be issued by means of a notice, circular, advertisement, document or information memoranda
or offer document". Hence option (d) is not correct.
• A buyback, also known as a share repurchase, is when a company buys its own outstanding shares to
reduce the number of shares available on the open market. Under the stock exchange route, a
company can buy back shares only on the stock exchanges having nationwide trading terminals. The
buyback of shares is made only through the order-matching mechanism. In this method, the promoters, or
persons in control of a company are not allowed to participate. Hence option (c) is not correct.

38 www.visionias.in ©Vision IAS

FREE BY KING R QUEEN P [ऋषभ राजपूत]


Q 96.A
• The Peninsular drainage system is older than the Himalayan one. This is evident from the broad, largely-
graded shallow valleys, and the maturity of the rivers.
• Three major geological events in the distant past have shaped the present drainage systems of Peninsular
India:
o Subsidence of the western flank of the Peninsula leading to its submergence below the sea during the
early tertiary period. Generally, it has disturbed the symmetrical plan of the river on either side of the
original watershed.
o Upheaval of the Himalayas when the northern flank of the Peninsular block was subjected to
subsidence and the consequent trough faulting. The Narmada and The Tapi flow in trough faults and
fill the original cracks with their detritus materials. Hence, there is a lack of alluvial and deltaic
deposits in these rivers.
o A slight tilting of the Peninsular block from the northwest to the southeastern direction gave
orientation to the entire drainage system except (Narmada and Tapi) towards the Bay of Bengal
during the same period.
• Hence option (a) is the correct answer.
Q 97.D
• Recent Context: Russia claimed that the USA had not complied with the New START treaty's provisions
and had attempted to undermine Russia's national security.
• About New START treaty
o The New START (Strategic Arms Reduction Treaty) is a nuclear arms reduction treaty between
the United States and Russia (signed in 2010). It entered into force in 2011. Hence statement 1 is
not correct.
o It replaced the previous START treaty that had expired in 2009.
o It’s objective was to limit and reduce the number of strategic nuclear warheads and delivery systems
deployed by both countries.
o It aimed to enhance strategic stability and build mutual trust between the two nations.
• Under the agreement, both sides are committed to the following:
o Deploying no more than 1,550 strategic nuclear warheads and a maximum of 700 long-range
missiles and bombers. Hence statement 2 is not correct.
o A limit of 800 intercontinental ballistic missiles in deployment.
o Each side can conduct up to 18 inspections of strategic nuclear weapons sites yearly to ensure the
other has not breached the treaty’s limits.
o In 2021, United States and the Russian agreed to extend the treaty till 2026.
Q 98.A
• In India, the mangrove forests are highly developed in the Andaman and Nicobar Islands and the
Sunderbans of West Bengal. Other areas of significance are the Mahanadi, the Godavari and the Krishna
deltas. As per the Indian State of Forest Report 2021, mangrove cover in the states are as follows :
• Andhra Pradesh - 405 sq km
• Gujarat - 1175 sq km
• Maharashtra - 324 sq km
• Odisha - 259 sq km
• West Bengal - 2114 sq km
• Andaman & Nicobar Islands - 616 sq km

• Hence option (a) is the correct answer.


39 www.visionias.in ©Vision IAS

FREE BY KING R QUEEN P [ऋषभ राजपूत]


Q 99.D
• The Warm Temperate Western Margin (Mediterranean) Climate is caused due to shifting of wind
belts. They are entirely confined to the western portion of continental masses, between 30° and 45° north
and south of the equator.
• Characteristics of Mediterranean climate:
o a warm summer with off-shore trades
o a concentration of rainfall in winter with onshore westerlies
o bright, sunny weather with hot dry summers and wet, mild winters
o the prominence of local winds around the Mediterranean Sea (Sirocco, Mistral).
• Mediterranean regions include areas around the Mediterranean sea, California (around San Francisco), the
south-western tip of Africa (around Cape Town), southern Australia (in southern Victoria and around
Adelaide, bordering St. Vincent and Spencer Gulfs), and south-west Australia (Swanland).
• Hence, option (d) is the correct answer.

Q 100.C
• An earthquake in simple words is the shaking of the earth. It is caused due to release of energy, which
generates waves that travel in all directions. The release of energy occurs along a fault. A fault is a sharp
break in the crustal rocks. Rocks along a fault tend to move in opposite directions. As the overlying rock
strata press them, the friction locks them together.
• However, their tendency to move apart at some point in time overcomes the friction. As a result, the
blocks get deformed and eventually, they slide past one another abruptly. This causes the dissipation of
energy, and the energy waves travel in all directions.
• The point inside the crust where the pressure is released is called the focus. Hence statement 1 is
correct. It is also called the hypocentre.
• The point on the Earth's surface above the focus is called the epicenter. Earthquake energy is released
in seismic waves. These waves spread out from the focus. Hence statement 2 is not correct.
• Epicenter is the first one to experience the waves. Hence statement 3 is correct.
• The waves are felt most strongly at the epicenter, becoming less strong as they travel further away.

Copyright © by Vision IAS


All rights are reserved. No part of this document may be reproduced, stored in a retrieval system or
transmitted in any form or by any means, electronic, mechanical, photocopying, recording or otherwise,
without prior permission of Vision IAS.

40 www.visionias.in ©Vision IAS

FREE BY KING R QUEEN P [ऋषभ राजपूत]


VISIONIAS
www.visionias.in

Test Booklet Series

TEST BOOKLET

GENERAL STUDIES (P) 2024 – Test – 4129


C
Time Allowed: Two Hours Maximum Marks: 200

INSTRUCTIONS

1. IMMEDIATELY AFTER THE COMMENCEMENT OF THE EXAMINATION, YOU SHOULD CHECK THAT THIS BOOKLET
DOES NOT HAVE ANY UNPRINTED OR TURN OR MISSING PAGES OR ITEMS, ETC. IF SO, GET IT REPLACED BY A
COMPLETE TEST BOOKLET.

2. ENCODE CLEARLY THE TEST BOOKLET SERIES A, B, C OR D AS THE CASE MAY BE IN THE APPROPRIATE PLACE IN
THE ANSWER SHEET.

3. You have to enter your Roll Number on the Test Booklet in the Box
provided alongside. Do NOT write anything else on the Test Booklet.

4. This Test Booklet contains 100 items (Questions). Each item is printed in English. Each item comprises four
responses (answers). You will select the response which you want to mark on the Answer Sheet. In case you
feel that there is more than one correct response with you consider the best. In any case, choose ONLY ONE
response for each item.

5. You have to mark all your responses ONLY on the separate Answer Sheet provided. See direction in the
answers sheet.

6. All items carry equal marks. Attempt all items. Your total marks will depend only on the number of correct
responses marked by you in the answer sheet. For every incorrect response 1/3rdof the allotted marks will be
deducted.

7. Before you proceed to mark in the Answer sheet the response to various items in the Test booklet, you have to
fill in some particulars in the answer sheets as per instruction sent to you with your Admission Certificate.

8. After you have completed filling in all responses on the answer sheet and the examination has concluded, you
should hand over to Invigilator only the answer sheet. You are permitted to take away with you the Test
Booklet.

9. Sheet for rough work are appended in the Test Booklet at the end.

DO NOT OPEN THIS BOOKLET UNTIL YOU ARE ASKED TO DO SO


1 www.visionias.in ©Vision IAS

FREE BY KING R QUEEN P [ऋषभ राजपूत]


1. World's richest monazite (from which 4. With reference to the ancient site 'Keeladi'
Thorium is mainly obtained) deposits in recently seen in the news, consider the
India are located in following statements:

(a) Kerala 1. It is an urban civilization site of the


Sangam Era in Tamil Nadu.
(b) Gujarat
2. It is located along the Vaigai River.
(c) Assam
Which of the statements given above is/are
(d) Karnataka
correct?
(a) 1 only
2. Which of the following is/are the (b) 2 only
characteristics of Zaid Season? (c) Both 1 and 2
1. It is a short-duration summer cropping (d) Neither 1 nor 2
season beginning after the harvesting of
Rabi crops. 5. Consider the following statements with
2. Some of the crops produced during this reference to Dryland farming in India:
season are watermelon, muskmelon, 1. It is largely confined to regions having

cucumber, vegetables and fodder crops. an annual rainfall of less than 100 cm.
2. These regions grow hardy and drought-
3. This type of distinction in the cropping
resistant crops such as ragi, bajra,
season does not exist in southern parts of
moong, gram and guar.
the country.
Which of the statements given above is/are
Select the correct answer using the code
correct?
given below. (a) 1 only
(a) 2 only (b) 2 only
(b) 1 and 3 only (c) Both 1 and 2
(c) 2 and 3 only (d) Neither 1 nor 2
(d) 1, 2 and 3
6. Consider the following statements in regard
3. With reference to phases of population to the Great Backyard Bird Count (GBBC)
2023:
growth in India, consider the following
1. It is a community science project
statements:
conducted once every four years.
1. The decades 1901–1921 are referred to
2. "The Bird Count India" organizes the
as the period of stagnant growth.
GBBC in the country.
2. The decades 1921–1951 are referred to
3. India has recorded the world's third-
as the period of population explosion. highest number of bird species.
Which of the statements given above is/are Which of the statements given above are
correct? correct?
(a) 1 only (a) 1, 2 and 3
(b) 2 only (b) 1 and 2 only
(c) Both 1 and 2 (c) 2 and 3 only

(d) Neither 1 nor 2 (d) 1 and 3 only


2 www.visionias.in ©Vision IAS

FREE BY KING R QUEEN P [ऋषभ राजपूत]


7. Consider the following: 10. Consider the following statements:
1. Religious beliefs in Japan 1. Mixed farming is found in highly
2. The indented coastline of Japan
developed parts of the world.
3. Presence of a Laurentian type of climate
4. Meeting of warm Kuroshio and cold 2. The system in which the farmers
Oyashio currents near the coast specialize in vegetables is known as
How many of the factors given above have
truck farming.
positively contributed to the development of
fishing activity in Japan? 3. In the Kolkhoz model of farming, the
(a) Only one factor individual farmers held small plots to
(b) Only two factors
meet their daily requirements.
(c) Only three factors
(d) All four factors Which of the statements given above is/are
correct?
8. Consider the following statements regarding
(a) 1 and 2 only
the Hoysaleswara Temple of Halebidu:
1. It contains images with themes drawn (b) 2 only
from Jainism and Shaivism. (c) 1 and 3 only
2. It is a twin temple dedicated to
(d) 1, 2 and 3
Hoysaleswara and Santaleswara Shiva
lingams, named after the masculine and
feminine aspects respectively. 11. With reference to the distribution of
3. Recently, India nominated the temple for vegetation in the world, consider the
the UNESCO World Heritage Sites
following statements:
recognition.
Which of the statements given above are 1. Mediterranean regions are dominated by
correct? the presence of short trees with scanty
(a) 1 and 2 only
leaves.
(b) 2 and 3 only
(c) 1 and 3 only 2. The Savanna vegetation is termed as
(d) 1, 2 and 3 ‘selvas’ due to its presence of trees along
with grasses.
9. Consider the following statements with
respect to the adolescent population of India, 3. The transition of steppe vegetation is
according to the 2011 Census: characterized by the presence of trees
1. Adolescents constitute approximately
towards the poles and shorter grass
one fifth of the total population of the
country. towards the equator.
2. The number of male adolescents is Which of the statements given above is/are
higher than that of female adolescents.
correct?
Which of the statements given above is/are
correct? (a) 1 only
(a) 1 only (b) 1 and 3 only
(b) 2 only (c) 2 and 3 only
(c) Both 1 and 2
(d) 1, 2 and 3
(d) Neither 1 nor 2
3 www.visionias.in ©Vision IAS

FREE BY KING R QUEEN P [ऋषभ राजपूत]


12. With reference to the distribution of 16. Consider the following pairs:
petroleum in the world, consider the Place Coal Field
following statements: 1. China : North Antelope Rochelle
1. Crude petroleum occurs in metamorphic
2. Germany : Ruhr Valley
rocks of the tertiary period.
3. Australia : Bowen Basin
2. More than 90% of the world’s proven oil
4. Japan : Fushun
reserves are located in the Middle East.
Which of the statements given above is/are Which of the pairs given above are correctly
correct? matched?
(a) 1 only (a) 1 and 4 only
(b) 2 only (b) 2 and 3 only
(c) Both 1 and 2 (c) 1 and 2 only
(d) Neither 1 nor 2 (d) 3 and 4 only

13. Which of the following statements is not


17. In the context of human geography, which of
correct with reference to market gardening?
the following best defines environmental
(a) They specialized in the cultivation of
high-value crops like vegetables, fruits, determinism?
and flowers. (a) It is the belief that the physical
(b) It is done solely for the urban markets. environment or the earth in general did
(c) In such agriculture farms are large and not have a divine but an anthropogenic
are poorly connected with the urban origin.
centers. (b) It is the ability or role of nature or
(d) It is both labor and capital-intensive.
physical environment in guiding or
shaping human development.
14. With reference to Mediterranean type of
(c) It is the ability of man to decide what
climate the term 'haciendas’ refer to:
(a) large farm holdings. kind of surrounding or physical
(b) nutritious grasses. environment it needs.
(c) labour specialised in viticulture. (d) It is a theory that nature or the
(d) export quality oranges. environment is going to become extinct
very soon.
15. Arrange the following ports in India in a
North-South direction:
18. With reference to footloose industries, which
1. Kandla (Deendayal Port Authority)
one of the following statements is not
2. Vishakhapatnam
3. Marmagao correct?

4. Tuticorin (a) These industries are dependent on the


5. Mangaluru availability of component parts.
Select the correct answer using the code (b) These employ a large labour force.
given below. (c) Accessibility by road network is an
(a) 1-2-3-4-5 important factor in their location.
(b) 2-3-4-1-5
(d) These are generally not polluting
(c) 3-5-1-4-2
industries.
(d) 1-2-3-5-4
4 www.visionias.in ©Vision IAS

FREE BY KING R QUEEN P [ऋषभ राजपूत]


19. With reference to Tsetse flies recently seen 21. Consider the following pairs with reference
to pillars of human development:
in the news, consider the following Pillar Description
1. Equity : Equal access to
statements:
opportunities
1. The bite of an infected Tsetse fly can be available to
everybody
fatal to humans. 2. Empowerment : To have the power to
make choices
2. Tsetse flies are found only in Africa. 3. Sustainability : continuity in the
availability of
Which of the statements given above is/are
opportunities
correct? 4. Productivity : a measure of output
per unit of input.
(a) 1 only How many of the pairs given above is/are
correctly matched?
(b) 2 only (a) Only one pair
(b) Only two pairs
(c) Both 1 and 2
(c) Only three pairs
(d) All four pairs
(d) Neither 1 nor 2

22. Consider the following pairs:


Cities Countries
20. Consider the following statements: 1. Mariupol : Ukraine
2. Adana : Syria
1. Asia's first cross-country pipeline for oil
3. Riyadh : Iran
Which of the pairs given above is/are
transport was constructed by Oil India
correctly matched?
Limited in India. (a) 1 only
(b) 2 only
2. Hazira-Vijaipur-Jagdishpur cross- (c) 3 only
(d) 1, 2 and 3
country gas pipeline connects Mumbai
23. Consider the following statements with
High with various industrial complexes
reference to the Border Road Organization
in western and northern India. (BRO):
1. It was established after the India-China
Which of the statements given above is/are War in 1962.
2. The Atal Tunnel which connects Manali
correct? to Lahaul-Spiti valley throughout the
year was built by the BRO.
(a) 1 only
Which of the statements given above is/are
(b) 2 only correct?
(a) 1 only
(c) Both 1 and 2 (b) 2 only
(c) Both 1 and 2
(d) Neither 1 nor 2 (d) Neither 1 nor 2
5 www.visionias.in ©Vision IAS

FREE BY KING R QUEEN P [ऋषभ राजपूत]


24. The Intensive subsistence agriculture 28. Consider the following pairs:
dominated by wet paddy cultivation is Waterways Drains into
generally characterized by: 1. Rhine : North Sea
1. Use of manure to maintain fertility
2. Volga Waterway : Black sea
2. High labor productivity
3. Mississippi-Ohio : Gulf of Mexico
3. Large scale use of machinery
waterway
4. High yield per unit area
Select the correct answer using the code Which of the pairs given above is/are
given below. correctly matched?
(a) 1 and 4 only (a) 1, 2 and 3
(b) 2, 3 and 4 only (b) 1 and 3 only
(c) 1, 2 and 3 only (c) 1 only
(d) 1, 2, 3 and 4
(d) 2 and 3 only

25. Bharat SHRI deals with which of the


29. Consider the following statements about
following?
(a) Digitization of inscriptions Yakshagana:
(b) Preservation of Wetlands 1. It is a traditional folk dance popular in
(c) Revision of prize money for Bharat the coastal districts of Tamil Nadu.
Ratna awardees 2. It is a temple art form that depicts
(d) Setting up of Textile Parks mythological stories from the puranas.
3. Large-sized headgear, colored faces, and
26. Pastoral nomadism is practiced in which of
elaborate costumes are common features
the following regions?
1. Himalayas of this dance.
2. Tundra region Which of the statements given above are
3. Atlantic shores of North Africa correct?
4. Island of Madagascar (a) 1 and 2 only
Select the correct answer using the code (b) 2 and 3 only
given below. (c) 1 and 3 only
(a) 1 and 2 only
(d) 1, 2 and 3
(b) 2 only
(c) 3 and 4 only
(d) 1, 2, 3 and 4 30. Consider the following statements regarding
Lignite:
27. Consider the following: 1. It is high-grade coal with less moisture
1. Natural disaster content.
2. High wage 2. Around 90 % of its deposits are found in
3. Pleasant climate Tamil Nadu.
4. Political turmoil
Which of the statements given above is/are
Which of the above is/are pull factors for
correct?
migration?
(a) 1 only (a) 1 only
(b) 2 and 3 only (b) 2 only
(c) 1 and 3 only (c) Both 1 and 2
(d) 1, 2 and 4 only (d) Neither 1 nor 2
6 www.visionias.in ©Vision IAS

FREE BY KING R QUEEN P [ऋषभ राजपूत]


31. Consider the following statements with 34. Consider the following pairs:
reference to the Maasai tribe of Kenya: Mineral Generally occurs in the
1. The Maasai tribe is a nomadic tribe who form
once wandered with their herds of cattle
1. Copper : Veins and Lodes
in the central highlands of East Africa.
2. Coal : Sedimentary Material
2. The cattle reared by them are Zebu cattle
3. Bauxite : Weathered Material
with humps and long horns.
3. They build circular huts with sticks, Which of the pairs given above is/are
bushes and mud. correctly matched?
Which of the statements given above are (a) 2 only
correct? (b) 1 and 2 only
(a) 1 and 2 only (c) 1 and 3 only
(b) 2 and 3 only
(d) 1, 2 and 3
(c) 1 and 3 only
(d) 1, 2 and 3
35. Consider the following statements with
32. Consider the following statements regarding reference to general demography:
palm oil: 1. In India, the sex ratio can be defined as
1. Andhra Pradesh is the largest palm oil- the number of females per thousand
producing state in India. males.
2. India is the largest importer of palm oil 2. In India, the literacy rate denotes the
in the world.
percentage of the population above 5
Which of the statements given above is/are
years of age, who can read, and write
correct?
(a) 1 only with understanding.
(b) 2 only Which of the statements given above is/are
(c) Both 1 and 2 correct?
(d) Neither 1 nor 2 (a) 1 only
(b) 2 only
33. In the context of the age-sex population
(c) Both 1 and 2
pyramid, consider the following graph. It
(d) Neither 1 nor 2
represents which of the following country’s
population demographic?
36. Consider the following pairs:
Region in the news Country
1. Odesa : Ukraine
2. Mount Aso : Philippines
3. Nicosia : Cyprus
How many pairs given above are correctly
matched?

(a) Nigeria (a) Only one pair


(b) Australia (b) Only two pairs
(c) Mexico (c) All the three pairs
(d) Bangladesh (d) None of the pairs
7 www.visionias.in ©Vision IAS

FREE BY KING R QUEEN P [ऋषभ राजपूत]


37. “SIGHT fund” often seen in the news is 41. Consider the following pairs:
associated with which of the following? Shifting Practised in the region
(a) Group of 20 (G-20)
cultivation
(b) National Green Hydrogen Mission
(c) National Solar Mission 1. Milpa : Indonesia
(d) Indian Space Research Organization 2. Ladang : Central America
(ISRO)
3. Jhuming : North Eastern India

4. Taungya : Sri Lanka


38. Which of the following tribes undertake
transhumance? How many pairs given above is/are correctly
1. Bakarwals matched?
2. Gaddis
(a) Only one pair
3. Toda
4. Bhotiyas (b) Only two pairs
Select the correct answer using the code (c) Only three pairs
given below.
(d) All four pairs
(a) 1, 2 and 4 only
(b) 1, 3 and 4 only
(c) 2 and 3 only 42. These ports are exclusively concerned with
(d) 2, 3 and 4 only the transportation of passengers and mail

across water bodies covering short distances.


39. Chennai is often called the 'Detroit of India'.
In this context, Detroit is famous for and has They occur in pairs located in such a way

in common with Chennai which of the that they face each other across the water
following industries? body.
(a) Silk production
Which of the following types of ports is best
(b) Automobile
(c) Shipbuilding described by the passage given above?
(d) Diamond processing (a) Entrepot Ports

(b) Ports of Call


40. Arrange the following dams/reservoirs from
(c) Out Ports
North to South:
1. Nagarjuna Sagar (d) Packet Station
2. Mettur
3. Tehri
43. Recently seen in the news Agasthyarkoodam
4. Gandhi Sagar
Select the correct answer using the code Observatory is situated in:
given below. (a) Telangana
(a) 3-4-2-1
(b) Kerala
(b) 4-3-2-1
(c) Andhra Pradesh
(c) 3-4-1-2
(d) 4-2-3-1 (d) Karnataka
8 www.visionias.in ©Vision IAS

FREE BY KING R QUEEN P [ऋषभ राजपूत]


44. Which of the following is referred to in the 47. Which of the following factors causes an
below-given map? increase in the population density of a

particular place?

1. Economic opportunities

2. Fertile soil

3. Emigration

Select the correct answer using the code

given below.

(a) 1 and 2 only


(a) Major industrial regions
(b) Minor industrial regions (b) 2 only

(c) IT hubs of India (c) 1 and 3 only


(d) Major mining areas (d) 1, 2 and 3

45. Consider the following pairs:


48. In the context of Indian economy, which one
Region Tribes
of the following migration streams
1. Malaysia : Orang Asli
2. Amazon basin : Indian tribes represents maximum share of migrants in the

3. Congo basin : Pygmies intra state migration as per census 2011?


Which of the pairs given above is/are (a) Rural to Rural male migration
correctly matched?
(b) Rural to Urban male migration
(a) 1, 2 and 3
(c) Rural to Rural female migration
(b) 1 and 3 only
(c) 1 only (d) Rural to Urban female migration

(d) 2 and 3 only


49. Arrange the following continents in
46. Which of the following industries are decreasing order of their population density.
affected by weight-losing raw materials?
1. Asia
1. Cotton Textile Industry
2. Europe
2. Iron and Steel Industry
3. Sugar Industry 3. North America

4. Computer Chip industry Select the correct answer using the code
Select the correct answer using the code given below.
given below.
(a) 2-1-3
(a) 1 and 2 only
(b) 1-2-3
(b) 2 and 3 only
(c) 3 and 4 only (c) 3-2-1

(d) 1 and 4 only (d) 1-3-2


9 www.visionias.in ©Vision IAS

FREE BY KING R QUEEN P [ऋषभ राजपूत]


50. In the context of urban settlements, which of 53. With reference to coal, consider the
the following statements best describes following statements:
'conurbation'? 1. The USA has largest share of
(a) It is the urban development that results recoverable coal reserves in the world.
in the increase in pollution level and
2. China is the top coal producing country
degradation in the quality of life.
in the world.
(b) It is the gradual deurbanaziation
3. South Africa is the top coal exporting
associated with the disappearance of
country in the world.
modern infrastructure.
(c) It is a large area of urban development Which of the statements given above are
that resulted from the merging of correct?
originally separate towns or cities. (a) 1 and 2 only
(d) It is the urban development associated (b) 2 and 3 only
with the development of green (c) 1 and 3 only
infrastructure. (d) 1, 2 and 3

51. Consider the following pairs:


54. Consider the following international
Iron ore Mine State
commercial centers/cities:
1. Gurumahisani : West Bengal
1. San Francisco
2. Noamundi : Jharkhand
3. Badampahar : Odisha 2. Vladivostok

Which of the pairs given above is/are 3. Wuhan


correctly matched? 4. Melbourne
(a) 3 only How many of the cities given above are
(b) 1 and 2 only located on the sea coasts?
(c) 2 and 3 only (a) Only one city
(d) 1, 2 and 3 (b) Only two cities
(c) Only three cities
52. Which one of the following statements about
(d) All four cities
the Sino-Tibetan family of languages spoken
in India is not correct?
55. Consider the following pairs:
(a) The Sino-Tibetan family of languages
are mostly spoken by the tribal groups in River Origin Country
India. 1. Rhine : Poland
(b) The Sino-Tibetan family of languages 2. Danube : Germany
has Tibeto-Myanmari and Siamese- 3. Volga : Russia
Chinese as its two main subfamilies. Which of the pairs given above are correctly
(c) The Tibeto-Himalayan group of matched?
languages is spoken only by the tribes (a) 1 and 2 only
living in Ladakh and Sikkim.
(b) 2 and 3 only
(d) The North Assam language speakers are
(c) 1 and 3 only
found in the hilly areas of Arunachal
(d) 1, 2, and 3
Pradesh.
10 www.visionias.in ©Vision IAS

FREE BY KING R QUEEN P [ऋषभ राजपूत]


56. Recently, the term “MISHTI” was in news 59. Ministry of Health and Family Welfare
with regard to: (MoH&FW) has recently introduced changes
(a) It is an initiative of Food Safety and to the National Organ Transplantation
Standards Authority of India to curb the
Guidelines. In this context, consider the
use of processed sugar.
following statements regarding the new
(b) It is a scheme by the Ministry of Health
and Family Welfare to raise awareness guidelines for organ transplantation in India:
about type-2 diabetes. 1. A citizen can now register for organ
(c) It is a mission launched for mangrove donation in any state without any
preservation and conservation. domicile requirement.
(d) It is a project to identify and track
2. The new guidelines allow even those
marsupials in India.
above 65 years of age to register to

57. With reference to the Securities and receive an organ for transplantation from
Exchange Board of India’s recent deceased donors.
notification on green debt securities, 3. There is now, no registration fees
consider the following statements: required to register for organ
1. YES Bank issued the first green bond for
transplantation.
financing renewable and clean energy
Which of the statements given above are
projects.
2. Yellow bonds and blue bonds will be correct?
raised for thermal energy generation and (a) 1 and 2 only
sustainable maritime sector respectively. (b) 2 and 3 only
Which of the statements given above is/are (c) 1 and 3 only
correct?
(d) 1, 2 and 3
(a) 1 only
(b) 2 only
(c) Both 1 and 2 60. Recently, NSE Indices Ltd., an arm of
(d) Neither 1 nor 2 National Stock Exchange has introduced the
country's first ever municipal bond index. In
58. It is a food as well as a fodder crop grown this context, consider the following
under semi-arid climatic conditions and over
statements regarding the new Nifty India
inferior soils. It requires 50-100 cm of
Municipal Bond index:
rainfall and a temperature ranging from 21°C
to 27°C. It is sown all over India except in 1. It will track the performance of
eastern and northeastern regions. The municipal bonds issued by municipal
leading producers are the states of Madhya corporations in India.
Pradesh, Andhra Pradesh, Karnataka, 2. The index will be reviewed quarterly.
Rajasthan, and Uttar Pradesh.
Which of the statements given above is/are
Identify the crop mentioned in the above
correct?
description.
(a) Wheat (a) 1 only
(b) Maize (b) 2 only
(c) Bajra (c) Both 1 and 2
(d) Sorghum (d) Neither 1 nor 2
11 www.visionias.in ©Vision IAS

FREE BY KING R QUEEN P [ऋषभ राजपूत]


61. Which of the following variable(s) is/are 64. Arrange the following categories of land use
displayed in population pyramids? in descending order as per their reporting
1. Birth rate area in India:
2. Sex indicators (male and female) 1. Barren and Waste Land

3. Death rate 2. Net Sown Area

4. Age-groups 3. Forest
Select the correct answer using the code
Select the correct answer using the code
given below.
given below:
(a) 2-3-1
(a) 1, 2 and 3
(b) 1-2-3
(b) 2 and 4 only
(c) 2-1-3
(c) 1, 3 and 4 only
(d) 3-2-1
(d) 2, 3 and 4 only

65. Consider the following statements:


62. Consider the following pairs: 1. Geographical area is the total area
Iron and steel Located in reported by the Land Revenue
centre department.
1. Ruhr region : Germany 2. Reporting area is the total area as
2. Duluth : France measured by the Survey of India.
3. Port Talbot : United Kingdom Which of the statements given above is/are
4. Krivoi Rog : Ukraine not correct?
How many pairs given above are correctly (a) 1 only
matched? (b) 2 only

(a) Only one Pair (c) Both 1 and 2

(b) Only two Pairs (d) Neither 1 nor 2

(c) Only three Pairs


66. With reference to the concept of growth and
(d) All four Pairs
development, consider the following
statements:
63. Consider the following statements with
1. Development is a qualitative change that
respect to the Footloose industries:
is always value positive.
1. They are not dependent on any specific
2. Growth is value-neutral; the change may
raw material.
be either positive or negative.
2. They are produced on large scale and 3. Population growth does not always lead
employ a large labor force. to development.
Which of the statements given above is/are Which of the statements given above is/are
correct? correct?
(a) 1 only (a) 1 only
(b) 2 only (b) 1, 2 and 3
(c) Both 1 and 2 (c) 1 and 2 only
(d) Neither 1 nor 2 (d) 2 only
12 www.visionias.in ©Vision IAS

FREE BY KING R QUEEN P [ऋषभ राजपूत]


67. Which of the following factors influence 70. Consider the following statements with
population distribution? regard to the sub-urbanization:
1. Landforms 1. It is a new trend of people moving away
2. Climate from congested urban areas to cleaner
3. Availability of water areas outside the city.
4. Minerals distribution 2. It generally develops around smaller
5. Urbanization cities.
Select the correct answer using the code Which of the statements given above is/are
given below. correct?
(a) 1 and 2 only (a) 1 only
(b) 3 and 4 only (b) 2 only
(c) 1, 2, 3 and 4 only (c) Both 1 and 2
(d) 1, 2, 3, 4 and 5 (d) Neither 1 nor 2

68. Consider the following statements with 71. Consider the following statements with
regard to the shapes of the settlements: reference to inland waterways:
1. Rectangular patterns of rural settlements 1. The Rhine waterway connects the
are found in plain areas or wide inter industrial areas of Switzerland and
montane valleys. Germany with the North Atlantic Sea
2. T -shaped settlements develop at tri- Route.
junctions of the roads. 2. The Mississippi-Ohio waterway
3. Cruciform settlements develop on the connects the interior part of the U.S.A.
cross-roads and houses extend in all the
with the Gulf of Mexico.
four direction.
Which of the statements given above is/are
Which of the statements given above are
not correct?
correct?
(a) 1 only
(a) 1 and 2 only
(b) 2 only
(b) 2 and 3 only
(c) Both 1 and 2
(c) 1 and 3 only
(d) Neither 1 nor 2
(d) 1, 2 and 3

72. With reference to the steel sector, consider


69. Consider the following statements with
the following statements:
regard to Coffee production in India:
1. India is the world's second-largest
1. It is cultivated in the highlands of
producer of crude steel.
Western Ghats in Karnataka, Kerala and
2. Steel Authority of India Limited (SAIL)
Tamil Nadu.
is the largest steel-making company in
2. India mostly grows superior quality
India.
Robusta coffee.
3. India does not have a dedicated National
3. India is the second-largest coffee-
Steel Policy yet.
producing country in the world.
Which of the statements given above are
Which of the statements given above is/are
correct?
correct?
(a) 1 only (a) 1 and 2 only
(b) 1 and 2 only (b) 2 and 3 only
(c) 2 and 3 only (c) 1 and 3 only
(d) 1, 2 and 3 (d) 1, 2 and 3
13 www.visionias.in ©Vision IAS

FREE BY KING R QUEEN P [ऋषभ राजपूत]


73. In these settlements, houses are spaced far 76. Which of the following are components of a
apart and often interspersed with fields. A population change?
cultural feature such as a place of worship or 1. Birth
a market, binds the settlement together. 2. Death
The above-mentioned passage refers to 3. Morbidity
which one of the following settlements? 4. Migration
(a) Dispersed Settlement Select the correct answer using the code
(b) Nucleated settlement given below.
(c) T-shaped settlement (a) 1, 2 and 4 only
(d) Rectangular settlement (b) 2 and 3 only

(c) 3 and 4 only


74. Which of the following states do not have a
(d) 1, 2, 3 and 4
nuclear power plant?
(a) Rajasthan
77. The quaternary sector mainly involves which
(b) Maharashtra
of the following activities/ occupations?
(c) Odisha
1. Personnel working in elementary
(d) Tamil Nadu
schools

2. Gold collar professions


75. With reference to land use criteria in India,
3. Home shoring
consider the following statements:
4. Accounting and brokerage firms
1. Current Fallow lands are those areas that
Select the correct answer using the code
are left without cultivation for more than
given below.
one year but less than five years.
(a) 1, 3 and 4 only
2. If the land is left uncultivated for more
(b) 2 and 3 only
than five years, it is categorized as
(c) 1 and 4 only
Barren and Wastelands.
(d) 1, 2, 3 and 4
3. Culturable wastelands can be brought
under cultivation with available
78. Among the following states, which one of
technology.
the following has the lowest percentage of
Which of the statements given above is/are
urban population (Census 2011)?
correct?
(a) Mizoram
(a) 1 and 2 only
(b) 2 and 3 only (b) Gujarat

(c) 3 only (c) Maharashtra

(d) 1 and 3 only (d) Goa


14 www.visionias.in ©Vision IAS

FREE BY KING R QUEEN P [ऋषभ राजपूत]


79. Which of the following statements is correct 82. Arrange the following capital cities in
regarding the Human Development Report Europe from West to East.
and the Human Development Index (HDI)? 1. Madrid
1. The Human Development Report is 2. Lisbon
annually published by the United 3. Vienna
Nations Development Programme. 4. Brussels
2. The HDI was created in 1990 by the Select the correct answer using the code
economist John Nash. given below.
3. Countries with very high HDI are all (a) 1-2-4-3
those which have a score of over 0.600.
(b) 1-2-3-4
Select the correct answer using the code
(c) 2-1-3-4
given below.
(d) 2-1-4-3
(a) 1 only
(b) 1 and 2 only
83. Consider the following statements with
(c) 1, 2 and 3
respect to Indian ports:
(d) 2 only
1. West coast has more seaports than the
east coast.
80. Broadly both fertility and mortality are high
2. Kochchi Port is situated at the head of
in which one of the following stages of
Vembanad Kayal.
demographic transition theory?
3. Haldia Port is situated in the Mahanadi
(a) Early expanding stage
delta.
(b) Low fluctuating stage
Which of the statements given above is/are
(c) Late expanding stage
correct?
(d) High fluctuating stage
(a) 1 only
(b) 1 and 2 only
81. Consider the following pairs:
Steel centers Country (c) 2 and 3 only

1. Donetsk : Ukraine (d) 1, 2 and 3

2. Pittsburg : Russia
3. Tienstin : China 84. Which of the following countries are

Which of the pairs given above is/are involved in the biennial trilateral coast guard

correctly matched? exercise ‘Dosti'?

(a) 1, 2 and 3 (a) India, Thailand and Myanmar

(b) 1 and 3 only (b) India, Bangladesh and Indonesia


(c) 1 only (c) India, Somalia and Iran
(d) 2 and 3 only (d) India, Sri Lanka and Maldives
15 www.visionias.in ©Vision IAS

FREE BY KING R QUEEN P [ऋषभ राजपूत]


85. With reference to Tundra vegetation, 88. Consider the following statements with
consider the following statements: regard to the rural settlements in India:
1. The deficiency of heat has resulted in the 1. Clustered Settlements are generally
permanent absence of mammals in the found in fertile alluvial plains and in the
Tundra region.
northeastern states.
2. In terms of vegetation, it is characterized
2. Semi-Clustered Settlements are
by the presence of mosses, lichens and
widespread in the Gujarat plains and
sedges.
Which of the statements given above is/are some parts of Rajasthan.

correct? Which of the statements given above is/are


(a) 1 only correct?
(b) 2 only (a) 1 only
(c) Both 1 and 2 (b) 2 only
(d) Neither 1 nor 2 (c) Both 1 and 2
(d) Neither 1 nor 2
86. Consider the following statements regarding
the railway network in India:
89. What is the aim of the Bhashini Mission?
1. India has more meter gauge tracks than
(a) To promote the recruitment of regional
Broad gauge tracks.
2. Narrow gauge lines are generally language teachers in the primary schools

confined to hilly areas. (b) To aid in local language translation


Which of the statements given above is/are (c) To improve optical fibre connectivity
correct? (d) To use satellite information for disaster
(a) 1 only management
(b) 2 only
(c) Both 1 and 2
90. Consider the following statements about
(d) Neither 1 nor 2
'short selling', a term often seen in the news:
1. It is the selling of stock and buying it
87. Consider the following statements regarding
later at a discounted price.
the NASA-ISRO Synthetic Aperture Radar
(NISAR) satellite, recently seen in the news: 2. Such stocks need to be owned by the

1. It is a low earth orbit (LEO) observatory seller before the selling takes place.
that will map the entire globe. 3. Hindenburg Research LLC is an
2. It is a dual-frequency (L-band and S- investment research firm with a focus on
band synthetic aperture radar) imaging activist short-selling.
radar satellite. Which of the statements given above is/are
Which of the statements given above is/are
correct?
correct?
(a) 1 only
(a) 1 only
(b) 2 only
(b) 2 only
(c) 3 only
(c) Both 1 and 2
(d) Neither 1 nor 2 (d) 1 and 3 only
16 www.visionias.in ©Vision IAS

FREE BY KING R QUEEN P [ऋषभ राजपूत]


91. Which of the following countries are 94. Consider the following pairs with respect to
bordered by Russia? ‘slash-and-burn agriculture:
1. Lithuania Agriculture Practice State
2. Latvia 1. Bewar : Madhya Pradesh
3. Estonia 2. Podu : Andhra Pradesh
4. Kyrgyzstan 3. Bringa : Odisha
5. Finland 4. Kuruwa : Jharkhand
Select the correct answer using the code How many pairs given above are correctly
given below. matched?
(a) 1, 2 and 3 only (a) Only one pair
(b) 1, 2, 3 and 5 only (b) Only two pairs
(c) 4 and 5 only (c) Only three pairs
(d) 1, 2, 3 and 4 only (d) All the four pairs

92. Arrange the following states/ UTs of India in 95. Consider the following statements:
the decreasing order of population density 1. It is under the permanent influence of
(Census 2011): the Westerlies all around the year.
1. Bihar 2. The natural vegetation of this climate
2. West Bengal type is a deciduous forest.
3. Uttar Pradesh 3. Market gardening is one of the highly
4. Arunachal Pradesh developed activities of the people of this
Select the correct answer using the code region.
given below. Which of the following types of climate is
(a) 3-1-4-2 described in the statements given above?
(b) 2-1-3-4 (a) Mediterranean Climate
(c) 1-3-2-4 (b) Steppe Climate
(d) 1-2-3-4 (c) British Type
(d) Siberian Climate
93. Consider the following pairs:
Inland Rivers system 96. Consider the following statements with
Waterways reference to the population of the world:
1. National : Ganga-Bhagirathi- 1. As per the United Nations, the world
Waterway 1 Hooghly River system population reached the 1 billion mark in
2. National : Brahmaputra River 1500 AD.
Waterway 2 2. It took less than 60 years for the world
3. National : Krishna River population to double from 1 billion to 2
Waterway 3 billion.
Which of the pairs given above is/are Which of the statements given above is/are
correctly matched? correct?
(a) 1 and 2 only (a) 1 only
(b) 3 only (b) 2 only
(c) 1, 2 and 3 (c) Both 1 and 2
(d) None (d) Neither 1 nor 2
17 www.visionias.in ©Vision IAS

FREE BY KING R QUEEN P [ऋषभ राजपूत]


97. Bharmaur is inhabited by ‘Gaddi’, a tribal 100. The Suez canal connects:
community who have maintained a distinct (a) Red Sea and the Persian gulf
identity as they practice transhumance and (b) Mediterranean Sea and the Red Sea
converse through the Gaddiali dialect.
(c) Caspian sea and the Mediterranean Sea
In which of the following states is the Gaddi
(d) Black sea and the Red sea
tribe found?
(a) Rajasthan
(b) Madhya Pradesh
(c) Gujarat
(d) Himachal Pradesh

98. Consider the following statements regarding


water resources in India:
1. India accounts for about 4 percent of the
world’s water resources.
2. The total water available in a year from
precipitation in India is relatively more
than water available from surface water.
Which of the statements given above is/are
correct?
(a) 1 only
(b) 2 only
(c) Both 1 and 2
(d) Neither 1 nor 2

99. Consider the following pairs:


Famous Mining Located in Desert
towns/regions
1. Chuquicamata : Kalahari Desert
2. Kalgoorlie : Great Australian
Desert
3. Utah : Nevada Desert
Which of the pairs given above is/are
correctly matched?
(a) 1 and 2 only
(b) 2 and 3 only
(c) 3 only
(d) 1 and 3 only

Copyright © by Vision IAS


All rights are reserved. No part of this document may be reproduced, stored in a retrieval system or
transmitted in any form or by any means, electronic, mechanical, photocopying, recording or otherwise,
without prior permission of Vision IAS.
18 www.visionias.in ©Vision IAS

FREE BY KING R QUEEN P [ऋषभ राजपूत]


VISIONIAS
www.visionias.in
ANSWERS & EXPLANATIONS
GENERAL STUDIES (P) TEST – 4129 (2024)

Q 1.A
• The world's richest monazite deposits are found in the coastal regions of India, particularly in the state
of Kerala. The monazite sands along the coast of Kerala are estimated to contain about 15% of the
world's monazite reserves. Hence option (a) is the correct answer.
• India has one of the world's largest reserves of monazite, with deposits estimated to be around 12 million
tonnes. However, the use of monazite in India is largely restricted due to its radioactive content, as
thorium is a radioactive element.
• About Monazite:
o Monazite is a rare earth element that is typically found in association with other minerals such as
ilmenite, rutile, and zircon.
o It is valued for its high concentrations of thorium and rare earth metals, which are used in a variety
of industrial applications, including the production of nuclear fuel, superconductors, and
permanent magnets.
o It is composed mainly of the elements cerium, lanthanum, neodymium, and thorium.
• In India, monazite is primarily used for the extraction of thorium, which is an important fuel for
nuclear reactors. India has a significant nuclear energy program, and thorium-based reactors are being
developed as a safer and more efficient alternative to traditional uranium-based reactors. Other countries
with significant monazite deposits include Brazil, Australia, Malaysia, Vietnam, and South Africa.

Q 2.D
• Zaid is a short-duration summer cropping season beginning after the harvesting of Rabi crops. Hence,
statement 1 is correct.
• In between the rabi and the kharif seasons, there is a short season during the summer months known as the
Zaid season. Some of the crops produced during ‘zaid’ are watermelon, muskmelon, cucumber,
vegetables, and fodder crops. Sugarcane takes almost a year to grow. Hence, statement 2 is correct.
• The Zaid crops require warm dry weather for major growth periods and longer day length for flowering
and fruiting. However, this type of distinction in the cropping season does not exist in southern parts of
the country. Hence, statement 3 is correct.

Q 3.A
• The growth rate of population in India over the last century has been caused by the annual birth rate, death
rate, and rate of migration, and thereby shows different trends. There are four distinct phases of growth
identified within this period:
• Phase I: The period from 1901-1921 is referred to as a period of stagnant or stationary phase of growth of
India’s population, since in this period the growth rate was very low, even recording a negative growth
rate during 1911-1921. Both the birth rate and death rate were high, keeping the rate of increase low. Poor
health and medical services, the widespread illiteracy of the population, and an inefficient distribution
system for food and other basic necessities were largely responsible for the high birth and death rates in
this period. Hence statement 1 is correct.
• Phase II: The decades 1921-1951 are referred to as the period of steady population growth. An
overall improvement in health and sanitation throughout the country brought down the mortality rate. At
the same time, better transport and communication systems improved the distribution system. The crude
birth rate remained high in this period, leading to a higher growth rate than the previous phase. This is
impressive against the backdrop of the Great Depression,1920s and World War II. Hence statement 2 is
not correct.
1 www.visionias.in ©Vision IAS

FREE BY KING R QUEEN P [ऋषभ राजपूत]


• Phase III: The decades 1951-1981 are referred to as the period of population explosion in India,
which was caused by a rapid fall in the mortality rate but a high fertility rate of the population in the
country. The average annual growth rate was as high as 2.2 percent. It is in this period, after the
Independence, that developmental activities were introduced through a centralised planning process and
the economy started showing up, ensuring the improvement of living conditions for the general
population. Consequently, there was a high natural increase and a higher growth rate. Besides, increased
international migration bringing in Tibetans, Bangladeshis, Nepalies and even people from Pakistan
contributed to the high growth rate.
• Phase IV: From 1981 until present, the growth rate of the country’s population though remained high, has
started slowing down gradually. A downward trend in the crude birth rate is held responsible for such
population growth. This was, in turn, affected by an increase in the mean age at marriage, improved
quality of life particularly education of females in the country.

Q 4.C
• The Keeladi excavation site, located along the Vaigai River, is an ancient settlement dating back to
the Sangam age. It is currently under excavation by the Tamil Nadu State Department of Archaeology
and the Archaeological Survey of India. Hence statement 2 is correct.
• Located about 12 km southeast of the temple city of Madurai, the site has revealed evidence that supports
the existence of an urban civilization during the Sangam age providing valuable insights into the ancient
history of Tamil Nadu. Hence statement 1 is correct.
• The Keeladi excavation site holds great significance as it has helped to push back the timeline of the
Sangam age to 800 BCE based on the archaeological findings. This is a significant discovery as it
provides new insights into the ancient history of Tamil Nadu.
• Moreover, the excavation site has the potential to provide vital evidence that can help to bridge the gap
between the Iron Age (12th century BCE to 6th century BCE) and the Early Historic Period (6th
century BCE to 4th century BCE). The findings from the Keeladi site can help shed light on the cultural
developments and societal changes that occurred during this critical period in ancient Tamil Nadu's
history.

Q 5.B
• On the basis of the main source of moisture for crops, farming can be classified as irrigated and rainfed
(barani).
• Rainfed farming is further classified on the basis of the adequacy of soil moisture during the cropping
season into dryland and wetland farming. In India, Dryland farming is largely confined to regions having
an annual rainfall of less than 75 cm. Hence, statement 1 is not correct.
• These regions grow hardy and drought-resistant crops such as ragi, bajra, moong, gram and guar
(fodder crops) and practice various measures of soil moisture conservation and rainwater harvesting.
Hence, statement 2 is correct.
• In Wetland farming, the rainfall is in excess of the soil moisture requirement of plants during the rainy
season. Such regions may face flood and soil erosion hazards. These areas grow various water-intensive
crops such as rice, jute, and sugarcane and practice aquaculture in the freshwater bodies.

Q 6.C
• The annual event called the Great Backyard Bird Count, which began in 1998, is organized by the
Cornell Lab of Ornithology and the National Audubon Society. It is a community science project, that
includes students, bird watchers, and nature enthusiasts, who can participate in observing and
counting birds in their backyard every year. Hence statement 1 is not correct.
• Bird Count India is a collaborative initiative among various organizations and groups to enhance our
understanding of birds' distribution and population. It encourages and facilitates the listing and
monitoring of birds in India, ranging from individuals keeping track of their bird sightings to groups of
students and bird watchers observing the local avian fauna. Additionally, it supports large-scale projects
that document the prevalence and geographic range of different species across India. It also helped Great
Backyard Bird Count conduct the annual bird count event in India. Hence, statement 2 is correct.
• According to the Great Backyard Bird Count (GBBC) 2023, West Bengal recorded the highest number
of bird species (489), followed by Uttarakhand (426), Arunachal Pradesh (407), Assam (397), and
Karnataka (371). Tamil Nadu and Kerala secured the eighth and ninth positions, respectively, with 349
and 325 species. The survey reveals that India's avian population is thriving in various habitats, ranging
from urban to rural areas. The country has witnessed a significant surge in participation, resulting in
India ranking second in the number of checklists submitted, just after the United States of America.
2 www.visionias.in ©Vision IAS

FREE BY KING R QUEEN P [ऋषभ राजपूत]


Additionally, India ranked third in the total number of species reported by any country during the
GBBC event. Hence statement 3 is correct.
• The 2022 edition of GBBC engaged over 3,782 birders who uploaded over 40,000 checklists and recorded
1,017 species whereas this year, it was over 46,000 checklists and 1067 species.

Q 7.D
• Fishing activity is well developed in Japan than in any other place in the world due to numerous factors:
• The scarcity of meat as there are few pastures for livestock rearing and religious reasons has
popularised fish as a principal item of diet and chief protein food of the Japanese.
• The lack of lowlands and pastures means that only a few animals can be kept to supply meat and other
protein food. Fish, in all its varied forms, fresh, canned, dried, frozen, and in the form of fish pastes, fish
sauce and spiced condiments takes the place of meat as Japan's primary source of protein food. Hence
option 1 is correct.
• The indented coastline of Japan provides sheltered fishing ports, calm waters and safe landing places,
ideal for the fishing industry, In Hokkaido, where the Laurentian type of climate is too cold for active
agriculture, fishing takes first place. Hakodate and Kushiro are large fishing ports, complete with
refrigeration facilities. Hence options 2 and 3 are correct.
• The continental shelves around the islands of Japan are rich in plankton, due to the meeting of
the warm Kuroshio and the cold Oyashio currents and provide excellent breeding grounds for all kinds
of fish including herring, cod, mackerel, bonito, salmon, sardine and tuna, as well as crabs and
lobsters. Hence option 4 is correct.
• There is a great demand for it locally, and for export to other east Asiatic neighbors that lack the
techniques of large-scale commercial fishing.

Q 8.D
• The Hoysaleswara temple also referred to simply as the Halebidu temple, is a 12th-century Hindu
temple dedicated to Lord Shiva. It is the largest monument in Halebidu, a town in the state of
Karnataka, India, and the former capital of the Hoysala Empire.
• The temple was built on the banks of a large man-made lake and sponsored by King Vishnuvardhana
of the Hoysala Empire. Its construction started around 1121 CE and was completed in 1160 CE.
• The Hoysaleswar is mainly a Shaivite monument, yet reverentially includes many themes from
Vaishnavism and Shaktism traditions of Hinduism. In fact, it even has some images from Jainism.
Hence statement 1 is correct.
• It is a twin temple dedicated to Hoysaleswara and Santaleswara Shiva lingas, named after the
masculine and feminine aspects. Hence statement 2 is correct.
• India's famed Hoysala Temples in Belur, Halebid, and Somanathapura could soon be included in the
list of UNESCO World Heritage Sites. According to the latest reports, India has filed for the nomination
of the temples for the year 2022-2023. Hence statement 3 is correct.
• The temples are situated in the Indian state of Karnataka and are collectively called The Sacred
Ensembles of Hoysala. These have been on the tentative list of UNESCO since 2014.

Q 9.C
• An important aspect of population growth in India is the growth of its adolescents. The share of
adolescents i.e., those up to the age group of 10–19 years is about 20.9 percent (2011), among
which male adolescents constitute 52.7 percent and female adolescents constitute 47.3 percent. Hence
both statements 1 and 2 are correct.
• The adolescent population, though, is regarded as the youthful population having high potentials, but at
the same time they are quite vulnerable if not guided and channelised properly.
• There are many challenges for the society as far as these adolescents are concerned, some of which are
lower age at marriage, illiteracy—particularly female illiteracy, school dropouts, low intake of nutrients, a
high rate of maternal mortality among adolescent mothers, a high rate of HIV and AIDS infections,
physical and mental disability or retardedness, drug abuse and alcoholism, juvenile delinquency and
commission of crimes, etc.
• In view of these, the Government of India has undertaken certain policies to impart proper education to
the adolescent groups so that their talents are better channelised and properly utilised. The National Youth
Policy is one example that has been designed to look into the overall development of our large youth and
adolescent population.

3 www.visionias.in ©Vision IAS

FREE BY KING R QUEEN P [ऋषभ राजपूत]


Q 10.D
• Mixed Farming is a form of agriculture found in the highly developed parts of the world, e.g. North-
Western Europe, Eastern North America, parts of Eurasia and the temperate latitudes of Southern
continents.
• Crop rotation and intercropping play an important role in maintaining soil fertility. Equal emphasis is laid
on crop cultivation and animal husbandry. Animals like cattle, sheep, pigs and poultry provide the main
income along with crops.
• Mixed farming is characterized by high capital expenditure on farm machinery and building, extensive
use of chemical fertilizers and green manures and also the skill and expertise of the farmers. Hence,
statement 1 is correct.
• The system in which the farmers specialize in vegetables is known as truck farming. The distance of
truck farms from the market is governed by the distance that a truck can cover overnight, hence the name
truck farming. Such a type of farming is developed in densely populated industrial districts of northwest
Europe, the northeastern United States of America and the Mediterranean regions. It is both labor and
capital-intensive and lays emphasis on the use of irrigation, HYV seeds, fertilizers, insecticides,
greenhouses and artificial heating in colder regions. Hence, statement 2 is correct.
• Collective Farming or the model of Kolkhoz was introduced in the erstwhile Soviet Union to improve
upon the inefficiency of the previous methods of agriculture and to boost agricultural production for self-
sufficiency. The farmers used to pool in all their resources like land, livestock and labor. However, they
were allowed to retain very small plots to grow crops in order to meet their daily requirements.
Hence, statement 3 is correct.

Q 11.B
• The vegetation in the Mediterranean lands is not luxuriant. Trees are widely spaced and smaller in
height. Though trees have branches they are short and carry few leaves. Thus, there is an absence of
shade. The absence of shade is a distinctive feature of the Mediterranean lands. Hence statement 1 is
correct.
• The savanna vegetation is characterized by tall grass and short trees. The lack of water makes the
savanna a difficult place for tall plants such as trees to grow. Grasses and trees that grow in the savanna
have adapted to life with little water and hot temperatures. Hence, the terms parklands, and bush-veld
are used to describe the Savanna grasslands.
• The term Selvas is used to characterize the dense forest/vegetative growth in the tropical rainforest
in the Amazon Basin. High temperatures and abundant rainfall support luxuriant tropical rainforests.
Hence statement 2 is not correct.
• Steppe vegetation' refers the temperate grassland all over the world. Trees are very scarce in such
grasslands because of the scant rainfall and severe winters. Towards the Poles, an increase in
precipitation gives rise to a transitional zone of wooded steppes in the form of conifers. Towards the
equator, the stepper grass becomes shorter and sparser, till it merges with the desert with thorny scrub.
Hence statement 3 is correct.

Q 12.D
• Petroleum is also called ‘black gold’ or ‘liquid gold’. It is second to coal in terms of sources of energy.
It is an essential source of energy for all internal combustion engines in automobiles, railways and
aircraft. Crude petroleum occurs in sedimentary rocks of the tertiary period. It is formed when large
quantities of dead organisms, usually zooplankton and algae, are buried underneath sedimentary rock and
subjected to intense heat and pressure. Hence statement 1 is not correct.
• Petroleum (and natural gas) are born and accumulate in the sedimentary mantle of the Earth. Small
amounts of these hydrocarbons are present throughout the mantle, but large accumulations are
encountered less frequently. About 600 sedimentary basins, characterized by oil and gas occurrence, are
found on the Earth.
• Unlike coal, Petroleum is not distributed evenly around the world. More than half of the world’s
proven oil reserves are located in the Middle East. Following the Middle East are Canada and the
United States, Latin America, Africa, and the region occupied by the former Soviet Union. Each of
those regions contains less than 15 percent of the world’s proven reserves. Hence statement 2 is not
correct.
• The two largest classes of fields are the super-giants, fields with 5 billion or more barrels of ultimately
recoverable oil, and world-class giants, fields with 500 million to 5 billion barrels of ultimately
recoverable oil. Fewer than 40 supergiant oil fields have been found worldwide. The Arabian-
Iranian sedimentary basin in the Persian Gulf region contains two- thirds of these supergiant fields.
4 www.visionias.in ©Vision IAS

FREE BY KING R QUEEN P [ऋषभ राजपूत]


The remaining super-giants are distributed as follows: two in the United States, two in Russia, two in
Mexico, one in Libya, one in Algeria, one in Venezuela, and two in China.

Q 13.C
• Market Gardening and Horticulture: Market gardening and horticulture specialize in the cultivation of
high-value crops such as vegetables, fruits, and flowers, solely for urban markets.
• Farms are small and are located where there are good transportation links with the urban center where a
high-income group of consumers is located. Hence option (c) is the correct answer.
• It is both labor and capital-intensive and lays emphasis on the use of irrigation, HYV seeds, fertilizers,
insecticides, greenhouses, and artificial heating in colder regions.
• This type of agriculture is well developed in densely populated industrial districts of northwest Europe,
the northeastern United States of America, and the Mediterranean regions.
• The Netherlands specializes in growing flowers and horticultural crops, especially tulips, which are flown
to all major cities of Europe.
• In the regions where farmers specialize in vegetables only, the farming is known as truck farming. The
distance of truck farms from the market is governed by the distance that a truck can cover overnight,
hence the name truck farming.

Q 14.A
• In the Mediterranean region the term 'haciendas’ refer to large farm holdings. Though in general
farms are small in the Mediterranean region. Large holdings (haciendas) can be found in Spain. These
haciendas engage a large number of paid laborers.
• The Mediterranean type of climate is characterized by highly developed orchard farming methods. These
lands are also known as the world's orchard lands. A wide range of citrus fruits such as oranges, lemons,
and grapefruit are grown.
• Hence option (a) is the correct answer.

Q 15.D
• India is surrounded by sea from three sides and is bestowed with a long coastline. Water provides a
smooth surface for very cheap transport provided there is no turbulence. Though ports have been in use
since ancient times, the emergence of ports as gateways of international trade became important after the
coming of European traders and the colonisation of the country by the British. At present, India has 12
major ports and more than 200 minor or intermediate ports. In the case of the major ports, the central
government decides the policy and plays regulatory functions. The minor ports are there whose policies
and functions are regulated by state governments. The major ports handle a larger share of the total traffic.
Some of the important ports are explained below:
o Kandla, now officially Deendayal Port Authority, is a seaport and town in the Kutch district of
Gujarat state in Western India, near the city of Gandhidham. Located on the Gulf of Kutch, it is one of
India's major ports on the west coast.
o Visakhapatnam Port in Andhra Pradesh is a land-locked harbour, connected to the sea by a channel
cut through solid rock and sand. An outer harbour has been developed for handling iron ore,
petroleum and general cargo. Andhra Pradesh and Telangana are the main hinterlands for this port.
o Tuticorin Port in Tamil Nadu was developed to relieve the pressure of Chennai port. It deals with a
variety of cargo, including coal, salt, food grains, edible oils, sugar, chemicals and petroleum
products.
o Mormugao Port, commissioned in 1885 is one of the oldest ports on the west coast of India in the
state of Goa and is blessed with a protected open type natural harbour.
o New Mangalore Port is a deep-water, all-weather port at Panambur, Mangalore in Karnataka state in
India, which is the deepest inner harbour on the west coast. This port is operated by New Mangalore
Port Trust.

5 www.visionias.in ©Vision IAS

FREE BY KING R QUEEN P [ऋषभ राजपूत]


• Hence, option (d) is the correct answer.

Q 16.B
• Major coalfields of the world are:
• North America
o Pennsylvania anthracite field
o Appalachian bituminous field
o Eastern Illinois field – Illinois, Indiana and Kentucky
o Western interior field – Iowa, Missouri, Oklahoma
o Gulf province – Texas, Alabama and Arkansas
o Rocky mountain province- Utah, Colorado, Wyoming, Montana, new Mexico
o Canada – Prairies, British Columbia coalfields, Nova Scotia Coal fields
o The largest coal mine in the world by reserves is the North Antelope Rochelle coal mine in the
Powder River Basin of Wyoming, US. The mine was estimated to contain more than 1.7 billion
tonnes of recoverable coal as of December 2018. Hence pair 1 is not correctly matched.
• Europe
o Donetz coal basin (anthracite and high grade bituminous coal)
o Moscow-Tula coalfields
o Kuznetsk coal basin
o Karaganda field
o Silesia coal fields
o Ruhr area of Germany. Hence pair 2 is correctly matched.
o Other coal fields in Urals, Taimyr fields of the Arctic, deposits of the Caucasus mountains.
• Asia
o China – Shanxi, Fushun, Inner Mongolia, Kansu. Hence pair 4 is not correctly matched.
o Japan – Chikugo coalfield, Ishikari coalfield
o India – Damodar valley, Raniganj, Bokaro, Jharia, Singareni. Singareni is in Telangana.
o Pakistan - Quetta, Kalabagh and Thar coalfields
• Australia – Bowen Basin coalfield, Galilee Basin coalfield, South Maitland coalfield, Sydney Basin
coalfield, and Latrobe valley coalfield. Hence pair 3 is correctly matched.

6 www.visionias.in ©Vision IAS

FREE BY KING R QUEEN P [ऋषभ राजपूत]


Q 17.B
• Environmental determinism is the study of how the physical environment predisposes societies and
states toward particular development trajectories. In the early stages of their interaction with their natural
environment or physical environment, humans were greatly influenced by it. They adapted to the dictates
of Nature. It is environmental determinism. Hence option (b) is the correct answer.
• With social and cultural development, humans develop better and more efficient technology. They move
from a state of necessity to a state of freedom. They create possibilities with the resources obtained from
the environment. Human activities create a cultural landscape. This is called possibilism.

Q 18.B
• The footloose industry is a general term for an industry that can be placed and located at any location
without effect from factors such as resources or transport. Computer chips and mobile manufacturing are
some main examples of the footloose industries.
• Footloose industries can be located in a wide variety of places. They are not dependent on any specific
raw material, weight loss or otherwise. They largely depend on component parts that can be obtained
anywhere. They produce in small quantities and also employ a small labor force. These are generally not
polluting industries. The important factor in their location is accessibility by road network. Hence,
option (b) is the correct answer.

Q 19.C
• Recent Context: A newly identified tsetse fly pheromone reveals new insights into how the insects
communicate—and may help in reducing disease spread.
• Tsetse, are large, biting flies that inhabit much of tropical Africa. Tsetse flies are exclusively found
in sub-Saharan Africa between latitudes 14° N and 20 ° S. Hence, statement 2 is correct.
• Tsetse flies include all the species in the genus Glossina, which are placed in their own family,
Glossinidae. The tsetse is an obligate parasite, which lives by feeding on the blood of vertebrate
animals.
• Tsetse flies are known to carry parasites called African trypanosomes.
• When the insects bite humans or animals, they transmit these parasites, spreading diseases such as
African sleeping sickness, which can be fatal to humans, and nagana, a disease that affects livestock
and other animals. Hence, statement 1 is correct.
Q 20.C
• Pipelines are the most convenient and efficient mode of transporting liquids and gases over long distances.
Even solids can also be transported by pipelines after converting them into a slurry.
• Oil India Limited (OIL) under the administrative setup of the Ministry of Petroleum and Natural Gas is
engaged in the exploration, production, and transportation of crude oil and natural gas. It was incorporated
in 1959 as a company. Asia’s first cross-country pipeline covering a distance of 1,157 km was
constructed by OIL from the Naharkatiya oilfield in Assam to the Barauni refinery in Bihar. It was
further extended up to Kanpur in 1966. Hence, statement 1 is correct.
• GAIL (India) Ltd. was set up in 1984 as a public sector undertaking to transport, process, and market
natural gas for its economic use. The first 1,700 km long Hazira-VijaipurJagdishpur (HVJ) cross-country
gas pipeline, constructed by GAIL (India), linked Mumbai High and Bassein gas fields with various
fertilizer, power, and industrial complexes in western and northern India. Hence, statement 2 is correct.
• This artery provided an impetus to the Indian gas market development. Overall, India’s gas infrastructure
has expanded over ten times from 1,700 km to 18,500 km of cross-country pipelines and is expected to
soon reach over 34,000 km as Gas Grid by linking all the gas sources and consuming markets across the
country including the North Eastern States.

7 www.visionias.in ©Vision IAS

FREE BY KING R QUEEN P [ऋषभ राजपूत]


Q 21.D
• Equity refers to making equal access to opportunities available to everybody. The opportunities
available to people must be equal irrespective of their gender, race, income, and in the Indian case, caste.
Hence pair 1 is correctly matched.
• Empowerment means having the power to make choices. Such power comes from increasing freedom
and capability. Good governance and people-oriented policies are required to empower people. Hence
pair 2 is correctly matched.
• Sustainability means continuity in the availability of opportunities. To have sustainable human
development, each generation must have the same opportunities. Hence pair 3 is correctly matched.
• Productivity is the measure of output per unit of input. Human labor productivity or productivity in
terms of human work means efforts to increase their knowledge or provide better health facilities
ultimately leading to better work efficiency. Hence pair 4 is correctly matched.
• Hence option (d) is the correct answer.

Q 22.A
• Mariupol, is located in southeastern Ukraine. It lies along the estuary of the Kalmius and Kalchik
rivers, 6 miles (10 km) from the Sea of Azov. recently Russian forces invaded Mariupol. Hence pair 1 is
correctly matched.

• Adana, city, south-central Turkey. It is situated in the plain of Cilicia, on the Seyhan River (the ancient
Sarus River). An agricultural and industrial centre and the country’s fourth largest city, Recently city of
Adana was worst hit by earthquake. Hence pair 2 is not correctly matched.

8 www.visionias.in ©Vision IAS

FREE BY KING R QUEEN P [ऋषभ राजपूत]


• Riyadh is the capital city of Saudi Arabia, located in the central region of the country. It is the largest
city in the kingdom and serves as a political, financial, and administrative center of the country. The
economy is primarily based on oil and gas exports, as well as other industries such as finance,
technology, and tourism. Hence pair 3 is not correctly matched.

Q 23.B
• The Border Road Organisation (BRO) was established in May 1960 for accelerating economic
development and strengthening defense preparedness through the rapid and coordinated improvement of
strategically important roads along the northern and northeastern boundaries of the country. It is a premier
multifaceted construction agency. It has constructed roads in high altitude mountainous terrain joining
Chandigarh with Manali (Himachal Pradesh) and Leh (Ladakh). This road runs at an average altitude of
4,270 meters above the mean sea level. Hence, statement 1 is not correct.
• Apart from the construction and maintenance of roads in strategically sensitive areas, the BRO also
undertakes snow clearance in high-altitude areas. The international highways are meant to promote a
harmonious relationship with the neighboring countries by providing effective links with India.
• Atal Tunnel (9.02 Km) has been built by Border Road Organisation. This tunnel connects Manali to
Lahaul-Spiti valley throughout the year. Earlier the valley was cut off for about 6 months each year owing
to heavy snowfall. The Tunnel is built with ultra-modern specifications in the Pir Panjal range of the
Himalayas at an altitude of 3000 meters from the Mean Sea Level. Hence, statement 2 is correct.

Q 24.A
• Intensive-subsistence farming is practiced in regions where the land is limited. And as land is passed
on from one generation to the next, it is fragmented into smaller plots to divide among the offspring of the
farmers. This type of agriculture is largely found in densely populated regions of monsoon Asia. Paddy,
wheat and millet are the important crops grown in this type of agriculture.
• In agriculture dominated by wet paddy cultivation rice crop is a dominant crop. Land holdings are very
small due to the high density of the population. Farmers work with the help of family labor leading to
intensive use of land. However, labor productivity is low. Hence, option 2 is not correct.
• The use of machinery is limited and most of the agricultural operations are done by manual labor.
Hence, 3 option is not correct. Farmyard manure is used to maintain the fertility of the soil. Hence,
option 1 is correct.
• In this type of agriculture, the yield per unit area is high. Hence, option 4 is correct.

Q 25.A
• Recent Context: Karnataka’s cultural capital Mysuru is set to host the Centre’s ambitious Bharat
Shared Repository of Inscriptions (Bharat SHRI) project that was announced by finance minister
Nirmala Sitharaman in her Union Budget speech.
• The project to be launched at the digital epigraphy museum aims to digitise one lakh ancient inscriptions
in the first phase. While all inscriptions have been published in annual reports of Indian Epigraphy
(ARIE), digital copies are unavailable to researchers and historians
9 www.visionias.in ©Vision IAS

FREE BY KING R QUEEN P [ऋषभ राजपूत]


• Currently, the Archaeological Survey of India’s (ASI) epigraphy division headquartered at Mysuru has
more than one lakh estampages of several prominent inscriptions which will soon be digitised for
widespread use by historians and researchers.
• Hence option (a) is the correct answer.

Q 26.D
• Nomadic herding or pastoral nomadism is a primitive subsistence activity, in which the herders rely on
animals for food, clothing, shelter, tools and transport.
• Pastoral nomadism is associated with three important regions. The core region extends from the
Atlantic shores of North Africa Eastwards across the Arabian peninsula into Mongolia and Central
China. The second region extends over the tundra region of Eurasia. In the southern hemisphere, there
are small areas in South-west Africa and on the island of Madagascar.
• In mountain regions, such as Himalayas, Gujjars, Bakarwals, Gaddis and Bhotiyas migrate from the
plains to the mountains in summers and to the plains from the high-altitude pastures in winters. Similarly,
in the tundra regions, the nomadic herders move from south to north in the summers and from north to
south in winter. This process is called transhumance. Hence, option (d) is correct.

Q 27.B
• Migration is usually defined as a geographical movement of people involving a change from their usual
place of residence.
• It is the third component of population change, the other two being mortality and fertility. However,
migration is different from the other two processes, in the sense that it is not a biological factor like the
other two but a human response to economic, social and demographic forces in the environment.
• Migration may be permanent, temporary or seasonal. It may take place from rural to rural areas, rural to
urban areas, urban to urban areas and urban to rural areas. There are two sets of factors that influence
migration.
• The push factors are those that compel or force a person, due to various reasons, to leave that place and
go to some other place. For example, unemployment, poor living conditions, political turmoil, an
unpleasant climate, natural disasters, epidemics and socio-economic backwardness may compel people
to leave their native place in search of better opportunities.
• Pull factors refer to those factors that attract migrants to an area, such as opportunities for better
employment, higher wages, better working conditions and better amenities of life, peace and stability,
security of life and property, a pleasant climate, etc.
• There is generally cityward migration when rapid expansion of industry, commerce and business takes
place. However, pull factors operate not only in the rural-urban migration, but also in other types of
internal as well as international migration.
• This classification of motives for migration into push and pull factors is very useful in analysing
determinants of migration, but all migratory movements cannot be explained by these factors alone.
Moreover, sometimes migration may occur not because of push or pull factors alone but as a result of the
combined effect of both.
• Hence option (b) is the correct answer.

Q 28.B
• Inland Waterways: Rivers, canals, lakes, and coastal areas have been essential waterways since time
immemorial. Boats and steamers are used as means of transport for cargo and passengers. The
development of inland waterways is dependent on the navigability width and depth of the channel,
continuity in the water flow, and transport technology in use.
o The Rhine Waterways: The Rhine flows through Germany and the Netherlands. It flows from two
small headways in the Alps of east-central Switzerland north and west to the North Sea, into which it
drains through the Netherlands. The Ruhr river joins the Rhine from the east. It flows through a rich
coalfield and the whole basin has become a prosperous manufacturing area. Dusseldorf is the Rhine
port for this region. This waterway is the world’s most heavily used. Hence pair 1 is correctly
matched.
o The Danube Waterway: This important inland waterway serves Eastern Europe. The Danube river
rises in the Black Forest and flows eastwards through many countries. It is navigable up to Taurna
Severin. The chief export items are wheat, maize, timber, and machinery.
o The Volga Waterway: Russia has a large number of developed waterways, of which the Volga is one
of the most important. It drains into the Caspian Sea. The Volga-Moscow Canal connects it with the

10 www.visionias.in ©Vision IAS

FREE BY KING R QUEEN P [ऋषभ राजपूत]


Moscow region and the Volga-Don Canal with the Black Sea. Hence pair 2 is not correctly
matched.
o The Great Lakes – St. Lawrence Seaway: The Great Lakes of North America Superior, Huron Erie,
and Ontario are connected by Soo Canal and Welland Canal to form an inland waterway. The estuary
of the St. Lawrence River, along with the Great Lakes, creates a unique commercial waterway in the
northern part of North America. The ports on this route like Duluth and Buffalo are equipped with all
facilities of ocean ports. As such large oceangoing vessels are able to navigate up the river deep inside
the continent to Montreal.
o The Mississippi Waterways: The Mississippi-Ohio waterway connects the interior part of the U.S.A.
with the Gulf of Mexico in the south. Large steamers can go through this route up to
Minneapolis. Hence pair 3 is correctly matched.

Q 29.B
• Recent Context: Famous Yakshagana singer and screenwriter Balipa Narayan Bhagwat passed away at
the age of 85. He had mastered a unique style of singing, due to which fans have given it the name
of ‘Balipa Style’. Rich in voice, Bhagwat has written more than 30 Yakshagana ‘Prasanga’ (scripts). He
was well versed in over 100 Yakshagana episodes, which he composed by heart. He had served in the
field of Yakshagana for about 60 years. He was the chief Bhagwat of Kateel Durgaparameshwari
Prasadita Yakshagana Mandali (Kateel Mela)·
• Yakshagaana is a traditional theatre, developed in Dakshina Kannada, Udupi, Uttara Kannada,
Shimoga and western parts of Chikmagalur districts, in the state of Karnataka and in Kasaragod
district in Kerala.
o It combines dance, music, dialogue, costume, make-up, and stage techniques with a unique style and
form.
o It is believed to have evolved from pre-classical music and theatre during the period of the Bhakti
movement. Hence, statement 1 is not correct.
o Elaborate and colorful costumes, makeup, and masks constitute some of the most striking
features of the art form. Hence, statement 3 is correct.
o Traditionally, yakshagana was performed in the open air by all-male troupes sponsored by various
Hindu temples. Since the mid-20th century, however, many performances have been held on indoor
stages, and women began to train in the tradition in the 1970s.
o With roots in Sanskrit literature and theatre, yakshagana emerged as a form of dance-drama in the
16th century.
o The narratives are drawn primarily from the great Hindu epics Ramayana and Mahabharata as well as
from the tales of the youthful god Krishna as recounted in the Bhagavata-Purana. Hence, statement 2
is correct.
o Historically, the cities of Tanjore (now Thanjavur) and Madura (now Madurai), both in the state of
Tamil Nadu, and Mysore, in Karnataka, were centers of yakshagana composition.

Q 30.B
• India has a diverse range of coal deposits and is the world's second-largest producer of coal after
China. The coal found in India can be broadly classified into four types based on its carbon content,
heating value, and other properties. These types of coal are:
o Anthracite Coal: This is hard, black coal with a high carbon content of 86-98% and a low
volatile matter content. Anthracite coal is found in Jammu and Kashmir and is primarily used for
heating and industrial processes.
o Bituminous Coal: This is the most common type of coal found in India, and accounts for over
80% of the country's coal reserves. Bituminous coal has a carbon content of 45-86% and a higher
volatile matter content than anthracite coal. It is found in the states of Jharkhand, Chhattisgarh,
Odisha, West Bengal, Madhya Pradesh, and Maharashtra. Bituminous coal is used primarily for
electricity generation, for making coke for steel production, and for heating.
o Sub-bituminous Coal: This type of coal has a lower carbon content than bituminous coal,
typically ranging from 35-45%. It has a higher moisture content and a lower heating value than
bituminous coal. Sub-bituminous coal is found in the states of Madhya Pradesh, Odisha, and
Chhattisgarh, and is primarily used for electricity generation.
o Lignite Coal:
✓ This is a low-grade coal with a carbon content ranging from 25-35%, and a high moisture
content of around 20-50%. It has a low heating value and is found in the states of Tamil Nadu,

11 www.visionias.in ©Vision IAS

FREE BY KING R QUEEN P [ऋषभ राजपूत]


Gujarat, and Rajasthan. Lignite coal is used primarily for electricity generation and for industrial
processes. Hence statement 1 is not correct.
✓ The lignite reserves in India stand at a level of 41.96 billion tonnes as of 1.4.2012, of which 90%
occur in the southern State of Tamil Nadu. Other states where lignite deposits have been located
are Rajasthan, Gujarat, Kerala, Jammu & Kashmir, and the union territory of Puducherry. The
largest lignite deposits of the country are at Neyveli in the state of Tamil Nadu. Hence
statement 2 is correct.

Q 31.D
• The Maasai tribe is a Nilotic ethnic group they are known for their distinctive dress, which includes
brightly colored shukas (cloths) and beaded jewelry, as well as their unique customs and traditions.
Maasai society is organized into clans, with each clan having its own territory and leadership. Men
traditionally serve as warriors and protectors, while women are responsible for household duties, such as
cooking, cleaning, and caring for children.
• The Masai are a nomadic tribe who once wandered with their herds of cattle in the central highlands of
East Africa – in Kenya, Tanzania and Uganda. They are now mainly confined to the 15,000 square miles
of Masai reserves in Kenya and Tanzania. Hence statement 1 is correct.
• The cattle reared by the Masai are Zebu cattle with humps and long horns. They are treated with great
respect and affection and are never slaughtered for food or for sale. Cattles are reared by every Masai
family. They are considered far more valuable than anything else. They are symbol of wealth. Hence
statement 2 is correct.
• The Masai tribe build circular huts with sticks, bushes and mud for temporary shelter. Hence
statement 3 is correct.
• Hence option (d) is the correct option.

Q 32.C
• Recent context: A projected 400,000 to 800,000 tonnes of Indonesian palm oil are expected to be blocked
from the global market over the next three months after the government said it would suspend two-thirds
of palm oil exports in order to shore up domestic cooking oil supplies, according to a Global Agricultural
Information Network report from the Foreign Agricultural Service o the US Department of Agriculture
(USDA) in the month of February.
• Palm oil is an edible vegetable oil derived from the mesocarp of the fruit of the oil palms. The oil is used
in food manufacturing, in beauty products, and as biofuel. Palm oil accounted for about 33% of global
oils produced from oil crops in 2014.
• In the year 2021, India was the leading importer of palm oil worldwide, with an import value of
about 9.6 billion U.S. dollars. Hence, statement 2 is correct.
• The biggest producers of palm oil are Indonesia, Malaysia, Thailand, and Nigeria. Indonesia produces
biodiesel primarily from palm oil.
• Andhra Pradesh is the major Oil Palm growing State in India with a production capacity of 20 lakh
tons. Andhra Pradesh (83.5 percent) along with Telangana accounts for about 97 percent of India's
278,000 tonnes of crude palm oil production. Hence, statement 1 is correct.

Q 33.B
• This is the population pyramid of Australia.
• It is bell shaped and tapered towards the top.
• This shows birth and death rates are almost equal leading to a near constant population. Hence,
option (b) is the correct answer.
o Whereas, Japan pyramid has a narrow base and a tapered top showing low birth and death rates.
• The age-sex pyramid of Nigeria is a triangular shaped pyramid with a wide base and is typical of less
developed countries.
• These have larger populations in lower age groups due to high birth rates.
• If you construct the pyramids for Bangladesh and Mexico, it would look the same.

Q 34.D
• Minerals are naturally occurring inorganic substances that have a definite chemical composition and
physical properties. They are formed by geological processes over millions of years and are found in
rocks, soil, and water. These minerals can be broadly classified into two main categories: metallic and
non-metallic minerals.

12 www.visionias.in ©Vision IAS

FREE BY KING R QUEEN P [ऋषभ राजपूत]


• They are usually found in “ores”. The term ore is used to describe an accumulation of any mineral mixed
with other elements. The mineral content of the ore must be in sufficient concentration to make its
extraction commercially viable.
• The mode of occurrence of minerals refers to the way in which minerals are distributed or concentrated
in the earth's crust. There are several modes of occurrence of minerals, including veins, disseminations,
stratiform deposits, and placers.
o Copper: Copper is a metallic mineral that is widely distributed in the earth's crust. It is commonly
found in the form of sulfide and oxide minerals, such as chalcopyrite, bornite, and malachite. Copper
deposits are typically formed in igneous and metamorphic rocks minerals which occur in the
cracks, crevices, faults, or joints. The smaller occurrences are called veins and the larger are
called lodes. In most cases, they are formed when minerals in liquid/ molten and gaseous forms
are forced upward through cavities toward the earth’s surface. They cool and solidify as they
rise. Copper can also occur in sedimentary rocks as a result of chemical weathering and leaching but
is generally found in veins and lodes. Hence pair 1 is correctly matched.
o Bauxite: Bauxite is a non-metallic mineral that is the primary source of aluminum. It is typically
found in tropical and subtropical regions, where it is formed through the weathering of aluminum-
rich rocks such as granite and shale. Bauxite deposits are typically formed in residual soils and are
often found on hillsides and plateaus. The mineral is commonly found as a mixture of gibbsite,
boehmite, and diaspore. Hence pair 3 is correctly matched.
o Coal: Coal is a non-metallic mineral that is formed from the remains of ancient plants and trees. It
is typically found in sedimentary rocks, where it is formed through a process known
as coalification. This process involves the transformation of plant material into coal through the
application of heat and pressure over millions of years. Coal deposits are typically found in
sedimentary basins, where they can occur as seams or layers of varying thicknesses. Hence pair 2 is
correctly matched.

Q 35.A
• Sex ratio refers to the number of females to males in a given population or group. Sex ratio can vary
widely depending on a number of factors, including geography, culture, and demographics. In most
human populations, the sex ratio at birth is slightly biased towards males.
• In India, the sex ratio is worked out using the formula: (Female Population/Male Population) *1000 or
the number of females per thousand males. Hence statement 1 is correct.
• Literacy rate is the percentage of the population who are able to read and write with understanding. It is a
measure of a country's educational level and human development. In India, literacy rate denotes the
percentage of the population above 7 years of age, who is able to read, write and have the ability to do
arithmetic calculations with understanding. Hence statement 2 is not correct.
• The literacy rate in rural India is 67.77% compared to 84.11% in urban India.
• Hence option (a) is the correct answer.

Q 36.B
• Recent Context: Russia invaded and occupied parts of Ukraine in a major escalation of the Russo-
Ukrainian War, which began in 2014. The invasion has resulted in many deaths on both sides and
instigated Europe's largest refugee crisis since World War II. Buildings and infrastructure were hit
in Kyiv, Odesa and Kharkiv, with power blackouts in several areas. Ukraine said Russia had fired
missiles, the largest number in a single wave for several weeks.
o Odesa is a UNESCO World Heritage Site and is located near the Black sea and is 500 km from
the capital of Ukraine, Kyiv Hence, pair 1 is correctly matched.
• Mount Aso, is also known as Aso Volcano and in this sense is the largest active volcano in Japan,
and is among the largest in the world. Mount Aso volcano in Japan, erupted on October 20, 2021 and
emitted a giant column of ash to about thousands of metres into the sky. Hence, pair 2 is not correctly
matched.
• Nicosia, also known as Lefkosia, is the divided capital city of Cyprus. Former foreign minister Nikos
Christodoulides was elected as the new president of Cyprus in a runoff election in 2023. Hence, pair 3
is correctly matched.

Q 37.B
• The Union Cabinet has approved National Green Hydrogen Mission with an initial outlay of
Rs.19,744 crore, including an outlay of Rs.17,490 crore for the SIGHT programme, Rs.1,466 crore for
pilot projects, Rs.400 crore for R&D, and Rs. 388 crores towards other Mission components.
13 www.visionias.in ©Vision IAS

FREE BY KING R QUEEN P [ऋषभ राजपूत]


• The MNRE will formulate the scheme guidelines for the implementation of the respective components
to help achieve the following outcomes by 2030:
o Development of green hydrogen production capacity of at least 5 MMT (Million Metric Tonne)
per annum with an associated renewable energy capacity addition of about 125 GW in the country.
o Over Rs. Eight lakh crore in total investments.
o Creation of over Six lakh jobs.
o The cumulative reduction in fossil fuel imports of over Rs. One lakh crore.
o Abatement of nearly 50 MMT of annual greenhouse gas emissions.
• The Mission will have wide-ranging benefits-
o Creation of export opportunities for Green Hydrogen and its derivatives
o Decarbonization of industrial, mobility, and energy sectors
o Reduction in dependence on imported fossil fuels and feedstock
o Development of indigenous manufacturing capabilities
o Creation of employment opportunities; and
o Development of cutting-edge technologies.
• The Mission will facilitate demand creation, production, utilization, and export of Green Hydrogen.
• Under the Strategic Interventions for Green Hydrogen Transition Programme (SIGHT), two distinct
financial incentive mechanisms – targeting domestic manufacturing of electrolyzers and production
of Green Hydrogen – will be provided.
• The Mission will also support pilot projects in emerging end-use sectors and production pathways.
Regions capable of supporting large-scale production and/or utilization of Hydrogen will be identified and
developed as Green Hydrogen.
• Hence option (b) is the correct answer.

Q 38.A
• Nomadic herding or pastoral nomadism is a primitive subsistence activity, in which the herders rely on
animals for food, clothing, shelter, tools, and transport. They move from one place to another along with
their livestock, depending on the amount and quality of pastures and water.
• A wide variety of animals is kept in different regions. In tropical Africa, cattle are the most important
livestock, while in Sahara and Asiatic deserts, sheep, goats, and camels are reared. In the mountainous
areas of Tibet and the Andes, yak, and llamas, and in the Arctic and Sub-Arctic areas, reindeer are the
most important animals.
• Movement in search of pastures is undertaken either over vast horizontal distances or vertically from one
elevation to another in the mountainous regions. The process of migration from plain areas to pastures on
mountains during summers and again from mountain pastures to plain areas during winters is known as
transhumance.
• In mountain regions, such as the Himalayas, Gujjars, Bakarwals, Gaddis and Bhotiyas migrate from
plains to the mountains in summers and to the plains from the high-altitude pastures in winters. Hence
option (a) is the correct answer.
• Similarly, in the tundra regions, the nomadic herders move from south to north in summer and from north
to south in winter.
• NOTE: The Five hill Tribes of the Nilgiris are: Irulas, Badagas, Todas, Kotas and Kurumbas.

Q 39.B
• General Motors, Ford Motor Company, and Chrysler Stellantis North America are often referred to
as the "Big Three", being the largest automakers in the United States of America. All three have
their headquarters in the Detroit area.
• Chennai is nicknamed "The Detroit of India", with more than one-third of India's automobile industry
being based in the city. Chennai is also referred to as the ‘Detroit of India’ with the Indian operations of
Ford, Hyundai, Renault, and Nissan headquartered in the city and BMW having an assembly plant on the
outskirts. Chennai accounts for 35% of the country’s automobile component industry and 60 percent of
the country’s automotive exports.
• Hence option (b) is the correct answer.

Q 40.C
• Tehri Dam: Tehri Dam is a hydroelectric dam located on the Bhagirathi River in the state of
Uttarakhand. It was built in 2006 and has a storage capacity of 4.0 billion cubic meters of water. The
dam is one of the tallest in the world, standing at a height of 260 meters. It is used for hydroelectric power

14 www.visionias.in ©Vision IAS

FREE BY KING R QUEEN P [ऋषभ राजपूत]


generation and flood control, and also provides drinking water to nearby towns and cities. The dam has
become a popular tourist attraction in the region.
• Gandhi Sagar: Gandhi Sagar is a reservoir located on the Chambal River in the states of Madhya
Pradesh and Rajasthan. It was built in 1960 and has a storage capacity of 7,322 million cubic meters of
water. The dam is used for irrigation, hydroelectric power generation, and flood control.
• Nagarjuna Sagar: Nagarjuna Sagar is a reservoir located on the Krishna River in the state of
Telangana and Andhra Pradesh. It is one of the largest man-made lakes in the world, with a storage
capacity of 11,472 million cubic meters of water. The dam was built in 1967 and is used for irrigation,
hydroelectric power generation, and drinking water supply.
• Mettur Dam: Mettur Dam is a large dam located on the Cauvery River in the state of Tamil Nadu. It
was built in 1934 and has a storage capacity of 93.47 thousand million cubic feet of water. The dam is
used for irrigation, hydroelectric power generation, and drinking water supply. It is also a popular tourist
destination.

Q 41.A
• Shifting cultivation is a Primitive Subsistence Agriculture in which the vegetation is usually cleared by
fire,and the ashes add to the fertility of the soil. It is also called as slash and burn agriculture. The
cultivated patches are very small and cultivation is done with very primitive tools such as sticks and hoes.
After some time (3 to 5 years) the soil loses its fertility and the farmer shifts to other parts and clears other
patches of the forest for cultivation.
• It is prevalent in tropical regions with different names, e.g. Jhuming in the Northeastern states of
India, Milpa in Central America and Mexico and Ladang in Indonesia and Malaysia, Taungya in
Myanmar, Chena in Sri Lanka and Caigin in the Philippines. Hence, only pair 3 is correctly matched.
15 www.visionias.in ©Vision IAS

FREE BY KING R QUEEN P [ऋषभ राजपूत]


Q 42.D
• Packet Stations are also known as ferry ports. These packet stations are exclusively concerned with the
transportation of passengers and mail across water bodies covering short distances. These stations occur in
pairs located in such a way that they face each other across the water body, e.g. Dover in England and
Calais in France across the English Channel. Hence option (d) is the correct answer.
• Entrepot Ports are collection centers where goods are brought from different countries for export.
Singapore is an entrepot for Asia. Rotterdam for Europe, and Copenhagen for the Baltic region.
• Out Ports are deep water ports built away from the actual ports. These serve the parent ports by receiving
those ships which are unable to approach them due to their large size. A classic combination, for
example, is Athens and its outport Piraeus in Greece.
• Ports of Call are the ports that originally developed as calling points and main sea routes where ships
used to anchor for refueling, watering and taking food items. Later on, they developed into commercial
ports. Aden, Honolulu and Singapore are good examples.

Q 43.B
• Recent Context: Research has shed light on the Agasthiyar Observatory which was one of the few
magnetic observatories in the world during the 19th century.
• Agasthyarkoodam, the misty peak, and trekkers destination on the Western Ghats straddling Tamil
Nadu and Kerala were once home to a forgotten and long-lost 19th-century observatory established by
Scottish meteorologist John Allan Broun.
• The Agasthyarkoodam Observatory is an astronomical research observatory in Kerala, India.
• The Indian Institute of Astrophysics owns and operates it.
• Magnetic observatories continuously measure and record Earth’s magnetic field at a number of
locations.
o In an observatory of this sort, magnetized needles with reflecting mirrors are suspended by quartz
fibers.
o Light beams reflected from the mirrors are imaged on a photographic negative mounted on a rotating
drum.
o Variations in the field cause corresponding deflections on the negative.
o Applications of magnetic observatories include the creation of world magnetic maps for
navigation and surveying; correction of data obtained in air, land, and sea surveys for mineral
and oil deposits; and scientific studies of the interaction of the Sun with Earth, etc
• Hence, option (b) is the correct answer.

Q 44.A
• Industries are not evenly distributed in the country. They tend to concentrate on certain locations because
of favourable locational factors. Several indices are used to identify the clustering of industries, important
among them are:
o Number of industrial units,
o Number of industrial workers,
o Quantum of power used for industrial purposes,
o Total industrial output,
o Value added by manufacturing, etc.

16 www.visionias.in ©Vision IAS

FREE BY KING R QUEEN P [ऋषभ राजपूत]


Q 45.A
• Life and development in the Equatorial Regions: The Equatorial Regions are generally sparsely
populated. In the forests, most primitive people live as hunters and collectors and the more advanced ones
practice shifting cultivation.
• In the Amazon basin, the Indian tribes collect wild rubber, in the Congo Basin the Pygmies gather
nuts and in the jungles of Malaysia, the Orang Asli make all sorts of cane products and sell them to
people in villages and towns. Hence option (a) is the correct answer.
• In the clearings for shifting cultivation, crops like manioc (tapioca), yams, maize, bananas, and
groundnuts are grown.
• When the fertility is exhausted, the clearing is abandoned and they move on to a new plot.

Q 46.B
• The location of an industry depends on many factors such as raw material, power, labor, transport
facilities, etc. Industries using weight-losing raw materials are located in the regions where raw materials
are located. Following are some of the industries which depend on weight-losing raw materials.
• The raw materials required for iron and steel industries are iron ore, coking coal, limestone,
dolomite, manganese, and fire clay. All these raw materials are gross (weight loss), therefore, the best
location for the iron and steel plants is near the source of raw materials.
• The sugar industry is the second most important agro-based industry in the country. Sugarcane is a
weight-losing crop. The ratio of sugar to sugarcane varies between 9 to 12 percent depending on its
variety. Its sucrose content begins to dry during haulage after it has been harvested from the field. Better
recovery of sugar is dependent upon its being crushed within 24 hours of its harvesting. Sugar factories
hence are located within the cane-producing regions.
• Copper smelting, pig iron, and pulp industry also depend on weight losing raw material.
• Cotton is a “pure” raw material that does not lose weight in the manufacturing process. so other factors,
like, power to drive the looms, labor, capital, or market may determine the location of the industry.
Similarly,
• The computer chip-making industries depend on silicon, Germanium, metals, etc which do not lose
weight in the process.
• Hence, option (b) is the correct answer.

Q 47.D
• The population density is the concentration of the individuals within a specific geographic location.
• It is influenced by many factors.
• Physical factors such as favorable landforms like river valleys have a higher concentration of population.
E.g. Ganga-Brahmaputra Plains in India.
• Fertile soil areas have high density across the globe. Hence option 2 is correct.
• Economic factors also influence population density. Regions with little or no economic opportunities
tend to be sparsely populated. The industrial regions across the world are densely populated. Osaka in
Japan and Mumbai in India are good examples. Hence option 1 is correct.
• Emigration is the outward movement of people from a particular area, it will lead to a reduction in
population.
• Immigration will lead to an increase in population density. Hence option 3 is correct.

Q 48.C
• Under the internal migration, four streams are identified: (a) rural to rural (R-R); (b) rural to urban (R-U);
(c) urban to urban (U-U); and (d) urban to rural (U-R).
• In India, during 2011, out of 455.0 million migrants, enumerated on the basis of the last residence, 141.9
million had changed their place of residence in the last ten years. Out of these, 118.7 million were intra-
state migrants. The stream was dominated by female migrants. Most of these were migrants related
to marriage.
• Though the intra-state migration is a major proportion of migration in India but the major reason
for this has been marriage. Marriage accounts for about 33% of intra-state migrant population
which is 51% for female intra-state migrant.

17 www.visionias.in ©Vision IAS

FREE BY KING R QUEEN P [ऋषभ राजपूत]


• Hence option (c) is the correct answer.

Q 49.B
• Patterns of population distribution and density help us to understand the demographic characteristics of
any area. Broadly, 90 per cent of the world population lives in about 10 per cent of its land area. The 10
most populous countries of the world contribute about 60 per cent of the world’s population. Of these 10
countries, 6 are located in Asia.

• Hence option (b) is the correct answer.

Q 50.C
• Types of Urban Settlements Depending on the size and the services available and functions
rendered, urban centers are designated as towns, cities, million cities, conurbations, and
megalopoleis.
• Town The concept of ‘town’ can best be understood with reference to ‘village’. Population size is not the
only criterion. Functional contrasts between towns and villages may not always be clear-cut, but specific
functions such as manufacturing, retail and wholesale trade, and professional services exist in towns.
• City: A city may be regarded as a leading town, which has outstripped its local or regional rivals. In the
words of Lewis Mumford, “ the city is in fact the physical form of the highest and most complex type of
associative life”. Cities are much larger than towns and have a greater number of economic functions.
They tend to have transport terminals, major financial institutions, and regional administrative offices.
When the population crosses the one million mark it is designated as a million city.
• Conurbation: The term conurbation was coined by Patrick Geddes in 1915 and applied to a large
area of urban development that resulted from the merging of originally separate towns or cities.
Greater London, Manchester, Chicago, and Tokyo are examples.
• Megalopolis This Greek word meaning “great city”, was popularised by Jean Gottman (1957) and
signifies ‘super- metropolitan’ region extending, as the union of conurbations. The urban landscape
stretching from Boston in the north to south of Washington in the U.S.A. is the best-known example of a
megalopolis.
• Hence, option (c) is the correct answer.

Q 51.C
• India is one of the largest producers of iron ore in the world, and there are several major iron ore
mines located in the country. Some of the major iron ore mines in India, along with their locations and
features, are as follows:
18 www.visionias.in ©Vision IAS

FREE BY KING R QUEEN P [ऋषभ राजपूत]


o The Gurumahisani iron ore mine: It is located in the Mayurbhanj district of Odisha, India. It is
India’s first iron ore mine, and widely dubbed as the ‘mother mine’ for the empire of Tata Steel that
has grown to a global steel behemoth having a crude steel capacity of 34 million tonnes per
annumThe Gurumahisani mine has an estimated reserve of around 67 million tonnes of iron ore, and
is known for its high-grade iron ore with low impurities. The iron ore from this mine is ideal for use in
steel production and is in high demand by steel plants in India and abroad. Hence pair 1 is not
correctly matched.
o Noamundi Iron Ore Mine: It is located in the West Singhbhum district of Jharkhand and is
owned by Tata Steel. The mine has an estimated reserve of around 310 million tonnes and supplies
iron ore to Tata Steel's Jamshedpur plant. Hence pair 2 is correctly matched.
o Badampahar iron ore Mine: It is located in the Mayurbhanj district of Odisha, India. The
Badampahar iron ore deposit is one of the largest iron ore reserves in the state of Odisha and is
estimated to contain around 200 million tonnes of high-grade iron ore. The iron ore from Badampahar
is known for its superior quality and high iron content, which makes it ideal for use in steel
production. Hence pair 3 is correctly matched.

Q 52.C
• The Sino-Tibetan languages are mostly spoken by the tribal groups residing in the geographical area
extending from Ladakh to the North-eastern frontier regions of India.
• This family is divided into the sub-families of Siamese-Chinese and Tibeto-Myanmari (Tibeto-
Burman). The first group does not belong to India except for the Khamti speech.
• The Tibeto-Myanmari (Tibeto-Burman) sub-family is divided into three branches: Tibeto-Himalayan,
North Assam and Assam-Myanmari (Assam-Burmese).
• The Tibeto-Himalayan branch is again divided into Tibetan, or Bhotia and Himalayan groups. They are
spread over the states of Jammu and Kashmir, Himachal Pradesh, Punjab, West Bengal and Sikkim.
• Languages like Bhotia, Tibetan, Balti, Ladakhi and Lahul belong to the Bhotia group. The Himalayan
group includes the Chhambba, Limba and Lepcha.
• North Assam branch includes Dafla, Miri, Mishmi and Mishing of Arunachal Pradesh.
• Assam-Myanmari (Assam-Burmese) language is spoken by Bodo, Naga and Kuki-chin. Important
languages of the area are Bodo, Garo, Tripuri, Reang, Kachari, Rabha and Dimasa. In Naga groups are
Sema, Angami, Lotha, Tangkhul and Konyak. Other important languages of this group are Manipuri,
Mizo,Thado, Hmar and Kuki.
• Hence option (c) is the correct answer.

Q 53.A
• Coal is a one of the important minerals which is mainly used in the generation of thermal power
and smelting of iron ore. It is the one of the most mined mineral from the earth. According to one
estimate, proven coal reserves are 860, 938 million tonnes.
• Of the three fossil fuels (Petroleum, natural gas and coal), coal has the most widely distributed
reserves; coal is mined in over 100 countries, and on all continents except Antarctica. The largest proved
reserves are found in the United States, Russia, China, Australia and India. The United States is the
country with the world's largest proven coal reserves. As of 2020, coal reserves in the country
amounted to nearly 248.9 billion metric tons. Russia ranked second, having proved coal reserves of
162.2 billion metric tons. Hence statement 1 is correct.

19 www.visionias.in ©Vision IAS

FREE BY KING R QUEEN P [ऋषभ राजपूत]


• A proved recoverable reserve is the tonnage of coal that has been proved by drilling etc. and is
economically and technically extractable. Coal is found majorly in forms of Lignite and Anthracite.
• In terms of production, China is the top coal producer since 1983. In 2011 China produced 3,520
millions of tonnes (mt) of coal – 49.5% of 7,695 million tonnes world coal production. In 2011 other large
producers were United States (993 mt), India (589 mt), European Union (576 mt) and Australia (416
mt). Hence statement 2 is correct.

• Top coal exporting countries are Indonesia and Australia. Hence statement 3 is not correct.

Q 54.C
• San Francisco is a commercial, financial, and cultural center of Northern California. The city proper is
the fourth most populous in California. It is located on the West Coast of the United States at the north
end of the San Francisco Peninsula and includes significant stretches of the Pacific Ocean and San
Francisco Bay within its boundaries. Hence option 1 is correct.
• Vladivostok is a major Pacific port city in Russia overlooking Golden Horn Bay, near the borders with
China and North Korea. It's known as the terminus of the Trans-Siberian Railway, which links the city to
Moscow in a 7-day journey. Hence option 2 is correct.
• Wuhan is the sprawling capital of Central China’s Hubei province, is a commercial center divided by
the Yangtze and Han rivers. It is not located on the sea coast. Hence option 3 is not correct.
• Melbourne is the capital and most populous city of the Australian state of Victoria, and the second-
most populous city in both Australia and Oceania. The metropolis occupies much of the northern and
eastern coastlines of Port Phillip Bay. Hence option 4 is correct.
20 www.visionias.in ©Vision IAS

FREE BY KING R QUEEN P [ऋषभ राजपूत]


Q 55.B
• The Rhine is a significant European river that originates in the south-eastern Swiss Alps in the Swiss
canton of Graubünden, and flows through the borders of Switzerland, Liechtenstein, Austria, and
Germany. It Flows through six countries: Switzerland, Liechtenstein, Austria, Germany, France, and the
Netherlands. It has a total length of approximately 1,233 km. Hence pair 1 is not correctly matched.
• The Danube is Europe's second-longest river Stretching through a significant part of Central and South-
eastern Europe. The river's source is in Donaueschingen, a town situated in the Black Forest of
Germany, which ends up in the Black Sea. Hence pair 2 is correctly matched.
• The Volga originates in Russia, and flows through Central and Southern Russia, eventually reaching the
Caspian Sea. It is the longest river of Europe. Hence pair 3 is correctly matched.

Q 56.C
• ‘Mangrove Initiative for Shoreline Habitats & Tangible Incomes’, MISHTI, is a scheme launched in
the recent budget. Under this scheme, multiple activities will be taken up focusing on mangrove
plantations along the coastline, on saltpan lands, and wherever feasible. The funds for the same would be
sourced from the convergence between MGNREGS, CAMPA Fund, and others. Hence, option (c) is
the correct answer.
• Mangroves have been the focus of conservationists for years and it is difficult to overstate their
importance in the global climate context. It provides multidimensional benefits as-
o Ecological- Biodiversity conservation, Groundwater recharge, habitat for multiple species including
the Royal Bengal tigers, etc.
o Economical- Mangrove forests offer a good ground for fisheries, these also provide access to minor
forest produce as well as timber.
o Social- Mangroves form a part of many cultures, they bind the community, host the local forest
deities and also provide avenues for tourism.
o Disaster Management- Mangroves and their roots can successfully moderate the speed of incoming
waves and storms thus protecting the shorelines, communities, and precious lives.

Q 57.A
• The Securities and Exchange Board of India (SEBI) strengthened the framework for green bonds
by introducing the concept of ‘blue’ and ‘yellow’ bonds as new modes of sustainable finance. Yes
Bank issued India's first green bond in 2015 to raise INR 5 billion. Hence, statement 1 is correct.
• The main objective of this amendment is to expand the definition of ‘green debt security’ and incidental
matters. It has also included new modes of sustainable finance in relation to pollution prevention and
control and eco-efficient products.
• These actions were taken against the backdrop of growing interest in sustainable finance both in India and
around the world. They also aim to align the existing framework for green debt securities (GBP) with the
updated Green Bond Principles, which are recognized by IOSCO.
• Blue bonds are modes of sustainable finance raised for sustainable maritime sector including
sustainable fishing, sustainable water management etc.
• Yellow bonds are modes of sustainable finance raised for solar energy generation and the associated
upstream and downstream industries. Hence, statement 2 is not correct.

Q 58.B
• Maize is a food as well as a fodder crop grown under semi-arid climatic conditions and over inferior soils.
It requires 50-100 cm of rainfall and a temperature ranging from 21°C to 27°C.
• Maize is sown all over India except in eastern and northeastern regions. The leading producers of maize
are the states of Madhya Pradesh, Andhra Pradesh, Karnataka, Rajasthan, and Uttar Pradesh. Yield level
of maize is higher than other coarse cereals. It is high in southern states and declines towards central
parts. Hence, option (b) is the correct answer.
• In India, maize is the third most important food crop after rice and wheat. According to advanced
estimates, it is cultivated in 8.7 m ha (2010-11) mainly during the Kharif season which covers 80% area.
Maize in India contributes nearly 9 % of the national food basket and more than Rs. 100 billion to the
agricultural GDP at current prices apart from generating employment to over 100 million man-days at the
farm and downstream agricultural and industrial sectors.
• In addition to staple food for human being and quality feed for animals, maize serves as a basic raw
material as an ingredient in thousands of industrial products that includes starch, oil, protein, alcoholic
beverages, food sweeteners, pharmaceutical, cosmetic, film, textile, gum, package, and paper industries,
etc.
21 www.visionias.in ©Vision IAS

FREE BY KING R QUEEN P [ऋषभ राजपूत]


Q 59.D
• In a recent move aimed at improving the organ donation and transplantation system in India, the Ministry
of Health and Family Welfare (MoH&FW) has introduced several changes to the National Organ
Transplantation Guidelines.
• New Guidelines:
o No domicile requirement: A citizen can now register for organ donation in any state, previous
requirement of registering in the state of domicile has been removed. Hence statement 1 is
correct.
o Upper limit age: New Guidelines allow even those above 65 years of age to register to receive an
organ for transplantation from deceased donors. Hence statement 2 is correct.
✓ Under the previous guidelines established by the National Organ and Tissue Transplant
Organization (NOTTO), patients over the age of 65 suffering from end-stage organ failure were
prohibited from registering to receive an organ for transplantation.
o No registration fees: Centre has asked states to stop governments to stop taking fees to register
a patient for organ transplants. Hence statement 3 is correct.
• These developments come in the backdrop of when MoH&FW is working on One Nation-One Policy for
organ donation and transplantation.

Q 60.C
• Recent Context: NSE Indices Ltd., an NSE (National Stock Exchange) arm has introduced the country's
first ever municipal bond index.
• About Nifty India Municipal Bond Index:
o Role of the index: The new Nifty India Municipal Bond Index will track the performance of
municipal bonds issued by Indian municipal corporations across maturities and having investment
grade credit rating. Hence statement 1 is correct.
o Index constituents: Presently, the index has 28 municipal bonds issued by 10 issuers all having credit
rating in the AA category. The index constituents are assigned weights based on their outstanding
amount.
o Base for Index: The index has a base date of January 1, 2021, and a base value of 1,000.
o Review: The index will be reviewed quarterly. Hence statement 2 is correct.

• Municipal Bond Market in India


o Municipal Bond Market is regulated by the Securities and Exchange Board of India (SEBI) in
India.
o The Indian municipal bond market has seen a resurgence of issuances after SEBI's Issue and Listing
of Municipal Debt Securities Regulations, 2015 came into effect.
o The Government of India has also provided incentives in the form of a lump-sum grant-in-aid
for municipal bond issuances.
o Largely investment-grade rating: 59% of municipal bonds issued received a rating of investment
grade or above, highlighting the underutilized potential for bond financing by Indian municipalities.

Q 61.B
• A population pyramid, also called an age-sex pyramid, is a graphical illustration that shows the
distribution of various age groups in a population (typically that of a country or region of the world),
which normally forms a pyramid. It typically consists of two back-to-back bar graphs, with the
population plotted on the X-axis and age on the Y-axis, one showing the number of males and one
showing females in a particular population in five-year age groups. Hence option (b) is the correct
answer.
• Males are conventionally shown on the left and females on the right, and they may be measured by a raw
number or as a percentage of the total population. The age–sex structure reflects the demographic and
22 www.visionias.in ©Vision IAS

FREE BY KING R QUEEN P [ऋषभ राजपूत]


socioeconomic history of a population over a period of time and, even their prospects for the future. It is
the result of various factors, such as fertility, mortality, and migration.
• Four main types of age-sex pyramids have been identified:
o A progressive age structure is one in which both birth and death rates are high. Such a structure
is common in developing countries where social, cultural, and perhaps religious and economic
conditions lead to high fertility and poor living conditions, bad diets, and little medical aid lead to
high levels of mortality.
o A regressive age structure is one in which birth and death rates are low and declining. This
pattern is common in developed countries (especially those in Western Europe), where high living
standards, education and social awareness are accompanied by good food and medicine.
o A stationary age structure is one in which birth and death rates are both low, children account for
about 35–40 percent of the total population, and the aged for about 10 percent. This pattern may
remain the same for many years.
o An intermediate age structure may vary in character and is most common in countries that are
passing through stages of development. Such countries may once have had progressive structures and
may, in the future, have regressive structures.

Q 62.C
• The Ruhr Coal field is located in Germany. It is responsible for 80 per cent of Germany’s total steel
production. Hence, pair 1 is correct.
• Duluth is located in northeastern Minnesota in the United States of America. It is one of the
important iron and steel-producing centres in the U.S. Besides this, most of the production in the U.S.A.
comes from the northern Appalachian region (Pittsburgh), Great Lake region (Chicago-Gary, Erie,
Cleveland, Lorain, Buffalo) and the Atlantic Coast (Sparrows Point and Morrisville). Hence, pair 2 is not
correct.
• Port Talbot is an important iron and steel centre in the United Kingdom. Other centres in the U.K. are
Scun Thorpe, Port Talbot, Birmingham and Sheffield. Hence, pair 3 is correct.
• Kryvyi Rih, also known as Krivoi Rog, is a city in central Ukraine. It is part of the Kryvyi Rih
Metropolitan Region. Hence, pair 4 is correct.
• Important iron and steel centres in Russia are Le Creusot and St. Ettiennein France; and Moscow, St.
Petersburgh, Lipetsk, and Tula.
• In Asia, the important centres include Nagasaki and Tokyo-Yokohamain Japan; Shanghai, Tienstin and
Wuhan in China; and Jamshedpur, Kulti-Burnpur, Durgapur, Rourkela, Bhilai, Bokaro, Salem,
Visakhapatnam and Bhadravati in India.

Q 63.A
• Footloose industries: Footloose industries can be located in a wide variety of places. They are not
dependent on any specific raw material, weight loss or otherwise. Hence, statement 1 is correct.
o They largely depend on component parts which can be obtained anywhere. They produce in small
quantities and also employ a small labour force. Hence, statement 2 is not correct.
o These are generally not polluting industries. The important factor in their location is accessibility by
road network.
o These are called footloose as these types of industries are prone to relocation. For example, diamond
and computer chips belong to the footloose industry.
o Important characteristics of footloose industries:
✓ Location: Footloose industries can be established at any place. These industries are affected by
component parts and they are available in all places.
✓ Less labour force: These industries produce their products in small numbers and they do not
require a large labour force.
✓ Eco-friendly: These are environment-friendly industries as the process involved in these
industries have a negligible carbon footprint. These industries emit less or no pollution.
✓ Less transport cost: Their products are having very high-value addition and are smaller in size
and so transportation cost is only a small fraction of the total cost.
✓ Small plant size: These industries require a small plant size compared to heavy and small
industries.
✓ Less raw material dependence: These are less dependent on specific raw materials, especially
weight-losing ones. Most of the raw materials are small and light and can be transported easily.
✓ Skilled workers: It needs skilled workers as the industrial process is advanced and major work
needs high-quality precision.
23 www.visionias.in ©Vision IAS

FREE BY KING R QUEEN P [ऋषभ राजपूत]


Q 64.A
• The land-use categories as maintained in the Land Revenue Records are as follows:
o Forests: According to the Survey of India report, a forest area is one that is notified by the department
as land under forests, irrespective of whether it has any tree cover or not. The land under forest cover
is the land exceeding one-hectare area having a minimum of 10 percent tree cover irrespective of any
other land use. Thus, the area under actual forest cover may be different from the area classified as
forest. Hence, there may be an increase in this category without any increase in the actual forest
cover.
o Barren and Wastelands: The land classified as a wasteland such as barren hilly terrains, desert lands,
ravines, etc. normally can not be brought under cultivation with the available technology. They
remain non-suitable for agriculture and generally remain fallow.
o Current Fallow: The land which has been left without cultivation for one or less than one agricultural
year is known as current fallow. The practice adopted for giving rest to the culturable land is called
the following. The land recoups the lost fertility through natural processes over a time duration.
o Net Area Sown: Net area sown represents the area sown with crops at least once in any of the crop
seasons of the year counting area sown more than once in the same year, only once.Net sown area is
of crucial importance for India because it is the land actually under cultivation of crops and India has
the highest percentage of Net Sown Area.

• Hence, option (a) is the correct answer.

Q 65.C
• In India, Land-use records are maintained by the Land revenue department. The land use categories add
up to Reporting area, which refers to the total area reported by the Land Revenue
department. Hence, statement 2 is not correct.
• The Reporting area stands for the area for which data on land use classification are available. In areas
where land utilization figures are based on land records, reporting area is the area according to village
papers, i.e. the papers prepared by the village accountants. In some cases, the village papers may not be
maintained in respect of the entire area of the State. For example, village papers are not prepared for the
forest areas but the magnitude of such areas is known. Also, there are tracts in many States for which no
village paper exists. In such cases, estimates of classification of the area from the agricultural census,
1995-96 and 2000-01 are adopted to complete the coverage.
• The land uses categories add up to reporting area, which is somewhat different from the geographical
area. The Survey of India is responsible for measuring the geographical area of administrative units
in India. Hence, statement 1 is not correct.
• The latest figures of the geographical area of the State/Union Territories are as provided by the Office of
the Surveyor General of India and remain fixed as per the international boundaries.

Q 66.B
• Development means a qualitative change that is always value positive. Development occurs when
positive growth takes place. Yet, positive growth does not always lead to development. Hence
statement 1 is correct.
24 www.visionias.in ©Vision IAS

FREE BY KING R QUEEN P [ऋषभ राजपूत]


• The growth is quantitative and value-neutral. It may have a positive or a negative sign. This means
that the change may be either positive (showing an increase) or negative (indicating a decrease).
Hence statement 2 is correct.
• If the population of a city grows from one lakh to two lakhs over a period of time, which can be said to
grow. However, if facilities like housing, provision of basic services, and other characteristics remain the
same, then this growth has not been accompanied by development. Thus growth (including population
growth does not always lead to development). Hence statement 3 is correct.

Q 67.D
• The term population distribution refers to the way people are spaced over the earth’s surface.
Broadly, 90 per cent of the world population lives in about 10 per cent of its land area.
• Factor that influence population distribution are Landforms: People prefer living on flat plains and
gentle slopes. This is because such areas are favourable for the production of crops and to build roads and
industries. The mountainous and hilly areas hinder the development of transport network.
• Climate: An extreme climate such as very hot or cold deserts are uncomfortable for human habitation.
Areas with a comfortable climate, where there is not much seasonal variation attract more people.
• Availability of water: Water is the most important factor for life. So, people prefer to live in areas where
fresh water is easily available. Water is used for drinking, bathing and cooking – and also for cattle, crops,
industries and navigation.
• Minerals: Areas with mineral deposits attract industries. Mining and industrial activities generate
employment. So, skilled and semi–skilled workers move to these areas and make them densely populated.
• Urbanisation: Cities offer better employment opportunities, educational and medical facilities, better
means of transport and communication.
• Hence option (d) is the correct answer.

Q 68.D
• Rural settlements may be classified on the basis of forms or shapes of the settlements: These may be a
number of geometrical forms and shapes such as Linear, rectangular, circular star like, T-shaped village,
double village, cross-shaped village etc.
• (a) Linear pattern: In such settlements houses are located along a road, railway line, river, canal edge of a
valley or along a levee.
• (b) Rectangular pattern: Such patterns of rural settlements are found in plain areas or wide inter
montane valleys. The roads are rectangular and cut each other at right angles. Hence statement 1 is
correct.
• (c) Circular pattern: Circular villages develop around lakes, tanks and sometimes the village is planned in
such a way that the central part remains open and is used for keeping the animals to protect them from
wild animals.
• (d) Star like pattern: Where several roads converge, star shaped settlements develop by the houses built
along the roads.
• (e) T-shaped, Y-shaped, Cross-shaped or cruciform settlements: T -shaped settlements develop at tri-
junctions of the roads (T) while Y-shaped settlements emerge as the places where two roads
converge on the third one and houses are built along these roads. Hence statement 2 is correct.
• Cruciform settlements develop on the cross-roads and houses extend in all the four direction. Hence
statement 3 is correct.

Q 69.A
• Coffee is a tropical plantation crop. Its seeds are roasted, and ground and are used for preparing a
beverage. There are three varieties of coffee i.e. arabica, robusta and liberica.
• Coffee is cultivated in the highlands of Western Ghats in Karnataka, Kerala, and Tamil Nadu.
Karnataka alone accounts for more than two-thirds of the total production of coffee in the country. Hence,
statement 1 is correct.
• Coffee is largely produced in the southern part of India. Karnataka is the largest producer accounting for
about 70% of the total coffee production in India. Kerala is the second-largest producer of coffee but lags
far behind, accounting only for about 23% of the total production. Tamil Nadu is the third-largest
producer, where India’s 6% of the coffee is produced. Nearly half of Tamil Nadu’s coffee is made in the
Nilgiri district, a major Arabica growing region. Orissa and the northeastern areas have a smaller
proportion of production.
• There are three varieties of coffee i.e. arabica, robusta, and liberica. India mostly grows superior quality
coffee, arabica, which is in great demand in the International market. Indian coffee is one of the best
25 www.visionias.in ©Vision IAS

FREE BY KING R QUEEN P [ऋषभ राजपूत]


coffees in the world due to its high quality and gets a high premium in the international markets. India
produces two types of coffee: Arabica and Robusta. Arabica has high market value than Robusta coffee
due to its mild aromatic flavor. Robusta coffee is mainly used in making various blends due to its strong
flavor. Robusta is the majorly manufactured coffee with a share of 72% of the total production. The
industry provides direct employment to more than 2 million people in India. Since coffee is mainly an
export commodity for India, domestic demand and consumption do not drastically impact the prices of
coffee. Hence, statement 2 is not correct.
• India is among the top 10 coffee-producing countries, with about 3% of the global export in 2020, and
ranks seventh after Brazil, Vietnam, Colombia, Indonesia, Ethiopia, and Mexico. Hence, statement 3 is
not correct.

Q 70.A
• It is widely accepted that settlements can be differentiated in terms of rural and urban, but there is no
consensus on what exactly defines a village or a town.
• Although population size is an important criterion, it is not a universal criterion since many villages in
densely populated countries of India and China have population exceeding that of some towns of Western
Europe and United States.
• At one time, people living in villages pursued agriculture or other primary activities, but presently in
developed countries, large sections of urban populations prefer to live in villages even though they work
in the city.
• The basic difference between towns and villages is that in towns the main occupation of the people is
related to secondary and tertiary sectors, while in the villages most of the people are engaged in primary
occupations such as agriculture, fishing, lumbering, mining, animal husbandry, etc.
• Sub Urbanisation:
o It is a new trend of people moving away from congested urban areas to cleaner areas outside the
city in search of a better quality of living. Hence statement 1 is correct.
o Important suburbs develop around major cities and everyday thousands of people commute
from their homes in the sub urbs to their work places in the city. Hence statement 2 is not
correct.

Q 71.D
• The Rhine flows through Germany and the Netherlands. It is navigable for 700 km from Rotterdam, at its
mouth in the Netherlands to Basel in Switzerland. Ocean-going vessels can reach up to Cologne. The
Ruhr river joins the Rhine from the east. It flows through a rich coalfield and the whole basin has become
a prosperous manufacturing area. Dusseldorf is the Rhine port for this region. Huge tonnage moves along
the stretch south of the Ruhr. This waterway is the world’s most heavily used.
• It connects the industrial areas of Switzerland, Germany, France, Belgium, and the Netherlands with
the North Atlantic Sea Route. Hence, statement 1 is correct.
• The Mississippi-Ohio waterway connects the interior part of U.S.A. with the Gulf of Mexico in the
south. Large steamers can go through this route up to Minneapolis. Hence, statement 2 is correct.

26 www.visionias.in ©Vision IAS

FREE BY KING R QUEEN P [ऋषभ राजपूत]


Q 72.A
• Recent context: The National Green Hydrogen Mission, approved by the Union Cabinet in January 2023,
identifies a significant role for green hydrogen in decarbonizing the steel sector to meet India’s climate
goals.
• Steel is a key sector of the Indian economy. India is the world’s second-largest producer of crude
steel and second-largest consumer of finished steel. In FY 21-22, the sector contributed approximately
2% to the country’s GDP and provided approximately 20 lakh jobs. Moreover, the sector is set for
significant growth: the National Steel Policy has set a target to reach 300 million tonnes (MT) of annual
production by 2030 from the existing level of 120 MT. Hence, statement 1 is correct.
• The availability of scrap is a major issue in India and in 2017 the deficit was to the tune of 7 million Tons.
This was imported at the cost of more than Rs. 24,500 crores (approx.) in 2017-18. The gap between
demand and supply is can be reduced in the future and the country may be self-sufficient by 2030. This is
mainly because, with the increase in consumption of steel in the recent past and end-of-life vehicles
(ELV), the generation of scrap is likely to be increased considerably. This scrap has to be channelized so
that the same can be utilized for steel production in an environmentally friendly manner.
• Steel Authority of India Limited (SAIL) is the largest steel-making company in India and one of the
seven Maharatnas of the country's Central Public Sector Enterprises. Hence, statement 2 is correct.
• National Steel Policy 2017 (NSP-2017) aims to develop a globally competitive steel industry by creating
300 Million Tonnes Per Annum (TPA) Steel production capacity by 2030 with a contribution of 35-40%
from the Electric Arc Furnaces (EAF)/Induction Furnaces (IF) route. Although, scrap is the main raw
material for the secondary sector primary sector also uses Scrap in the charge mix of BOF to the tune of
15% to improve efficiency, and minimize the cost of production and other process needs. The availability
of raw materials at competitive rates is imperative for the growth of the steel industry and to achieve the
NSP-2017 target. Thus, the availability of the right quality of scrap, in adequate quantity is one of the
critical factors for the future growth of both the EAF/IF sector & primary sector. Hence, statement 3 is
not correct.

Q 73.A
• Dispersed Settlements: In these settlements, houses are spaced far apart and often interspersed with
fields. A cultural feature such as a place of worship or a market, binds the settlement together.
• Compact or Nucleated settlements: These settlements are those in which large number of houses are built
very close to each other. Such settlements develop along river valleys and in fertile plains. Communities
are closely knit and share common occupations.
• On the basis of forms or shapes of the settlements: These may be a number of geometrical forms and
shapes such as Linear, rectangular, circular star like, T-shaped village, double village, cross-shaped
village etc.
• Hence, option (a) is the correct answer.

Q 74.C
• Recent Context: Union Minister of State for Science and Technology has said that North India's first
nuclear power plant will come up in Haryana in the village of Gorakhpur in Fatehabad district, 150 km
north of the national capital.
• India plans to commission 20 nuclear power plants by 2031, adding nearly 15,000 MW in power
generating capacity.
• The first of these 20 nuclear power plants, a 700 MW unit, is expected to be commissioned in 2023 at
Kakrapar in Gujarat, which already has three atomic power generating units operational.
• According to a written reply by Minister of State in the PMO, the 500 MW Prototype Fast Breeder
Reactor at Kalpakkam is likely to be operational in 2024, followed by two 1,000 MW units at
Kudankulam in 2025.
o Two 700 MW units at Rawatbhata in Rajasthan are likely to be completed by 2026, while another
two 1,000 MW units are likely to be completed at Kudankulam by 2027,
o Two 700 MW units are expected to be completed at Gorakhpur in Haryana by 2029.
• Nuclear power is the fifth-largest source of electricity in India after coal, gas, hydroelectricity and wind
power. As of November 2020, India has 22 nuclear reactors in operation in 8 nuclear power plants, with a
total installed capacity of 7,380 MW.
• Rawatbhata Plant is situated in Rajasthan, Tarapur Plant is situated in Maharashtra and Tamil
Nadu has Kalapakkam Nuclear power plant..
• Hence, option (c) is the correct answer.

27 www.visionias.in ©Vision IAS

FREE BY KING R QUEEN P [ऋषभ राजपूत]


Q 75.C
• The Survey of India is responsible for measuring the geographical area of administrative units in India.
The land-use categories as maintained in the Land Revenue Records are as follows:
• Forests, Land put to Non-agricultural Uses, Barren and Wastelands, Area under Permanent Pastures and
Grazing Lands, Area under Miscellaneous Tree Crops and Groves(Not included is Net sown Area),
Culturable Waste-Land, Current Fallow, Fallow other than Current Fallow, and Net Area Sown.
• Current Fallow is the land that is left without cultivation for one or less than one agricultural year. It
is a cultural practice adopted for giving land rest. The land recoups the lost fertility through natural
processes.
• Fallow other than Current Fallow is the cultivable land that is left uncultivated for more than a year
but less than five years. If the land is left uncultivated for more than five years, it would be categorized as
a culturable wasteland. Hence, statement 1 is not correct.
• Culturable Waste-Land is any land that is left fallow (uncultivated) for more than five years and is
included in this category. Thus, if the land is left uncultivated for more than five years, it is categorized as
Culturable Waste-Land and not Barren and Wasteland. Hence, statement 2 is not correct.
• Barren and Wastelands includes wasteland such as barren hilly terrains, desert lands, ravines, etc. While
the Culturable Waste-Land can be brought under cultivation after improving it through
reclamation practices the Barren and Wastelands cannot be brought under cultivation with the available
technology. Hence, statement 3 is correct.
• The physical extent of land on which crops are sown and harvested is known as net sown area.

Q 76.A
• Components of Population Change
o There are three components of population change – births, deaths, and migration. Hence option (a) is
the correct answer.
o The crude birth rate (CBR) is expressed as the number of live births in a year per thousand of the
population. Death rate plays an active role in population change. Population growth occurs not only
by increasing birth rate but also due to decreasing death rate.
o Crude Death Rate (CDR) is a simple method of measuring mortality in any area. CDR is expressed in
terms of the number of deaths in a particular year per thousand of the population in a particular region
o By and large mortality rates are affected by the region’s demographic structure, social advancement,
and levels of economic development.
o Apart from birth and death migration is another way by which the population size changes.

Q 77.C
• Quaternary activities involve the collection, production and dissemination of information or even the
production of information. Quaternary activities center around research, and development and may be
seen as an advanced form of services involving specialized knowledge and technical skills.
• The Quaternary Sector along with the Tertiary Sector has replaced most of the primary and secondary
employment as the basis for economic growth. Over half of all workers In developed economies are in the
‘Knowledge Sector’. Personnel working in office buildings, elementary schools and university
classrooms, hospitals and doctors’ offices, theatres, and accounting and brokerage firms all belong to
this category of services. Hence, points 1 and 4 are correct.
• Quinary activities are services that focus on the creation, re-arrangement and interpretation of new and
existing ideas; data interpretation and the use and evaluation of new technologies. Often referred to as
‘gold collar’ professions, they represent another subdivision of the tertiary sector representing special
and highly paid skills of senior business executives, government officials, research scientists, financial
and legal consultants, etc. Hence, point 2 is not correct.
• Home shoring is a new trend in Quinary activities. It has emerged as an alternative to outsourcing.
Home shoring is an organizational operational model in which employees work and perform all official
tasks from a home or external office. Homeshoring is the hiring, management and tasking of employees
remotely, usually over the Internet, although it may include other forms of digital communication. Hence,
point 3 is not correct.

28 www.visionias.in ©Vision IAS

FREE BY KING R QUEEN P [ऋषभ राजपूत]


Q 78.B

• Hence, option (b) is the correct answer.

Q 79.A
• Since 1990, the United Nations Development Programme (UNDP) has been publishing the Human
Development Report every year. This report provides a rank-wise list of all member countries according
to the level of human development. The Human Development Index and the Human Poverty index are
two important indices to measure human development used by the UNDP. Hence statement 1 is correct.

29 www.visionias.in ©Vision IAS

FREE BY KING R QUEEN P [ऋषभ राजपूत]


• Countries with very high human development index are those which have a score of over 0.800. Hence
statement 3 is not correct.
• The concept of human development was introduced by Dr. Mahbub-ul-Haq. Dr. Haq has described human
development as development that enlarges people’s choices and improves their lives. People are central to
all development under this concept. These choices are not fixed but keep on changing. The basic goal of
development is to create conditions where people can live meaningful lives. A man of vision and
compassion, Pakistani economist Dr. Mahbub-ul-Haq created the Human Development Index in 1990.
Hence statement 2 is not correct.

Q 80.D
• The process by which fertility rates eventually decline to low and stable levels has been called
demographic transition. The fertility rate is defined as the average number of children per woman in the
reproductive age group. Demographic transition postulates three-stage sequences of birth and death rates
that are associated with economic development.
• First Stage of Demographic Transition or High Fluctuating Stage
o In this stage, the death rates are high due to the absence of effective medical aid, primitive sanitation,
and poor diets. The birth rates are also high on account of the lack of knowledge about family
planning techniques, the early age of marriage, illiteracy and deep-rooted social beliefs, and customs
about the size of the family, including as an insurance against high child mortality rates.
o The actual rate of growth of the population is low since a high birth rate is balanced by a high death
rate.
• Second Stage of Demographic Transition or Expanding Stage
o With economic development resulting in high incomes and improvements in public health facilities,
there is a marked decline in mortality that raises life expectancy from under 40 years to 60 years.
o However, the decline in death rate is not immediately accompanied by a decline in fertility. In this
stage of demographic transition, with a declining death rate, the birth rate does not fall
correspondingly.
o This leads to the transition from a stable or slow-growing population to one with a rapidly increasing
population. The second stage is further classified into Early Expanding Stage and Late Expanding
Stage.
• Third Stage of Demographic Transition or Low Fluctuating Stage
o The forces and influences of modernisation (including an increase in female work participation rate
and a move towards nuclear families) and economic development cause the fertility rate to decline so
that the falling birth rate eventually converges with the death rate, leaving little or no population
growth.
o The characteristics of the third stage are a low birth rate, a low death rate, a small family size, and a
low growth rate of the population.
o This demographic transition has been witnessed in contemporary developed nations as they developed
and one can identify the developing nations as they move through the different stages of this
transition.

• Hence option (d) is the correct answer.


30 www.visionias.in ©Vision IAS

FREE BY KING R QUEEN P [ऋषभ राजपूत]


Q 81.B
• Iron and Steel Industry: The iron and steel industry forms the base of all other industries and, therefore,
is called a basic industry. It is basic because it provides the raw materials for other industries such as
machine tools used for further production. It may also be called a heavy industry because it uses large
quantities of bulky raw materials and its products are also heavy.
o Distribution:
✓ The industry is one of the most complex and capital-intensive industries and is concentrated in the
advanced countries of North America, Europe, and Asia.
✓ In the U.S.A, most of the production comes from the northern Appalachian region (Pittsburgh),
Great Lake region (Chicago-Gary, Erie, Cleveland, Lorain, Buffalo, and Duluth), and the Atlantic
Coast (Sparrows Point and Morrisville).
✓ The industry has also moved towards the southern state of Alabama. Pittsburg area is now losing
ground. It has now become the “rust bowl” of the U.S.A. Hence pair 2 is not correctly
matched.
✓ In Europe, the U.K., Germany, France, Belgium, Luxembourg, the Netherlands, and Russia are
the leading producers. The important steel centers are Scun Thorpe, Port Talbot, Birmingham, and
Sheffield in the U.K.; Duisburg, Dortmund, Dusseldorf, and Essen in Germany; Le Creusot and
St. Ettienne in France; and Moscow, St. Petersburgh, Lipetsk, Tula, in Russia and Kryvyi Rih,
and Donetsk in Ukraine. Hence pair 1 is correctly matched.
✓ In Asia, the important centers include Nagasaki and Tokyo-Yokohama in Japan; Shanghai,
Tienstin, and Wuhan in China; and Jamshedpur, Kulti-Burnpur, Durgapur, Rourkela, Bhilai,
Bokaro, Salem, Visakhapatnam and Bhadravati in India. Hence pair 3 is correctly matched.

Q 82.D
• Madrid is the capital city of Spain.
• Lisbon is the capital city of Portugal.
• Brussels is the capital city of Belgium.
• Vienna is the capital city of Austria.

• Hence option (d) is the correct answer.

Q 83.B
• India is surrounded by sea from three sides and is bestowed with a long coastline. Water provides a
smooth surface for very cheap transport provided there is no turbulence. India has a long tradition of
seafaring and developed many ports with place names suffixed with pattan meaning port. An interesting
fact about ports in India is that its west coast has more ports than its east coast. Hence, statement 1 is
correct.
• Kochchi Port, situated at the head of Vembanad Kayal, popularly known as the ‘Queen of the Arabian
Sea’, is also a natural harbor. This port has an advantageous location being close to the Suez-Colombo
route. It caters to the needs of Kerala, southern Karnataka and southwestern Tamil Nadu. Hence,
statement 2 is correct.
• Haldia Port is located 105 km downstream from Kolkata on the Hooghly river. It has been constructed to
reduce the congestion at Kolkata port. It handles bulk cargo like iron ore, coal, petroleum, petroleum
products and fertilizers, jute, jute products, cotton and cotton yarn, etc. Hence, statement 3 is not
correct.
31 www.visionias.in ©Vision IAS

FREE BY KING R QUEEN P [ऋषभ राजपूत]


Q 84.D
• The aim of the India-Maldives-Sri Lanka Tri-Lateral Exercise ‘DOSTI’ is to further fortify the
friendship, enhance the mutual operational capability, exercise interoperability, and to build cooperation
between the Coast Guards of Maldives, India, and Sri Lanka.
• The Exercises carried out over the past ten years have focused on exercises and drills on providing
assistance in sea accidents, eliminating sea pollution, and the Coast Guard's procedure and conduct during
situations such as oil spills.
• Exercise Dosti was initiated in 1991, between the Indian and Maldives Coast Guard. Sri Lanka
joined the exercise for the first time in 2012.
• Hence, option (d) is the correct answer.

Q 85.B
• The polar type of climate and vegetation are found mainly north of the Arctic Circle in the northern
hemisphere. The ice caps are confined to Greenland and to the highlands of these high-latitude regions,
where the ground is permanently snow-covered. The lowlands, with a few months ice-free, have tundra
vegetation.
• In such an adverse environment as the tundra, few plants survive. The greatest inhibiting factor is the
region's deficiency in heat. With a growing season of fewer than three months and the warmest month not
exceeding 50°F. (the tree-survival line), there are no trees in the tundra. Such an environment can
support only the lowest form of vegetation, mosses, lichens and sedges. Drainage in the tundra is
usually poor as the subsoil is permanently frozen. Hence statement 2 is correct.
• Despite the deficiency of heat Tundra vegetation does support a few species of mammals. This
includes herbivorous animals like reindeer, wolves, foxes, musk-ox, Arctic hares, etc.
• In the brief summer, when the snows melt and the days are warmer and longer, berry-bearing bushes and
Arctic flowers bloom. Though short-lived, they brighten the monotonous tundra landscape into Arctic
prairies'. In the summer, the tundra is full of activities. Birds migrate north to prey on the numerous
insects which emerge when the snow thaws. Hence statement 1 is not correct.

Q 86.B
• Indian Railways has launched an extensive programme to convert the metre and narrow gauges to broad
gauges since 1992.
• The gauge of a railway track is defined as the clear minimum perpendicular distance between the inner
faces of the two rails.
• On the basis of the width of the track of the Indian Railways, three categories have been made:
o Broad gauge: The distance between rails in broad gauge is 1.676 metres. The total length of broad
gauge lines was 86526 km. Hence statement 1 is not correct.
o Metre gauge: The distance between rails is one metre. Its total length was 15529 km.
o Narrow gauge: The distance between the rails in this case is 0.762 metres or 0.610 metres. The total
length of the narrow gauge was 3651 km. It is generally confined to hilly areas.
• Narrow gauge railways usually cost less to build because they are usually lighter in construction, using
smaller cars and locomotives (smaller loading gauge), as well as smaller bridges, smaller tunnels (smaller
structure gauge) and tighter curves. Narrow gauge is thus often used in mountainous terrain, where
the savings in civil engineering work can be substantial. Hence statement 2 is correct.
• It is also used in sparsely populated areas, with low potential demand, and for temporary railways that will
be removed after short-term use, such as for construction, the logging industry, the mining industry, or
large-scale construction projects, especially in confined spaces.
• Broader gauge railways are generally more expensive to build but are able to handle heavier and faster
traffic.

Q 87.C
• Recently, Indian Space Research Organisation (ISRO) received the NASA-ISRO SAR (NISAR)
satellite from the U.S. space agency.
• NASA-ISRO SAR (NISAR) is a Low Earth Orbit (LEO) observatory being jointly developed by
NASA and ISRO. Hence statement 1 is correct.
o NISAR will map the entire globe in 12 days and provide spatially and temporally consistent data for
understanding changes in Earth’s ecosystems, ice mass, etc.
o The 2,800 kilograms satellite consists of both L-band and S-band synthetic aperture radar
(SAR) instruments, which makes it a dual-frequency imaging radar satellite. Hence statement 2
is correct.
32 www.visionias.in ©Vision IAS

FREE BY KING R QUEEN P [ऋषभ राजपूत]


o The SAR payloads mounted on Integrated Radar Instrument Structure (IRIS) and the spacecraft bus
are together called an observatory.
o NASA is to provide the L-band radar, GPS, a high-capacity solid-state recorder to store data,
and a payload data subsystem. ISRO is to provide the S-band radar, the GSLV launch system,
and spacecraft for the mission.
✓ After the commissioning period, the mission will conduct science operations with the L- band
radar to satisfy NASA’s requirements for a minimum of three years, while the S-band radar
will be used by India for its specific needs for a period of five years.
o NISAR will acquire data over the Indian Coasts and monitor annual changes in the bathymetry
along the deltaic regions.
o The NISAR mission will observe sea ice characteristics over the seas surrounding India’s
Antarctic polar stations, which can be used to detect marine oil spills and disseminate the spill
location during accidental oil seepage for preventive measures.
o It will provide a means of disentangling and clarifying spatially and temporally complex phenomena,
ranging from ecosystem disturbances, to ice sheet collapse and natural hazards including earthquakes,
tsunamis, volcanoes, and landslide.

Q 88.C
• Clustered Settlements: The clustered rural settlement is a compact or closely built-up area of houses. In
this type of village, the general living area is distinct and separated from the surrounding farms, barns, and
pastures.
o The closely built-up area and its intervening streets present some recognizable pattern or geometric
shape, such as rectangular, radial, linear, etc.
o Such settlements are generally found in fertile alluvial plains and in the northeastern
states. Sometimes, people live in compact villages for security or defense reasons, such as in the
Bundelkhand region of central India and in Nagaland. In Rajasthan, scarcity of water has necessitated
compact settlement for maximum utilization of available water resources. Hence statement 1 is
correct.
• Semi-Clustered Settlements: Semi-clustered or fragmented settlements may result from the tendency of
clustering in a restricted area of dispersed settlement. More often such a pattern may also result from
segregation or fragmentation of a large compact village.
o In this case, one or more sections of the village society choose or is forced to live a little away from
the main cluster or village. In such cases, generally, the land-owning and dominant community
occupies the central part of the main village, whereas people of lower strata of society and menial
workers settle on the outer flanks of the village. Such settlements are widespread in the Gujarat plain
and some parts of Rajasthan. Hence statement 2 is correct.

Q 89.B
• Recent Context: The government has integrated Bhasini Mission's capabilities with the unified
payment interface (UPI) ecosystem.
• Bhashini Mission is a local language translation mission that aims to break the barrier between
various Indian tongues by using available technology. It aims to build a National Public Digital
Platform for languages.
• Bhashini, the National Language Translation Mission (NLTM), was launched by the Hon’ble Prime
Minister in July 2022 to provide artificial intelligence (AI) and Natural Language Processing (NLP) based
open-source language technology solutions for speech and text translation through the Bhashini platform
to bridge the language barriers leveraging startup-academia ecosystem. These technologies would enable
organizations to create innovative and artificial intelligence-based multilingual interfaces including voice-
based interfaces in their websites and apps to provide better citizen services and digital resources. As of
date, 289 pre-trained AI models for language translation in 10 Indian languages have been made available
on the Bhashini platform.
• Hence, option (b) is the correct answer.

Q 90.D
• In finance, being short on an asset means investing in such a way that the investor will profit if the
value of the asset falls. This is the opposite of a more conventional "long" position, where the investor
will profit if the value of the asset rises.
• Hence, short selling means borrowing a stock that you do not own, selling the borrowed stock, and
then buying and returning the stock if and when the price drops. Here, the speculation is that
33 www.visionias.in ©Vision IAS

FREE BY KING R QUEEN P [ऋषभ राजपूत]


the value of the asset will decrease in the near future thus ensuring profit for the investor. Thus, short
selling is mostly done to earn in a falling market (bearish). Hence, statement 2 is not correct.
• For example, an investor borrows a certain number of shares from a broker and sells these shares in the
market for Rs 100. In the future, when the price of the shares falls to Rs 80, then the investor can buy
them at a lesser price, and return them to the broker, thereby making a profit of Rs 20. Hence, statement
1 is correct.
• At present, in a realistic sense, there is no short selling in India. It's done in the form of day trading,
futures trading, etc. But actual short selling – which involves borrowing shares for a multi-day horizon –
is absent in India.
• The Hindenburg Research LLC is an investment research firm with a focus on activist short-selling.
In a report, they said that the Adani Group has “substantial debt” and that the stock prices of the listed
companies are overvalued by 85%. Such a forecast caused panic selling making the stocks hit the
bottom of the market. Hence, statement 3 is correct.
• It is speculated that Hindenburg is thus involved in short selling of the Adani stocks (outside of
India) to make a profit.
• SEBI has said that it will conduct a detailed examination of the company's stocks.

Q 91.B
• Russia shares borders with 14 countries. They are Norway, Finland, Estonia, Latvia, Lithuania,
Poland, Belarus, Ukraine, Georgia, Azerbaijan, Kazakhstan, Mongolia, China, and North Korea.
To the south, Russia borders North Korea, China, Mongolia, and Kazakhstan, Azerbaijan, and Georgia.
To the southwest and west, it borders Ukraine, Belarus, Latvia, and Estonia, as well as Finland and
Norway.
• Russia does not share a border with Kyrgyzstan. Kyrgyzstan is bounded by Kazakhstan on the
northwest and north, by China on the east and south, and by Tajikistan and Uzbekistan on the south and
west.
• Hence option (b) is the correct answer.

Q 92.D
• The density of population is expressed as the number of persons per unit area. It is a crude measure of the
human-land relationship. It helps in getting a better understanding of the spatial distribution of population
in relation to land.
• The density of population in India (2011) is 382 persons per sq km. There has been a steady increase of
more than 200 persons per sq km over the last 50 years as the population density increased from 117
persons/ sq km in 1951 to 382 persons/sq km in 2011.
• The spatial variation of population densities in the country ranges from as low as 17 persons per sq km
in Arunachal Pradesh to 11,297 persons in the National Capital Territory of Delhi.
• Among the northern Indian states, Bihar (1102), West Bengal (1029) and Uttar Pradesh (828) have
higher densities, while Kerala (859) and Tamil Nadu (555) have higher densities among the peninsular
Indian states.
• States like Assam, Gujarat, Andhra Pradesh, Haryana, Jharkhand, Odisha have moderate densities. The
hill states of the Himalayan region and the north-eastern states of India (excluding Assam) have relatively
low densities, while the Union Territories (excluding the Andaman and Nicobar Islands) have very high
densities of population.
• Hence option (d) is the correct answer.

Q 93.A
• To promote Inland Water Transport (IWT) in the country, 111 waterways (including 5 existing and 106
new) have been declared National Waterways (NWs) under the National Waterways Act, 2016. The
Inland Waterways Authority of India (IWAI) was constituted in October 1986 for the development and
regulation of inland waterways for shipping and navigation.
• The Ganga - Bhagirathi - Hooghly river system from Allahabad to Haldia was declared National
Waterway-1 (NW-1) vide the National Waterway (Allahabad-Haldia stretch of the Ganga Bhagirathi-
Hooghly river) Act 1982. It became operative from 27th October 1986 after the formation of the IWAI.
The waterway extends from Haldia to Allahabad for a distance of 1620 km. It is being used by tourism
vessels, ODC carriers, and IWAI vessels. Many coal based plants are located along Ganga and thus, are
potential revenue sources for the inland navigation sector. Hence pair 1 is correctly matched.
• The Brahmaputra from Dhubri to Sadiya was declared National Waterway-2 (NW-2) vide the National
Waterway (Sadiya-Dhubri stretch of the Brahmaputra river) Act 1988. From Dhubri to Sadiya, the
34 www.visionias.in ©Vision IAS

FREE BY KING R QUEEN P [ऋषभ राजपूत]


waterway extends for a distance of 891 km. The river Brahmaputra flows down the centre of the Assam
Valley. It is used by tourism vessels, Border security forces, Assam government, and private
vessels. Hence pair 2 is correctly matched.
• The West Coast Canal from Kottapuram to Kollam was declared National Waterway-3 (NW-3) vide the
National Waterway (Kollam-Kottapuram stretch of the West Coast Canal and Champakara and
Udyogmandal Canals) Act 1992 and notified on 1st Feb. 1993. From Kottapuram to Kollam, including the
Champakara and Udyogmandal canals, it has a navigable length of 205 km. This waterway comprises
natural lakes, backwaters, river sections, and man-made canal sections. It is one of the most navigable and
tourism potential areas in India. Raw material for fertilizer plants is a major part of movement. Hence
pair 3 is not correctly matched.
• Kakinada-Puducherry canal stretch (767 km) along with the Godavari River stretch (171 km) between
(Bhadrachalam and Rajahmundry) and the Krishna River stretch (157 km) between (Wazirabad and
Vijayawada) is termed as National Waterway-4 (NW-4). The total length of the NW-4 is 1095 km. Coal
on Godavari river, Cement on Krishna river and rice on both rivers, and other such food commodities are
major transport on this waterway.

Q 94.D
• It is a ‘slash and burn’ agriculture. Farmers clear a patch of land and produce cereals and other food crops
to sustain their families. When the soil fertility decreases, the farmers shift and clear a fresh patch of land
for cultivation. This type of shifting allows Nature to replenish the fertility of the soil through natural
processes; land productivity in this type of agriculture is low as the farmer does not use fertilizers or other
modern inputs. It is known by different names in different parts of the country.

35 www.visionias.in ©Vision IAS

FREE BY KING R QUEEN P [ऋषभ राजपूत]


• It is jhumming in north-eastern states like Assam, Meghalaya, Mizoram and Nagaland; Pamlou in
Manipur, Dipa in Bastar district of Chhattisgarh, and in Andaman and Nicobar Islands.
• Jhumming: The ‘slash and burn’ agriculture is known as ‘Milpa’ in Mexico and Central America,
‘Conuco’ in Venezuela, ‘Roca’ in Brazil, ‘Masole’ in Central Africa, ‘Ladang’ in Indonesia, ‘Ray’ in
Vietnam.
• In India, this primitive form of cultivation is called ‘Bewar’ or ‘Dahiya’ in Madhya Pradesh, ‘Podu’ or
‘Penda’ in Andhra Pradesh, ‘Pama Dabi’ or ‘Koman’ or Bringa’ in Odisha, ‘Kumari’ in Western Ghats,
‘Valre’ or ‘Waltre’ in South-eastern Rajasthan, ‘Khil’ in the Himalayan belt, ‘Kuruwa’ in Jharkhand,
and ‘Jhumming’ in the North-eastern region. Hence, all the pairs are correctly matched.
Q 95.C
• The British type of climate is also known as the cool temperate western climate. The cool temperate
western margins are under the permanent influence of the Westerlies all around the year. From
Britain, the climatic belt stretches far inland into the lowlands of North-West Europe, including such
regions as northern and western France, Belgium, the Nether- lands, Denmark, western Norway and also
north-western Iberia.
• There is so much oceanic influence on both the temperature and the precipitation that the climate is
also referred to as the North-West European Maritime Climate. In North America, the high Rockies
prevent the on-shore Westerlies from penetrating far inland and the British type of climate is confined
mainly to the coastlands of British Columbia.
• The natural vegetation of this climate type is a deciduous forest. The trees shed their leaves in the cold
season. Common species of this climate include oak, poplar, birch and hornbeam.
• Market gardening is one of the highly developed activities of the people of this region. Market
gardening is the production of fruits, vegetables and flowers as cash crops on relatively small scales and
sold directly to consumers. It is both labour and capital-intensive.
• The annual temperature range in this type of climate is comparatively small. The summers are never
very warm and winters are abnormally mild. Thus the climate is considered ideal for maximum comfort
and mental alertness. People can work for long hours without feeling drowsy and lethargic as they do in
the tropics.
• Hence option (c) is the correct answer.

Q 96.D
• According to sources, it is believed that the world population reached the 8 billion mark in November
2022 though 2025 was the estimated year for the same.

• From the above table, it is clear that the world population reached the 1 billion mark in 1804 AD. Also, it
took nearly 123 years for the world population to reach from 1 billion to 2 billion. Hence both
statements 1 and 2 are not correct.

Q 97.D
• Bharmaur tribal area comprises Bharmaur and Holi tehsils of Chamba district of Himachal
Pradesh.
• It is a notified tribal area since 21 November 1975.
• Bharmaur is inhabited by ‘Gaddi’, a tribal community who have maintained a distinct identity in
the Himalayan region as they practised transhumance and conversed through Gaddiali dialect.
• Bharmaur tribal region has harsh climate conditions, low resource base and fragile environment.
• These factors have influenced the society and Economy of the region.
• According to the 2011 census, the total population of Bharmaur sub-division was 39,113 i.e., 21 persons
per sq km. It is one of the most (economically and socially) backward areas of Himachal Pradesh.
36 www.visionias.in ©Vision IAS

FREE BY KING R QUEEN P [ऋषभ राजपूत]


• Historically, the Gaddis have experienced geographical and political isolation and socio-economic
deprivation.
• The economy is largely based on agriculture and allied activities such as sheep and goat rearing.
• This region lies between 32° 11’ N and 32°41’ N latitudes and 76° 22’ E and 76° 53’E longitudes.
• Spread over an area of about 1,818 sq km, the region mostly lies between 1,500 m to 3,700 m above the
mean sea level.
• This region popularly known as the homeland of Gaddis is surrounded by lofty mountains on all sides.
• It has Pir Panjal in the north and Dhaula Dhar in the south.
• In the east, the extension of Dhaula Dhar converges with Pir Panjal near Rohtang Pass.
• The river Ravi and its tributaries– the Budhil and the Tundahen, drain this territory, and carve out deep
gorges.
• These rivers divide the region into four physiographic divisions called Holi, Khani, Kugtiand Tundah
areas.
• Bharmaur experiences freezing weather conditions and snowfall in winter. Its mean monthly
temperature in January remains 4°C and in July 26°C.
• Hence, option (d) is the correct answer.

Q 98.C
• Water is a cyclic resource with abundant supplies around the globe. Approximately, 71 percent of the
earth’s surface is covered with it but freshwater constitutes only about 3 percent of the total water. In fact,
a very small proportion of freshwater is effectively available for human use. The availability of freshwater
varies over space and time.
• India accounts for about 2.45 percent of the world’s surface area and 4 percent of the world’s water
resources and about 16 percent of the world’s population. Hence statement 1 is correct.
• The total water available from precipitation in the country in a year is about 4,000 cubic km. The
availability of surface water and replenishable groundwater is 1,869 cubic km. Out of this, only 60
percent can be put to beneficial use. Thus, the total utilizable water resource in the country is only 1,122
cubic km.
o Reasons for higher water availability from Precipitation:
✓ Firstly, India receives a large amount of precipitation, mainly due to its monsoon climate. The
southwest monsoon, which occurs from June to September, accounts for around 75% of India's
annual rainfall. The northeast monsoon, which occurs from October to December, brings rainfall
to the southern and eastern parts of the country. This high amount of rainfall contributes to the
total water available from precipitation.
✓ Secondly, not all the water from precipitation flows as surface water. A significant portion of
the precipitation infiltrates the soil and recharges groundwater. In India, groundwater is a
significant source of water for irrigation, drinking, and industrial purposes. The recharge of
groundwater through precipitation is, therefore, an important component of the total water
available in India.
✓ Lastly, India has significant water storage capacity through its dams and reservoirs. This
storage capacity allows for the capture and management of water from precipitation, which
contributes to total water availability. Hence statement 2 is correct.
Q 99.B
• Chuquicamata is the world's largest copper town located in Chile in the Atacama Desert. Other towns
in Chile known for copper production are Arica, Iquique and Antofagasta. Hence, pair 1 is not correctly
matched.
• Kalgoorlie is a gold mine located in Australia and it is part of the Great Australian Desert. Another
important town for gold mining in Australia is Coolgardie. Besides this, the Kalahari desert is also known
for its diamond and copper reserves. Hence, pair 2 is correctly matched.
• Utah is located in the Nevada desert in the U.S.A. It is known for Uranium mining. Hence, pair 3 is
correctly matched.

Q 100.B
• The Suez Canal had been constructed in 1869 in Egypt between Port Said in the north and PortSuez in
the south linking the Mediterranean Sea and the Red Sea. It gives Europe a new gateway to the Indian
Ocean and reduces the direct sea-route distance between Liverpool and Colombo compared to the Cape
of Good Hope route. It is a sea-level canal without locks which is about 160 km and 11 to 15 m deep.
About 100 ships travel daily and each ship takes 10-12 hours to cross this canal. Hence option (b) is the
correct answer.
37 www.visionias.in ©Vision IAS

FREE BY KING R QUEEN P [ऋषभ राजपूत]


• Image Source: NCERT

Copyright © by Vision IAS


All rights are reserved. No part of this document may be reproduced, stored in a retrieval system or
transmitted in any form or by any means, electronic, mechanical, photocopying, recording or otherwise,
without prior permission of Vision IAS.

38 www.visionias.in ©Vision IAS

FREE BY KING R QUEEN P [ऋषभ राजपूत]


VISIONIAS
www.visionias.in

Test Booklet Series

TEST BOOKLET

GENERAL STUDIES (P) 2024 – Test – 4130


C
Time Allowed: Two Hours Maximum Marks: 200

INSTRUCTIONS

1. IMMEDIATELY AFTER THE COMMENCEMENT OF THE EXAMINATION, YOU SHOULD CHECK THAT THIS BOOKLET
DOES NOT HAVE ANY UNPRINTED OR TURN OR MISSING PAGES OR ITEMS, ETC. IF SO, GET IT REPLACED BY A
COMPLETE TEST BOOKLET.

2. ENCODE CLEARLY THE TEST BOOKLET SERIES A, B, C OR D AS THE CASE MAY BE IN THE APPROPRIATE PLACE IN
THE ANSWER SHEET.

3. You have to enter your Roll Number on the Test Booklet in the Box
provided alongside. Do NOT write anything else on the Test Booklet.

4. This Test Booklet contains 100 items (Questions). Each item is printed in English. Each item comprises four
responses (answers). You will select the response which you want to mark on the Answer Sheet. In case you
feel that there is more than one correct response with you consider the best. In any case, choose ONLY ONE
response for each item.

5. You have to mark all your responses ONLY on the separate Answer Sheet provided. See direction in the
answers sheet.

6. All items carry equal marks. Attempt all items. Your total marks will depend only on the number of correct
responses marked by you in the answer sheet. For every incorrect response 1/3rdof the allotted marks will be
deducted.

7. Before you proceed to mark in the Answer sheet the response to various items in the Test booklet, you have to
fill in some particulars in the answer sheets as per instruction sent to you with your Admission Certificate.

8. After you have completed filling in all responses on the answer sheet and the examination has concluded, you
should hand over to Invigilator only the answer sheet. You are permitted to take away with you the Test
Booklet.

9. Sheet for rough work are appended in the Test Booklet at the end.

DO NOT OPEN THIS BOOKLET UNTIL YOU ARE ASKED TO DO SO


1 www.visionias.in ©Vision IAS

FREE BY KING R QUEEN P [ऋषभ राजपूत]


1. Consider the following statements regarding 5. With reference to the Vaisheshika
Harappan technology and crafts: philosophical system, consider the following
1. Harappans used only iron to make statements:
weapons.
1. It declares that salvation depends on
2. Harappans practiced boatmaking.
fully recognizing the atomic nature of
3. Harappans were aware of potter's wheel.
Which of the statements given above is/are the universe and its difference from the
correct? soul.
(a) 1 and 2 only 2. It classifies objects strictly under three
(b) 2 and 3 only elements - land, water, and air.
(c) 1 and 3 only Which of the statements given above is/are
(d) 1, 2 and 3
correct?
(a) 1 only
2. Madanna and Akhanna, the two bothers and
prominent statesman during the seventeenth (b) 2 only
century rose to prominence under the (c) Both 1 and 2
(a) Bijapur Sultanate (d) Neither 1 nor 2
(b) Golconda Sultanate
(c) Nizam Shahi of Ahmednagar 6. Consider the following statements regarding
(d) Maratha Kingdom
Satavahanas:
1. The Satavahanas started the practice of
3. In the context of ancient India, which of the
following books talk about India? granting tax-free villages to Brahmanas.
1. The Periplus of the Erythrean Sea 2. The official language of the Satavahanas
2. Ptolemy's Geography was Prakrit.
3. Pliny’s Naturalis Historia 3. They issued mostly coins of gold.
Select the correct answer using the code Which of the statements given above is/are
given below.
correct?
(a) 1 only
(a) 1 only
(b) 1 and 2 only
(c) 2 and 3 only (b) 1 and 2 only
(d) 1, 2 and 3 (c) 2 and 3 only
(d) 1, 2 and 3
4. Consider the following statements regarding
the Dhrupad genre of Hindusthani classical 7. With reference to the later Vedic period,
music:
which of the following statements is not
1. Man Singh Tomar, the Maharaja of
correct?
Gwalior mainly responsible for the
popularity of dhrupad. (a) King maintained a standing army.
2. Tansen, who was in Akbar’s court, is a (b) The society came to be divided into four
significant dhrupad singer. varnas.
Which of the statements given above is/are (c) The grant of land as sacrificial fee is not
correct? well established in the later Vedic
(a) 1 only
period.
(b) 2 only
(d) Sacrifices and performing rituals became
(c) Both 1 and 2
(d) Neither 1 nor 2 dominant mode of worshipping the gods.
2 www.visionias.in ©Vision IAS

FREE BY KING R QUEEN P [ऋषभ राजपूत]


8. Consider the following statements with 11. Consider the following statements regarding
regard to the state of agricultural system in the Rig Vedic period:
the Mauryan period: 1. Women slaves were prevalent during the
1. The Mauryan state provided irrigation Rig Vedic period.
2. Land was a well-established property
facilities and regulated water supply.
during the Rig Vedic age.
2. The Mauryan state employed slaves in
3. Child marriage was widely prevalent
agriculture.
during the Rig Vedic age.
3. To bring the virgin soil under Which of the statements given above is/are
cultivation, the new peasants were correct?
allowed remission in tax and supplied (a) 1 only
with cattle, seeds and money. (b) 2 and 3 only
Which of the statements given above is/are (c) 1 and 2 only
correct? (d) 1, 2 and 3
(a) 1 only
12. Sensuous and lyrical, this dance form is a
(b) 1 and 2 only
dance of love and passion touching on the
(c) 2 and 3 only
divine and the human, the sublime and the
(d) 1, 2 and 3
mundane. It is a highly stylised dance and to
some extent is based on the classical Natya
9. With reference to the Gupta period, who Shastra and the Abhinaya Darpana. The
among the following were the court scholars/ techniques of movement are built around the
poets of Chandragupta II? two basic postures of the Chowk and the
1. Ravikirti Tribhanga. The torso movement is very
2. Amarasimha important and is an unique feature of this
3. Kalidasa dance form.
Which of the following dance form is best
Select the correct answer using the code
described by the above passage?
given below.
(a) Kathak
(a) 1 and 3 only
(b) Bharatnatyam
(b) 2 and 3 only (c) Odissi
(c) 3 only (d) Sattriya
(d) 1 and 2 only
13. Consider the following statements about
10. Consider the following statements regarding satellite internet or space-based internet:
Jainism: 1. Satellites for space-based internet can be
1. It denied the existence of gods. put in both Low Earth Orbit (LEO) and
Geostationary Orbit.
2. Unlike Buddhism, it condemned the
2. Unlike DTH broadcasting, it is a two-
Varna system.
way communication.
3. It mainly aims at the attainment of
3. Starlink is an example of a
freedom from worldly bonds. Geostationary satellite-based internet.
Which of the statements given above is/are Which of the statements given above is/are
correct? correct?
(a) 1 only (a) 1 and 2 only
(b) 1 and 2 only (b) 2 and 3 only
(c) 3 only (c) 1 only
(d) 2 and 3 only (d) 1, 2, and 3
3 www.visionias.in ©Vision IAS

FREE BY KING R QUEEN P [ऋषभ राजपूत]


14. Consider the following statements with 17. In the context of ancient India, the terms
respect to the Ancient Monuments and bhuktis, vishayas and vithis refer to
Archeological Sites and Remains (AMASR)
(a) coins
Act of 1958:
1. The act regulates the archeological (b) administrative divisions
excavation and looks into the (c) administrative officials
preservation of sculptures and carvings.
(d) books
2. According to the AMASR Act of 1958,
Archeological Site means any area
which contains ruins and relics of 18. With reference to the Mansabdari system
historical or archeological importance
during the Mughal rule, consider the
which have been in existence for not less
than 1000 years. following statements:
3. The recent Amendment to the act gives 1. It was established by Babur after his
state governments the power to extend
victory in the first battle of Panipat.
the area beyond 100 meters around
protected areas. 2. Under this system, the Mansabdars were
Which of the statements given above is/are supposed to maintain their own military
correct?
force.
(a) 1 only
(b) 2 only 3. The salaries of the Mansabdars were

(c) 1 and 3 only stated in rupees and paid by assigning


(d) 2 and 3 only
them jagirs.

Which of the statements given above is/are


15. Consider the following statements regarding
Harappan civilization: correct?
1. Harappans worshipped goddesses such (a) 1 and 2 only
as Durga, Amba, Kali.
(b) 2 and 3 only
2. Harappans practised phallu worship.
Which of the statements given above is/are (c) 1 and 3 only
correct? (d) 2 only
(a) 1 only
(b) 2 only
(c) Both 1 and 2 19. The term "Startup20 Engagement group
(d) Neither 1 nor 2 (SUMup)" is often seen in news in the

context of:
16. Which one of the following philosophical
systems believes only in materialism? (a) BRICS
(a) Samkhya (b) World Bank
(b) Lokayata
(c) OECD
(c) Vaisesika
(d) Mimamsa (d) G20
4 www.visionias.in ©Vision IAS

FREE BY KING R QUEEN P [ऋषभ राजपूत]


20. Consider the following statements regarding 23. Consider the following pairs:
particularly vulnerable tribal groups Writer Literature
(PVTGs):
1. Vishakhadatta : Mudrarakshasha
1. On the recommendation of Xaxa
2. Banabhatta : Harshacharita
Committee, the government identified
3. Bilhana : Vikramankadeva-
the most vulnerable groups as PVTGs.
2. The criteria for identification of PVTGs charita
include a declining or stagnant Which of the pairs given above is/are
population and pre-agriculture level of correctly matched?
technology.
(a) 1 only
3. Kathodi, Jarawas, and Koraga are
(b) 1 and 2 only
examples of PVTGs.
(c) 2 and 3 only
Which of the above statements is/are
correct? (d) 1, 2 and 3
(a) 1 only
(b) 2 and 3 only 24. In the context of prehisoric period of India,
(c) 1 and 3 only
pit dwellings were mainly associated with
(d) 1, 2, and 3
(a) Maski
(b) Burzahom
21. With reference to the history of India, the
terms "khud-kashta" and "pahi-kashta" were (c) Piklihal
used for (d) Mirzapur
(a) warriors
(b) traders
25. Recently, India has called upon G20
(c) weavers
countries to adopt multilateral action for
(d) peasants
faster extradition of fugitive economic

22. Consider the following statements regarding offenders. In this context, consider the
Rudradaman I: following statements:
1. He took measures to improve the 1. The term ‘fugitive economic offender’
Sudarsana lake in the semi-arid zone of
(FEO) is not defined under any statute in
Kathiawar.
India.
2. He issued the first-ever long inscription
in chaste Sanskrit. 2. The government of India can confiscate

3. It was during his reign that Saint the properties of economic offenders
Thomas is said to have come to India for who have left the country.
the propagation of Christianity. Which of the statements given above is/are
Which of the statements given above is/are
correct?
correct?
(a) 1 only
(a) 1 only
(b) 2 only
(b) 1 and 2 only
(c) 2 and 3 only (c) Both 1 and 2
(d) 1, 2 and 3 (d) Neither 1 nor 2
5 www.visionias.in ©Vision IAS

FREE BY KING R QUEEN P [ऋषभ राजपूत]


26. Consider the following statements regarding 28. With reference to different foreign
the legal system during the Gupta period: personalities that visited the Mughal Empire,
consider the following statements:
1. Civil and criminal laws were clearly
1. Antonio Monserrate was a Portuguese
defined and demarcated during this
priest sent to the Mughal court to
period. convert Akbar to Christianity.
2. Laws were based on differences in 2. François Bernier was a Frenchman who
varnas. served as a physician to Prince Dara
Shikoh.
3. Guilds of artisans, merchants were
3. Sir Thomas Roe was an English
governed by their own laws.
Diplomat who visited Emperor Shah
Which of the statements given above is/are Jahan’s court to obtain trading rights for
correct? the British East India Company.
(a) 2 and 3 only Which of the statements given above is/are
(b) 1, 2 and 3 correct?
(a) 1 and 3 only
(c) 1 and 2 only
(b) 2 and 3 only
(d) 3 only (c) 2 only
(d) 1 and 2 only
27. Which of the following statements correctly
defines the term "Sealed Cover 29. With reference to the Virashaiva tradition,
consider the following statements:
Jurisprudence"?
1. Its founder Basavanna was a
(a) It is a practice used by the Supreme
contemporary of the Sufi saint Shaikh
Court and High Courts for conveying Nizamuddin Auliya.
their judgments on cases involving 2. The tradition believes that every devotee
terrorists in sealed envelopes to ensure has to go through multiple cycles of
rebirth before he/she is united with Lord
National Security.
Shiva.
(b) It is a practice used by the Supreme
3. Virashaivism did not advocate the
Court and High Courts, for asking worship of idols.
information from strategic agencies from Which of the statements given above is/are
abroad including the FBI, CIA, Mossad, correct?
(a) 1 and 2 only
and others.
(b) 2 and 3 only
(c) It is a practice used by the Supreme
(c) 3 only
Court and sometimes lower courts for (d) 1, 2 and 3
asking for information from government
agencies in sealed envelopes that can 30. In the context of medieval India, Qarachil
expedition was launched during which of the
only be accessed by judges.
following rulers?
(d) It is a practice used by the Supreme
(a) Muhammad bin Tughlaq
Court for delivering judgements in (b) Ghiyasuddin Tughlaq
sealed envelopes in cases where judges (c) Qutubuddin Aibak
themselves are involved. (d) Ibrahim Lodi
6 www.visionias.in ©Vision IAS

FREE BY KING R QUEEN P [ऋषभ राजपूत]


31. Siksha (phonetics), Kalpa (rituals), Vyakarna 35. Consider the following statements with
(grammar), Nirukta (etymology), Chhanda regard to the traditional theatre form Bhavai:
(metrics), and Jyotisha (astronomy) are 1. The centers of this form are Kutch and
classified under which one of the following? Kathiawar.
(a) Vedangas 2. Only tabla and flute are used as musical
(b) Brahmanas instruments in it.
(c) Aranyakas 3. There is a rare synthesis of devotional
(d) Upanishads and romantic sentiments in this form.
Which of the statements given above is/are
32. Consider the following statements:
correct?
1. Asthadhyayi was written by Patanjali.
(a) 1 only
2. Sarvavarman, a scholar of great repute,
(b) 2 and 3 only
composed the grammar of Katantra.
(c) 1 and 3 only
3. Hala wrote a great poetical work
(d) 3 only
Gathasaptasati in Sanskrit.
Which of the statements given above is/are
36. With the breakup of the Gujara-Pratihara
correct?
empire, a number of Rajput states came into
(a) 1 only
(b) 2 only existence in north India, which of the
(c) 2 and 3 only following are Rajput dynasties?
(d) 1 and 3 only 1. Gahadavalas of Kannauj
2. Parmar of Malwa
33. The Sixth Assessment Report (AR6), 3. Chauhans of Ajmer
sometimes seen in the news, is published by: 4. Kalachuris
(a) Intergovernmental Panel on Climate 5. Chandela of Bundelkhand
Change (IPCC) Select the correct answer using the code
(b) United Nations Environment Program given below.
(c) World Wide Fund for Nature (WWF) (a) 1 and 3 only
(d) NITI Aayog (b) 3 and 5 only
(c) 1, 2 and 4 only
34. Which of the following are the features of (d) 1, 2, 3, 4 and 5
the Nagara style of temple architecture?
1. An entire temple is built on a stone 37. Muzarians during the medieval period of
platform. Indian history were generally
2. The temples have elaborate boundary (a) Village Headmen
walls. (b) Tenant farmers
3. The Garbhagriha is located under the
(c) Grain merchants
tallest tower.
(d) Landless labourers
4. A large water reservoir is present in the
temple complex.
38. Which of the following rulers is credited to
Select the correct answer using the code
be the founder of the Rashtrakuta dynasty?
given below.
(a) Dantidurga
(a) 1 and 3 only
(b) Amoghvarsha
(b) 1, 3 and 4 only
(c) Krishna I
(c) 2 and 4 only
(d) 1, 2 and 3 only (d) Vallabhraja
7 www.visionias.in ©Vision IAS

FREE BY KING R QUEEN P [ऋषभ राजपूत]


39. In the context of ancient history of India, 43. In the context of Mahajanapadas of ancient
samaharta refers to India, which of the following factors were
(a) spy mainly responsible for the rise of Magadha
(b) tax assessment officer empire?
(c) army commander 1. Availability of iron mines
(d) village headman 2. Use of elephants in military
3. Orthodox character of Magadhan society
40. Akbar divided the empire into twelve
Select the correct answer using the code
provinces called Subhas in 1580. With
given below.
reference to this, which of the following
(a) 1 and 2 only
officers were appointed by Akbar at the
(b) 2 only
provincial level?
1. Amalguzar (c) 2 and 3 only
2. Bakhshi (d) 1 and 3 only
3. Faujdar
4. Waqia-navis 44. With reference to the cultural history of
Select the correct answer using the code India, 'Arabesque' was
given below. (a) a form of artistic decoration.
(a) 1, 2 and 3 only (b) a form of Sufi song.
(b) 1 and 2 only (c) a type of jewellery worn by aristocratic
(c) 2 and 4 only women during the Mughal period.
(d) 1, 3 and 4 only (d) a Mughal delicacy prepared during
special occasions.
41. With reference to the Gol Gumbad of
Bijapur, consider the following statements:
45. Which one of the following ancient works is
1. It is the mausoleum of Muhammad Adil
not written by Kalidasa?
Shah.
(a) Kumarasambhavam
2. The monument lacks walls and is both
(b) Vikramorvasiyam
outwardly and inwardly circular.
Which of the statements is/are correct? (c) Sishupalavadham
(a) 1 only (d) Abhigyanashakuntalam
(b) 2 only
(c) Both 1 and 2 46. Consider the following statements regarding
(d) Neither 1 nor 2 Rig Vedic period:
1. King's post was generally hereditary.
42. Consider the following statements regarding 2. The assemblies Vidatha and Gana were
Project ELLORA:- the most important from political point
1. It is an initiative of the Ministry of of view.
Culture. 3. The Rig Veda does not mention any
2. It aims to bring 'rare' Indian languages officer for administering justice.
online and preserve them.
Which of the statements given above is/are
Which of the statements given above is/are
correct?
not correct?
(a) 1 only
(a) 1 only
(b) 1 and 3 only
(b) 2 only
(c) Both 1 and 2 (c) 2 and 3 only
(d) Neither 1 nor 2 (d) 1, 2 and 3
8 www.visionias.in ©Vision IAS

FREE BY KING R QUEEN P [ऋषभ राजपूत]


47. Which of the following is not musical form 51. Consider the following statements, with
of Carnatic music? reference to the Vijayanagar Empire:
(a) Gitam 1. The empire was founded by Harihara
(b) Svarjati
and Bukka.
(c) Pada
2. The extent of the empire was limited to
(d) Khayal
modern-day Karnataka and Andhra

48. The art form deals with themes that narrate Pradesh only.
the social lifestyle and activities of village 3. The rulers of the empire had friendly ties
life in the Maharashtra region. Spiral with the sultan of Madurai.
formations of men and women and Which of the statements given above is/are
concentric circular designs in these paintings correct?
are symbolic of the Tarpa dance. They (a) 1 only
simply paint on mud and cow dung based
(b) 3 only
surface, coated with geru (red mud) first and
(c) 1 and 3 only
then with rice paste for the colour white.
(d) 1, 2 and 3
Salati grass or bamboo sticks are used in
place of brush for painting.
Which of the following is best described in 52. Consider the following statements with
the passage given above? respect to "Organ-on-a-chip" technology:
(a) Gond painting 1. It is an integrated circuit that simulates
(b) Santhal Painting the activities and physiological response
(c) Phad Painting
of an organ.
(d) Warli Painting
2. It is entirely artificial and does not
contain any human cells.
49. Consider the following pairs:
Painting school Region 3. It can be used to replace animals for the
1. Cherial Scroll Painting : Kerala testing of drugs.
2. Madhubani Painting : Bihar Which of the statements given above are
3. Patachitra Painting : Odisha correct?
4. Picchvai Painting : Rajasthan (a) 1 and 3 only
Which of the pairs given above are correct? (b) 1 and 2 only
(a) 1 and 3 only
(c) 2 and 3 only
(b) 2 and 4 only
(d) 1, 2, and 3
(c) 2, 3 and 4 only
(d) 1, 3 and 4 only
53. Krishnattam and Mudiyettu are theatres
50. Recently Salal-Haimana area was reported to associated with which of the following
have huge resources of Lithium. It is located states?
in which of the following regions of India? (a) Kerala
(a) Arunachal Pradesh (b) Tamil Nadu
(b) Bundelkhand
(c) Andhra Pradesh
(c) Hyderabad
(d) Karnataka
(d) Jammu and Kashmir
9 www.visionias.in ©Vision IAS

FREE BY KING R QUEEN P [ऋषभ राजपूत]


54. Consider the following statements about 57. In the context of ancient history,
Yellowstone National Park: Buddhacharita and Saundarananda were
1. It is considered to be the first national
written by
park in the world.
2. It is situated east of the Mississippi (a) Banabhatta
River. (b) Asvaghosha
3. It is home to grizzly bears and wolves
(c) Bhavabhuti
Which of the above statements is/are not
correct? (d) Vishakhadatta
(a) 1 only
(b) 2 only
58. Consider the following statements regarding
(c) 3 only
(d) 1 and 2 only the Primary Agricultural Cooperative Credit
Societies (PACS):
55. With reference to the Mughal rule in India, 1. They provide short-term, and medium-
consider the following statements:
term agricultural loans to the farmers for
1. The first Mughal invasion of Deccan
happened during the reign of Akbar. various agricultural and farming
2. The rebellion of Mahabat Khan which activities.
led to the arrest of the Mughal Emperor
2. They constitute the lowest tier of the
occurred during the reign of Jahangir.
3. The Nizam Shahi state of Ahmednagar three tiers including State Cooperative
came to an end during the rule of Banks (SCBs) and District Central
Aurangzeb.
Cooperative Banks (DCCBs).
Which of the statements given above is/are
correct? 3. They are regulated by the Reserve Bank
(a) 1 only of India.
(b) 1 and 2 only Which of the statements given above is/are
(c) 1 and 3 only
correct?
(d) 3 only
(a) 1 and 2 only
56. Mauryans in ancient India controlled (b) 2 and 3 only
numerous economic activities through an
(c) 3 only
elaborate economic administration. In this
context, in which of the following areas did (d) 1, 2 and 3
the state maintained a monopoly?
1. Mining
59. With reference to the history of medieval
2. Sale of liquor
3. Manufacture of arms India, the terms 'Uluq' and 'Dawa' refer to
Select the correct answer using the code types of
given below. (a) Slaves
(a) 1 only
(b) Coins
(b) 1 and 3 only
(c) 2 and 3 only (c) Postal systems
(d) 1, 2 and 3 (d) Lands
10 www.visionias.in ©Vision IAS

FREE BY KING R QUEEN P [ऋषभ राजपूत]


60. Consider the following statements: 63. "Women, Business and the Law 2023
1. Vakataka bronze images show the report", is released by:
influence of the Mathura style. (a) World Economic Forum
(b) UN Women
2. The lost-wax technique of sculpture-
(c) World Bank
making was introduced in India by the
(d) BRICS
Cholas in the 1st century CE.
3. The Gupta bronze sculptures were 64. His teachings were based on the Bhagavad
portable and carried by monks to Gita and the Bhagavata Purana. He
different regions. emphasised the need for naam kirtan, the
Which of the statements given above is/are recitation of the names of the lord in
correct? congregations of pious devotees. He
encouraged the establishment of satra or
(a) 1 only
monasteries for the transmission of spiritual
(b) 2 and 3 only
knowledge, and naam ghar or prayer halls.
(c) 3 only
His major compositions include the Kirtana-
(d) 2 and 3 only ghosha.
Who among the following bhakti saints is
61. Arrange the following architectural features described in the paragraph given above?
chronologically: (a) Baba Guru Nanak Dev
1. Virupaksha temple of Pattadkal (b) Chaitanya Mahaprabhu
2. Sun temple at Konark (c) Tulsidas
(d) Shankaradeva
3. Lakshama Temple of Khajuraho
4. Sun temple at Modhera
65. He was a Turkish adventurer who is believed
Select the correct answer using the code to have risen in the service of a Brahmana.
given below. He was the founder of the Bahmani Empire.
(a) 2-1-3-4 Which of the following personalities has
(b) 4-1-2-3 been referred to in the passage given above?
(c) 1-3-4-2 (a) Krishna Devaraya
(d) 2-3-1-4 (b) Hasan Gangu
(c) Mahmud Gawan
(d) Ibrahim Adil Shah
62. It is a classical dance form pertaining to
Tamil Nadu in southern India. It has grown 66. The Tungabhadra Doab is a region between
out of the art of dancers dedicated to temples the Tungabhadra and Krishna rivers. In this
and was earlier known as Sadir or Dasi context, which of the following empires/
Attam. It is the first of India's traditional dynasties controlled the Tungbhandra doab
dances to be refashioned as theatre art and to in various periods of time?
be exhibited widely both at home and 1. Western Chalukyas
2. Hoysalas
abroad.
3. Rashtrakutas
Which among the following dance form is
Select the correct answer using the code
being described in the above given passage? given below.
(a) Bharatanatyam (a) 1 and 2 only
(b) Kathakali (b) 2 and 3 only
(c) Manipuri (c) 1 and 3 only
(d) Kuchipudi (d) 1, 2 and 3
11 www.visionias.in ©Vision IAS

FREE BY KING R QUEEN P [ऋषभ राजपूत]


67. Consider the following statements regarding 70. Consider the following pairs:
The Ajanta and Ellora caves: Dynasty King
1. While the Ajanta caves have double- 1. Chola : Senguttuvan
storeyed caves, the Ellora caves have
2. Chera : Karikala
unique three-storeyed caves.
3. Chalukyas of Badami : Pulakesin II
2. While the images of Padmapani and
How many of the pairs given above are
Vajrapani are present in Ajanta caves,
they are absent in Ellora caves. correctly matched?
Which of the statements given above is/are (a) None of the pairs
correct? (b) Only one pair
(a) 1 only (c) Only two pairs
(b) 2 only (d) All three pairs
(c) Both 1 and 2
(d) Neither 1 nor 2
71. Which of the following statements is not
true regarding the Kuchipudi danceform?
68. With reference to the Maratha administration
under King Shivaji, consider the following (a) It grew as a product of Bhakti movement
pairs: in Andhra Pradesh.
Important Their role/description (b) Initially male only performed the dance
functionaries but now female kuchipudi dancers also
1. Peshwa : Commander in chief participate.
2. Dabir : Foreign Affairs
(c) A Kuchipudi recital is usually concluded
3. Mirasdars : Loose auxiliaries
with tarangam.
Which of the pairs given above is/are
(d) Rukmini Devi Arundale was a
correctly matched?
(a) 1 only prominent Indian dancer who patronized
(b) 2 only this dance form.
(c) 1 and 3 only
(d) 1 and 2 only 72. With reference to the Treaty of Purandar,
consider the following statements:
69. With reference to the Firuz Shah Tughlaq,
1. It was concluded between Maratha king
consider the following statements:
Shivaji and Raja Jai Singh of Amber.
1. He transferred the capital from Delhi to
2. As per the provisions of the treaties the
Deogir.
2. He introduced bronze token currency. Marathas surrendered more than half
3. He extended the principle of heredity to their forts to the Mughals.
the army. 3. The treaty granted a portion of Bijapur
4. During his reign, Jizyah became a territory in Balaghat to the Marathas.
separate tax. Which of the statements given above is/are
Which of the statements given above are
correct?
correct?
(a) 1 only
(a) 1 and 2 only
(b) 2 and 3 only
(b) 2 and 3 only
(c) 1 and 3 only (c) 1 and 3 only
(d) 3 and 4 only (d) 1, 2 and 3
12 www.visionias.in ©Vision IAS

FREE BY KING R QUEEN P [ऋषभ राजपूत]


73. With reference to the Maratha Kingdom, 76. Consider the following statements regarding
consider the following statements: Gupta period:
1. Chandalas appeared for the first time
1. Chauth was a levy imposed on the during this period.
territories not directly controlled by the 2. The castes proliferated into numerous
Marathas. subcastes during this period.
Which of the statements given above is/are
2. Sardeshmukhi was a 25 percent levy correct?
imposed on the peasants as a matter of (a) 1 only
right by the Maratha king. (b) 2 only
(c) Both 1 and 2
Which of the statements given above is/are (d) Neither 1 nor 2
correct?
(a) 1 only 77. Consider the following statements regarding
the reign of emperor Harshavardhana:
(b) 2 only 1. He made Kanauj the seat of his power.
(c) Both 1 and 2 2. Land grants to priests were stopped
(d) Neither 1 nor 2 during his reign.
3. As per Hsuan Tsang, law and order was
not well maintained during his reign.
74. Consider the following statements regarding Which of the statements given above is/are
the Udaigiri-Khandagiri caves: correct?
(a) 1 only
1. The caves have inscriptions of the Pala (b) 2 and 3 only
kings. (c) 1 and 3 only
2. According to the inscriptions, the caves (d) 1, 2 and 3

were meant for Jain monks. 78. Consider the following passage :
Which of the statements given above is/are This school of philosophy's name literally
correct? means the art of reasoning and
interpretation. It provided reasoning for
(a) 1 only various Vedic rituals, and the attainment of
(b) 2 only salvation was made dependent on their
(c) Both 1 and 2 performance. According to this school, the
Vedas contain the eternal truth. The
(d) Neither 1 nor 2 principal object of this philosophy was to
acquire heaven and salvation. In order to
75. Which of the following is a campaign attain salvation this school strongly
recommended the performance of Vedic
initiated by UNICEF in Rajasthan, which sacrifices, which needed the services of the
aims to cover tankas, the traditional priests and legitimized the social distance
rainwater harvesting system, in order to between various varnas.
Which of the following schools of
reduce the incidents of suicides in the state? philosophy is being described in the passage
(a) Jal Jeevan Abhiyan given above?
(b) Anmol Jeevan Abhiyan (a) Sankhya
(b) Mimamsa
(c) Nirmal Jal Abhiyan (c) Vedanta
(d) Vidyut Jal Abhiyan (d) Lokayata
13 www.visionias.in ©Vision IAS

FREE BY KING R QUEEN P [ऋषभ राजपूत]


79. Consider the following statement with 82. MISHTI Scheme has been launched in
reference to the market control and agrarian Union Budget 2023 for mangrove
policy of Alauddin: plantations along the coastlines. In this
1. He fixed the cost of a wide range of context, consider the following statements:
commodities. 1. MISHTI will be implemented through
2. He set up a common market for all convergence between MGNREGS and
commodities. CAMPA.
3. He set up warehouses to stock food grain
2. The state of Odisha alone has about 40%
to supply during shortfalls.
of India’s mangroves.
Which of the statements given above is/ are
Which of the statements given above is/are
correct?
correct?
(a) 1 only
(a) 1 only
(b) 2 only
(b) 2 only
(c) 1 and 3 only
(c) Both 1 and 2
(d) 2 and 3 only
(d) Neither 1 and 2

80. Which of the following best describes the


term 'windfall tax', recently seen in the 83. With reference to Indian history, the term

news? 'agrahara' refers to:

(a) It is a tax levied on an unforeseen or (a) religious sacrifices to the deities during
unexpectedly large profit. Mauryan period.
(b) It is a tax levied to limit tax exemptions (b) land grants to Brahmanas by royal
availed by companies. patronage.
(c) It is a type of transfer tax on money or (c) officials below district level during
property that one living person or Gupta period.
corporate entity gives to another. (d) type of idol casting method used during
(d) It is a tax levied on profit from the sale Post – mauryan period
of property or an investment.
84. With reference to medieval India, which of
81. With reference to Ulugh Khan or Balban, the following foreign travelers visited the
consider the following statements: Vijayanagar Empire?
1. He introduced the Chahalgani system.
1. Fernao Nuniz
2. He began an era of decentralized
2. Nicolo de Conti
government for the first time.
3. Abdur Razzak
3. He was the first who set up a separate
4. Ibn Battuta
ariz’s department in India.
Select the correct answer using the code
Which of the statements given above is/are
given below.
correct?
(a) 1 and 2 only
(a) 1 only
(b) 1 and 3 only
(b) 2 only
(c) 2, 3 and 4 only
(c) 1 and 3 only
(d) None (d) 1, 2, 3, and 4
14 www.visionias.in ©Vision IAS

FREE BY KING R QUEEN P [ऋषभ राजपूत]


85. With reference to the dahsala system during 88. Consider the following pairs:
the Mughal rule, consider the following Theatre Region
statements: 1. Dashavatar Goa

1. It was the modified version of the Zabti 2. Koodiyaattam Kerala

system of land revenue. 3. Yakshagaana Telangana


4. Therukoothu Tamil Nadu
2. Under this system, the produce was
How many pairs given above are correctly
equally divided between the peasants
matched?
and the state after it had been thrashed,
(a) Only one pair
or when it had been cut and tied in
(b) Only two pairs
stacks.
(c) Only three pairs
Which of the statements given above is/are
(d) All four pairs
not correct?
(a) 1 only 89. Consider the following pairs:
(b) 2 only Regional music Region
(c) Both 1 and 2 1. Lotia : Andhra Pradesh
(d) Neither 1 nor 2 2. Chhakri : Kashmir
3. Daskathia : Odisha
86. In the context of trade in Post Mauryan 4. Pandavani : Chhattisgarh

period, which of the following famous ports Which of the pairs given above are correctly

are located on the eastern coast of India? matched?


(a) 1 and 3 only
1. Broach
(b) 2, 3 and 4 only
2. Sopara
(c) 1, 2 and 3 only
3. Tamralipti
(d) 2 and 4 only
4. Arikamedu
Select the correct answer using the code
90. It is a category of musical instruments in
given below.
which sound is produced by blowing air into
(a) 1 and 2 only an hollow column. The pitch of the note is
(b) 2, 3 and 4 only determined by controlling the air passage
(c) 1, 3 and 4 only and the melody is played by using the
(d) 3 and 4 only fingers to open and close in the instrument
Above mentioned the paragraph refers to
87. Alexander invaded India during the reign of which of the following category of
which of the following dynasties? instruments?

(a) Shunga (a) Avanaddh Vadya

(b) Haryanka (b) Sushir Vadya


(c) Ghana Vadya
(c) Nanda
(d) Tata Vadya
(d) Gupta
15 www.visionias.in ©Vision IAS

FREE BY KING R QUEEN P [ऋषभ राजपूत]


91. With reference to Sufism, consider the 94. With reference to the Tamil literature,
following pairs: consider the following pairs:
Terms Description/Meaning Work Associated With
1. Haidaris : Community Kitchens 1. Silappadikaram : Chattanar

2. Ziyarat : Practice of pilgrimage 2. Manimekalai : Ilango-Adigal

3. Khalifa : Successor of a Sufi saint 3. Thirukkural : Thiruvalluvar


How many pairs given above are correctly
4. Masnavis : Sufi poems
matched?
How many pairs given above are correctly
(a) Only one pair
matched?
(b) Only two pairs
(a) Only one pair
(c) Only three pairs
(b) Only two pairs
(d) None
(c) Only three pairs
(d) All four pairs
95. Consider the following statements regarding
Harappan trade:
92. With reference to the Dharmapala, consider 1. Harappans used punch-marked coins in
the following statements: trade.
1. He was the founder of the Pala dynasty. 2. Harappans had trade relations with
2. He founded Vikramshila University Afghanistan and Iran.
which became second only to Nalanda in Which of the statements given above is/are
fame. correct?
3. He was involved in a conflict with the (a) 1 only
Pratihara king Vatsaraja due to the claim (b) 2 only
over the Kannauj region. (c) Both 1 and 2

Which of the statements given above are (d) Neither 1 nor 2

correct?
96. Consider the following statements with
(a) 1 and 2 only
regard to Qutub Minar:
(b) 2 and 3 only
1. The construction of the Minar was
(c) 1 and 3 only
started by Qutubuddin Aibak and
(d) 1, 2 and 3
completed by Balban.
2. Alai Darwaza associated with Qutub
93. With reference to Indian history, Rekhapida,
Minar was constructed by Alauddin
Pidhadeul and Khakra are related to:
Khilji.
(a) Rich cuisine culture of Western India Which of the statements given above is/are
(b) Temple architecture in eastern India not correct?
(c) Geometric scales given in Surya (a) 1 only
Siddhanta (b) 2 only
(d) Revenue collection systems in various (c) Both 1 and 2
parts of India (d) Neither 1 nor 2
16 www.visionias.in ©Vision IAS

FREE BY KING R QUEEN P [ऋषभ राजपूत]


97. Which of the following statements correctly 100. Consider the following statements about the
describes the term “ChatGPT”? National Recruitment Agency:
(a) It is a Government of India portal to 1. It will be a multi-agency body that will
create sensitization about mental issues. conduct tests for all non-gazetted posts,
(b) It is an initiative of Ministry of including Group B and C (non-technical)
Education to help students deal with jobs.
exam anxiety. 2. It will be registered as a society under
(c) It is a Generative Transformer from the Societies Registration Act, of 1860.
META for an interactive metaverse. 3. It will be responsible for conducting the
(d) It is an artificial intelligence chatbot Common Eligibility Test (CET).
developed by OpenAI. Which of the statements given above is/are
correct?
98. The term 'Windsor framework' is seen in the (a) 1 only
news in the context of relations between: (b) 1 and 3 only
(a) South Korea and Japan (c) 2 and 3 only
(b) Great Britain and Northern Ireland (d) 1, 2 and 3
(c) France and Spain
(d) United Kingdom and France

99. "The festival is celebrated every year on the


full moon of Lamta (February-March) of the
Meitei lunar calendar. It begins just after
sunset followed by the "Burning of the Straw
Hut". Unlike Holi, it is celebrated with a
traditional twist. During these five days,
Manipur comes alive with sporting events
during the day and traditional “thabal
chongba” dance the night."
Which of the following festivals is being
described in the passage given above?
(a) Yoshang Festival
(b) Magh Bihu
(c) Losar
(d) Saga Dawa

Copyright © by Vision IAS


All rights are reserved. No part of this document may be reproduced, stored in a retrieval system or
transmitted in any form or by any means, electronic, mechanical, photocopying, recording or otherwise,
without prior permission of Vision IAS.

17 www.visionias.in ©Vision IAS

FREE BY KING R QUEEN P [ऋषभ राजपूत]


VISIONIAS
www.visionias.in
ANSWERS & EXPLANATIONS
GENERAL STUDIES (P) TEST – 4130 (2024)

Q 1.B
• The Harappan culture belongs to the Bronze Age. The people of Harappa used many tools and
implements of stone, but they were very well acquainted with the manufacture and use of bronze.
• Bronze was made by the smiths by mixing tin with copper. Since none of the two metals was easily
available to the Harappans, bronze tools are not prolific in Harappa.
• The bronze tools and weapons recovered from the Harappan sites contain a smaller percentage of tin.
However, the kit of bronze goods left by the Harappans is considerable, which suggests that the bronze
smiths constituted an important group of artisans in the Harappan society. They produced not only
images and utensils but also various tools and weapons such as axes, saws, knives and spears. But
this civilization was much before Iron age, reference and use of iron is not there in Harappan
civilization. Hence statement 1 is not correct.
• Several other important crafts flourished in the Harappan towns. A piece of woven cotton has been
recovered from Mohenjodaro, and textiles impressions found on several objects. Spindle whorls were
used for spinning. Weavers wove ‘cloth of wool and cotton. Huge brick structures suggest that brick-
laying was an important craft. They also attest to the existence of a class of masons.
• The Harappans also practiced boat-making. As will be shown later, seal-making and terracotta
manufacture were also important crafts. The goldsmiths made jewellery of silver, gold and precious
stones; the first two may have been obtained from Afghanistan and the last from south India. The
Harappans were also experts in bead-making. Hence statement 2 is correct.
• The potter's wheel was in full use, and the Harappans produced their own characteristic
pottery, which was made glossy and shining. Hence statement 3 is correct.

Q 2.B
• Madanna and Akkanna were two Brahmin brothers who rose to prominence in the 17th century in the
final two decades of the Golconda sultanate.
• Abul Hasan Qutb Shah (1626-72 CE) was the Sultan of Golconda at that time. He had purchased peace
with the Mughals in 1657 CE by giving away his daughter in marriage to Prince Muhammad Sultan, the
eldest son of Aurangzeb. He was a Shia, and had entrusted the work of administration to his two capable
Brahmin ministers, Madanna and Akhanna.
• Madanna and Akkanna helped the Maratha king Shivaji to an expedition into Bijapur, Karnataka. Hence,
option (b) is the correct answer.

Q 3.D
• Indigenous literature about India can be supplemented by foreign accounts. To India came the Greek,
Roman and Chinese visitors, either as travelers or religious converts, and they left behind accounts of the
things that they saw. It is remarkable that Alexander's invasion finds no mention in Indian sources, and it
is entirely on the basis of the Greek sources that we have to reconstruct the history of his Indian exploits.
• The Greek visitors mention Sandiokottas, a contemporary of Alexander the Great who invaded India in
324 B C. Prince Sandrokottas is identified with Chandragupta Maurya, whose date of accession is fixed at
322 BC.
o This identification has served as the sheet-anchor in ancient Indian chronology The Indika of
Megasthenes, who came to the court of Chandragupta Maurya, has been preserved only in fragments
quoted by subsequent classical writers.
• Greek and Roman accounts of the first and second centuries A.D. mention many Indian ports and
enumerate items of trade between India and the Roman Empire.
1 www.visionias.in ©Vision IAS

FREE BY KING R QUEEN P [ऋषभ राजपूत]


o The Periplus of the Erythrean Sea and Ptolemy's Geography, both written in Greek, provide
valuable data for the study of the ancient geography and commerce of India. The date ascribed
to the first ranges between A.D. 80 and 115, while the second is attributed to about A.D. 150.
o Pliny’s Naturalis Historia, which belongs to the first century AD., was written in Latin and tells
us about trade between India and Italy.
• Hence option (d) is the correct answer.

Q 4.C
• Dhrupad is primarily a form of chant or worship where a singer offers prayer to the Lord through Nada or
sounds. It is believed to have been a further elaboration of the prabandha structure. While it might have
had an impetus for popularity even by the 14th century, it finds a blossoming period from the 15th century
onwards to about the 18th century.
• During these centuries we meet the most respected and renowned singers and patrons of this form.
There was Man Singh Tomar, the Maharaja of Gwalior. It was he who was mainly responsible for
the enormous vogue of dhrupad. There were Baiju, Bakshu and others. Swami Haridasa a hermit of
Brindavan was not only a dhrupadiya, but one of the most central figures in the Bhakti cult in the
Northern areas of India. By tradition, he was the guru of Tansen, one of the best-known dhrupad
singers and one of the nine jewels of Emperor Akbar’s court. Hence, statements 1 and 2 are correct.
• In structure, dhrupad has two parts, the anibaddha section and the sanchari dhrupad proper. The first is
free alap. The dhrupad proper is a song in four parts: the asthayee, the antara, the Sanchari and the
abhoga. The essential quality of the dhrupadic approach is its sombre atmosphere and emphasis on
rhythm.
• There were four schools or vanis of singing the dhrupad. The Gauhar vani developed the raga or
unadorned melodic figures. The Dagarvani emphasized melodic curves and graces. The Khandar vani
specialised in quick ornamentation of the notes. Nauhar vani was known for its broad musical leaps and
jumps. These vanis ‘are now indistinguishable.
• The dhrupad is even now highly respected and can be heard on the concert platform but more often in
temples of North India. The dhrupad has somewhat receded to the background and is not so popular with
the masses. The Been and Pakhawaj which were closely associated with the dhrupad also do not find
much patronage these days.

Q 5.A
• Vaisheshika philosophical system: The Sanskrit philosopher Kanada Kashyapa is believed to have
expounded its theories and is credited with founding the school.
• The Vaisheshika system is a realistic, analytic, and objective philosophy of the world.
• It tries to distinguish between the various kinds of ultimate things and to classify all the objects
under five elements - Earth, Water, Air, Fire, and Ether - existing in the form of Atoms, Time, Space,
Minds, and Self. Hence statement 2 is not correct.
• The creation of the world begins when the atoms of these five elements start to combine, and when they
disintegrate, the world comes to an end.
• Vaisheshika, thus postulates a dualism of the matter and soul and declares that salvation depends on
fully recognizing the atomic nature of the universe and its difference from the soul. Hence statement 1 is
correct.

Q 6.B
• The most important of the native successors of the Mauryas in the Deccan and in central India were the
Satavahanas., The Satavahanas are considered to be identical with the Andhras who are mentioned in the
Puranas The Puranas speak only of the Andhra rule and not of the Satavahana rule.
o On the other hand, the name Andhra does not occur in Satavahana inscriptions. According to some
Puranas altogether the Andhras ruled for 300 years and this period from about mid-1st century BCE to
early 3rd century CE is assigned to the rule of the Satavahana dynasty.
• The Satavahanas started the practice of granting tax-free villages to Brahmanas and Buddhist
monks. The cultivated fields and villages granted to them were declared free from molestation by royal
policemen and soldiers, and all kinds of royal officers. These areas therefore became small independent
islands within the Satavahana kingdom. Hence statement 1 is correct.
• The official language of the Satavahanas was Prakrit. All inscriptions were composed in this language
and written in the Brahmi script, as was the case in Asokan times. Some Satavahana kings may have
composed Prakrit books. One Prakrit text called Gathasaptasati is attributed to a Satavahana king called
Hala. Hence statement 2 is correct.
2 www.visionias.in ©Vision IAS

FREE BY KING R QUEEN P [ऋषभ राजपूत]


• The Satavahanas may have used gold as bullion, for they did not issue gold coins as the Kushans
did. They issued mostly coins of lead, which is found in the Deccan. They also issued potin, copper and
bronze money. Hence statement 3 is not correct.

Q 7.A
• In later Vedic times popular assemblies lost in importance, and ‘royal power increased at their cost The
vidatha completely disappeared,’ The sabha and samiti continued to hold the ground, but their character
changed. They came to be dominatéd by princes and rich nobles. Women were no longer permitted to sit
on the sabha, and it was now dominated by nobles and brahmanas.
o The formation of wider kingdoms made the king more powerful. Tribal authority tended to become
territorial.
o But even in later Vedic times the king did not possess a standing army. Tribal units were
mustered in times of war, and, according to one ritual for success in war, the king had to eat along
with his people from the same plate.
• The later Vedic society came to be divided into four varnas called the brahmanas, rajanyas or
kshatriyas, vaisyas and sudras. The growing cult of sacrifices enormously added to the power of the
brahmanas.
• In later Vedic period, in addition to cows, which were usually given as sacrificial gifts, gold, cloth and
horses were also givan, Sometimes the priests claimed portions of territory as dakshina, but the grant of
land as sacrificial fee is not well established in the later Vedic period.
• People worshipped gods for the same material reasons in later Vedic period as they did in earlier times.
However, the mode of worship changed considerably.
o Prayers continued to be recited, but they ceased to be the dominant mode of placating the gods.
Sacrifices became far more important, and they assumed both public and domestic character. Public
sacrifices involved the king and the whole of the community.
• Hence option (a) is the correct answer.

Q 8.D
• The Mauryan state also provided irrigation facilities and regulated water supply for the benefit of
agriculturists. Megasthenes informs us that in the Maurya empire the officer measured the land as in
Egypt and inspected the channels through which water was distributed into smaller channels. Hence
statement 1 is correct.
• Striking social development of the Maurya period was the employment of slaves in agricultural
operations. Megasthenes states that he did not notice any slaves in India. But there is no doubt that
domestic slaves were found in India from Vedic times onwards.
• For the first time in the Maurya period slaves were engaged in agricultural work on a large scale.
The state maintained farms, on which numerous slaves and hired labourers were employed. Hence
statement 2 is correct.
• Kautilya advised that new settlements should be founded with the help of cultivators, who were
apparently vaisyas, and with that of sudra labourers who should be drafted from overpopulated areas.
• In order to bring the virgin soil under cultivation, the new peasants were allowed remission in tax
and supplied with cattle, seeds and money. The state followed this policy in the hope that it would get
back what it had given. Hence statement 3 is correct.

Q 9.B
• Harisena was a 4th-century Sanskrit poet and a court official. He was an important figure in the
court of Gupta emperor, Samudragupta. His most famous poem, written c. 345 C.E., describes the
bravery of Samudragupta and is inscribed on the Allahabad Pillar.
o In a long inscription the poet enumerates the peoples and countries that Samudragupta conquered. The
inscription is engraved at Allahabad on the same pillar as carries an inscription of the peace-loving
Asoka.
• Amarasimha was a Sanskrit grammarian and poet in the court of Chandragupta II.
o He wrote a book on Sanskrit grammar called Amarakosa.
• Kalidasa was a Sanskrit poet and dramatist in the court of Chandragupta II.
o His works include Abhijnanashakuntala, Vikramorvashi, Malavikagnimitra, the epic poems
Raghuvamsha and Kumarasambhava and the lyric “Meghaduta”.
• Ravikirti was court poet of Pulakesin II (Chalukyan King) who wrote famous Aihole inscription.
• Hence option (b) is the correct answer.

3 www.visionias.in ©Vision IAS

FREE BY KING R QUEEN P [ऋषभ राजपूत]


Q 10.C
• Jamism taught five doctrines:
o do not commit violence
o do not speak a lie
o do not steal
o do not acquire property
o observe contwence (brahmacharya)
✓ It is said that only the fifth doctrine was added by Mahavira, the other four being taken over by
him from previous teachers.
• Jainism recognized the existence of the gods but placed them lower than the jina. Hence statement 1 is
not correct.
• It did not condemn the varna system, as Buddhism did. Hence statement 2 is not correct.
• According to Mahavira, a person is born in a high or in a lower varna is consequence of the sins or the
virtues acquired by him in the previous birth. Mahavira looks for human values even in a chandala. In his
opinion through pure and meritorious life members of the lower castes can attain liberation.
• Jainism mainly aims at the attainment of freedom from worldly bonds. It is not necessary to use any
ritual for acquiring such liberation. It can be obtained through full knowledge and action. Full knowledge,
action and liberation are considered to be the three gems or ratnas of Jaimsm. Hence statement 3 is
correct.
• In order to spread the teachings of Jainism, Mahavira organized an order of his followers that admitted
both men and women.

Q 11.A
• Women slaves were prevalent during Rig Vedic age. For example, we hear of gifts made fo priests is
stated in terms of cows and women slaves and never in terms of the measurement of land.
o The term for family (kula) is mentioned rarely in the Rig Veda. It comprised not only mother, father,
sons, slaves. Hence statement 1 is correct.
• The Rig Vedic people may have occasionally occupied pieces of land, but land did not form a well-
established type of private property. Hence statement 2 is not correct.
• In Rig Vedic times, we also notice the practice of levirate and widow remarriage. There are no
examples of child marriage and the marriageable age in the Rig Veda seems to have been 16 to
17. Hence statement 3 is not correct.
• The fourth division called the sudras appeared towards the end of the Rig Vedic period, because it is
mentioned for the first time in the tenth Book of the Rig Veda, which is the latest addition.

Q 12.C
• Odisha, on the eastern sea coast, is the home of Odissi, one of the many forms of Indian classical
dance. Sensuous and lyrical, Odissi is a dance of love and passion touching on the divine and the
human, the sublime and the mundane. The Natya Shastra mentions many regional varieties, such as the
south-eastern style known as the Odhra Magadha which can be identified as the earliest precursor of
present day Odissi.
• Archaeological evidence of this dance form dating back to the 2nd century B.C. is found in the caves of
Udayagiri and Khandagiri near Bhubaneshwar. Later, innumerable examples of the Buddhist sculptures,
the tantric images of dancing Yoginis, the Nataraja, and other celestial musicians and dancers of early
Shaivite temples bear testimony to a continuing tradition of dance from the 2nd century B.C.E to the 10th
century C.E.
• Odissi is a highly stylised dance and to some extent is based on the classical Natya Shastra and the
Abhinaya Darpana. In fact, it has derived a great deal from the Abhinaya Darpana Prakasha by
Jadunatha Sinha, the Abhinaya Chandrika by Rajmani Patra, and the Abhinaya Chandrika by
Maheshwara Mahapatra.
• The techniques of movement are built around the two basic postures of the Chowk and the
Tribhanga. The chowk is a position imitating a square – a very masculine stance with the weight of the
body equally balanced. The tribhanga is a very feminine stance where the body is deflected at the neck,
torso and the knees.
• The torso movement is very important and is an unique feature of the Odissi style. With the lower
half of the body remaining static, the torso moves from one side to the other along the axis passing
through the centre of the upper half of the body. Great training is required for this control so as to
avoid any shoulder or hip movement. There are certain foot positions with flat, toe or heel contact. These

4 www.visionias.in ©Vision IAS

FREE BY KING R QUEEN P [ऋषभ राजपूत]


are used in a variety of intricate combinations. There are also numerous possibilities of leg movements.
Almost all leg movements are spiral or circular, whether in space or on the ground.
• For centuries maharis were the chief repositories of this dance. The maharis, who were originally temple
dancers came to be employed in royal courts which resulted in the degeneration of the art form. Around
this time, a class of boys called gotipuas were trained in the art, they danced in the temples and also for
general entertainment. Many of today’s gurus of this style belong to the gotipua tradition. Hence, option
(c) is the correct answer.

Q 13.A
• Recent context:
o Starlink has placed multiple satellites in the orbit.
o Bharti-backed satellite operator, OneWeb plans to complete the launch of all of its low-Earth
connectivity satellites as early as 26 March, aboard a rocket by Indian Space Research
Organisation's commercial arm, NewSpace India Limited.
• Space-based internet is the ability to use satellites to send and receive data. It is much faster and has the
ability to work across the globe.
• The major difference between satellite-based internet and a satellite TV (DTH) is that, unlike DTH,
satellite internet is 2-way communication. Hence, statement 2 is correct.
• Most of the existing space-based Internet systems use satellites in geostationary orbit, however, some
also use the Low Earth Orbit (LEO). Hence statement 1 is correct.
• Starlink is the world's first and largest satellite constellation using a low Earth orbit to deliver
broadband internet capable of supporting streaming, online gaming, video calls and more. Hence,
statement 3 is not correct.
• Geostationary orbit is located at the height of 35,786 km over the Earth’s surface, directly above the
Equator. Due to the larger distance, the transmission from a satellite in geostationary orbit has
a latency of about 600 milliseconds. The longer distance in comparison to the LEO results in greater
latency.
• Space Internet with Low Earth Orbit: The LEO extends up to 2,000 km above the Earth’s surface.
The presence of satellites at a lower height helps to bring the lag down to 20-30 milliseconds, roughly the
time it takes for terrestrial systems to transfer data.
• Satellites in geostationary orbit (GEO) circle Earth above the equator from west to east following
Earth’s rotation – taking 23 hours 56 minutes and 4 seconds – by traveling at exactly the same rate as
Earth. This makes satellites in GEO appear to be ‘stationary’ over a fixed position. In order to perfectly
match Earth’s rotation, the speed of GEO satellites should be about 3 km per second at an altitude of
35,786 km. Compared to this, LEO is relatively close to Earth’s surface (less than 1000 km). Its close
proximity makes it useful for several reasons including satellite imaging, placement of the International
Space Station (ISS), etc. Satellites in this orbit travel at a speed of around 7.8 km per second; at this
speed, a satellite takes approximately 90 minutes to circle Earth, meaning the ISS travels around Earth
about 16 times a day.

Q 14.A
• Recent context: Ancient Monuments and Archaeological Sites and Remains Amendment Bill has been
reintroduced in the Budget session.
• The AMASR Act was passed by the Parliament in 1958 for the purpose of protection and preservation
of archaeological and historical monuments and sites. It also provides for the regulation of
archaeological excavations and for the protection of sculptures, carvings, and other such objects.
Hence statement 1 is correct.
• In the original Act of 1958, “ancient monument” is defined as “any structure, erection, or monument,
or any tumulus or place of interment, or any cave, rock-sculpture, inscription, or monolith which is
of historical, archaeological, or artistic interest and which has been in existence for not less than 100
years”. Also, “Archaeological sites and remains” mean “any area which contains or is reasonably
believed to contain ruins or relics of historical or archaeological importance which have been in
existence for not less than 100 years”. Hence statement 2 is not correct.
• The AMASR Amendment Bill was first introduced in Lok Sabha on July 18, 2017. Amendments in the
Bill include allowing the construction of public works in “prohibited areas”, and the approval and
impact assessment of such public works.
• The original Act prohibits construction in an area of 100 meters around protected monuments, and
the central government (not State) can extend this area beyond 100 meters. Hence statement 3 is not
correct. With the Amendment, the government will be allowed to take up infrastructure projects for
5 www.visionias.in ©Vision IAS

FREE BY KING R QUEEN P [ऋषभ राजपूत]


public works in this prohibited area. For the construction (or reconstruction, repair, or renovation), the
relevant central government department will be required to submit an application to the competent
authority. The central government will determine whether the construction works in question qualify
as public works, and the authority will then convey the decision of the central government to the
applicant within ten days of the receipt of the decision.

Q 15.B
• The Harappans looked upon the earth as a fertility goddess and worshipped her in the same manner as the
Egyptians worshipped the Nile goddess "Isis".
• Some Vedic texts show reverence to the earth goddess, although she is not given any prominence. It took
a long time for the worship of | the supreme goddess to develop in Hinduism, only from the sixth
century A D. various mother-goddesses such as Durga, Amba, Kali, ‘Chandi, etc., came to be
regarded as goddesses in the Puranas and in the Tantra literature. In course of time, every village
came to have its own separate goddess. Hence statement 1 is not correct.
• The male deity is represented on a seal ‘This god has three heads and has horns. The seal immediately
recalls to our mind the traditional image of Pasupati Mahadeva. The four animals surrounding the god
look towards the four directions of the earth.
o In addition to the use of the image of Siva, we also come across the prevalence of phallus
worship, which became so intimately connected with Siva in later times. Numerous symbols of the
phallus and female sex organs made of stone have been found m Harappa.
o They were possibly meant for worship. The Rig Veda speaks of the non-Aryan people, who were
phallus worshippers. The phallus worship which started in the days of Harappa came to be
recognized as a respectable form of worship in Hindu society. Hence statement 2 is correct.
• The people of the Indus region also worshipped trees. The picture of a god is represented on a seal in the
midst of the branches of the pipal. This tree continues to be worshipped to this day.
• Animals were also worshipped in Harappan times, and many of them are represented on seals. The most
important of them is the humped bull.
• Similarly the animals surrounding Pasupati Mahadeva indicate that these were worshipped.
• Obviously, the inhabitants of the Indus region worshipped gods in the form of trees, animals and human
beings. But the gods were not placed in temples, a practice that was common in ancient Egypt and
Mesopotamia. Nor can we say anything about the religious beliefs of the Harappans without being able to
read their script.
• Amulets have been found in large numbers. Probably the Harappans believed that ghosts and evil forces
were capable of harming them and therefore used amulets against them. The Atharva Veda, which is
considered to be a non-Aryan work, contains many charms and spells, and recommends amulets for
warding off diseases and evil forces.

Q 16.B
• The continuous quest by the sages gave birth to great philosophical systems, which looked upon man and
the universe with an unbiased, free and rational mindset.
• The important systems are Charvaka, Jaina, Buddha, Vaisesika, Nyaya, Samkhya, Yoga, Mimamsa and
Vedanta.
• The first three systems are nastika i.e. do not believe in the authority of the Vedas and the God, while all
others are astika, i.e. believe in the Vedas and the God.
• The Charvaka system (also called Lokayata) believes only in materialism.
• The physical body composed of material elements is the only essence of man.
• Death only is the end of man ; and enjoyment of pleasures are the only objects in life.
• There is no life beyond death, no heaven or hell, no 'Law of karma, and no rebirth.
• The Charvaka system does not believe in soul, god, or any other life beyond the present one.
• Hence, option (b) is the correct answer.

Q 17.B
• The Gupta bureaucracy was not as elaborate as that of the Mauryas, The most important officers in the
Gupta empire were the kumaramatyas. They were appointed by the king in the home provinces and
possibly paid in cash.
o Since the Guptas were possibly vaisyas, recruitment was not confined to the upper varnas only. But
several offices came to be combined in the hands of the same person, and posts became hereditary.
This naturally weakened the royal control.

6 www.visionias.in ©Vision IAS

FREE BY KING R QUEEN P [ऋषभ राजपूत]


• The Guptas organized a system of provincial and local administration. The empire was divided into
divisions (bhuktis), and each bhukti was placed under the charge of an uparika. The bhuktis were
divided into districts (vishayas), which were placed under the charge of vishayapati. In eastern
India, the vishayas were divided into vithis, which again were divided into villages.
• The village headman became more important in Gupta times He managed the village affairs with the
assistance of elders. With the administration, of a village or a small town leading local elements were
associated. No land transactions could be effected without their consent.
• Hence option (b) is the correct answer.

Q 18.B
• The Mansabdari system was a unique administrative system developed by the Mughals in India. It was
designed and established by Mughal Emperor Akbar. Hence statement 1 is not correct.
• The term Mansabdar referred to an individual who holds a mansab, meaning a position or rank. It was a
grading system used by the Mughals to fix Zat or rank, Salary and Sawar or military responsibilities. The
Mansabdars were military commanders, high civil and military officers
• Ahadis were the individual troopers maintained/recruited by the Mughal rulers. They were a highly
trusted corps, being recruited directly by the emperors and having their own muster master. They were
not associated with the Manasabdars. The Mansabdars had to maintain their own force. The sawar
rank of a mansabdar indicated the number of horsemen needed to the kept by the Mansabdar. Hence
statement 2 is correct.
• The salaries of the mansabdars were stated in rupees but they were generally not paid in cash, but
by assigning them a jagir. Mansabdars preferred a jagir because cash payments were likely to be delayed
and sometimes entailed a lot of harassment. However, Jagirs were not hereditary and were transferred by
rulers at any time. Hence, statement 3 is correct.
• Tajwiz was a petition presented by a nobleman to the emperor, recommending that an applicant be
recruited as mansabdar. If the applicant was found suitable a mansab was granted to him.

Q 19.D
• Recent Context: As India assumes its G20 presidency, it has introduced the Startup20 Engagement
group (SUMup) which has the potential to revolutionize the G20.
• It is the only new group by which G20 has turned itself into an ambidextrous institution, one where
both large corporations and startups have an equal voice in taking the economies forward.
• In the new architecture, while the existing B20 Engagement Group continues its focus on
corporations, the Startup20 takes on the policy issues concerning the global startup ecosystem, with
the necessary linkages between the two groups.
• B20 Engagement Group-
o It is a forum for international business leaders from the G20 countries.
o Established in 2010, it is among the most prominent Engagement Groups in G20, with companies and
business organizations as participants.
o It aims to provide recommendations to the G20 on issues such as economic growth, trade,
investment, digitalization, sustainability, and job creation.
o The Confederation of Indian Industry (CII) has been designated as the B20 Secretariat for India’s
G20 Presidency.
• Startup20-
o Initiated under India’s G20 Presidency.
o It aims to create a global narrative for supporting startups and enabling synergies between
startups, corporates, investors, innovation agencies, and other key ecosystem stakeholders.
o The engagement group comprises three task forces where delegates will come together to discuss
efficient policy frameworks to promote the scaling up of startups in the G20 nations-
✓ Foundation & Alliance
✓ Finance, an
✓ Inclusion & Sustainability.

Q 20.B
• Recent Context: Pradhan Mantri particularly vulnerable tribal groups (PVTG) Development Mission will
be launched as part of ‘Reaching The Last Mile’.
• In order to improve the socio-economic conditions of the particularly vulnerable tribal groups
(PVTGs), the Pradhan Mantri PVTG Development Mission will be launched. This will saturate
PVTG families and habitations with basic facilities such as safe housing, clean drinking water, and
7 www.visionias.in ©Vision IAS

FREE BY KING R QUEEN P [ऋषभ राजपूत]


sanitation, improved access to education, health and nutrition, road and telecom connectivity, and
sustainable livelihood opportunities.
• Rs.15,000 crores will be made available to implement the Mission in the next three years under
the Development Action Plan for the Scheduled Tribes.
• The Pradhan Mantri PVTG Mission will be launched as part of ‘Reaching The Last Mile’, one of the
seven Saptarishi priorities enlisted in this year’s Budget. There are 75 PVTG groups in India that will
benefit from this scheme.
• In 1973, the Dhebar Commission created Primitive Tribal Groups (PTGs) as a separate category,
which are less developed among the tribal groups. Hence statement 1 is not correct.
• PVTGs are more vulnerable among the tribal groups. Due to this factor, more developed and assertive
tribal groups take a major chunk of the tribal development funds, because of which PVTGs need more
funds directed for their development. In this context, in 1975, the Government of India initiated to
identify the most vulnerable tribal groups as a separate category called PVTGs and declared 52 such
groups, while in 1993 an additional 23 groups were added to the category, making it a total of 75 PVTGs
out of 705 Scheduled Tribes, spread over 17 states and one Union Territory (UT), in the country
(2011 census). In 2006, the Government of India renamed the PTGs as Particularly Vulnerable Tribal
Groups (PVTGs).
• PVTGs have some basic characteristics –
o they are mostly homogenous,
o they have a small population,
o they are relatively physically isolated,
o their social institutes are cast in a simple mold,
o there is an absence of written language,
o the technology used is relatively simple, etc.
• However, they are identified Government of India by following the below-mentioned criteria-
o Pre-agricultural level of technology
o Low level of literacy
o Economic backwardness
o A declining or stagnant population. Hence statement 2 is correct.
• Examples of PVTGs include: Cholanaikayan (Kerala), Kathodi (Gujarat), Jarawas (Andaman &
Nicobar Islands), Koraga (Karnataka). Hence statement 3 is correct.

Q 21.D
• The term that Indo-Persian sources of the Mughal period most frequently used to denote a peasant was
raiyat (plural, riaya) or muzarian. In addition, we also encounter the terms kisan or asami.
• Sources from the seventeenth century refer to two kinds of peasants – khud-kashta and pahi-kashta.
o The former were residents of the village in which they held their lands. The latter were non-resident
cultivators who belonged to some other village but cultivated lands elsewhere on a contractual basis.
o People became pahi-kashta either out of choice—for example, when terms of revenue in a distant
village were more favourable—or out of compulsion—for example, forced by economic distress after
a famine.
• Seldom did the average peasant of north India possess more than a pair of bullocks and two ploughs; most
possessed even less.
• In Gujarat, peasants possessing about six acres of land were considered to be affluent; in Bengal, on the
other hand, five acres was the upper limit of an average peasant farm; 10 acres would make one a rich
asami. Cultivation was based on the principle of individual ownership. Peasant lands were bought and
sold in the same way as the lands of other property owners.
• Hence option (d) is the correct answer.

Q 22.B
• Although Sakas established their rule in different parts of the country, only those who ruled in western
India held power for any considerable length of time, for about four centuries or so. The most famous
Saka ruler in India was Rudradaman I (AD 130-150) He ruled not only over Sindh, Kutch and Gujarat, but
had also recovered from the Satavahanas Konkan, the Narmada valley, Malwa and Kathiawar.
• He is famous in history because of the repairs he undertook to improve the Sudarsana lake in the
semi-arid zone of Kathiawar. Hence statement 1 is correct.
o This lake had been in use for irrigation for a long time and was as old as the time of the Mauryas.
• Rudradaman was a great lover of Sanskrit. Although a foreigner settled in India, he issued the first-
ever long inscription in chaste Sanskrit. Hence statement 2 is correct.
8 www.visionias.in ©Vision IAS

FREE BY KING R QUEEN P [ऋषभ राजपूत]


o All the earlier longer inscriptions that we have in this country were composed in Prakrit.
• It was during the reign of Parthian king Gondophernes, St, Thomas is said to have come to India
for the propagation of Christianity. Hence statement 3 is not correct.

Q 23.D
• Kautilya's Arthashastra, a book on statecraft was written in the Maurya period. The text is divided into 15
chapters known as books.
• Different books deal with different subject matter concerning polity, economy and society.
• It appears that even before the final version of Arthashastra was written in the fourth century B.C. by
Kautilya, there appeared a tradition of writing on and teaching of statecrafts because Kautilya
acknowledges his debt to his predecessors in the field.
• Mudrarakshasha, a play written by Vishakhadatta, also gives a glimpse of society and culture.
• Kalidasa's Malavikagnimitram is based on some events of the reign of Pusyamitra Sunga, a dynasty which
followed the Mauryas.
• Bhasa and Sudraka are other poets who have written plays based on historical events.
• Banabhatta's Harshacharita throws light on many historical facts about which we could not have
known otherwise.
• Vakpati wrote Gaudavaho, based on the exploits of Yasovarman of Kanauj.
• Similarly, Bilhana's Vikramankadevacharita describes the victories of the later Chalukya king
Vikramaditya.
• Hence, option (d) is the correct answer.

Q 24.B
• The people of the Neolithic age used tools and implements of polished stone. They particularly used stone
axes, which have been found in large numbers throughout the greater part of the country.
• Burzahom is an important Neolithic settlement in Kashmir. The Neolithic people lived here on a
plateau in pits, and probably had a hunting and fishing economy.
• The people of Burzahom used coarse grey pottery. It is interesting that the Burzahom domestic dogs were
buried with their masters in their graves. Pit dwelling and the placing of domestic dogs in the graves of
the masters do not seem to be the practice with Neolithic people in any other part of India. The
earliest date for Burzahom is about 2400 B.C.
• Hence option (b) is the correct answer.

Q 25.B
• Recent context: India has called upon G20 countries to adopt multilateral action for faster
extradition of fugitive economic offenders.
• Fugitive Economic Offenders Act, 2018:
o The Act defines a Fugitive Economic Offender (FEO) as a person against whom an arrest
warrant has been issued for committing an offense listed in the Act and the value of the offense is at
least Rs. 100 crore. Hence, statement 1 is not correct.
o It seeks to confiscate the properties of economic offenders who have left the country to avoid
facing criminal prosecution or refuse to return to the country to face prosecution. A notice is issued
under the FEO Act requiring the accused to appear before the court within 6 weeks of the date of
notice failing which the court shall declare him a fugitive and confiscate his properties to the
Central Government. Hence, statement 2 is correct.
• Some of the offenses listed in the act are:
o Counterfeiting government stamps or currency.
o Cheque dishonor.
o Money laundering.
o Transactions defrauding creditors.
• The G20 or Group of 20 is an intergovernmental forum comprising 19 countries and the European
Union. It works to address major issues related to the global economy, such as international financial
stability, climate change mitigation, and sustainable development.
Q 26.B
• The judicial system was well developed under the Guptas than in earlier times. Several law-books were
compiled in this period.
• For the first time civil and criminal law were clearly defined and demarcated. Theft and adultery
came under criminal law. Disputes regarding various types of property came under civil law. Elaborate
laws were laid down about inheritance. Hence statement 1 is correct.
9 www.visionias.in ©Vision IAS

FREE BY KING R QUEEN P [ऋषभ राजपूत]


• During this period also many laws continued to be based on differences in varnas. It was the duty of
the king to uphold the law. The king tried cases with the help of brahmana priests. Hence statement 2 is
correct.
• The guilds of artisans, merchants and others were governed by their own laws, Seals from Vaisali
and from Bhita near Allahabad indicate that these guilds flourished exceedingly well in Gupta times.
Hence statement 3 is correct.

Q 27.C
• Recent context: Recently in a case the CJI refused to accept information in a 'sealed cover’.
• Sealed Cover Jurisprudence is a practice used by the Supreme Court and sometimes lower courts, of
asking for or accepting information from government agencies in sealed envelopes that can only
be accessed by judges.
• While a specific law does not define the doctrine of sealed cover, the Supreme Court derives its
power to use it from Rule 7 of Order XIII of the Supreme Court Rules and Section 123 of the Indian
Evidence Act of 1872.
• Rule 7 of Order XIII of the Supreme Court Rules: According to the rule, if the Chief Justice or court
directs certain information to be kept under sealed cover or considers it of confidential nature, no
party would be allowed access to the contents of such information, except if the Chief Justice himself
orders that the opposite party be allowed to access it. It also mentions that information can be kept
confidential if its publication is not considered to be in the interest of the public.
• Section 123 of the Indian Evidence Act of 1872: Under this act, official unpublished documents
relating to state affairs are protected and a public officer cannot be compelled to disclose such
documents.
• Other instances where information may be sought in secrecy or confidence are when its publication
impedes an ongoing investigation, such as details that are part of a police case diary.

Q 28.D
• Antonio Monserrate (1536-1600) was a Portuguese priest who accompanied two other priests, Father
Rodolfo Acquaviva, and Father Francisco Enriquez, on the first Jesuit mission to the court of Emperor
Akbar (1542-1605; reigned 1556-1605). Aquaviva and Monserrate went back in 1583, belying the
hopes the Portuguese entertained of Akbar’s conversion to the Christian faith. Hence, statement 1 is
correct.
• François Bernier, a Frenchman, was a doctor, political philosopher, and historian. Like many others,
he came to the Mughal Empire in search of opportunities. He was in India for twelve years, from 1656 to
1668, and was closely associated with the Mughal court, as a physician to Prince Dara Shukoh, the
eldest son of Emperor Shah Jahan.
• Jahangir was the Mughal emperor who ruled from 1605 until his death in 1627. Thus, his reign preceded
the arrival of François Bernier to India. Hence, statement 2 is correct.
• Sir Thomas Roe was an English Diplomat who visited Mughal emperor Jahangir’s court. In 1615,
Sir Thomas Roe came as an accredited ambassador of James I to the court of Jahangir, staying there till
February 1619. Sir Thomas Roe obtained more trading rights and privileges for the East India Company.
Accordingly, the English set up business centers at Agra, Ahmedabad and Broach. Slowly the English
East India Company succeeded in expanding its area of trade. Hence, statement 3 is not correct.
• Earlier Captain Hawkins arrived in the court of Jahangir in April 1609. But the mission to establish a
factory at Surat didn’t succeed due to opposition from the Portuguese, and Hawkins left Agra in
November 1611.

Q 29.C
• The Virashaiva Tradition emerged during the twelfth century in Karnataka. Basavanna (1106-68) is
regarded as the founder of this Virashaiva tradition. He was a minister in the court of a Kalachuri
ruler. His followers were known as Virashaivas (heroes of Shiva) or Lingayats (wearers of the linga).
• Shaikh Nizamuddin Auliya (1238 to 1325) was a Sufi saint of the Chishti Order. He thus belonged to
the thirteen and fourteen centuries AD and was not a contemporary of Basavanna. Hence, statement 1 is
not correct.
• Lingayats believe that on death the devotee will be united with Shiva and will not return to this
world. Thus they do not believe in the theory of rebirth. Therefore they do not practice funerary rites
such as cremation, prescribed in the Dharmashastras. Instead, they ceremonially bury their dead. Hence,
statement 2 is not correct.

10 www.visionias.in ©Vision IAS

FREE BY KING R QUEEN P [ऋषभ राजपूत]


• The vachanas (literally, sayings) composed in Kannada by women and men who joined the movement
serve as an important source to study the Virshaiva Movement.
• Virashaivism did not advocate the worship of idols and the worship of a multitude of Gods and
Goddesses. It emphasized the worship of only one God namely Lord Shiva. Shiva was the only Supreme
Being to be worshipped in the form of a Linga. Hence, statement 3 is correct.

Q 30.A
• During the year 1333, Muhammad bin Tughlaq commanded the Qarachil expedition to the Kullu-
Kangra region of present-day Himachal Pradesh in India. According to historical accounts from Badauni
and Ferishta, Tughluq's initial plan was to cross the Himalayas and launch an attack on China.
• Despite his intentions, Muhammad bin Tughlaq encountered opposition from the locals in Himachal.
The army of Tughlaq, which was not accustomed to fighting in the hills, was defeated by Prithvi Chand II
of the Katoch clan, a Hindu Rajput kingdom of Kangra. As a result, almost all of Tughlaq's troops,
which numbered around 100,000, were killed and compelled to withdraw.
• Hence option (a) is the correct answer.

Q 31.A
• Vedic Literature
o Six vedangas (limbs of Vedas) were evolved for the proper understanding of the Vedas.
o These are Siksha (phonetics), Kalpa (rituals), Vyakarna (grammar), Nirukta (etymology),
Chhanda (metrics), and Jyotisha (astronomy).
o Each vedanga has developed a credible literature around it which are in the sutra form i.e., precepts.
o This is a very precise and exact form of expression in prose which was developed by the ancient
Indians.
o Panini's Ashtadhyayi, book on grammar in eight chapters is the final culmination of this excellent art
of writing in sutra (precepts) in which every chapter is precisely interwoven.
o Besides the Vedas, the Brahmanas, the Aranyakas and the Upanishads are also included in the vedic
literature and are known as the later vedic literature.
• Hence, option (a) is the correct answer.

Q 32.B
• The smritis have played a very important role in Hindu life during the last two thousand years.
• These smritis define the religious duties, usage, laws, and social customs. In general, the smritis may be
regarded as the expanded and contemporary version of the Dharmasutras which covered the period from
about the sixth century to the third century B.C.
• The works on the smritis follow almost immediately afterward and continue for almost eight hundred
years or even more.
• The Manava Dharmasastra or Manusmriti is not only the oldest work of this class but is also the
most well-known and has its hold even today all over India.
• This was composed in about the first century B.C. Some other important smritis are Naradasmriti,
Vishnusmriti, Yajnavalkyasmriti, Brihaspatismriti, and Katyayanasmriti.
• They are all very important sources of law and social customs of contemporary society. These smritis
were declared to be of divine origin.
• The most outstanding work in the field of grammar, Mahabhasya written by Patanjali in the second
century B.C., is a commentary on Panini's Asthadhyayi. Hence statement 1 is not correct.
• After Patanjali, the center of Sanskrit grammar learning shifted to the Deccan where the Katantra school
flourished in the first century A.D.
• Sarvavarman, a scholar of great repute in the court of the Satavahana King Hala, composed the
grammar of Katantra. This work was short and handy which helped the learn of Sanskrit in about six
months. Hence statement 2 is correct.
• Hala wrote a great poetical work Gathasaptasati in Prakrit. Hence statement 3 is not correct.
• An important literary figure of the period was Asvaghosha. He was not only a play writer and poet but a
great Buddhist philosopher.
• He wrote Saundarananda, Buddhacharita, Vajrasuchi, and a number of other works. Buddhacharita is a
complete life of Buddha written in the form of Mahakavya.
Q 33.A
• Recent Context: The Intergovernmental Panel on Climate Change (IPCC) published its Synthesis
Report (SYR) on March 20, 2023, summarizing the findings of reports released during its Sixth
Assessment Cycle —
11 www.visionias.in ©Vision IAS

FREE BY KING R QUEEN P [ऋषभ राजपूत]


o The 1.5 C report of 2018,
o The Special Reports on Land and Oceans of 2019, and
o The three Assessment Reports published between 2021 and 2022.
• The SYR is presented in the wake of major global upheavals brought about by the COVID-19 pandemic,
the Russian invasion of Ukraine, and the subsequent global energy crisis.
• Key Findings of the Report:
o The sea surface temperature over the Indian Ocean is likely to increase by 1° to 2° C when there
is 1.5°C to 2°C global warming.
o In the report, scientists showed that the western Indian Ocean region experienced the largest
increase in marine heatwaves at a rate of about 1.5 events per decade, followed by the north Bay
of Bengal at a rate of 0.5 events per decade. The western Indian Ocean experienced 66
events while the Bay of Bengal had 94 events over the last four decades, between 1982 and 2018.
o Temperature breaching the 1.5° C threshold: Average temperatures have already touched 1.1°C
(about 1.07°C was contributed by human activities) above pre-industrial times.
o By 2030 there is a 50% chance that global surface temperature in any single year could exceed
1.5°C.
o The world is still not doing enough to limit global temperatures from breaching the 1.5° C threshold
despite there being multiple, feasible, and effective options to do so.
o Overshooting 1.5°C will result in irreversible adverse impacts on certain ecosystems with low
resilience, such as polar, mountain, coastal ecosystems, etc.
o To keep within the 1.5°C limit, emissions need to be reduced by at least 43% by 2030 compared
to 2019 levels, and at least 60% by 2035. This is the decisive decade to make that happen.
Q 34.A
• The style of temple architecture that became popular in northern India is known as nagara. In North
India, it is common for an entire temple to be built on a stone platform with steps leading up to
it. Further, unlike in South India it does not usually have elaborate boundary walls or gateways.
While the earliest temples had just one tower or shikhara, later temples had several. The
Garbhagriha is always located directly under the tallest tower.
• Unlike the Nagara temple, the Dravida temple is enclosed within a compound wall. The front wall has an
entrance gateway in its center, which is known as a gopuram. The shape of the main temple tower known
as Vimana in Tamil Nadu is like a stepped pyramid that rises up geometrically rather than the curving
shikhara of North India.
• In the South Indian temple, the word ‘shikhara’ is used only for the crowning element at the top of the
temple which is usually shaped like a small stupika or an octagonal cupola— this is equivalent to the
amlak and kalasha of North Indian temples. Whereas at the entrance to the North Indian temple’s
garbhagriha, it would be usual to find images such as mithunas and the river goddesses, Ganga and
Yamuna, in the south you will generally find sculptures of fierce dvarapalas or the door-keepers guarding
the temple. It is common to find a large water reservoir or a temple tank enclosed within the
complex. Subsidiary shrines are either incorporated within the main temple tower, or located as distinct,
separate small shrines beside the main temple. The North Indian idea of multiple shikharas rising together
as a cluster was not popular in South India. Hence, option (a) is the correct answer.

Q 35.C
• Bhavai is the traditional theatre form of Gujarat.
• The centers of this form are Kutch and Kathiawar. Hence statement 1 is correct.
• The instruments used in Bhavai are:
o bhungal,
o tabla,
o flute,
o pakhaawaj,
o rabaab,
o sarangi, manjeera, etc. Hence statement 2 is not correct.
• In Bhavai, there is a rare synthesis of devotional and romantic sentiments. Hence statement 3 is
correct.

Q 36.D
• The Gurjara-Pratihara was a dynasty that ruled much of Northern India from the mid-8th to the 11th
century. They ruled first at Ujjain and later at Kannauj. The power of the Gurjara-Pratihara dynasty was
weakened by dynastic strife. It was further diminished as a result of a great raid led by the Rashtrakuta
12 www.visionias.in ©Vision IAS

FREE BY KING R QUEEN P [ऋषभ राजपूत]


ruler Indra III who, in about 916, sacked Kannauj. Under a succession of rather obscure rulers, the
dynasty never regained its former influence.
• Pratihara break into numbwr of Rajput states. These are the Gahadavalas of Kannauj, which was a
Rajput dynasty that ruled parts of present-day Uttar Pradesh and Bihar, during the 11th and 12th centuries.
Their capital was located at Banaras. in the Gangetic plains, and for a brief period, they also controlled
Kannauj.
• The Paramara was a Rajput dynasty that ruled the Malwa region between the 9th and 14th centuries.
• Chauhans of Delhi and Ajmer were a Rajput dynasty, also known as the Chahamanas of Shakambhari,
who ruled over part of modern-day Rajasthan and neighboring areas.
• Kalachuri dynasty ruled between the 10-12th century. Chandelas of Jejakabhukti was a Rajput
dynasty in Central India. The Chandelas ruled much of the Bundelkhand region (then called Jejakabhukti)
between the 9th and 13th centuries. They belonged to the Chandel clan of the Rajputs.
• Hence option (d) is the correct answer.

Q 37.B
• During the Mughal Period the term ‘muzarians’ was used to describe the tenants who generally paid
land revenue at a higher rate. These two groups were the largest section among the cultivators in the
village. Hence, option (b) is the correct answer.
• The peasants who owned the land they tilled were called khudkasht. They paid land revenue at
customary rates. Some of them had many ploughs and bullocks which they let out: to their poorer brethren
tenants (Muzarians).
• The landless peasants and laborers often belonged to the class of people called kamin. Whenever there
was a famine—and famines were frequent—it was this class of peasants and the village artisans who
suffered the most.

Q 38.A
• The Rashtrakuta dynasty, a Hindu royal family that governed the Deccan region and adjacent territories
of India, held power between approximately 755 and 975 CE. The kingdom was founded by
Dantidurga who fixed his capital at Manyakheta or Malkhed near modern Sholapur.
• The greatest Rashtrakuta rulers were Govinds III (793-814) and Amoghavarsha (814-878).
Amoghavarsha ruled for 68 years. He was a great builder and built the capital city Manyakheta so as to
excel in the city of Indra.
• Al-Masudi paid a visit to India, During the period of 914 to 928 CE when Indra-III was in power and
ruled over the Rashtrakuta Empire. According to Al-Masudi, the Rashtrakuta king Balhara or
Vallbharaja was the greatest king of India and most of the Indian rulers accepted his suzerainty and
respected his envoy.
• The Kailash Rock-cut temple was constructed between 756 and 773 AD by the Rashtrakuta King
Krishna I, who ruled during the 8th century. The renowned temple is located at Ellora.
• Hence option (a) is the correct answer.

Q 39.B
• The Maurya period constitutes a landmark in the system of taxation in ancient India. Kautilya names
many taxes to be collected from peasants, artisans and traders. This required a strong and efficient
machinery for assessment, collection and storage.
• The Mauryas attached greater importance to assessment than to storage and depositing.
• The samaharta was the highest officer in charge of assessment and the sannidhata was the chief
custodian of the state treasury and store-house.
• The harm done to the state by the first is thought to be more serious than the harm caused by the second.
In fact, an elaborate machinery for assessment first appears in the Maurya period The list of taxes
mentioned in the Arthasastra is impressive, and if these wore really collected very little would be left to
the people to live on.
• Hence option (b) is the correct answer.

Q 40.C
• Akbar divided the empire into twelve provinces called Subhas in 1580. These were Bengal, Bihar,
Allahabad, Awadh, Agra, Delhi, Lahore, Multan, Kabul, Ajmer, Malwa and Gujarat. A governor
(subedar), a diwan, a bakhshi, a sadr, a qazi, and a waqia-navis were appointed to each of the
provinces. Thus, an orderly government based on the principle of checks and balances was extended to
the provinces. Hence options 2 and 4 are correct.
13 www.visionias.in ©Vision IAS

FREE BY KING R QUEEN P [ऋषभ राजपूत]


• The waqia-navis were the news reporters posted to all parts of the empire. Their reports were presented to
the emperor at the court through the mir bakhshi.
• The provinces were divided into sarkars which were further divided into Parganas. The chief
officers of the sarkar were the faujdar and the amalguzar, the former being in charge of law and
order, and the latter responsible for the assessment and collection of the land revenue. Thus, Amalguzar
was one of the principal officials at the Sarkar level and not the provincial level. Hence options 1 and 3
are not correct.
• The amalguzar was required to exercise general supervision over all types of land holdings so that
the imperial rules and regulations for the assessment and collection of land revenue were followed
uniformly.
• Faujdar was responsible for maintaining law and order (In the Sarkars/districts, the Faujdars were
usually in charge of maintenance of law and order) in his jurisdiction and executed the royal decrees
and regulations. He also kept the powerful Zamindars under check.

Q 41.A
• Gol Gumbad
• It is situated in Bijapur in the Bijapur District of Karnataka.
• It is the mausoleum of Muhammad Adil Shah (1626–1656) the seventh Sultan of the Adil Shahi
Dynasty of Bijapur (1489–1686).
• Built by the ruler himself it is a striking edifice in spite of being unfinished. Hence statement 1 is
correct.
• The tomb is a complex of buildings such as a gateway, a Naqqar Khana, a mosque, and a sarai located
within a large-walled garden.
• The Gumbad is a monumental square building topped with a circular drum over which rests a
majestic dome, giving the building its nomenclature. It has an outer wall (which is square in
shape). Hence statement 2 is not correct.
• It is built of dark gray basalt and decorated plasterwork.
• Each wall of the tomb is one hundred and thirty-five feet long and one hundred and ten feet high and ten
feet thick.
• With the drum and the dome, the building rises to a height of over two hundred feet.
• The tomb has only one square chamber and a dome, with a diameter of one hundred and twenty-five
feet.
• The tomb covers an uninterrupted floor space of 18,337 square feet, the largest in the world.

Q 42.A
• Recent context: Microsoft Research India is creating tools to help preserve fast disappearing languages.
• To bring ‘rare’ Indian languages online, Microsoft launched Project ELLORA i.e., Enabling Low
Resource Languages in 2015. Under this project, researchers are building digital resources of the
languages. Hence statement 1 is not correct and statement 2 is correct.
• The purpose is to preserve a language for posterity so that users of these languages “can participate and
interact in the digital world.” The researchers are mapping out resources, including printed literature, to
14 www.visionias.in ©Vision IAS

FREE BY KING R QUEEN P [ऋषभ राजपूत]


create a dataset to train their AI model. The team is also working with these communities on the project.
By involving the community in the data collection process, researchers hope to create a dataset that is
both accurate and culturally relevant.
• It includes rare languages like Gondi, Mundari, Idu Mishmi, etc.
• Marathi, Telugu, Bengali, Malayalam, etc. are popular languages in India.

Q 43.A
• The march of the Magadhan empire during the two centuries preceding the rise of the Mauryas is like the
march of the Iranian empire during the same period.
• The formation of the largest state in India during this period was the work of several enterprising and
ambitious rulers such as Bimbisara, Ajatasatru and Mahapadma Nanda. They employed all means, far and
foul, at their disposal to enlarge their kingdoms and to strengthen their states. But this was not the only
reason for the expansion of Magadha.
• Magadha enjoyed an advantageous geographical position in the age of iron because the richest iron
deposits were situated not far away from Rajgir, the earliest capital of Magadha.
o The ready availability of the rich iron ores in the neighbourhood enabled the Magadhan princes to
equip themselves with effective weapons, which were not easily available to their rivals.
• Magadha enjoyed certain other advantages The two capitals of Magadha, the first at Rajgir and the second
at Pataliputra, were situated at very strategic points. Rajgir was surrounded by a group of five hills, and it
was rendered impregnable in those days.
• Magadha lay at the centre of the middle Gangetic plain. The alluvium, once cleared of the jungles, proved
immensely fertile.
• Magadha enjoyed a special advantage in military organization, Although the Indian states were well
acquainted with the use of horses and chariots, it was Magadha that first used elephants on a large
scale in its wars against its neighbours. The eastern part of the country could supply elephants to
the princes of Magadha, and we learn from Greek sources that the Nandas maintained 6000 elephants.
• The unorthodox character of the Magadhan society. It was inhabited by the Kiratas and Magadhas,
who were held in low esteem by the orthodox Brahmanas.
o But it underwent a happy racial admixture on account of the advent of the Vedic people. Since
it was recently Aryanized it showed more enthusiasm for expansion than the kingdoms which
had been brought under the Vedic influence earlier, On account of all these reasons Magadha
succeeded in defeating the other kingdoms and in founding the first empire in India.
• Hence option (a) is the correct answer.

Q 44.A
• Arabesque is a form of artistic decoration consisting of "surface decorations based on rhythmic linear
patterns of scrolling and interlacing foliage, tendrils" or plain lines, often combined with other elements. It
was extensively used in Indo-Islamic architecture. Hence option (a) is the correct answer.
• The ‘arabesque’ is a French term derived from the Italian word arabesco meaning Arabic style. The
design can be ‘tiled’ or repeated seamlessly to create an endless pattern in the eyes of the viewer. The
arabesques made in the West are based on either ancient Roman ornament or derived from Islamic art.
The patterns are used to mostly decorate architecture and seem to disappear under a framing edge without
an ending.

Q 45.C
• The tallest figure in the sphere of epic poetry is Kalidasa (between A.D. 380-A.D. 415).
• He wrote two great epics, Kumarasambhava (the birth of Kumar), and Raghuvamsa (the dynasty of
the Raghus).
• In the Kavya tradition, more care is bestowed on the form, such as the style, figure of speech, conceits,
descriptions, etc., and the story theme is pushed to the background.
• The overall purpose of such a poem is to bring out the efficacy of a religious and cultured way of life,
without flouting any ethical norms.
• Other distinguished poets, like Bharavi (550 A.D.), wrote Kiratarjuniyam (Kirat and Arjun) and
Magha (65-700 A.D.) wrote Sishupalavadham (the killing of Shishupal).
• There are several other poets like Sriharsha and Bhatti who are of great merit.
• The main purpose of Kavya or even Nataka (drama) is to offer the reader or spectator diversion or
entertainment, (Lokaranjana), and also stimulate his feelings, and ultimately give him a perspective to
illuminate his vision of life.

15 www.visionias.in ©Vision IAS

FREE BY KING R QUEEN P [ऋषभ राजपूत]


• The drama is, therefore, stylized and packed with poetry and descriptive prose. It moves on a level of
worldliness as well as on another level of other-worldliness.
• The rules and prescriptions regarding performance, the theatre hall, acting, gestures, Rasa, and stage
direction, are all given in the first book of dramaturgy, Natyashastra, by Bharata (1st-century B.C.-1st
century A.D.).
• Kalidasa is the most distinguished dramatist and his treatment of the rasa of love in all its possible
manifestations in the three plays Malavikagnimitram (Malavika and Agnimitra), Vikramorvasiyam
(Vikram and Urvasi) and Abhigyana Shakuntala or Abhigyanashakuntalam (the recognition of
Shakuntala) is unparalleled.
• He is the poet of love and beauty, and believes in the affirmation of life, the joy of which lies in pure,
sacred, and ever-widening love.
• Hence, option (c) is the correct answer.

Q 46.B
• The administrative machinery of the Aryans in the Rig Vedic period worked with the tribal chief in the
centre, because of his successful leadership in war, He was called Rajan. It seems that in the Rig Vedic
period, the king's post was hereditary.
o However, the king did not exercise unlimited power, for he had to reckon with the tribal
organizations, Although his post was hereditary, we have also some traces of election by the
tribal assembly called the samiti. The king was called the protector of his tribe. He protected its
cattle, fought its wars and offered prayers to gods on its behalf. Hence statement 1 is correct.
• Several tribal assemblies such as: the sabha, samiti, vidatha, gana are mentioned in the Rig Veda. These
assemblies exercised deliberative, military and religious functions.
o Even women attended the samiti and vidatha in Rig Vedic times, But the two most important
assemblies from the political point of view seem to have been the sabha and the samiti. These
"two were so important that the kings showed’ eagerness to win their support. Hence statement 2 is
not correct.
o In the day-to-day administration, the king was assisted by a few functionaries. The most important
functionary seems to have been the pirohita.
• The Rig Veda does not mention any officer for administering justice. But it was not an ideal society.
There were cases of theft and burglary, and especially we hear of the theft of cows. Spies were employed
to keep an eye on such unsocial activities. Hence statement 3 is correct.
• The officer who enjoyed authority over the pasture ground is called vrajapati.

Q 47.D
• The course of the evolution of Indian music saw the emergence of two different sub-systems as
Hindustani and Carnatic music. In Haripala’s “Sangeeta Sudhakara”, written in the 14th century A.D., the
terms Carnatic and Hindustani are found for the first time. The two distinct styles, Hindustani and
Carnatic came into vogue after the advent of the Muslims, particularly during the reign of the Mughal
Emperors of Delhi.
• Both systems of music received their nourishment from the same original source. Whereas the Indian
music of the Northern part of India assimilated some features of the music of the Persian and Arabic
musicians who adorned the courts of the Mughal rulers of Delhi, the music of the South continued to
develop along its own original lines. But the fundamental aspects of both the systems of the North and
South have been the same. Following are some of the significant forms of Carnatic music:
• GITAM: Gitam is the simplest type of composition. Taught to beginners of music, the gitam is very
simple in construction, with an easy and melodious flow of music. The music of this form is a simple
melodic extension of the raga in which it is composed. The tempo is uniform. It has no sections dividing
one part of the song from the other. It is sung without repetition from the beginning to the end. There are
no intricate variations in the music. Purandaradasa’s introductory gitas in praise of Ganesha, Maheswara
and Vishnu, collectively referred to as Pillari gitas, form the very first set of gitas taught to the students of
music.
• SULADI: Very much like the gitam in musical structure and arrangement, the Suladis are of a higher
standard than the gitam. The Suladi is a talamalika, the sections being in different talas. The sahitya
syllables are fewer than in the gitas and there is a profusion of vowel extensions. The theme is devotional.
Suladis are composed in different tempos vilambita, madhya and druta. Purandaradasa has composed
many Suladis.
• SSVARAJATI: This is learned after a course in gitams. More complicated than the gitas, the Svarajati
paves the way for the learning of the Varnams. It consists of three sections, called Pallavi, Anupallavi and
16 www.visionias.in ©Vision IAS

FREE BY KING R QUEEN P [ऋषभ राजपूत]


Charanam. The theme is either devotional, heroic or amorous. It originated as a dance form with jatis
(tala, solfa syllables like taka tari kita naka tatin gina tam) tagged on.
• JATISVARAM: Very similar to the svarajati in musical structure, this form- Jatisvaram-has no sahitya or
words. The piece is sung with solfa syllables only. It is noted for its rhythmical excellence and the jati
pattern used in it. This is a musical form belonging to the realm of dance music. In some Jatisvarams, the
Pallavi and Anupallavi are sung to jatis and the Charanas are sung to a mixture of svaras and jatis. There
are also Ragamalika Jatisvarams.
• PADA: Padas are scholarly compositions in Telegu and Tamil. Though they are composed mainly as
dance forms, they are also sung in concerts, on account of their musical excellence and aesthetic appeal. A
padam also has the sections, pallavi, anupallavi and charana. The music is slow-moving and dignified.
There is a natural flow of music and, a sustained balance between the words and the music is maintained
throughout. The theme is madhura bhakti, portrayed as bahir sringara and antar bhakti.
• Khyals are musical forms of Hindustani music. The pride of place in classical Hindustani Music is
occupied by the Khyal. As sung today, the khyal has two varieties: the slow or vilambit khyal and
the fast or drut khyal. In form both are similar, they have two sections – the asthayee and the
antara. The vilambit is sung in slow tempo and the drut at a faster speed. In technique. the
exposition is less grave than the dhrupad. There are more delicate gamkas and ornamentations.
Hence, option (d) is the correct answer.

Q 48.D
• Warli paintings are narrative paintings that require performance gestures and movement with little scope
for facial expression. Most of the characters are in dialogue with each other. The art form deals with
themes that narrate the social lifestyle and activities of the Warlis i.e. hunting, fishing, farming,
forest activities and village life. During the harvest season, weddings and births, their houses are adorned
with paintings. The Warlis are predominantly farmers whose life is governed by the monsoon cycle. As
their life is closely linked to nature, they worship it in different forms–Sun and moon, god of thunder,
lightning, wind, rain and several others. Gods are worshipped according to the seasons. Men and women
are depicted engaged in a harvest scene, cultivating land and hunting and many more day-to-day
activities. Spiral formations of men and women and concentric circular designs in Warli Paintings are
symbolic of the Tarpa dance.
• The Warli painting is simplistic in character with human figures made of triangles and stick-like
hands and legs and a variety of flora and fauna in geometrical shapes. Warli paintings are practised
generation after generation and there is no formal training given to the artists.
• They simply paint on mud and cow dung-based surface, coated with geru (red mud) first and then with
rice paste for the color white. These paintings are simplistic but full of life. Salati grass or bamboo sticks
are used in place of a brush for painting.
• Warli is the main tribe to be found on the northern outskirts of Mumbai, in Western India and extends up
to the Gujarat border. The Warlis inhabit small villages of thatched mud huts, which are constructed in
such a way that they all surround a central cell. Hence, option (d) is the correct answer.

Q 49.C
• Cherial scroll paintings belong to Andhra Pradesh. Cherial in the Warangal district is the
traditional center of this art. The traditional caste-based groups from Jingor, Muchi and Mera known as
nakkash, paint these paintings. Such scrolls were used by the story tellers accompanied by the musicians,
along with dolls and masks made of a coconut shell while the larger ones are made out of sawdust and
wood called tella puniki smeared with tamarind seed paste. The common themes are from the Krishna
Leela, Ramayana, Mahabharata, Shiva Puranam, Markandeya Puranam interspersed with the ballads and
folk stories of communities like Gauda, Madiga and so on. Irrespective of the subject matter, Cherial
paintings begin with the painting of Lord Ganesha, a remover of obstacles and then of Saraswati, goddess
of learning. Hence, pair 1 is not correctly matched.
• Madhubani is a village in the district of Mithilanchal in Bihar. The Madhubani painting originated
in the area named ‘Mithilanchal’ where King Janak ruled or in other words the birthplace of
Sita. The origin of this art form can be traced to the epic of Ramayana as Tulsidas gives a vivid account of
Mithilanchal being decorated with Mithila paintings at the wedding of Sita with Ram. These paintings are
drawn on auspicious occasions such as the birth of a child, the sacred-thread ceremony of the Brahmins,
etc. -sathhudi, a ritual observed during the seventh and ninth month of pregnancy; chatiyar puja, done on
the sixth day after the birth of a child; annaprashan, to mark a child’s first eating of grain; akshararambh,
to mark a child’s entry into the world of learning. One of the important themes of Mithila painting is
kohabara. The main area of the house where these paintings are done is – the kohabara ghar– a room
17 www.visionias.in ©Vision IAS

FREE BY KING R QUEEN P [ऋषभ राजपूत]


prepared for a groom where he stays at his in-laws place after the Wedding. Hence, pair 2 is correctly
matched.
• The Patachitra is the traditional painting of Odisha. The word is derived from the Sanskrit word
“pata” which means canvas and “chitra” which means picture. This art style is dedicated to Lord
Jagannath of Puri. Unlike the scroll paintings of Bengal, Rajasthan or South India, the Patachitras of Puri,
Kalighat paintings of Bengal and Chitrakathi paintings of Maharashtra are separate pieces of paintings
complete in themselves. The Patachitras are oblong, stiff and glazed pieces of painting. Painted on cloth, it
follows a traditional process of canvas preparation. It is painted with stone colours, shell powders and
organic lac on paper or cloth stiffened with tamarind seed and chalk powder. Hence, pair 3 is correctly
matched.
• The word ‘Pichhvai’ literally means ‘at the back’. Pichhvai paintings are done on cloth and serve as
painted backdrops for installed icons of Shrinathji at Nathadwara in Rajasthan. There are three
different types of Pichhvais–those made for specific festivals (celebrating the life of Krishna); those made
for specific seasons (monsoon and winter); and those which express general themes not necessarily linked
to Shrinathji. There are pichhvais inspired by the festival of Sharad Purnima Govardhan Dhanna (the
Mount Govardhan pichhvai), and Varsha (the Varsha pichhvai). Pichhvai paintings are done on similar
lines as that of miniature paintings. Hence, pair 4 is correctly matched.
Q 50.D
• Recent context: The Geological Survey of India has for the first-time established Lithium ‘inferred’
resources(G3) of 5.9 million tonnes in the Salal-Haimana area of the UT of Jammu & Kashmir. This
recent discovery of lithium reserves is a significant development that has the potential to revolutionize
various industries, including electric vehicles (EVs) and electronics.
• Lithium is a rare and valuable mineral. Known as the ‘white gold of energy storage technology’, it is
one of the key components used in electric batteries. Besides EV, Lithium-based semiconductors are
also playing a key role in areas of national security, nuclear medicine, and other scientific research.
• Due to a lack of sufficient resources until yet, India has been heavily dependent on imports to meet its
lithium requirements, with more than 80% of its lithium needs imported from countries like China,
Argentina, and Chile.
• The discovery of lithium reserves in India presents an opportunity for the country to reduce its
dependence on imports, become self-reliant in the production of lithium-ion batteries, and position
itself as a major player in the global lithium-ion battery market.
• The current discovery has also placed India in the seventh position in terms of Lithium reserves in
the world.
• Hence option (d) is the correct answer.

Q 51.A
• The Vijayanagara Empire was a South Indian empire that ruled from the 14th to the 17th century, based
in the city of Vijayanagara (now Hampi, Karnataka). The Vijayanagara Empire was founded in 1336
and was attributed to the Sangama dynasty siblings Harihara I and Bukka Raya I, who hailed from
a cowherd community with a Yadava lineage. Hence statement 1 is correct.
• The Vijayanagara Empire, held dominion over a vast area in South India, encompassing present-day
Karnataka, Andhra Pradesh, Tamil Nadu, Kerala, Goa, and portions of Telangana and Maharashtra.
Hence statement 2 is not correct.
• The rising power Vijayanagar empire brought it into a clash with many powers both in the south and
to the north. In the south, its main rivals were the sultans of Madurai. The struggle between the sultan
and Madurai lasted for about four decades. By 1377, the sultana of Madurai had been wiped out. Hence
statement 3 is not correct.

Q 52.A
• Recent Context: US-FDA has approved computer-based and experimental alternatives to animals to
test new drugs. This move is expected to boost the research and development of “organ on a
chip”. Hence statement 3 is correct.
• Organ-on-a-chip is a microfluidic device that aims to mimic the structure and function of specific
human organs or tissues in vitro. It is a multi-channel 3-D microfluidic cell culture, integrated circuit
(chip) that simulates the activities, mechanics, and physiological response of an entire organ or an
organ system. It constitutes the subject matter of significant biomedical engineering research. Hence,
statement 1 is correct.
• It is a small, transparent chip made of biocompatible materials such as silicon, glass, or polymers,
and contains tiny channels lined with living cells. These living cells are derived from human tissues and
18 www.visionias.in ©Vision IAS

FREE BY KING R QUEEN P [ऋषभ राजपूत]


can be cultured to replicate the microenvironment of the specific organ being modeled. Hence, statement
2 not is correct.
• Mechanism- Each organ-on-a-chip contains a complex network of microfluidic channels and chambers
that can simulate the mechanical and chemical environment of a specific organ. It can mimic the flow of
blood and air, while the living cells provide a realistic environment for drug testing and disease
modeling.
• Potential applications-
o It has numerous potential applications, including drug development, disease modeling, and toxicity
testing.
o By replicating the structure and function of human organs, researchers can study how organs
interact with drugs and other compounds.
o This could lead to the development of more effective and personalized treatments for a variety of
diseases.
o It provides a more ethical and effective approach to testing drugs and other compounds, reducing
the reliance on animal testing.

Q 53.A
• Krishnattam, folk theatre of Kerala, came into existence in the middle of 17th century A.D. under
the patronage of King Manavada of Calicut.
• Krishnattam is a cycle of eight plays performed for eight consecutive days. The plays are Avataram,
Kaliamandana, Rasa krida, kamasavadha, Swayamvaram, Bana Yudham, Vivida Vadham, and
Swargarohana.
• The episodes are based on the theme of Lord Krishna – his birth, childhood pranks and various deeds
depicting victory of good over evil.
• Mudiyettu, traditional folk theatre form of Kerala is celebrated in the month of Vrischikam
(November-December).
• It is usually performed only in the Kali temples of Kerala, as an oblation to the Goddess. Hence option
(a) is the correct answer.
• It depicts the triumph of goddess Bhadrakali over the asura Darika. The seven characters in
Mudiyettu-Shiva, Narada, Darika, Danavendra, Bhadrakali, Kooli and Koimbidar (Nandikeshvara) are all
heavily made-up.

Q 54.B
• Yellowstone National Park is widely considered to be the first national park in the world. It is the first
national park in the US that set forth the process of protecting places for their intrinsic and recreational
value. Hence, statement 1 is correct.
• It is located in the northwest corner of Wyoming and extends into Montana and Idaho. The Gardner
River originates in the northwest corner of the park and flows to the Missouri River. The Gardner
flows into Yellowstone first, joining near Rattlesnake Butte at the north entrance to the park. Hence,
statement 2 is not correct.

19 www.visionias.in ©Vision IAS

FREE BY KING R QUEEN P [ऋषभ राजपूत]


• it was established by the 42nd United States Congress with the Yellowstone National Park Protection
Act signed into law on March 1, 1872. It spans an area of over 9,000 sq. km comprising lakes, canyons,
rivers, iconic geothermal features such as the Old Faithful geyser, and mountain ranges.
• Over the years, it has been at the center of many successful conservation endeavors, and today is the
most famous megafauna location in the contiguous United States, home to grizzly bears, wolves, and
free-ranging herds of the endangered bison and elk. Hence, statement 3 is correct.

Q 55.B
• The first Mughal invasion of Deccan happened during the reign of Akbar (reign 1556-1605). In 1591,
Akbar launched a diplomatic offensive. He sent embassies to all the Deccani states ‘inviting’ them to
accept Mughal suzerainty. However, only Khandesh accepted this proposal and rest others did not pay
heed.
• The instability in Ahmednagar gave an opportunity for the Mughals to intervene in the Deccan. The
Mughal invasion was led by prince Murad, who was the governor of Gujarat, and by Abdur Rahim Khan-
i-Khanan.
• The fight against invading Mughal forces was led by Chand Bibi. However, the factionalism in the
Deccani forces led to their defeat. A treaty was signed in 1596 by the Mughals got Berar and the Mughal
suzerainty was also recognised by Ahmednagar. The Mughals in return for their recognition of the claim
of Bahadur. Hence, statement 1 is correct.
• The Coup D’etat of Mahabat Khan occurred during the reign of Jahangir in 1626. Mahabat Khan
was one of the most capable commanders of Jahangir. He was loyal to the throne and but he was one of
those nobles who did not like the increased influence of Nur Jahan in the administration of the state.
• In 1626, Mahabat Khan came with a trusted body of Rajputs and seized the emperor at an opportune
moment when the royal camp was crossing the river Jhelum on its way to Kabul. Nur Jahan after a failed
attempt to defeat Mahabat Khan surrendered herself in order to be close to Jahangir. However, later she
was able to wean away most of the nobles from Mahabat Khan’s side. Realising his precarious position,
Mahabat Khan abandoned Jahangir and fled from the court. Hence, statement 2 is correct.
• The Mughal emperor Shah Jahan (1628 –1658) annexed the state of Ahmednagar in 1636 and a
treaty was concluded with Bijapur. According to the treaty, the Nizam Shahi state came to an end. The
territory of Ahmednagar was divided between the Mughals and Bijapur; the territory lying in the north of
the Bhima river went to the Mughals while the area lying in the south to Adil Shah.
• Aurangzeb became the Mughal ruler in 1658, twenty years later when the state of Ahmadnagar came to
an end. Hence, statement 3 is not correct.

Q 56.D
• Mauryan state controlled almost all the economic activities in the realm The state brought new land under
cultivation with the help of cultivators and sudra labourers.
• The virgin land which was opened to cultivation yielded handsome income to the state in the form of
revenue collected from the newly settled peasants.
• It seems that taxes collected from the peasants varied from one-fourth to one-sixth of the produce. Those
who were provided with irrigation facilities by the state had to pay for it.
• In addition to this in times of emergency peasants were compelled to raise more crops. Tolls were levied
on commodities brought to town for sale, and they were collected at the gates.
• Moreover, the state enjoyed a monopoly mining, sale of liquor, manufacture of arms, etc. This
naturally brought money to the royal exchequer.
• Hence option (d) is the correct answer.

Q 57.B
• During the post-Mauryan period, foreign princes patronized and cultivated Sanskrit literature. The earliest
specimen of kavya style is found in the inscription of Rudradaman in Kathiawar in about A D. 150. From
then onwards inscriptions began to be composed in chaste Sanskrit, although the use of Prakrit in
composing inscriptions continued till the fourth century A.D and even later.
• It seems that some of the great creative writers such as Asvaghosha enjoyed the patronage of the
Kushans. Asvaghosha wrote the Buddhacharita, which is a biography of the Buddha, He also
composed Saundarananda, which is a fine example of Sanskrit kavya.
• Hence option (b) is the correct answer.

20 www.visionias.in ©Vision IAS

FREE BY KING R QUEEN P [ऋषभ राजपूत]


Q 58.D
• Recent context: Budget has announced computerization of over 63000 PACS. A Primary
Agricultural Credit Society (PACS) is a basic unit and the smallest cooperative credit institution in
India. It works on the grassroots level (gram panchayat and village level).
• PACSs serve as the last link in a three-tier cooperative credit structure headed by the State
Cooperative Banks (SCB) at the state level. Hence statement 2 is correct.
• These provide short-term, and medium-term agricultural loans to the farmers for the various
agricultural and farming activities for the period of the crop cycle. Credit from the SCBs is transferred
to the district cooperative central banks, or DCCBs, that operate at the district level. The DCCBs
work with PACSs, which deal directly with farmers. At the start of the cropping cycle, farmers avail
credit to finance their requirement of seeds, fertilizers etc. Banks extend this credit at 7% interest, of
which 3% is subsidized by the Centre and 2% by the state government. Effectively, farmers avail the
crop loans at 2% interest only.
• Hence, statement 1 is correct.
• Since these are cooperative bodies, individual farmers are members of the PACS, and office-bearers
are elected from within them.
• A village can have multiple PACS, which are regulated by the RBI and governed by Banking
Regulations Act, of 1949 and the Banking Laws (Co-operative Societies) Act of 1955. Hence
statement 3 is correct.
• A report published by the Reserve Bank of India in 2022 put the number of PACS at 1.02 lakh.

Q 59.C
• India had a unique system of communication during the fourteenth century, Ibn Battuta in his
book ‘Rihla’ explained the unique system of communication of India which impressed him very much.
• In India, the postal system was of two kinds. The horse post, called uluq, is run by royal horses
stationed at a distance of every four miles. The foot-post has three stations per mile; it is called dawa.
• At every third of a mile, there is a well-populated village, outside which are three pavilions in which men
sit with girded loins ready to start. This foot post is quicker than the horse post, and often it is used to
transport the fruits of Khurasan which are much desired in India. Also, this postal system allowed
merchants to not only send information and remit credit across long distances but also to dispatch
goods required at short notice.
• Hence option (c) is the correct answer.

Q 60.C
• Indian sculptors had mastered the bronze medium and the casting process as much as they had
mastered terracotta sculpture and carving in stone. The cire-perdu or ‘lost-wax’ process for casting
was learned as long ago as the Indus Valley Culture. Along with it was discovered the process of
making an alloy of metals by mixing copper, zinc and tin which is called bronze. Hence, statement 2
is not correct.
• Vakataka bronze images of the Buddha from Phophnar, Maharashtra, are contemporary with the
Gupta period bronzes. They show the influence of the Amaravati style of Andhra Pradesh in the
third century CE and at the same time there is a significant change in the draping style of the monk’s
robe. Buddha’s right hand in abhaya mudra is free so that the drapery clings to the right side of the body
contour. The result is a continuous flowing line on this side of the figure. At the level of the ankles of the
Buddha figure the drapery makes a conspicuous curvilinear turn, as it is held by the left hand. Hence,
statement 1 is not correct.
• The additional importance of the Gupta and Vakataka bronzes is that they were portable and
monks carried them from place to place for the purpose of individual worship or to be installed in
Buddhist viharas. In this manner, the refined classical style spread to different parts of India and to
Asian countries overseas. Hence, statement 3 is correct.

Q 61.C
• The hybridisation and incorporation of several styles was the hallmark of Chalukyan buildings. The
most elaborate of all Chalukyan temples at Pattadakal made in the reign of Vikramaditya II (733-
44) by his chief queen Loka Mahadevi, for instance, shows complete knowledge of Pallava buildings at
Kanchipuram and as a corollary, Mahabalipuram. The temple is one of the best early examples of the
Dravida tradition. Virupaksha Temple is the oldest known temple built by a queen in India.
• The Lakshmana temple dedicated to Vishnu is the grandest temple of Khajuraho, built in 954 by
the Chandela king, Dhanga. A Nagara temple is placed on a high platform accessed by stairs. There are
21 www.visionias.in ©Vision IAS

FREE BY KING R QUEEN P [ऋषभ राजपूत]


four smaller temples in the corners, and all the towers or shikharas rise high, upward in a curved
pyramidal fashion, emphasizing the temple’s vertical thrust ending in a horizontal fluted disc called an
amalak topped with a kalash or vase. The crowning elements: amalak and kalash, are to be found on all
nagara temples of this period. The temple also has projecting balconies and verandahs.
• At Konark, on the shores of the Bay of Bengal, lie the majestic ruins of the Surya or Sun temple
built in stone around 1240. Its shikhara was a colossal creation said to have reached 70m, which,
proving too heavy for its site, fell in the nineteenth century. The vast complex is within a quadrilateral
precinct of which the jagamohana or the dance-pavillion (mandapa) has survived, which though no longer
accessible is said to be the largest enclosed space in Hindu architecture. The Sun temple is set on a high
base, its walls covered in extensive, detailed ornamental carving. These include twelve pairs of enormous
wheels sculpted with spokes and hubs, representing the chariot wheels of the Sun god who, in mythology,
rides a chariot driven by eight horses, sculpted here at the entrance staircase. The whole temple thus
comes to resemble a colossal processional chariot.
• The Sun temple at Modhera dates back to the early eleventh century and was built by Raja
Bhimdev I of the Solanki Dynasty in 1026. The Solankis were a branch of the later Chalukyas. There is
a massive rectangular stepped tank called the surya kund in front of it. The proximity of sacred
architecture to a water body such as a tank, a river or a pond has been noticed right from the earliest times.
By the early eleventh century they had become a part of many temples. This hundred-square-meter
rectangular pond is perhaps the grandest temple tank in India. A hundred and eight miniature shrines are
carved in between the steps inside the tank. A huge ornamental arch-torana leads one to the sabha
mandapa (the assembly hall) which is open on all sides, as was the fashion of the times in western and
central Indian temples. Hence, option (c) is the correct answer.

Q 62.A
• Bharatanatyam of Tamil Nadu in southern India has grown out of the art of dancers dedicated to
temples and was earlier known as Sadir or Dasi Attam. It is the first of India's traditional dances to
be refashioned as theatre art and to be exhibited widely both at home and abroad. Hence option (a)
is the correct answer.
• Bharatanatyam rests on principles of performance and aesthetics set down in classics such as Bharata's
Natyashastra. It has a rich repertoire of songs in Telugu, Tamil and Sanskrit. The present-day format of
a Bharatanatyam recital, as well as a valuable part of its musical compositions, was created by the
famed ‘Tanjore Quartet’ of the nineteenth century: the brothers Ponniah, Chinnaiah, Sivanandam
and Vadivelu.
• Alarippu is the opening item which is a nritta item. Alarippu can be sometimes preceded by an item
called Todayamangalam or Pushpaanjali. Todayamangalam is quite often than not followed by an
item called "Kautavam".
• Tillaanaa is a pure nritta item performed as the concluding offering in a Bharata Natyam recital.
• Apart from these, the repertoire of Bharatanatyam consists of the following:
o Jatisvaram: This again is a nritta item. It is a combination of jatis (string of adavus) danced to the
svaras of a raaga.
o Shabdam: A nritya item that makes use of the saattvika abhinaya for the first time in a recital. It is
addressed to a deity and expresses spiritual love.
o Varnam:An intricate and complex item that has a judicious blend of both nritta and nritya. It is usually
addressed to a deity wherein bhakti for the Lord is expressed in terms of endearment. It has both jatis
as well as svaras of the raaga employed. These are interspersed with lyrics as saahitya.
o Jaavali : Yet another variety of nritya composition which is usually secular in character. It usually
represents the love of a naayikaa (heroine) for the naayaka (hero).

Q 63.C
• Recent context: The World Bank has released "Women, Business and the Law 2023 report".
• Women, Business and the Law 2023 is the 9th in a series of annual reports that analyze laws and
regulations affecting women’s economic opportunity in 190 economies. The data is available for the
period from 1971 to 2023
• It has eight indicators- Mobility, Workplace, Pay, Marriage, Parenthood, Entrepreneurship, Assets,
and Pension.
• The Data can be used to build evidence of the relationship between legal gender equality and women’s
entrepreneurship and employment.
• Findings-
o Globally:
22 www.visionias.in ©Vision IAS

FREE BY KING R QUEEN P [ऋषभ राजपूत]


✓ Only 14 scored a perfect 100: Belgium, Canada, Denmark, France, Germany, Greece, Iceland,
Ireland, Latvia, Luxemburg, The Netherlands, Portugal, Spain and Sweden.
✓ In 2022, the global average score is 76.5 out of 100.
✓ At the current pace of reform, it would take at least 50 years to approach legal gender
equality everywhere.
o India:
✓ India has a score is 74.4 out of 100.
✓ This score is above the regional average observed across South Asia (63.7). Within this region,
the maximum score observed is 80.6 (Nepal).
✓ The report used data on laws and regulations in Mumbai.
✓ India received a perfect score for laws related to freedom of movement, women's work
decisions, and marriage constraints.
✓ In India, a thriving civil society also contributed to identifying gaps, drafting legislation, and
organizing public opinion through campaigns, discussions, and protests, leading to the enactment
of the 2005 Domestic Violence Act.
• More: World Bank Index on the Life Cycle of Working Women- It is a tool developed by the World
Bank to measure and track the progress of women’s economic participation and opportunities over
their lifetimes. It is based on a set of indicators that measure factors such as laws and regulations
affecting women’s employment, access to finance, and gender-based violence and harassment in the
workplace.
o The purpose of the index is to provide policymakers and stakeholders with data and insights that can
be used to inform policies and programs aimed at improving women’s economic opportunities and
outcomes.
o India has scored 74.4 out of 100 on the World Bank Index on the life cycle of working
women. This score places India at 140th out of 190 countries surveyed in the index.
• Hence option (c) is the correct answer.

Q 64.D
• Saint Shankaradeva emerged as one of the leading proponents of Vaishnavism in Assam in the late
fifteenth century. His teachings, often known as the Bhagavati dharma because they were based on
the Bhagavad Gita and the Bhagavata Purana, focused on absolute surrender to the supreme deity, in
this case, Vishnu.
• He emphasized the need for naam kirtan, recitation of the names of the lord in sat sanga or
congregations of pious devotees. He also encouraged the establishment of satra or monasteries for the
transmission of spiritual knowledge, and naam ghar or prayer halls. Many of these institutions and
practices continue to flourish in the region. His major compositions include the Kirtana-ghosha. Hence,
option (d) is the correct answer.

Q 65.B
• Ala-Ud-Din Hasan Bahman Shah also known as Hasan Gangu was the founder of the Bahmani
Sultanate. He reigned over Sultanate from 1347 to 1358. The capital of the Bahmani dynasty was
Gulbarga (Hasanbad). Hasan Gangu began his career as a general serving under Sultan Mohammed Bin
Tuglaq. He was made a governor in Daultabad. After establishing his kingdom, he handed it over his
kingdom to his eldest son.
• Alaudin Hasan was an Afghan adventurer.
• The Bahamani kingdom was founded in 1347 by Ala-ud-Din Bahman Shah or Alauddin Hasan.
• He had risen in the service of a brahmana, named Ganga, and is therefore known as Hasan Gangu.
After his coronation, he assumed the title of Alaudin Hasan Bahman Shah.
• Hence option (b) is the correct answer.

Q 66.D
• The Tungabhadra Doab is a region located in southern India, specifically in the states of Karnataka
and Andhra Pradesh. It is the area between the Tungabhadra and Krishna rivers. During the medieval
period in India, the Tungabhadra Doab region played a significant role in the history and politics of the
Deccan Plateau. The area was ruled by various dynasties, including the Chalukyas, the Rashtrakutas, the
Hoysalas, and the Vijayanagara Empire. Hence option (d) is the correct answer.
• From 992 CE to 1120 CE, the Chola Empire and the Western Chalukya Empire engaged in a series of
battles, now known as the Chola-Chalukya Wars, in the southern region of India. The battles were
fought on two fronts: the Western Front, where the capture of Manyakheta and Kalyani were the
23 www.visionias.in ©Vision IAS

FREE BY KING R QUEEN P [ऋषभ राजपूत]


Cholas' objectives, and the Eastern Front, which revolved around the strategic location of Vengi,
essential to both sides.
• There were several instances of conflict between the Yadavas and the Hoysalas over control of the
Tungabhadra Doab region.
• The clashes between the two powers Vijayanagar and Bahmani over the control of the Tungabhadra
Doab region were frequent and intense. The Bahmani Sultanate launched several campaigns against the
Vijayanagara Empire, seeking to gain control of the region. However, the Vijayanagara Empire was able
to repel these attacks and establish its dominance over the region.
• Kakatiya dynasty ruled over the eastern Deccan region and the Gajpati dynasty ruled over present-
day Odisha and North coastal Andhra, so there was no direct fight between the two empires to control
Tungbhadra doab.

Q 67.A
• The Ajanta and Ellora caves are the finest examples of cave architecture in Ancient India. While Ajanta is
mostly about beautiful paintings made on cave walls on the theme of Buddhism, Ellora is all about
sculpture and architecture belonging to three different religions prevailing in the country during those
times—Buddhism, Hinduism and Jainism.
• Ajanta is a cluster of 30 caves of different sizes excavated in a horseshoe-shaped stretch of rock
embedded in a hill facing a narrow stream called Waghora. Each cave was connected to the stream
by a flight of steps, which are now demolished with few remnants left behind. The caves were built
in two phases—starting around the 2nd century BC, with the second group of caves built around
400–650 AD. While Ajanta has two-storeyed caves, three-storeyed caves in Ellora is a unique
achievement. Hence, statement 1 is correct.
• Both Ajanta and Ellora have elaborate Buddhist paintings and structures. At Ajanta, The themes of the
paintings are the events from the life of the Buddha, the Jatakas and the Avadanas. Some paintings such as
Simhala Avadana, Mahajanaka Jataka and Vidhurpundita Jataka cover the entire wall of the cave. It is
worth noting that Chaddanta Jataka has been painted in the early Cave No. 10 with many details and
events grouped according to their geographical locations. Events that happened in the jungle and events
that happened in the palace are separated by their locations. The best-preserved images of Padmapani and
Vajrapani are in cave no. 1 of Ajanta.
• Ellora is also unique in terms of stylistic eclecticism, i.e., the confluence of many styles at one place.
The caves of Ellora and Aurangabad show the ongoing differences between the two religions—
Buddhism and Brahmanical. There are twelve Buddhist caves having many images belonging to
Vajrayana Buddhism like Tara, Mahamayuri, Akshobhya, Avalokiteshwara, Maitreya, Amitabha,
etc. Buddhist caves are big in size and are of single, double and triple storeys. Their pillars are
massive. The shrine Buddha images are big in size; they are generally guarded by the images of
Padmapani and Vajrapani. Hence, statement 2 is not correct.

Q 68.B
• The Maratha administration under King Shivaji was organized into eight councils of ministers known as
Ashtapradhan.
• Peshwa was one most important ministers of the Ashtapradhan. Peshwas looked after the finances and
general administration. The Sar-i-naubat (Senapati) was the commander-in-chief of the army. was a
post of honor and was generally given to one of the leading Maratha chiefs. Hence, pair 1 is not
correctly matched.
• Other important members of the Ashtapradhan were the Majumdar, Surunavis or Chitnis,Dabir,
Nyayadhish and Panditrao, etc.
• The Majumdar was the accountant, while the Wakenavis was responsible for intelligence, posts and
household affairs.
• The Surunavis or Chitnis helped the king with his correspondence.
• Senapati was the Commander-in-Chief.
• The Dabir was the master of ceremonies and also helped the king in his dealings with foreign
powers. The Nyayadhish and Panditrao were in charge of justice and charitable grants. Hence, pair 2 is
correctly matched.
• The Mirasdars were hereditary holders of rights in land in the Maratha kingdom. They belong to the
affluent strata of rural society.
• The ‘silahdars’ were the loose auxiliaries in the army Maratha army placed under havaldars who
received fixed salaries. Hence, pair 3 is not correctly matched.

24 www.visionias.in ©Vision IAS

FREE BY KING R QUEEN P [ऋषभ राजपूत]


Q 69.D
• The Tughlaq dynasty was a Muslim royal family of Turko-Mongol or Turkic descent that held sway
over the Delhi Sultanate during medieval India. The dynasty came into power in Delhi in 1320 CE Their
reign continued until 1412 CE when the dynasty came to an end. The Tughlaqs provided three competent
rulers: Ghiyasuddin, Muhammad bin Tughlaq, and Firuz Shah Tughlaq.
• Muhammad bin Tughlaq transferred the capital from Delhi to Deogir. As Deogir had been a base for
the expansion of Turkish rule in South India. The attempt to bring entire South India under control had led
to serious political difficulties. Later Deogir was renamed to Daulatabad. Soon he decided to abandon
Daulatabad. Hence statement 1 is not correct.
• Muhammad bin Tughlaq introduced token currency in order to avoid dependency upon the supply of
gold, and silver. There was a shortage of silver in the world in the 14th century. Later people started
forging the coins, this experiment turned out to be a failure. Hence statement 2 is not correct.
• Firuz Shah Tughlaq extended the principle of heredity to the army. Old soldiers were allowed to rest
in peace and to send in their place their sons or sons-in-law, and if they were not available, their slaves.
Hence statement 3 is correct.
• During the time of Firuz Shah Tughlaq, Jizyah became a separate tax. Earlier it was a part of jizyah
since this was not provided for in the sharia. Hence statement 4 is correct.

Q 70.B
• The Chola kingdom was called Cholamandalam or Coromandal. It was situated to the north-east of the
territory of the Pandyas, between the Pennar and the Velur rivers. We have some idea of the political
history of the Cholas, whose chief centre of political power lay at Uraiyur, a place famous for cotton
trade.
o It seems that in the middle of the second century B.C. a Chola king named Elara conquered Sr1 Lanka
and ruled over it for nearly 50 years.
o A firmer history of the Cholas begins in the second century A.D. with their famous king
Karikala, who flourished around A.D. 100. He founded Puhar and constructed 160 km of
embankment along the Kaveri river. Hence pair 1 is not correctly matched.
o This was built with the labour of 12,000 slaves who were brought as captives from Sri Lanka. Puhar is
identical with Kaveripattanam, which was the” Chola capital. It was a gréat centre of trade and
commerce, and excavations show that it had a large dock.
• The Chera or the Kerala country was situated to the west and north of the land of the Pandyas. It
included the narrow strip of land between the sea and the mountains and covered a portion of modern
Kerala state.
o In the early centuries of the Christian era, the Chera country was as important as the country of the
Cholas and the Pandyas. It owed its importance to trade with the Romans. The Romans set up two
regiments at Muziris identical with Cranganore tn the Chera country to protect their interests. It is said
that they also built there a temple of Augustus.
o According to the Chera poets their greatest king was Senguttuvan, the Red Chera, He routed his
rivals and established his cousin securely on the throne. Hence pair 2 is not correctly matched.
• The Chalukyas set up their kingdom towards the beginning of the sixth century A.D. in the western
Deccan. They established their capital at Vatapi, modern Badami, in the district of Bijapur which forms a
part of Karnataka.
o Later they branched off into several independent ruling houses, but the main branch continued to rule
at Vatapi for two centuries. In this period no other power in the Deccan was so important as the
Chalukyas of Badami until we come to Vijayanagara in late medieval times.
o Pulakesin II (609- 642) is the most famous Chalukya king. He is known to us from his eulogy
written by the court poet Ravikirti in the Aihole inscription. This inscription is an example of poetic
excellence reached in Sanskrit, and in spite of its exaggeration is a valuable source for the biography
of Pulakesin. Hence pair 3 is correctly matched.
o Pulakesin overthrew the Kadamba capital at Banavasi and compelled the Gangas of Mysore to
acknowledge his suzerainty. He also defeated Harsha’s army on the Narmada and checked his
advance towards the Deccan.
o In his conflict with the Pallavas, he almost reached the Pallava capital, but the Pallavas purchased
peace by ceding their northern provinces to Pulakesin II.
Q 71.D
• Kuchipudi derives its name from the village of Kuchelapuram, in Andhra Pradesh, India. As a
classical form of dance, drama and music Kuchipudi enjoys a unique place among the Indian
classical idioms. Kuchipudi grew largely as a product of the Bhakti movement beginning in the
25 www.visionias.in ©Vision IAS

FREE BY KING R QUEEN P [ऋषभ राजपूत]


seventh century A.D. However, it was in the 14th century that the ascetic Siddhendra Yogi
appeared on the scene and gave Kuchipudi a new definition and direction.
• Kuchipudi was originally a male dance tradition. Groups of men traveled from village to village
enacting stories from Hindu mythology. As in Elizabethan theatre men portrayed the roles of
women. It was only in the last 9 or 10 decades that women were introduced to this art form.
Kuchipudi in its present form is the result of the vision of stalwarts like Vempati Chinna Satyam
and the late Vedantam Laxminarayana Shastry. It consists of both male and female performers.
• A recital of Kuchipudi begins with an invocatory number, as is done in some other classical dance styles.
Earlier the invocation was limited to Ganesha Vandana. Now, other gods are also invoked. It is followed
by nritta, that is, non-narrative and abstract dancing. Usually, jatiswaram is performed as the nritta
number. Next is presented a narrative number called shabdam. The Shabdam is followed by a natya
number called Kalaapam. Many Kuchipudi dancers prefer to perform entry of Satyabhama from the
traditional dance-drama Bhaamaakalaapam. Next in the sequence comes a pure nritya abhinaya number
based on literary-cum musical forms like padam, jaavli, shlokam, etc. In such a number each of the sung
words is delineated in space through dance, drishya-kavita (visual poetry). A Kuchipudi recital is
usually concluded with tarangam. Excerpts of Krishna-leela-tarangini are sung with this number.
In this, the dancer usually stands on a brass plate locking the feet in shakatavadanam paada and
moves the plate rhythmically with great dexterity.
o Rukmini Devi Arundale was Bharatnatiyyam dancer. Hence, option (d) is the correct answer.
• The music that accompanies the dance is according to the classical school of Carnatic music and is
delightfully syncopatic. The accompanying musicians, besides the vocalist, are: a mridangam player
to provide percussion music, a violin or veena player or both for providing instrumental melodic
music, and a cymbal player who usually conducts the orchestra and recites the sollukattus
(mnemonic rhythm syllables).

Q 72.D
• The Treaty of Purandar was signed in June 1665, between the Maratha King Shivaji and Raja Jai
Singh I representing Aurangzeb. Hence, statement 1 is correct.
• The terms of the treaty were:
o Out of 35 forts held by Shivaji, 23 forts with the surrounding territory which yielded a revenue of
four lakhs of huns every year were to be surrendered to the Mughals, while the remaining 12 forts
with an annual income of one lakh of huns were to be left to Shivaji ‘on condition -of service and
loyalty to the throne’. Hence, statement 2 is correct.
o Territory worth four lakhs of huns a year in the Bijapuri Konkan, which Shivaji had already held, was
granted to him. In addition, Bijapur territory worth five lakhs of huns a year in the uplands
(Balaghat), which Shivaji was to conquer, was also granted to him. In return for these, he was to
pay 40 lakhs huns in installments to the Mughals. Hence, statement 3 is correct.
o Shivaji asked to be excused from personal service. Hence, a mansab of 5000 was granted in his place
to his minor son, Sambhaji.

Q 73.A
• King Shivaji supplemented his income by levying a contribution on the neighboring Mughal
territories. This contribution which came to one-fourth of the land revenue began to be called chauthai
(one- fourth) or chauth.
• Chauth was a levy imposed on the territories not directly controlled by the Marathas. It was a levy of
twenty-five percent paid to the Marathas by the adjoining/outside rulers to the Maratha kingdom in order
to avoid the Maratha raids. Hence, statement 1 is correct.
• Sardeshmukhi was imposed by Shivaji in his own dominion (swaraj) on the basis of his claim as
hereditary sardeshmukh (landlord) of the dominion. Thus, it was claimed by Shivaji as a matter of
right, unlike Chauth. It was 10 percent of the total revenue realized. Thus, it was a tax on landlords
and not peasants. Hence, statement 2 is not correct.

Q 74.B
• The rock-cut cave tradition also existed in Odisha. The earliest examples are the Udaigiri-
Khandagiri caves in the vicinity of Bhubaneswar. These caves are scattered and have inscriptions of
Kharavela kings. Hence, statement 1 is not correct.
• According to the inscriptions, the caves were meant for Jain monks. There are numerous single-cell
excavations. Some have been carved in huge independent boulders and given the shape of animals.

26 www.visionias.in ©Vision IAS

FREE BY KING R QUEEN P [ऋषभ राजपूत]


The big caves include a cave with a pillared veranda with cells at the back. Hence, statement 2 is
correct.
• The upper part of the cells is decorated with a series of chaitya arches and narratives that still continue in
the folklores of the region. The figures in this cave are voluminous, move freely in the picture space, and
are an excellent example of qualitative carving. Some caves in this complex were excavated later,
sometime in the eighth–ninth centuries CE.

Q 75.B
• Recent Context: A recent initiative called the ‘Anmol Jeevan Abhiyan’ (Precious Life Campaign)
in Barmer, Rajasthan has motivated village panchayats and homeowners to add hand pumps and
locked covers to tankas (associated with local deities) or sacred forest groves 10 km away from the
village.
• The lightweight hand pumps made of fiber serve a dual purpose-
o Preventing accidents and suicides (171 suicide cases reported last year)
o Drawing water from the tank.
• The campaign has been started jointly by the district administration, United Nations Children’s Fund
(UNICEF), and Action Aid.
• Though the campaign has made an impact during the last three to four months, it cannot be measured in
quantitative terms at present because of its continuity, even as the reports of suicides have gradually
reduced.
• Tanka- It is a traditional rainwater harvesting technique indigenous to the Thar desert region of
Rajasthan. A Taanka is a cylindrical paved underground pit into which rainwater from rooftops,
courtyards or artificially prepared catchments flows.
• Other traditional water conservation systems in India include- Jhalaras, Bawari, Ahar Pynes, etc.
• Hence option (b) is the correct answer.

Q 76.C
• Land grants to the brahmanas on a large scale suggest that the brahmana supremacy continued in Gupta
times. The Guptas who were originally vaisyas came to be looked upon as kshatriyas by the brahmanas.
The brahmanas represented the Gupta kings as possessing the attributes of gods, and the Gupta princes
became great supporters of the brahmanrical order.
• The castes proliferated into numerous subcastes as a result of two factors (there was no
consolidation of castes). On the one hand a large number of foreigners had been assimilated into the
Indian society, and each group of foreigners was considered a kind of Hindu caste. Hence statement 2 is
correct.
o Since the foreigners mainly came as conquerors they were given the status of the kshatriya in society.
The Hunas, who appeared in India towards the close of the fifth century, ultimately came to be
recognized as one of the thirty-six clans of the Rajputs.
• The other reason for the increase in the number of castes was the absorption of many tribal peqples into
brahmanical society by way of land grants. To the ruling chiefs of the tribes was ascribed a respectable
origin. But most of the rest of the tribal people were given a low origin, and every tribe now became a
kind of caste in Hindu society.
• In some ways. the position of sudras and women improved in this period. They were now permitted
to listen to the epics and the Puranas, They could also worship a new god called Krishna.
o From the seventh century onwards the sudras were mainly represented as agriculturists; in the earlier
period they always appeared as servants, slaves and agricultural labourers working for the three higher
varnas.
• But during this period the untouchables increased in number, especially the chandalas. The
chandalas appeared in society as early as the fifth century B.C. Hence statement 1 is correct.
o By the fifth century A.D their number had become so enormous and their disabilities so glaring that it
attracted the attention of the Chinese pilgrim Fa-hsien. He informs us that the chandalas lived outside
the village and dealt in meat and flesh.

Q 77.C
• The Guptas with their seat of power in Uttar Pradesh and Bihar ruled over north and western India for
about 160 years, till the middle of the sixth century A D. Then north India again split up into several
kingdoms. The white Hunas established their supremacy over Kashmir, Punjab and western India from
about A.D. 500 onwards. North and western India passed under the control of about half a dozen
feudatories who parcelled out Gupta empire among themselves.
27 www.visionias.in ©Vision IAS

FREE BY KING R QUEEN P [ऋषभ राजपूत]


o Gradually one of these dynasties ruling at Thanesar in Haryana extended its authority over all the
other feudatories. The ruler who brought it about was Harshavardhana (A.D. 606-647).
• Harsha made Kanauj the seat of his power and there from extended his authority in all directions.
During this period Pataliputra fell on bad days and Kanauj came to the forefront. Hence statement 1 is
correct.
• The early history of Harsha’s reign is reconstructed from a study of Banabhatta, who was his court poet
and who wrote a book called - Harshacharita. This can be supplemented by the account of the Chinese
pilgrim Hsuan Tsang, who visited India in the seventh century “A.D. and stayed in the country for about
15 years.
• Harsha governed his empire on the same lines as the Guptas did, except that his administration had
become more feudal and decentralized.
• Land grants continued to be made to priests for special services rendered to the state. In addition,
Harsha is credited with the grant of land to the officers by charters. These grants allowed the same
concessions to priests as were allowed by the earlier grants. Hence statement 2 is not correct.
• The feudal practice of rewarding and paying officers with grants of land seems to have begun under
Harsha. This is natural because we do not have many coins issued by Harsha.
• In the empire of Harsha, law and order was not well maintained. The Chinese pilgrim Hsuan
Tsang, about whom special care may have been taken by the government, was robbed of his
belongings, although he reports that according to the laws of the land severe punishments were inflicted
for the crime. Hence statement 3 is correct.

Q 78.B
• Mimamsa literally means the art of reasoning and interpretation. But reasoning was used to provide
justifications for various Vedic rituals, and the attainment of salvation was made dependent on their
performance.
• According to the Mimamsa, the Vedas contain the eternal truth. The principal object of this
philosophy was to acquire heaven and salvation.
• A person will enjoy the bliss of heaven as long as his accumulated acts of virtue last. When his
accumulated virtues are exhausted, he will come back to the earth.
• But if he attains salvation, he will be completely free from the cycle of birth and death in the world. In
order to attain salvation the Mimamsa strongly recommended the performance of Vedic sacrifices,
which needed the services of the priests and legitimised the social distance between varlous varnas.
• Through the propagation of the Mimamsa the brahmanas wanted to maintain their ritual authority and
preserve the social hierarchy based on brahmanism.
• Hence option (b) is the correct answer.

Q 79.C
• Alaudin Khalji (1296-1316) introduced market policy, which was a groundbreaking and distinctive
measure that garnered the interest of both domestic and international historians.
• During the early 14th century, Alauddin Khalji implemented a series of reforms that included price
controls throughout his empire. This involved setting fixed prices for a wide range of commodities,
such as grains, cloth, slaves, and animals. Hence statement 1 is correct.
• Alauddin also prohibited hoarding and regrating, employed overseers and spies to enforce the regulations,
and imposed harsh penalties on those who disobeyed.
• According to Barani, Alauddin set up three markets in Delhi, first for food grains, the second for
cloth and expensive items such as sugar, ghee, oil, dry fruits, etc., and the third for horses, slaves, and
cattle. Hence statement 2 is not correct.
• Grains were stored in government warehouses. The storage was meant for emergencies like times of
scarcity and famine. Any trader or vendor who cheated in weights and measurements was punished with
cutting an equal weight of flesh from his body. Hence statement 3 is correct.

Q 80.A
• Recent context: Windfall tax on crude oil slashed by the Indian government.
• A windfall tax refers to the tax levied against certain industries by the government when economic
conditions permit those industries to experience significantly above-average profits. Such economic
conditions come from a sudden windfall gain to a certain business or industry, typically as a result of a
geopolitical disruption, natural disaster, or war that causes unusual spikes in demand or supply
interruptions. A good example is a confrontation between Russia and Ukraine.

28 www.visionias.in ©Vision IAS

FREE BY KING R QUEEN P [ऋषभ राजपूत]


• They are also imposed if there is an acute need for a temporary spurt in public spending at the same
time.
• There have been varying rationales for governments worldwide to introduce windfall taxes-
o Redistribution of unexpected gains when high prices benefit producers at the expense of consumers
o Funding social welfare schemes, and
o The supplementary revenue stream for the government.
• Windfall taxes are imposed retrospectively, one-off over and above normal taxes are often influenced by
unexpected events, and they can cause uncertainty in the market about future taxes.
• IMF says that taxes in response to price surges may suffer from design problems—given their expedient
and political nature. It added that introducing a temporary windfall profit tax reduces future
investment because prospective investors will internalize the likelihood of potential taxes when making
investment decisions.
• Another issue is who should be taxed- only the big companies responsible for the bulk of high-priced
sales or smaller companies as well.

Q 81.D
• Ulugh Khan, known in history by his later title of Balban, ascended the throne in 1265. The growing
authority of Balban alienated many of the Turkish chiefs who had hoped to continue their former power
and influence in the affairs of government since Nasiruddin Mahmud was young and inexperienced.
• Iltutmish, who is widely regarded as the true founder of the slave dynasty, established the 'Chalisha',
Turk-i-Chahalghani, and Chahalgani systems. This arrangement comprised a group of 40 nobles who
held significant sway in the governance of the Delhi Sultanate. Hence statement 1 is not correct.
• During the earlier period, Balban held the position of naib or deputy to Nasiruddin Mahmud whom
Balban had helped in securing the throne in 1246.
• An era of centralized government began for the first time. Balban constantly sought to increase the
prestige and power of the monarchy, because he was convinced that this was the only way to face the
internal and external dangers facing him. He refused to entertain important government posts for anyone
who did not belong to a noble family. Hence statement 2 is not correct.
• He reorganized the military department (diwan-i-arz), and pensioned off those soldiers and troopers
who were no longer fit for service. While Alauddin Khalji first set up a separate ariz's department.
Hence statement 3 is not correct.

Q 82.A
• MISHTI (Mangrove Initiative for Shoreline Habitats & Tangible Incomes) is a new program declared
in Union Budget 2023-23. It will facilitate mangrove plantations along India’s coastline and on
saltpan lands.
• Importance of MISHTI Initiative-
o Mangrove areas are some of the most bio-diverse locations in India.
o They also protect the coastlines from the vagaries of inclement weather.
o As climate change increases the incidence of extreme weather events across the world, mangrove
plantations have been shown to make coastal lands resilient, preventing flooding, and land
erosion and acting as a buffer for cyclones.
o They are also excellent carbon sinks and can sequester up to four times more carbon than tropical
rainforests.
• MISHTI program will be operated through convergence between MGNREGS, CAMPA Fund
(Compensatory Afforestation Fund Management and Planning Authority), and other sources, which
will be a game changer for protecting Mangroves. Mangrove saplings will be planted along the coastal
regions of the country. Its main objective is to “preserve Mangrove forests”. Hence statement 1 is
correct.
• Mangrove forests are spread across Andhra Pradesh, West Bengal, Odisha, Andaman and Nicobar,
Goa, Karnataka, Kerala, Maharashtra, and Tamil Nadu.
• Distribution of mangrove cover-
o About 40% of the world’s mangrove cover is found in South East Asia and South Asia.
o India has about 3% of the total Mangrove cover in South Asia.
• Mangroves distribution in India-
o Mangroves are found in the following states and Union Territories in India: West Bengal, Odisha,
Andhra Pradesh, Tamil Nadu, Andaman and Nicobar Islands, Kerala, Karnataka, Goa,
Maharashtra, and Gujarat.

29 www.visionias.in ©Vision IAS

FREE BY KING R QUEEN P [ऋषभ राजपूत]


o The major concentrations of mangroves are in the Sunderbans delta and Andaman & Nicobar
Islands.
o Mangroves are also found in the deltaic areas of the Mahanadi, Godavari, and Krishna river basins.
o West Bengal has 42.45% of India’s mangrove cover, followed by Gujarat at 23.66% and A&N
Islands at 12.39%. Hence statement 2 is not correct.

Q 83.B
• From the early centuries of the Common Era, we find grants of land being made, many of which were
recorded in inscriptions.
• Some of these inscriptions were on stone, but most were on copper plates which were probably given as a
record of the transaction to those who received the land. The records that have survived are generally
about grants to religious institutions or to Brahmanas.
• An agrahara was land granted to a Brahmana, who was usually exempted from paying land revenue
and other dues to the king and was often given the right to collect these dues from the local people. Hence
option (b) is the correct answer.

Q 84.D
• Fernao Nuniz was a traveler, chronicler, and horse trader of Portuguese-Jewish origin who resided in the
city of Vijayanagara, the capital of the Vijayanagara Empire, for a duration of three years spanning
from 1535 to 1537 during the reign of Achyuta Raya.
• Nicolo de Conti, a merchant hailing from Venice, journeyed through the eastern lands between 1414 and
1438. Among his destinations was the Vijayanagara Kingdom, where he arrived during the reign of
Dev Raya II. In his writings, he describes the city's fortifications and the vast number of soldiers serving
in the rulers' army.
• Abdur Razzak, a Persian traveler, traveled to India and visited the Vijayanagara Kingdom during
the reign of Dev Raya II. In "Matla as-Sadain Wa Majma ul-Bahrain," he provides a detailed
narrative of the rule of Devaraya II.
• Ibn Battuta, a renowned geographer, and explorer of the 14th century. Hailing from Morocco, he spent
three decades of his life journey in different parts including the Indian subcontinent, Central Asia,
Southeast Asia, and East China. In his book "Rehla," he also recounted the reign of Harihara I.
• Hence option (d) is the correct answer.

Q 85.B
• Akbar instituted a new system called the dahsala system of land revenue in 1580. It was a further
development of the Zabti system of land revenue. Zabti system was the system of measurement and
assessment based on the productivity of the soil. It was influenced by the Sher Shah’s system of land
revenue.
• Akbar introduced this system in the area from Lahore to Allahabad, and in Malwa and Gujarat. However
local prices and productivity was not taken into consideration in the Zabti system.
• The qanungos, who were hereditary holders of land as well as local officials conversant with local
conditions often concealed the real produce in many cases. So to reform the system Akbar appointed
officials known as Karoris who reported the true prices. Based on these findings the dahsala system was
introduced. Hence, statement 1 is correct.
• Under dahsala system of land revenue, the average produce of different crops as well as the average
prices prevailing over the last ten (dah) years were calculated. One-third of the average produce
was the state share. The state demand was, however, stated in cash. This was done by converting the
state share into money on the basis of a schedule of average prices over the past ten years. Thus, the
produce of a bigha of land under share was given in maunds. But on the basis of average prices, the state
demand was fixed in rupees per bigha.
• Besides, dahsala system other systems were also prevalent during the Mughal rule. One such was batai
or ghallabakhshi. In this system, the produce was divided between the peasants and the state in a
fixed proportion. The crop was divided after it had been thrashed, or when it had been cut and tied in
stacks, or while it was standing in the field. This system was considered a very fair one, but it needed an
army of honest officials to be present at the time of the ripening or the reaping of the crops. The peasants
were allowed to choose between zabti and batai under certain conditions. Hence, statement 2 is not
correct.

30 www.visionias.in ©Vision IAS

FREE BY KING R QUEEN P [ऋषभ राजपूत]


Q 86.D
• The most important economic development of the post Mauryan period was the thriving trade between
India and the eastern Roman Empire. In the beginning a good deal of this trade was carried on by land, but
the movement of the Sakas, Parthians and Kushans from the first century B.C. disrupted trade by land
route.
• Although the Parthians of Iran imported iron and steel from India they presented great obstacles to India's
trade with the lands further west of Iran. But from the first century A.D. trade was carried on mainly by
sea.
• It seems that around the beginning of the Christian era, the monsoons were discovered, So the sailors now
could sail in much less time directly from the eastern coast of the Arabian Sea to its western coast.
• They could call easily at the various ports such as Broach and Sopara situated on the western coast of
India, and Arikamedu and Tamralipti situated on its eastern coast.
o Of all these ports Broach seems to have been the most important and flourishing. To it were brought
not only the commodities produced in the Satayahana kingdom but also the goods produced in the
Saka and Kushan kingdoms.
• The Sakes and the Kushans used two routes from the north-western frontier to the western sea coast. Roth
these routes converged at Taxila, and were connected with the lower Indus basin from where it “passed on
to Broach. The second route called the uttarapatha was in more frequent use.
o From Taxila it passed through the modern Panjab up to the western coast of the Yamuna. Following
the course of the Yamuna it went southward to Mathura. From Mathura it passed on to Ujjain in
Malwa and again from Ujjain to Broach on the western coast. Ujjain was the meeting point of another
route that started from Kausambi near Allahabad.

• Hence option (d) is the correct answer.

Q 87.C
• Magadha came to prominence under the leadership of Bimbisara, who belonged to the Haryanka dynasty.
He was a contemporary of the Buddha. He started the policy of conquests and aggrandizement which
ended with the Kalinga war of Asoka.
• Magadha’s most serious rival was Avanti with its capital at Ujjain. Its king Chanda Pradyota Mahasena
fought Bimbisara but ultimately the two thought it wise to become friends. Later when Pradyota was
attacked by jaundice at the Avanti king’s request Bimblisara sent the royal physician Jivaka to Ujjain.
• The Sisunagas of Magadhas were succeeded by the Nandas, who proved to be the most powerful
rulers of Magadha. So great was their power that Alexander, who invaded Punjab at that time, did
not dare to move towards the east.
• All this took place in the reign of Mahapadma Nanda, He claimed to be ekarat, the sole sovereign who
destroyed all the other ruling princes.
• It seems that he acquired not only Kalinga but also Kosala which had probably rebelled against him.
• Hence option (c) is the correct answer.

Q 88.C
• Major traditional theatre form of South India:
• Dashavatar is the most developed theatre form of the Konkan and Goa regions. The performers
personify the ten incarnations of Lord Vishnu-the god of preservation and creativity. The ten
31 www.visionias.in ©Vision IAS

FREE BY KING R QUEEN P [ऋषभ राजपूत]


incarnations are Matsya (fish), Kurma (tortoise), Varaha (boar), Narsimha (lion-man), Vaman (dwarf),
Parashuram, Rama, Krishna (or Balram), Buddha and Kalki. Apart from stylized make-up, the Dashavatar
performers wear masks of wood and papier mache.
• Koodiyaattam, one of the oldest traditional theatre forms of Kerala, is based on Sanskrit theatre
traditions. The characters of this theatre form are: Chakyaar or actor, Naambiyaar, the instrumentalists and
Naangyaar, those taking on women’s roles. The Sutradhar or narrator and the Vidushak or jesters are the
protagonists. It is the Vidushak alone who delivers the dialogues. Emphasis on hand gestures and eye
movements makes this dance and theatre form unique.
• Yakshagaana, traditional theatre form of Karnataka, is based on mythological stories and
Puranas. The most popular episodes are from the Mahabharata i.e. Draupadi swayamvar, Subhadra
vivah, Abhimanyu vadh, Karna-Arjun yuddh and from Ramayana i.e. Raajyaabhishek, Lav-kush Yuddh,
Baali-Sugreeva yuddha and Panchavati.
• Therukoothu, the most popular form of folk drama of Tamil Nadu, literally means “street play”. It is
mostly performed at the time of annual temple festivals of Mariamman (Rain goddess) to achieve
rich harvest. At the core of the extensive repertoire of Therukoothu there is a cycle of eight plays based
on the life of Draupadi. Kattiakaran, the Sutradhara of the Therukoothu performance, gives the gist of the
play to the audience and Komali entertains the audience with his buffoonery.
• Hence, option (c) is the correct answer.

Q 89.B
• Cultural traditions from various regions of the country reflect the rich diversity of Regional Music of
India. Each region has its own particular style. Tribal and folk music is not taught in the same way that
Indian classical music is taught. There is no formal period of apprenticeship where the student is able to
devote their entire life to learning the music, the economics of rural life does not permit this sort of thing.
The musical practitioners must still attend to their normal duties of hunting, agriculture or whatever their
chosen profession is. Some of the important regional music traditions are discussed below:
• Rasiya Geet, Uttar Pradesh: The rich tradition of singing Rasiya Geet flourished in Braj which is the
sacred land of Lord Krishna’s charming leelas from time immemorial. This is not confined to any
particular festival but is closely woven into the very fabric of daily life and day-to-day chores of its
people. ‘Rasiya’ word is derived from the word rasa (emotion) because rasiya means that which is filled
with rasa or emotion. It reflects the personality of the singer as well as the nature of the song.
• Lotia, Rajasthan: ‘Lotia’ is sung in the Chaitra month during the festival – ‘Lotia’. Women bring
lotas (a vessel to fill water) and kalash (a vessel considered to be auspicious to fill water during
worship) filled with water from ponds and wells. They decorate them with flowers and come home.
Hence, pair 1 is not correctly matched.
• Pandavani, Chhattisgarh: In Pandavani, tales from Mahabharata are sung as a ballad and one or two
episodes are chosen for the night’s performance. The main singer continuously sits throughout the
performance and with powerful singing and symbolic gestures he assumes all the characters of the episode
one after another. Hence, pair 4 is correctly matched.
• Alha, Uttar Pradesh: Alha, typical ballad of Bundelkhand narrates the heroic deeds of Alha and Udal, the
two warrior brothers who served Raja Parmal of Majoba. This is the most popular regional music of
Bundelkhand which is popular elsewhere in the country as well.
• Chhakri, Kashmir: Chhakri is a group song that is the most popular form of Kashmir’s folk music.
It is sung to the accompaniment of the noot (earthen pot) rababs, sarangi and tumbaknari (an
earthen pot with high neck). Hence, pair 2 is correctly matched.
• Teej Songs, Rajasthan: Teej is celebrated with great involvement by women of Rajasthan. This is a
festival celebrated on the third day after the new moon or amavasya of shraavana month. The
theme of the songs sung during this festival revolves around the union of Shiva and Parvati, the
magic of monsoon, greenery, peacock dance, etc.
• Daskathia, Odisha: Daskathia is a form of ballad singing prevalent in Odisha. Daskathia is a name
derived from a unique musical instrument called “Kathi” or “Ram Tali”, wooden clappers used
during the presentation. The performance is a form of worship and offering on behalf of the “Das”,
the devotee. Hence, pair 3 is correctly matched.

Q 90.B
• In the Sushira Vadya group, the sound is produced by blowing air into a hollow column. The pitch
of the note is determined by controlling the air passage and the melody is played by using the
fingers to open and close the instrument. The simplest of these instruments is the flute. Generally,
flutes are made of bamboo or wood and the Indian musician prefers these due to the tonal and
32 www.visionias.in ©Vision IAS

FREE BY KING R QUEEN P [ऋषभ राजपूत]


musical attributes of these materials. However, there are references to flutes made of red
sandalwood, black wood, cane, ivory, bronze, silver and gold also. Hence, option (b) is the correct
answer.
• In the Avanaddha Vadya category of instruments, the sound is produced by striking the animal
skin which has been stretched across an earthen or metal pot or a wooden barrel or frame. The
earliest references to such instruments have been found in the Vedas where there is mention of Bhumi
Dundhubhi; this was a hollow pit dug in the ground and covered with the hide of a buffalo or ox which
was stretched across the pit. The tail of the animal was used for striking the animal hide and thus sound
was produced.
• The earliest instruments invented by man are said to be the Ghana Vadya. Once constructed, this
variety of instrument does not need special tuning prior to playing. In early times these instruments
were the extension of the human body such as sticks, clappers, rods, etc. and were also closely
related to objects of utility in daily life such as pots and pans, jhanj, falams, etc. They are
principally rhythmic in function and are best suited as an accompaniment to folk and tribal music
and dance.
• The tata vadya is a category of instruments in which sound is produced by the vibration of a string
or chord. These vibrations are caused by plucking or by bowing on the string which has been pulled taut.
The length of the vibrating string or wire, the degree to which it has been tightened, determines the pitch
of the note and also to some extent the duration of the sound.

Q 91.C
• Qalandars, Madaris, Malangs, and Haidaris were the Sufi saints who observed extreme forms of
asceticism. They initiated movements based on a radical interpretation of sufi ideals. They defied the laws
of shari‘a. Thus, they were often referred to as be-shari‘a, in contrast to the ba-shari‘a sufis who complied
with it. Hence, pair 1 is not correctly matched.
• Ziyarat in Sufism referred to a practice of pilgrimage to the grave of a sufi saint. Such pilgrimages
were undertaken particularly on his death anniversary or urs (or marriage, signifying the union of his soul
with God). Hence, pair 2 is correctly matched.
• Khalifa was the successor of a Sufi saint appointed by the saint himself. The Sufi saints were known by
different names such as shaikh (in Arabic), pir or murshid (in Persian). Saints established rules for
spiritual conduct and interaction between inmates as well as between laypersons and the master. He
enrolled disciples (murids) and appointed a successor (khalifa). Hence, pair 3 is correctly matched.
• Masnavis were a series of long poems written by the Sufi saints to express ideas of divine love using
human love as an allegory. Hence, pair 4 is correctly matched.

Q 92.B
• The Pala dynasty was a powerful Indian dynasty that ruled parts of eastern India from the 8th to the
12th century CE. The Pala empire was founded by Gopala, probably in 750 AD. He was succeeded in
770 by his son Dharmapala who ruled till 810. He was defeated by the Rashtrakuta ruler, Dhruva. Hence
Statement 1 is not correct.
• The Pala rulers were great patrons of Buddhist learning and religion. Dharmapala revived Nalanda
University and founded the Vikramshila University which became second only to Nalanda in
fame. Hence statement 2 is correct.
• A conflict between Dharmapala and Vatsaraja, the Pratihara king, arose as a consequence of the Kannauj
dispute. Later, Dharmapala recaptured Kannauj and installed his vassal Chakrayudha as the
ruler. Hence statement 3 is correct.
• This move solidified his position as the most dominant leader in Northern India, and he proclaimed
himself as the Uttarapatha Swamin or "Lord of Northern India".

Q 93.B
• Odisha is famous for temple architecture and with varieties of temples presents the Kalingan School
of Architecture in its well-developed forms. According to Silpa Sastras, there are three different
types of temples in the Kalingan School. These are ‘Rekha’, ‘Bhadra’ or ‘Pidha’ and ‘Khakhara’.
Hence, option (b) is the correct answer.
• The rekha temple or the vimana is characterized by a curvilinear superstructure. It can be divided into
four parts. The four divisions are pista, bada, gandi and sira or mastaka. From the bottom to the finial,
each part of the temple has a separate name. The Odishan craftsmen considered the temple as the body of
the Cosmic Being. Therefore, the different parts. of the temple are named after limbs of the body. Just as

33 www.visionias.in ©Vision IAS

FREE BY KING R QUEEN P [ऋषभ राजपूत]


the different parts of a human body are organically related to each other, so the different divisions of the
temple bear a vital relationship with each other and are integrated into an artistic composition.
• The Khakhara temple is very unique in its style. This type is very limited in Odisha. This is exclusively
meant for the Sakti worship. The gandi of the khakhara is composed either like that of a rekha or of a
pidha with certain minor differences.25 The plan of the deula is oblong, and its mastaka is distinguished
by its barrel vaulted elongated roof called khakhara by the treatisers due to its faint resemblance to
kakharu or voita kakharu. Over the khakhara are placed either miniature amlas or kalasa flanked by lions.
• There is no difference between Rekha temple and Pidha temple in the treatment of the bada, but they
differ in the disposition of the gandi. The gandi of the jagamohana is of pyramidal shape. It is composed
of a number of pidhas or horizontal platforms, piled up in the form of a pyramid. The pidhas rapidly
decrease in size from bottom upwards. The diminution proceeds until the topmost pidha is half in size to
the lower most one. The pidhas may be arranged in one or two tiers, with moderate height of vertical wall
intervening between them. Each of these tiers is called a potala. The cross-section at any point of the
gandi is square. Above the gandi comes the mastaka, composed of several elements which are circular in
cross-section. First comes the beki, then the ghanta, an enormous ribbed structure shaped like a bell. On
the top of the ghanta is a succession of beki, amla, khapuri and kalasa as in the Rekha.

Q 94.A
• The Sangam classics, consisting of 18 works (eight anthologies of lyrics and ten long poems), are well
known for their directness of expression.
• These were written by 473 poets, among whom 30 were women, the famous poetess Avvaiyar being one
of them.
• In the case of 102 poems, the authors are unknown. Most of these anthologies are of the 3rd century B.C.
• During this time, a Tamil grammar Tolkappiyam was written, to understand early Tamil poetry.
• Tolkappiyam indicates five landscapes or types of love and outlines their symbolic conventions.
• Critics say that Sangam literature is not just the earliest evidence of the Tamil genius. The Tamils, in all
their 2000 years of literary effort, wrote nothing better.
• The famous Thirukkural by Thiruvalluvar, in the 6th century A.D., serves as a manual of precepts to
guide one to noble living. It expounds a secular, moral, and practical attitude towards life.
• The twin epics, Silappadikaram (the story of the anklet), written by Ilango-Adigal, and
Manimekalai (the story of Manimekalai) by Chattanar, were written sometimes in A.D. 200-300 and
give vivid accounts of Tamil society during that period.
• These are valuable storehouses and epics of dignity and sublimity, laying stress on the cardinal virtues of
life. In Manimekalai there is an elaborate exposition of the doctrines of Buddhism.
• If Tamil reveals a triumph of Brahmanic and Buddhist knowledge, Kannada shows Jain's ascendency in
its ancient phase. Malayalam absorbed a rich treasure contained in the Sanskrit language.
• Nannaya (A.D.1100) was the first Telugu poet. In ancient times, Tamil and Telugu spread to distant
places.
• Hence, option (a) is the correct answer.

Q 95.B
• The Harappan cities did not possess the necessary raw material for the commodities they produced. They
did not use metallic money. We have no idea about their currency. Most probably they carried on
all exchanges through barter in return for finished goods and possibly foodgrains. Hence statement 1 is
not correct.
• They procured metals from the neighboring areas by boats and bullock carts. They practiced navigation on
the coast of the Arabian Sea.
• They knew the use of wheel and carts with, solid wheels were in use in Harappa. It also appears that the
Harappans used some kind of modern ekka.
• The Harappans had commercial links with Rajasthan, Afghanistan and Iran. Their cities also carried
on commerce with those in the land of the Tigris and Euphrates. Many Harappan seals have been
discovered in Mesopotamia, and it seems that the Harappans imitated some cosmetics used by the urban
people of Mesopotamia. Hence statement 2 is correct.
• The Mesopotamian records from about 2350 B.C. refer to trade relations with Meluha, which was the
ancient name given to the Indus region. The Mesopotamian texts speak of two intermediate trading
stations called Dilmun and Makan, which lay between Mesopotamia and Meluha: Dilmun can probably be
identified with Bahrain on the Persian Gulf.

34 www.visionias.in ©Vision IAS

FREE BY KING R QUEEN P [ऋषभ राजपूत]


Q 96.A
• The Qutb Minar also spelled Qutub Minar and Qutab Minar, is a minaret and "victory tower" that
forms part of the Qutb complex, which lies at the site of Delhi’s oldest fortified city, Lal Kot, founded by
the Tomar Rajputs. It is a UNESCO World Heritage Site in the Mehrauli area of South Delhi, India. The
monument was started by Qutubuddin Aibak, but completed by Iltutmish. It is believed to have been
dedicated to the Sufi saint Qutubuddin Bakhtiyar Kaki. Hence statement 1 is not correct.
• The Qutub Minar, built in the thirteenth century, is a 234-foot-high tapering tower divided into five
stories. Hence statement 1 is not correct.
• The minar is a mix of polygonal and circular shapes.
• It is largely built of red and buff sandstone with some use of marble in the upper storeys. Hence
statement 3 is not correct.
• It is characterized by highly decorated balconies and bands of inscriptions intertwined with foliated
designs.
• Alai Darwaza (southern door to the Qutub Minar) is the southern gateway of the Quwwat-ul-Islam
Mosque in the Qutub complex, Mehrauli, Delhi, India. Built by Sultan Alauddin Khalji in 1311 and made
of red sandstone, it is a square domed gatehouse with arched entrances and houses a single chamber.
Hence statement 2 is correct.

Q 97.D
• Recently, OpenAI introduced a new chatbot called ChatGPT. It is a ‘conversational’ AI (hence the
word “chat”) that can answer queries similar to how a human would. It can answer “follow-up
questions”, and can also “admit its mistakes, challenge incorrect premises, and reject inappropriate
requests.” Hence option (d) is the correct answer.
• Here, GPT stands for Generative Pre-trained Transformer 3 and its relevance is derived from the fact
that it was trained using Reinforcement Learning from Human Feedback (RLHF).
• Applications: Its usages are immense right from coding to drafting a personal message for a loved one.

Q 98.B
• Recent context: Northern Ireland Protocol will be replaced by the “Windsor Framework”.
• A new agreement to alter the Northern Ireland Protocol’s functioning was announced by the European
Commission and the UK. The Protocol entered into force in January 2021, and since then, the EU and
the UK have been in negotiations for two years.
• According to the Windsor Framework, Northern Ireland can continue to be a part of the EU’s single
market and customs union. It will remove many of the onerous procedures that were put in place on
products coming from the British mainland as part of the first Brexit agreement.
• With the new arrangement, commodities moving from Great Britain to Northern Ireland and
those continuing into the EU will be divided into “green” and “red” lanes.
• Significance- The Windsor Framework will still give the EU some influence over Northern Ireland.
Several industries will continue to be subject to EU legislation, particularly those that deal with food and
animal products, and the European Court of Justice will continue to be the deciding body in these cases.
• Hence option (b) is the correct answer.

Q 99.A
• Yaosang: The festival is celebrated every year on the full moon of Lamta (February-March) of
the Meitei lunar calendar. It begins just after sunset followed by Yaosang Mei Thaba (Burning of the
Straw Hut). Unlike Holi, it is celebrated with a traditional twist. During these five days, Manipur comes
alive with sporting events during the day and traditional “thabal chongba” dance the night. The thabal
chongba is a traditional dance of the Meitei, where boys and girls gathered in an open ground and dance
in a circle. But these days thabal chongba is performed throughout the month of Lamta. Business activities
and public transportation come to a grinding halt during Yaoshang. All educational institutions, both
private and government, will also remain shut. Hence, option (a) is the correct answer.
• Losar: It is a major festival in Arunachal Pradesh as it marks the Tibetan New Year. Tribes like
Monpa, Sherdukpens, Memba, Khamba and Nah following the Mahayana sect of Buddhism
celebrate this festival with all pomp. This three-day festival begins on February 11 each year. On the first
day of the festival, the priests make offering to the highest priest called the Dharmapala or Palden
Lhamo, whereas the masses visit friends and family and wish them Tashi Delek (best wishes). Also
according to the local tradition, sprouted barley seeds and buckets of tsampa (roasted barley flour
with butter) and other grains are offered on home altars to ensure a good harvest. On the second day of
the festival, which is also called Gyalpo Losar, national leaders and kings are honoured. On the third and
35 www.visionias.in ©Vision IAS

FREE BY KING R QUEEN P [ऋषभ राजपूत]


final day of the festival known as Choe-Kyong Losar, people make offerings to Dharmapala and tie
prayer flags on rooftops and across the locality. Although the spiritual observance ends on the third day,
the celebration can extend for 10 to 15 days.
• Saga Dawa is one of the major Buddhist festivals in Sikkim and is celebrated in the Tibetan lunar
month. The full moon day that falls in the middle of the lunar month is known as Saga Dawa and it is
considered as an auspicious day by the Buddhists. It is called the ‘Month of Merits’ and is celebrated
between May and June. The festival commemorates the birth, enlightenment and death of the
Buddha. On the occasion of Saga Dawa in Sikkim, people gather at the monasteries and offer water,
incense sticks and dhog. Also, many participate in the circumambulation of the Gompas, where they chant
mantras, carry the religious text and turn the prayer wheels.

Q 100.D
• Recent Context: Common Eligibility Test is to be conducted in India. It aims to replace multiple
examinations conducted by different recruiting agencies for selection to government jobs advertised each
year, with a single online test.
• At present, candidates seeking government jobs have to appear for separate examinations
conducted by multiple recruiting agencies for various posts. The common eligibility test (CET) aims
to replace multiple examinations conducted by different recruiting agencies for selection to government
jobs advertised each year, with a single online test.
• The Union Cabinet has approved the setting up of the National Recruitment Agency, an independent
body to conduct examinations for government jobs. It will be a Society registered under the Societies
Registration Act of 1860. Hence statement 2 is correct
• Initially, it will organize a CET to screen/shortlist candidates for the Group B and C (non-technical)
posts, which are now being conducted by the Staff Selection Commission (SSC), Railways
Recruitment Board (RRBs) and Institute of Banking Personnel Selection (IBPS). Later on, more
exams may be brought under it. Hence statement 1 is correct.
• As the multiple recruitment examinations are a burden on the candidates, as also on the respective
recruitment agencies, involving avoidable/repetitive expenditure, law and order/security-related
issues and venue-related problems. On average, 2.5 crore to 3 crore candidates appear in each of these
examinations. A CET would enable these candidates to appear once and apply to any or all of
these recruitment agencies for a higher level of examination. This would indeed be a boon to all the
candidates.
• Salient features of the NRA and CET:
o NRA to conduct online CET twice a year.
o Registration of applicants, generation of roll number/admit cards, display of marks, merit list, etc, will
be done online.
o The CET would be available in a number of languages. This would greatly facilitate people from
different parts of the country to take the exam and have an equal opportunity of being selected.
o The CET will be based on multiple choice objective type question paper and it will be a computer-
based exam.
o The CET score of a candidate will be valid for three years. Hence statement 3 is correct.

Copyright © by Vision IAS


All rights are reserved. No part of this document may be reproduced, stored in a retrieval system or
transmitted in any form or by any means, electronic, mechanical, photocopying, recording or otherwise,
without prior permission of Vision IAS.
36 www.visionias.in ©Vision IAS

FREE BY KING R QUEEN P [ऋषभ राजपूत]


VISIONIAS
www.visionias.in

Test Booklet Series

TEST BOOKLET

GENERAL STUDIES (P) 2024 – Test – 4131


C
Time Allowed: Two Hours Maximum Marks: 200

INSTRUCTIONS

1. IMMEDIATELY AFTER THE COMMENCEMENT OF THE EXAMINATION, YOU SHOULD CHECK THAT THIS BOOKLET
DOES NOT HAVE ANY UNPRINTED OR TURN OR MISSING PAGES OR ITEMS, ETC. IF SO, GET IT REPLACED BY A
COMPLETE TEST BOOKLET.

2. ENCODE CLEARLY THE TEST BOOKLET SERIES A, B, C OR D AS THE CASE MAY BE IN THE APPROPRIATE PLACE IN
THE ANSWER SHEET.

3. You have to enter your Roll Number on the Test Booklet in the Box
provided alongside. Do NOT write anything else on the Test Booklet.

4. This Test Booklet contains 100 items (Questions). Each item is printed in English. Each item comprises four
responses (answers). You will select the response which you want to mark on the Answer Sheet. In case you
feel that there is more than one correct response with you consider the best. In any case, choose ONLY ONE
response for each item.

5. You have to mark all your responses ONLY on the separate Answer Sheet provided. See direction in the
answers sheet.

6. All items carry equal marks. Attempt all items. Your total marks will depend only on the number of correct
responses marked by you in the answer sheet. For every incorrect response 1/3rdof the allotted marks will be
deducted.

7. Before you proceed to mark in the Answer sheet the response to various items in the Test booklet, you have to
fill in some particulars in the answer sheets as per instruction sent to you with your Admission Certificate.

8. After you have completed filling in all responses on the answer sheet and the examination has concluded, you
should hand over to Invigilator only the answer sheet. You are permitted to take away with you the Test
Booklet.

9. Sheet for rough work are appended in the Test Booklet at the end.

DO NOT OPEN THIS BOOKLET UNTIL YOU ARE ASKED TO DO SO


1 www.visionias.in ©Vision IAS

FREE BY KING R QUEEN P [ऋषभ राजपूत]


1. With reference to Sayyid Ahmad Khan, 4. With reference to the Swadeshi Movement,
consider the following statements: consider the following statements:

1. In 1875, he founded the Mohammedan 1. It had its genesis in the anti-partition


movement of Bengal.
Anglo-Oriental College in Lahore.
2. The big zamindars of Bengal did not
2. He interpreted the Quran in the light of
extend support to the Swadeshi cause.
contemporary rationalism and science. 3. Samitis (Corps of volunteers) helped in
Which of the statements given above is/are mobilisation of the mass support for the
correct? movement.
(a) 1 only Which of the statements given above is/are
(b) 2 only correct?
(a) 1 only
(c) Both 1 and 2
(b) 2 and 3 only
(d) Neither 1 nor 2
(c) 1 and 3 only
(d) 1, 2 and 3
2. Who among the following is associated with
‘The Philosophy of the Bomb’, a critique of 5. In the context of Indian freedom struggle,
non-violence that sought to explain the Bengalee, Hitabadi and Sanjibani were:
thinking behind revolutionary actions? (a) journals and newspapers
(b) revolutionary organizations
(a) Sardar Bhagat Singh
(c) societies focused on social reform
(b) Jatin Das
(d) educational organizations
(c) Sachindranath Sanyal
(d) Bhagwati Charan Vohra 6. The object of the Cunningham circular of
1930 was
3. With reference to the Office of Secretary of (a) to defend the honour of the National
State during the British colonial rule in flag.

India, consider the following statements: (b) to forbid students from participating in
political activities.
1. It was created by the Government of
(c) to refuse to pay chowkidara tax.
India Act 1858 to supervise the
(d) to suppress the separate manjari sena or
administration of India. cat army.
2. The Secretary of the State served as the
personal representative of the British 7. In which of the following congress sessions,
Crown in India and was accountable to the congress formally announced its changed
it. policy towards princely states, from that of
non-intervention to the removal of any self-
Which of the statements given above is/are
imposed constraint on its participation in the
correct?
princely states?
(a) 1 only
(a) Nagpur session, 1920
(b) 2 only (b) Lahore session, 1929
(c) Both 1 and 2 (c) Tripuri session, 1939
(d) Neither 1 nor 2 (d) Haripura session, 1938
2 www.visionias.in ©Vision IAS

FREE BY KING R QUEEN P [ऋषभ राजपूत]


8. Consider the following statements regarding 11. The Charter Act of 1813, provided for
the Animal Birth Control (ABC) Rules, 1. opening of Indian trade for all British
2023:
merchants.
1. It provides guidelines for the
2. allocation of Company resources for the
sterilization and immunization of stray
dogs through Animal Birth Control spread of modern sciences in India.
programs. 3. ending of the East India Company's
2. The rules classify stray dogs into a new monopoly in the tea trade.
class of community animals.
Select the correct answer using the code
3. Resident welfare associations will be
given below.
responsible for caring for stray dogs and
feeding them at fixed intervals. (a) 1 only
Which of the statements given above is/are (b) 1 and 2 only
correct? (c) 2 and 3 only
(a) 1, 2 and 3
(d) 1, 2 and 3
(b) 2 and 3 only
(c) 1 and 2 only
(d) 3 only 12. Consider the following statements with
respect to the INA Relief and Enquiry
9. Champaran Satyagraha was launched against Committee:
the: 1. It was set up by Subhash Chandra Bose.
(a) withdrawal of plague bonus given to the
2. Its main objective was to rehabilitate the
workers.
(b) increase in land revenues imposed by the INA prisoners after their trial.
colonial government in the district. Which of the statements given above is/are
(c) exploitation of peasants working on correct?
indigo plantations by the European (a) 1 only
planters.
(b) 2 only
(d) refusal of the government to remit land
revenue in light of the crop failure. (c) Both 1 and 2
(d) Neither 1 nor 2
10. They are tribes that live in several states in
west and south India, especially near forest 13. Who among the following established
areas. They are a semi-nomadic tribe,
educational institutions for the study of
traditionally of bird catchers and hunters.
They follow Hindu traditions and celebrate Indian traditions/philosophy during the
all Hindu festivals. The eldest son in a family British colonial rule in India?
is not supposed to cut his hair so that he can 1. Francis Buchanan
be identified easily. In different regions, they 2. Jonathan Duncan
are known by different names, such as Mel-
3. Warren Hastings
Shikari in northern Karnataka and
Maharashtra. Select the correct answer using the code
Which of the following tribes of India has given below.
been described in the passage given above? (a) 1 and 2 only
(a) Hakki Pikki (b) 2 and 3 only
(b) Chenchus
(c) 3 only
(c) Apatanis
(d) Baigas (d) 1, 2 and 3
3 www.visionias.in ©Vision IAS

FREE BY KING R QUEEN P [ऋषभ राजपूत]


14. With reference to the Marketing and 17. In the context of Indian history, the
Logistics Development for Promotion of
Standstill Agreement is related to which of
Tribal Products from North-Eastern Region
(PTP-NER) scheme, consider the following the following?
statements:
1. It is a centrally sponsored scheme which (a) As an aftermath to 1857 mutiny an
aims to strengthen livelihood agreement signed between Princely
opportunities for tribal artisans.
2. The state governments will organize states rulers and the British Queen to not
Tribal Artisans Melas throughout the
annex princely states in future.
year to showcase tribal products.
Which of the statements given above is/are (b) As part of Princely states'
not correct?
reorganization, an agreement signed
(a) 1 only
(b) 2 only between Indian government and Nizam
(c) Both 1 and 2
of Hyderabad.
(d) Neither 1 nor 2
(c) Agreement signed between British
15. With reference to Lucknow Pact of 1916,
consider the following statements: government and Congress to abandon
1. Both Congress and Muslim league put Civil Disobedience movement.
forward a joint scheme of political
reforms. (d) Agreement signed between the congress
2. Both Congress and Muslim league
and depressed classes after the Ramsay
rejected separate electorate.
Which of the statements given above is/ are MacDonald's Communal Award.
not correct?
(a) 1 only
(b) 2 only 18. Consider the following statements with
(c) Both 1 and 2
(d) Neither 1 nor 2 respect to the Desai-Liaqat Plan:

1. It provided for the nomination of


16. Consider the following statements, with
reference to Rehnumai Mazdayasan Sabha: persons from all major political parties
1. S.S. Benaglee and Dadabhai Naoroji
in the central legislature.
were the founding members of the
Sabha. 2. It proposed to provide 20% reserved
2. The campaign initiated the
seats to minorities.
modernization of Parsi social customs.
3. This sabha created a great impact on Which of the statements given above is/are
Western scholarship to reinterpret
correct?
Zoroastrianism.
Which of the statements given above are (a) 1 only
correct?
(a) 1 and 3 only (b) 2 only
(b) 1 and 2 only (c) Both 1 and 2
(c) 2 and 3 only
(d) 1, 2, and 3 (d) Neither 1 nor 2
4 www.visionias.in ©Vision IAS

FREE BY KING R QUEEN P [ऋषभ राजपूत]


19. Consider the following statements with 21. Consider the following statements about Bal
Gangadhar Tilak:
respect to National Quantum Mission:
1. He founded the newspaper Kesari in
1. It aims to promote indigenous English and Mahratta in Marathi.
2. He was arrested under Section 124A of
production of quantum computers, the Indian Penal Code on the charge of
sedition against the government.
quantum materials and satellite based
3. He started the practice of using
quantum communications. traditional religious festivals like the
Ganapati and Shivaji festivals to
2. Four Thematic Hubs will be set up in top propagate nationalist ideas through
national R&D institutes on the domains patriotic songs and speeches.
Which of the statements given above is/are
of Quantum Technology. correct?
(a) 1 and 2 only
3. It is being implemented by the Union
(b) 2 and 3 only
Ministry of Electronics and Information (c) 1 and 3 only
(d) 1, 2 and 3
Technology.
22. With reference to Pitt’s India Act of 1784,
Which of the statements given above is/are
consider the following statements:
correct? 1. It established a board of six
commissioners including the two cabinet
(a) 1 only
ministers for the affairs of India.
(b) 2 and 3 only 2. The act gave British Parliament the final
authority to appoint and dismiss
(c) 1 and 2 only company officials in India.
Which of the statements given above is/are
(d) 1, 2 and 3
correct?
(a) 1 only
(b) 2 only
20. Consider the following pairs: (c) Both 1 and 2
Product State (d) Neither 1 nor 2

1. Agasaim brinjal : Odisha 23. Consider the following personalities


associated with Gandhi during his early
2. Malcorada mango : Goa
career and activism:
3. Manamadurai pottery : Kerala 1. Ambalal Sarabhai
2. Brij Kishore
Which of the pairs given above is/are 3. Mahadev Desai
4. Narhari Parekh
correctly matched?
Which of the personalities given above
(a) 1 and 2 only actively supported Gandhiji in the 1918
Ahmedabad Mill owners dispute?
(b) 2 only
(a) 1 and 3 only
(c) 1 and 3 only (b) 1 only
(c) 1, 2, 3 and 4
(d) 3 only (d) None
5 www.visionias.in ©Vision IAS

FREE BY KING R QUEEN P [ऋषभ राजपूत]


24. Who among the following advised Gandhiji 27. What was the objective of Rajagopalachari
to spend a year traveling around British Formula (1944)?
India upon his arrival in 1915? (a) To frame a new constitution by the
(a) Bal Gangadhar Tilak newly constituted executive council
(b) Bipin Chandra Pal having Indian members.
(c) Motilal Nehru (b) To bifurcate regions for elections
(d) Gopal Krishna Gokhale according to proportional representation.
(c) To seek cooperation of Muslim League
25. Consider the following statements with
and Congress in forming a provisional
regard to Ranjit Singh:
government at the centre.
1. To improve land revenue, he brought
(d) To demand a new formula for giving
drastic changes in the system of land
representation to depressed classes and
revenue promulgated earlier by the
minorities.
Mughals.
2. He built up a powerful, disciplined, and
28. In the context of Indian history, the naval
well-equipped army along European
ratings of HMIS Talwar went on strike to
lines.
protest against
3. He set up modern factories to
1. Racial discrimination
manufacture cannons at Lahore.
Which of the statements given above are 2. Unpalatable food

correct? 3. New recruitment policy

(a) 1 and 2 only Select the correct answer using the code
(b) 2 and 3 only given below.
(c) 1 and 3 only (a) 1 and 2 only
(d) 1, 2 and 3 (b) 2 and 3 only
(c) 1 and 3 only
26. With reference to the Congress session, (d) 1, 2 and 3
consider the following statements:
1. The resolution on Fundamental Rights 29. Which of the following are the members of
and Economic Policy was passed at the the Price Cap Coalition?
Lahore session of the Indian National 1. Australia
Congress in 1929. 2. New Zealand
2. The resolutions on 'purna swaraj' were 3. France
passed at the Faizpur session of the 4. European Union
Indian National Congress in 1936.
Select the correct answer using the code
Which of the statements given above is/are
given below.
correct?
(a) 1, 3 and 4 only
(a) 1 only
(b) 2 and 3 only
(b) 2 only
(c) 1, 2, 3 and 4
(c) Both 1 and 2
(d) 2 and 4 only
(d) Neither 1 nor 2
6 www.visionias.in ©Vision IAS

FREE BY KING R QUEEN P [ऋषभ राजपूत]


30. Recently, the Ministry of Earth Sciences has 33. With reference to the peasant movement,
launched a Research, Education, and consider the following pairs:
Training Outreach (REACHOUT) scheme Peasant movement Associated leader
for capacity building. It consists of which of 1. Kuka Movement : Baba Ram Singh
the following sub-schemes? 2. Ramosi peasant : Gauri Shankar
1. R&D in Earth System Science rebellion Mishra
2. International Training Centre for 3. Kisan Sabha : Vasudev Balwant
Operational Oceanography movement in the Phadke
3. Program for Development of Skilled United Provinces
Manpower in Earth System Sciences Which of the pairs given above is/are
Select the correct answer using the code correctly matched?
given below. (a) 1 only
(a) 1 and 2 only (b) 2 and 3 only
(b) 2 and 3 only (c) 1 and 3 only
(c) 1 and 3 only (d) 1, 2 and 3
(d) 1, 2 and 3
34. Arrange the following organizations in the
31. Consider the following statements regarding chronological order of their formation.
Ceramic Radome Technology: 1. The Ramakrishna mission
1. Radomes are structures designed to 2. Atmiya Sabha
protect an antenna from the surrounding 3. Satyasodhak Samaj
environment. Select the correct answer using the code
2. It is the state-of-the-art technology for given below.
shielding missiles from getting (a) 3-1-2
overheated. (b) 1-2-3
Which of the statements given above is/are (c) 2-3-1
correct? (d) 1-3-2
(a) 1 only
(b) 2 only 35. Which of the following were the demands of
(c) Both 1 and 2 the Indian nationalist in the later part of the
(d) Neither 1 nor 2 19th century?
1. Reduction of import duties on textile
32. With reference to the General Elections held import
in 1945, consider the following statements: 2. No expansion in Afghanistan or Burma
1. The Indian National Congress won a 3. The right to bear arms and reduction of
majority of seats in the Central military expenditure
Legislative Assembly. 4. Higher expenditure on famine relief.
2. The Muslim League assumed power in 5. The right of Indians to join the semi-
Bengal, Punjab and Sind. military volunteer corps.
Which of the statements given above is/are Select the correct answer using the code
correct? given below.
(a) 1 only (a) 1, 2, and 3 only
(b) 2 only (b) 3, 4, and 5 only
(c) Both 1 and 2 (c) 1, 2, and 5 only
(d) Neither 1 nor 2 (d) 2, 3, 4 and 5 only
7 www.visionias.in ©Vision IAS

FREE BY KING R QUEEN P [ऋषभ राजपूत]


36. Arrange the following historical events on a 39. In the context of Montague-Chelmsford
chronological sequence:
reforms, which of the statements are correct?
1. Second Round Table Conference
2. Gandhi-Irwin Pact 1. Dyarchy was introduced at the level of
3. Karachi Resolution the central government.
4. Execution of Bhagat Singh
2. Majority of the members of the
Select the correct answer using the code
given below. Provincial Legislative Councils were to
(a) 4-2-3-1
be elected.
(b) 2-4-1-3
(c) 4-2-1-3 3. Congress leader Surendranath Banerjea
(d) 2-4-3-1 was in favor of accepting the reforms.

37. Consider the following statements about the Select the correct answer using the code
Government of India Act, 1935: given below.
1. It introduced a new system of dyarchy
(a) 1 and 3 only
for the executive at the level of the
provincial government. (b) 2 and 3 only
2. It allowed Governors to veto legislative
(c) 1 and 2 only
action of the provincial assemblies and
legislate on their own. (d) 1, 2 and 3
Which of the statements given above is/are
correct?
40. Which of the following are the terms and
(a) 1 only
(b) 2 only conditions of the Subsidiary Alliance?
(c) Both 1 and 2
1. The British would be responsible for
(d) Neither 1 nor 2
protecting their ally from external and
38. Consider the following pairs: internal threats.
Types of Reason
Eclipse 2. In the territory of the ally, a British
1. Annular : The moon passes between armed contingent would be stationed.
Eclipse the Earth and the Sun but is
3. The ally could enter into agreements
not perfectly aligned
2. Partial : Only a small ring-like sliver with other rulers only with the
Eclipse of light is seen from the
permission of the British.
sun's disc
3. Hybrid : Caused by the curvature of Select the correct answer using the code
Eclipse the earth's surface given below.
Which of the pairs given above is/are
(a) 1 and 2 only
correctly matched?
(a) 1, 2 and 3 (b) 1 and 3 only
(b) 1 and 2 only
(c) 2 and 3 only
(c) 3 only
(d) 2 and 3 only (d) 1, 2 and 3
8 www.visionias.in ©Vision IAS

FREE BY KING R QUEEN P [ऋषभ राजपूत]


41. What were the reasons for the change in the 44. In the context of Individual Satyagraha, who
attitude of the British Indian Government among the following was the first
satyagrahi?
towards the Indian National movement post-
(a) Jawahar Lal Nehru
INA trials?
(b) Acharya Vinoba Bhave
1. The USA and the Soviet Union (c) Brahma Dutt
supported India’s demand for freedom. (d) Sarojini Naidu
2. Many members of the Conservative
45. Arrange the following events in
Party which replaced the Labour Party
chronological order of their occurence.
supported Congress's demands.
1. Passing of Pakistan Resolution by
3. Loss of faith in Indian personnel for Muslim League
suppressing the national movement. 2. Launch of Individual Satyagraha
Select the correct answer using the code movement to affirm the rights to speech
3. Proposal of August Offer
given below.
Select the correct answer using the code
(a) 1 and 2 only
given below.
(b) 2 and 3 only (a) 1-2-3
(c) 1 and 3 only (b) 2-1-3
(d) 1, 2 and 3 (c) 3-2-1
(d) 1-3-2

42. Good Friday Agreement, sometimes seen in


46. Consider the following statements regarding
the news, is an agreement between: the MAHARISHI Initiative:
(a) Ukraine and E.U 1. It aims to promote research and
(b) Israel and U.A.E awareness about Ayurveda and
traditional medicine.
(c) Britain and Ireland
2. The Central Council for Research in
(d) Poland, Moldova and Russia Ayurvedic Sciences will be the
secretariat for this initiative.
43. This city, which lies on the coast of the Bay Which of the statements given above is/are
of Bengal, was granted on lease by a local correct?
(a) 1 only
Raja to the company in 1639. The Raja
(b) 2 only
authorized East India Company to fortify the (c) Both 1 and 2
place, to administer it, and to coin money on (d) Neither 1 nor 2
condition of payment to him.
Which of the following cities has been 47. In 1510, Goa was captured from the Sultan
of Bijapur by which of the following
described in the passage given above?
Portuguese viceroys?
(a) Masulipatam (a) Francisco de Almeida
(b) Surat (b) Tristao da Cunha
(c) Balasore (c) Alfonso de Albuquerque
(d) Vasco da Gama
(d) Madras
9 www.visionias.in ©Vision IAS

FREE BY KING R QUEEN P [ऋषभ राजपूत]


48. Consider the following report: 51. Recently, Prime Minister of India
Report/Index Released by inaugurated Kochi Water Metro. In this
1. Global Food Policy : Food and context, consider the following statements:
Report Agriculture
1. It a project undertaken by the Union
Organization
Ministry of Shipping to connect the
2. Logistic Performance : World Bank
Malabar coast with the Konkan coast.
Index
3. Data Threat Report : Thales 2. It includes boats that are hybrid, battery-
Which of the pairs given above is/are powered and disabled-friendly.
correctly matched? Which of the statements given above is/are
(a) 1, 2 and 3 correct?
(b) 1 and 3 only
(a) 1 only
(c) 2 only
(b) 2 only
(d) 2 and 3 only
(c) Both 1 and 2

49. Consider the following statements regarding (d) Neither 1 nor 2


the Indian Independence Act of 1947:
1. It declared India as an independent state 52. Which of the following personalities were
from August 15, 1947. tried and sentenced to imprisonment in the
2. It abolished the office of Viceroy and
Kanpur Bolshevik Case, 1924?
established the office of Governor
1. S.A.Dange
General.
2. Muzaffar Ahmed
3. It abolished the office of the secretary of
state for India. 3. Nalini Gupta

Which of the statements given above is/are 4. Shaukat Usmani


correct? Select the correct answer using the code
(a) 1 only given below.
(b) 2 and 3 only (a) 1 and 2 only
(c) 1 and 3 only
(b) 2 and 3 only
(d) 1, 2 and 3
(c) 1 and 4 only
(d) 1, 2, 3 and 4
50. Mission 50K-EV4ECO, to promote the
growth and development of value chain of
India’s electric vehicle industry has been 53. Subhash Chandra Bose set up two
launched by: headquarters of Indian National Army
(a) Indian Renewable Energy Development (INA). One was at Rangoon and the other
Agency Limited (IREDA)
one was at
(b) Small Industries Development Bank of
(a) Singapore
India (SIDBI)
(b) Tokyo
(c) NITI Aayog
(d) The Ministry of Road Transport and (c) Mauritius

Highways (d) Malaya


10 www.visionias.in ©Vision IAS

FREE BY KING R QUEEN P [ऋषभ राजपूत]


54. Consider the following statements with 57. Tolstoy farm, the precursor of the later
regard to the C-Veda project: Gandhian Ashrams was set up during which
1. It aims to investigate environmental and among the following events?
genetic risk factors affect brain function (a) Issue of the bill to disenfranchise
in industrialized countries.
Indians.
2. It is a project jointly launched by India
(b) Legislation making it compulsory for
and the UK.
Indians to carry certificates of
Which of the statements given above is/are
registration.
correct?
(c) Indians from Natal crossing the frontier
(a) 1 only
(b) 2 only into Transvaal to defy the new

(c) Both 1 and 2 immigration laws.

(d) Neither 1 nor 2 (d) Invalidation of marriages not conducted


according to Christian rites.
55. Which of the social reformers is popularly
known as Lokahitwadi? 58. What is the significance of the Haripura
(a) Narayan Malhar Joshi Session of the Indian National Congress
(b) Gopal Hari Deshmukh
(INC) in 1938?
(c) Gopal Ganesh Agarkar
(a) It was the first time that INC session was
(d) Jyotibai Phule
held in a village.
(b) National Planning Committee was setup
56. With reference to legislative reforms
under the leadership of Jawaharlal
introduced by the Indian Councils Act of
1861, consider the following statements: Nehru.

1. The act for the first time separated the (c) The Independence Resolution was

legislative and executive functions of the adopted by INC.


British government in India. (d) New Constitution for INC was framed.
2. The act provided for a majority of non-
official members in the Governor 59. SuperBIT, recently seen in the news, is a:
General Executive Council for (a) supercomputer used for bitcoin mining.
legislative purposes. (b) state-of-the-art scientific instrument
3. The Central Legislative Council
designed to measure precise weak
constituted under this act did not have
gravitational lensing of galaxy clusters.
the right to vote on the Budget.
(c) decentralised blockchain platform used
Which of the statements given above is/are
to build decentralised apps and smart
correct?
contacts among others.
(a) 1 and 2 only
(b) 2 and 3 only (d) secure communication technology that

(c) 3 only uses quantum physics to construct a


(d) 1 and 3 only cryptographic protocol.
11 www.visionias.in ©Vision IAS

FREE BY KING R QUEEN P [ऋषभ राजपूत]


60. With reference to the revolt of 1857, 63. What is Wagner Group, recently seen in the
consider the following pairs: news?
Place of revolt Leader (a) It is a group of para-military forces of
1. Kanpur : Nana Saheb Somalian fighting armies.
2. Bareilly : Bakht Khan (b) It is a grouping of the nuclear-free zone
3. Delhi : Khan Bahadur of North Africa.
4. Lucknow : Birjis Qadir (c) It is a network of contractors that supply
How many pairs given above are correctly
soldiers for hire.
matched?
(d) It is a voluntary export control regime of
(a) Only one pair
conventional weapons and dual-use
(b) Only two pairs
goods and technologies.
(c) Only three pairs
(d) All four pairs
64. Consider the following statements regarding
the Provincial Elections and formation of
61. Consider the following statements with
popular Ministries in Provinces, 1937:
regard to National Health Claims Exchange
(HCX): 1. Congress ministries were formed in July
1. It is an initiative under the Ayushman 1937 in ten out of eleven provinces.
Bharat Digital Mission that aims to bring 2. In Punjab, the Muslim League and
interoperability to health claims. Krashak Praja Party came into a
2. It has been announced by the National coalition to form the government.
Health Authority. 3. Gandhiji campaigned for the Congress
Which of the statements given above is/are during the elections which led to
correct? massive support to the Congress.
(a) 1 only Which of the statements given above is/are
(b) 2 only correct?
(c) Both 1 and 2 (a) 1 and 3 only
(d) Neither 1 nor 2 (b) 2 and 3 only
(c) 2 only
62. Consider the following statements with
(d) None
reference to the VAIBHAV Fellowship’ for
NRI researchers:
65. Consider the following statements regarding
1. It has been launched by the Ministry of
Montevideo Maru, seen in the news recently:
External Affairs.
1. It was a Japanese cargo ship that was
2. To avail of the benefits applicant should
used to transport prisoners of war and
be NRI or PIO or OCI.
civilians during World War II.
3. An applicant must have obtained a
Ph.D./M. D/M.S degree from a 2. It was destroyed and sunk by a US
recognized University. submarine in the South China Sea.
Which of the statements given above is/are Which of the statements given above is/are
correct? correct?
(a) 1 only (a) 1 only
(b) 1 and 2 only (b) 2 only
(c) 2 and 3 only (c) Both 1 and 2
(d) 1 and 3 only (d) Neither 1 nor 2
12 www.visionias.in ©Vision IAS

FREE BY KING R QUEEN P [ऋषभ राजपूत]


66. Consider the following statements regarding 69. Consider the following statements with
the Unified Portal of the Central Bureau of respect to Cripps Proposal, 1942:
Narcotics: 1. Sir Stafford Cripps, the head of the
1. It is an initiative of the Ministry of mission, strongly supported the Indian
Home Affairs. National Movement.
2. It is a portal to track the illicit trade of 2. It aimed for the earliest possible
drugs and psychotropic substances. realization of self-government in India.
Which of the statements given above is/are Which of the statements given above is/are
correct? correct?
(a) 1 only (a) 1 only

(b) 2 only (b) 2 only


(c) Both 1 and 2
(c) Both 1 and 2
(d) Neither 1 nor 2
(d) Neither 1 nor 2

70. Consider the following statement, with


67. Consider the following statements with
reference to the Paramahansa Mandali:
reference to the All India States’ People’s
1. It aimed at fighting idolatry and the caste
Conference (AISPC):
system.
1. The convening of AISPC was initiated
2. It was founded by Dadoba Pandurang
by Balwantrai Mehta, Maniklal Kothari
and Mehtaji Durgaram.
and G.R. Abhayankar.
Which of the statements given above is/are
2. The first session of AISPC was at
correct?
Lucknow.
(a) 1 only
Which of the statements given above is/are (b) 2 only
correct? (c) Both 1 and 2
(a) 1 only (d) Neither 1 nor 2
(b) 2 only
(c) Both 1 and 2 71. With reference to the communal policies of
(d) Neither 1 nor 2 the British during the Civil Disobedience
Movement, consider the following
68. Which of the following issues were taken up statements:
by Congress during the Indian General 1. The Congress while strongly disagreeing
Elections of 1945? with the Communal Award decided to
1. Brutal repression of the 1942 movement reject it.
2. Equal representation of Muslims and 2. According to the Poona Pact, more than
Hindus in the Constituent Assembly fifteen percent of the total seats in the
3. Fate of Indian National Army prisoners Central Legislature were reserved for the
of war depressed classes.
Select the correct answer using the code Which of the statements given above is/are
given below. correct?
(a) 1 and 2 only (a) 1 only
(b) 2 only (b) 2 only
(c) 1 and 3 only (c) Both 1 and 2

(d) 3 only (d) Neither 1 nor 2


13 www.visionias.in ©Vision IAS

FREE BY KING R QUEEN P [ऋषभ राजपूत]


72. The Nehru Report of 1928 demanded which 75. Consider the following statements regarding
of the following? Chagas disease (American trypanosomiasis):
1. Reservation for Muslims in all the 1. It is a communicable parasitic disease
provinces which remains most prevalent in the
2. Equal rights for women Americas.
3. Freedom to form unions 2. The disease propagates by casual contact
4. Dissociation of the state from religion in with infected humans or animals.
any form 3. There are currently no vaccines available
Select the correct answer using the code for the disease.
given below. Which of the statements given above is/are
(a) 1, 2 and 3 only correct?
(b) 1 and 4 only (a) 1 only
(c) 2, 3 and 4 only (b) 1 and 3 only
(d) 1, 2, 3 and 4 (c) 2 and 3 only
(d) 1, 2 and 3
73. With reference to Indian history, consider
the following pairs: 76. Consider the following statements with
Movement/Incident Viceroy respect to the Jallianwala Bagh massacre:
1. Partition of Bengal : Lord Curzon 1. Large crowd gathered at Jallianwalla
2. Communal Award : Lord Irvin Bagh to protest against the arrest of Dr.
3. Chauri Chaura : Lord Reading Saifuddin Kitchlew and Dr. Satyapal.
4. Ghadar Mutiny : Lord Chelmsford 2. After the incident, Mahatma Gandhi
How many pairs given above are correctly
formed Satyagraha Sabha to organize an
matched?
all-India hartal against the massacre.
(a) Only one pair
Which of the statements given above is/are
(b) Only two pairs
correct?
(c) Only three pairs
(a) 1 only
(d) All four pairs
(b) 2 only
(c) Both 1 and 2
74. On 18 October 1939, he spoke in the House
(d) Neither 1 nor 2
of Lords and stressed differences among
Indians, especially among Hindus and
77. With reference to Woods Despatch of 1854,
Muslims. He branded the Congress as a
consider the following statements:
purely 'Hindu organization’. With
1. It was intended to resolve the
Churchill's assumption of the Prime
Orientalist-Anglicist controversy with
Ministership in 1940, he then offered his
regard to education in India.
resignation, feeling that his ideas and
2. It directed the British government to
Churchill's regarding India were so different
assume the responsibility of mass
that he remarked "I could only end by
education.
becoming an embarrassment to him."
Which of the statements given above is/are
Who among the following personalities is
correct?
discussed in the passage given above?
(a) Lord Willingdon (a) 1 only
(b) Lord Linlithgow (b) 2 only
(c) Lord Zetland (c) Both 1 and 2
(d) Lord Wavell (d) Neither 1 nor 2
14 www.visionias.in ©Vision IAS

FREE BY KING R QUEEN P [ऋषभ राजपूत]


78. With reference to different opinions on the 81. Consider the following statements with
question of Indian support to British war regard to the Shahu, grandson of Shivaji:
efforts in World War II, consider the 1. He had been a prisoner in the hands of
following statements: Aurangzeb for more than 15 years.
1. Jawahar Lal Nehru advocated 2. He had a conflict with his aunt Tarabai
unconditional support to the Allied over the seat of power.
powers. 3. He made Balaji Vishwanath as his
2. Subhas Bose and other socialists leaders Peshwa.
were of the view to take advantage of Which of the statements given above is/are
the situation of World War II. correct?
3. Mahatma Gandhi was of the view that (a) 1 only
no Indian participation in World War II (b) 1, 2 and 3
and at the same, no opportunistic view (c) 3 only
of the situation either. (d) 2 and 3 only
Which of the statements given above is/are
correct? 82. With reference to the Simon Commission
(a) 1 only 1927, which of the following was the
(b) 2 only primary reason for its boycott by most of the
(c) 1 and 3 only important leaders and parties of India?
(d) 2 and 3 only (a) It proposed for an increase in British
personnel in the armed forces
79. In 1722, the autonomous kingdom of Avadh (b) It accepted the idea of separate
was founded by which of the following communal electorates in India.
rulers? (c) It violated principle of self-
(a) Saadat Khan Burhan-ul-Mulk determination as all members of the
(b) Alivardi Khan commission were Englishmen.
(c) Safdar Jung (d) It proposed to annul the system of
(d) Shuja ud Daula Dyacrchy in British provinces in India.

80. With reference to the Congress Working 83. Consider the following statements regarding
Committee resolution of 12th February 1922 the proposals made under the Wavell Plan:
popularly known as the Bardoli resolution, 1. With the exception of the governor-
which one of the following statements is general and the commander-in-chief, all
correct? members of the executive council were
(a) The Bardoli Satyagraha was launched to be Indians.
for the farmers of Bardoli against the 2. Caste Hindus and Muslims were to have
unjust raising of taxes. equal representation.
(b) The Congress met and accepted non- Which of the statements given above is/are
cooperation as its own. correct?
(c) C. R.Das moved the main resolution on (a) 1 only
non-cooperation. (b) 2 only
(d) Peasants were asked to pay taxes and (c) Both 1 and 2
tenants to pay rents. (d) Neither 1 nor 2
15 www.visionias.in ©Vision IAS

FREE BY KING R QUEEN P [ऋषभ राजपूत]


84. All-India Tilak Memorial Swaraj Fund was 87. With reference to British colonial rule in
created to raise funds for which among the India, the term ‘Damin-i-koh’ appears in the
following movements? context of
(a) Swadeshi Movement (a) Commission paid by the British East
(b) Non Cooperation Movement India Company to its agents

(c) Civil Disobedience Movement (b) Temporary facilities in the British

(d) Quit India Movement factories for storage of foods grains


(c) Procedure of auctioning the estates of
Zamindars in case of arrear of revenue
85. With reference to the Rajkot Satyagraha,
(d) Area demarcated by the British for
consider the following statements:
settlement of cultivators
1. It started against the British
government’s attempt of selling off
88. Consider the following statements regarding
monopolies for the sale of matches,
the Vernacular Press Act, 1878:
sugar, rice, and cinema licenses to
1. It was directed only against the Indian
individual merchants. language newspapers.
2. The main demands included responsible 2. It provided for the confiscation of the
government, reduction in taxes and state printing press, paper and other materials
expenditure. of a newspaper if the Government
3. It was led by Jethalal Joshi and UN believed that it was publishing seditious
Dhebar. materials.
Which of the statements given above is/are 3. It was repealed in 1881 by Lord Ripon.
correct? Which of the statements given above is/are
(a) 2 only correct?
(b) 1 and 2 only (a) 1 and 2 only
(c) 2 and 3 only (b) 3 only

(d) 1 and 3 only (c) 2 and 3 only


(d) 1, 2 and 3

86. Consider the following statements with


89. Which of the following proposals was/were
regard to the Dutch East India Company:
part of the August Offer?
1. The company established trading depots
1. Promise of the expansion of the
at Surat and Ahmedabad in West India.
Executive Council.
2. The Dutch parliament empowered the
2. A dominion status for Indian Union.
company to conclude treaties on its
3. No future constitution to be adopted
own.
without the consent of minorities.
Which of the statements given above is/are Select the correct answer using the code
correct? given below.
(a) 1 only (a) 1 and 2 only
(b) 2 only (b) 2 and 3 only
(c) Both 1 and 2 (c) 3 only
(d) Neither 1 nor 2 (d) 1, 2 and 3
16 www.visionias.in ©Vision IAS

FREE BY KING R QUEEN P [ऋषभ राजपूत]


90. With reference to Indian history, which of 93. With reference to the Ghadar movement,
the following predecessors of the Indian consider the following statements:
National Congress was established the
1. It was organized by overseas Indian
earliest?
immigrants to Canada and the USA.
(a) Bengal British Indian Society
2. The founding president of the Ghadar
(b) Poona Sarvajanik Sabha
(c) Landholder's Society party was Sohan Singh Bhakna.
(d) Madras Native Association 3. The party weekly newspaper The
Ghadar first issue was published in
91. With reference to the Animal Pandemic
Gurumukhi.
Preparedness Initiative (APPI), consider the
Which of the statements given above is/ are
following statements:
correct?
1. It will create an integrated disease
reporting system for enhanced (a) 1 only
surveillance of zoonotic diseases. (b) 2 and 3 only
2. It has been launched by the Ministry of (c) 1 and 2 only
Health and Family Welfare under One
(d) 1, 2 and 3
Health Approach.
Which of the statements given above is/are
94. Consider the following statements with
correct?
(a) 1 only reference to the farman of 1717:
(b) 2 only 1. It was issued during the reign of the
(c) Both 1 and 2 Mughal emperor Farrukh Siyar.
(d) Neither 1 nor 2
2. The farman granted freedom to both the
East India Company and its officials to
92. With reference to the changes made to
get engaged in tax-free trade.
Congress Constitution after the 1920 Nagpur
session, consider the following statements: Which of the statements given above is/are
1. Provincial Congress Committees were correct?
now to be organized on a linguistic (a) 1 only
basis.
(b) 2 only
2. The Congress was now to have a
(c) Both 1 and 2
Working Committee of fifteen members
to look after its day-to-day affairs (d) Neither 1 nor 2

3. The requirement to pay an annual


membership fee was done away with to 95. Which of the following acts introduced the
enable the poor to become members. system of open competitive examination for
Which of the statements given above are
civil services during British Indian rule?
correct?
(a) Charter Act of 1813
(a) 1 and 2 only
(b) Charter Act of 1853
(b) 2 and 3 only
(c) 1 and 3 only (c) Indian Councils Act of 1861
(d) 1, 2 and 3 (d) Government of India Act 1858
17 www.visionias.in ©Vision IAS

FREE BY KING R QUEEN P [ऋषभ राजपूत]


96. With reference to the CLEANaction, 99. With reference to the terms of the Gandhi-
consider the following statements: Irwin Pact, consider the following
1. It is a partnership to protect nature statements:
during the energy transition.
1. The agreement included the immediate
2. It is a clean energy coalition of the
return of all lands confiscated from the
United Nations Environment
Programme. peasants.
Which of the statements given above is/ are 2. The Government also conceded the right
correct? to make salt for consumption to villages
(a) 1 only along the coast.
(b) 2 only
Which of the statements given above is/are
(c) Both 1 and 2
correct?
(d) Neither 1 nor 2
(a) 1 only
97. Khongjom Day is observed to pay homage (b) 2 only
to: (c) Both 1 and 2
(a) the martyrs of Anglo-Manipur war. (d) Neither 1 nor 2
(b) Rani Gaidinliu, a famous personality of
the Indian independence movement.
100. Which one of the following changes was not
(c) those who sacrificed their lives against
introduced by Tipu Sultan?
Japanese aggression in the second world
war. (a) new calendar
(d) the freedom fighters of the Khasi Hills. (b) new system of coinage
(c) new scales of weights and measures
98. Consider the following statements, with (d) new jagirdari system
reference to the doctrine of lapse:
1. It was introduced by Lord Dalhousie.
2. According to this doctrine, if an Indian
ruler died without a natural heir, his
kingdom would lapse to the British
Empire.
3. Awadh was annexed because the prince
died without a natural heir.
Which of the statements given above are
correct?
(a) 1 and 2 only
(b) 1 and 3 only
(c) 2 and 3 only
(d) 1, 2 and 3

Copyright © by Vision IAS


All rights are reserved. No part of this document may be reproduced, stored in a retrieval system or
transmitted in any form or by any means, electronic, mechanical, photocopying, recording or otherwise,
without prior permission of Vision IAS.
18 www.visionias.in ©Vision IAS

FREE BY KING R QUEEN P [ऋषभ राजपूत]


VISIONIAS
www.visionias.in
ANSWERS & EXPLANATIONS
GENERAL STUDIES (P) TEST – 4131 (2024)

Q 1.B
• Sayyid Ahmad Khan was one of the most important reformers among Muslims. He was tremendously
impressed by modern scientific thought and worked all his life to reconcile it with Islam.
• The Muslim upper class tended to avoid contact with Western education and culture, and it was mainly
after the revolt of 1857 that modern ideas of education and religious reform began to appear. A beginning
in this direction was made when the Mohammedan literary society was founded at Calcutta in 1863.
• In 1875 he founded at Aligarh the Muhammedan Anglo-Oriental College at Aligarh as a center for
spreading Western science and culture, later this college grew into an Aligarh Muslim university. Hence
statement 1 is not correct.
• In his view, any interpretation of the Quran that conflicted with human reason, science, or nature was in
reality a misinterpretation. He also interpreted Quran in the light of contemporary rationalism and
science. Hence statement 2 is correct.
• He was opposed to communal friction, appealing to Hindus and Muslims to unite, he was a great believer
in religious toleration.

Q 2.D
• 'The Philosophy of the Bomb' was written by Bhagwati Charan Vohra in early January 1930 as a
polemical intervention into debates among nationalist circles about the role of violence in the anti-
colonial movement in India.
• Bhagwati Charan Vohra was part of the Hindustan Socialist Republican Association (HSRA), and he
also authored the manifesto of HSRA.
• Hence, option (d) is the correct answer.

Q 3.A
• The Government of India Act of 1858 ended the Company rule and the system of Double Government
by the Board of Control in England and the Court of Directors of the company introduced by the Pitt's
India Act, of 1784. Indian Administration came directly under the Crown. The Act created the office of
the Secretary of State who was a cabinet minister in the British cabinet. His salary and establishment
were paid from the Indian revenue. He was assisted by a council of fifteen members to make him familiar
with Indian affairs.
• The Secretary of State was given the power of sending and receiving secret messages and despatches from
the Governor General without the necessity of communicating them to the Indian Council. The Secretary
of State was to present to the House of Commons periodically report on the moral and material progress
of India. Hence, statement 1 is correct.
• As per the administrative changes introduced by the Government of India Act 1858, the Governor
General got an additional title of Viceroy or Crown's representative. Thus, the Governor-general
served as the personal representative of the British Crown in India. The Secretary of State sat in the
British Parliament and was accountable to the Parliament. Hence, statement 2 is not correct.

Q 4.C
• In pursuance of his reactionary policies in India Lord Curzon announced the partition of Bengal in
December 1903 which took effect on 16 October 1905. A massive anti-partition movement was
started in Bengal which evolved into the Swadeshi and Boycott movement. The formal proclamation
of the Swadeshi Movement was, made on 7 August 1905, in a meeting held at the Calcutta town hall. At
this meeting, the famous Boycott Resolution was passed. Hence statement 1 is correct.
1 www.visionias.in ©Vision IAS

FREE BY KING R QUEEN P [ऋषभ राजपूत]


• Samitis (corps of volunteers) helped in the mobilization of the mass to support the Swadeshi
Movement. The samitis took the swadeshi message to the villages through magic lantern lectures and
swadeshi songs, gave physical and moral training to the members, did social work during famines and
epidemics, organized schools, trained in the swadeshi craft, and organized arbitration courts. The
Swadesh Bandhab Samiti of Ashwini Kumar Dutt is such a prominent example. Hence statement 3
is correct.
• Indian National Congress took up the Swadeshi call and at the 1905 Banaras Session, presided over by
G.K. Gokhale, a resolution supporting the Swadeshi and Boycott Movement for Bengal was
passed. During Swadeshi Movement, the big zamindars, who had so far remained loyal to the Raj,
joined forces with the Congress leaders in support of the movement. Hence statement 2 is not
correct.
• A very important characteristic of the Movement is the active participation of women in the movement, it
marked the first instance of women participating in large numbers in the national movement. The woman
of the urban middle class came out in large numbers and participated in the Swadeshi movement they
carried out positions and picketing work.

Q 5.A
• Bengalee was a newspaper started by Surendranath Banerjee in 1879. Sanjibani (by Krishna Kumar
Mitra) and Hitabadi (by Dwijendranath Tagore) were journals. A powerful press campaign was Launched
against the partition proposals through these journals and newspapers
• Hence option (a) is the correct answer.

Q 6.B
• To discourage and stop students to participate in the Civil Disobedience Movement, J.R.
Cunningham, the then powerful Director of Public Information of Assam, issued the Cunningham
circular in 1930 imposing a blanket ban on any anti-British and pro-swadeshi activity by students.
• In Assam, a powerful agitation led by students was launched against the infamous ‘Cunningham circular’
which forced students and their guardians to furnish assurances of good behavior.
• It forced parents, guardians and students to furnish assurances of good behavior and also asked them to
sign an undertaking that they would have to quit their schools and colleges if they participated in anti-
government demonstrations or movements.
• The circular was aimed to forbid students from participating in political activities and so, it raised a very
strong response.
• Thousands of students, throughout Assam, left their educational institutions. Many Swadeshi educational
institutions like Kamrup Academy of Guwahati and Sibsagar Vidyapeeth were also established at the
same time to accommodate the students leaving British Government schools & colleges.
• The Kamrup Academy, the first Swadeshi school in the Northeast, was conceived with Rai Bahadur
Kalicharan Sengupta as its president and Gaurikanta Talukdar as its secretary.
• The school started functioning on July 15, 1930, and initially, lessons were imparted in both Bengali and
Assamese languages. Hence, option (b) is the correct answer.

Q 7.C
• The policy of the Indian National Congress towards the Indian states had been first enunciated in
1920 at Nagpur when a resolution calling upon the Princes to grant full responsible government in
their States had been passed. Simultaneously, however, the Congress, while allowing residents of the
States to become members of Congress, made it clear that they could not initiate political activity in
the States in the name of Congress but only in their individual capacity or as members of the local
political organizations.
• In the mid-thirties, two associated developments brought about a distinct change in the situation in the
Indian States.
o First, the Government of India Act of 1935 projected a scheme of federation in which the Indian
States were to be brought into a direct constitutional relationship with British India and the States
were to send representatives to the Federal Legislature.
o The second development was the assumption of office by Congress Ministries in the majority of
the provinces in British India in 1937. The fact that Congress was in power created a new sense of
confidence and expectation in the people of the Indian States and acted as a spur to greater political
activity. The Princes too had to reckon with a new political reality — the Congress was no longer just

2 www.visionias.in ©Vision IAS

FREE BY KING R QUEEN P [ऋषभ राजपूत]


a party in opposition but a party in power with a capacity to influence developments in contiguous
Indian States.
• The years 1938-39, in fact, stand out as years of a new awakening in the Indian States and were witness to
a large number of movements demanding responsible government and other reforms. Praja mandals
mushroomed in many States that had earlier no such organizations. Major struggles broke out in Jaipur,
Kashmir, Rajkot, Patiala, Hyderabad, Mysore, Travancore, and the Orissa States.
• These new developments brought about a significant change in Congress policy as well. Whereas,
even in the Haripura session in 1938, the Congress had reiterated its policy that movements in the
States should not be launched in the name of the Congress but should rely on their own independent
strength and fight through local organizations, a few months later, on seeing the new spirit that was
abroad among the people and their capacity to struggle. Gandhiji and the Congress changed their attitude
on this question. The radicals and socialists in Congress, as well as political workers in the States, had in
any case been pressing for this change for quite some time.
• Following upon this, the Congress at Tripuri in March 1939 passed a resolution enunciating its new
policy: ‘The great awakening that is taking place among the people of the States may lead to a relaxation,
or to a complete removal of the restraint which the Congress imposed upon itself, thus resulting in
an ever-increasing identification of the Congress with the States’ peoples’. Also in 1939, the AISPC
elected Jawaharlal Nehru as its President for the Ludhiana session, thus setting the seal on the fusion of
the movements in Princely India and British India.
• Hence option (c) is the correct answer.

Q 8.A
• Animal Birth Control Rules, 2023: The government has announced new Animal Birth Control (ABC)
Rules 2023, which require strays to be caught, vaccinated, neutered, and released back into the
community. The ABC 2023 rules will supersede the Animal Birth Control (Dog) Rules, 2001. Hence,
statement 1 is correct.
o To reduce the stray dog population by addressing animal welfare issues.
o The program for the sterilization and immunization of stray dogs shall be carried out by
the respective local bodies, municipalities, municipal corporations, and panchayats, with the
help of the Animal Welfare Board of India (AWBI)-a recognized organization.
o Local bodies need to implement the ABC and Anti-Rabies Program jointly.
o In carrying out the ABC program, ‘cruelty to animals should be addressed.
o The Rules transform stray dogs into a new class of “community animals.” Hence, statement 2 is
correct.
o Resident welfare associations are responsible for caring for stray dogs and feeding them at fixed
intervals, away from children and the elderly. Hence, statement 3 is correct.
o The Rules provide guidelines on how to deal with human and stray dog conflicts without relocating
the dogs in an area.
o The Rules prohibit the improper disposal of solid waste and casual feeding of dogs.
o Local authorities “will be held responsible for any violation (during birth-control procedures) and
animal-human conflicts.”

Q 9.C
• Gandhi's first great experiment in satyagraha came in 1917 in Champaran, a district in Bihar. The
peasantry on the indigo plantations in the district was excessively oppressed by the European
planters. They were compelled to grow indigo on at least 3/20th of their land and to sell it at prices fixed
by the planters. In the Champaran Satyagraha, the Government appointed a Commission of Inquiry to go
into the whole issue and nominated Gandhiji as one of its members. Armed with evidence collected from
8,000 peasants, he had little difficulty in convincing the Commission that the tinkathia system needed to
be abolished and that the peasants should be compensated for the illegal enhancement of their dues. As a
compromise with the planters, he agreed that they refund only twenty-five percent of the money
they had taken illegally from the peasants. Answering critics who asked why he did not ask for a full
refund, Gandhiji explained that even this refund had done enough damage to the planters’ prestige and
position. As was often the case, Gandhiji’s assessment was correct and, within a decade, the planters left
the district altogether. Hence, option (c) is the correct answer.

3 www.visionias.in ©Vision IAS

FREE BY KING R QUEEN P [ऋषभ राजपूत]


Q 10.A
• Recent context: More than 181 members of the Hakki Pikki tribal community from Karnataka are
stuck in violence-hit Sudan, even as the government is making efforts to bring them back.
• The Hakki Pikki is a tribe that lives in several states in the west and south India, especially near forest
areas. Hakki Pikkis (Hakki in Kannada means ‘bird’ and Pikki means ‘catchers’) are a semi-nomadic
tribe, traditionally of bird catchers and hunters.
• According to the 2011 census, the Hakki Pikki population in Karnataka is 11,892, and they live majorly in
Davangere, Mysuru, Kolar, Hassan and Shivmogga districts. In different regions, they are known by
different names, such as Mel-Shikari in northern Karnataka and Maharashtra.
• The Hakki Pikki move in groups from place to place in search of livelihood. They are divided into four
clans, called Gujaratia, Panwar, Kaliwala and Mewaras. These clans can be equated with castes in
traditional Hindu society. In the olden days, there was a hierarchy among the clans, with the Gujaratia at
the top and the Mewaras at the bottom. The forest is the main natural resource of the Hakki Pikki.
• Hakki Pikki people are believed to hail originally from the bordering districts of Gujarat and Rajasthan.
• Hakki Pikkis in Karnataka follow Hindu traditions and celebrate all Hindu festivals. They are non-
vegetarians. The eldest son in a family is not supposed to cut his hair so that he can be identified easily.
• The tribe prefers cross-cousin marriages.
• Hence option (a) is the correct answer.

Q 11.B
• The Charter Act of 1813 was passed by the British Parliament due to increasing pressure from the British
industrialists who wanted to take part in the profitable Indian trade.
• By the Charter Act of 1813, the trade monopoly of the Company in India was ended and trade with
India was thrown open to all British subjects (private merchants). But trade in tea and trade with
China was still exclusive to the Company. Hence option 1 is correct.
• A humble beginning for the spread of education was made in 1813 through the Charter Act. It
incorporated the principle of encouraging learned Indians and promoting the knowledge of modern
sciences in the country. The Act directed the Company to spend the sum of one lakh of rupees for
the purpose. But even this petty amount was not made available by Company authorities till 1823. Hence
option 2 is correct.
• The Charter Act of 1833 brought the Company’s monopoly of tea trade and trade with China to an
end. At the same time, the debts of the Company were taken over by the Government of India, which
was also to pay its shareholders a 10½ percent dividend on their capital. Hence option 3 is not correct.

Q 12.B
• The issue which caught the popular imagination was the fate of the members of Subhas Chandra
Bose’s Indian National Army (INA), who were captured by the British in the eastern theatre of
War.
• An announcement by the Government, limiting trials of the INA personnel to those guilty of brutality or
active complicity, was due to be made by the end of August 1945.
• However, before this statement could be issued. Nehru raised the demand for leniency at a meeting in
Srinagar on 16 August 1945 — making the proposed statement seem a response to his call rather than an
act of generosity on the part of the Government.
• The defense of the INA prisoners was taken up by the Congress and Bhulabhaj Desai, Tej Bahadur
Sapru, K.N. Katju, Nehru, and Asaf All appeared in court at the historic Red Fort trials.
• The Congress organized an INA Relief and Enquiry Committee, which provided small sums of
money and food to the men on their release, and attempted, though with marginal success, to secure
employment for these men. Hence, statement 1 is not correct and statement 2 is correct.
• A significant feature of the INA campaign was its wide geographical reach and the participation of
diverse social groups and political parties.
• The Muslim League, the Communist Party of India, the Unionist Party, the Akalis, the Justice Party, the
Abrars in Rawalpindi, the Rashtriya Swayamsevak Sangh, the Hindu Mahasabha, and the Sikh League
supported the NA cause in varying degrees.

4 www.visionias.in ©Vision IAS

FREE BY KING R QUEEN P [ऋषभ राजपूत]


Q 13.B
• Francis Buchanan was a physician who came to India and served in the Bengal Medical Service (from
1794 to 1815). For a few years, he was a surgeon to the Governor-General of India, Lord Wellesley.
• During his stay in Calcutta (present-day Kolkata), he organized a zoo that became the Calcutta Alipore
Zoo.
• He was also in charge of the Botanical Gardens for a short period. At the request of the Government of
Bengal, he undertook detailed surveys of the areas under the jurisdiction of the British East India
Company. However, he did not establish any institution in India for the study of Indian
traditions/philosophy. Hence option 1 is not correct.
• Jonathan Duncan served as a resident of the East India Company in the late 18th century. He started
the Sanskrit College in Varanasi in 1791 for the study of Hindu law and philosophy. Hence option 2
is correct.
• Warren Hastings was the governor-general of Bengal between 1772-1785. He established the Calcutta
in 1781 for the study of Muslim law and related subjects. The Calcutta Madrasah and the Sanskrit
College were designed to provide a regular supply of qualified Indians to help the administration of law in
the Company’s court, and the knowledge of classical languages and vernaculars was useful in
correspondence with Indian states. Hence option 3 is correct.
• Hence, option (b) is the correct answer.

Q 14.C
• Marketing and Logistics Development for Promotion of Tribal Products from North-Eastern
Region (PTP-NER) scheme:
o Aim: It is a Central Sector Scheme which aims to strengthen livelihood opportunities for tribal
artisans through increased efficiency in procurement, logistics and marketing of tribal products from
North Eastern States. Hence, statement 1 is not correct.
o States covered: The scheme will apply to the states of Arunachal Pradesh, Assam, Manipur,
Meghalaya, Mizoram, Nagaland, Tripura and Sikkim.
o Ministry of Tribal Affairs is the nodal ministry and Tribal Cooperative Marketing Development
Federation (TRIFED) is the Nodal Agency.
o Key Features of the scheme: The scheme will facilitate tribal artisans to avail opportunities of
enhancing income through the provision of backward and forward linkages through incubation
support, aggregation, skill and entrepreneurship development, sourcing and procurement, marketing,
transportation and publicity.
o The Central government will organize Tribal Artisans Melas (TAMs) in April and May to showcase
tribal products. Hence, statement 2 is not correct.

Q 15.B
• Lucknow Session of the Indian National Congress (December 1916) was very important for the Indian
freedom struggle. The session produced two historic developments.
o First, the two wings of the Congress that is the Moderates and the Extremist were reunited.
o Second, the session also became significant because of the famous Congress-League Pact,
popularly known as the Lucknow Pact wherein the two organisations overcame their old
differences and put out a set of common political demands before the government. An important
role in bringing the two together was played by Lokmanya Tilak and Muhammad Ali
Jinnah because the two believed that India could win self-government only through Hindu Muslim
unity.
o Together Congress and the League passed the same resolutions at their sessions and put forward
a joint scheme of political reforms based on separate electorate and demand that the British
government should make a declaration that it would consider self-government in India at an early
date. Hence statement 1 is correct.
o The immediate effect of the signing of Lucknow Pact was that, at Lucknow was seen tremendous
unity between the moderate nationalist and the militant nationalist and between the national Congress
and the Muslim league aroused great political enthusiasm in the country. At the same time it accepted
the principle of separate electorate. Thus, it left the way open to the future resurgence of
communalism in Indian politics. Hence statement 2 is not correct.

5 www.visionias.in ©Vision IAS

FREE BY KING R QUEEN P [ऋषभ राजपूत]


Q 16.D
• Rehnumai Madaysan sabha was founded in 1851 by Naoroji Furdnji, Dadabhai Naoroji, S.S.
Bengalee, and others. It campaigned against the entrenched orthodoxy in the religious field and
initiated the modernization of Parsi social customs regarding the education of women. Hence statements
1 and 2 are correct.
• The Rahnumai Mazdayasnan Sabha had a significant influence on both the colonial milieu and the way in
which Zoroastrianism was reinterpreted by Western scholars. Hence statement 3 is correct.
• Scholars from the West studied the Parsis' sacred texts, rituals, and customs, and the knowledge they
gained formed the foundation for religious change and the education of a new generation of Parsi priests.
• It works to promote the welfare and interests of the Zoroastrian community in India and to preserve
and promote the teachings and traditions of Zoroastrianism. It also provides various services to the
community, including education,
• Hence option (d) is the correct answer.

Q 17.B
• On 4 June 1947, the Viceroy, Mountbatten, announced at a press conference that the British would
soon leave India for good on 15 August. On 12 June, the Nizam of Hyderabad announced that on the
lapse of British paramountcy, he would become a sovereign monarch. The intention was clear: he would
not accede to the Indian Union.
• The first open session of the Hyderabad State Congress which demanded accession to the Indian
Union and the grant of responsible government was held from 16 to 18 June. The State Congress, with
the full support of the Indian National Congress, had also thwarted an attempt by the Nizam a few months
earlier, to foist an undemocratic constitution on the people. The boycott of the elections launched by
them received tremendous support. With this new confidence, they began to take a bold stand against
Nizam’s moves. The decision to launch the final struggle was taken by the leaders of the State Congress
in consultation with the national leaders in Delhi.
o On 13 August, the Nizam banned the ceremonial hoisting of the national flag. As the movement
gathered force and gained momentum, the Nizam and his administration cracked down on it.
o But the most ominous development was the encouragement given to the storm troopers of the
Ittihad ul Muslimin, the Razakars, by the State to act as a paramilitary force to attack the
peoples’ struggle. Razakars were issued arms and let loose on protesting crowds; they set up camps
near rebellious villages and carried out armed raids.
o On 29 November 1947, the Nizam signed a Standstill Agreement with the Indian Government,
but simultaneously the repression was intensified, and the Razakar menace became even more
acute.
o Many thousands of people who could afford to do so fled the State and were housed in camps in
neighbouring Indian territory. In organizing the defence against the Razakars and attacks on
Razakar camps, the Communists played a very important role, especially in the areas of
Nalgonda, Warangal and Khammam that were their strongholds. Peasants were organized, given
training in arms, and mobilized for the anti-Nizam struggle. In these areas, the movement also took
an anti-landlord stance and many cruel landlords were attacked, some even killed, and illegally
occupied land was returned to the original owners. Virtually all the big landlords had run away, and
their land was distributed to and cultivated by those with small holdings or no land.
o Outside the Communist strongholds in the Telengana areas, it was the State Congress that was the
main vehicle for organizing popular resistance.
o By September 1948, it became clear that all negotiations to make the Nizam accede to the Union
had failed. On 13 September, 1948, the Indian Army moved in and on 18 September the Nizam
surrendered. The process of the integration of the Indian Union was finally complete.
o The people welcomed the Indian Army as an army of liberation, an army that ended the
oppression of the Nizam and the Razakars. Scenes of jubilation were evident all over, and the national
flag was hoisted.
• Hence option (b) is the correct answer.

Q 18.B
• Bhulabhai Desai, leader of the Congress Party in the Central Legislative Assembly, met Liaqat Ali Khan,
deputy leader of the Muslim League in that Assembly, and both of them came up with a draft proposal for
the formation of an interim government at the centre, consisting of:

6 www.visionias.in ©Vision IAS

FREE BY KING R QUEEN P [ऋषभ राजपूत]


o an equal number of persons nominated by the Congress and the League in the central legislature.
Hence, statement 1 is not correct.
o 20% reserved seats for minorities. Hence, statement 2 is correct.
• No settlement could be reached between the Congress and the League on these lines, but the fact that a
sort of parity between the Congress and the League was decided upon had far-reaching consequences.

Q 19.C
• National Quantum Mission:
o It’ll be implemented by the Department of Science & Technology (DST) under the Ministry of
Science & Technology. Hence, statement 3 is not correct.
o The mission planned for 2023-2031 aims to seed, nurture, and scale up scientific and industrial R&D
and create a vibrant & innovative ecosystem in Quantum Technology (QT).
o With the launch of this mission, India will be the seventh country to have a dedicated quantum
mission after the US, Austria, Finland, France, Canada and China.
o Salient features of NQM:
✓ It will target developing intermediate scale quantum computers with 50-100 physical qubits in 5
years and 50-1000 physical qubits in 8 years.
✓ Just like bits (1 and 0) are the basic units by which computers process information, ‘qubits’ or
‘quantum bits’ are the units of process by quantum computers.
✓ The mission will help develop magnetometers with high sensitivity for precision timing (atomic
clocks), communications, and navigation.
✓ It will also support design and synthesis of quantum materials such as superconductors, novel
semiconductor structures and topological materials for fabrication of quantum devices.
✓ The mission will also help developing:
▪ Satellite based secure quantum communications between ground stations over a range of 2000
km within India. Hence, statement 1 is correct.
▪ Long distance secure quantum communications with other countries
▪ Inter-city quantum key distribution over 2000 km
▪ Multi-node Quantum network with quantum memories
✓ Four Thematic Hubs (T-Hubs) would be set up in top academic and National R&D institutes on
the domains of Quantum Technology:
▪ Quantum computation
▪ Quantum communication
▪ Quantum Sensing & Metrology
▪ Quantum Materials & Devices. Hence, statement 2 is correct.

Q 20.B
• Recent context: Tamil Nadu’s Manamadurai pottery gets GI tag. Hence pair 3 is not correctly
matched.
o The Vaigai river enriches the clay used for the Manamadurai pottery. A unique type of clay is sourced
from water bodies like Nedunkulam, Nathapurakki, Sundaranadappu, Seikalathur to make these pots.
• Four products that are unique to Goa - bebinca, malcorada mango, seven ridge okra and Agasaim
brinjal - have been published in the journal of the Geographical Indication (Gl) Registry, which is the
final stage in the process of obtaining a Gl status for these items. Hence pair 1 is not correctly matched
and pair 2 is correctly matched.

7 www.visionias.in ©Vision IAS

FREE BY KING R QUEEN P [ऋषभ राजपूत]


Q 21.B
• Bal Gangadhar Tilak, the outstanding leader of militant nationalism. Born in 1856, Tilak devoted his
entire life to the service of his country. In 1881, along with G.G. Agarkar, he founded the newspaper
Kesari (in Marathi) and Mahratta (in English). In 1888, he took over the two papers and used their
columns to spread discontent against British rule and to preach national resistance to it. Hence statement
1 is not correct.
• In 1893, he started the practice of using the traditional religious Ganapati festival to propagate
nationalist ideas through patriotic songs and speeches. In 1896, he started the Shivaji festival to
stimulate nationalism among young Maharashtrians. In the same year, he organized an all-Maharashtra
campaign for the boycott of foreign cloth in protest against the imposition of the excise duty on
cotton. He was, perhaps the first among the national leaders to grasp the important role that the lower
middle classes, peasants, artisans and workers could play in the national movement and, therefore, he saw
the necessity of bringing them into the Congress fold. Hence statement 3 is correct.
• In pursuance of this objective, he initiated a no-tax campaign in Maharashtra during 1896-97 with
the help of the young workers of the Poona Sarvajanik Sabha. Referring to the official famine code
whose copies he got printed in Marathi and distributed by the thousand, he asked the famine-stricken
peasants of Maharashtra to withhold payment of land revenue if their crops had failed.
• In 1897, plague broke out in Poona and the Government had to undertake severe measures of
segregation and house- searches. Unlike many other leaders, Tilak stayed in Poona, supported the
Government and organized his own measures against the plague. But he also criticized the harsh and
heartless manner in which the officials dealt with the plague-stricken people. Popular resentment
against the official plague measures resulted in the assassination of Rand, the Chairman of
the Plague Committee in Poona, and Lt. Ayerst by the Chaphekar brothers on 27 June 1898.
o The British- owned Press and the bureaucracy were quick to portray the Rand murder as a
conspiracy by the Poona Brahmins led by Tilak. The Government investigated the possibility of
directly involving Tilak in Rand’s assassination. But no proof could be found. And so,
the Government decided to arrest him under Section 124A of the Indian Penal Code on the
charge of sedition, that is, spreading disaffection and hatred against the Government. Hence
statement 2 is correct.
o Tilak was arrested on 27 July 1879 and tried before Justice Strachey and a jury of six Europeans and
three Indians. The charge was based on the publication in the Kesari of 15 June of a poem titled
‘Shivaji’s Utterances’ ‘read out by a young man at the Shivaji Festival and on a speech Tilak had
delivered at the Festival in defense of Shivaji’s killings of Afzal Khan.
o Tilak’s defense of Shivaji’s killing of Afzal Khan was portrayed by the prosecution as an incitement
to kill British officials. The overall accusation was that Tilak propagated the views in his newspaper,
that the British had no right to stay in India and any and all means could be used to get rid of them.
o The jury gave a 6 to 3 verdict holding Tilak guilty, the three dissenters being its Indian members. The
Judge passed a barbarous sentence of rigorous imprisonment for eighteen months, and this when Tilak
was a member of the Bombay Legislative Council. Simultaneously several other editors of the
Bombay Presidency were tried and given similar harsh sentences.
o Tilak’s imprisonment led to widespread protests all over the country
• In 1898, the Government amended Section 124A and added a new Section 153A to the penal code,
making it a criminal offence for anyone to attempt ‘to bring into contempt’ the Government of India or to
create hatred among different classes, that is vis-a-vis Englishmen in India. This once again led to nation-
wide protest.

Q 22.A
• The Pitt’s India Act was passed in 1784 to remove the defects of the regulating act of 1773 and also
to establish British Parliamentary control over the company affairs in India. This Act gave the British
government supreme control over the company’s affairs and its administration in India.
• It established six commissioners for the affairs of India, popularly known as the Board of
Control, including two cabinet ministers. The Board of Control was to guide and control the work of the
Court of Directors and the Government of India.
• The Act placed the Government of India in the hands of the Governor-General and a Council of three so
that if the Governor-General could get the support of even one member, he could have his way. The Act
clearly subordinated the Bombay and Madras Presidencies to Bengal in all questions of war, diplomacy,
and revenues. Hence, statement 1 is correct.
• The British Parliament passed a series of Acts in the late eighteenth century to regulate and control
Company rule in India. It forced the Company to produce regular reports on the administration of India
8 www.visionias.in ©Vision IAS

FREE BY KING R QUEEN P [ऋषभ राजपूत]


and appointed committees to enquire into the affairs of the Company. The Fifth Report was one such
report produced by a Select Committee. It was submitted to the British Parliament in 1813. It
became the basis of intense parliamentary debates on the nature of the East India Company’s rule in
India.
• As per the provisions of the act, the directors of the company retained the profitable right of
appointing and dismissing its British officials in India. Moreover, the Government of India was to be
carried on through their agency. Hence, statement 2 is not correct.

Q 23.C
• The story of Champaran begins in the early nineteenth century when European planters had involved the
cultivators in agreements that forced them to cultivate indigo on the 3/20th of their holdings (known as the
tinkathia system). Towards the end of the nineteenth century, German synthetic dyes forced indigo out of
the market, and the European planters of Champaran, keen to release the cultivators from the obligation
of cultivating indigo, tried to turn their necessity to their advantage by securing enhancements in rent and
other illegal dues as a price for the release.
• Resistance had surfaced in 1908 as well, but the exactions of the planters continued till Raj Kumar
Shukla, a local man, decided to follow Gandhiji all over the country to persuade him to come to
Champaran to investigate the problem. Raj Kumar Shukla’s decision to get Gandhiji to Champaran is
indicative of the image he had acquired as one who fought for the rights of the exploited and the poor.
• Gandhi and his colleagues, who now included Brij Kishore, Rajendra Prasad, and other members
of the Bihar intelligentsia, Mahadev Desai and Narhari Parikh, two young men from Gujarat who
had thrown in their lot with Gandhiji, and J.B. Kripalani, toured the villages and from dawn to
dusk recorded the statements of peasants, interrogating them to make sure that they were giving
correct information. Hence, option (c) is the correct answer.
• Gandhiji then turned his attention to the workers of Ahmedabad. A dispute was brewing between them
and the mill owners over the question of a ‘plague bonus’ the employers wanted to withdraw once the
epidemic had passed but the workers insisted it stay since the enhancement hardly compensated for the
rise in the cost of living during the War.
• Ambalal Sarabhai, one of the leading mill owners of the town, was a friend of Gandhiji and had just
saved the Sabarmati Ashram from extinction by a generous donation. The strike began and Gandhiji
addressed the workers every day on the banks of the Sabarmati River. He brought out a daily news
bulletin and insisted that no violence be used against employers or blacklegs. Ambalal Sarabhai’s sister,
Anasuya Behn, was one of the main lieutenants of Gandhiji in this struggle in which her brother,
and Gandhiji’s friend, was one of the main adversaries.

Q 24.D
• Gandhiji returned to India on January 9th, 1915, after starting two settlements for Indians in South
Africa and launching a successful satyagraha against unfair laws and taxes on the Indian community there.
• Gandhiji was advised by his mentor Gopalkrishna Gokhale, who belonged to the ilk of Moderates
within the Congress to tour India for a year before embarking upon any political work.
• Gokhale’s instructions suited Gandhi well, as the latter realised that there was much about the country that
he had not seen, or knew about. In South Africa, differences of religion, caste and language were often
elided over, as the community of Indians stood as one against the imperial powers. In India, however, the
differences were wide and various, and Gandhi needed time to understand them.
• Hence, option (d) is the correct answer.

Q 25.B
• The Punjab under Ranjit Singh: At the end of the 18th century, Ranjit Singh, chief of the Sukerchakia
misl, rose into prominence.
• A strong and courageous soldier, an efficient administrator, and a skilful diplomat, he was a born leader of
men. He captured Lahore in 1799 and Amritsar in 1802.
• He soon brought all Sikh chiefs west of the Sutlej under his control and established his own kingdom in
the Punjab. Later, he conquered Kashmir, Peshawar, and Multan.
• The old Sikh chiefs were transformed into big zamindars and jagirdars.
• He did not make any changes in the system of land revenue promulgated earlier by the Mughals.
Hence statement 1 is not correct.
• The amount of land revenue was calculated on the basis of 50 per cent of the gross produce.

9 www.visionias.in ©Vision IAS

FREE BY KING R QUEEN P [ऋषभ राजपूत]


• Ranjit Singh built up a powerful, disciplined, and well-equipped army along European lines with
the help of European instructors. Hence statement 2 is correct.
• His new army was not confined to the Sikhs. He also recruited Gurkhas, Biharis, Oriyas, Pathans, Dogras,
and Punjabi Muslims.
• He set up modern factories to manufacture cannon at Lahore and employed Muslim gunners to man
them. Hence statement 3 is correct.
• It is said that he possessed the second best army in Asia, the first being the army of the English East India
Company.
• Ranjit Singh had great capacity for choosing his ministers and officials, He was tolerant and liberal in
religious matters.
• While a devout Sikh he was “known to step down from his throne to wipe the dust off the feet of Muslim
mendicants with his long grey beard.” Many of his important ministers and commanders were Muslims
and Hindus.
• The most prominent and trusted of his ministers was Fakir Azizuddin, while his Finance Minister was
Dewan Dina Nath.
• Infact, in no sense was the Punjab, ruled by Ranjit Singh, a Sikh state, Political power was not used for
exclusive Sikh benefit.

Q 26.D
• The Karachi Congress Session in 1931, was held following the Gandhi–Irwin Pact. It reflected the
impact of the Left on the national movement through the Resolution on Fundamental Rights and
Economic Policy. The socio-economic provision in the Karachi Resolution went on to influence the
Constituent Assembly in drawing up Part IV of the Indian Constitution – the Directive Principles of State
Policy. Hence, statement 1 is not correct.
• The Indian National Congress, on 19 December 1929, passed the historic ‘Purna Swaraj’ (total
independence) resolution at the Lahore session. Hence, statement 2 is not correct.
• The resolutions on economic policy was passed at the Faizpur session in 1936. It was, here, for the first
time that Congress held its Annual Session in a backward rural setting. A large number of peasants
participated in the session.

Q 27.C
• C. Rajagopalachari, the veteran Congress leader, prepared a formula for Congress-League
cooperation in 1944.
o It was a tacit acceptance of the League’s demand for Pakistan.
o Gandhiji supported the formula.
• The main points in the CR Plan were:
o Muslim League to endorse Congress's demand for independence.
o League to cooperate with Congress in forming a provisional government at the centre.
o After the end of the war, the entire population of Muslim-majority areas in North-West and North-
East India was to decide by a plebiscite, whether or not to form a separate sovereign state.
o In case of acceptance of partition, an agreement is to be made jointly for safeguarding defence,
commerce, communications, etc.
o The above terms were to be operative only if England transferred full powers to India.
o Jinnah wanted Congress to accept the two-nation theory. He wanted only the Muslims of the North-
West and North-East to vote in the plebiscite and not the entire population. He also opposed the idea
of a common centre.
o While the Congress was ready to cooperate with the League for the independence of the Indian Union,
the League did not care for the independence of the Union. It was only interested in a separate nation.
o Hindu leaders led by Vir Savarkar condemned the Plan.
• Hence option (c) is the correct answer.

Q 28.A
• The Royal Indian Navy revolt, also known as the 1946 Naval Uprising, occurred in February 1946 and
was a significant event in the Indian independence movement. The naval ratings stationed at HMIS (His
Majesty's Indian Ship) Talwar, a training establishment of the Royal Indian Navy, went on strike
and refused to obey their British officers, leading to widespread protests and demonstrations in cities like
Bombay, Karachi, Calcutta, and Madras.

10 www.visionias.in ©Vision IAS

FREE BY KING R QUEEN P [ऋषभ राजपूत]


• The naval ratings faced racial discrimination and abusive treatment despite their qualifications and
experience. They were paid less than their British counterparts and given menial tasks, while the
quality of food and living conditions on the ships were poor. The arrest of a rating for scrawling "Quit
India" on the HMIS Talwar was also a source of resentment. Hence options 1 and 2 are correct.
• The naval ratings demanded better pay, improved living conditions, and an end to racial discrimination,
which was not limited to HMIS Talwar but shared by naval ratings serving in the Royal Indian Navy
across India. The revolt quickly spread and involved over 20,000 ratings, leading to a widespread
disruption of naval operations. The naval ratings also demanded the release of Indian National Army
soldiers who were imprisoned after the INA trials.
• There was no issue of a new recruitment policy in the Royal Indian Navy at the time of the strike.
Hence option 3 is not correct.

Q 29.A
• Oil hypocrisy:
o A new study by the Centre for Research on Energy and Clean Air (CREA) has found that countries
that imposed crude oil sanctions on Russia used India, China, United Arab Emirates, Singapore, and
Turkey as ‘laundromats’ for refined products.
o During the last year, India has emerged as the leading exporter of refined oil products, followed by
China and the United Arab Emirates.
o Major exporting port: Sikka port and Vadinar port (Both in Gujarat) exported the highest amount
of seaborne refined oil to the Price Cap Coalition countries
o India now imports about 87% of its crude oil requirement (worth over US $190 bn) while it exported
petroleum products worth over $86 billion (accounting for more than 21 percent of India’s total
commodity exports)
o Laundromat: Laundromats’ is a term used to describe a type of financial fraud where large amounts
of money, often obtained through illegal means, are moved through a complex web of transactions
and accounts to disguise their origin and make them appear legitimate.
o Price Cap plan: The Price Cap Coalition is composed of Australia, Canada, the European Union,
France, Germany, Italy, Japan, the United Kingdom, and the United States. Hence option (a) is the
correct answer.
o NOTE: The Centre for Research on Energy and Clean Air (CREA) (est. 2019; HQ: Helsinki) is a
non-profit think tank researching energy and air pollution. It aims at tracking the impacts of air
pollution by providing data-backed research products.

Q 30.D
• REACHOUT:
o Ministry of Earth Sciences is implementing an umbrella scheme Research, Education, and Training
Outreach (REACHOUT) for capacity building
o Aim of the scheme: To support various R&D activities, develop useful collaborations with
international organizations, develop skilled and trained manpower in Earth Sciences, etc.
o It consists of the following sub-schemes:
✓ R&D in Earth System Science (RDESS).
✓ International Training Centre for Operational Oceanography (ITCOocean).
✓ Program for Development of Skilled Manpower in Earth System Sciences (DESK). Hence option
(d) is the correct answer.

Q 31.C
• ‘Ceramic Radome’ technology:
o A ceramic is a non-metallic, inorganic solid material that is typically produced by heating natural clay
or other minerals at high temperatures. E.g. Pottery, Tiles.
✓ Properties: The majority of ceramics are excellent insulators and can withstand high
temperatures.
o Radomes are structures or enclosures designed to protect an antenna and associated electronics from
the surrounding environment and elements such as rain, UV light, etc. Hence, statement 1 is correct.
o What are Ceramic radomes?
✓ Ceramic Radome Technology is the state-of-the-art technology for shielding Missiles across the
world from getting overheated. Hence, statement 2 is correct.
✓ Currently, ceramic radomes have been developed indigenously by Research Centre Imarat (RCI)
which has developed India’s missile arsenal.
11 www.visionias.in ©Vision IAS

FREE BY KING R QUEEN P [ऋषभ राजपूत]


✓ RCI is a DRDO laboratory for carrying out R&D in the technologies of control engineering,
inertial navigation, imaging infrared seekers, radio frequency seekers and systems, onboard
computers, and mission software.

Q 32.A
• In the elections to the central assembly in December 1945, out of 102 seats, Congress won 57, the
League 30, Independents 5, Akalis 2 and Europeans 8. Congress received 91 percent of non-Muslim
votes. Hence statement 1 is correct.
• In the elections to the provincial assemblies, Congress won 923 of 1585 seats, including 23 of the 38
labour seats, but was defeated by the Muslim League in the Muslim seats.
• In the NWFP, the League contested all 33 Muslim seats and won 15 of them. 19 Muslim seats were won
by the Congress, and 58.75 percent of the Muslim vote went to the non-League parties.
• In Punjab, the League gained 73 seats, with its share of the Muslim vote being 65.10 percent. The League
gained 83.6 percent of the Muslim vote in Bengal. It won 76 percent of the total Muslim vote in India.
This was a sharp rise from the 4.8 percent vote in its tally in 1937.
• The elections of 1946 were a watershed. The results made it clear that the Congress represented the large
masses of the country. It was equally clear, however, that the Muslim League spoke for most Muslims.
• The Congress was to go on to form governments in the provinces of Madras, Bombay, United Provinces,
Bihar, Orissa, Central Provinces and Berar and NWFP. The League formed ministries in Bengal and
Sind.
• In Punjab, after the 1946 elections, some negotiations between the Muslim League and Akalis were
carried out with a possible coalition in mind. This had been unsuccessful, and Akalis joined with
Unionists and Congress to form a coalition ministry. Hence statement 2 is not correct.

Q 33.A
• The peasant movement played an important role in the fight for independence from British rule. Peasant
movements were led by farmers, laborers, and rural workers who were discontent with the oppressive
policies of the British colonial government and the exploitative practices of the landlords and
moneylenders.
• Baba Ram Singh initiated the Kuka Movement, which marked the Sikh community's foremost
resistance to the British Empire's post-1849 political structure in Punjab. This movement was a blend of
political and religious beliefs and represented a notable uprising against the new British authority. Hence
pair 1 is correctly matched.
• Vasudev Balwant Phadke led the Maharashtra-based Ramosi Uprising as a protest against the British
government's failure to take any measures against the famine. This farmer uprising took a violent turn
instead of peaceful demonstrations. Hence pair 2 is not correctly matched.
• The Kisan sabhas were organized in UP mainly due to the efforts of the home rule activists. The United
Provinces Kisan Sabha was set up in 1918 by Gauri Shankar Mishra and Indra Narayan
Dwivedi, Madan Mohan Malaviya. Hence pair 3 is not correctly matched.

Q 34.C
• In 1897, Vivekananda founded the Ramakrishna Mission to carry on humanitarian relief and social
work. It laid emphasis not on personal salvation but on good or social service.
• Atmiya Sabha was established in Calcutta in 1815 by Ram Mohan Roy. The association conducted
debates and discussions on philosophical subjects, as well as advocated for free and collective thinking
and social reform.
• Truth seeker society (Satya Shodhak) was founded by Jyotiba Phule in 1873 with the leadership of
the samaj coming from the backward classes, malis, telis, kunbis, sans, dhangar.
• Hence option (c) is the correct answer.

Q 35.D
• The foundation of the Indian National Congress in 1885 was not a sudden event or a historical accident. It
was the culmination of a process of political awakening that had its beginnings in the 1860s and 1870s
and took a major leap forward in the late 1870s and early 1880s.
• The year 1885 marked a turning point in this process, for that was the year the political Indians, the
modem intellectuals interested in politics, who no longer saw themselves as spokesmen of narrow group
interests, but as representatives of national interest vis-a-vis foreign rule, as a ‘national party,’ saw their
12 www.visionias.in ©Vision IAS

FREE BY KING R QUEEN P [ऋषभ राजपूत]


efforts bear fruit. The all-India nationalist body that they brought into being was to be the platform, the
organizer, the headquarters, and the symbol of the new national spirit and politics.
• The nationalist Indian demands of those years —
o no reduction of import duties on textile import
o no expansion in Afghanistan or Burma,
o the right to bear arms,
o freedom of the Press,
o reduction of military expenditure,
o higher expenditure on famine relief,
o Indianization of the civil services,
o the right of Indians to join the semi-military volunteer corps,
o the right of Indian judges to try Europeans in criminal cases,
• Hence option (d) is the correct answer.

Q 36.D
• After deliberating amongst itself for close to three weeks, and after long discussions with delegates who
had returned from London's first RTC, and with other leaders representing a cross-section of political
opinion, the Congress Working Committee authorized Gandhiji to initiate discussions with the Viceroy.
The fortnight-long discussion culminated on 5 March 1931 in the Gandhi-Irwin Pact, which was
variously described as a ‘truce’ and a provisional settlement.’ The Pact was signed by Gandhiji on behalf
of the Congress and by Lord Irwin on behalf of the Government, a procedure that was hardly popular with
officialdom as it placed the Congress on an equal footing with the Government.
• The Congress met at Karachi on 29 March 1931 to endorse the Gandhi-Irwin or Delhi Pact. Bhagat
Singh, Sukhdev and Rajguru had been executed six days earlier.
• The Karachi Resolution was passed by the Indian National Congress at its 1931 Karachi session. The
Session was conducted in the shadow of three major events. First, Mahatma Gandhi had just been released
from prison following his Salt Satyagraha. Second, the Gandhi-Irwin pact had just been concluded which
had brought the civil disobedience movement to an end. And third, the British government had, a week
before the session, executed Bhagat Singh and two of his associates in connection with the Kakori
Conspiracy case.The Resolution is three pages long and is mostly written in a quasi-legal style. It
reiterated the Congress Party’s commitment to ‘Purna Swaraj’ or ‘complete independence’. In addition to
fundamental rights which protected civil liberties, the Resolution for the first time put forward a list of
socio-economic principles/rights that the Indian state had to adhere to. These included: protections for
industrial workers, abolishing of child labour, free primary education and protections for agricultural
labour. The Resolution also, which seems to be a Gandhian influence, prohibited intoxicating drinks and
drugs.
• Bhagat Singh, Rajguru and Sukhdev were sentenced to death in the Lahore conspiracy case and
ordered to be hanged on 24 March 1931. The schedule was moved forward by 11 hours and the
three were hanged on 23 March 1931 at 7:30 pm in the Lahore jail.
• Gandhiji sailed for London on 29 August 1931 to attend the Second Round Table Conference. It
was held in London from 7 September 1931 to 1 December 1931. Nothing much was expected from
the Conference for the imperialist political and financial forces, which ultimately controlled the British
Government in London, were opposed to any political or economic concessions being given to India
which could lead to its independence from their control. Hence, option (d) is the correct answer.

Q 37.B
• The discussions of the Third Round Table Conference were held in December 1932. It led to the
issue of a white paper in 1933, which declared that according to the new constitution, there would be
a dyarchy at the center and secretary of state of India. Thus, The Third Round Table Conference
eventually led to the passing of the Government of India Act, 1935.
• The Government of India Act, of 1919 introduced a new system of dyarchy for the executive at the
level of the provincial government. The 1935 act provided for the establishment of an All India
Federation and a new system of government for the provinces on the basis of provincial autonomy. The
federation was to be based on a union of the provinces of British India and the Princely states. Hence,
statement 1 is not correct.
• According to The Government of India Act, 1935, the Governors-General and the Governors were to be
appointed by the British government and were to be responsible for it. In the provinces, local power was
to be increased and ministers were given the power to control all departments, however, the Governors
could veto their legislative actions. Hence, statement 2 is correct.
13 www.visionias.in ©Vision IAS

FREE BY KING R QUEEN P [ऋषभ राजपूत]


Q 38.C
• Ningaloo Eclipse:
o It is a rare ‘hybrid solar eclipse’, caused by the curvature of the earth's surface and a shift from
annular to total eclipse. The last one was seen in 2013, and the next one will appear in 2031.
o Its uniqueness is such that it has already been named Ningaloo, a part of western Australia from
which the eclipse was most visible. The Ningaloo region is also designated as a UNESCO World
Heritage Site.
o Types of Solar Eclipse:
✓ Total Solar Eclipse: A total eclipse happens when the Moon completely blocks out the Sun while
passing between the Earth and the Sun. The Baily's Beads effect, also known as the diamond ring
effect, is a phenomenon that occurs during a total solar eclipse or annular solar eclipse.
✓ Annular Eclipse: It happens when the Moon is at its farthest point from the Earth. The sun is
covered in such a way that only a small ring-like sliver of light is seen from the sun's disc. This
ring is known as the ring of fire. Hence pair 1 is not correctly matched.
✓ Partial Eclipse: It occurs when the Moon passes between the Earth and the Sun but is not
perfectly aligned. Hence, only a part of the Sun appears covered. Hence pair 2 is not correctly
matched.
✓ Hybrid Eclipse: A hybrid solar eclipse occurs when the eclipse is total from some locations on
Earth and annular from others, due to the viewer's position relative to the Moon's shadow. Hence
pair 3 is correctly matched.
✓ It means that for some observers, the Moon appears to fully cover the Sun, resulting in a total
solar eclipse, while for others, the Moon only partially covers the Sun, resulting in an annular
solar eclipse.

Q 39.B
• In line with the government policy contained in Montagu’s statement of August 1917, the government
announced further constitutional reforms in July 1918, known as Montagu-Chelmsford or Montford
Reforms. Based on these, the Government of India Act, 1919 was enacted.
• Main Features:
o The Provincial Legislative Councils were enlarged and the majority of their members were to be
elected. Hence statement 2 is correct.
o The Act introduced dyarchy for the executive at the level of the provincial government. The Dyarchy
system provided more powers to the Provincial governments. Under this system some subjects, such
as finance and law and order, were called 'reserved' subjects and remained under the direct control of
the Governor; others such as education, public health, and local self-government, were called
'transferred' subjects and were to be controlled by ministers responsible to the legislatures. The
Governor could overrule the ministers on any grounds that he considered special. Hence statement 1
is not correct.
o At the centre, there were to be houses of the legislature, the lower house, the Legislative Assembly,
was to have 41 nominated members in a total strength of 144. The upper house, the Council of State,
was to have 26 nominated and 34 elected members. The legislature had virtually no control over the
Governor-General and his Executive Council. On the other hand, the Central Government had
unrestricted control over the provincial governments.
• The Indian National Congress met in a special session at Bombay in August 1918 under the presidentship
of Hasan Imam to consider the reform proposals. It condemned them as disappointing and unsatisfactory.
Some of the veteran Congress leaders led by Surendanath Banerjea were in favour of accepting the
government proposals and left the Congress at this time. They founded the Indian Liberal Federation.
Hence statement 3 is correct.

Q 40.D
• The Subsidiary Alliance played a significant role in the expansion of British rule in India, and many
Indian states, including Mysore, Hyderabad, and Awadh, were forced to accept it. The policy remained
in effect until the Indian Rebellion of 1857, which led to the end of the British East India Company's rule
in India and the beginning of direct British rule under the British Crown. The Subsidiary Alliance was a
policy introduced by Lord Wellesley, the Governor-General of India from 1798 to 1805, during the
British colonial rule in India.
• British were responsible for protecting their ally from external and internal threats under the terms of
the Subsidiary Alliance. Hence statement 1 is correct.

14 www.visionias.in ©Vision IAS

FREE BY KING R QUEEN P [ऋषभ राजपूत]


• The Indian rulers who accepted the Subsidiary Alliance were required to pay for the maintenance of
British troops stationed in their territories, and in return, the British promised to protect the Indian ruler
from external aggression and internal rebellion. Hence statement 2 is correct.
• the stationing of a British armed contingent in the territory of the ally was a key feature of the Subsidiary
Alliance. Under this policy, Indian rulers were required to accept the presence of British troops in
their territories. Hence statement 3 is correct.
• Under this policy, Indian rulers were required to accept British control over their foreign policy and
were not allowed to make treaties or alliances with other powers without British permission. This
condition of the Subsidiary Alliance was intended to prevent Indian rulers from forming alliances that
could potentially threaten British interests in India.

Q 41.C
• Even though the Court Martial held the INA prisoners guilty, the Government felt it expedient to set them
free.
• The changed attitude of the British Government is explained by several factors.
o Firstly, the war had changed the balance of power in the world. Not Britain, but the United States of
America and the Soviet Union emerged as winners of the war as big powers. Both supported India's
demand for freedom. Hence, statement 1 is correct.
o Secondly, even though Britain was on the winning side in the war, its economic and military power
was shattered. It would take Britain years to rehabilitate itself. Moreover, there was a change of
government in Britain.
o The Conservatives were replaced by the Labour Party many of whose members supported the
Congress demands. Hence, statement 2 is not correct.
o The British soldiers were weary of war. Having fought and shed their blood for nearly six years, they
had no desire to spend many more years away from home in India suppressing the Indian people's
struggle for freedom.
o Thirdly, the British Indian Government could no longer rely on the Indian personnel of its civil
administration and armed forces to suppress the national movement. Hence, statement 3 is
correct.
o The INA had shown that patriotic ideas had entered the ranks of the professional Indian army, the
chief instrument of British rule in India. Another straw in the wind was the famous revolt of the
Indian naval ratings at Bombay in February 1946.
o The ratings had fought a seven-hour battle with the army and navy and had surrendered only when
asked to do so by the national leaders. Moreover, there were also widespread strikes in the Indian Air
Force.
o The Indian Signal Corps at Jabalpur also went on strike.
o The other two major instruments of British rule, the police and the bureaucracy were also showing
signs of nationalist leanings.
o They could no longer be safely used to suppress the national movement. For example, the police force
in Bihar and Delhi went on strike.

Q 42.C
• US President Joe Biden visited Belfast, Northern Ireland, to mark the 25th anniversary of the Good
Friday Agreement (GFA), a peace deal that brought an end to decades of sectarian violence on the island
of Ireland known as “the Troubles.”
• Good Friday Agreement
o The Good Friday Agreement was signed on April 10, 1998, between factions of Northern Ireland,
and the governments of Britain and Ireland.
o It ended 30 years of the violence in Northern Ireland which is known as ‘The Troubles’, has been
hailed as a model deal to end long-standing conflicts.
o It was signed to end decades of violence in Northern Ireland among those who wished to remain
with the United Kingdom (UK) and those who wanted to join Ireland.
• Terms of the Good Friday Agreement:
o Northern Ireland would continue to be a part of the UK.
o It could join Ireland if a majority of voters on both sides supported it in a referendum.
o People born in Northern Ireland could have Irish or British nationality or both.
o Northern Ireland would get a new government, which would have powers over local matters, while
the UK government would look after security, foreign policy, tax laws, immigration rules, etc.

15 www.visionias.in ©Vision IAS

FREE BY KING R QUEEN P [ऋषभ राजपूत]


o On May 22 1998, a referendum was held in Ireland and Northern Ireland, and the agreement was
approved by 94% of voters in Ireland and 71% in Northern Ireland.

Q 43.D
• In 1625 the East India Company’s authorities at Surat made an attempt to fortify their factory but
the chiefs of the English factory were immediately imprisoned and put in irons by the local
authorities of the Mughal Empire which was still in its vigour.
• Similarly, when the Company’s English rivals made piratical attacks on Mughal shipping, the Mughal
authorities imprisoned in retaliation the President of the Company at Surat and members of his Council
and released them only on payment of £ 18,000.
• Conditions in the South were more favourable to the English as they did not have to face a strong Indian
Government there.
• The great Vijayanagar Kingdom had been overthrown in 1565 and its place was taken by a number of
petty and weak states.
• It was easy to appeal to their greed or overawe them with armed strength.
• The English opened their first factory in the South at Masulipatam in 1611.
• But they soon shifted the centre of their activity to Madras the lease of which was granted to them by the
local Raja in 1639.
• Madras was then a strip of coastal territory six miles long and one mile broad.
• The Raja authorised them to fortify the place, to administer it, and to coin money on condition of
payment to him of half of the customs revenue of the port. Here the English built a small fort
around their factory called Fort St. George.
• Hence, option (d) is the correct answer.

Q 44.B
• In the late 1940s, Gandhiji decided to initiate a limited satyagraha on an individual basis. The aims were:
o To show that nationalist patience was not due to weakness;
o to express that Indians made no distinction between Nazism and autocracy that ruled India; and
o to give another opportunity to the government to accept Congress’s Demands peacefully.
• The carefully chosen Satyagrahis — Vinoba Bhave was to be the first Satyagrahi on 17 October 1940
and Jawaharlal Nehru the second — were surrounded by huge crowds when they appeared on the
platform, and the authorities could often arrest them only after they had made their speeches. And if the
Government did not arrest a Satyagrahi, he or she would not only repeat the performance but move into
the villages and start a trek towards Delhi, thus participating in a movement that came to be known as
the ‘Delhi Chalo’ (onwards to Delhi) movement.
• Third Satyagrahi was Brahma Datt, one of the inmates of the Gandhi’s Ashram.
• Hence, option (b) is the correct answer.

Q 45.D
• March,1940: Pakistan Resolution was passed by Muslim League. The All India Muslim League met
in Lahore in March 1940 and adopted a resolution that came to be known as the Lahore Resolution
or Pakistan Resolution. Its the first hand demand was partition of the country and creation of Muslim
nation named Pakistan. The Muslim League demanded that the areas in which the Muslims are
numerically in a majority as in the North-Western and Eastern Zones of India should be grouped to
constitute Independent autonomous States.
• August,1940: August offer promising Dominion Status. On 8 August 1940, early in the Battle of
Britain, the Viceroy of India, Lord Linlithgow, made the so-called "August Offer" at Simla, a fresh
proposal promising the expansion of the Executive Council to include more Indians, the establishment of
an advisory war council, giving weight to minority opinion, and the recognition of Indians' right to frame
their own constitution (after the end of the war). In return, it was hoped that all parties and communities in
India would cooperate in Britain's war effort.
• October, 1940: Individual Satyagraha movement was launched to affirm the rights to speech.
Individual Satyagraha was the result of August offer. It was started with the mass Civil Disobedience
Movement by M.K Gandhi on Individual Satyagraha. This was movement for not only to seek
independence but also to affirm the right of Speech. The demand of the Satyagrahi was using freedom of
Speech against the war through an anti-war declaration. If government did not arrest the Satyagrahi, he or
she will move repeating it in villages and start march towards Delhi (“Delhi Chalo Movement”).
• Hence, option (d) is the correct answer.
16 www.visionias.in ©Vision IAS

FREE BY KING R QUEEN P [ऋषभ राजपूत]


Q 46.D
• MAHARISHI Initiative:
o It will focus on Research and Awareness about agro-biodiversity, food security, and nutrition aligning
with the International Year of Millets 2023. Hence statement 1 is not correct.
o Secretariat: Indian Institute of Millets Research (IIMR), Hyderabad. Hence statement 2 is not
correct.
o Technical Support: International Crops Research Institute for the Semi-Arid Tropics (ICRISAT),
One CGIAR Centres, and other international organizations
o Millets are known as nutri-cereals or superfoods, mainly grown on marginal land in dry areas of
temperate, sub-tropical, and tropical regions.

Q 47.C
• Alfonso de Albuquerque, who succeeded Almeida as the Portuguese governor in India, was the real
founder of Portuguese power in the East, a task he completed before his death.
• He secured for Portugal the strategic control of the Indian Ocean by establishing bases overlooking all the
entrances to the sea.
• There were Portuguese strongholds in East Africa, off the Red Sea, at Hormuz; in Malabar; and at
Malacca. They had strongholds over the entire Asian coast from Hormuz in the Persian Gulf to
Malacca in the Malaya.
• The Portuguese, under Albuquerque, bolstered their stranglehold by introducing a permit system for other
ships and exercising control over the major ship-building centers in the region.
• The nonavailability of timber in the Gulf and Red Sea regions for ship-building also helped the
Portuguese in their objectives.
• Albuquerque acquired Goa from the Sultan of Bijapur in 1510 with ease; the principal port of the
Sultan of Bijapur became “the first bit of Indian territory to be under the Europeans since the time of
Alexander the Great”. Hence option (c) is the correct answer.
• In the words of James Mill, the famous British historian of the 19th century: “The Portuguese followed
their merchandise as their chief occupation, but like the English and the Dutch of the same period, had no
objection to plunder when it fell in their way.”
• The Portuguese were intolerant and fanatical in religious matters. Their approach in this respect was
particularly hateful to the people of India where religious tolerance was the rule.
• They also indulged in inhuman cruelties and lawlessness. In spite of their barbaric behavior their
possessions in India survived for a century because they enjoyed control over the high seas, their soldiers
and administrators maintained strict discipline.

Q 48.D
• Logistics Performance Index
o It is released by the World Bank, which is an interactive benchmarking tool that helps countries to
identify the challenges and opportunities they face in their performance on trade logistics. The 2023
LPI for the first time measures the speed of trade with indicators derived from big datasets tracking
shipments.
o India has climbed six places on the World Bank's Logistic Performance Index (LPI) 2023. India was
ranked 44th on the index in 2018 and has now climbed to 38th in the 2023 listing.
• The Thales Data Threat Report, 2023:
o It is a comprehensive analysis of cybersecurity trends and threats. It has revealed some concerning
findings about ransomware attacks and cloud data breaches in India and globally. According to the
report by global technology company Thales, there has been a significant increase in these attacks,
o The report highlights a significant increase in these attacks, indicating the need for businesses to
implement robust security measures to protect their sensitive data.
• Global Food Policy Report, 2023:
o It is released by the International Food Policy Research Institute (IFPRI), urging stakeholders to
invest in long-term solutions for building resilient and equitable food systems.
o The report highlights multiple crises, such as the COVID-19 pandemic, climate change, natural
disasters, civil unrest, and political instability, which have led to a rise in food insecurity worldwide
from 2020-2022. Hence, option (d) is the correct answer.

17 www.visionias.in ©Vision IAS

FREE BY KING R QUEEN P [ऋषभ राजपूत]


Q 49.D
• Some of the features of the Indian Independence Act of 1947:
o It ended British rule in India and declared India as an independent and sovereign state from
August 15, 1947. Hence statement 1 is correct.
o It abolished the office of viceroy and provided, for each dominion, a governor-general, who was to
be appointed by the British King on the advice of the dominion cabinet. His Majesty’s Government in
Britain was to have no responsibility with respect to the Government of India or Pakistan. Hence
statement 2 is correct.
o It abolished the office of the secretary of state for India and transferred his functions to the
secretary of state for Commonwealth Affairs. Hence statement 3 is correct.

Q 50.B
• Small Industries Development Bank of India (SIDBI) recently announced a new scheme Mission 50K-
EV4ECO to boost the electric vehicle (EV) ecosystem in the country.
• About Mission 50K-EV4ECO:
o The objective of the mission is to promote the growth and development of the value chain of India’s
electric vehicle industry.
o It aims at strengthening the EV ecosystem, including uptake for two, three and four-wheelers through
direct and indirect lending.
o The pilot scheme, which is the precursor to EVOLVE scheme by SIDBI-World Bank, has two
components -direct lending and indirect lending.
o Under direct lending, SIDBI will directly give loans to eligible MSMEs (including aggregators,
fleet operators, and EV leasing companies) for the purchase of electric vehicles and develop charging
infrastructure, including battery swapping.
o The indirect scheme targeted at NBFCs, including small unrated focused and emerging NBFCs,
actively engaged in EV financing, will reach out to the last mile by inducing access to funds as also
reducing landed cost.
• Key facts about SIDBI
o It was established under an Act of Parliament in 1990.
o It is the Principal Financial Institution engaged in the promotion, financing & development of the
Micro, Small and Medium Enterprises (MSMEs) sector and coordination of the functions of the
various institutions engaged in similar activities.

Q 51.B
• Recent context: PM Modi inaugurated Kochi Water Metro. The Kochi Water Metro is a project being
implemented by Kochi Metro Rail Corporation Limited (KMRL) with the assistance of a German funding
agency, Kreditanstalt für Wiederaufbau.
• It includes boats that are hybrid, battery-powered, air-conditioned and disabled-friendly among other
features. The water metro will operate on water bodies like any other ferry or traditional boat service, but
with modern facilities, enhanced safety and security measures. Hence statement 2 is correct.
• Kochi Water metro has been envisaged as a feeder service of the Kochi metro rail, which has been
operational since 2017. While boats have been designed as coaches of Kochi Metro, boat terminals,
passenger entry and exit gates, ticket counters and safety measures mirror the features of the metro rail
service. All jetties feature electronic display boards about boat service. Announcements will be made in
English, Hindi and Malayalam when the services are operating in full swing. Passenger entry and exit to
boats, with air-conditioned cabins, are similar to the system in Kochi metros. The water metro boat
service will operate in the backwaters of Kochi, connecting 10 nearby islands with the mainland of
Kochi. Hence statement 1 is not correct.

Q 52.D
• Kanpur Bolshevik conspiracy case was a controversial court case initiated in British India in 1924.
• It was against the newly turned communists by the British government. The charge on them was 'to
deprive King-Emperor of his sovereignty of British India, by complete separation of India from
imperialistic Britain in a violent revolution.
• S.A.Dange, Muzaffar Ahmed, Nalini Gupta and Shaukat Usmani in the Kanpur Bolshevik Conspiracy
Case, all four were sentenced to four years of imprisonment.
• Hence, option (d) is the correct answer.

18 www.visionias.in ©Vision IAS

FREE BY KING R QUEEN P [ऋषभ राजपूत]


Q 53.A
• The second phase of the Indian National Army (INA) began when Subhas Chandra Bose was brought
to Singapore on 2 July 1943, by means of German and Japanese submarines.
• He went to Tokyo and Prime Minister Tojo declared that Japan had no territorial designs on India.
• Bose returned to Singapore and set up the Provisional Government of Free India on 21 October 1943.
• The Provisional Government then declared war on Britain and the United States and was recognised by
the Axis powers and their satellites.
• Subhas Bose set up two INA headquarters, in Rangoon and in Singapore, and began to reorganize
the INA.
• Hence option (a) is the correct answer.

Q 54.C
• C-Veda
o A recent study, under the C-Veda project, has evaluated and compared neurological development and
generated brain-development charts across ages, similar to charts for physical growth.
o C-Veda is an India-UK project (jointly funded by ICMR and UK’s Newton Grant from Medical
Research Council (MRC))
o Objective: It aims to investigate if environmental and genetic risk factors in industrialized
countries and emerging societies distinctly shape brain function and behavior. It has now established
the largest neurodevelopmental database in India. Hence option (c) is the correct answer.

Q 55.B
• Gopal Hari Deshmukh (18 February 1823 – 9 October 1892) was a social reformer and rationalist
from Maharashtra. He wrote weekly prabhakar under the pen name of Lokahitawadi on social
reform issues. He started a weekly Hitechhu and also played a leading role in founding the
periodicals, Gyan Prakash, Indu Prakash and Lokahitawadi.
• Narayan Malhar Joshi founded a social service league in Bombay with an aim to secure for the masses
better and reasonable conditions of life and work.
• Gopal Ganesh Agarkar was a co-founder of new english school, the deccan education society, and
Fergusson College. He started his own periodical sudharak which spoke against untouchability and the
caste system.
• Jyotiba phule founded satyasodhak samaj in 1873 with the leadership of the samaj coming from the
backward classes, malis, telis, kunbis, sans, dhangar.
• Hence option (b) is the correct answer.

Q 56.C
• The Indian Councils Act of 1861 enlarged the Governor General’s Executive Council for the purpose
of making laws. The Governor General's executive council consisted of five members. And for the
purpose of the legislation, the council was reinforced by six to twelve nominated members for a two-year
term. Half of these were to be nonofficials, both European and Indian not in the service of the Crown.
There were similar councils in the provinces. Thus, the act did not provide for the non-official
majority. The executive council was still dominated by officials. Hence, statement 1 is not correct.
• The Central legislative council constituted under the Indian Councils Act of 1861 came to be known as
the Imperial Legislative Council. The Council possessed no powers at all. It could not discuss the
budget or a financial measure or any other important bill without the previous approval of the
Government. It could not discuss the actions of the administration. The council did not have the right to
vote on the Budget. Hence, statement 3 is correct.
• The Charter Act of 1853 for the first time separated the legislative and executive functions of the
British government in India. The Council under the act of 1853 in its legislative capacity was to consist
of 12 members. These included the Governor General, Commander-in-Chief, four members of his council
and six legislative members. Hence, statement 2 is not correct.

Q 57.C
• In June 1894 the Natal Legislature planned to introduce the Indian Franchise Bill intended for
disenfranchising Indians. Having settled the lawsuit for which he had come, Gandhiji prepared to leave
for India. But on the eve of his departure from Durban, he raised the issue of the bill to
disenfranchise Indians which was in the process of being passed by the Natal legislature. The Indians
in South Africa begged Gandhiji to stay on for a month and organize their protest as they could not do so
19 www.visionias.in ©Vision IAS

FREE BY KING R QUEEN P [ऋषभ राजपूत]


on their own, not knowing even enough English to draft petitions, and so on. Gandhiji agreed to stay on
for a month and stayed for twenty years.
• The second phase of the struggle in South Africa, which began in 1906, was characterized by the use
of the method of passive resistance or civil disobedience, which Gandhiji named Satyagraha. It was
first used when the Government enacted legislation making it compulsory for Indians to take out
certificates of registration that held their fingerprints. It was essential to carry these on person at all
times. At a huge public meeting held on 11 September 1906, in the Empire Theatre in Johannesburg,
Indians resolved that they would refuse to submit to this law and would face the consequences. The
Government remained adamant, and so did the Indians. Gandhiji formed the Passive Resistance
Association to conduct the campaign. The last date for registration being over, the Government started
proceedings against Gandhiji and twenty-six others. The passive resisters pleaded guilty, were ordered to
leave the country and, on refusing to do so, were sent to jail. Others followed, and their numbers swelled
to 155. The fear of jail had disappeared, and it was popularly called King Edward’s Hotel.
• Meanwhile, the Government brought in new legislation, this time to restrict Indian
immigration. The campaign widened to oppose this. In August 1908, a number of prominent
Indians from Natal crossed the frontier into Transvaal to defy the new immigration laws and were
arrested. Other Indians from Transvaal opposed the laws by hawking without a license; traders who had
Licenses refused to produce them. All of them were jailed. Gandhiji himself landed in jail in October
1908 and, along with the other Indians, was sentenced to a prison term involving hard physical labour and
miserable conditions. At this stage, the movement reached an impasse. The more committed Satyagrahis
continued to go in and out of jail, but the majority were showing signs of fatigue. Gandhiji’s own legal
practice had virtually ceased since 1906, the year he had started devoting all his attention to the
struggle. At this point in 1910, Gandhiji set up Tolstoy Farm, made possible through the generosity of
his German architect friend, Kallenbach, to house the families of the Satyagrahis and give them a way to
sustain themselves. Hence, option (c) is the correct answer.
• Further fuel was added to the already raging fire by a judgment of the Supreme Court in 1913
which invalidated all marriages not conducted according to Christian rites and registered by the
Registrar of Marriages. By implication, Hindu, Muslim and Parsi marriages were illegal and the
children born through these marriages were illegitimate. The Indians treated this judgment as an insult to
the honor of their women and many women were drawn into the movement because of this indignity.
Gandhiji decided that the time had now come for the final struggle into which all the resisters’ resources
should be channeled. The campaign was launched by the illegal crossing of the border by a group of
sixteen Satyagrahis, including Kasturba, Gandhiji’s wife, who marched from Phoenix Settlement in Natal
to Transvaal and were immediately arrested.
Q 58.B
• The first time that INC session was held in a village was at the Faizpur Session of 193 under the
presidentship of Jawaharlal Nehru.
• National Planning Committee was set up under the leadership of Jawaharlal Nehru at the Haripura
Session of 1938 under the presidentship of Subhash Chandra Bose. Hence, option (b) is the correct
answer.
• The Independence Resolution was adopted by INC at the Madras session of 1927 under the presidentship
of M.A. Ansari.
• New Constitution for INC was framed at the Nagpur Session of 1920 under the presidentship of C.
Vijayaraghavachariar.

Q 59.B
• Recent context: The first images for research purposes captured by NASA’s Super Pressure Balloon
Imaging Telescope (SuperBIT) instrument have been released.
• The dark matter observatory consists of a telescope mounted on a balloon lofted to an altitude of 32.9
kilometres on April 16, 2023. The platform allows researchers to investigate scientific targets from a near-
space environment. Using a balloon is much more economical than a launch vehicle to deploy an
instrument in orbit. The super-pressure balloon can gather scientific data by circumnavigating the world
for up to 100 days after deployment. The height of the balloon prevents most of the atmospheric
interference for the imaging instrument on board.
• The goal of the observatory is to map the amount of dark matter in its targets, by measuring the
amount of light from distant sources that is amplified and distorted. The targets of SuperBIT exhibit
weak gravitational lensing, as against the strong gravitational lensing of more massive targets, such as
galaxy clusters. Hence option (b) is the correct answer.

20 www.visionias.in ©Vision IAS

FREE BY KING R QUEEN P [ऋषभ राजपूत]


Q 60.B
• The Revolt of 1857, also known as the First War of Indian Independence, was a widespread uprising
against British rule in India. It began on May 1857, in the town of Meerut and quickly spread
throughout the country, involving soldiers, peasants, landlords, and princes. The Revolt spread to
different parts of the country: Kanpur, Lucknow, Benares, Allahabad, Bareilly, Jagdishpur, and
Jhansi. The rebel activity was marked by intense anti-British feelings and the administration.
• At Kanpur, the leader was Nana Saheb, the adopted son of the last Peshwa, Baji Rao II. Hence pair 1 is
correctly matched.
• At Bareilly, Khan Bahadur, a descendant of the former ruler of Rohilkhand, was placed in
command. Hence pair 2 is not correctly matched.
• At Delhi, the nominal and symbolic leadership belonged to the Mughal emperor, Bahadur Shah, but the
real command lay with a court of soldiers headed by General Bakht Khan. Hence pair 3 is not
correctly matched.
• At Lucknow, Begum Hazrat Mahal took over the reins. Her son Birjis Qadir was proclaimed the nawab
and a regular administration was organized with important offices. Hence pair 4 is correctly matched.

Q 61.C
• National Health Claims Exchange (HCX):
o The Health Claims Exchange (HCX) is a new initiative under the Ayushman Bharat Digital Mission
(ABDM) that aims to bring interoperability of health claims and streamline the process of claim
settlement. Hence, statement 1 is correct.
o The HCX was announced by the National Health Authority (NHA) on September 23rd, 2022, as a
solution to the challenges posed by the current manual and non-digital process of exchanging health
insurance claims in the country.
o Features of HCX
✓ The HCX is designed to be interoperable, machine-readable, auditable, and verifiable, ensuring
that the information being exchanged is accurate and trustworthy.
✓ Insurers and third-party administrators (TPAs) are required to send responses for each e-claim
submitted by providers via the HCX through standard protocols (APIs). The HCX acts as a
gateway for the ecosystem, validating and routing the responses.
o HCX – Sandbox Environment
✓ To ensure the success and adoption of the claims network, the NHA has introduced the HCX –
Sandbox Environment.
✓ This is a testing ground for contributors and developers from the open community, especially
those working in the Health Insurance and Healthcare Provider space, to test and contribute to the
HCX ecosystem.
✓ The HCX – Sandbox enables participants to test their specific components against communication
standards and get certified to become a part of the system. The successful completion of the HCX
– Sandbox process enables the participants to gain access to the HCX production
environment. Hence, statement 2 is correct.

Q 62.C
• VAIBHAV Fellowship’ for NRI researchers
o Ministry of Science and Technology unveiled a fellowship to bring Indian-origin researchers to
higher educational institutions in India for a maximum period of two months per year. Hence,
statement 1 is not correct.
o Aim of the fellowship: It aims to improve the research ecosystem of India’s higher educational
institutions by facilitating academic and research collaborations with the best institutions in the world
through the mobility of researchers from overseas institutions to India. Hence, statement 2 is correct.
o Criteria: Applicant should be NRI or PIO or OCI and must have obtained a Ph.D./M. D/M. S degree
from a recognized University.
o Eligibility:
✓ The applicant should be Non-Resident Indian (NRI), Persons of Indian Origin (PIO)and Overseas
Citizen of India (OCI), currently living abroad.
✓ The applicant must have obtained Ph.D/M.D/M.S degree from a recognized University.
✓ Applicant must be a researcher engaged in an overseas academic / research /
industrialorganization with proven track record of research & development working in the top500
QS World University Ranking

21 www.visionias.in ©Vision IAS

FREE BY KING R QUEEN P [ऋषभ राजपूत]


✓ Plan to pursue research work for minimum of 1 month to a maximum of 2 months ayear in a
research institution / academic institution in India, spread over 3 years.
✓ Applicants can submit their proposal only once in a calendar year.
o Hence, statement 3 is correct.

Q 63.C
• Wagner Group
o According to reports in The Washington Post, it is a network of contractors that supply soldiers for
hire.
o It is said to have been founded by Dmitry Utkin, a former special forces officer, a member of Russia’s
military intelligence service, and a veteran of both Chechen wars. Hence option (c) is the correct
answer.
o It is ostensibly private, but according to CSIS, “its management and operations are deeply intertwined
with the Russian military and intelligence community” under Vladimir Putin.
o The Wagner Group first surfaced in 2014, during Russia’s annexation of Crimea. It has been involved
in conflicts in Africa, Latin America, and the Middle East.
o Its forces are known to be fighting in parts of Ukraine, including during a brutal, long-running battle
for the small city of Bakhmut.

Q 64.D
• Though Congress was opposed to the Government of India Act, 1935, it decided to contest the elections
under the Act, though with the desired aim of showing how unpopular the Act was.
• The election campaign of Congress met with massive response, even though Gandhiji didn’t address a
single election meeting. Hence, statement 3 is not correct.
• The elections, held in February 1937, conclusively demonstrated that a large majority of Indian people
supported the Congress which swept the polls in most of the provinces. Congress ministries were formed
in July 1937 in seven out of eleven provinces. Later, Congress formed coalition governments in two
others. Only Bengal and Punjab had non-Congress ministries. Hence, statement 1 is not correct.
• Punjab was ruled by the Unionist Party and Bengal by a coalition of the Krashak Praja Party and the
Muslim League. Hence, statement 2 is not correct.

Q 65.C
• Recent context: Explorers find WWII ship sunk with over 1,000 Allied prisoners of war.
• The Montevideo Maru was a ship that sunk during World War II, resulting in the loss of about 1,060
prisoners of war and civilians from 14 different countries. For 81 years, the wreckage of the ship remained
elusive, until a recent discovery by the Silentworld Foundation, led by Captain Roger Turner.
• The Montevideo Maru was a Japanese cargo ship that was used to transport prisoners of war and
civilians during World War II. On July 1, 1942, the ship was torpedoed and sunk by a US submarine in
the South China Sea. The ship was carrying about 979 Australian troops and civilians, as well as prisoners
from other countries, including the United Kingdom, New Zealand, and the Netherlands, among others.
Tragically, no one on board survived the sinking, and it became one of the deadliest maritime disasters in
Australian history. Hence statements 1 and 2 are correct.
• The wreck of the Montevideo Maru was located after a 12-day search at a depth of over 4000 meter
(13,120 feet) — deeper than the Titanic — off Luzon island in the South China Sea, using an autonomous
underwater vehicle with in-built sonar.
• The Montevideo Maru was transporting prisoners and civilians who were captured after the fall of Rabaul
in Papua New Guinea. The ship was not marked as carrying POWs, and on July 1, 1942, the American
submarine Sturgeon, after stalking the ship through the night, fired four torpedoes, which found their
target, sinking the vessel in less than 10 minutes.
Q 66.D
• Central Bureau of Narcotics
o Central Bureau of Narcotics is a central government organization dealing with the international
trade of narcotic drugs, psychotropic substances, and precursor chemicals under the ambit of
various United Nations Conventions and the provisions of The Narcotic Drugs and Psychotropic
Substances Act(NDPS), 1985.
o Ministry of Finance launched the unified portal of the Central Bureau of Narcotics (CBN), which
will instil efficiency and transparency and reduce the time taken by the pharma industry to get
a trade licence. Hence statement 1 is not correct.

22 www.visionias.in ©Vision IAS

FREE BY KING R QUEEN P [ऋषभ राजपूत]


o The unified portal of Central Bureau of Narcotics will instil the efficiency, transparency and
accountability in department users with twin objective to cater the need of pharma and chemical
industry for synergistic growth of the drugs & pharma sector and boosting economy for
“Aatma Nirbhar Bharat" and ensure availability of the “essential narcotic drugs" and
medicines to the patients & their attendant families. Hence statement 2 is not correct.
o Exporters, importers, and manufacturers of NDPS, and controlled substances are expected to benefit
in an ecosystem which supports and harbours user-friendly, secure transactions, cloud-based storage,
simplified process for applicants to obtain various licences.
• Narcotics Control Bureau is different from Central Bureau of Narcotics
o Narcotics Control Bureau is the drug law enforcement and intelligence agency of India responsible
for fighting drug trafficking and the abuse of illegal substances.
o It is affiliated with the Ministry of Home Affairs, which was made responsible for administering
the Narcotic Drugs and Psychotropic Substances (NDPS) Act, 1985.

Q 67.A
• The advance of the national movement in British India, and the accompanying increase in political
consciousness about democracy, responsible government and civil liberties had an inevitable impact on
the people of the princely states. In the first and second decade of the twentieth century, runaway terrorists
from British India seeking shelter in the princely states became agents of politicization.
• A much more powerful influence was exercised by the Non-Cooperation and Khilafat Movement
launched in 1920; around this time and under its impact, numerous local organizations of the States’
people came into existence.
• Some of the States in which praja mandals or States’ People’s Conferences were organized were
Mysore, Hyderabad, Baroda, the Kathiawad States, the Deccan States, Jamnagar, Indore, and
Nawanagar. This process came to a head in December 1927 at Bombay (first session) with the
convening of the All India States’ People’s Conference (AISPC) which was attended by 700 political
workers from the States. It was presided by Dewan Bahadur Ram Chandra Rao. The men chiefly
responsible for this initiative were Balwantrai Mehta, Maniklal Kothari and G.R. Abhayankar.
Hence statement 1 is correct and statement 2 is not correct.
• The main emphasis was that people of the States should build up their own strength and demonstrate their
capacity to struggle for their demands.
• Informal links between the congress and the various organisations of the people of the States,
including the AISPC, always continued to be close. In 1939, the AISPC elected Jawaharlal Nehru as its
President for the Ludhiana session, thus setting the seal on the fusion of the movements in Princely
India and British India.
Q 68.C
• The victory of the Labour Party in Britain in the post-war elections had significant implications for India.
Civil liberties were restored, the ban on Congress was lifted, and elections were declared both at the
centre and in the provinces for the winter of 1945–1946.
• The Congress welcomed the opportunity to form ministries in the provinces and to elect representatives to
the constitution-making body that would be set up.
• The election campaign became a massive exercise in the mobilisation of the people.
• A major issue taken up in the election campaign related to the unprecedented repression witnessed
against the 1942 movement. Hence option 1 is correct.
o A typical election speech would relate the tale of repression with all the details of brutality, move on
to condemning the officials who were guilty, and end with the promise (or threat) that Congress
governments would enquire into these cases and mete out punishment to the guilty officials.
• The second issue taken up in the election campaign was that of the fate of the members of Subhas
Bose’s Indian National Army taken prisoner by the Allies and put on trial for brutalities and war
crimes. Hence option 3 is correct.
o Jawaharlal Nehru hailed them as misguided patriots and called for leniency, given that big changes
were imminent in India. The Congress followed this up by passing a resolution in support of the
cause. Well-known Congress leaders like Bhulabhai Desai, K.N. Katju, M. Asaf Ali and Jawaharlal
Nehru argued the case in court when the trials began at the historic Red Fort. In addition to legal help,
Congress organised relief funds and arranged employment for the INA men. Congress election
meetings were often indistinguishable from INA meetings.
• Equal representation of Muslims and Hindus in Constituent Assembly was not specifically taken up
during the election campaign of 1945. Hence option 2 is not correct.

23 www.visionias.in ©Vision IAS

FREE BY KING R QUEEN P [ऋषभ राजपूत]


Q 69.C
• The British Government desperately wanted the active cooperation of Indians in the war effort during
World War II.
• To secure Indian cooperation it sent to India in March 1942 a mission headed by a Cabinet
Minister, Sir Stafford Cripps, who had earlier been a radical member of the Labour Party and a strong
supporter of the Indian national movement. Hence, statement 1 is correct.
• Even though Cripps declared that the aim of British policy in India was "the earliest possible realisation of
self-government in India", detailed negotiations between him and the Congress leaders broke
down. Hence, statement 2 is correct.
• The British Government refused to accept the Congress's demand to immediately transfer effective power
to Indians.
• On the other hand, the Indian leaders could not be satisfied by mere promises for the future while the
Viceroy retained his autocratic powers in the present.
• They were anxious to cooperate in the war effort, especially as the Japanese army endangered Indian
territory. But they could do so, they felt, only when a national government was formed in the country.
• The failure of the Cripps Mission embittered the people of India. While they still fully sympathised with
the anti-Fascist forces, they felt that the existing political situation in the country had become intolerable,
The Congress now decided to take active steps to compel the British to accept the Indian demand for
independence.
• The All India Congress Committee met in Bombay on 8 August 1942.
• It passed the famous 'Quit India' Resolution and proposed the start of a non-violent mass struggle under
Gandhi’s leadership to achieve this aim.

Q 70.C
• Paramahansa Mandali was a secret socio-religious group, established in 1849, in Bombay and is
closely related to Manav Dharma Sabha which was founded in the 1840s in Surat.
• It was the first socio-religious organization in Maharashtra, Its founders of these Mandli believed in one
god. They were primarily interested in breaking caste rules. At their meetings food cooked by lower
caste people was taken by the members. The Mandali also advocated women's education and widow
remarriage. Hence statement 1 is correct.
• It was started by Durgaram Mehtaji, Dadoba Pandurang, and a group of his friends. Hence statement
2 is correct.
• Balshastri Jambhekar started Digdardshan in 1840, which published articles on scientific subjects as
well as history.

Q 71.B
• The British policy of ‘Divide and Rule’ found another expression in the announcement of the Communal
Award in August 1932. The Award allotted to each minority a number of seats in the legislatures to be
elected on the basis of a separate electorate that is Muslims would be elected only by Muslims and Sikhs
only by Sikhs, and so on. Muslims, Sikhs and Christians had already been treated as minorities.
• The Award declared the Depressed Classes (Scheduled Castes of today) also to be a minority community
entitled to separate electorate and thus separated them from the rest of the Hindus.
• The Congress was opposed to a separate electorate for Muslims, Sikhs and ‘Christians as it encouraged
the communal notion that they formed separate groups or communities having interests different from the
general body of Indians. The inevitable result was to divide the Indian people and prevent the growth of a
common national consciousness.
• But the idea of a separate electorate for Muslims had been accepted by Congress as far back as 1916 as a
part of the compromise with the Muslim League. Hence, Congress took the position that though it was
opposed to separate electorates, it was not in favour of changing the Award without the consent of the
minorities.
• Consequently, though strongly disagreeing with the Communal Award, it decided neither to accept
it nor to reject it. Hence, statement 1 is not correct.
• But the effort to separate the Depressed Classes from the rest of the Hindus by treating them as separate
political entities was vehemently opposed by all the nationalists. Gandhiji, in Yeravada jail at the time, in
particular, reacted very strongly.’ He saw the Award as an attack on Indian unity and nationalism, harmful
to both Hinduism and the Depressed Classes, for it provided no answers to the socially degraded position
of the latter. Once the Depressed Classes were treated as a separate community, the question of abolishing
untouchability would not arise, and the work of Hindu social reform in this respect would come to a halt.
24 www.visionias.in ©Vision IAS

FREE BY KING R QUEEN P [ऋषभ राजपूत]


Gandhiji argued that whatever harm separate electorates might do to Muslims or Sikhs, it did not affect
the fact that they would remain Muslims or Sikhs. But while reformers like himself were working for the
total eradication of untouchability, separate electorates would ensure that ‘untouchables remain
untouchables in perpetuity.’ What was needed was not the protection of the so-called interests of the
Depressed Classes in terms of seats in the legislatures or jobs but the ‘root arid branch’ eradication of
untouchability. Gandhiji demanded that the representatives of the Depressed Classes should be elected by
the general electorate under a wide if possible universal, common franchise. At the same time, he did not
object to the demand for a larger number of reserved seats for the Depressed Classes. He went on a fast
unto death on 20 September 1932 to enforce his demand.
• Political leaders of different political persuasions, including Madan Mohan Malaviya, M.C. Rajah and
B.R. Ambedkar, now became active. In the end, they succeeded in hammering out an agreement,
known as the Poona Pact, according to which the idea of separate electorates for the Depressed
Classes was abandoned but the seats reserved for them in the provincial legislatures were increased
from seventy-one in the Award to 147 and in the Central Legislature to eighteen per cent of the
total. Hence, statement 2 is correct.

Q 72.C
• Lord Birkenhead, the Conservative Secretary of State responsible for the appointment of the Simon
Commission, had constantly harped on the inability of Indians to formulate a concrete scheme of
constitutional reforms which had the support of wide sections of Indian political opinion
• This challenge, too, was taken up and meetings of the All-Parties Conference were held in February, May
and August 1928 to finalize a scheme which popularly came to be known as the Nehru Report after
Motilal Nehru, its principal author.
o This report defined Dominion Status as the form of government desired by India.
o It also rejected the principle of separate communal electorates on which previous constitutional
reforms had been based.
o Seats would be reserved for Muslims at the Centre and in provinces in which they were in a
minority, but not in those where they had a numerical majority. Hence, option 1 is not correct.
o The Report also recommended Nineteen fundamental rights including universal adult suffrage,
equal rights for women, freedom to form unions and dissociation of the state from religion in
any form. Hence, options 2, 3 and 4 are correct.
o Linguistic provinces
o Responsible government at the Centre and in provinces
o The Indian Parliament at the Centre consists of a 500-member House of Representatives elected on
the basis of adult suffrage, a 200-member Senate to be elected by provincial councils; the House of
Representatives to have a tenure of 5 years and the Senate, one of 7 years; the central government to
be headed by a governor-general, appointed by the British government but paid out of Indian
revenues, who would act on the advice of the central executive council responsible to the Parliament.
o Provincial councils to have a 5-year tenure, headed by a governor acting on the advice of the
provincial executive council.
o Full protection to the cultural and religious interests of Muslims.
• A section of the Muslim League had in any case dissociated itself from these deliberations, but by the end
of the year, it became clear that even the section led by Jinnah would not give up the demand for
reservation of seats for Muslims, especially in Muslim majority provinces.
• The dilemma in which Motilal Nehru and other secular leaders found themselves was not one that was
easy to resolve: if they conceded more to Muslim communal opinion, then Hindu communalists would
withdraw support and if they satisfied the latter, then Muslim leaders would be estranged.
• In the event, no further concessions were forthcoming and Jinnah withdrew his support to the report and
went ahead to propose his famous ‘Fourteen Points’ which were basally a reiteration of his objections to
the Nehru Report.

Q 73.B
• Lord Curzon (1899-1905) was the viceroy during the partition of Bengal in 1905. His tenure also
include the second Delhi Durbar in 1903 and the creation of the North West Frontier Province in 1901 to
curb the uprising by the frontier tribes. Hence pair 1 is correctly matched.
• Lord Willingdon (1931-36) was the viceroy during the announcement of the Communal Award
(1932). Further, the Second Round Table Conference (1931), and failure of the conference, and the
resumption of the Civil Disobedience Movement are also associated with his tenure. Lord Irvin was the
viceroy during 1926-31. Hence pair 2 is not correctly matched.
25 www.visionias.in ©Vision IAS

FREE BY KING R QUEEN P [ऋषभ राजपूत]


• Lord Reading (1921-26) was the viceroy during the Chauri Chaura incident (February 5, 1922) and
the subsequent withdrawal of the Non-Cooperation Movement. Hence pair 3 is correctly matched.
• Lord Hardinge II (1910-16) was the viceroy during the Ghadar mutiny in 1915. Further, the creation of
the Bengal Presidency (like Bombay and Madras) in 1911 and the transfer of capital from Calcutta to
Delhi (1911) occurred during his tenure. Lord Chelmsford was the viceroy during 1916-21. Hence pair
4 is not correctly matched.

Q 74.C
• In a private communication to Zetland, the Secretary of State, Linlithgow was to remark a few months
later: ‘I am not too keen to start talking about a period after which British rule will have ceased in India. I
suspect that that day is very remote and I feel the least we say about it in all probability the better."
• Zetland's term as Secretary of State — and the experiment with democracy represented by the 1935 Act
— came to an end with Churchill's assumption of the Prime Ministership in 1940: Zetland then
offered his resignation, feeling that his ideas and Churchill's regarding India were so different that "I
could only end by becoming an embarrassment to him."
• On 18 October, Zetland spoke in the House of Lords and stressed differences among Indians,
especially among Hindus and Muslims. He branded the Congress as a purely Hindu organization.’
• Hence, option (c) is the correct answer.

Q 75.B
• what is Chagas disease?
o Chagas disease, also called American trypanosomiasis, is a communicable parasitic disease which
is caused by the parasite protozoan Trypanosoma cruzi.that has infected 6-7 million people and
claims around 12,000 lives every year across the globe, according to WHO.
o Chagas is currently endemic in 21 countries in the Americas, and shows an annual average incidence
of 30,000 new cases, according to Pan-American Health Organisation. Hence statement 1 is correct.
• Symptoms:
o The systemic, chronic disease manifests as fever, headaches, rashes and inflammatory nodules,
nausea or diarrhoea and muscle or abdominal pain.
o A majority of the patients (70-80 per cent) show an asymptomatic clinical course throughout their
lives, making early detection challenging. Because of this it is often referred to as the “silent and
silenced disease”.
• Transmission of Disease:
o It is caused by the parasite protozoan Trypanosoma cruzi. The parasites are mainly transmitted
by a family of bugs called ‘triatomines’, also known as the ‘kissing bug’.
o These blood-sucking bugs transmit the disease from infected to healthy individuals through bites and
by defecating on them.
o Other ways of Transmission:
✓ An individual can also contract this disease through congenital transmission (pregnant woman
to their baby), blood transfusions, organ transplantation, consumption of uncooked food
contaminated with faecal matter of infected bugs or even accidental laboratory exposure.
✓ This disease, however, cannot propagate by casual contact with infected humans or animals.
Hence statement 2 is not correct.
• Prevention and Treatment:
o There are currently no vaccines available for Chagas disease. However, the disease can be treated
with antiparasitic medicines Benznidazole and Nifurtimox. These have a 100 per cent efficacy rate if
the medicine is administered at the onset of the acute stage. Hence statement 3 is correct.

Q 76.A
• In Amritsar, the arrest of two local leaders, Dr. Saifuddin Kitchlew and Dr. Satyapal on 10 April led to an
attack on the town hall and the post office; telegraph wires were cut and Europeans including women
were attacked. The army was called in and the city handed over to General Dyer, who issued an order
prohibiting public meetings and assemblies.
• An unarmed but large crowd gathered on 13 April 1919 at Amritsar in the Jallianwalla Bagh, to protest
against the arrest of the leaders Saifuddin Kitchlew and Satyapal. Jallianwalla Bagh was a large open
space that was enclosed on three sides by buildings and had only one exit. General Dyer surrounded the
Bagh with his army unit, closed the exit with his troop, and then ordered his men to shoot into the trapped
crowd with rifles and machine guns. Thousands were killed and wounded. After this massacre, martial

26 www.visionias.in ©Vision IAS

FREE BY KING R QUEEN P [ऋषभ राजपूत]


law was proclaimed throughout Punjab and the people were submitted to the most uncivilized atrocities.
Hence statement 1 is correct.
• A wave of horror ran through the country as the knowledge of the Punjab happenings spread. The popular
shock was expressed by the great poet and humanist Rabindranath Tagore who renounced his knighthood
in the protest.
• In February 1919, Mahatma Gandhi founded the Satyagraha Sabha whose members took a pledge to
disobey the Rowlatt Act and thus to court arrest and imprisonment. Hence statement 2 is not correct.
• The Hunter Committee was appointed by the Government to enquire into the Punjab disturbances. It was
just an eyewash that the House of Lords had voted in favour of General Dyer’s action and that the British
public had demonstrated its support by helping the Morning Post collect 30,000 pounds for General Dyer.

Q 77.B
• The orientalist-Anglicist Controversy during British rule was related to educational development in India.
It erupted in the early nineteenth century (1830s) over the issue of the system of education and the
language of modern Education to be introduced in India.
• The officials who favoured the introduction of Western scientific knowledge in India came to be known as
the Anglicists while the Orientalists were in favour of promoting and disseminating traditional Oriental
learning.
• However, the famous Lord Macaulay’s Minute of 1835 settled the row in favour of Anglicists. The
limited government resources thus were to be devoted to the teaching of Western sciences and literature
through the medium of the English language alone. Thus, Orientalist-Anglicist Controversy was settled
much before the Woods despatch (1854). Hence, statement 1 is not correct.
• The Wood’s Dispatch was a document dispatched from the Court of Directors and popularly named
after Sir Charles Wood, President of the Board of Control in 1854.
• It was another important step in the development of education in India. The Dispatch asked the
Government of India to assume responsibility for the education of the masses. It thus repudiated the
“downward filtration” theory, at least on paper. Hence, statement 2 is correct.
• It systematised the hierarchy from vernacular primary schools in villages at the bottom, followed by
Anglo-Vernacular High Schools and an affiliated college at the district level, and affiliating universities in
the presidency towns of Calcutta, Bombay and Madras. It recommended English as the medium of
instruction for higher studies and vernaculars at the school level.
• The despatch directed the government to establish Departments of Education at the provincial levels and
affiliated Universities on the model of the London University.
• Consequently, Departments of Education were instituted in all provinces and affiliating universities
were set up in 1857 at Calcutta, Bombay and Madras. Bankim Chandra Chatterjee, the famous Bengali
novelist, became 1858 one of the first two graduates of Calcutta University.

Q 78.B
• World War II broke Out On 1 September 1939 when Nazi Germany invaded Poland. Earlier Germany
had occupied Austria in March 1938 and Czechoslovakia in 1939. Britain and France, which had been
following a policy of appeasement towards Hitler, were now forced to go to Poland’s aid and declare war
on Germany. This they did on 3 September 1939. The Government of India immediately declared
India to be at war with Germany without consulting the Congress or the elected members of the
central legislature.
• Different opinions on the question of Indian support to British war efforts in WW2:
o Mahatma Gandhi advocated an unconditional support to the Allied powers. Hence, statement 1
is not correct.
o Subhas Bose and other socialists leaders such as Acharya Narendra Dev and Jaya prakash Narayan
were of the view to take advantage of the situation of World War II. Hence, statement 2 is
correct.
o Nehru's View: No Indian Participation in WW2 and at the same, no opportunistic view of the
situation either. Hence, statement 3 is not correct.
o The Muslim League viewed the war situation as one from which it could profit in getting a British
assurance on its demands.

27 www.visionias.in ©Vision IAS

FREE BY KING R QUEEN P [ऋषभ राजपूत]


Q 79.A
• The founder of the autonomous kingdom of Avadh was Saadat Khan Burhan-ul-Mulk who was
appointed Governor of Avadh in 1722. Hence option (a) is the correct answer.
• He was an extremely bold, energetic, iron-willed, and intelligent person. At the time of his appointment,
rebellious zamindars had raised their heads everywhere in the province. They refused to pay the land tax,
organized their own private armies, erected forts, and defied the Imperial Government. For years Saadat
Khan had to wage war upon them.
• He succeeded in suppressing lawlessness and disciplining the big zamindars and thus, increasing the
financial resources of his government. Most of the defeated zamindars were, however, not displaced.
• They were usually confirmed in their estates after they had submitted and agreed to pay their dues (land
revenue) regularly Moreover, they continued to be refractory.
• Whenever Nawab’s military hold weakened or he was engaged in some other direction, they would rebel,
thus weakening Nawab’s power.
• As Safdar Jang, Saadat Khan's successor, later wrote. “The Avadh chiefs were capable of creating a
disturbance in the twinkling of an eye and were more dangerous than the Marathas of the Deccan ”.
• Saadat Khan also carried out a fresh revenue settlement in 1723. He is said to have improved a lot
of the peasants by levying equitable land revenue and by protecting him from oppression by the big
zamindars.
• Like the Bengal Nawabs, he too did not discriminate between Hindus and Muslims.
• Many of his commanders and high officials were Hindus and he curbed refractory zamindars, chiefs, and
nobles irrespective of their religion.
• His troops were well-paid, well-armed, and well-trained.
• Alivardi Khan: Alivardi Khan was the Nawab of Bengal from 1740 to 1756. He toppled the Nasiri
dynasty of Nawabs by defeating Sarfaraz Khan in 1740 and assumed power himself.
• Safdar Jung: Abul Mansur Mirza Muhammad Muqim Ali Khan, better known as Safdar Jang, was a
major figure at the Mughal court during the declining years of the Mughal Empire. He became the second
Nawab of Awadh when he succeeded Saadat Ali Khan I in 1739.
• Shuja ud Daula: Shuja-ud-Daula was the son of Safdar Jung. He was the Subedar and Nawab of Oudh
from 5 October 1754 to 26 January 1775.

Q 80.D
• The Non-Cooperation movement was launched formally on 1 August 192O, after the expiry of the notice
that Gandhiji had given to the Viceroy in his letter of 22 June. in which he had asserted the right
recognized ‘from time immemorial of the subject to refuse to assist a ruler who misrules.’
• The Congress met in September at Calcutta and accepted non-cooperation as its own. The main
opposition, led by C.R. Das, was to the boycott of legislative councils, elections to which were to be held
very soon. But even those who disagreed with the idea of a boycott accepted the Congress discipline and
withdrew from the elections. The voters, too, largely stayed away. By December, when the Congress met
for its annual session at Nagpur, the opposition had melted away; the elections were over and, therefore,
the boycott of councils was a non-issue, and it was CR. Das who moved the main resolution on non-
cooperation
• Gandhiji’s, decision to withdraw from the Non-cooperation movement in response to the violence at
Chauri Chaura raised a Controversy whose heat can still be felt in staid academic seminars and sober
volumes of history. Motilal Nehru, C.R. Das, Jawaharlal Nehru, Subhas Bose, and many others have
recorded their utter bewilderment on hearing the news.
• They could not understand why the whole country had to pay the price for the crazy behavior of some
people in a remote village. Many in the country thought that the Mahatma had failed miserably as a leader
and that his days of glory were over. Many later commentators, following, the tradition established by R.
Palme Dutt in India Today, have continued to condemn the decision taken by Gandhiji, and seen in it
proof of the Mahatma’s concern for the propertied classes of Indian society.
• Their argument is that Gandhiji did not withdraw from the movement simply because of his belief in the
necessity of non-violence. He withdrew it because the action at Chauri Chaura was a symbol and an
indication of the growing militancy of the Indian masses, of their growing radicalization, of their
willingness to launch an attack on the status quo of property relations. Frightened by this radical
possibility and by the prospect of the movement going out of his hands and into the hands of radical
forces, and in order to protect the interests of landlords and capitalists who would inevitably be at the
receiving end of this violence, Gandhiji cried halt to the movement.

28 www.visionias.in ©Vision IAS

FREE BY KING R QUEEN P [ऋषभ राजपूत]


• They have found supportive proof in the resolution of the Congress Working Committee of 12
February 1922 popularly known as the Bardoli resolution which while announcing the withdrawal,
asked the peasants to pay taxes and tenants to pay rents. This, they say, was the real though hidden
motive behind the historic decision of February 1922. Hence, option (d) is the correct answer.

Q 81.B
• The Rise of the Maratha Power
• The most important challenge to the decaying Mughal power came from the Maratha Kingdom which was
the most powerful of the succession states. In fact, it alone possessed the strength to fill the political
vacuum created by the disintegration of the Mughal empire.
• Moreover, it produced a number of brilliant commanders and statesmen needed for the task, But the
Maratha sardars lacked unity, and they lacked the outlook and program which were necessary for
founding an all-India empire,
• And so they failed to replace the Mughals. They did, however, succeed in waging continuous war against
the Mughal Empire, till they destroyed it.
• Shahu, the grandson of Shivaji, had been a prisoner in the hands of Aurangzeb since 1689.
• Aurangzeb had treated him and his mother with great dignity, honour, and consideration, paying
full attention to their religious, caste, and other needs, hoping perhaps to arrive at a political
agreement with Shahu. Shahu was released in 1707 after Aurangzeb’s death. Hence statement 1 is
correct.
• Very soon a civil war broke out between Shahu at Satara and his aunt Tara Bai at Kolhapur who
had carried out an anti-Mughal struggle since 1700 in the name of her son Shivaji Il after the death of
her husband Raja Ram. Hence statement 2 is correct.
• Maratha sardars, each one of whom had a large following of soldiers loyal to himself alone, began to side
with one or the other contender for power. They used this opportunity to increase their power and
influence by bargaining with the two contenders for power.
• Several of them were even intrigued with the Mughal viceroys of the Deccan.
• Arising out of the conflict between Shahu and his rival at Kolhapur, a new system of Maratha
government evolved under the leadership of Balaji Vishwanath, the Peshwa of King Shahu.
• With this change began the second period—the period of Peshwa domination in Maratha history in which
the Maratha state was transformed into an empire.
• Balaji Vishwanath, a Brahmin, started life as a petty revenue official and then rose step by step as an
official. He rendered Shahu loyal and useful service in suppressing his enemies. He excelled in diplomacy
and won over many of the big Maratha sardars to Shahu's cause.
• In 1713, Shahu made him his Peshwa. Hence statement 3 is correct.
• Balaji Vishwanath gradually consolidated Shahu's hold and his own over Maratha sardars and over
most of Maharashtra except for the region south of Kolhapur where Raja Ram’s descendants ruled.
• The Peshwa concentrated power in his office and eclipsed the other ministers and sardars. In fact he and
his son Baji Rao made the Peshwa' the functional head of the Maratha Empire.
• Balaji Vishwanath took full advantage of the internal conflicts of the Mughal officials to increase
Maratha's power.

Q 82.C
• Simon Commission was appointed by the British government in November 1927 to go into the question
of further constitutional reforms.
• All the members of the commission were Englishmen, this announcement was greeted by a chorus of
disenchantment from all sections of India as it was seen as a violation of the principle of self-
determination and a deliberate insult to the self-respect of the Indians.
• The Congress Party decided to boycott the Commission at their session at Madras in 1927. The Muslim
League led by M A Jinnah and the Hindu Mahasabha also decided to support the Congress decision.
• Hence, option (c) is the correct answer.

Q 83.C
• The Conservative Government in Britain led by Winston Churchill was keen to reach a solution to the
constitutional question in India. The Viceroy, Lord Wavell, was permitted to start negotiations with
Indian leaders. Congress leaders were released from jails in June 1945.
o The idea was to reconstruct the governor general’s executive council pending the preparation of
a new constitution.
29 www.visionias.in ©Vision IAS

FREE BY KING R QUEEN P [ऋषभ राजपूत]


o For this purpose, a conference was convened by the viceroy, Lord Wavell, at Shimla in June
1945.
• The main proposals of the Wavell Plan were as follows:
o With the exception of the governor-general and the commander-in-chief, all members of the
executive council were to be Indians. Hence, statement 1 is correct.
o Caste Hindus and Muslims were to have equal representation. Hence, statement 2 is correct.
o The reconstructed council was to function as an interim government within the framework of the 1935
Act (i.e. not responsible to the Central Assembly).
o The governor general was to exercise his veto on the advice of ministers.
o Representatives of different parties were to submit a joint list to the viceroy for nominations to the
executive council. If a joint list was not possible, then separate lists were to be submitted.
o Possibilities were to be kept open for negotiations on a new constitution once the war was finally
won.
o The League wanted all Muslim members to be League nominees because it feared that since the aims
of other minorities—depressed classes, Sikhs, Christians, etc.—were the same as those of the
Congress, this arrangement would reduce the League to a one-third minority. (Wavell wanted Khizr
Hyat Khan as the Muslim representative from Western Punjab.)
o The League claimed some kind of veto in the council with decisions opposed to Muslims needing a
two-thirds majority for approval.
o The Congress objected to the plan as “an attempt to reduce the Congress to the status of a purely caste
Hindu party and insisted on its right to include members of all communities among its nominees”.
o Wavell announced a breakdown of talks thus giving the League a virtual veto.
o This strengthened the League’s position, as was evident from the elections in 1945–46, boosted
Jinnah’s position; and exposed the real character of the Conservative government of Churchill.

Q 84.B
• When the All-India Congress Committee met at Bezwada (now Vijayawada in Andhra Pradesh) on March
31, 1921, they devised a constructive program wherein amongst other aims, it was decided to raise one
crore rupees by the end of June 1921. Bombay played the central arena of activity for Mahatma Gandhi
from April to June 1921.
• He devised several measures to build the fund and strategized to raise 60 lakhs from Bombay and 40 lakhs
from the rest of the country. Support for the Fund came from multiple angles and received a thunderous
response, and even surpassed the target.
• The AICC, at its session at Vijayawada in March 1921, directed that for the next three months,
Congressmen should concentrate on the collection of funds, enrolment of members and distribution of
charkhas. As a result, a vigorous membership drive was launched and though the target of one crore
members was not achieved, Congress membership reached a figure roughly of 50 lakhs.
• The overwhelming support and generosity that Bombay displayed earned itself the sobriquet “Bombay the
Beautiful” by Gandhi in the Young India magazine on July 6, 1921.
• In the Nagpur session of Congress which was held on December 1921, a decision was taken to create
the All-India Tilak Memorial Swaraj Fund (in the memory of Bal Gangadhar Tilak) to raise funds
for the Non-Cooperation Movement. Bombay played a central role in the collection of funds for it.
Hence, option (b) is the correct answer.

Q 85.C
• Lakhajiraj died in 1939 and his son Dharmendra Singhji, a complete contrast to the father, soon took
charge of the State. The new Thakore was interested only in pleasure, and effective power fell into the
hands of Dewan Virawala, who did nothing to stop the Thakore from frittering away the State’s wealth,
and finances reached such a pass that the State began to sell monopolies for the sale of matches, sugar,
rice, and cinema licences to individual merchants. This immediately resulted in a rise in prices and
enhanced the discontent that had already emerged over the Thakore’s easy-going life-style and his
disregard for popular participation in government as reflected in the lapse of the Pratinidhi Sabha as well
as the increase in taxes. The ground for struggle had been prepared over several years of political work by
political groups in Rajkot and Kathiawad. Hence, statement 1 is not correct.
• The first struggle emerged under the leadership of Jethalal Joshi, a Gandhian worker, who organized the
800 labourers of the state-owned cotton mill into a labour union and led a twentyone day strike in 1936 to
secure better working conditions. The Durbar had been forced to concede the union’s demands. This
victory encouraged Joshi and UN Dhebar to convene, in March 1937, the first meeting of the Kathiawad
Rajakiya Parishad to be held in eight years. The conference, attended by 15,000 people, demanded
30 www.visionias.in ©Vision IAS

FREE BY KING R QUEEN P [ऋषभ राजपूत]


responsible government, reduction in taxes and state expenditure. Hence, statement 2 and 3 are
correct.

Q 86.C
• In 1595, four Dutch ships sailed to India via the Cape of Good Hope.
• In 1602, the Dutch East India Company was formed and the Dutch States General —the Dutch
parliament—gave it a Charter empowering it to make war, conclude treaties, acquire territories
and build fortresses. Hence statement 2 is correct.
• The main interest of the Dutch lay not in India but in the Indonesian Islands of Java, Sumatra, and the
Spice Islands where spices were produced.
• They soon turned out the Portuguese from the Malay Straits and the Indonesian Islands and in 1623,
defeated English attempts to establish themselves there.
• It appeared at the time that the Dutch had successfully seized the most important profitable part of Asian
trade.
• They did not, however, entirely abandon Indian trade.
• They also established trading depots at Surat, Broach, Cambay, and Ahmedabad in Gujarat in
West India, Cochin in Kerala, Nagapatam in Madras, Masulipatam in Andhra, Chinsura in Bengal,
Patna in Bihar, and Agra in Uttar Pradesh. Hence statement 1 is correct.
• In 1658 they also conquered Ceylon from the Portuguese.
• They exported indigo, raw silk, cotton textiles, saltpetre, and opium from India.
• Coffee was introduced to India during the late seventeenth century. The story goes that an Indian
pilgrim to Mecca – known as Baba Budan – smuggled seven beans back to India from Yemen in 1670 (it
was illegal to take coffee seeds out of Arabia at the time) and planted them in the Chandragiri hills of
Karnataka.

Q 87.D
• Damon-i-Koh was a large area of land demarcated by the British East India Company for the
settlement of Santhals near the Rajmahal hills. Santhals were given lands and persuaded to practice
settled cultivation. The Santhals were to live within it, practice plough agriculture, and become settled
peasants. The land grant to the Santhals stipulated that at least one-tenth of the area was to be cleared and
cultivated within the first ten years. The territory was surveyed and mapped. Enclosed with boundary
pillars, it was separated from both the world of the settled agriculturists of the plains and the Paharias of
the hills.
• However, the Santhals soon found that the land they had brought under cultivation was slipping away
from their hands. The state was levying heavy taxes on the land that the Santhals had cleared,
moneylenders (dikus) were charging them high rates of interest and taking over the land when debts
remained unpaid, and zamindars were asserting control over the Damin area. The discontent of the
Santhals was reflected in the Santhal Revolt (1855-56).
• Hence option (d) is the correct answer.

Q 88.D
• Indian newspapers in the 1870s became highly critical of Lord Lytton’s administration, especially
regarding its inhuman approach toward the victims of the famine of 1876-77. As a result, the Government
decided to make a sudden strike at the Indian language newspapers, since they reached beyond the
middle-class readership.
• The Vernacular Press Act (VPA) of 1878, directed only against Indian language newspapers,was
conceived in great secrecy and passed at a single sitting of the Imperial Legislative Council. Hence
statement 1 is correct.
o The Act provided for the confiscation of the printing press, paper, and other materials of a
newspaper if the Government believed that it was publishing seditious materials and had
flouted an official warning. Hence statement 2 is correct.
o When the Vernacular Press Bill came up before the Council, only one Indian member, Maharaja
Jitendra Mohan Tagore, the leader of the Zamindari-dominated British Indian Association was
present. He voted for it.
o Indian nationalist opinion firmly opposed the Act. The first great demonstration on an issue of
public importance was organized in Calcutta on this question when a large meeting was held in the
Town Hall. Various public bodies and the Press also campaigned against the Act.
o Consequently, it was repealed in 1881 by Lord Ripon. Hence statement 3 is correct.
31 www.visionias.in ©Vision IAS

FREE BY KING R QUEEN P [ऋषभ राजपूत]


o Under VPA, proceedings were instituted against Som Prakash, Bharat Mihir, Dacca Prakash,
and Samachar. Incidentally, the Amrita Bazar Patrika turned overnight into an English
newspaper to escape the VPA.
o VPA, 1878 is also known as 'gagging act'.

Q 89.D
• On 8 August 1940, early in the Battle of Britain, the Viceroy of India, Lord Linlithgow, made the so-
called "August Offer" at Simla, a fresh proposal promising the expansion of the Executive Council to
include more Indians, the establishment of an advisory war council, giving weight to minority opinion,
and the recognition of Indians' right to frame their own constitution (after the end of the war). In return, it
was hoped that all parties and communities in India would cooperate in Britain's war effort.
• British got into conciliatory mood owing to Hitler’s advances to seek Indian Support, so this offer was
proposed:
o Promise of the expansion of the Executive Council. Hence option 1 is correct.
o Dominion status as the objective for India. Hence option 2 is correct.
o Promise of Constituent Assembly (mainly Indians) post WW2.
o No future constitution to be adopted without the consent of minorities. Hence option 3 is
correct.
• For the first time, inherent right of Indians to frame their Constitution was recognised. Both congress &
Muslim League rejected it. Nehru said, “Dominion status concept is dead as a doornail.”

Q 90.C
• Landholder's Society (Zamindari Association) was established in 1838 to protect the landlords' interests
of Bengal, Bihar and Orissa. Despite its limited objectives, the Landholders' Society marked the
beginning of organized political activity and the use of constitutional agitation methods for the redressal
of grievances.
• Bengal British Indian Society was organized in 1843 to protect and promote the interests of the general
public. Later on, the Landholders Society and Bengal British Indian Society were merged to form British
India Association.
• Madras Native Association was established in 1852 along with the Bombay Association.
• Poona Sarvajanik Sabha was organised in the 1870s by Justice Ranade and others. It is mainly
devoted to criticism of important legislative and administrative measures.
• Hence, option (c) is the correct answer.

Q 91.A
• Animal Pandemic Preparedness Initiative (APPI):
o The initiative is in line with the World Health Organization's (WHO) Global One Health strategy,
which emphasizes the importance of multisectoral collaboration in addressing the threat of zoonotic
diseases.
o It will help to improve veterinary services and infrastructure, disease surveillance capabilities, early
detection and response, build the capacity of animal health professionals, and awareness among
farmers through community outreach.
o It will create an “integrated disease reporting system for enhanced surveillance of zoonotic and other
animal diseases. Hence, statement 1 is correct.
o It has been launched by the Ministry of Fisheries, Animal Husbandry & Dairying under One Health
Approach to address the threat of diseases that can be transmitted from animals to humans. Hence,
statement 2 is not correct.
o Pillars of APPI:
✓ Disease Surveillance and Monitoring.
✓ Disease Model Algorithms and Early Warning System.
✓ Outbreak Investigation and Response.
✓ Ecosystem Coordination.
✓ Vaccine Development and Research and Development.
✓ Building Disaster Resilience.
✓ Funding.
✓ Regulatory Framework.
o Aim: The initiative aims to enhance India's preparedness and response capabilities to prevent and
control zoonotic diseases, thereby safeguarding the health of both animals and humans.

32 www.visionias.in ©Vision IAS

FREE BY KING R QUEEN P [ऋषभ राजपूत]


• Animal Health System Support for One Health (AHSSOH):
o It aims to create an ecosystem for a better animal health management system using the One Health
approach.
o The project will be implemented over a five-year period as Central sector scheme.
o It has set a goal to cover 151 districts in five participating states, focusing on upgrading 75
district/regional laboratories, as well as strengthening 300 veterinary hospitals/dispensaries.
• NOTE: The NOHM is a cross-ministerial effort approved by the Prime Minister's Science, Technology,
and Innovation Advisory Council (PM-STIAC). The NOHM will be implemented by the Ministry of
Health and Family Welfare in collaboration with other ministries.

Q 92.A
• By December 1920, when the Congress met for its annual session at Nagpur, the opposition had melted
away; the elections were over and, therefore, the boycott of councils was a non-issue, and it was CR. Das
moved the main resolution on non-cooperation. The programme of non-cooperation included within its
ambit the surrender of titles and honours, boycott of government-affiliated schools and colleges, law
courts, and foreign cloth, and could be extended to include resignation from government service and mass
civil disobedience including the non-payment of taxes.
• National schools and colleges were to be set up, panchayats were to be established for settling disputes,
hand-spinning and weaving were to be encouraged and people were asked to maintain Hindu- Muslim
unity, give up untouchability and observe strict non-violence. Gandhiji promised that if the programme
was fully implemented, Swaraj would be ushered in within a year. The Nagpur session, thus, committed
Congress to a programme of extra-constitutional mass action.
• Many groups of revolutionary terrorists, especially in Bengal, also pledged support to the movement. To
enable the Congress to fulfil its new commitment, significant changes were introduced in its creed as well
as in its organizational structure. The goal of the Congress was changed from the attainment of self-
government by constitutional and legal means to the attainment of Swaraj by peaceful and legitimate
means. The new constitution of the Congress, the handiwork of Gandhiji, introduced other important
changes.
• The Congress was now to have a Working Committee of fifteen members to look after its day-to-
day affairs. This proposal, when first made by Tilak in 1916, had been shot down by the Moderate
opposition. Gandhiji, too, knew that the Congress could not guide a sustained movement unless it
had a compact body that worked around the year. Hence, statement 2 is correct.
• Provincial Congress Committees were now to be organized on a linguistic basis so that they could
keep in touch with the people by using the local language. Hence, statement 1 is correct.
• The Congress organization was to reach down to the village and the mohalla level by the formation of
village and mohalla or ward committees. The membership fee was reduced to four annas per year to
enable the poor to become members. Hence, statement 3 is not correct.
• Mass involvement would also enable Congress to have a regular source of income. In other ways, too, the
organisational structure was both streamlined and democratized. The Congress was to use Hindi as far as
possible.

Q 93.C
• Ghadar means 'revolt' or rebellion. The Ghadar party (started in 1913) was a revolutionary group
organized to overthrow British rule in India. It was organized by overseas Indian immigrants to
Canada and the USA. The party was organized around a weekly newspaper The Ghadar which was
published from its headquarters, the Yugantar Ashram in San Francisco. The founding president of the
Ghadar party was Sohan Singh Bhakna and Lala Hardayal was a co-founder of this party. Hence
statement 1 and 2 are correct.
• The leadership also included Bhagwan Singh, Barkatullah, and Ram Chandra. The Ghadar militants
immediately began an extensive propaganda campaign against British rule. They toured extensively,
visiting mills and farms where most of the Punjabi immigrant labour worked. The Yugantar Ashram
became the home and headquarters and refuge of these political workers.
• The first issue of Ghadar was published in Urdu on 1st November 1913, the Gurumukhi edition was
letter started on 9th December. The newspaper carried the captions on the masthead: ‘Angrezi Raj ka
Dushman’ or ‘An Enemy of British Rule.’ On the front page of each issue was a feature titled Angrezi Raj
Ka Kacha Chittha or ‘An Expose of British Rule.’ This exposes consisted of 14 points enumerating the
harmful effect of the British rule in India and lost two point dealt with solutions. Hence statement 3 is
not correct.

33 www.visionias.in ©Vision IAS

FREE BY KING R QUEEN P [ऋषभ राजपूत]


• During the first World War revolutionaries of Ghadar tried to violently overthrow the British government,
they called Rash Behari Bose to lead the movement in Punjab, but the movement was successfully
dismantled by the British India government using intelligence (CID) and force.

Q 94.A
• Bengal was the most fertile and the richest of India’s provinces. Its industries and commerce were well
developed.
• As has been noted earlier, the East India Company and its servants had highly profitable trading interests
in the province.
• The Company had secured valuable privileges in 1717 under a royal farman by the Mughal
Emperor Farrukh Siyar, which had granted the Company the freedom to export and import their
goods in Bengal without paying taxes and the right to issue passes or dastaks for the movement of
such goods. Hence statement 1 is correct.
• The Company's servants were also permitted to trade but were not covered by this farman. They
were required to pay the same taxes as Indian merchants. Hence statement 2 is not correct.
• This farman was a perpetual source of conflict between the Company and the Nawabs of Bengal.
• For one, it meant loss of revenue to the Bengal Government.
• Secondly, the power to issue dastaks for the Company's goods was misused by the Company’s servants to
evade taxes on their private trade.
• All the Nawabs of Bengal, from Murshid Quli Khan to Alivardi Khan, had objected to the English
interpretation of the farman of 1717.
• They had compelled the Company to pay Jump sums to their treasury and firmly suppressed the misuse of
dastaks.
• The. The company had been compelled to accept the authority of the Nawabs in the matter, but its
servants had taken every opportunity to evade and defy this authority.

Q 95.B
• The Charter Act of 1853 carried further the separation of the executive and the legislative functions by
providing additional members of the council for the purpose of the legislation.
• The Law Member was made a full member of the Executive Council of the Governor General. The
consent of the Governor General was made necessary for all legislative proposals.
• Another important provision of the act was that it dissolved the Company’s patronage. The Charter Act
decreed that all recruits to the Civil Service were to be selected through a competitive
examination. Till 1853, all appointments to the Civil Service were made by the directors of the East
India Company, who placated the members of the Board of Control by letting them make some of the
nominations.
• The number of Directors was reduced from 24 to 18. Six of them were to be nominated by the Crown. The
Company was allowed to retain possessions of the Indian territories in trust for her majesty, her heirs and
successors until Parliament shall otherwise provide. Thus, no definite period was specified for the control
of the company over Indian affairs.
• Hence option (b) is the correct answer.

Q 96.A
• CLEANaction
o The Coalition Linking Energy and Nature for Action (CLEANaction) is a partnership to protect nature
during the energy transition. Hence, statement 1 is correct.
o The clean energy coalition of WWF says all forms of renewable power are better than fossil fuels.
CLEANaction is a partnership of several non-profits and organisations like WWF, IRENA and
BirdLife International formed to protect nature during energy transition. Hence, statement 2 is not
correct.
o It aims to highlight the need for new renewable energy generation projects to be carefully assessed for
their impacts on biodiversity.
o CLEANaction urged governments to consider the impact on nature at the earliest stage by evaluating
the renewable energy value chain (from sourcing material to disposal).
o It also urges the governments to develop national regulatory schemes that require energy developers
to contribute to national conservation targets.

34 www.visionias.in ©Vision IAS

FREE BY KING R QUEEN P [ऋषभ राजपूत]


o Findings
✓ Solar and wind energy: These transitions help achieve the International Energy Agency’s
standards of a power system capable of holding global temperature rise below 1.5 degrees
Celsius. The impact of wind energy is minimal compared to other major energy technologies,
such as fossil fuels, bioenergy, and nuclear energy.
✓ Projects: The construction of renewable energy projects, such as hydropower dams, leads to the
inundation of vast swathes of natural habitats and alters natural flow regimes and downstream
habitats.
✓ Bioenergy: In all its forms, including biomass, biofuel, and biogas, has a larger biodiversity
impact per unit of energy than that of wind and solar.

Q 97.A
• Recent context: On April 23, Manipur honours to the courageous soldiers who gave up their lives during
the war. The event is typically held at Kheba Ching in Khongjom, where Manipur’s Chief Minister and
Governor, along with other dignitaries, pay their respects to the fallen heroes.
• Khongjom Day is a significant event in the history of Manipur that commemorates the bravery of the
state’s soldiers who fought in the 1891 Anglo-Manipuri War. The battle occurred at Khongjom in
Manipur’s Thoubal district. It marked the start of the British colonialism in the region.
• After the graceful defeat at the Battle of Khongjom where the Manipuri armies fought gallantly under
Major Paona Brajabashi, Manipur lost its sovereignty to the British. And, on the fateful day of 13th
August 1891, martyrs of Manipur- Yubraj Tikendrajit, Thangal General were hanged in front of the public
• Hence option (a) is the correct answer.

Q 98.A
• The Doctrine of Lapse was used to justify the annexation of several Indian states, including Satara,
Sambalpur, Jhansi, and Nagpur. The policy was criticized by some British officials, as well as by many
Indians, who saw it as a violation of their traditional rights and customs.
• The Doctrine of Lapse was a policy introduced by Lord Dalhousie, the Governor-General of India from
1848 to 1856, during the British colonial rule in India. Hence statement 1 is correct.
• According to this doctrine, any Indian state ruled by a prince or ruler who did not have a natural
heir could be annexed by the British East India Company. Hence statement 2 is correct.
• The British annexation of Awadh, also known as Oudh, took place in 1856 under the rule of Lord
Dalhousie, the Governor-General of India. The annexation was carried out under the pretext of misrule
and maladministration by the then Awadh ruler. Hence statement 3 is not correct.
• Hence option (a) is the correct answer.

Q 99.B
• Fortnight-long discussions culminated on 5 March 1931 in the Gandhi-Irwin Pact, which was variously
described as a ‘truce’ and a ‘provisional settlement.’ The Pact was signed by Gandhiji on behalf of the
Congress and by Lord Irwin on behalf of the Government, a procedure that was hardly popular with
officialdom as it placed the Congress on an equal footing with the Government.
• The terms of the agreement included
o The immediate release of all political prisoners not convicted for violence,
o The remission of all fines not yet collected, the return of confiscated lands not yet sold to third
parties,
o Lenient treatment for those government employees who had resigned.
o The Government also conceded the right to make salt for consumption to villages along the
coast. Hence, statement 2 is correct.
o As also the right to peaceful and non-aggressive picketing.
o The Congress's demand for a public inquiry into police excesses was not accepted, but Gandhiji’s
insistent request for an inquiry was recorded in the agreement.
o The Congress, on its part, agreed to discontinue the Civil Disobedience Movement.
• It was also understood that the Congress would participate in the next Round Table Conference.
• The terms on which the Pact was signed, it's timing, the motives of Gandhiji in signing the Pact, his
refusal to make the Pact conditional on the commutation of the death sentences of Bhagat Singh and his
comrades, (even though he had tried his best to persuade the Viceroy to do so), have generated
considerable controversy and debate among contemporaries and historians alike.

35 www.visionias.in ©Vision IAS

FREE BY KING R QUEEN P [ऋषभ राजपूत]


• The Pact has been variously seen as a betrayal, as proof of the vacillating nature of the Indian
bourgeoisie and of Gandhiji succumbing to bourgeois pressure. It has been cited as evidence of
Gandhiji’s and the Indian bourgeoisie’s fear of the mass movement taking a radical turn; a betrayal
of peasants’ interests because it did not immediately restore confiscated land, already sold to a third
party, and so on. Hence, statement 1 is not correct.

Q 100.D
• Tipu Sultan was born in November 1750 to Haidar Ali and Fatima.
• A well educated man, he could freely converse in Arabic, Persian, Kanarese and Urdu.
• Tipu was a great warrior (he was known as the ‘Tiger of Mysore’) and gave maximum care to the raising
and maintenance of an efficient military force.
• He organised his army on the European model with Persian words of command.
• Though he took the help of the French officers to train his soldiers, he never allowed them (French) to
develop into a pressure group. Like his father, Tipu realised the importance of a naval force.
• In 1796, he set up a Board of Admiralty and planned for a fleet of 22 battleships and 20 large frigates.
• Three dockyards were established at Mangalore, Wajedabad and Molidabad. However, his plans did not
fructify.
• Tipu was a patron of science and technology. He is credited as the ‘pioneer of rocket technology’ in
India.
• He wrote a military manual explaining the operation of rockets. He was also a pioneer in introducing
sericulture to the Mysore State.
• Tipu was a great lover of democracy and a great diplomat.
• He gave his support to the French soldiers at Seringapatam in setting up a Jacobin Club in 1797. He
ordered a salute of 2,300 cannons and 500 rockets to celebrate the occasion.
• Tipu himself became a member of the Jacobin Club and allowed himself to be called Citizen Tipu.
• His desire to change with the times was symbolised in the introduction of a new calendar, a new
system of coinage, and new scales of weights and measures.
• His personal library contained books on such diverse subjects as religion, history, military science,
medicine, and mathematics.
• He showed a keen interest in the French Revolution.
• He planted the Tree of Liberty at Seringapatam.
• He tried to do away with the custom of giving jagirs, and thus improve state income.
• Hence, option (d) is the correct answer.

Copyright © by Vision IAS


All rights are reserved. No part of this document may be reproduced, stored in a retrieval system or
transmitted in any form or by any means, electronic, mechanical, photocopying, recording or otherwise,
without prior permission of Vision IAS.
36 www.visionias.in ©Vision IAS

FREE BY KING R QUEEN P [ऋषभ राजपूत]


VISIONIAS
www.visionias.in

Test Booklet Series

TEST BOOKLET

GENERAL STUDIES (P) 2024 – Test – 4132


C
Time Allowed: Two Hours Maximum Marks: 200

INSTRUCTIONS

1. IMMEDIATELY AFTER THE COMMENCEMENT OF THE EXAMINATION, YOU SHOULD CHECK THAT THIS BOOKLET
DOES NOT HAVE ANY UNPRINTED OR TURN OR MISSING PAGES OR ITEMS, ETC. IF SO, GET IT REPLACED BY A
COMPLETE TEST BOOKLET.

2. ENCODE CLEARLY THE TEST BOOKLET SERIES A, B, C OR D AS THE CASE MAY BE IN THE APPROPRIATE PLACE IN
THE ANSWER SHEET.

3. You have to enter your Roll Number on the Test Booklet in the Box
provided alongside. Do NOT write anything else on the Test Booklet.

4. This Test Booklet contains 100 items (Questions). Each item is printed in English. Each item comprises four
responses (answers). You will select the response which you want to mark on the Answer Sheet. In case you
feel that there is more than one correct response with you consider the best. In any case, choose ONLY ONE
response for each item.

5. You have to mark all your responses ONLY on the separate Answer Sheet provided. See direction in the
answers sheet.

6. All items carry equal marks. Attempt all items. Your total marks will depend only on the number of correct
responses marked by you in the answer sheet. For every incorrect response 1/3rdof the allotted marks will be
deducted.

7. Before you proceed to mark in the Answer sheet the response to various items in the Test booklet, you have to
fill in some particulars in the answer sheets as per instruction sent to you with your Admission Certificate.

8. After you have completed filling in all responses on the answer sheet and the examination has concluded, you
should hand over to Invigilator only the answer sheet. You are permitted to take away with you the Test
Booklet.

9. Sheet for rough work are appended in the Test Booklet at the end.

DO NOT OPEN THIS BOOKLET UNTIL YOU ARE ASKED TO DO SO


1 www.visionias.in ©Vision IAS

FREE BY KING R QUEEN P [ऋषभ राजपूत]


1. Consider the following situations in an 4. In the context of the functions of the central
economy: bank, the Reserve Bank of India carries out
1. Surge in Exports sterilization to
2. Increased government spending on (a) protect the commercial banks from the
large-scale projects rising non-performing assets (NPA).
3. High economic growth
(b) stabilize the stock of money in the
How many of the above situations can lead
economy from external shocks.
to demand-pull inflation?
(c) prevent the circulation of black money.
(a) Only one
(d) invalidate the licenses of banks violating
(b) Only two
the banking rules.
(c) All three
(d) None
5. Consider the following pairs with reference

2. Which of the following statements best to India's "Mission COVID Suraksha":

describes the ‘head count ratio’? Covid Vaccination Type


(a) It is the percentage of the unemployed 1. iNcovacc : Adenovirus vector
population in a country. Vaccine
(b) It is the percentage of the population 2. COVAXIN : Inactivated vaccine
living below the national poverty line. 3. Zydus Cadila : DNA-based vaccine
(c) It is the measure of the dropout ratio Which of the pairs given above is/are
from school education to higher correctly matched?
education.
(a) 1 and 2 only
(d) It is the percentage of people employed
(b) 2 and 3 only
in a job but not utilised for the
(c) 1 only
production of goods and services.
(d) 1, 2 and 3

3. Which of the following statements is correct


6. In order to arrive at the market price of the
about the Foreign Direct Investment (FDI)
and Foreign Portfolio Investment (FPI)? product, which of the following are added to

(a) Foreign investment greater than 10 the factor cost of the product?

percent of the paid-up capital of a 1. Total direct taxes


company can be considered as the FDI. 2. Total indirect taxes
(b) FDI is more liquid asset for an investor 3. Total subsidies
than FPI. Select the correct answer using the code
(c) FPI investors are active investors and given below.
take controlling positions in domestic (a) 1 only
firms. (b) 2 only
(d) All the statements (a), (b) and (c) are
(c) 2 and 3 only
correct.
(d) 1, 2 and 3
2 www.visionias.in ©Vision IAS

FREE BY KING R QUEEN P [ऋषभ राजपूत]


7. Consider the following statements with 10. A decision of the Reserve Bank of India to
respect to Capital Account Convertibility: increase the Cash Reserve Ratio (CRR) is
most likely to result in
1. It means freedom to convert local
(a) an increase in the inflation rate.
financial assets into foreign ones at (b) an increase the interest rates.
market-determined exchange rates. (c) an increase the aggregate demand for
2. At present, India allows only partial goods.
(d) a decrease in household savings.
convertibility in the capital account.
3. Tarapore Committee has recommended 11. Which of the following curves describes the
moving towards full capital account ‘inverse relationship between rates of
convertibility. unemployment and corresponding rates of
inflation’?
Which of the statements given above is/are
(a) Laffer curve
correct? (b) Kuznets curve
(a) 1 and 2 only (c) Philips curve
(d) Engel curve
(b) 3 only
(c) 2 and 3 only
12. Consider the following statements with
(d) 1, 2 and 3 respect to Managed Floating Exchange Rate:
1. In this exchange rate system, the central
banks intervene to buy and sell foreign
8. Under which of the following types of
currencies in an attempt to moderate
unemployment more people are doing work exchange rate movements.
than actually required? 2. It is also called the 'dirty floating'
(a) Voluntary unemployment system.
Which of the statements given above is/are
(b) Disguised unemployment
correct?
(c) Casual Unemployment (a) 1 only
(d) Chronic Unemployment (b) 2 only
(c) Both 1 and 2
(d) Neither 1 nor 2
9. Consider the following statements regarding
Participatory Notes (P-Notes): 13. To obtain personal disposable income, which
1. It is a derivative instrument. of the following should be deducted from
2. It is issued in a foreign jurisdiction by an personal Income?
1. Tax payments
RBI registered Foreign Institutional
2. Non-tax payments
Investor (FII). 3. Transfer payments to households from
Which of the statements given above is/are the government.
correct? Select the correct answer using the code
given below.
(a) 1 only
(a) 1 only
(b) 2 only (b) 1 and 2 only
(c) Both 1 and 2 (c) 1 and 3 only
(d) Neither 1 nor 2 (d) 2 and 3 only
3 www.visionias.in ©Vision IAS

FREE BY KING R QUEEN P [ऋषभ राजपूत]


14. With reference to the inflation, consider the 17. Which of the following are different sources
following statements: of data on unemployment in India?
1. Deflation is the decrease in the level of 1. Reports of Census of India
inflation. 2. Annual Reports of Periodic Labour
2. Disinflation is a general decline in prices Force Survey
for goods and services. 3. Data from registration with Employment
Which of the statements given above is/are Exchanges
correct? Select the correct answer using the code
(a) 1 only given below.
(b) 2 only (a) 1 and 2 only

(c) Both 1 and 2 (b) 2 and 3 only

(d) Neither 1 nor 2 (c) 1 and 3 only


(d) 1, 2 and 3

15. Consider the following statements regarding


18. Which of the following statements best
the GDP deflator:
explains the term ‘high-powered money’?
1. It reflects the prices of all domestically
(a) It is the portion of the public deposits
produced goods and services in the
that banks use for lending.
economy.
(b) It is the total liability of monetary
2. GDP deflator is calculated by the
authority of country.
Department of Expenditure under the
(c) It refers to only that amount of reserves
Ministry of Finance.
kept by commercial banks with the
Which of the statements given above is/are
Central bank.
correct?
(d) It is the value of Net Time and Demand
(a) 1 only
Deposits of the public held by the banks.
(b) 2 only
(c) Both 1 and 2 19. Recently, the Centre for Social and
(d) Neither 1 nor 2 Economic Progress (CSEP) in its report cited
concerns about poor critical mineral supply
16. With reference to the economy, what does chain management in India. In this context,
the term de minimis refer to? which of the above minerals can be
(a) It is the minimum level of special considered as 'critical minerals' in India?
drawing rights (SDRs) that has to be 1. Cobalt
maintained by IMF members. 2. Graphite
(b) It is the minimal amount of subsidies or 3. Niobium
domestic support that is allowed under 4. Lithium
the Agreement on Agriculture. 5. Nickel
(c) It is the minimum level of capital Select the correct answer using the code
required for a foreign bank to operate in given below.
India. (a) 1, 3 and 4 only
(d) It is the minimum amount of trade (b) 2, 4 and 5 only
required for a country to get the status of (c) 1, 2, 3 and 5 only
Most Favoured Nation (MFN) (d) 1, 2, 3, 4 and 5
4 www.visionias.in ©Vision IAS

FREE BY KING R QUEEN P [ऋषभ राजपूत]


20. Which of the following statements best 23. Which of the following are the objectives of
describes WTO's MFN (Most Favoured the Reserve Bank of India?
Nation) principle? 1. Ensuring adequate flow of credit to
(a) MFN principle states that imported and productive sectors of the economy
locally produced goods should be treated 2. Maintaining financial stability
equally after foreign goods have entered 3. To facilitate external trade
the market. 4. To give the public an adequate quantity
(b) Nations should treat all trade partners of supplies of coins
equally without any bias.
Select the correct answer using the code
(c) Among all trade partners, the country
given below.
recognized as the Most Favoured Nation
(a) 1 and 4 only
should be favored.
(b) 1, 2 and 3 only
(d) Countries with MFN status should have
(c) 2, 3 and 4 only
a Free Trade Agreement.
(d) 1, 2, 3 and 4

21. Recently, India launched Operation Kaveri


24. In the context of monetary policy, which of
to:
(a) aid in search-and-rescue in Turkey and the following are qualitative tools used by

Syria following deadly earthquakes. the Reserve Bank of India?


(b) evacuate the Indian citizens amidst the 1. Margin requirements
2022 Russian invasion of Ukraine, who 2. Moral suasion
had crossed over to neighboring 3. Changing the SLR (Statutory Liquidity
countries. Ratio)
(c) develop indigenous engines for Select the correct answer using the code
powering our own aircrafts such as LCA given below.
Tejas. (a) 2 only
(d) evacuate its nationals owing to the (b) 1 and 2 only
current crisis in Sudan. (c) 1, 2 and 3
(d) None
22. Recently, World’s second-deepest blue hole
was discovered in Chetumal Bay, off the 25. Which of the following statements best
coast of the Yucatan Peninsula in Mexico. In describes the primary deficit?
this context consider the following
(a) It refers to the excess of government’s
statements regarding blue holes:
revenue expenditure over revenue
1. They are found on coastal karst
receipts.
platforms around the world.
(b) It is the fiscal deficit of the current year
2. They are believed to have formed during
minus interest payments on previous
the latter ice ages.
borrowings.
Which of the statements given above is/are
(c) It is the difference between revenue
correct?
deficit and grants for creation of capital
(a) 1 only
(b) 2 only assets.

(c) Both 1 and 2 (d) It is the excess of total expenditure over


(d) Neither 1 nor 2 total receipts excluding borrowings.
5 www.visionias.in ©Vision IAS

FREE BY KING R QUEEN P [ऋषभ राजपूत]


26. Consider the following statements with 29. Consider the following statements:
respect to currency swap agreements: 1. Piezometers measure groundwater levels
while digital water level recorders
1. It is an agreement between the central
transmit the recorded information
banks of the two countries. digitally.
2. One country exchanges its national 2. Groundwater contamination is mostly
currency for that of another or even a “geogenic” rather than anthropogenic.
Which of the statements given above is/are
third one.
correct?
Which of the statements given above is/are (a) 1 only
correct? (b) 2 only
(a) 1 only (c) Both 1 and 2
(d) Neither 1 nor 2
(b) 2 only
(c) Both 1 and 2
30. Consider the following statement:
(d) Neither 1 nor 2 1. a shift away from calorie consumption
based poverty estimation
2. a uniform poverty line basket (PLB)
27. Consider the following pairs:
across rural and urban India
Air Exercises of Partner Country
3. a change in the price adjustment
India procedure to correct spatial and temporal
1. Cope India : The United States issues with price adjustment
2. Veer Guardian : Japan 4. incorporation of private expenditure on
health and education while estimating
3. Pitch Black : Australia
poverty
Which of the pairs given above is/are How many of the above were recommended
correctly matched? by the Tendulkar committee, constituted to
(a) 1 and 2 only review methodology for poverty estimation?
(a) Only one
(b) 2 only
(b) Only two
(c) 1 and 3 only (c) Only three
(d) 1, 2 and 3 (d) All four

31. Consider the following statements with


28. The Basel Norms were introduced by the
reference to the measures of money supply
Reserve Bank of India (RBI) mainly to in India:
(a) Improve the banking sector’s ability to 1. Among all the measures of money
absorb shocks arising from financial supply, M4 is the most liquid.
2. M1 is the most commonly used measure
stress
of money supply and is known as
(b) Depolitise the appointment of Board of aggregate monetary resources.
Directors in Nationalised Banks Which of the statements given above is/are
(c) reduce the supervision of the RBI over correct?
(a) 1 only
foreign banks with more than 10
(b) 2 only
branches in the country
(c) Both 1 and 2
(d) Expand bank network in remote areas (d) Neither 1 nor 2
6 www.visionias.in ©Vision IAS

FREE BY KING R QUEEN P [ऋषभ राजपूत]


32. Which of the following is not used to 35. With reference to the Marginal Standing
Facility (MSF), consider the following
calculate national income?
statements:
(a) Expenditure method 1. Its aim is to reduce volatility in the
(b) Product method overnight lending rates in the interbank
market.
(c) Balance of Payment method
2. Borrowing rate under this facility is
(d) Income method always higher than the repo rate.
3. The facility was introduced by the
Reserve Bank of India during the 2016
33. Consider the following statements: demonetization.
1. Depreciation is the regular wear and tear Which of the statements given above is/are
correct?
of the capital
(a) 1 only
2. Depreciation is accounted for even if no (b) 2 only
actual expenditure has occurred in an (c) 1 and 2 only
(d) 1, 2 and 3
economy.
3. Depreciation also takes into account the 36. In the context of capital markets, consider
the following statements:
sudden unexpected destruction of
1. Capital markets provide direct funding
capital. from savers to users via the issuance of
Which of the statements given above is/are securities.
2. In this market, capital funds comprising
not correct?
both equity and debt are issued and but
(a) 1 only not traded.
(b) 1 and 3 only Which of the statements given above is/are
correct?
(c) 2 and 3 only (a) 1 only
(d) 3 only (b) 2 only
(c) Both 1 and 2
(d) Neither 1 nor 2
34. Consider the following statements:
1. Production taxes and subsidies are 37. Which of the following currencies form a
part of Special Drawing Rights (SDR)'s
independent of the volume of
Basket of currencies:
production. 1. US Dollar
2. Product taxes and subsidies are paid pair 2. Japanese Yen
3. Chinese Renminbi
or received per unit product. 4. Indian Rupee
Which of the statements given above is/are 5. Euro
6. British Pound sterling
correct?
select the correct answer using the code
(a) 1 only given below.
(b) 2 only (a) 1, 2, 3 and 4 only
(b) 2, 3, 4, 5 and 6 only
(c) Both 1 and 2
(c) 1, 4, 5 and 6 only
(d) Neither 1 nor 2 (d) 1, 2, 3, 5 and 6 only
7 www.visionias.in ©Vision IAS

FREE BY KING R QUEEN P [ऋषभ राजपूत]


38. Which of the following financial institutions 41. Consider the followings receipts of the
in India are required to maintain the Union Government:
Statutory Liquidity Ratio (SLR) norms? 1. Recoveries of loans and advances given
1. Regional Rural Banks to state governments
2. Local Area Banks 2. Disinvestment proceeds of PSUs
3. Co-operative Banks 3. Recoveries of loans from foreign
4. Small Finance Banks governments
Select the correct answer using the code How many of the above receipts are non-
given below. debt capital receipts?
(a) 1 and 4 only (a) Only one
(b) 2 and 3 only (b) Only two
(c) 1, 2 and 4 only (c) All three
(d) 1, 2, 3 and 4 (d) None

39. "The experiment consists of detectors that 42. In context of the economy, what does
are blind to the light from the Universe,
'autarky' mean?
arranged at right angles to each other. The
(a) An economy having no taxation
Union Cabinet recently approved a project to
mechanism
build such an advanced facility in
(b) An open economy is one that trades with
Maharashtra. By doing so, “Indian science
other nations in goods and services and
and technology is expected to leap-frog in
also in financial assets.
quantum-sensing and metrology.
(c) It is a closed economy where no external
Which of the following experiments has
trade takes place at all.
been described in the passage given above?
(d) An economy where sectors are only
(a) Laser Interferometer Gravitational-wave
managed by state having no private
Observatory (LIGO)
sector,
(b) The Large Hadron Collider (LHC)
(c) Evolved Laser Interferometer Space
43. Consider the following statements:
Antenna (eLISA)
1. India is expected to overtake China as
(d) LISA pathfinder
the most populous nation in the world by
the end of 2026.
40. Which of the following are the major
2. India has more population in the age
sources of non-tax revenue?
1. Fees for Communication Services group 0-14 years compared to the age

2. License fee group above 65.

3. Interests 3. India has a better life expectancy

4. Fines and penalties compared to China.


Select the correct answer using the code Which of the statements given above is/are
given below. correct?
(a) 1, 2, and 4 only (a) 1 and 2 only
(b) 2 and 3 only (b) 2 only
(c) 1, 3, and 4 only (c) 3 only
(d) 1, 2, 3 and 4 (d) 1, 2 and 3
8 www.visionias.in ©Vision IAS

FREE BY KING R QUEEN P [ऋषभ राजपूत]


44. With reference to the Markets in Crypto 47. Consider the following statements:
Assets (MiCA) regulation, consider the 1. Real GDP changes express changes in
production output in the country.
following statements:
2. A nominal GDP increase necessarily
1. It has been approved by the European means that the production output has
Union (EU) to bring largely unregulated increased.
cryptocurrency markets under Which of the statements given above is/are
correct?
government regulation.
(a) 1 only
2. It will also regulate central bank digital (b) 2 only
currencies issued by the European (c) Both 1 and 2
Central Bank. (d) Neither 1 nor 2

Which of the statements given above is/are


48. Consider the following statements:
correct? 1. An increase in the GDP always leads to
(a) 1 only the welfare of people.
(b) 2 only 2. Negative externalities are not taken into
account while calculating GDP
(c) Both 1 and 2
3. Non-monetary exchanges are not
(d) Neither 1 nor 2 registered as part of economic activities
in GDP calculation.
45. Which of the following items are reflected Which of the statements given above are
correct?
on the assets side of a balance sheet of a
(a) 1 and 2 only
Scheduled Commercial bank in India? (b) 2 and 3 only
1. Loan advances (c) 1 and 3 only
2. Net Time Deposits held by a bank (d) 1, 2 and 3
3. Government Securities held by the bank
49. Which of the following is not a flow
4. Cash Reserve Ratio (CRR) variable?
Select the correct answer using the code (a) Depreciation
given below. (b) National Income
(c) Wealth
(a) 1 and 3 only
(d) Import
(b) 1, 3 and 4 only
(c) 2 and 4 only 50. Consider the following statements:
(d) 1, 2, 3 and 4 1. The real effective exchange rate (REER)
is the relative price of the goods of two
countries.
46. Recently, which of the following rice variety
2. The nominal effective exchange rate
of Bihar has been accorded ‘Geographical (NEER) is the relative price of the
Indication’ status for its unique aroma and currencies of two countries.
palatability? Which of the statements given above is/are
correct?
(a) Mircha rice
(a) 1 only
(b) Gobindobhog rice (b) 2 only
(c) Joha rice (c) Both 1 and 2
(d) Tulaipanji rice (d) Neither 1 nor 2
9 www.visionias.in ©Vision IAS

FREE BY KING R QUEEN P [ऋषभ राजपूत]


51. With reference to 5th cycle of India’s Tiger 54. It refers to the time lag between jobs when
Census in India, consider the following an individual is searching for a new job or is
statements: switching between jobs. Which types of
1. The only landscape in India where the unemployment have been referred to above?
tiger population has gone down is the (a) Frictional unemployment
Eastern Ghats. (b) Technological unemployment
2. The National Tiger Conservation (c) Seasonal unemployment
Authority, is the nodal authority to (d) Cyclical Unemployment
conduct the estimation of tiger census.
Which of the statements given above is/are 55. Recently, which of the following
correct? organisations has launched 'The
(a) 1 only Preparedness and Resilience for Emerging
(b) 2 only Threats' (PRET) Initiative?
(c) Both 1 and 2 (a) World Health Organisation
(d) Neither 1 nor 2 (b) United Nations Framework Convention
on Climate Change
52. Consider the following statements regarding (c) World Bank
the Wholesale price index (WPI): (d) Food and Agriculture Organization
1. It is published by the Economic Advisor
in the Ministry of Commerce and 56. Which of the following statements are
Industry. correct with reference to the contribution of
2. Fuel and power are given highest Jagdish Chandra Bose?
weightage in it. 1. He was one of the pioneers in wireless
3. It tracks inflation at the producer level communication in India.
only. 2. He invented the crescograph for
4. It does not capture changes in the prices measuring the growth of plants.
of services. 3. He was the first to demonstrate radio
Which of the statements given above is/are communication with millimeter
not correct? wavelengths.
(a) 2 only Select the correct answer using the code
(b) 1 and 4 only given below.
(c) 2 and 3 only (a) 1 and 2 only
(d) 3 and 4 only (b) 2 and 3 only
(c) 1 and 3 only
53. Which of the following are included in the (d) 1, 2 and 3
quota formula for Special Drawing Rights in
International Monetary Fund (IMF) 57. Which of the following indices are published
finances? by the World Bank?
1. Population Size 1. World Economic Outlook
2. Weighted average of GDP 2. Global Economic Prospect (GEP) Report
3. Openness 3. World Development Report
4. Economic Variability 4. Global Financial Stability Report
Select the correct answer using the code Select the correct answer using the code
given below. given below.
(a) 2, 3 and 4 only (a) 1, 2 and 4 only
(b) 1, 3 and 4 only (b) 1, 3 and 4 only
(c) 1, 2 and 4 only (c) 3 and 4 only
(d) 1, 2 and 3 only (d) 2 and 3 only
10 www.visionias.in ©Vision IAS

FREE BY KING R QUEEN P [ऋषभ राजपूत]


58. The idea of Five Year Plans was taken from 63. Which one of the following statements best
which of the following countries? reflects the idea behind the " PSLV Orbital
(a) Japan Experimental Module " often seen in the
(b) Ireland news?
(c) Australia (a) A reusable launch vehicle has parts that
(d) Soviet Union
can be recovered and reflown, while
carrying payloads from the surface to
59. “Ricardian Equivalence” is associated with
outer space
which of the following?
(a) Foreign exchange reserves (b) An orbital platform to carry out
(b) Trade deficit scientific experiments through non-
(c) Government spending separating payloads
(d) Money supply (c) A warhead delivery system that uses a
low earth orbit towards its target
60. Consider the following statements with destination on the Earth
respect to masala bonds: (d) An early warning system in space to
1. They are bonds issued in the Indian detect debris and other hazards to Indian
capital market.
satellites
2. In masala bonds, the currency risk lies
with the investor and not the issuer.
64. Consider the following documents:
Which of the statements given above is/are
correct? 1. Finance Bill
(a) 1 only 2. Outcome Budget
(b) 2 only 3. Medium Term Fiscal Policy Statement
(c) Both 1 and 2 4. Economic Survey
(d) Neither 1 nor 2 5. Demands for Grants
How many of the above documents comprise
61. In the context of external sector, 'Special the Budget presented in the Parliament?
Drawing Rights' (SDR) is an international (a) Only two
reserve asset to supplement its member
(b) Only three
countries’ official reserves. It is created by
(c) Only four
which of the following institutions?
(d) All five
(a) World Bank
(b) World Trade Organization
(c) International Monetary Fund 65. The term ‘Rules of Origin’ is sometimes
(d) US Federal Reserve seen in the news with reference to
(a) Permitting foreign educational
62. Which of the following are direct taxes? institutions to set up their campuses in
1. Wealth tax the domestic country.
2. Gift tax (b) Regulating investment through
3. Excise duty Participatory-Notes to prevent round
4. Corporation tax
tripping of black money.
Select the correct answer using the code
(c) Granting licenses to foreign e-commerce
given below.
firms to operate in the domestic country.
(a) 1, 2, and 4 only
(d) Developing trading norms to prevent
(b) 2 and 3 only
(c) 1, 3, and 4 only dumping of foreign goods in the
(d) 1, 2, 3 and 4 domestic market.
11 www.visionias.in ©Vision IAS

FREE BY KING R QUEEN P [ऋषभ राजपूत]


66. With reference to Olive Ridley Turtles, 69. Consider the following statements regarding
consider the following statements: the Consumer Price Index (CPI):
1. They are the largest of all sea turtle 1. It measures the change in prices paid by
species in the world and are categorized consumers for goods and services both.
as vulnerable by the International Union 2. It is measured by the Ministry of
for Conservation of Nature. Statistics and Programme
2. Generally, an Olive Ridley turtle lays Implementation with base year 2012.
100-150 eggs. 3. The weightage of food and beverages in
3. Rushikulya Beach is a notified wildlife the CPI is close to 50%.
sanctuary for Olive Ridley Turtles in the How many of the statements given above are
state of Odisha. correct?
Which of the statements given above is/are (a) Only one
correct? (b) Only two
(a) 1 and 3 only (c) All three
(b) 2 and 3 only (d) None
(c) 2 only
(d) 1, 2 and 3 70. Which of the following spacecraft has
operated outside the heliosphere, considered
67. Consider the following pairs: to be the border of our solar system?
Biodiversity Heritage Site Located in (a) New Horizons
1. Gandhamardan Hill : Jharkhand (b) Cassini Huygens
2. Arittapatti : Tamil Nadu (c) Juno spacecraft
3. Namthing Pokhari : Assam (d) Voyager 2
How many pairs given above are correctly
matched? 71. Which of the following are the main items of
(a) All three pairs capital receipts?
(b) Only one pair 1. Market borrowings
(c) Only two pairs 2. Profits of public enterprises
(d) None of the pairs 3. National Savings Certificate
4. Provident funds
68. Dakar Declaration, recently seen in the Select the correct answer using the code
news, is related to: given below.
(a) Global elimination of Tuberculosis (a) 1, 2, and 4 only
(b) Cyber security (b) 2 and 3 only
(c) Global elimination of AIDS (c) 1, 3, and 4 only
(d) Road safety (d) 1, 2, 3 and 4
12 www.visionias.in ©Vision IAS

FREE BY KING R QUEEN P [ऋषभ राजपूत]


72. In the context of banking reforms for 75. Consider the following statements with
strengthening the financing system in rural reference to Regional Rural Banks (RRBs):
India, the year 1982 is important because 1. Regional Rural Banks were set up on the
(a) the first regional rural bank came into basis of the recommendations of the
being that year. Narasimham Working Group (1975).
(b) it was the year in which the first phase of 2. The equity of a regional rural bank is
nationalisation of banks took place. completely owned by the state
(c) National Bank for Agriculture and Rural government.
Development (NABARD) was set up. Which of the statements given above is/are
(d) the Reserve Bank of India introduced correct?
and formalised the concept of priority (a) 1 only
sector lending. (b) 2 only
(c) Both 1 and 2
73. Consider the following statements regarding (d) Neither 1 nor 2
the inflation targeting in Indian economy:
1. Inflation targeting is a statutory and 76. In the context of economy, which of the
institutionalised framework under the following best describes the term "official
RBI Act, 1934. reserve sale"?
2. Inflation target is set up by the RBI in (a) It is the sale of foreign exchange by the
consultation with the central reserve bank when there is a deficit in its
government. balance of payments.
Which of the statements given above is/are (b) It is the sale of government stakes in
correct? public sector units for the process of
(a) 1 only disinvestment.
(b) 2 only (c) It is the printing of currency by the
(c) Both 1 and 2 reserve bank for financing the fiscal
(d) Neither 1 nor 2 deficit.
(d) It is the sale of government securities by
74. Consider the following: the RBI.
1. Medium Term Fiscal Policy Statement
2. Macroeconomic Framework Statement 77. Which of the following statements best
3. Fiscal Policy Strategy Statement describes the term inflationary gap?
Under the FRBM Act, how many of the (a) It is the difference between the real and

above are mandatory for the government to nominal GDP of a country.

place along with the Union Budget (b) It is the difference between the current

documents in the Parliament annually? real GDP and the GDP of an economy

(a) Only one operating at full employment.

(b) Only two (c) It is the difference between the GNP and

(c) All three GDP of a country

(d) None (d) None


13 www.visionias.in ©Vision IAS

FREE BY KING R QUEEN P [ऋषभ राजपूत]


78. With reference to PM Street Vendor’s 80. Consider the following pairs:

AtmaNirbhar Nidhi (PM SVANidhi) General Agreements Description


scheme, consider the following statements: on Trade in Services
1. It is a Central Sector Scheme to facilitate (GATS) Mode
street vendors' access to collateral-free 1. Mode 1 : Presence of
capital loans.
natural persons
2. It facilitates an initial working capital
2. Mode 2 : Cross-border
loan of up to 10 lakhs at a subsidized
trade
rate of interest.
3. Mode 3 : Consumption
3. Street vendors/ hawkers of both the
abroad
urban and rural areas are covered under
4. Mode 4 : Commercial
it.
presence
Which of the statements given above is/are
How many pairs given above are correctly
correct?
matched?
(a) 1 and 2 only
(b) 2 only (a) Only one pair

(c) 1 and 3 only (b) Only two pairs

(d) 3 only (c) All four pairs

(d) None

79. Which of the following statements is correct


about the Balance of Payments? 81. With reference to India's biodiversity,
(a) It is a record of only goods and services "Mangrove pitta" seen in eastern India is a
exchanged between the residents of a (a) Tree
country with the rest of the world.
(b) Bird
(b) It is a systematic record of all economic
(c) Snake
transactions between the residents of one
(d) Butterfly
country and the rest of the world.

(c) It includes transactions consisting of


82. The "National Credit Framework" has been
imports and exports of goods, services,
formulated to fulfill the vision of
and capital but excludes foreign aid and
(a) National Education Policy
remittances.
(b) New industrial policy
(d) It includes all the domestic assets and
(c) Biofuel Policy
liabilities of the government of India,
kept by RBI. (d) National Health Policy

14 www.visionias.in ©Vision IAS

FREE BY KING R QUEEN P [ऋषभ राजपूत]


83. Consider the following statements with 86. With reference to the 'Logistics Performance
respect to subsidies in World Trade Index', consider the following statements:
Organization: 1. It is released by the United Nations
Conference on Trade and Development.
1. Green box subsidies are certain direct
2. India's ranking in the Logistics
payments to farmers where the farmers
Performance Index has improved in the
are required to limit production.
last five years.
2. Amber box subsidies are domestic Which of the statements given above is/are
policies that do not have a direct effect correct?
on production and trade. (a) 1 only
Which of the statements given above is/are (b) 2 only
correct? (c) Both 1 and 2
(d) Neither 1 nor 2
(a) 1 only
(b) 2 only
87. Which of the following are known as the
(c) Both 1 and 2 Bretton Woods Institutions (BWIs)?
(d) Neither 1 nor 2 1. International Monetary Fund
2. World Bank
84. It reflects people’s preference for liquidity 3. World Trade Organization
and is a purely behavioral parameter that 4. World Economic Forum
depends, among other things, on the seasonal Select the correct answer using the code
given below.
pattern of expenditure. Which of the
(a) 1 and 2 only
following ratios has been described in the
(b) 1, 2 and 3 only
passage given above? (c) 3 and 4 only
(a) Currency Deposit Ratio (d) 1, 2, 3 and 4
(b) Reserve Deposit Ratio
(c) Statutory Liquidity Ratio 88. The terms 'Ernie and Bard', sometimes
(d) Cash Reserve Ratio mentioned in the news recently are
(a) Antivirus software
(b) Trojans
85. Which of the following are part of the World
(c) Generative Artificial Intelligence
Bank Group?
(d) Ransomware
1. The Multilateral Investment Guarantee
Agency (MIGA) 89. Which of the following statements is not
2. The International Development correct about the Pradhan Mantri Jan-Dhan
Association Yojana (PMJDY), the National Mission for
3. The International Centre for Settlement Financial Inclusion?
of Investment Disputes (a) No interest is earned on the deposit in
PMJDY accounts.
4. The International Finance Corporation
(b) An overdraft (OD) facility up to Rs.
Select the correct answer using the code
10,000 to eligible account holders is
given below. available.
(a) Only one (c) Accident Insurance Cover of Rs. 2 lakh
(b) Only two for new PMJDY accounts is available.
(c) Only three (d) Rupay Debit card is provided to PMJDY
(d) All four account holders.
15 www.visionias.in ©Vision IAS

FREE BY KING R QUEEN P [ऋषभ राजपूत]


90. Consider the following statements regarding 93. Consider the following statements regarding
the direct monetisation of the fiscal deficit: India’s Civil Liability for Nuclear Damage
1. It is not allowed under the Fiscal
Act (CLNDA), 2010:
Responsibility and Budget Management
Act (FRBM Act), 2003. 1. This act deals with a speedy
2. The exercise reduces the total money compensation mechanism for victims of
supply in the economy. a nuclear accident.
Which of the statements given above is/are
2. It provides for strict and no-fault liability
correct?
(a) 1 only on the operator of the nuclear plant,
(b) 2 only where it will be held liable for damage
(c) Both 1 and 2
regardless of any fault on its part.
(d) Neither 1 nor 2
3. It sets a financial cap on the operator’s

91. Which of the following best defines the term liability at INR 1,500 crore for each
'liquidity'? nuclear incident.
(a) The ease with which an asset can be
Which of the statements given above are
converted into cash without a significant
correct?
loss of value.
(b) The degree to which an asset is able to (a) 1, 2 and 3
appreciate in value over time. (b) 2 and 3 only
(c) The ability of an asset to generate
(c) 1 and 3 only
income over time.
(d) The potential return on investment in a (d) 1 and 2 only
particular asset.
94. Consider the following statements regarding
92. Consider the following statements:
Self-Reliant India (SRI) fund:
Statement-I: The RBI Governor is not a
member of the Monetary Policy Committee. 1. This fund provides equity funding to
Statement-II: The Monetary Policy micro, small, and medium enterprises
Committee replaced the RBI Governor as the (MSMEs) in India.
authority for inflation-targeting.
2. It is a SEBI-registered category-II
Which one of the following is correct in
respect of the above statements? Alternative Investment Fund (AIF).
(a) Both Statement-I and Statement-II are 3. The SRI fund is operated by a
correct and Statement-II is the correct consortium of private agencies approved
explanation for Statement-I.
by the SEBI.
(b) Both Statement-I and Statement-II are
correct and Statement-II is not the Which of the statements given above are
correct explanation for Statement for correct?
Statement-I.
(a) 1 and 2 only
(c) Statement-I is correct but Statement-II is
(b) 1 and 3 only
incorrect
(d) Statement-I is incorrect but Statement-II (c) 2 and 3 only
is correct (d) 1, 2 and 3
16 www.visionias.in ©Vision IAS

FREE BY KING R QUEEN P [ऋषभ राजपूत]


95. Consider the following pairs: 97. With reference to Open Market Operations
(OMOs), consider the following statements:
Type of Derivative Description
1. In outright open operations, the
1. Forwards : settlement takes place injection/absorption of the money is of
on a specific date in permanent nature.
2. A Repo is a type of open market
the future at today's
operation in which the central bank
pre-agreed price
promises to buy the securities at a fixed
2. Options : give the right, but not date and fixed price.

an obligation Which of the statements given above


is/are correct?
3. Warrants : longer-dated option (a) 1 only
How many pairs given above are correctly (b) 2 only
(c) Both 1 and 2
matched?
(d) Neither 1 nor 2
(a) Only one pair
(b) Only two pairs 98. Consider the following statements with
(c) Only three pairs respect to the Balance of Current Account:
1. It is a measure of all payments made for
(d) None
currently produced goods and services.
2. It includes factor income from abroad
96. Consider the following statements regarding excluding the international transfer
payments.
the fiscal policy:
Which of the statements given above is/are
1. A countercyclical fiscal policy means correct?
increased government spending and (a) 1 only
(b) 2 only
reduced taxes during a boom period, and
(c) Both 1 and 2
reduced spending and increased taxes (d) Neither 1 nor 2
during a recession.
2. A procyclical fiscal policy means 99. Consider the following statements with
respect to International Monetary Fund:
reducing spending and raising taxes
1. It was established along with the
during a boom period, and increasing International Bank for Reconstruction
spending and cutting taxes during a and Development.
2. It is a specialized agency of the United
recession.
Nations.
Which of the statements given above is/are 3. India is a founder member of the IMF.
correct? Which of the statements given above are
correct?
(a) 1 only
(a) 2 and 3 only
(b) 2 only (b) 1 and 3 only
(c) Both 1 and 2 (c) 1, 2 and 3
(d) Neither 1 nor 2 (d) 1 and 2 only
17 www.visionias.in ©Vision IAS

FREE BY KING R QUEEN P [ऋषभ राजपूत]


100. The Mahalanobis Plan, which focused on
rapid industrialization- heavy & basic
industries is related to which of the
following five year plans?
(a) First Five Year Plan
(b) Second Five Year Plan
(c) Third Five Year Plan
(d) Eighth Five Year Plan

Copyright © by Vision IAS


All rights are reserved. No part of this document may be reproduced, stored in a retrieval system or
transmitted in any form or by any means, electronic, mechanical, photocopying, recording or otherwise,
without prior permission of Vision IAS.
18 www.visionias.in ©Vision IAS

FREE BY KING R QUEEN P [ऋषभ राजपूत]


VISIONIAS
www.visionias.in
ANSWERS & EXPLANATIONS
GENERAL STUDIES (P) TEST – 4132 (2024)

Q 1.C
• Demand-pull inflation is a type of inflation that is influenced by growing demand for a good or service.
When the aggregate demand is higher than the aggregate supply, prices will rise. It is commonly described
as "too much money chasing too few goods."
• What Causes Demand-Pull Inflation?
o Economic Growth: When an economy is thriving, people and businesses tend to feel more confident
in spending their money. When this happens, general demand rises and many businesses may have
trouble keeping pace with the increased demand.
o Surge in Exports: Exchange rate depreciation can drive aggregate demand and create demand-pull
inflation by encouraging a high level of exports. Typically when this happens people in a country buy
fewer imports while at the same time exports from their country increase.
o Government Spending: When the government begins spending on large-scale projects, this often
drives prices up. This is because substantial projects funded by massive amounts of capital that
governments provide creates more demand overall. Fiscal policies that drive demand can also create
demand-pull inflation.
o Inflation Forecasts: When economists, the government, or major media outlets forecast inflation, this
can unintentionally cause demand-pull inflation through a couple of avenues. First, some companies
may raise their prices preemptively to meet the expected inflation. Second, some consumers may
make major purchases preemptively to avoid paying higher prices later. This can create greater
demand and result in demand-pull inflation.
o Exorbitant Supply of Money: When a government prints too much money, this can create demand-
pull inflation. In this case, the popular definition of demand-pull inflation"too much money chasing
too few goods" applies quite literally.
• Hence option (c) is the correct answer.

Q 2.B
• A common method used to estimate poverty in India is based on the income or consumption levels and if
the income or consumption falls below a given minimum level, then the household is said to be Below the
Poverty Line (BPL).
• Poverty: According to the World Bank, Poverty is pronounced deprivation in well-being and comprises
many dimensions. It includes low incomes and the inability to acquire the basic goods and services
necessary for survival with dignity.
• Poverty Line: The conventional approach to measuring poverty is to specify a minimum expenditure (or
income) required to purchase a basket of goods and services necessary to satisfy basic human needs and
this minimum expenditure is called the poverty line.
• Poverty Line Basket: The basket of goods and services necessary to satisfy basic human needs is the
Poverty Line Basket (PLB).
• Poverty Ratio: The proportion of the population below the poverty line is called the poverty ratio or
headcount ratio (HCR). Hence option (b) is the correct answer.

Q 3.A
• According to IMF and OECD definitions, the acquisition of at least ten percent of the ordinary shares
or voting power in a public or private enterprise by non-resident investors makes it eligible to be
categorized as a foreign direct investment (FDI). In India, a particular FII is allowed to invest up to

1 www.visionias.in ©Vision IAS

FREE BY KING R QUEEN P [ऋषभ राजपूत]


10% of the paid-up capital of a company, which implies that any investment above 10% will be
construed as FDI, though officially such a definition did not exist. It may be noted that there is no
minimum amount of capital to be brought in by the foreign direct investor to get the same categorized as
FDI. Hence option (a) is the correct answer.
• FDI investors cannot easily liquidate their assets and depart from a nation, since such assets may be
very large and quite illiquid. FPI investors can exit a nation literally with a few mouse clicks, as financial
assets are highly liquid and widely traded.
• FDI can be used to develop infrastructure, set up manufacturing facilities and service hubs, and invest in
other productive assets such as machinery and equipment, which contributes to economic growth and
stimulates employment. FDI is obviously the route preferred by most nations for attracting foreign
investment since it is much more stable than FPI and signals long-lasting commitment.
• FDI investors typically take controlling positions in domestic firms or joint ventures and are actively
involved in their management. FPI investors, on the other hand, are generally passive investors who
are not actively involved in the day-to-day operations and strategic plans of domestic companies,
even if they have a controlling interest in them.

Q 4.B
• Sterilization by the Reserve Bank of India (RBI): The RBI often uses its instruments of money
creation for stabilizing the stock of money in the economy from external shocks. Suppose due to
future growth prospects in India investors from across the world increase their investments in Indian
bonds which under such circumstances, are likely to yield a high rate of return. They will buy these
bonds with foreign currency.
• Since one cannot purchase goods in the domestic market with foreign currency, a person or a financial
institution that sells these bonds to foreign investors will exchange its foreign currency holding into a
rupee at a commercial bank. The bank, in turn, will submit this foreign currency to RBI and its deposits
with RBI will be credited with an equivalent sum of money. The commercial bank’s total reserves and
deposits remain unchanged (it has purchased the foreign currency from the seller using its vault cash,
which, therefore, goes down; but the bank’s deposit with RBI goes up by an equivalent amount – leaving
its total reserves unchanged).
• There will, however, be increments in the assets and liabilities on the RBI balance sheet. RBI’s
foreign exchange holding goes up. On the other hand, the deposits of commercial banks with RBI also
increase by an equal amount. But that means an increase in the stock of high-powered money – which, by
definition, is equal to the total liability of RBI.
• With a money multiplier in operation, this, in turn, will result in an increased money supply in the
economy. This increased money supply may not altogether be good for the economy’s health. If the
volume of goods and services produced in the economy remains unchanged, the extra money will lead to
an increase in the prices of all commodities.
• People have more money in their hands with which they compete each other in the commodities
market for buying the same old stock of goods. As too much money is now chasing the same old
quantities of output, the process ends up in bidding up prices of every commodity – an increase in the
general price level, which is also known as inflation.
• RBI often intervenes with its instruments to prevent such an outcome. In the above example, RBI will
undertake an open market sale of government securities of an amount equal to the amount of foreign
exchange inflow in the economy, thereby keeping stock of high-powered money and total money supply
unchanged. Thus it sterilizes the economy against adverse external shocks. This operation of RBI is
known as sterilization. Hence option (b) is the correct answer.

Q 5.D
• The Department of Biotechnology (DBT) in the Ministry of Science & Technology has through
“Mission COVID Suraksha”, delivered four vaccines, augmented the manufacturing of Covaxin,
and created the necessary infrastructure for the smooth development of future vaccines so that our
country is pandemic ready. These vaccines were developed in collaboration with various
organizations.
• The four vaccines are- ZyCoV-D- World’s 1st and India’s indigenously developed DNA Vaccine,
CORBEVAXTM-India’s first protein subunit vaccine, GEMCOVAC™-19 - World’s 1st and
India’s indigenously developed mRNA vaccine and iNCOVACC-World’s 1st and India’s
indigenously developed intranasal COVID-19 Vaccine.

2 www.visionias.in ©Vision IAS

FREE BY KING R QUEEN P [ऋषभ राजपूत]


• The world’s first intranasal vaccine, christened as iNcovacc, is based on a delivery vehicle, a virus,
that causes the common cold (adenovirus vector). The spike protein on the virus is the one that attaches to
human cells to infect them. The spike protein on the vector is stabilized with appropriate changes, for a
stable form at delivery, which is more effective in producing immunity or resistance to a virus. It is a
recombinant replication-deficient adenovirus vectored vaccine with a pre-fusion stabilized spike
protein. Hence pair 1 is correctly matched.
o It is different from Covaxin as the latter is an inactivated (killed) vaccine administered through
intramuscular injections. It will be delivered as nasal drops, not as a spray, and to be given as
four drops/each nostril, a total of 8 drops in a slow drip process of delivery, rather than a single
shot/spray.
• COVAXIN, India's indigenous COVID-19 vaccine by Bharat Biotech is developed in collaboration with
the Indian Council of Medical Research (ICMR) - National Institute of Virology (NIV). The
indigenous, inactivated vaccine is developed and manufactured in Bharat Biotech's BSL-3 (Bio-Safety
Level 3) high containment facility
• The vaccine is developed using Whole-Virion Inactivated Vero Cell-derived platform technology.
Inactivated vaccines do not replicate and are therefore unlikely to revert and cause pathological effects.
They contain dead viruses, incapable of infecting people but still able to instruct the immune system to
mount a defensive reaction against an infection. Hence pair 2 is correctly matched.
• Why develop Inactivated Vaccine? Conventionally, inactivated vaccines have been around for decades.
Numerous vaccines for diseases such as Seasonal Influenza, Polio, Pertussis, Rabies, and Japanese
Encephalitis use the same technology to develop inactivated vaccines with a safe track record of >300
million doses of supplies to date. It is the well-established, and time-tested platform in the world of
vaccine technology.
• Zydus Cadila, the world’s first and India’s indigenously developed DNA-based vaccine is for Children
and adults 12 years and above. It was developed in partnership with the Department of Biotechnology,
Government of India under the ‘Mission COVID Suraksha’ and implemented by BIRAC. Hence pair 3 is
correctly matched.

Q 6.B
• Factor cost is the cost of an item of goods or a service in terms of the various factors which have
played a part in its production or availability.
• In order to arrive at the market prices, we have to add to the factor cost the total indirect taxes less
total subsidies. Also, it does not generally include direct taxes. Hence option (b) is the correct
answer.
• Once goods and services are produced they are sold in a marketplace at a set market price. The market
price is the price that consumers will pay for the product when they purchase it from the sellers. Taxes
charged by the government will be added to the factor price while subsidies provided will be reduced
from the factor price to arrive at the market price. Taxes are added on because taxes are costs that increase
the price, and subsidies are reduced because subsidies are already included in the factor cost, and cannot
be double-counted when the market price is calculated. The market price will be decided, depending on
the cost of production, demand for the product, and prices that are charged by competitors. In economics,
the market price is identified as the price at which demand for the product or service is equal to its supply.
Changes in the levels of demand and supply, cost of factor inputs, and other economic and environmental
conditions can affect the market price of a good or service.

Q 7.D
• Capital Account Convertibility (CAC)
o Currency convertibility is the ease with which a country's currency can be converted into gold or
another currency.
o It means freedom to convert local financial assets into foreign ones at market-determined exchange
rates. Hence, statement 1 is correct.
o Currently, India allows full convertibility in the current account but only partial convertibility in the
capital account. Hence, statement 2 is correct.
o S. S. Tarapore Committee has recommended moving towards full CAC. Hence, statement 3 is
correct.
o Should India move towards full Capital Account convertibility?
o Positives:
✓ RBI recently allowed Indian companies to raise rupee debt offshore.

3 www.visionias.in ©Vision IAS

FREE BY KING R QUEEN P [ऋषभ राजपूत]


✓ Convertibility would facilitate further liberalisation and increase foreign investment.
✓ Increasing openness to international trade may create opportunities for avoiding capital account
restrictions.
✓ It can lead to the free exchange of currency at lower rates. Also, it promotes unrestricted mobility
of capital – which may impact the economy in times of global recession.
o Negatives:
✓ It could destabilise an economy in case there are massive capital flows in and out of the country;
✓ Currency appreciation/depreciation could affect the trade balance.

Q 8.B
• Disguised Unemployment
o It is a situation in which more people are doing work than actually required. Even if some are
withdrawn, production does not suffer. In other words, it refers to a situation of employment with
surplus manpower in which some workers have zero marginal productivity. Hence option (b) is the
correct answer.
o Overcrowding in agriculture due to the rapid growth of the population and lack of alternative job
opportunities may be cited as the main reasons for disguised unemployment in India.
o Note:
✓ Casual Unemployment: When a person is employed on a day-to-day basis, casual
unemployment may occur due to short-term contracts, shortage of raw materials, fall in demand,
change of ownership, etc.
✓ Chronic Unemployment: If unemployment continues to be a long-term feature of a country, it is
called chronic unemployment. The rapid growth of the population and inadequate level of
economic development on account of the vicious circle of poverty are the main causes of chronic
unemployment.

Q 9.A
• Statement 1 is correct and 2 is not correct: A Participatory Note (PN or P-Note) is a derivative
instrument issued in foreign jurisdictions, by a SEBI registered Foreign Institutional Investor (FII)
or its sub-accounts or one of its associates, against underlying Indian securities. The underlying
Indian security instrument may be equity, debt, derivatives or may even be an index. Further, a basket of
securities from different jurisdictions can also be constructed in which a portion of the underlying
securities is Indian securities or indices.
• Being derivative instruments and freely tradable, PNs can be easily transferred, creating multiple
layers, thereby obfuscating the real beneficial owner. It is in this respect, that concerns have been raised
about the misuse of P-Notes for money laundering and terror financing.
• Derivatives
o A derivative is a financial security with a value that is reliant upon or derived from an underlying
asset or group of assets. Its price is determined by fluctuations in the underlying asset. The most
common underlying assets include stocks, bonds, commodities, currencies, interest rates and market
indexes.

Q 10.B
• Cash Reserve Ratio (CRR) is the average daily balance that a bank is required to maintain with the
Reserve Bank as a percent of its net demand and time liabilities (NDTL) as of the last Friday of the
second preceding fortnight that the Reserve Bank may notify from time to time in the Official Gazette.
• Cash Reserve Ratio (CRR) = Percentage of deposits which a bank must keep as cash reserves with the
bank.
• CRR is one of the monetary policy tools that the RBI uses to control inflation. During high inflation
in the economy, RBI increases the CRR to lower the bank’s loanable funds. Thus, when banks are
required to deposit more cash with the RBI the total loanable funds with the banks will reduce. The
less availability of funds with the banks will lead to an increase in the interest rates charged by the
Banks. Hence, option (b) is the correct answer.
• The rise in interest rates decreases the liquidity in the market which further seeks to reduce the
aggregate demand and thereby inflation in the economy. Hence, options (a) and (c) are not correct.
• A high-interest rate by the banks is likely to attract households to save more money with banks. Thus
an increase in CRR is likely to increase household savings with the banks. Hence, option (d) is not
correct.

4 www.visionias.in ©Vision IAS

FREE BY KING R QUEEN P [ऋषभ राजपूत]


Q 11.C
• The Phillips curve is an economic theory that inflation and unemployment have a stable and inverse
relationship. Higher inflation is associated with lower unemployment and vice versa. Hence option (c) is
the correct answer.
o The concept behind the Phillips curve states the change in unemployment within an economy has a
predictable effect on price inflation. The inverse relationship between unemployment and inflation is
depicted as a downward sloping, concave curve, with inflation on the Y-axis and unemployment on
the X-axis. Increasing inflation decreases unemployment, and vice versa. Alternatively, a focus on
decreasing unemployment also increases inflation, and vice versa.
• Engel's Law is an economic theory put forth in 1857 by Ernst Engel, a German statistician. It states that
the percentage of income allocated for food purchases decreases as a household's income rises, while the
percentage spent on other things (such as education and recreation) increases.
• Kuznets curve graphs the hypothesis that as an economy develops, market forces first increase and then
decrease economic inequality. The hypothesis was first advanced by economist Simon Kuznets in the
1950s and '60s.
• Laffer curve describes higher tax rates would increase revenue, but at some point further increases in tax
rates would cause revenue to fall, for instance by discouraging people from working. The curve became
the basis of supply-side economics.

Q 12.C
• Managed Floating Exchange Rate: Without any formal international agreement, the world has moved
on to what can be best described as a managed floating exchange rate system. India is having this type of
exchange rate system. This system is also known as 'dirty floating'. It is a mixture of a flexible
exchange rate system (the floating part) and a fixed rate system (the managed part).
• In this hybrid exchange rate system, the exchange rate is basically determined in the foreign exchange
market through the operation of market forces. Market forces mean the selling and buying
activities by various individuals and institutions. So far, the managed floating exchange rate system is
similar to the flexible exchange rate system. But during extreme fluctuations, the central bank under a
managed floating exchange rate system (like the RBI) intervenes in the foreign exchange market.
The objective of this intervention is to minimize the fluctuation in the exchange rate of the
rupee. Since, the exchange rate is basically determined by market forces, the upward and downward
movements in the value of the rupee are appreciation and depreciation. Hence statement 1 is correct and
statement 2 is correct.

Q 13.B
• Personal income is part of the National income which is received by households. By subdividing National
income into smaller categories, we can understand it better.
• First, let us note that out of NI, which is earned by the firms and government enterprises, a part of the
profit is not distributed among the factors of production. This is called Undistributed Profits (UP). We
have to deduct UP from NI to arrive at PI, since UP does not accrue to the households. Similarly,
Corporate Tax, which is imposed on the earnings made by the firms, will also have to be deducted from
the NI, since it does not accrue to the households.
• On the other hand, the households do receive interest payments from private firms or the government on
past loans advanced by them. And households may have to pay interest to the firms and the government as
well, in case they had borrowed money from either. So, we have to deduct the net interest paid by the
households to the firms and government. The households receive transfer payments from the government
and firms (pensions, scholarships, prizes, for example) which have to be added to calculate the Personal
Income of the households.
o Thus, Personal Income (PI) ≡ NI – Undistributed profits – Net interest payments made by
households – Corporate tax + Transfer payments to the households from the government and
firms.
• However, even PI is not the income over which the households have a complete say. They have to
pay taxes from PI. If we deduct the Personal Tax Payments (income tax, for example) and Non-tax
Payments (such as fines) from PI, we obtain what is known as Personal Disposable Income.
o Personal Disposable Income (PDI ) ≡ PI – Personal tax payments – Non-tax payments. Hence,
option (b) is the correct answer.
• Personal Disposable Income is part of the aggregate income which belongs to the households. They may
decide to consume a part of it and save the rest.

5 www.visionias.in ©Vision IAS

FREE BY KING R QUEEN P [ऋषभ राजपूत]


Q 14.D
• Deflation is a general decline in prices for goods and services, typically associated with a contraction
in the supply of money and credit in the economy. During deflation, the purchasing power of currency
rises over time. Hence statement 1 is not correct.
o Deflation causes the nominal costs of capital, labor, goods, and services to fall, though their relative
prices may be unchanged.
o By definition, monetary deflation can only be caused by a decrease in the supply of money or
financial instruments redeemable in money. Reduced investment spending by the government or
individuals may also lead to this situation.
o Deflation leads to a problem of increased unemployment due to slack in demand.
• Disinflation is a temporary slowing of the pace of price inflation and is used to describe instances
when the inflation rate has reduced marginally over the short term. Hence statement 2 is not correct.
o Unlike inflation and deflation, which refer to the direction of prices, disinflation refers to the rate of
change in the rate of inflation.
o A healthy amount of disinflation is necessary since it prevents the economy from overheating.
• Deflation is represented as a negative growth rate, such as -1%, while disinflation is shown as a change in
the inflation rate, say, from 3% one year to 2% the next. Disinflation is considered the opposite of
reflation, which occurs when a government stimulates an economy by increasing the money supply.

Q 15.A
• The Gross Domestic Product (GDP) deflator is a measure of general price inflation. It is calculated by
dividing nominal GDP by real GDP and then multiplying by 100. Nominal GDP is the market value of
goods and services produced in an economy, unadjusted for inflation (It is the GDP measured at current
prices). Real GDP is nominal GDP, adjusted for inflation to reflect changes in real output (It is the GDP
measured at constant prices).
o GDP Deflator = (Nominal GDP/Real GDP) x 100.
• There are other measures of inflation too like Consumer Price Index (CPI) and Wholesale Price Index (or
WPI); however, the GDP deflator is a much broader and more comprehensive measure. Since Gross
Domestic Product is an aggregate measure of production, being the sum of all final uses of goods
and services (fewer imports), the GDP deflator reflects the prices of all domestically produced goods
and services in the economy whereas, other measures like CPI and WPI are based on a limited basket of
goods and services, thereby not representing the entire economy (the basket of goods is changed to
accommodate changes in consumption patterns but after a considerable period of time). Another important
distinction is that the basket of WPI (at present) has no representation of the services sector. The GDP
deflator also includes the prices of investment goods, government services, and exports, and excludes the
price of imports. Changes in consumption patterns or the introduction of new goods and services or
structural transformation are automatically reflected in the deflator which is not the case with other
inflation measures. Hence, statement 1 is correct.
• However, WPI and CPI are available on a monthly basis whereas deflator comes with a lag (yearly
or quarterly after quarterly GDP data is released). Hence, the monthly change in inflation cannot
be tracked using a GDP deflator, limiting its usefulness.
• The Ministry of Statistics and Programme Implementation (MOSPI) comes out with a GDP
deflator in National Accounts Statistics as price indices. The base of the GDP deflator is revised
when the base of the GDP series is changed. Hence, statement 2 is not correct.

Q 16.B
• Under the WTO Agreement on Agriculture (AoA), domestic agri-subsidies are classified into three
categories; green, blue and amber. Under WTO principles, "amber box" subsidies create trade distortions
because they encourage excessive production through farm subsidies to fertilizers, seeds, electricity, and
irrigation.
• They are also called as Aggregate Measure of Support. As per the WTO norms, the AMS can be given up
to 10 % of a country’s agricultural GDP (at 1986-88 prices) in the case of developing countries. On the
other hand, the limit is 5% for a developed economy. This limit is called the de minimis level of
support. It is thus minimal amounts of domestic support that are allowed even though they distort
trade — up to 5% of the value of production for developed countries, 10% for developing countries.
• Hence option (b) is the correct answer.

6 www.visionias.in ©Vision IAS

FREE BY KING R QUEEN P [ऋषभ राजपूत]


Q 17.D
• Sources of Data on Unemployment: The data regarding unemployment can come from four potential
sources viz. Household Surveys; Enterprise Surveys; Administrative Data and Data from Government
Schemes.
o Household Surveys: Currently, there are three datasets on household surveys as follows:
✓ Employment-Unemployment Survey of NSSO under MOPSI
✓ Annual Labour Force Survey by MoLE
✓ Population/census data from the Office of the Registrar General & Census Commissioner and
Data from registration with Employment Exchanges. Hence option (d) is the correct answer.
o Annual Labour Force Survey (Labour Bureau)
✓ To obtain more regular data on the labour force, the Labour Bureau under MoLE started
conducting the Annual Labour Force Survey in 2009-10. Four more surveys have been conducted
since then. The most recent one was conducted during 2015-16. The problem with this survey is
that it collects data only for a part of a year and is unable to catch the information covering the
entire year. Further, while the EUS covers the entire population, this survey reports data only for
the population aged 15 and above.
o Population Census
✓ Population Census collects data on main, marginal and non-workers. However, since census data
comes every 10 years, these figures hardly get any attention.

Q 18.B
• One of the core functions of the Central bank is to issue the currency of the country. This currency issued
by the central bank can be held by the public or by commercial banks and is called the ‘high-
powered money’ or ‘reserve money’ or ‘monetary base’ as it acts as a basis for credit creation.
• High high-powered money is also defined as the total liability of the monetary authority of the
country (RBI). It consists of currency (notes and coins in circulation with the public and vault cash of
commercial banks) and deposits held by the Government of India and commercial banks with RBI. If a
member of the public produces a currency note to RBI the latter must pay her value equal to the figure
printed on the note. Similarly, the deposits are also refundable by RBI on demand from deposit-holders.
These items are claims that the general public, government, or banks have on RBI and hence are
considered to be the liability of RBI. Hence, option (b) is the correct answer.
• High-powered money is the base for the expansion of Bank deposits and the creation of a money supply.
The supply of money varies directly with changes in the monetary base and inversely with the currency
and reserve ratios.
• The amount of money that banks generate with deposits by the public is called a Money Multiplier.

Q 19.D
• Recently, The Centre for Social and Economic Progress (CSEP) in its report cited concerns about
poor critical mineral supply chain management in India.
• Critical minerals refer to mineral resources, both primary and processed, which are essential inputs in
the production process of an economy, and whose supplies are likely to be disrupted due to the risks
of non-availability or unaffordable price spikes. They are important to support military, industrial,
or commercial purposes that are essential to the economy, defense, medicine, or infrastructure.
• Minerals such as antimony, cobalt, gallium, graphite, lithium, nickel, niobium, and strontium are
among the 22 assessed to be critical for India. Many of these are required to meet the manufacturing
needs of green technologies, high-tech equipment, aviation, and national defense. Hence, option (d) is
the correct answer.

Q 20.B
• Most-favoured-nation (MFN): Under the WTO agreements, countries cannot normally discriminate
between their trading partners. Grant someone a special favour (such as a lower customs duty rate for one
of their products) and you have to do the same for all other WTO members. A most-favored-nation
(MFN) clause requires a country to provide any concessions, privileges, or immunities granted to one
nation in a trade agreement to all other World Trade Organization member countries. Although its
name implies favoritism toward another nation, it denotes the equal treatment of all countries. Hence
option (b) is the correct answer

7 www.visionias.in ©Vision IAS

FREE BY KING R QUEEN P [ऋषभ राजपूत]


• Equal treatment of imported and locally produced goods after foreign goods enter the domestic market
is a clause under WTO's another principle known as National Treatment: Treating foreigners and
locals equally.

Q 21.D
• Recent context: India started ‘Operation Kaveri’ to evacuate its nationals owing to the Current
Crisis in Sudan. Around 3,000 Indians were stuck in various parts of Sudan, including capital
Khartoum and in distant provinces like Darfur. Hence option (d) is the correct answer.
• Operation Kaveri is a codename for India's evacuation effort to bring back its citizens stranded in Sudan
amid intense fighting between the army and a rival paramilitary force there.
• The operation involves the deployment of the Indian Navy’s INS Sumedha, a stealth offshore patrol
vessel, and two Indian Air Force C-130J special operations aircraft on standby in Jeddah.
• There are about 2,800 Indian nationals in Sudan, and there is also a settled Indian community of about
1,200 in the country.
• What is the current crisis in Sudan.
o The conflict in Sudan has its roots in the overthrowing of long-serving President Omar al-Bashir by
military generals in April 2019, following widespread protests.
o This led to an agreement between the military and protesters, under which a power-sharing body
called the Sovereignty Council was established to lead Sudan to elections at the end of 2023.
o However, the military overthrew the transitional government led by Abdalla Hamdok in October
2021, with Burhan becoming the de-facto leader of the country and Dagalo his second-in-command.
o Soon after the 2021 coup, a power struggle between two military (SAF) and paramilitary (RSF)
generals arose, interrupting a plan to transition to elections.
o A preliminary deal was reached in December 2021 for a political transition, but negotiations hit a
roadblock over the integration of the paramilitary Rapid Support Forces (RSF) with the Sudanese
Armed Forces (SAF), due to disagreements over the timetable and security sector reforms.
o Tensions escalated over the control of resources and RSF integration, leading to clashes.
o There was disagreement over how the 10,000-strong RSF should be integrated into the army, and
which authority should oversee that process.
o Also, Dagalo (RSF general) wanted to delay the integration for 10 years but the army said it would
take place in the next two years.
• The RSF is a group, evolved from Janjaweed militias, which fought in a conflict in the 2000s in the
Darfur region in West Sudan nearing the Border of Chad. Over time, the militia grew and made into the
RSF in 2013, and its forces were used as border guards in particular.
Q 22.C
• Recently, World’s second-deepest blue hole (the deepest is Dragon Hole in the South China Sea)
was discovered in Chetumal Bay, off the coast of the Yucatan Peninsula in Mexico.
• Blue holes are large, undersea vertical caves or sinkholes found in coastal regions. Many contain a high
diversity of plant and marine life, including corals, sea turtles, and sharks.
• Blue Holes are found on coastal karst platforms around the world. It is typically formed out of eroded
limestone, with an open rounded mouth on the sea floor. Hence, statement 1 is correct.
• They are believed to be formed during the latter ice ages around 11,000 years ago because of the
erosion of limestone terrains. Hence, statement 2 is correct.
• The world's deepest known deep hole was discovered in the South China Sea in 2016 and is known as
the Dragon Hole. As per the records, it's more than 980 ft deep.

Q 23.D
• The Reserve Bank of India began operations on April 01, 1935, in accordance with the provisions of the
Reserve Bank of India Act, 1934. The Preamble of the Reserve Bank of India describes the basic
functions of the Reserve Bank as: "to regulate the issue of Bank notes and keeping of reserves with a view
to securing monetary stability in India and generally to operate the currency and credit system of the
country to its advantage; to have a modern monetary policy framework to meet the challenge of an
increasingly complex economy, to maintain price stability while keeping in mind the objective of
growth."
• The main Functions of the RBI are:
o Monetary Authority: The RBI Formulates, implements, and monitors the monetary policy with the
objective of maintaining price stability and financial stability while keeping in mind the objective
of growth.
8 www.visionias.in ©Vision IAS

FREE BY KING R QUEEN P [ऋषभ राजपूत]


o Regulator and supervisor of the financial system: RBI prescribes broad parameters of banking
operations within which the country's banking and financial system functions with the objective of
maintaining public confidence in the system, protecting depositors' interests, and providing cost-
effective banking services to the public.
o Manager of Foreign Exchange: The RBI manages the Foreign Exchange Management Act with the
objective to facilitate external trade and payment and promote orderly development and
maintenance of the foreign exchange market in India.
o The issuer of currency: The RBI Issues, exchanges and destroys currency notes as well as puts into
circulation coins minted by the Government of India with the objective to give the public an
adequate quantity of supplies of currency notes and coins in good quality.
o Developmental role: The RBI Performs a wide range of promotional functions to support national
objectives.
o Regulator and Supervisor of Payment and Settlement Systems: The RBI Introduces and upgrades
safe and efficient modes of payment systems in the country to meet the requirements of the public at
large. Its objective is to maintain public confidence in the payment and settlement system.
o Hence option (d) is the correct answer.

Q 24.B
• The RBI controls the money supply in the economy in various ways. The tools used by the Central bank
to control money supply can be quantitative or qualitative. Quantitative tools control the extent of money
supply by changing the CRR and SLR, or bank rate or open market operations. Hence option 3 is not
correct.
• Qualitative tools include persuasion by the Central bank in order to make commercial banks discourage or
encourage lending which is done through moral suasion, margin requirement, etc. Hence options 1 and 2
are correct.

Q 25.B
• Primary deficit is fiscal deficit of the current year minus interest payments on previous borrowings.
While fiscal deficit represents the government's total borrowing including interest payments, primary
deficit shows the amount of borrowing excluding interest payments. Hence option (b) is the correct
answer.
• It shows the amount of government borrowings specifically to meet the expenses by removing the interest
payments. Therefore, a zero Primary Deficit means the need for borrowing to meet interest payments.
• Fiscal Deficit: The excess of total expenditure over total receipts excluding borrowings is called fiscal
deficit. In other words, the fiscal deficit gives the amount needed by the government to meet its expenses.
Thus a large fiscal deficit means a large amount of borrowings.
• Revenue Deficit: Revenue deficit is the excess of its total revenue expenditure to its total revenue
receipts. Revenue deficit is only related to revenue expenditure and revenue receipts of the government.
• Effective Revenue Deficit: It is the difference between revenue deficit and grants for the creation of
capital assets.

Q 26.C
• Currency Swap Agreement:
o The currency swap agreement between the two countries is entered between the Central Banks of the
two countries. Hence, statement 1 is correct.
o One country exchanges its national currency for that of another or even a third one. Hence, statement
2 is correct.
o Examples: India and Japan signed a currency swap agreement in 2018 worth $ 75 billion. India
can/will get Yen(or dollars) from Japan worth a max of $ 75 billion and Japan will get equivalent
Indian Rupees as per the market exchange rate at the time of transaction.
o In July 2020, India and Sri Lanka signed a currency swap agreement worth $ 400 million in which
India will give dollars to SL and in return India will get a ‘Sri Lankan Rupee’.

Q 27.D
• Air Exercises With Quad Countries:
o Cope India began in 2004 as a fighter training exercise held at Air Station Gwalior, India. The
exercise has evolved to incorporate subject matter expert exchanges, air mobility training, airdrop
training and large-force exercises, in addition to fighter-training exercises.

9 www.visionias.in ©Vision IAS

FREE BY KING R QUEEN P [ऋषभ राजपूत]


o The exercise showcases U.S. and India’s efforts and commitment to a free and open Indo-Pacific
region. Exercise Cope India 23 was held at Air Force Stations Arjan Singh (Panagarh), Kalaikunda
and Agra. Hence pair 1 is correctly matched.
o The inaugural edition of the bilateral air exercise 'Veer Guardian 2023' between the Indian Air
Force (IAF) and Japan Air Self-Defence Force (JASDF) concluded in Japan, on 26 January 2023.
The JASDF participated in the exercise with its F-2 and F-15 aircraft, while the IAF contingent
participated with the Su-30 MKI aircraft. The IAF fighter contingent was complemented by one IL-78
Flight Refueling Aircraft and two C-17 Globemaster strategic airlift transport aircraft.
✓ During the joint training spanning 16 days, the two Air Forces engaged in complex and
comprehensive aerial manoeuvres in multiple simulated operational scenarios. The exercise
involved precise planning and skillful execution by both air forces. IAF and JASDF engaged in
air combat maneuvering, interception, and air defence missions, both in Visual and Beyond
Visual Range settings. Aircrew of the two participating Air Forces also flew in each other's
fighter aircraft to gain a deeper understanding of each other's operating philosophies. Hence pair
2 is correctly matched.
o Exercise Pitch Black 22 was hosted by the Royal Australian Air Force at its Darwin Air Base.
Spanning over three weeks, the exercise saw the participation of 17 Air Forces and over 2500 military
personnel. The IAF contingent included four Su-30 MKI & two C-17 aircraft. The participating forces
took part in various multi-aircraft combat drills by day & night, simulating complex aerial scenarios,
involving large formations.
✓ The exercise provided an opportunity for the air forces to share best practices and experiences.
The event witnessed a collaborative spirit that has led to a better understanding of each other's
capabilities & a bonhomie that will lead to enduring ties between the participating nations. Hence
pair 3 is correctly matched.

Q 28.A
• The Basel Accords are a set of agreements set by the Basel Committee on Bank Supervision
(BCBS), which provides recommendations on banking regulations in regard to capital risk, market
risk, and operational risk. The purpose of the accords is to ensure that financial institutions have enough
capital on account to meet obligations and absorb unexpected losses. The Basel Committee has issued
three sets of regulations which are known as Basel-I, II, and III.
• In Basel-I norms the capital adequacy ratio was agreed upon—a requirement was imposed upon the banks
to maintain a certain amount of free capital to their assets cushion against probable losses in investments
and loans.
• The capital adequacy ratio is the percentage of total capital to the total risk—weighted asset. CAR, a
measure of a bank’s capital, is expressed as a percentage of a bank’s risk-weighted credit exposures:
CAR= Total of Tier 1 & Tier 2 capitals ÷ Risk-Weighted Assets.
• The Reserve Bank of India decided in April 1992 to introduce a risk-asset ratio system for banks
(including foreign banks) in India as a capital adequacy measure in line with the Capital Adequacy Norms
prescribed by Basel Committee. It was aimed at Improving the banking sector’s ability to absorb
shocks arising from financial and economic stress.
• Market risk refers to the risk to a bank resulting from movements in market prices in particular changes in
interest rates, foreign exchange rates, and equity and commodity prices. In simpler terms, it may be
defined as the possibility of loss to a bank caused by changes in the market variables.
• Credit risk is most simply defined as the potential that a bank’s borrower or counterparty may fail to meet
its obligations in accordance with agreed terms.
• Currently, Basel III norms are implemented in India with effect from April 1, 2013.
• Hence option (a) is the correct answer.

Q 29.C
• The Ministry of Jal Shakti has released the report of India’s first water bodies census, a
comprehensive database of ponds, tanks, lakes, and reservoirs in the country. The census was
conducted in 2018-19, and enumerated more than 2.4 million water bodies across all states and
Union Territories.
• The Water Bodies: First Census Report considers “all natural or man-made units bounded on all sides
with some or no masonry work used for storing water for irrigation or other purposes (e.g. industrial,
pisciculture, domestic/ drinking, recreation, religious, groundwater recharge etc.)” as water bodies. The
water bodies “are usually of various types known by different names like tank, reservoirs, ponds etc.

10 www.visionias.in ©Vision IAS

FREE BY KING R QUEEN P [ऋषभ राजपूत]


• As per the report, West Bengal’s South 24 Pargana has been ranked as the district having the highest (3.55
lakh) number of water bodies across the country. The district is followed by Andhra Pradesh’s
Ananthapur (50,537) and West Bengal’s Howrah (37,301).
• So did the census cover all water bodies that fit this definition? No. Seven specific types of water bodies
were excluded from the count. They were: 1) oceans and lagoons; 2) rivers, streams, springs, waterfalls,
canals, etc. which are free-flowing, without any bounded storage of water; 3) swimming pools; 4) covered
water tanks created for a specific purpose by a family or household for their own consumption; 5) a water
tank constructed by a factory owner for consumption of water as raw material or consumable; 6)
temporary water bodies created by digging for mining, brick kilns, and construction activities, which may
get filled during the rainy season; and 7) pucca open water tanks created only for cattle to drink water.
• The census found that 1.6% of enumerated water bodies — 38,496 out of 24,24,540 — had been
encroached upon.Uttar Pradesh accounted for almost 40% (15,301) of water bodies under encroachment,
followed by Tamil Nadu (8,366) and Andhra Pradesh (3,920). No encroachment was reported from West
Bengal, Sikkim, Arunachal Pradesh, and Chandigarh.
• The Jal Shakti Ministry is working on an ambitious plan to deploy a vast network of groundwater sensors
that will continuously relay information on groundwater levels as well as the degree of contamination
down to the taluk level. Currently, such information is only measured a handful of times a year and
communicated via reports of the Central Groundwater Board.
• Under the new initiative, around 16,000-17,000 digital water level recorders will be connected to
piezometers in the wells. Piezometers measure groundwater levels, the recorders will transmit the
information digitally. Hence, statement 1 is correct.
• The CGWB is in charge of the National Aquifer Mapping Program (NAQUIM), that as of March has
mapped the country’s aquifers at a resolution of 1:50000 and – under the second phase of the programme
– expects to improve the resolution by five times in the country. So far, an area of 25.15 lakh square km
has been covered under the NAQUIM studies.
• In the latest Ground Water Resource Assessment-2022, the average stage of groundwater extraction for
the country as a whole works out to be about 60.08%. Anything above 70% is considered “critical”
though there are regions in Punjab, Haryana, Delhi and Rajasthan with groundwater blocks with over
100% extraction.
• Groundwater contamination, the CGWB says, is mostly “geogenic” (natural) and hasn’t
significantly changed over the years. However, nitrate contamination – a result of the use of
nitrogenous fertilisers—has been observed. Sections of nearly 409 districts have been confirmed
with fluoride contamination and parts of 209 districts have noted arsenic contamination. Hence,
statement 2 is correct.

Q 30.D
• Expert group constituted by the Planning Commission and, chaired by Suresh Tendulkar, was
constituted to review methodology for poverty estimation and to address the following shortcomings
of the previous methods:
o Obsolete Consumption Pattern: Consumption patterns were linked to the 1973-74 poverty line
baskets (PLBs) of goods and services, whereas there were significant changes in the consumption
patterns of the poor since that time, which were not reflected in the poverty estimates.
o Inflation Adjustment: There were issues with the adjustment of prices for inflation, both spatially
(across regions) and temporally (across time).
o Health and Education Expenditure: Earlier poverty lines assumed that health and education would
be provided by the state and formulated poverty lines accordingly.
• Recommendations:
o Shift from Calorie Consumption based Poverty Estimation: It based its calculations on the
consumption of the items like cereal, pulses, milk, edible oil, non-vegetarian items, vegetables, fresh
fruits, dry fruits, sugar, salt & spices, other food, intoxicants, fuel, clothing, footwear, education,
medical (non-institutional and institutional), entertainment, personal & toilet goods. Hence option 1
is correct.
o Uniform Poverty line Basket: Unlike Alagh committee (which relied on separate PLB for rural and
urban areas), Tendulkar Committee computed new poverty lines for rural and urban areas of each
state based on the uniform poverty line basket and found that all India poverty line (2004-05) was:
✓ ₹446.68 per capita per month in rural areas
✓ ₹578.80 per capita per month in urban areas. Hence option 2 is correct.

11 www.visionias.in ©Vision IAS

FREE BY KING R QUEEN P [ऋषभ राजपूत]


o Private Expenditure: Incorporation of private expenditure on health and education while estimating
poverty. Hence option 4 is correct.
o Price Adjustment Procedure: The Committee also recommended a new method of updating poverty
lines, adjusting for changes in prices and patterns of consumption (to correct spatial and temporal
issues with price adjustment), using the consumption basket of people close to the poverty line. Hence
option 3 is correct.
o Mixed Reference Period: The Committee recommended using Mixed Reference Period based
estimates, as opposed to Uniform Reference Period based estimates that were used in earlier methods
for estimating poverty.
• Tendulkar committee computed poverty lines for 2004-05 at a level that was equivalent, in Purchasing
Power Parity (PPP) terms to Rs 33 per day.
o Purchasing Power Parity: The PPP model refers to a method used to work out the money that would
be needed to purchase the same goods and services in two countries.

Q 31.D
• Money supply, like money demand, is a stock variable. The total stock of money in circulation among the
public at a particular point of time is called the money supply.
• RBI publishes figures for four alternative measures of money supply, viz. M1, M2, M3 and M4.
They are defined as followsM1 = CU + DDM2 = M1 + Savings deposits with Post Office savings banks
M3 = M1 + Net time deposits of commercial banksM4 = M3 + Total deposits with Post Office savings
organizations (excluding National Savings Certificates) where CU is currency (notes plus coins) held by
the public and DD is net demand deposits held by commercial banks.
• The word ‘net’ implies that only deposits of the public held by the banks are to be included in the money
supply. The interbank deposits, which a commercial bank holds in other commercial banks, are not to be
regarded as part of the money supply. M1 and M2 are known as narrow money. M3 and M4 are known as
broad money.
• These measures are in decreasing order of liquidity. M1 is the most liquid and easiest for transactions
whereas M4 is the least liquid of all. M3 is the most commonly used measure of money supply. It is also
known as aggregate monetary resources. Hence both statements 1 and 2 are not correct.

Q 32.C
• GDP is the market value of all final goods and services produced within a domestic territory of a country
measured in a year. There are three ways to calculate the national income; namely product
method, expenditure method, and income method.
o The Product or Value-Added Method: In the product method, the aggregate annual value of goods
and services produced (if a year is the unit of time) is calculated. If we sum the gross value added of
all the firms of the economy in a year, we get a measure of the value of the aggregate amount of
goods and services produced by the economy in a year (just as we had done in the wheat-bread
example). Such an estimate is called Gross Domestic Product (GDP).
o Expenditure Method: An alternative way to calculate the GDP is by looking at the demand side of
the products. This method is referred to as the expenditure method.
o The expenditure approach is the most commonly used GDP formula, which is based on the money
spent by various groups that participate in the economy. GDP = C + G + I + NX;
✓ C = consumption or all private consumer spending within a country’s economy, including,
durable goods (items with a lifespan greater than three years), nondurable goods (food &
clothing), and services.
✓ G = total government expenditures, including salaries of government employees, road
construction/repair, public schools, and military expenditures.
✓ I = sum of a country’s investments spent on capital equipment, inventories, and housing.
✓ NX = net exports or a country’s total exports less total imports.
o Income Method: As we mentioned in the beginning, the sum of final expenditures in the economy
must be equal to the incomes received by all the factors of production taken together (final
expenditure is the spending on final goods, it does not include spending on intermediate goods). Total
National Income – the sum of all wages, rent, interest, and profits.
• Balance of Payment: In international economics, the balance of payments of a country is the difference
between all money flowing into the country in a particular period of time and the outflow of money to the
rest of the world. Balance of Payment is primarily an indicator of an economy's link with the outside
world and is not used as a method to calculate the national income. Hence option (c) is the correct
answer.
12 www.visionias.in ©Vision IAS

FREE BY KING R QUEEN P [ऋषभ राजपूत]


Q 33.D
• A significant part of the current output of capital goods goes into maintaining or replacing part of the
existing stock of capital goods. This is because the already existing capital stock suffers wear and tear and
needs maintenance and replacement. A part of the capital goods produced this year goes for the
replacement of existing capital goods and is not an addition to the stock of capital goods already existing
and its value needs to be subtracted from gross investment for arriving at the measure for net investment.
This deletion, which is made from the value of a gross investment in order to accommodate the regular
wear and tear of capital, is called depreciation. Hence, statement 1 is correct.
• So new addition to the capital stock in an economy is measured by net investment or new capital
formation, which is expressed as:
o Net Investment = Gross investment – Depreciation
• Depreciation is an accounting concept. No real expenditure may have actually been incurred each
year yet depreciation is annually accounted for. In an economy with thousands of enterprises with
widely varying periods of life of their equipment, in any particular year, some enterprises are
actually making the bulk replacement spending. Thus, we can realistically assume that there will be a
steady flow of actual replacement spending which will more or less match the amount of annual
depreciation being accounted for in that economy. Hence, statement 2 is correct.
• Depreciation does not take into account unexpected or sudden destruction or disuse of capital as
can happen with accidents, natural calamities, or other such extraneous circumstances. Hence,
statement 3 is not correct.

Q 34.C
• GVA is the value of total output produced in the economy less the value of intermediate consumption (the
output which is used in the production of output further, and not used in final consumption). It is
important to understand the concepts such as basic prices, factor cost, and market price to understand
GVA.
• The distinction between factor cost, basic prices, and market prices are based on the distinction between
net production taxes (production taxes fewer production subsidies) and net product taxes (product taxes
less product subsidies).
• Production taxes and subsidies are paid or received in relation to production and are independent of the
volume of production such as land revenues, stamp and registration fees. Product taxes and subsidies, on
the other hand, are paid or received per unit or product, e.g., excise tax, service tax, export, and import
duties, etc. Hence, statements 1 and 2 are correct.
• Factor cost includes only the payment to factors of production, it does not include any tax. In order to
arrive at the market prices, we have to add to the factor cost the total indirect taxes less total subsidies.
The basic prices lie in between: they include production taxes (fewer production subsidies) but not
product taxes (fewer product subsidies). Therefore in order to arrive at market prices we have to add
product taxes (less product subsidies) to the basic prices.

Q 35.C
• Marginal Standing Facility (MSF) is a new scheme announced by the Reserve Bank of India (RBI) in
its Monetary Policy (2011-12) and refers to the penal rate at which banks can borrow money from the
central bank over and above what is available to them through the LAF window. Hence statement 3 is
not correct.
• MSF, being a penal rate, is always fixed above the repo rate. The MSF would be the last resort for
banks once they exhaust all borrowing options including the liquidity adjustment facility by pledging
government securities, where the rates are lower in comparison with the MSF. Hence statement 2 is
correct.
• The MSF would be a penal rate for banks and the banks can borrow funds by pledging government
securities within the limits of the statutory liquidity ratio. The scheme has been introduced by RBI with
the main aim of reducing volatility in the overnight lending rates in the inter-bank market and enabling
smooth monetary transmission in the financial system. Hence statement 1 is correct.

Q 36.A
• Financial markets comprise both capital and money markets. Capital markets refer to markets that
trade financial instruments with maturities longer than one year. Money markets trade debt securities
or instruments of maturities of a year or less.

13 www.visionias.in ©Vision IAS

FREE BY KING R QUEEN P [ऋषभ राजपूत]


• In the simplest terms, capital markets can be defined as a marketplace where buyers and sellers can
engage in the trade of long-term financial securities. Long-term here means for a period greater than one
year.
• Capital Markets vis-a-vis Commercial Banks
o Intermediation between lenders (or savers) and borrowers (or users of funds) is a fundamental
function of the financial system in an economy and is performed primarily by commercial banks and
primary capital markets.
o The key distinction is that capital markets provide direct funding from saver to user via the issuance
of securities, while bank intermediation involves indirect funding with banks as the go-between
connecting the saver and user. Hence, statement 1 is correct.
• The following characteristics are typical of Capital Markets:
o The capital market is the market for securities, where companies and governments can raise long-term
funds.
o The market in which corporate equity and longer-term debt securities maturing in more than one year
are issued and traded.
o The capital market is the market for long-term debt equity shares. In this market, capital funds
comprising both equity and debt are issued and traded. Hence, statement 2 is not correct.
o The market in which long-term securities such as stocks and bonds are bought and sold.
o The capital market comprises financial securities, government securities, and semi-government
securities. The capital market concerns two broad types of securities traded - debts and equity. Buying
stock allows investors to gain an equity interest in the company and become an owner of the
company.

Q 37.D
• The SDR basket of currencies includes the US dollar, Euro, Japanese yen, pound sterling and the
Chinese renminbi (included in 2016).
• The SDR currency value is calculated daily (except on IMF holidays or whenever the IMF is closed for
business) and the valuation basket is reviewed and adjusted every five years.
• Quota (the amount contributed to the IMF) of a country is denominated in SDRs. Members’ voting power
is related directly to their quotas.
• IMF makes the general SDR allocation to its members in proportion to their existing quotas in the IMF.
• Hence option (d) is the correct answer.

Q 38.D
• Statutory Liquidity Ratio (SLR) refers to the minimum percentage of deposits that commercial
banks are mandated to maintain as gold assets, cash, or government-approved securities, in their
own vaults. These deposits have to be maintained by the banks themselves and not with the Reserve Bank
of India. By definition, the SLR is the ratio of a bank’s liquid assets to their Net Demand and Time
Liabilities (NDTL). The SLR is an essential instrument in the RBI’s monetary policy that helps regulate
the cash flow in the economy and ensures the bank’s stability.
• As per the Reserve Bank of India SLR norms are applicable to all Scheduled Commercial Banks
(SCBs) (including Regional Rural Banks), Small Finance Banks (SFBs), Payments Banks, Local
Area Banks (LABs), Primary (Urban) Co-operative Banks (UCBs), State Co-operative Banks
(StCBs) and District Central Co-operative Banks (DCCBs). Hence, option (d) is the correct answer.
• SLR is used by the RBI to control credit flow in the banks. In a way, SLR also makes commercial banks
invest in government securities. Making banks invest a portion of their deposits in government securities
also ensures the solvency of such banks.
• SLR is decided by the RBI from time to time. The maximum limit of SLR is 40% and the minimum limit
is Zero. If the bank fails to control the required level of the statutory liquidity ratio, then it becomes
responsible to pay a penalty to the Reserve Bank of India (RBI).

Q 39.A
• The Union Cabinet recently approved a project to build an advanced gravitational-wave detector in
Maharashtra at an estimated cost of Rs 2,600 crore. The facility’s construction is expected to be
completed by 2030. By building it, “Indian S&T is expected to leap-frog in a number of cutting-edge
frontiers of great national relevance, in particular quantum-sensing and metrology. It will come up in the
Hingoli district, where land has been acquired to the tune of 174 acres.

14 www.visionias.in ©Vision IAS

FREE BY KING R QUEEN P [ऋषभ राजपूत]


• LIGO stands for "Laser Interferometer Gravitational-wave Observatory". It is the world's largest
gravitational wave observatory and a marvel of precision engineering. Comprising two enormous laser
interferometers located 3000 kilometers apart, LIGO exploits the physical properties of light and of space
itself to detect and understand the origins of gravitational waves (GW). Currently, there are three
operational gravitational wave observatories around the world - two in the United States (Hanford and
Livingston), one in Italy (Virgo), and one in Japan (Kagra). For accurate detection, four comparable
detectors need to be operated simultaneously across the globe.
• The LIGO detectors consist of two 4-km-long vacuum chambers, arranged at right angles to each
other, with mirrors at the end. The experiment works by releasing light rays simultaneously in both
chambers.
• Normally, the light should return at the same time in both chambers. However, if a gravitational wave
passes through, one chamber gets elongated while the other gets squished, resulting in a phase difference
in the returning light rays. Detecting this phase difference confirms the presence of a gravitational wave.
• Three things fundamentally distinguish LIGO from a stereotypical astronomical observatory:
o LIGO is blind to the light from the Universe. Unlike optical or radio telescopes, LIGO does not see
electromagnetic radiation (e.g., visible light, radio waves, microwaves). But it doesn't have to because
gravitational waves are not part of the electromagnetic spectrum.
o It doesn't need to focus on starlight or point at a particular part of the sky.
o It is difficult for a single detector to make a discovery on its own.
✓ The Large Hadron Collider (LHC) is three things. First, it is large – so large that it’s the world’s
largest science experiment. Second, it’s a collider. It accelerates two beams of particles in
opposite directions and smashes them head-on. Third, these particles are hadrons. The LHC, built
by the European Organisation for Nuclear Research (CERN), is on the energy frontier of physics
research, conducting experiments with highly energized particles.
✓ Laser Interferometer Space Antenna(LISA) is a space-based gravitational wave observatory
building on the success of LISA Pathfinder and LIGO. Led by ESA, the LISA mission is a
collaboration of ESA, NASA, and an international consortium of scientists. Gravitational waves
were first theorized by Albert Einstein. They are created during events such as supermassive
black hole mergers, or collisions between two black holes that are billion times bigger than our
Sun. These collisions are so powerful that they create distortions in spacetime, known as
gravitational waves. Studying gravitational waves gives enormous potential for discovering the
parts of the universe that are invisible by other means, such as black holes, the Big Bang, and
other, as yet unknown, objects. Gravitational waves were first directly detected by the Laser
Interferometer Gravitational-Wave Observatory (LIGO) in 2015.
▪ Three types of detectors have been designed to look for gravitational radiation, which is
very weak. The changes of curvature of space-time would correspond to dilation in one
direction and a contraction at right angles to that direction. One scheme, first tried out about
1960, employed a massive cylinder that might be set in mechanical oscillation by a
gravitational signal. The authors of this apparatus argued that signals had been detected, but
their claim was not substantiated.
▪ In a second scheme, an optical interferometer is set up with freely suspended reflectors at the
ends of long paths that are at right angles to each other. Shifts of interference fringes
corresponding to an increase in the length of one arm and a decrease in the other would
indicate the passage of gravitational waves. One such interferometer is the Laser
Interferometer Gravitational-Wave Observatory (LIGO), which consists of two
interferometers with arm lengths of 4 km (2 miles), one in Hanford, Washington, and
the other in Livingston, Louisiana.
▪ A third scheme, the Evolved Laser Interferometer Space Antenna (eLISA), is planned that
uses three separate, but not independent, interferometers installed in three spacecraft located
at the corners of a triangle with sides of some 5 million km (3 million miles). A mission to
test the technology for eLISA, LISA Pathfinder, was launched in 2015.
▪ LISA Pathfinder, formerly Small Missions for Advanced Research in Technology-2, was an
ESA spacecraft that was launched on 3 December 2015 onboard Vega flight VV06. The
mission tested technologies needed for the Laser Interferometer Space Antenna, an ESA
gravitational wave observatory planned to be launched in 2037. Hence, option (a) is the
correct answer.

15 www.visionias.in ©Vision IAS

FREE BY KING R QUEEN P [ऋषभ राजपूत]


Q 40.D
• Non-Tax Revenue: Non-Tax Revenue is the recurring income that is earned from sources other than
taxes by the government. They are the revenue receipts that are not generated by taxing the public. Some
of the major sources of non-tax revenue are mentioned below:
o Interests that the government receives through the loans provided by it to the state governments, UTs,
private enterprises, and the general public are an important source of non-tax revenue.
o Power Supply Fees: This includes fees received by the central power authority of any nation. In the
case of India, this includes fees received by the Central Electricity Authority.
o Fees: These are the charges that cover the cost of recurring services that are provided and imposed by
the government. It is a compulsory contribution like a tax.
o License Fee: It is a form of tax charged by the government and its allied entities for conducting an
activity that can be anything such as opening a restaurant or operating a heavy vehicle.
o Fines and Penalties: Fines are mostly used in the context of criminal law wherein a court of law will
punish a person convicted of a crime by imposing a fine. Penalty, meanwhile, is used in both civil as
well as criminal law. It includes both monetary and physical forms of punishment.
o Escheats: Escheats is the transfer of estate assets or property to the government if an individual
passes away without leaving a legally biding bill or legal heirs
o Several grants are received by the government from various International Organisations and foreign
governments. Such grants are not a fixed source of revenue and are generally received during a
national crisis such as war, flood, etc.
o Forfeitures: Forfeiture is the loss of any property without compensation as a result of defaulting on
the obligations of a contract or a penalty for illegal conduct. Under the terms of a contract, forfeiture
refers to the requirement by the defaulting party to give up ownership of an asset or cash flows from
an asset, as compensation for the resulting losses to the other party.
o Interests: It comprises of interests of loans and insurance given to the government for non-plan
schemes and planned schemes and also interest on loans that have been advanced to Public Sector
Enterprises or other statutory bodies.
o Fees for Communication Services: This mainly includes the license fees from telecom operators on
account of spectrum usage charges that licensed Telecom Service Providers pay to the government
ministry that handles telecommunications. Hence option (d) is the correct answer.

Q 41.C
• Usually, there are two main sources of the government’s income – revenue receipts and capital receipts.
Revenue receipts comprise both tax and non-tax revenues while capital receipts consist of capital receipts
and non-debt capital receipts.
• Non-debt creating capital receipts refer to those receipts of the government which lead to a decrease in
assets, and not an increase in liabilities. Non-debt capital receipts, also known as NDCR, account for just
3% of the central government’s total receipts.
• Broadly, there are two kinds of non debt capital receipts:
o Recoveries of loans and advances and
o Miscellaneous capital receipts
• Recoveries of loans and advances: This kind of non debt capital receipts includes:
o Recovery of loans and advances from state governments and union territories with legislature
o Recovery of loans given to foreign governments
o Recovery of loans and advances from PSUs and other autonomous bodies
• Miscellaneous Capital Receipt: This includes proceeds from disinvestment in public sector undertakings.
The government further classifies disinvestment proceeds into:
o Disinvestment receipts
o Strategic disinvestment
o Listing of PSUs in stock markets and
o Issue of bonus shares
• Hence option (c) is the correct answer.

Q 42.C
• Types of Economies- Closed/Inward and Open/Outward:
o Closed Economy: In a closed economy, no external trade takes place. This means that there are
no imports or exports. It indicates a self-sufficient, self-reliant economy primarily growing via
its domestic sectors. Another term for a closed economy is autarky.

16 www.visionias.in ©Vision IAS

FREE BY KING R QUEEN P [ऋषभ राजपूत]


o It is said that India had a near-closed economy focusing more on self-development after the 1950s,
running up to the 1980s, and finally opening its economy after the economic crisis of 1991.
o Open Economy: An open economy is one that trades with other nations in goods and services and also
in financial assets.
• Hence, option (c) is the correct answer.

Q 43.B
• Recently, The demographic data from the United Nations Population Fund’s (UNFPA) “State of
World Population Report, 2023” titled ‘8 Billion Lives, Infinite Possibilities: The Case for Rights
and Choices’ was released.
• The UNFPA’s ‘The State of World Population Report, 2023’, said India’s population is projected to
be 1,428.6 million while that of China’s 1,425.7 million, a difference of 2.9 million by the mid of the
year 2023. Hence, statement 1 is not correct.
• The UNFPA report said 25% of India’s population is in the age group of 0-14, 18% in 10-19, 26% in
10-24, 68% in 15-64, and 7% above 65. Hence, statement 2 is correct.
• China is doing better than India in the context of life expectancy, which in the case of women is 82
and 76 that of men. The figures for India are 74 and 71, according to the report. Hence, statement 3 is
not correct.

Q 44.A
• Recently, European Union Parliament approved Crypto Licensing, Funds Transfer Rules.
• Markets in Crypto Assets (MiCA) forms part of the European Union’s (EU) broader digital financial
package, which includes the Digital Operational Resilience Act and Pilot Regime on Distributed Ledger
Technology. It aims to embrace innovation and new financial technologies while ensuring consumer
protection and financial stability across the EU. Hence, statement 1 is correct.
• The MiCA legislation will apply to crypto assets including traditional cryptocurrencies like Bitcoin,
Ethereum, and newer ones like stablecoins.
• MiCA will not regulate central bank digital currencies issued by the European Central Bank,
nonfungible tokens (NFTs) and digital assets issued by national central banks of EU member countries.
Hence, statement 2 is not correct.

Q 45.B
• Commercial banks are the other type of institutions that are a part of the money-creating system of the
economy. Scheduled banks are banks that are listed in the 2nd schedule of the Reserve Bank of India Act,
1934. The Balance sheet is a record of the assets and liabilities of any firm/bank.
• Assets are things a firm owns or what a firm can claim from others. In the case of a bank, apart from
buildings, furniture, etc., its assets are loans given to the public. Hence option 1 is correct.
• Another asset that a bank has is reserves (Cash Reserve Ratio). Reserves are deposits that commercial
banks keep with the Central Bank, Reserve Bank of India (RBI) and its cash. These reserves are kept
partly as cash and partly in the form of financial instruments (bonds and treasury bills) issued by the
RBI. Hence option 4 is correct.
• Besides reserves with RBI, other classes of assets held by commercial banks are investments in
Government Securities. Government securities are securities of both the central and state government
including treasury bills, treasury deposit certificates, and postal obligations such as national plan
certificates, and national savings certificates. Hence option 3 is correct.
• Liabilities for any firm are its debts or what it owes to others. For a bank, the main liability is the
deposits that people keep with it. Liabilities for a bank = Total Deposits of the public held by
banks. Hence option 2 is not correct.
• Hence option (b) is the correct answer.

Q 46.A
• Recently, Mircha’ rice of Bihar’s West Champaran has been awarded the GI tag. The size and shape of
the grain appear like that of black pepper, hence it is known as Mircha or Marcha Rice.
• This rice is famous for its aroma, palatability and its aromatic chura (rice flakes) making
qualities. The cooked rice is fluffy, non-sticky, sweet and easily digestible with a popcorn-like aroma.
• Gobindobhog rice and Tulaipanji rice belong to West Bengal, whereas, Joha rice belongs to Assam.
• Hence, option (a) is the correct answer.

17 www.visionias.in ©Vision IAS

FREE BY KING R QUEEN P [ऋषभ राजपूत]


Q 47.A
• While calculating GDP, if prices change, then there may be difficulties in comparing GDPs. If we
measure the GDP of a country in two consecutive years and see that the figure for the GDP of the latter
year is twice that of the previous year, we may conclude that the volume of production of the country has
doubled. But it is possible that only prices of all goods and services have doubled between the two years
whereas the production has remained constant.
• Therefore, in order to compare the GDP figures (and other macroeconomic variables) of different
countries or to compare the GDP figures of the same country at different points of time, we cannot rely on
GDPs evaluated at current market prices. For comparison, we take the help of real GDP. Real GDP is
calculated in a way such that the goods and services are evaluated at some constant set of prices (or
constant prices). Since these prices remain fixed, if the Real GDP changes we can be sure that it is the
volume of production that is undergoing changes. An increase in real GDP implies an increase in the
production of goods and services. Hence, statement 1 is correct.
• Nominal GDP, on the other hand, is simply the value of GDP at the current prevailing prices.
Hence, it may happen that the numbers of nominal GDP have increased due to price rise and not
necessarily due to production increase. Actually, production may have remained constant or
decreased in an economy but due to price rise, the nominal GDP figures may have improved.
Hence, statement 2 is not correct.

Q 48.B
• If the GDP of the country is rising, the welfare may not rise as a consequence. This is because the rise in
GDP may be concentrated in the hands of very few individuals or firms. For the rest, the income may in
fact have fallen. In such a case the welfare of the entire country cannot be said to have increased. the GDP
is not taking into account such negative externalities. Therefore, if we take GDP as a measure of the
welfare of the economy we shall be overestimating the actual welfare. This was an example of a negative
externality. There can be cases of positive externalities as well. In such cases, GDP will underestimate the
actual welfare of the economy. Hence statement 1 is not correct.
• Because of development, there will be pollution of rivers and air. This may cause harm to the people who
use the water of the river. Hence their well-being will fall. Pollution may also kill fish or other organisms
of the river on which fish survive. As a result, the fishermen of the river may be losing their livelihood.
Such harmful effects that the project is inflicting on others, for which it will not bear any cost, are called
externalities. In this case, the GDP is not taking into account such negative externalities. Hence statement
2 is correct.
• Many activities in an economy are not evaluated in monetary terms. For example, the domestic services
women perform at home are not paid for. The exchanges which take place in the informal sector without
the help of money are called barter exchanges. In barter exchanges, goods (or services) are directly
exchanged against each other. But since money is not being used here, these exchanges are not registered
as part of economic activity. Hence statement 3 is correct.

Q 49.C
• A variable is a measurable quantity that varies (changes). There are two types of variables i.e. stocks and
flows. The basis of distinction is measurability at a point of time or period of time.
• A flow is a quantity that is measured with reference to a period of time. Thus, flows are defined with
reference to a specific period (length of time), i.e. weeks, months, or years. For example, National
income is a flow, as it describes and measures the flow of goods and services which become available
to a country during a year.
• Other examples of flow variables are expenditure, savings, depreciation, exports, imports, change in
inventories (not mere inventories), change in money supply, output, rent, profit, etc. because the
magnitude (size) of all these are measured over a period of time. A stock is a quantity that is
measurable at a particular point in time. For example, capital is a stock variable as capital goods
continue to serve us through different cycles of production. The buildings or machines in a factory are
there irrespective of the specific time period. There can be an addition to or deduction from, these if a new
machine is added or a machine falls in disuse and is not replaced. A flow shows change during a period of
time whereas a stock indicates the quantity of a variable at a point in time. Thus, wealth is a stock since
it can be measured at a point in time, but income is a flow because it can be measured over a period of
time. Other examples of stock are foreign debts, loans, inventories (not change in inventories), population,
etc. Hence option (c) is the correct answer.

18 www.visionias.in ©Vision IAS

FREE BY KING R QUEEN P [ऋषभ राजपूत]


Q 50.D
• Types of Exchange Rates
o Nominal Exchange Rate is the relative price of the currencies of two countries. For example, if
the exchange rate between the U.S. dollar and the Indian Rupee is Rs. 60 per dollar, then you can
exchange one dollar for 60 Rupees in world markets for foreign currency. When people refer to “the
exchange rate’’ between two countries, they usually mean the nominal exchange rate.
✓ Nominal exchange rates are established on currency financial markets called "forex markets",
which are similar to stock exchange markets.
o Real Exchange Rate is the relative price of the goods of two countries. That is, the real exchange rate
tells us the rate at which we can trade the goods of one country for the goods of another. The real
exchange rate is sometimes called the terms of trade.
o Thus, the Nominal Effective Exchange Rate (NEER) is the weighted average value of the
nominal exchange rate of the rupee against the currencies of major trading partners of India.
Hence, statement 2 is not correct.
o On the other hand, the Real Effective Exchange Rate (REER) is the weighted average of the Real
Exchange Rates of the Rupee against the currencies of major trading partners of India. Hence,
statement 1 is not correct. The weights are determined by the importance that a home country places
on all other currencies traded within the pool, as measured by the balance of trade.
o Unlike NER and RER, NEER and REER are not determined for each foreign currency
separately.
o There is a concept of Effective Exchange Rate which describes the relative strength of a currency
relative to a basket of other currencies.

Q 51.B
• The Prime Minister released the figures of the 5th cycle of India’s Tiger Census revealing that tiger
numbers have once again increased in the country and now stand at 3,167 in the wild as of 2022. The
2018 Tiger Census, released in July 2019, established the presence of 2,967 tigers in India. The animal
population in the country has increased by 200 or 6.7 percent in the past four years. While the tiger
numbers in the country stood at 1,411 in 2006, it increased to 1,706 in 2010 and 2,226 in the 2014 cycle
of evaluations.
• PM released the Tiger Census while inaugurating the International Big Cat Alliance in Karnataka’s
Mysuru, the first of its kind in the country, organised to mark 50 years of Project Tiger. The three-
day conference will focus on the protection and conservation of seven major big cats of the world –
tigers, lions, leopards, snow leopards, pumas, jaguars and cheetahs.
• Being the 50th year of Project Tiger, it is notable that governments, since 1973, have consistently devoted
attention to ensuring that tigers — generally vulnerable to environmental degradation and extinct in
several countries — continue to populate India’s forests. However, this does not mean that tiger numbers
are ordained to grow in perpetuity. The ‘Status of Tiger’ report warns that all of India’s five main tiger
zones, while largely stable, face challenges of deforestation and loss of tiger habitat.
• The only landscape in India where the tiger population has gone down is the Western Ghats, where
declaring of an ecologically sensitive zone has been hanging since 2010, according to the latest tiger
estimation report. The estimation was done in five landscapes. Population increase is substantial in
Shivalik & Gangetic flood plain which is followed by Central India, North Eastern Hills and
Brahmaputra flood plains and Sundarbans while Western Ghats population showed decline with
major populations being stable. Hence, statement 1 is not correct.
• The overlap between “wildlife and humans” because of developmental activities has led to a fall in the
tiger population in the areas under the World Heritage Western Ghats landscape, which is the “most
biodiverse” in the country, according to the report.
• The Western Ghats tiger landscape is 1,600 km long and covers an area of about 1,40,000 square km. It
spans six states --- Karnataka, Maharashtra, Goa, Kerala, Tamil Nadu and Telangana -- and contains 12
Tiger Reserves, 20 National Parks, and 68 Wildlife Sanctuaries and is home to several endemic species
such as lion-tailed macaque, the Malabar giant squirrel, and the Nilgiri tahr.
• From nine tiger reserves in 1973 to 53 today, the increase in numbers has not translated to all of these
reserves becoming suitable habitats for tigers. Serious conservation efforts are needed to help, for
instance, tiger population recovery in Jharkhand, Odisha, Chhattisgarh, Telangana, and Andhra Pradesh.
Experts have said India’s reserves, in their present state, ought to be able to sustain populations of up to
4,000, and with expanded efforts at improving fledgling reserves, these numbers can increase.

19 www.visionias.in ©Vision IAS

FREE BY KING R QUEEN P [ऋषभ राजपूत]


• The National Tiger Conservation Authority, which conducts the estimation, has not released state
or tiger reserve-wise tiger estimation but has provided some state-specific insight in the overall
report. Hence, statement 2 is correct.

Q 52.C
• The Wholesale Price Index (WPI) reflects changes in the average prices of goods at the wholesale level —
that is, commodities sold in bulk and traded between businesses or entities rather than goods bought by
consumers.
o WPI captures the price changes at the point of bulk transactions and may include some taxes levied
and distribution costs up to the stage of wholesale transactions. Hence statement 3 is not correct.
• There are certain limitations in using WPI as a measure for inflation, as WPI does not consider the price
of services, and it does not reflect the consumer price situation in the country. The wholesale market is
only for goods, you cannot buy services on a wholesale basis. So WPI does not include services.
Hence statement 4 is correct.
• WPI is released by the Economic Advisor in the Ministry of Commerce and Industry. The purpose of WPI
is to inspect movement in prices of goods that reflect supply and demand in industry, construction and
manufacturing. Hence statement 1 is correct.
• The index basket of WPI categorises commodities under three groups — primary articles, fuel and power
& manufactured products. The biggest basket is Manufactured Goods. It spans across a variety of
manufactured products such as Textiles, Apparels, Paper, Chemicals, Plastic, Cement, Metals, and
more. Hence statement 2 is not correct.
• In April 2017, the government revised the base year for WPI from 2004-05 to 2011-12. WPI is
extensively used for short-term policy intervention as it is the only index that is available on a weekly
basis.

Q 53.A
• International Monetary Fund (IMF) Finances: The money for lending comes from the member
countries primarily through their payment of quotas. Each member country of the IMF is assigned a quota,
based broadly on its relative position in the world economy.
o Quotas are denominated in Special Drawing Rights (SDRs), the IMF’s unit of account.
o The current quota formula is a weighted average of GDP (50%), Openness (30%), Economic
Variability (15%) and International Reserves (5%). Hence, option (a) is the correct answer.
o The capital subscriptions or quota is now made up of 25 p.c. of its quota in SDRs or widely accepted
currencies (such as the US dollar, euro, the yen or the pound sterling) instead of gold and 75 p.c. in
the country’s own currency.
Q 54.A
• Frictional Unemployment:
o Frictional Unemployment, also called Search Unemployment, refers to the time lag between jobs
when an individual is searching for a new job or is switching between jobs. Hence option (a) is the
correct answer.
o In other words, an employee requires time for searching for a new job or shifting from the existing
one to a new job, this inevitable time delay causes frictional unemployment.
o It is often considered voluntary unemployment because it is not caused due to the shortage of jobs, but
in fact, the workers themselves quit their jobs in search of better opportunities.
Q 55.A
• The World Health Organization (WHO) has launched the 'Preparedness and Resilience for
Emerging Threats' (PRET) to better prepare for future outbreaks of a similar scale and devastation as
the COVID-19 pandemic.
• It aims to provide “guidance on integrated planning for responding to any respiratory pathogen such as
influenza or coronaviruses”.
• The initiative was announced at the Global Meeting for Future Respiratory Pathogen Pandemics
held on 24-26 April 2023 in Geneva, Switzerland.
• Hence, option (a) is the correct answer.

Q 56.D
• Jagdish Chandra Bose:
• Recent context: In April month, a group of researchers from Tel Aviv University in Israel reported that
they had been able to pick up distress noises made by plants. The researchers said these plants had been
20 www.visionias.in ©Vision IAS

FREE BY KING R QUEEN P [ऋषभ राजपूत]


making very distinct, high-pitched sounds in the ultrasonic range when faced with some kind of stress,
like when they were in need of water. This was the first time that plants had been caught making any kind
of noise.
o Jagadish Chandra Bose (1858 – 1937) was an Indian physicist and plant physiologist.
o He earned a B.Sc. from University College London, which was connected with the University of
London in 1883, and a BA (Natural Sciences Tripos) from the University of Cambridge in 1884.
o Contributions
✓ In 1917, he established Bose Institute – Asia’s first modern research center devoted to
interdisciplinary studies.
✓ He discovered wireless communication and was named Father of Radio Science by the Institute
of Electrical and Electronics Engineering. Hence, statement 1 is correct.
✓ He invented the crescograph, a device for measuring the growth of plants. He for the first time
demonstrated that plants have feelings. Hence, statement 2 is correct.
✓ He was the first to demonstrate radio communication with millimeter wavelengths, which fall
in the 30GHz to 300GHz spectrum. Hence, statement 3 is correct.
✓ Bose is considered the father of Bengali science fiction. A crater on the moon has been named in
his honor.
✓ In 1896, he published Niruddesher Kahini, the first work of science fiction in the Bengali
language
✓ Bose was the first Asian to be awarded a US patent. In 1904, he was awarded a patent for his
invention of a detector for electrical disturbances.
✓ Books: Response in the Living and Non-Living, The Nervous: Mechanism of Plants, etc.

Q 57.D
• Ease of doing business is an index published by the World Bank. It is an aggregate figure that includes
different parameters which define the ease of doing business in a country.
• World Economic Outlook - A Survey by the IMF staff usually published twice a year. It presents IMF
staff economists' analyses of global economic developments during the near and medium term. It gives an
overview as well as a more detailed analysis of the world economy; considers issues affecting industrial
countries, developing countries, and economies in transition to market; and addresses topics of pressing
current interest. Hence, option 1 is not correct.
• Global Economic Prospects (GEP) is a flagship report of the World Bank Group, which examines
global economic developments and prospects, with a special focus on emerging markets and developing
economies, on a semi-annual basis (in January and June).Each edition includes analytical pieces on topical
policy challenges faced by these economies. Hence, option 2 is correct.
• The World Development Report (WDR) is an annual report published since 1978 by the
International Bank for Reconstruction and Development (IBRD) or World Bank. Each WDR
provides an in-depth analysis of a specific aspect of economic development. Past reports have considered
such topics as agriculture, youth, equity, public services delivery, the role of the state, transition
economies, labour, infrastructure, health, the environment, risk management, and poverty. The reports are
the Bank's best-known contribution to thinking about development. Hence, option 3 is correct.
• The Global Financial Stability Report by IMF provides an assessment of the global financial system
and markets and addresses emerging market financing in a global context. It focuses on current market
conditions, highlighting systemic issues that could pose a risk to financial stability and sustained market
access by emerging market borrowers. Hence, option 4 is not correct.

Q 58.D
• India has adopted the five-year plan model which was practiced in the earlier communist Soviet
Union. The Five-Year Plan exercise is a detailed work plan. To begin with an Approach Paper is prepared
to identify the growth target and the sectors to be prioritised in the five year plan. After the Approach
Paper is discussed and finalised in the highest policy making body viz; the National Development
Council, the subject divisions in Planning Commission representing the different Central Ministries set up
Working Groups wherein subject experts, state government officials and central government officials are
Members and they discuss and chart out the course of action to be implemented in the next five
years. Hence option (d) is the correct answer.
• Our plan documents upto the year 2017 not only specify the objectives to be attained in the five years of a
plan but also what is to be achieved over a period of twenty years. This long-term plan is called
‘perspective plan’. The five year plans were supposed to provide the basis for the perspective plan. The
goals of the five year plans were: growth, modernisation, self-reliance and equity. This does not mean that
21 www.visionias.in ©Vision IAS

FREE BY KING R QUEEN P [ऋषभ राजपूत]


all the plans have given equal importance to all these goals. Due to limited resources, a choice has to be
made in each plan about which of the goals is to be given primary importance.

Q 59.C
• Ricardian equivalence:
o This theory was developed by David Ricardo in the early 19th century and later was elaborated upon
by Harvard professor Robert Barro. For this reason, Ricardian equivalence is also known as the
Barro-Ricardo equivalence proposition.
o It is an economic theory that says that financing government spending out of current taxes or future
taxes (and current deficits) will have equivalent effects on the overall economy. Hence option (c) is
the correct answer.
o This means that attempts to stimulate an economy by increasing debt-financed government spending
will not be effective because investors and consumers understand that the debt will eventually have to
be paid for in the form of future taxes.
o The theory argues that people will save based on their expectation of increased future taxes to be
levied in order to pay off the debt and that this will offset the increase in aggregate demand from the
increased government spending. This also implies that Keynesian fiscal policy will generally be
ineffective at boosting economic output and growth.

Q 60.B
• Masala Bonds: They are Indian rupee-denominated bonds issued in offshore capital markets. Hence,
statement 1 is not correct.
o They are issued to offshore investors settled in dollars and, therefore, the currency risk lies with the
investor and not the issuer, unlike external commercial borrowings – where Indian companies raise
money in foreign currency loans. Hence, statement 2 is correct.
• Benefits of Masala Bonds:
o Companies do not have to worry about rupee depreciation.
o Masala bonds help protect corporate balance sheets from exchange rate risks, however, their issuance
should be used in moderation.
o Masala bonds can have implications for the rupee, interest rates, and the economy as a whole.

Q 61.C
• The Special Drawing Rights (SDR) is an international reserve asset, created by the IMF in 1969 to
supplement its member countries’ official reserves. The SDR is neither a currency nor a claim on the
IMF. Rather, it is a potential claim on the freely usable currencies of IMF members. SDRs can be
exchanged for these currencies. The SDR serves as the unit of account of the IMF and some other
international organizations. The currency value of the SDR is determined by summing the values in US
dollars, based on market exchange rates, of a SDR basket of currencies.
• Hence option (c) is the correct answer.

Q 62.A
• Direct Taxes:
o Direct taxes are levied on an individual’s property or company's property and revenue. Direct taxes
are levied on businesses and individuals and are paid directly to the government.
o Direct taxes have an impact on people's income levels as well as their purchasing power. It also aids
in the adjustment of the economy's aggregate demand. Direct taxation can be proportional,
progressive, or regressive.
o Tax revenues, an important component of revenue receipts, comprise direct taxes – which fall directly
on individuals (personal income tax) and firms (corporation tax), and indirect taxes like excise taxes
(duties levied on goods produced within the country), customs duties (taxes imposed on goods
imported into and exported out of India) and service tax.
o Other direct taxes are wealth tax, gift tax, and estate duty (now abolished). Hence option (a) is the
correct answer.

Q 63.B
• Recent context: Indian Space Research Organisation (ISRO), Polar Satellite Launch Vehicle C55 (PSLV-
C55) mission was launched on 22 April 2023. This was a dedicated commercial mission through NSIL
with TeLEOS-2 as the primary satellite and Lumelite-4 as a co-passenger satellite weighing 741 kg and 16
kg respectively.
22 www.visionias.in ©Vision IAS

FREE BY KING R QUEEN P [ऋषभ राजपूत]


• The PSLV-C55 mission has the PSLV Orbital Experimental Module (POEM), where the spent PS4 of the
launch vehicle would be utilised as an orbital platform to carry out scientific experiments through non-
separating payloads.
• This is the third time that PS4 will be used after satellite separation as a platform for experiments.
According to the space agency, POEM has seven experimental non-separable payload.
• PSLV Orbital Experimental Module, Poem is the spent fourth stage of the launch vehicle that would be
used as an orbital platform to carry out scientific experiments through non-separating payloads. PSLV is a
four-stage rocket, and while the first three stages are jettisoned into the ocean after they push the mission
to desired orbit, the four-stage remains in orbit and becomes space junk.
• Isro is now repurposing this fourth stage to use an experimental platform. The mission will carry seven
non-separable payloads from Isro, Bellatrix, Dhruva Space, and the Indian Institute of Astrophysics neatly
packed on this fourth stage. The fourth stage or the orbital experimental platform, Poem, is powered by
solar panels and is fitted with its own Navigation Guidance and Control (NGC) system, which helps in
attitude stabilization. This is not the first time that Isro is repurposing the fourth stage to use for in-house
experiments, it has done that a couple of times in the past as well. Hence, option (b) is the correct
answer.
o A reusable launch vehicle has parts that can be recovered and reflown, while carrying payloads from
the surface to outer space.
o A Fractional Orbital Bombardment System is a warhead delivery system that uses a low earth orbit
towards its target destination. Just before reaching the target, it deorbits through a retrograde engine
burn.
o 'Project NETRA' is an early warning system in space to detect debris and other hazards to Indian
satellites.

Q 64.C
• The Budget can also be presented to the House in two or more parts and when such presentation takes
place, each part shall be dealt with as if it were the budget. Further, there shall be no discussion of the
budget on the day on which it is presented to the House.. The finance minister presents the budget with a
speech known as the ‘budget speech’. At the end of the speech in the Lok Sabha, the budget is laid before
the Rajya Sabha, which can only discuss it and has no power to vote on the demands for grants. The
budget documents presented to the Parliament comprise of the following :
o Budget Speech
o Annual Financial Statement
o Demands for Grants
o Appropriation Bill
o Finance Bill
o Statements mandated under the FRBM Act:
✓ Macro-Economic Framework Statement
✓ Fiscal Policy Strategy Statement
✓ Medium Term Fiscal Policy Statement
o Expenditure Budget
o Receipts Budget
o Expenditure Profile
o Memorandum Explaining the Provisions in the Finance Bill (xi) Budget at a Glance
o Outcome Budget
• Earlier, the Economic Survey also used to be presented to the Parliament along with the budget. Now, it
is presented one day or a few days before the presentation of the budget. This report is prepared by the
finance ministry and indicates the status of the national economy. Hence option (c) is the correct
answer. Hence option (c) is the correct answer.

Q 65.D
• Rules of origin are the rules to attribute a country of origin to a product in order to determine its
"economic nationality". The need to establish rules of origin stems from the fact that the
implementation of trade policy measures, such as tariffs, quotas, trade remedies, in various cases,
depends on the country of origin of the product at hand.
• Rules of origin are used:
o to implement measures and instruments of commercial policy such as anti-dumping duties and
safeguard measures;

23 www.visionias.in ©Vision IAS

FREE BY KING R QUEEN P [ऋषभ राजपूत]


o to determine whether imported products shall receive most-favoured-nation (MFN) treatment or
preferential treatment;
o for the purpose of trade statistics;
o for the application of labelling and marking requirements; and
o for government procurement.
• General Agreement on Tariffs and Trade (GATT) has no specific rules governing the determination
of the country of origin of goods in international commerce. Each contracting party was free to
determine its own origin rules, and could even maintain several different rules of origin depending on
the purpose of the particular regulation.
• During international trade, an exporting country needs to show a certificate under norms of "rules of
origin" to prove that the commodity or a product originates there. Rules of origin norms help in
containing dumping of goods. Hence option (d) is the correct answer.

Q 66.C
• Millions of baby Olive Ridley sea turtles crawled towards the Bay of Bengal after emerging from
eggshells along Odisha’s Rushikulya beach in Ganjam district, recording one of the most successful mass
nesting and hatchings in the past few decades.
• Usually, mass nesting takes place for three to four days. However, turtles had come to lay their eggs over
a period of nine days this year. After laying eggs, they disappeared into the sea.
• After 50 days, baby turtles have emerged from eggs themselves, without mother turtles, and started their
journey towards an unknown destination using a vast seawater route.
• Generally, an Olive Ridley turtle lays 100-150 eggs in a cavity created by them with their front
flippers. They scoop out sand for hours to create the void. After laying eggs in one go, these
creatures cover it again with sand. Before sunrise, the turtles return to the sea, leaving behind the
eggs to hatch after 40-60 days. Sometimes, a turtle lays eggs in a pit at a place which was previously
used by another turtle, leading to the loss of thousands of eggs. Hence, statement 2 is correct.
• The Rushikulya beach is a unique phenomenon, which is not a wildlife sanctuary, yet turtles feel
safe to carry out mass nesting. Baby turtles started coming out from eggshells in the second week of
April this year. Hence, statement 3 is not correct.
• Once a turtle finds the right spot, it settles down to make a shallow nest where it will lay about 100 eggs
on average. A turtle uses its flippers to scoop out sand to create a cavity 30 cm to 50 cm deep to lay eggs.
• Growing to about half a metre and 50 kg in weight, the Olive Ridley turtle gets its name from its green-
grey carapace (top shell). It is the smallest of all the sea turtle species. Olive Ridley turtles, a
vulnerable species according to the International Union for Conservation of Nature, come ashore to
the Rushikulya river mouth in Odisha to lay eggs. Hence, statement 1 is not correct.
• The Olive Ridley turtles do not wait to see the eggs hatch but come back next season to lay eggs again.
They spend their lives in the ocean, inhabiting the tropical and subtropical waters of the Pacific, Indian,
and Atlantic oceans.
• Turtles also arrive at Gahirmatha Beach in Odisha’s Kendrapara district, known as the world’s largest
rookery. Besides, Puri and Devi river mouth beaches too host Olive Ridley turtles this time around.
• As part of a long-term study, researchers of the Zoological Survey of India (ZSI) continued tagging of
Olive Ridley turtles at three mass nesting sites – Gahirmatha, Devi river mouth, and Rushikulya. The
metal tags affixed to turtles are non-corrosive and they do not harm their body. The metal can be removed
later. The tags are uniquely numbered containing details such as the name of the organization, country
code, and email address.

Q 67.B
• Recently, West Bengal notified Char Balidanga (Nadia), Namthing Pokhari (Darjeeling), Amkhoi Wood
Fossil Park (Birbhum) and State Horticulture Research and Development Station (Nadia) as the four latest
Biodiversity Heritage Sites (BHS) making it the state with the highest number of BHS in India.
• Biodiversity Heritage Sites are well-defined areas that are unique, ecologically fragile ecosystems with a
high diversity of wild and domesticated species, the presence of rare and threatened species, and
keystone species.
• The State Government, in cooperation with local bodies, may designate sites of biological importance
as Biodiversity Heritage Sites under the Biological Diversity Act of 2002.
o Gandhamardan Hill Range is located in Bargarh and Balangir districts, Odisha. Hence pair 1 is not
correctly matched.

24 www.visionias.in ©Vision IAS

FREE BY KING R QUEEN P [ऋषभ राजपूत]


✓ This is the third BHS of Odisha after Mandasuru Gorge in the Kandhamal district and
Mahendragiri Hill Range in the Gajapati district.The Gandhamardan Hill, located in both Balangir
and Bargarh, is considered as the treasure trove of medicinal plants and an Ayurvedic paradise of
Odisha. This ecologically fragile ecosystem is rich in floral and faunal diversity having diverse
socio-economic, ecological and biological significance for the people of Odisha.As per the
government, the floral diversity of the hill comprises 1,055 plant species that include 849
angiosperms, 56 pteridophytes, 40 bryophytes, 45 lichens and two gymnosperms and 63 species
of macrofungi. Moreover, the faunal diversity comprises 500 species of animals that include 43
mammals, 161 birds, 44 reptiles, 16 amphibians, 118 butterflies, 27 dragonflies and 7 damselflies,
and 83 species of spiders. One angiosperm, Ficus conccina var dasycarpa and one spider, Peucetia
harishankarensis, are endemic to this hill. Besides, two historical monuments such as the
Nrusinghanath Temple located on the northern slope and the Harishankar Temple situated on the
southern slope of the foothills of Gandhamardan, have immense cultural significance.The two hill
shrines are major pilgrimage sites of Odisha. In his chronicles, Hiuen Tsang had described the hill
shrine as a Buddhist heritage site named Parimalagiri.In the 1980s, then public sector Bharat
Aluminium Company Limited (BALCO), now under the control of Vedanta Group, had almost
started mining of bauxite from Gandhamardan.
o Arittapatti is located in Madurai district, Tamil Nadu. Hence pair 2 is correctly matched.
✓ Arittapatti village, known for its ecological and historical significance, houses around 250 species
of birds including three important raptors - birds of prey, namely the Laggar Falcon, the Shaheen
Falcon and Bonelli’s Eagle.Arittapatti village of Melur block in Madurai. It is also home to
wildlife such as the Indian Pangolin, Slender Loris and pythons. The area is surrounded by a chain
of seven hillocks or inselbergs that serve as a watershed, charging “72 lakes, 200 natural springs
and three check dams,” the notification said. The Anaikondan tank, built during the reign of
Pandiyan kings in the 16th century is one among them. Several megalithic structures, rock-cut
temples, Tamil Brahmi inscriptions and Jain beds add to the historical significance of the region.
o Namthing Pokhari is located in Darjeeling, West Bengal. Hence pair 3 is not correctly matched.
✓ Titled as the natural habitat of the rare endangered species of Himalayan Salamander, Namthing
Pokhari is the only place except for Jorpokhari in Darjeeling to inhabit these rarest species in the
world. The Himalayan Salamander in Namthing Pokhari is cited to fall under the Tylototriton
verrucosus species which is found only in Darjeeling. Therefore, this unique amphibian is
conserved under Schedule II Part I of the Indian Wildlife Protection Act, 1972. The Himalayan
Salamander found in Namthing Pokhari is a keystone species of the lentic zones in the Eastern
Himalayas. This endangered species belongs to an offshoot of the ancient family Salamandridae,
which is known to have existed in Europe during the Miocene age, that is, between 13-25 million
years ago. These salamanders are now entirely vanished in the other part of the world.
Fortunately, they are still surviving in the solitary Namthing Lake of Shelpu Hills.

Q 68.D
• Dakar Declaration
o ‘Dakar Declaration’ was adopted by 21 African countries committing to strengthening reporting on
road crash fatalities, enhancing data capture, analysis, sharing, and coordination to shape better road
safety policies. Hence option (d) is the correct answer.
o It is being conducted at the first African sub-regional conference on implementing the Global Plan for
Road Safety in Dakar, Senegal.
o Other treaties on road safety:
✓ The decade of Action for road safety 2021–2030 (target of preventing at least 50% of road traffic
deaths and injuries by 2030);
✓ Stockholm Declaration on Road Safety 2020;
✓ National Road Safety Policy 2010;
✓ Delhi Declaration on Road Safety 2021

Q 69.C
• Consumer Price Indices (CPI) measures change over time in the general level of prices of goods and
services that households acquire for the purpose of consumption. Hence statement 1 is correct.
• In India, CPI measured by the Ministry of Statistics and Programme Implementation with Base
Year 2012. In India, there are four consumer price index numbers, which are calculated, and these are as
follows:
o CPI for Industrial Workers (IW)
25 www.visionias.in ©Vision IAS

FREE BY KING R QUEEN P [ऋषभ राजपूत]


o CPI for Agricultural Labourers (AL)
o CPI for Rural Labourers (RL) and
o CPI for Urban Non-Manual Employees (UNME). Hence statement 2 is correct.
• While the Ministry of Statistics and Program Implementation collects CPI (UNME) data and compiles it,
the remaining three are collected by the Labour Bureau in the Ministry of Labour.
• Inflation is measured using CPI. The percentage change in this index over a period of time gives the
amount of inflation over that specific period, i.e. the increase in prices of a representative basket of goods
consumed.
• CPI is calculated for a fixed list of items including food, housing, apparel, transportation, electronics,
medical care, education, etc. The weightage of food and beverages in the CPI is close to 50%. Hence
statement 3 is correct.

Q 70.D
• Recently, the Voyager 2 spacecraft, which is now travelling in interstellar space, has gotten a new
lease of life after mission engineers developed a new plan to keep its instruments running for longer
till 2026.
• Voyager 2 was launched by NASA in 1977, with its initial mission objective of probing space.
Currently, it is so far away from Earth that radio signals from the spacecraft take 18 hours to reach us.
• Voyager 2 and Voyager 1 are the only spacecraft that has ever operated outside the heliosphere,
which is considered to be the border of our solar system. The heliosphere is a bubble of particles and
magnetic fields generated by the Sun.
• NASA's New Horizons spacecraft was the first spacecraft to explore Pluto up close; Cassini-Huygens,
commonly called Cassini, was a space-research mission by NASA, the European Space Agency, and the
Italian Space Agency to send a space probe to study the planet Saturn; Juno is a NASA space probe
orbiting the planet Jupiter.
• Hence, option (d) is the correct answer.

Q 71.C
• The Capital Account: The Capital Budget is an account of the assets as well as liabilities of the central
government, which takes into consideration changes in capital. It consists of capital receipts and capital
expenditure of the government. This shows the capital requirements of the government and the pattern of
their financing.
o Capital Receipts: The main items of capital receipts are loans raised by the government from the
public which are called market borrowings, borrowing by the government from the Reserve Bank and
commercial banks and other financial institutions through the sale of treasury bills, loans received
from foreign governments and international organizations, and recoveries of loans granted by the
central government. Other items include small savings (Post-Office Savings Accounts, National
Savings Certificates, etc), provident funds, and net receipts obtained from the sale of shares in Public
Sector Undertakings (PSUs). Hence option (c) is the correct answer.
o Capital Expenditure: This includes expenditure on the acquisition of land, buildings, machinery,
equipment, investment in shares, and loans and advances by the central government to state and union
territory governments, PSUs, and other parties. Capital expenditure is also categorized as plan and
non-plan in the budget documents.

26 www.visionias.in ©Vision IAS

FREE BY KING R QUEEN P [ऋषभ राजपूत]


✓ Plan capital expenditure, like its revenue counterpart, relates to the central plan and central
assistance for state and union territory plans.
✓ Non-plan capital expenditure covers various general, social, and economic services provided by
the government.
o Note: Revenue receipts are current income receipts from all sources such as taxes, profits of public
enterprises, grants, etc. Revenue receipts neither create any liability nor cause any reduction in the
assets of the government. Capital receipts, on the other hand, are the receipts of the government that
either create liability or cause any reduction in the assets of the government. e.g., borrowings,
recovery of loans, and disinvestment, etc.

Q 72.C
• National Bank for Agriculture and Rural Development (NABARD) was set up in 1982 as an apex body to
coordinate the activities of all institutions involved in the rural financing system. It was established in
1982 under the National Bank for Agriculture and Rural Development Act, 1981. Hence option (c) is the
correct answer.
• NABARD is a development bank in India that focuses on providing financial and other support to the
agriculture and rural development sectors. It offers a range of financial products and services, including
various funds, to meet the needs of its clients.
• NABARD is responsible for supervising and providing financial assistance to the Regional Rural Banks
(RRBs) in India. RRBs are public sector banks that were established to provide banking and financial
services to the rural and semi-urban areas of the country.
• Some of the funds managed by NABARD include:
o Rural Infrastructure Development Fund (RIDF),
o Dairy Processing and Infrastructure Development Fund (DIDF),
o Rural Innovation Fund (RIF)
• The origins of priority sector (PS) lending can be traced back to 1966 when Morarji Desai saw a
need for increasing credit to agriculture and small industries. However, the definition of PS was
only formalized based on a Reserve Bank of India (RBI) report in the National Credit Council in
1972. The Priority Sector norms are applicable to apply to every Commercial Bank [including Regional
Rural Bank (RRB), Small Finance Bank (SFB), and Local Area Bank], and Primary (Urban) Co-operative
Bank (UCB) other than Salary Earners’ Bank. As per the PSL norms, the Domestic commercial banks
(excluding RRBs & SFBs) & foreign banks with 20 branches and above are mandated to allocate 40
percent of Adjusted Net Bank Credit (ANBC) to the priority sectors.
• Rural Banks were set up on the basis of the recommendations of the Narasimham Working Group (1975),
and after the legislation of the Regional Rural Banks Act, 1976. The first Regional Rural Bank “Prathama
Grameen Bank” was set up on October 2, 1975.
• The first phase of the nationalization of banks took place in 1969. 14 banks were nationalized that year.

Q 73.A
• In 2016, the Reserve Bank of India Act, 1934 (RBI Act) was amended to provide for a statutory and
institutionalised framework for a Monetary Policy Committee (MPC). The Preamble in the RBI Act, as
amended by the Finance Act, 2016, now provides that the primary objective of the monetary policy is to
maintain price stability, while keeping in mind the objective of growth, and to meet the challenge of an
increasingly complex economy. RBI would, accordingly, operate a Monetary Policy Framework. Thus,
now there is a statutory basis for a Monetary Policy Framework and the MPC. Hence statement 1 is
correct.
• A Committee-based approach will add lot of value and transparency to monetary policy decisions. Out of
the six Members of MPC, three Members will be from the Reserve Bank of India (RBI), including the
Governor, RBI, who will be the ex-officio Chairperson, the Deputy Governor, RBI and one officer of
RBI. The other three Members of MPC will be appointed by the Central Government, on the
recommendations of a Search-cum-Selection Committee. These three Members of MPC will be experts in
the field of economics or banking or finance or monetary policy and will be appointed for a period of 4
years and shall not be eligible for re-appointment. The meetings of the MPC shall be held at least 4 times
a year and it shall publicise its decisions after each such meeting.
• Under the new statutory framework, the central government would, in consultation with the Reserve
Bank of India (RBI), set an inflation target based on the consumer price index (CPI) once every five
years. The RBI was entrusted with the responsibility of meeting this target (“accountability”), for which it
would be given “independence” in the conduct of monetary policy. Hence statement 2 is not correct.

27 www.visionias.in ©Vision IAS

FREE BY KING R QUEEN P [ऋषभ राजपूत]


• The inflation target of the Reserve Bank of India is 4 percent, with a 2 per cent extension in the
upper bound and 2 per cent in the lower bound.

Q 74.C
• The Fiscal Responsibility and Budget Management Act (FRBM Act), 2003, establishes financial
discipline to reduce fiscal deficit. The FRBM Act aims to introduce transparency in India's fiscal
management systems. The Act’s long-term objective is for India to achieve fiscal stability and to give the
Reserve Bank of India (RBI) flexibility to deal with inflation in India. The FRBM Act was enacted to
introduce more equitable distribution of India's debt over the years.
• Key features of the FRBM Act:
o The FRBM Act made it mandatory for the government to place the following along with the Union
Budget documents in Parliament annually:
✓ Medium Term Fiscal Policy Statement
✓ Macroeconomic Framework Statement
✓ Fiscal Policy Strategy Statement
o The FRBM Act proposed that revenue deficit, fiscal deficit, tax revenue and the total outstanding
liabilities be projected as a percentage of gross domestic product (GDP) in the medium-term fiscal
policy statement. Hence option (c) is the correct answer.
• On grounds of national security, calamity, etc, the set targets of fiscal deficits and revenue could be
exceeded.

Q 75.A
• Regional Rural Banks (RRBs) are financial institutions that ensure adequate credit for agriculture and
other rural sectors. Regional Rural Banks were set up on the basis of the recommendations of the
Narasimham Working Group (1975), and after the legislation of the Regional Rural Banks Act, 1976. The
first Regional Rural Bank “Prathama Grameen Bank” was set up on October 2, 1975. Hence statement 1
is correct.
• At present, there are around 40 RRBs in India (43 as of 2020). The equity of a regional rural bank is held
by the Central Government, the concerned State Government, and the Sponsor Bank in the proportion of
50:15:35. Hence statement 2 is not correct.
• The RRBs combine the characteristics of a cooperative in terms of familiarity with rural problems and a
commercial bank in terms of its professionalism and ability to mobilize financial resources. Each RRB
operates within the local limits as notified by Government.
• The main objectives of RRBs are to provide credit and other facilities‚ especially to the small and
marginal farmers‚ agricultural laborers artisans and small entrepreneurs in rural areas with the objective of
bridging the credit gap in rural areas, checking the outflow of rural deposits to urban areas and reduce
regional imbalances and increase rural employment generation.

Q 76.A
• Balance of Payments Surplus and Deficit
o The essence of international payments is that just like an individual who spends more than her income
must finance the difference by selling assets or by borrowing, a country that has a deficit in its current
account (spending more than it receives from sales to the rest of the world) must finance it by selling
assets or by borrowing abroad. Thus, any current account deficit must be financed by a capital
account surplus, a net capital inflow.
o In this case, in which a country is said to be in balance of payments equilibrium, the current account
deficit is financed entirely by international lending without any reserve movements.
o Alternatively, the country could use its reserves of foreign exchange in order to balance any
deficit in its balance of payments. The reserve bank sells foreign exchange when there is a
deficit. This is called an official reserve sale. Hence option (a) is the correct answer.
o The decrease (increase) in official reserves is called the overall balance of payments deficit (surplus).
The basic premise is that the monetary authorities are the ultimate financiers of any deficit in the
balance of payments (or the recipients of any surplus).

Q 77.B
• An inflationary gap measures the difference between the current real GDP and the GDP of an economy
operating at full employment. Hence option (b) is the correct answer.
• The current real GDP must be higher than the potential GDP for the gap to be considered inflationary.

28 www.visionias.in ©Vision IAS

FREE BY KING R QUEEN P [ऋषभ राजपूत]


• Policies that reduce an inflationary gap include reductions in government spending, tax increases, bond
and securities issues, interest rate increases, and transfer payment reductions. H
• An inflationary gap exists when the demand for goods and services exceeds production due to factors such
as higher levels of overall employment, increased trade activities, or elevated government expenditure.
The real GDP can exceed the potential GDP, resulting in an inflationary gap. The inflationary gap is
named as such because the relative rise in real GDP causes an economy to increase its consumption,
leading prices to climb in the long run.

Q 78.C
• PM Street Vendor’s AtmaNirbhar Nidhi (PM SVANidhi):
o A total of 42.7 lakh loans amounting to ₹5,152.37 crore had been disbursed to street vendors under
the PM SVANidhi scheme
o This is a Central Sector Scheme to facilitate street vendors to access affordable working capital loans
for resuming their livelihood activities, after the easing of the lockdown. Hence, statement 1 is
correct.
o The COVID-19 pandemic and consequent lockdowns have adversely impacted the livelihoods of
street vendors. They usually work with a small capital base, which they might have consumed during
the lockdown. Therefore, credit for working capital to street vendors will be helpful to resume their
livelihoods.
o To facilitate an initial working capital loan of up to 10,000 at a subsidized rate of interest.
Hence, statement 2 is not correct.
o To incentivize regular repayment of loans; and
o To reward digital transactions.
✓ Initial working capital of up to `10,000/- rupees
✓ Interest subsidy on timely/ early repayment@ 7%
✓ Monthly cash-back incentive on digital transactions
✓ Higher loan eligibility on timely repayment of the first loan.
o Street vendors/ hawkers vending in urban areas, as on or before March 24, 2020, including the
vendors of surrounding peri-urban and rural areas. Hence, statement 3 is correct.
o No collateral security is required.
o The interest subsidy amount will be credited directly to the beneficiary account on a quarterly basis.
In case of early payment, the admissible amount of subsidy will be credited in one go.

Q 79.B
• Balance of Payments (BoP) of a country is a systematic record of all economic transactions between
the residents of one country and the rest of the world during a given period of time. It summarizes all
transactions that a country's individuals, companies and government bodies complete with individuals,
companies and government bodies outside the country. These transactions consist of imports and exports
of goods, services and capital, as well as transfer payments such as foreign aid and remittances. Hence,
option (b) is the correct answer.
• An example of Balance of Payments as per Economic Survey 2022-23.

29 www.visionias.in ©Vision IAS

FREE BY KING R QUEEN P [ऋषभ राजपूत]


Q 80.D
• General Agreements on Trade in Services (GATS)
o The GATS was inspired by essentially the same objectives as its counterpart i.e. General
Agreement on Tariffs and Trade (GATT) covers international trade in goods. The workings of
the GATT agreement are the responsibility of the Council for Trade in Goods (Goods Council)
which is made up of representatives from all WTO member countries.
o Services negotiations in the WTO follow the so-called positive list approach, whereby members’
schedules of specific commitments list all of the services sectors and sub-sectors where they
undertake to bind the market opening and the granting of national treatment to foreign service
suppliers, apart from the listed barriers that remain.
30 www.visionias.in ©Vision IAS

FREE BY KING R QUEEN P [ऋषभ राजपूत]


o Sectors and sub-sectors not included in the schedule are exempt from any obligations as regards
market access and national treatment. West is pushing hard to move from a positive list approach
to a negative list approach.
o In the negative list approach, services, where GATS is not applicable, will have to be negotiated,
agreed upon and specified. India is against this concept as it will throw open almost the whole
Indian services sector to Western multinational giants.
o Negotiations in services under GATS are classified in 4 modes, interests of different countries depend
upon this classification –
✓ Mode 1 – Cross border trade- It includes the cross-border supply of services without the
movement of natural persons. For eg. Business Process Outsourcing, KPO or LPO services. Here,
it’s in India’s interest to push for liberalization given its large human resource pool and
competitive IT industry. Hence, pair 1 is not correctly matched.
✓ Mode 2 – Consumption abroad- It refers to in the territory of one Member to the service
consumer of any other Member. Nationals of A have moved abroad as tourists, students, or
patients to consume the respective services. Hence, pair 2 is not correctly matched.
✓ Mode 3 – Commercial presence – which covers services provided by a service supplier of one
country in the territory of any other country. This opens the door of relevant sectors in one
country to investments from another country. Accordingly, it is in the West’s interest to push for
liberalization here. There has been sustained pressure to open up the higher education sector,
insurance sector, Medical sector etc through this mode. Hence, pair 3 is not correctly matched.
✓ Mode 4 – Presence of natural persons – which covers services provided by a service supplier of
one country through the presence of natural persons in the territory of any other country. E.g.
Infosys or TCS sending its engineers for onsite work in US/Europe or Australia. Here again, it’s
in India’s interest to push for liberalization. In 2012, India dragged the US to the World Trade
Organization’s (WTO) dispute settlement body (DSB) over an increase in the professional visa
fee (H1B/L1). Hence, pair 4 is not correctly matched.

Q 81.B
• In a first-ever census of mangrove pitta birds carried out in two coastal districts of Odisha, 179 such birds
were sighted. Mangrove pitta is a resident non-migratory bird that generally feeds on crustaceans,
mollusks and insects. Hence, option (b) is the correct answer.
• Mangrove pitta birds are a nearly threatened species found in a few pockets of eastern India, including
Odisha’s Bhitarkanika and West Bengal’s Sundarbans.
• The first census of mangrove pitta (Pitta megharencha) birds were mainly focused on the mangrove
patches all along the coasts of the Kendrapara and Jagatsingpur districts. “The mangrove pitta is found in
coastal mangrove forests of India, foraging on the ground and resting on the trees,”
• The census was carried out by point count method, either by walking in the forest or using country boats
in the creeks. In this census, a total of 179 individual mangrove pitta birds were counted. The highest
concentration of the birds has been found in the mangroves near the Mahipura river mouth inside
the Bhitarkanika National Park.
• The first-ever population census of the mangrove pitta bird (Pitta megarhyncha) in Bhitarkanika national
park revealed that the Ramsar site is home to around 179 of the avian species. “Reports indicate a
population of the bird in the core areas is good. The present report reflects results from the direct count
method during which all water bodies and mangrove forest areas of the park were monitored and a survey
was carried out accordingly.
• The breeding season of this species ranges from April to August in Bhitarkanika. Abundant fish in the
river and creeks and distance from human habitation has made Bhitarkanika a suitable congenial breeding
place for this bird species.
• The Bhitarkanika National Park witnessed the arrival of 1,39,959 birds of 140 species in January this year.
The birds were counted by direct sighting and from their chirping.

Q 82.A
• Recent context: University Grants Commission (UGC) has released the final draft of the National
Credit Framework (NCrF).
• To fulfill the vision of National Education Policy 2020, making education more holistic and effective
and lay emphasis on the integration of general (academic) education, vocational education and
Experiential learning including relevant experience and professional levels acquired, it becomes
imperative to establish and formalize a national credit accumulation and transfer system which will

31 www.visionias.in ©Vision IAS

FREE BY KING R QUEEN P [ऋषभ राजपूत]


integrate both general & vocational education while ensuring mobility of candidates between the
two systems. Hence, option (a) is the correct answer.
• The National Credit Framework (NCrF) has been jointly developed by UGC, AICTE, NCVET, NIOS,
CBSE, NCERT, Ministry of Education, DGT, and Ministry of Skill Development to achieve this vision
and intent of NEP. NCrF is a comprehensive framework encompassing elementary, school, higher, and
vocational education & training, integrating learning on all dimensions i.e. academics, vocational skills
and experiential learning including relevant experience and professional levels acquired.
• The National Credit Framework (NCrF) shall be an inclusive umbrella Framework to seamlessly integrate
the credits earned through school education, higher education and vocational & skill education. For
creditisation and integration of all learning, the National Credit Framework (NCrF) shall encompass the
qualification frameworks for higher education, vocational & skill education and school education, namely
National Higher Education Qualification Framework (NHEQF), National Skills Qualification Framework
(NSQF) and National School Education Qualification Framework (NSEQF) also popularly known as
National Curricular Framework (NCF) respectively.
• The implementation of NCrF would be a game changer in realising the vision and intent of NEP by
removing distinction, ensuring flexibility & mobility and establishing academic equivalence between
general and vocational education. Such integration shall open numerous options for further progression of
students and inter-mingling of school & higher education with vocational education & Experiential
learning including relevant experience and professional levels acquired, to further enable entry and re-
entry from vocational stream to general education and vice-versa, thus mainstreaming the vocational
education and skilling.
• The National Credit Framework (NCrF) provides for broad-based, multi-disciplinary, holistic education,
allowing imaginative and need-based curricular structures and enabling creative combinations of subjects
and disciplines. The Framework has been built on the strength of existing regulations, guidelines and
qualification frameworks of UGC, AICTE, NCVET, NCERT, CBSE & NIOS so that the options for
Multiple Entry-Multiple Exit (ME-ME) are accessible and applicable across the higher education, school
education and vocational education.

Q 83.D
• In WTO terminology, subsidies in general are identified by “boxes” which are given the colours of
traffic lights: green (permitted), amber (slow down — i.e. need to be reduced), red (forbidden). In
agriculture, things are, as usual, more complicated. The Agreement on Agriculture has no red box,
although domestic support exceeding the reduction commitment levels in the amber box is prohibited; and
there is a blue box for subsidies that are tied to programmes that limit production.
• Amber Box: Nearly all domestic support measures considered to distort production and trade (with some
exceptions) fall into the amber box. These include measures to support prices, or subsidies directly related
to production quantities. Hence statement 2 is not correct.
• Green Box: The green box is defined in Annex 2 of the Agreement on Agriculture.
o In order to qualify, green box subsidies must not distort trade, or at most cause minimal distortion.
o They have to be government-funded (not by charging consumers higher prices) and must not involve
price support.
o They tend to be programmes that are not targeted at particular products, and include direct income
supports for farmers that are not related to (are “decoupled” from) current production levels or
prices. Hence statement 1 is not correct.
o They also include environmental protection and regional development programmes.
o Green box subsidies are therefore allowed without limits, provided they comply with the policy-
specific criteria. India's Public distribution system does not come under the Green Box.
• Blue Box: This is the “amber box with conditions” — conditions designed to reduce distortion. Any
support that would normally be in the amber box, is placed in the blue box if the support also requires
farmers to limit production. At present there are no limits on spending on blue box subsidies.

Q 84.A
• Reserve Deposit Ratio: It is the proportion of the total deposits commercial banks keep as reserves.
Banks hold a part of the money people keep in their bank deposits as reserve money and loan out the rest
to various investment projects. Reserve money consists of two things – vault cash in banks and deposits of
commercial banks with RBI. Banks use this reserve to meet the demand for cash by account holders.

32 www.visionias.in ©Vision IAS

FREE BY KING R QUEEN P [ऋषभ राजपूत]


• Statutory Liquidity Ratio: SLR is an obligatory reserve that commercial banks must maintain
themselves. It is a percentage of commercial banks' net demand and time liabilities, maintained as
approved securities.
• Cash Reserve Ratio: Under the cash reserve ratio (CRR), commercial banks have to hold a certain
minimum amount of deposits as reserves with the central bank.
• Currency Deposit Ratio: The currency deposit ratio (cdr) is the ratio of money held by the public in
currency to that they hold in bank deposits. It reflects people’s preference for liquidity. It is a purely
behavioral parameter that depends, among other things, on the seasonal pattern of expenditure. For
example, cdr increases during the festive season as people convert deposits to cash balances for meeting
extra expenditures during such periods. Hence option (a) is the correct answer.

Q 85.D
• The World Bank Group is a family of five international organizations that make leveraged loans to
developing countries. It is the largest and best-known development bank in the world and an
observer at the United Nations Development Group. The bank is headquartered in Washington,
D.C. in the United States.
o The International Bank for Reconstruction and Development offers assistance to middle-income
and poor, but creditworthy, countries. It also works as an umbrella for more specialized bodies under
the World Bank. It was the original arm of the World Bank that was responsible for the reconstruction
of post-war Europe. Before gaining membership in the other four WBG's affiliates a country must be
a member of the IBRD.
o The International Development Association offers loans to the world's poorest countries. These
loans come in the form of "credits" and are essentially interest-free. Hence, option 2 is correct.
o The International Finance Corporation (IFC) works to promote private sector investments by both
foreign and local investors. It provides advice to investors and businesses, and it offers normalized
financial market information through its publications, which can be used to compare across markets.
The IFC also acts as an investor in capital markets and will help governments privatize inefficient
public enterprises. Hence, option 4 is correct.
o The Multilateral Investment Guarantee Agency (MIGA) supports direct foreign investment into a
country by offering security against the investment in the event of political turmoil. These guarantees
come in the form of political risk insurance, meaning that MIGA offers insurance against the political
risk that an investment in a developing country may bear. Hence, option 1 is correct.
o The International Centre for Settlement of Investment Disputes facilitates and works toward a
settlement in the event of a dispute between a foreign investor and a local country. Hence, option 3 is
correct.

Q 86.B
• Recently, Logistic Performance Index (LPI) 2023 was released by World Bank. The LPI is an
interactive benchmarking tool to help countries identify the challenges and opportunities they face in their
performance of trade logistics and what they can do to improve their performance. Hence, statement 1 is
not correct.
• India now ranks 38th in the 139 countries index, this is a significant improvement from its previous
ranking of 44th in 2018. It is a result of significant investments in both soft and hard infrastructure as
well as technology. Hence, statement 2 is correct.
• India's policy interventions include - PM Gati Shakti initiative, National Logistics Policy, Sagarmala,
Bharatmala, etc.

Q 87.A
• The International Monetary Fund (IMF) and the International Bank for Reconstruction and
Development (IBRD), commonly known as the World Bank, were created in 1944 at a meeting of 44
nations (the Allies) at Bretton Woods, New Hampshire to rebuild devastated economies after the war
and to promote economic stability and economic development on a global scale. These institutions thus
came to be known as Bretton Woods Institutions.
• The founders wished to protect future generations against a repeat of a worldwide economic crisis like the
one experienced during the Great Depression which was believed to have partially caused the subsequent
rise of fascism and the outbreak of World War II.
• Hence, option (a) is the correct answer.

33 www.visionias.in ©Vision IAS

FREE BY KING R QUEEN P [ऋषभ राजपूत]


Q 88.C
• Generative artificial intelligence (AI) describes algorithms (such as ChatGPT) that can be used to
create new content, including audio, code, images, text, simulations, and videos. Recent
breakthroughs in the field have the potential to drastically change the way we approach content
creation.
• That’s why ChatGPT—the GPT stands for generative pre-trained transformer—is receiving so much
attention right now. It’s a free chatbot that can generate an answer to almost any question it’s asked.
Developed by OpenAI, and released for testing to the general public in November 2022, it’s already
considered the best AI chatbot ever. And it’s popular too: over a million people signed up to use it in just
five days.
• Starry-eyed fans posted examples of the chatbot producing computer code, college-level essays, poems,
and even halfway-decent jokes. Others, among the wide range of people who earn their living by creating
content, from advertising copywriters to tenured professors, are quaking in their boots
• Bard is a conversational generative artificial intelligence chatbot developed by Google, based on the
LaMDA family of large language models. It was developed as a direct response to the rise of
OpenAI's ChatGPT and was released in a limited capacity in March 2023 to lukewarm responses.
• Ernie Bot, full name Enhanced Representation through Knowledge Integration, is an AI chatbot
service product of Baidu, under development since 2019. It is based on a large language model
named "Ernie 3.0-Titan". It was released on March 17, 2023. Hence, option (c) is the correct
answer.
• A chatbot (originally a chatterbot) is a software application that aims to mimic human conversation
through text or voice interactions, typically online. The term "ChatterBot" was originally coined by
Michael Mauldin (creator of the first Verbot) in 1994 to describe conversational programs.

Q 89.A
• Pradhan Mantri Jan-Dhan Yojana (PMJDY) is National Mission for Financial Inclusion to ensure access
to financial services, namely, basic savings & deposit accounts, remittance, credit, insurance, and pension
in an affordable manner. Under the scheme, a basic savings bank deposit (BSBD) account can be opened
in any bank branch or Business Correspondent (Bank Mitra) outlet, by persons not having any other
account
• Benefits under PMJDY
o One basic savings bank account is opened for unbanked people.
o There is no requirement to maintain any minimum balance in PMJDY accounts.
o Interest is earned on the deposit in PMJDY accounts. Hence, option (a) is the correct answer.
o Rupay Debit card is provided to PMJDY account holders.
o Accident Insurance Cover of Rs.1 lakh (enhanced to Rs. 2 lakhs for new PMJDY accounts
opened after 28.8.2018) is available with a RuPay card issued to the PMJDY account holders.
o An overdraft (OD) facility of up to Rs. 10,000 to eligible account holders is available.
o PMJDY accounts are eligible for Direct Benefit Transfer (DBT), Pradhan Mantri Jeevan Jyoti Bima
Yojana (PMJJBY), Pradhan Mantri Suraksha Bima Yojana (PMSBY), Atal Pension Yojana (APY),
Micro Units Development & Refinance Agency Bank (MUDRA) scheme.
• In the last two financial years, only 329 claims for the accident insurance cover provided to bank account
holders under the Pradhan Mantri Jan Dhan Yojana (PMJDY) have been settled out of the 647 claims that
were filed. The key condition to avail the accident insurance is that the beneficiary must have
performed at least one successful transaction (financial or non-financial) using the card in the 90
days prior to the date of the accident.

Q 90.D
• Direct monetization of deficit refers to a scenario where a central bank prints currency to the tune of
accommodating massive deficit spending by the government. RBI does so by purchasing government
securities directly in the primary market. Such a monetization process used to be automatic only until
1997, when it was later decided to end this practice by entrusting RBI to conduct such OMOs (Open
Market Operations) only in the secondary market. It is only placed as a last resort.
o The printing of new currency is inflationary in nature and increases money supply in the
economy. Hence statement 2 is not correct.
• This is different from the “indirect” monetisation that RBI does when it conducts the Open Market
Operations (OMOs) and/or purchases bonds in the secondary market.

34 www.visionias.in ©Vision IAS

FREE BY KING R QUEEN P [ऋषभ राजपूत]


• An escape clause in the 2017 amendment of the FRBM (Fiscal Responsibility and Budget Management
Act) act permits such direct monetisation under special circumstances. Hence statement 1 is not correct.

Q 91.A
• Liquidity refers to the degree to which an asset, security, or market can be bought or sold quickly
without causing significant price changes. It is a measure of how easily an asset can be converted
into cash or used to facilitate transactions.
• In financial markets, liquidity is essential because it ensures that there is a sufficient number of
buyers and sellers to execute trades efficiently. Highly liquid markets allow investors to enter or exit
positions with minimal impact on the price of the asset. On the other hand, illiquid markets have fewer
participants, making it more challenging to buy or sell assets without affecting their prices.
• Cash is the most liquid of assets, while tangible items are less liquid. The two main types of liquidity are
market liquidity and accounting liquidity. Current, quick, and cash ratios are most commonly used to
measure liquidity.
• Hence, option (a) is the correct answer.

Q 92.D
• The Monetary Policy Committee (MPC) is a committee of the Central Bank in India (Reserve Bank of
India), headed by its Governor, which is entrusted with the task of fixing the benchmark policy interest
rate (repo rate) to contain inflation within the specified target level.
• Monetary Policy Committee is defined in Section 2(iii) (cci) of the Reserve Bank of India Act, 1934, and
is constituted under Sub-section (1) of Section 45ZB of the same Act.
• The MPC replaced the system where the RBI governor, with the aid and advice of his internal team and a
technical advisory committee, had complete control over monetary policy decisions. A Committee-based
approach will add a lot of value and transparency to monetary policy decisions. Thus Statement II is
correct.
• The committee comprises six members - three officials of the Reserve Bank of India and three external
members nominated by the government of India. The governor of the Reserve Bank of India is the
chairperson ex officio of the committee. Decisions are taken by the majority with the governor having the
casting vote in case of a tie. Thus Statement-I is not correct.
• Hence option (d) is the correct answer.

Q 93.A
• Context-The plan to build six nuclear power reactors in Maharashtra's Jaitapur, which is currently the
world's biggest nuclear power generation site under consideration, has been delayed for over a decade due
to issues related to India's nuclear liability law.
• India’s Civil Liability for Nuclear Damage Act (CLNDA) of 2010:
o India enacted the CLNDA in 2010 to put in place a speedy compensation mechanism for victims
of a nuclear accident. Hence statement 1 is correct.
o Liability on Operator: The CLNDA provides for strict and no-fault liability on the operator of
the nuclear plant, where it will be held liable for damage regardless of any fault on its part.
Hence statement 2 is correct.
o It specifies the amount the operator will have to pay in case of damage caused by an accident at
₹1,500 crore. It also requires the operator to cover liability through insurance or other financial
security. Hence statement 3 is correct.
o If the compensation amount exceeds this cap, the central government is responsible for providing
additional funds up to the rupee equivalent of 300 million Special Drawing Rights (SDRs), which is
approximately INR 3,300 crore (USD 450 million).
o Right of recourse: The Act addresses supplier liability in Section 17, which grants the operator a
right of recourse against the supplier under certain conditions. This right of recourse can be invoked if
(a) the contract between the operator and supplier contains such provisions, (b) the nuclear incident
occurs due to the supplier’s negligence, or (c) the supplier provided defective equipment or services
that caused the incident. This provision aims to ensure accountability among suppliers and share the
burden of liability in case of a nuclear accident.
o Claims Commission: The Act provides for the establishment of a Nuclear Damage Claims
Commission to adjudicate claims arising from nuclear accidents. This commission ensures a
streamlined process for victims to seek compensation and resolves disputes between operators,
suppliers, and affected individuals or communities.

35 www.visionias.in ©Vision IAS

FREE BY KING R QUEEN P [ऋषभ राजपूत]


o Time limits for claims: The CLND Act sets a time limit for filing claims for compensation. Claims
related to personal injury or death must be filed within 20 years of the nuclear incident, whereas
claims for damage to property must be filed within 10 years.
o Mandatory insurance: The Act requires nuclear plant operators to obtain insurance or financial
security to cover their liability. This ensures that funds are available for compensation in the event of
an accident.

Q 94.A
• SRI Fund: Self-Reliant India (SRI) fund provides equity funding to micro, small and medium
enterprises (MSMEs) in India. Hence statement 1 is correct.
• Its objectives:
o Supporting faster growth of MSME, supporting enterprises which have the potential to graduate
beyond the MSME bracket and become National/International champions.
o Supporting MSMEs which will help make India self-reliant by producing relevant technologies, goods
and services.
• SRI fund is a SEBI-registered category-II Alternative Investment Fund (AIF) announced in May
2020 that operates through a mother-fund and daughter-fund structure and aims to invest Rs
50,000 crore into MSMEs. Hence statement 2 is correct.
• SRI fund is implemented by NSIC Venture Capital Fund Limited (NVCFL), a wholly-owned subsidiary
of the government’s MSME growth agency National Small Industries Corporation (NSIC). The
government is the sole anchor investor in the SRI fund with the initial support of Rs 10,000 as the mother
fund. The rest 80 per cent of the Rs 50,000 crore fund will be raised by daughter funds from banks,
financial institutions, HNIs, and others. Hence Statement 3 is not correct.

Q 95.C
• Derivatives Types of derivatives
o Forwards: A forward contract is a customized contract between two entities, where settlement takes
place on a specific date in the future at today's pre-agreed price. Hence, pair 1 is correctly matched.
o Futures: A futures contract is an agreement between two parties to buy or sell an asset at a certain
time in the future at a certain price. Futures contracts are special types of forward contracts in the
sense that the former are standardized exchange-traded contracts, such as futures of the Nifty index.
o Options: An Option is a contract that gives the right, but not an obligation, to buy or sell the
underlying at a stated date and price. While a buyer of an option pays the premium and buys the right
to exercise his option, the writer of an option is the one who receives the option premium and is
therefore obliged to sell/buy the asset if the buyer exercises it on him. Hence, pair 2 is correctly
matched.
✓ Options are of two types - Calls and Puts options:
▪ "Calls' give the buyer the right but not the obligation to buy a given quantity of the underlying
asset, at a given price on or before a given future date.
▪ "Puts' give the buyer the right, but not the obligation to sell a given quantity of an underlying
asset at a given price on or before a given future date. •
o Warrants: Options generally have lives of up to one year. The majority of options traded on
exchanges have a maximum maturity of nine months. Longer-dated options are called Warrants and
are generally traded over the counter. Hence, pair 3 is correctly matched.

Q 96.D
• Fiscal policy, in simple terms, is an estimate of taxation and government spending that impacts the
economy. The fiscal policy helps mobilise resources for financing projects. The central theme of fiscal
policy includes development activities like expenditure on railways, infrastructure, etc. Non-development
activities include spending on subsidies, salaries, pensions, etc. It gives incentives to the private sector to
expand its activities.
• Types of fiscal policy- There are two types of fiscal policy:
o A procyclical fiscal policy means increased government spending and reduced taxes during a boom
period, and reduced spending and increased taxes during a recession. Hence statement 2 is not
correct.
o A countercyclical fiscal policy means reducing spending and raising taxes during a boom period, and
increasing spending and cutting taxes during a recession. Hence statement 1 is not correct.

36 www.visionias.in ©Vision IAS

FREE BY KING R QUEEN P [ऋषभ राजपूत]


Q 97.C
• Open Market Operations is a tool by which the Reserve Bank of India influences money supply. Open
Market Operations refers to buying and selling of bonds issued by the Government in the open market.
This purchase and sale is entrusted to the Central bank on behalf of the Government.
• There are two types of open market operations: outright and repo. Outright open market
operations are permanent in nature: when the central bank buys these securities (thus injecting money
into the system), it is without any promise to sell them later. Similarly, when the central bank sells these
securities (thus withdrawing money from the system), it is without any promise to buy them later. As a
result, the injection/absorption of the money is of permanent nature. Hence, statement 1 is correct.
• There is another type of operation in which when the central bank buys the security, this agreement of
purchase also has specifications about the date and price of the resale of this security. This type
of agreement is called a repurchase agreement or repo.
• The interest rate at which the money is lent in this way is called the repo rate. Similarly, instead of an
outright sale of securities the central bank may sell the securities through an agreement that has a
specification about the date and price at which it will be repurchased. This type of agreement is called a
reverse repurchase agreement or reverse repo. The rate at which the money is withdrawn in this manner is
called the reverse repo rate. The Reserve Bank of India conducts repo and reverse repo operations at
various maturities: overnight, 7-day, 14-day, etc. Hence, statement 2 is correct.

Q 98.A
• Components of Balance of Payments
o There are two main accounts in the BoP – Current Account and Capital Account.
o Balance of Current Account
o It includes the BoT (visibles), and the balance of invisibles (services or transfer payments).
o It is a measure of all payments made for currently produced goods and services plus non-trade flows
of funds between a country and the rest of the world. Hence, statement 1 is correct.
✓ Non-trade flows comprise Factor Income from abroad (interest, profits, wages, etc.)
and international transfer payments. Hence, statement 2 is not correct.
✓ Invisible Balance shows the value of imports and exports of services or invisible items.
• Trade-in services are denoted as invisible trade (because they are not seen to cross national borders).
Services trade includes both factor and non-factor income. Factor income includes net international
earnings on factors of production (like labour, land, and capital). Non-factor income is the net sale of
service products like shipping, banking, tourism, software services, etc.
• Transfer payments are receipts that the residents of a country receive ‘for free’, without having to make
any present or future payments in return. They consist of remittances, gifts and grants. They could be
official or private. The balance of exports and imports of goods is referred to as the trade balance. Adding
trade-in services and net transfers to the trade balance, we get the current account balance. The capital
account records all international purchases and sales of assets such as money, stocks, bonds, etc.

Q 99.C
• The International Monetary Fund (IMF) was created in 1945 with the aim to (i) foster global
monetary cooperation, (ii) secure financial stability, (iii) facilitate international trade, (iv) promote high
employment and (v) sustainable economic growth, and (vi) reduce poverty around the world.
• It was established along with the International Bank for Reconstruction and Development at the
Conference of 44 Nations held at Bretton Woods, New Hampshire, USA in July 1944. Hence,
statement 1 is correct.
• It was created out of a need to prevent economic crises like the Great Depression.
• With its sister organization, the World Bank, the IMF is the largest public lender of funds in the world.
• It is a specialized agency of the United Nations Membership that is open to any country that
conducts foreign policy and accepts the organization's statutes. Hence, statement 2 is correct.
• The IMF membership currently includes 190 countries (189 UN countries and Kosovo) that make up its
near-global membership. India is a founder member of the IMF. Hence, statement 3 is correct.

37 www.visionias.in ©Vision IAS

FREE BY KING R QUEEN P [ऋषभ राजपूत]


Q 100.B
• The second plan was to set India on the path of industrialisation. P.C. Mahalanobis was the moving spirit
behind the second five year plan. He gave the highest priority to strengthening the industrial base of the
economy. There was emphasis on the expansion of the public sector and establishment of a socialistic
pattern of society. Hence option (b) is the correct answer.
• Objectives:
o A sizeable increase in national income so as to raise the level of living.
o Rapid industrialisation of the country with particular emphasis on the development of basic and key
industries.
o A large expansion of employment opportunities by developing labour-intensive projects and small
scale industries.
o Reduction in inequalities of income and distribution.
o To attain the annual growth rate of 5%.

Copyright © by Vision IAS


All rights are reserved. No part of this document may be reproduced, stored in a retrieval system or
transmitted in any form or by any means, electronic, mechanical, photocopying, recording or otherwise,
without prior permission of Vision IAS.
38 www.visionias.in ©Vision IAS

FREE BY KING R QUEEN P [ऋषभ राजपूत]


VISIONIAS
www.visionias.in

Test Booklet Series

TEST BOOKLET

GENERAL STUDIES (P) 2024 – Test – 4133


C
Time Allowed: Two Hours Maximum Marks: 200

INSTRUCTIONS

1. IMMEDIATELY AFTER THE COMMENCEMENT OF THE EXAMINATION, YOU SHOULD CHECK THAT THIS BOOKLET
DOES NOT HAVE ANY UNPRINTED OR TURN OR MISSING PAGES OR ITEMS, ETC. IF SO, GET IT REPLACED BY A
COMPLETE TEST BOOKLET.

2. ENCODE CLEARLY THE TEST BOOKLET SERIES A, B, C OR D AS THE CASE MAY BE IN THE APPROPRIATE PLACE IN
THE ANSWER SHEET.

3. You have to enter your Roll Number on the Test Booklet in the Box
provided alongside. Do NOT write anything else on the Test Booklet.

4. This Test Booklet contains 100 items (Questions). Each item is printed in English. Each item comprises four
responses (answers). You will select the response which you want to mark on the Answer Sheet. In case you
feel that there is more than one correct response with you consider the best. In any case, choose ONLY ONE
response for each item.

5. You have to mark all your responses ONLY on the separate Answer Sheet provided. See direction in the
answers sheet.

6. All items carry equal marks. Attempt all items. Your total marks will depend only on the number of correct
responses marked by you in the answer sheet. For every incorrect response 1/3rdof the allotted marks will be
deducted.

7. Before you proceed to mark in the Answer sheet the response to various items in the Test booklet, you have to
fill in some particulars in the answer sheets as per instruction sent to you with your Admission Certificate.

8. After you have completed filling in all responses on the answer sheet and the examination has concluded, you
should hand over to Invigilator only the answer sheet. You are permitted to take away with you the Test
Booklet.

9. Sheet for rough work are appended in the Test Booklet at the end.

DO NOT OPEN THIS BOOKLET UNTIL YOU ARE ASKED TO DO SO


1 www.visionias.in ©Vision IAS

FREE BY KING R QUEEN P [ऋषभ राजपूत]


1. With reference to the management of the 3. Consider the following statements with

Solid Waste, consider the following respect to the procedure for divorce under
the Hindu Marriage Act, 1955:
statements:
1. It lays down the law for divorce which
1. Composting is the process of degrading
applies to Hindus only.
decomposable solid waste in a properly
2. It is mandatory to give six-month time to
dug pit. the parties time to withdraw their plea.
2. Land filling is the process of dumping 3. A petition for divorce by mutual consent

wastes or other substances in low land can be filed only a year after the
marriage.
areas and covering the same with soil.
Which of the statements given above is
3. In incineration, the dried solid waste is
correct?
safely burnt in closed compartments
(a) 1 and 2 only
with proper provision of treatment of (b) 1 and 3 only
harmful gases. (c) 2 and 3 only
How many of the above statements are (d) 1, 2 and 3

correct?
4. Business Ready (B-Ready) Project which is
(a) Only one
recently seen in news is an initiative of
(b) Only two
(a) World Trade Organisation (WTO)
(c) All three (b) World Bank
(d) None (c) United Nations Conference on Trade and
Development (UNCTAD)

2. Consider the following statements about acid (d) The Organization for Economic
Cooperation and Development (OECD)
rain:

1. It occurs only in tropical areas.


5. Consider the following statements with
2. It is caused by a reaction of oxides of
respect to Kambala:
sulphur and nitrogen with water. 1. It is a traditional buffalo race in paddy
3. One of the reasons for the plaque on the fields filled with slush and mud.

Taj Mahal is the acid rain in the region. 2. It takes place mainly in the Mysore
region of Karnataka.
How many of the above statements are not
Which of the statements given above is/are
correct?
correct?
(a) Only one
(a) 1 only
(b) Only two (b) 2 only
(c) All three (c) Both 1 and 2

(d) None (d) Neither 1 nor 2


2 www.visionias.in ©Vision IAS

FREE BY KING R QUEEN P [ऋषभ राजपूत]


6. Consider the following statements regarding 9. Recently, Article 299 of the Indian
nitrogen pollution: constitution was seen in the news. It is
1. Its contamination causes blue baby’ related to
syndrome.
(a) contracts made in the exercise of the
2. Excessive use of nitrogen fertilizers
executive power of the Union or of a
contributes towards nitrogen pollution.
State.
3. Runoff of nitrogen from fields to water
bodies causes eutrophication. (b) official languages of the Union and

How many of the above statements are states.


correct? (c) seat of Supreme Court in India.
(a) Only one (d) setting up of Union Public Commission.
(b) Only two
(c) All three
10. In the context of global efforts for the
(d) None
protection of the environment, which of the
following happened earliest?
7. Which of the following statements best
describes the term bioremediation? (a) Release of Brundtland Report titled ‘Our

(a) It is an increase in concentration of a Common Future’


substance within a trophic level in (b) United Nations Conference on
certain tissues of organisms' bodies. Environment and Development
(b) It is the application of biotic agents like
(c) United Nations Conference on Human
microorganisms in the correction and
Environment
recovery of environmental damage.
(d) Adoption of Kyoto Protocol
(c) It is an increase in concentration of a
substance in the bodies of consumers as
one moves up the food chain. 11. Consider the following statements:

(d) It is the practice of commercial 1. Anaerobic decomposition occurs in the


exploitation of biochemicals or genetic absence of oxygen and a lot of energy is
materials which occur naturally. produced in this process.

2. Anaerobic decomposition kills seeds of


8. Solid Waste Management Rules 2016 and E-
weeds and germs present in the waste.
Waste (Management) Rule 2016 were
Which of the statements given above is/are
notified under which of the following Acts?
(a) Air (Prevention and Control of correct?

Pollution) Act, 1981 (a) 1 only


(b) Environment (Protection) Act, 1986 (b) 2 only
(c) Biological Diversity Act, 2002 (c) Both 1 and 2
(d) Wild Life (Protection) Act, 1972 (d) Neither 1 nor 2
3 www.visionias.in ©Vision IAS

FREE BY KING R QUEEN P [ऋषभ राजपूत]


12. Consider the following statements with 15. Which of the following harmful materials
respect to the National Automated is/are found in e-waste?
Fingerprint Identification System (NAFIS): 1. Lead
2. Cadmium
1. It is a web-based application that serves
3. Chromium
as a central repository of criminal
Select the correct answer using the code
fingerprints.
given below.
2. It assigns a unique 10-digit National (a) 1 only
Fingerprint Number to each person (b) 1 and 2 only
arrested for a crime. (c) 1, 2 and 3
3. It is managed by the Crime and Criminal (d) 2 and 3 only

Tracking Network and Systems.


16. Consider the following statements with
How many of the above statements are
respect to the eutrophication of lakes:
correct?
1. It is a manmade process and does not
(a) Only one occur naturally.
(b) Only two 2. The prime contaminants for
(c) All three eutrophication are nitrates and
(d) None phosphates.
Which of the statements given above is/are
correct?
13. The species dodo, quagga, thylacine and
(a) 1 only
Steller’s Sea Cow are
(b) 2 only
(a) invasive species
(c) Both 1 and 2
(b) endemic to Africa (d) Neither 1 nor 2
(c) marine mammals
(d) extinct species 17. Consider the following statements:
1. As compared to wildlife sanctuaries
14. Arrange the following biodiversity national parks are given a greater degree
of protection, with human activity
conservation efforts in India in chronological
greatly restricted.
order from earliest to latest.
2. Wildlife sanctuaries do not have fixed
1. Project Elephant
boundaries whereas the boundaries of
2. Project Snow Leopard National Parks are well marked.
3. Project Hangul 3. Unlike wildlife sanctuaries, national
4. The Asiatic Lion Reintroduction Project parks can be notified only by the Central
Select the correct answer using the code Government.

given below. How many of the above statements are


correct?
(a) 1-4-3-2
(a) Only one
(b) 3-1-2-4
(b) Only two
(c) 3-1-4-2 (c) All three
(d) 1-4-2-3 (d) None
4 www.visionias.in ©Vision IAS

FREE BY KING R QUEEN P [ऋषभ राजपूत]


18. Consider the following statements: 20. With reference to ecological succession,
Statement-I: Termites are counted among consider the following statements:
natural sources of methane like wetlands, 1. Ecological succession is defined as the
wild animals, livestock and geological
process in which one generation of a
features like volcanoes.
species replaces another without
Statement-II: Methanogenic microorganisms
allowing other species to grow.
that live in the gut of termites break down
the cellulose entering the body and release 2. Primary succession takes place in areas
methane. that are newly exposed and essentially
Which one of the following is correct in lifeless.
respect of the above statements? Which of the statements given above is/are
(a) Both Statement-I and Statement-II are
correct?
correct and Statement-II is the correct
(a) 1 only
explanation for Statement-I
(b) 2 only
(b) Statement-I and Statement-II are correct
and Statement-II is not the correct (c) Both 1 and 2
explanation for Statement-I (d) Neither 1 nor 2
(c) Statement-I is correct but Statement-II is
incorrect 21. Consider the following statements regarding
(d) Statement-I is incorrect but Statement-II
First Loss Default Guarantee (FLDG), which
is correct
is recently seen in the news:
1. It is contractual arrangement between
19. Consider the following statements regarding
ionizing radiations: banks, non-banking finance companies

1. They include long wavelength and lending service providers (LSP)


ultraviolet radiations (UV), X-rays, and whereby the LSPs guarantees to
gamma rays. compensate the banks.
2. Electrically charged particles produced
2. Losses only to a certain threshold are
in nuclear processes can knock electrons
covered under this arrangement.
out of atoms or molecules, producing
3. The LSP-providing default-loss-
ions.
3. Gamma rays passing through a cell can guarantee must be incorporated as a

ionize water molecules near the DNA, company under the Companies Act,
potentially causing DNA to break and 2013.
damage living tissues. Which of the statements given above are
How many of the above statements are not correct?
correct?
(a) 1 and 2 only
(a) Only one
(b) 2 and 3 only
(b) Only two
(c) All three (c) 1 and 3 only

(d) None (d) 1, 2 and 3


5 www.visionias.in ©Vision IAS

FREE BY KING R QUEEN P [ऋषभ राजपूत]


22. Consider the following statements: 25. A zone of gradual but continuous change
1. Quantum physics is the study of matter from one ecosystem to another when there is
and energy at the most fundamental
no sharp boundary between the two in terms
level and it aims to uncover the
properties and behaviors of the very of species composition, is best described as
building blocks of nature. a/an
2. Quantum effects only manifest at very
(a) habitat
small length and mass scales, or when
temperatures approach absolute zero. (b) niche
Which of the statements given above is/are (c) ecocline
correct?
(d) biome
(a) 1 only
(b) 2 only
(c) Both 1 and 2 26. Which of the following criteria are used to
(d) Neither 1 nor 2 identify biodiversity hotspots?
1. Species richness
23. Consider the following statements:
Statement-I: Most Herbicides are toxic to 2. Presence of invasive species
mammals. 3. High endemism
Statement-II: Herbicides become
4. Threat perception
concentrated in the food web.
Which one of the following is correct in Select the correct answer using the code
respect of the above statements? given below.
(a) Both Statement-I and Statement-II are
(a) 1 and 2 only
correct and Statement-II is the correct
explanation for Statement-I. (b) 2 and 4 only
(b) Both Statement-I and Statement-II are (c) 1, 3 and 4 only
correct and Statement-II is not the (d) 1, 2, 3 and 4
correct explanation for Statement-I.
(c) Statement-I is correct but Statement-II is
incorrect. 27. Consider the following statements about
(d) Statement-I is incorrect but Statement-II Ozone:
is correct.
1. It can only be formed naturally in the

24. Which of the following are the possible atmosphere.


impacts of ozone layer depletion? 2. In the stratosphere, it forms a shield
1. Delay in plant formation and timing of
protecting us from the UV radiation of
development
2. Increase in tropospheric ozone the sun.
3. Relative cooling of the troposphere 3. Ozone is not found in the troposphere.
4. Increased incidence of cataracts in How many of the above statements are
humans
Select the correct answer using the code correct?
given below. (a) Only one
(a) 1 and 2 only (b) Only two
(b) 1, 3 and 4 only
(c) All three
(c) 3 and 4 only
(d) 1, 2, 3 and 4 (d) None
6 www.visionias.in ©Vision IAS

FREE BY KING R QUEEN P [ऋषभ राजपूत]


28. The Global Green House Gas Watch, which 31. Consider the following statements:
is recently seen in news is an initiative of 1. Species evenness describes the
(a) World Metrological Organisation distribution of abundance across the
(WMO) species in a community.
(b) National Oceanic and Atmospheric 2. Species richness is the number of

Administration (NOAA) species within a defined region.


Which of the statements given above is/are
(c) United Nations Environment Programme
correct?
(UNEP)
(a) 1 only
(d) GermanWatch
(b) 2 only
(c) Both 1 and 2
29. Consider the following statements:
(d) Neither 1 nor 2
Statement I: Very small animals are rarely
found in the polar regions.
32. Consider the following pairs:
Statement II: Small animals have a small National Park State
surface area relative to their volume. 1. Kaziranga National : Arunachal
Which one of the following is correct in Park Pradesh
respect of the above statements? 2. Ranthambore National : Rajasthan
(a) Both Statement-I and Statement-II are Park
correct and Statement-II is the correct 3. Kanha National Park : Madhya
explanation for Statement-I Pradesh
(b) Both Statement-I and Statement-II are 4. Simlipal National Park : West Bengal
correct and Statement-II is not the How many pairs given above are correctly
correct explanation for Statement-I matched?

(c) Statement-I is correct but Statement-II is (a) Only one pair


(b) Only two pairs
incorrect
(c) Only three pairs
(d) Statement-I is incorrect but Statement-II
(d) All four pairs
is correct

33. Consider the following processes:


30. The term ‘forum shopping’ recently seen in
1. Agricultural runoff
the news is the
2. Open Defecation
(a) practice of choosing the court which is 3. Discharge of Industrial Effluents
likely to provide the most favorable 4. Floods
outcome. 5. Discharge of coolant from Nuclear
(b) common digital marketplace platform reactors
for day-to-day grocery shopping. Which of the above given processes are
(c) university forums in which students can responsible for water pollution?
post multiple topics and replies. (a) 1, 3 and 4 only
(d) consumer forums in which the case may (b) 2 and 5 only
most be tried for the interests of all the (c) 2, 3 and 4 only
parties. (d) 1, 2, 3, 4 and 5
7 www.visionias.in ©Vision IAS

FREE BY KING R QUEEN P [ऋषभ राजपूत]


34. Which of the following statements best 37. 'The Evil Quartet' is the sobriquet used to
describes the term “Biotic potential”? describe the major causes that have led to the
(a) It is the maximum rate at which a
rapid extinction of species across the world.
population can increase when resources
In this context which of the following are the
are unlimited and environmental
conditions are ideal. major causes of loss of biodiversity?

(b) It is the maximum amount of fertilizer 1. Over-exploitation


that can be used without increasing the 2. Habitat loss and fragmentation
pH of soil. 3. Alien species invasions
(c) It is the maximum rate of respiration of
4. Co-extinctions
species in the tropics.
Select the correct answer using the code
(d) It is the maximum time taken by a plant
species to grow under a polluted given below.

environment. (a) 1, 2 and 3 only


(b) 1, 2 and 4 only
35. Consider the following statements:
(c) 3 and 4 only
1. Biological Oxygen Demand (BOD) is an
(d) 1, 2, 3 and 4
indicator of eutrophication in water
bodies.
2. A rise in BOD means there is a presence 38. Consider the following statements:
of higher inorganic pollution in the Statement-I: Generally, species diversity
water body. decreases as we move away from the equator
Which of the statements given above is/are
towards the poles.
correct?
Statement-II: Unlike temperate
(a) 1 only
(b) 2 only environments, tropical ones are more

(c) Both 1 and 2 seasonal and relatively unpredictable.


(d) Neither 1 nor 2 Which one of the following is correct in
respect of the above statements?
36. With reference to the earth’s biodiversity,
(a) Both Statement-I and Statement-II are
consider the following statements:
correct and Statement-II is the correct
1. More than 70 per cent of all the species
recorded are plants. explanation for Statement-I
2. Among animals, insects are the most (b) Both Statement-I and Statement-II are
species-rich taxonomic group. correct and Statement-II is not the
Which of the statements given above is/are
correct explanation for Statement-I
correct?
(c) Statement-I is correct but statement-II is
(a) 1 only
(b) 2 only incorrect

(c) Both 1 and 2 (d) Statement-I is incorrect but Statement-II


(d) Neither 1 nor 2 is correct
8 www.visionias.in ©Vision IAS

FREE BY KING R QUEEN P [ऋषभ राजपूत]


39. Consider the following statements about the 42. Consider the following pairs:
Rehabilitation Council of India:
Drying Lakes State
1. It is a statutory body under the Ministry
of Social Justice and Empowerment. 1. Mettur : Tamil Nadu
2. It prescribes minimum standards of
education and training for professionals. 2. Krishna Raja Sagar : Karnataka
Which of the statements given above is/are
3. Nagarjuna Sagar : Maharashtra
correct?
(a) 1 only 4. Idamalayar : Kerala
(b) 2 only
(c) Both 1 and 2 How many pairs given above are correctly
(d) Neither 1 nor 2 matched?

40. With regards to the Great Indian Bustard, (a) Only one pair
consider the following statements: (b) Only two pairs
1. It is endemic to the Indian Sub-
continent. (c) Only three pairs
2. It is one of the species identified for the
(d) None of the pairs
Species Recovery Programme under the
Integrated Development of Wildlife
Habitats of the Ministry of Environment
43. With reference to the Environment Impact
and Forests.
3. It is the state bird of Rajasthan. Assessment, consider the following
How many of the statements given above are
statements:
correct?
(a) Only one 1. Environmental Clearance (EC) has been
(b) Only two
made mandatory for modernisation of
(c) All three
(d) None any activity or for setting up new

41. Consider the following statements regarding projects under Environmental


the Sustainable Development Goals (SDGs): (Protection) Act 1986.
1. The SDGs are 17 sets of goals adopted
under the Rio Declaration. 2. It provides an exemption from the EIA
2. The Ministry of Environment, Forest
requirements to certain industries based
and Climate Change coordinates India’s
SDG implementation. on the investment value of their projects.
3. The SDG India Index is released by the
Which of the statements given above is/are
NITI Aayog in association with
International Monetary Fund. correct?
Which of the statements given above is/are
(a) 1 only
not correct?
(a) 1 and 2 only (b) 2 only
(b) 2 only
(c) Both 1 and 2
(c) 1 and 3 only
(d) 1, 2 and 3 (d) Neither 1 nor 2
9 www.visionias.in ©Vision IAS

FREE BY KING R QUEEN P [ऋषभ राजपूत]


44. Consider the following statements: 47. It is one of the biogeographic zones of India.
Statement-I: Carbon dating cannot be used to The tropical evergreen forests occupy about
determine the age of non-living things like one-third of the total area of this zone. About
rocks. two-thirds of India’s endemic plants are
Statement-II: Carbon dating is based on the confined to this region. Grizzled Squirrel is
radioactive decay of the Carbon-12 isotope restricted to a few forests in this zone.
of Carbon. Which of the following biogeographic zones
Which one of the following is correct in of India is described in the above passage?
respect of the above statements?
(a) Gangetic plains
(a) Both Statement-I and Statement-II are
(b) North-East India
correct and Statement-II is the correct
(c) Islands
explanation for Statement-I
(d) Western Ghats
(b) Both Statement-I and Statement-II are
correct and Statement-II is not the
48. Article 355 empowers the Union
correct explanation for Statement-I
government:
(c) Statement-I is correct but Statement-II is
1. To protect every state in India against
incorrect
internal disturbances.
(d) Statement-I is incorrect but Statement-II
is correct 2. To issue directions to any state to ensure
compliance with the Union’s laws and
45. “Endosulfan” considered to be a carcinogen, regulations.
neurotoxin and genotoxin, is most likely 3. To impose President’s Rule in a state in
used in which of the following? case of a failure or breakdown of
(a) pesticides in agriculture constitutional machinery.
(b) reused plastic containers How many of the above statements are
(c) fruit ripening substances correct?
(d) food preservatives (a) Only one
(b) Only two
46. With reference to the decomposition of (c) All three
organic matter in an ecosystem, consider the (d) None
following statements:
1. Anaerobic conditions inhibit the process 49. Consider the following pairs:
of decomposition of organic matter.
Pollutant Source
2. Decomposition involves a series of
1. Ozone : Vehicular exhaust
processes including fragmentation,
2. Plutonium : Nuclear reactor
leaching, catabolism, humification, and
mineralization 3. Mercury : Dental industry
Which of the statements given above is/are How many of the pairs given above are
correct? correctly matched?
(a) 1 only (a) Only one pair
(b) 2 only (b) Only two pairs
(c) Both 1 and 2 (c) Only three pairs
(d) Neither 1 nor 2 (d) All four pairs
10 www.visionias.in ©Vision IAS

FREE BY KING R QUEEN P [ऋषभ राजपूत]


50. The term "Green Shoe Option" recently seen 53. Consider the following statements with
in the news is associated with which of the respect to National Adaptation Fund For
following? Climate Change (NAFCC):
(a) Initial Public Offer in share market 1. It was launched in 1992 as a result of the
(b) Climate change agreements curbing earth summit negotiations.
greenhouse gas emissions 2. The National Bank for Agriculture and
(c) Construction of green highways Rural Development (NABARD) is the
(d) Giving boost to marine economy National Implementing Entity (NIE).
Which of the statements given above is/are
51. Which of the following statements best correct?
describes the term 'bioprospecting'? (a) 1 only
(a) It is the exploration of plant and animal (b) 2 only
species for the utilization of their genetic (c) Both 1 and 2
resources in the production of (d) Neither 1 nor 2
commercial products.
(b) It refers to unauthorized appropriation of 54. Gomphonema rajaguruii, which is recently
biological materials. seen in news is a type of
(c) It is an ex situ treatment technology that (a) Algae
leverages biological processes to convert (b) Fish
contaminants to low-toxicity byproducts. (c) Turtle
(d) It is a remediation technique that uses (d) Butterfly
indigenous microorganisms to stimulate
in-situ aerobic biological activity. 55. Consider the following statements regarding
the Polar Stratospheric Clouds (PSC):
52. Consider the following pollutants: 1. PSCs are readily formed in the extreme
1. Oxides of sulfur cold winters of the Antarctic
2. Particulate Matter atmosphere.
3. Oxides of nitrogen 2. Ozone is destroyed in Antarctica at the
4. Carbon monoxide maximum rate during the winters due to
5. Carbon dioxide the abundance of PSCs.
Which of the above given pollutants are Which of the statements given above is/are
released during the production of steel? correct?
(a) 1, 3 and 5 only (a) 1 only
(b) 1, 4 and 5 only (b) 2 only
(c) 2, 3, 4 and 5 only (c) Both 1 and 2
(d) 1, 2, 3, 4 and 5 (d) Neither 1 nor 2
11 www.visionias.in ©Vision IAS

FREE BY KING R QUEEN P [ऋषभ राजपूत]


56. With reference to food chains, consider the 59. Consider the following statements in relation
following statements: to Pradhan Mantri Jan Vikas Karyakram
1. All food chains begin with autotrophs.
(PMJVK):
2. In an aquatic ecosystem, Detritus Food
Chain (DFC) is the major conduit for 1. It is a central sector scheme.
energy flow as Grazing Food Chain 2. Its objective is to enhance the basic
(GFC) is totally absent.
infrastructure facilities and socio-
3. Saprotrophs form an important part of
Detritus Food Chains. economic amenities for the minority

Which of the statements given above is/are communities.


correct? 3. Projects under it are implemented and
(a) 1 and 3 only
managed by the concerned State/ Union
(b) 2 and 3 only
(c) 3 only Territory government.
(d) 1 and 2 only How many of the statements given above are

correct?
57. Consider the following compounds:
(a) Only one
1. Chlorofluorocarbons
2. Hydrochlorofluorocarbons (b) Only two
3. Methyl Bromide (c) Only three
4. Hydrofluorocarbons
(d) None
How many compounds given above are
ozone-depleting?
(a) Only one 60. Consider the following statements about the
(b) Only two
Central Pollution Control Board (CPCB):
(c) Only three
1. It is a statutory organization under the
(d) All four
Ministry of Environment Forest and
58. Which of the following are the methods of Climate Change.
carbon sequestration?
2. It was formed under the Air (Prevention
1. Iron fertilization of oceans
2. Subterranean injection of carbon dioxide and Control Pollution) Act 1981.
3. Growing seaweeds Which of the statements given above is/are
4. Wetland restoration
correct?
Select the correct answer using the code
(a) 1 only
given below.
(a) 1 and 2 only (b) 2 only
(b) 1, 2 and 3 only (c) Both 1 and 2
(c) 3 and 4 only
(d) Neither 1 nor 2
(d) 1, 2, 3 and 4
12 www.visionias.in ©Vision IAS

FREE BY KING R QUEEN P [ऋषभ राजपूत]


61. Consider the following statements: 63. With regard to the functioning of the
National Green Tribunal, consider the
Statement-I: Debt for Climate Swaps is an
following statements:
agreement where a creditor forgoes a portion 1. It has been established under the Wild
Life (Protection) Act, 1972 for effective
of the debtor's foreign debt or provides debt
and expeditious disposal of cases
relief in exchange for the debtor's relating to environmental protection.
2. The tribunal is not bound by the
commitment to invest in a specific
procedure laid down under the Code of
environmental project. Civil Procedure, 1908.
Which of the statements given above is/are
Statement-II: Many developing nations are
correct?
facing issues of rising debt loads, climate (a) 1 only
change, and nature loss due to such (b) 2 only
(c) Both 1 and 2
agreements. (d) Neither 1 nor 2
Which one of the following is correct in
64. Consider the following places:
respect of the above statements? 1. Three Mile Island
(a) Both Statement-I and Statement-II are 2. Chernobyl
3. Fukushima
correct and Statement-II is the correct 4. Deepwater Horizon (Gulf of Mexico)
explanation for Statement-I How many of the above given places are
associated with nuclear accidents?
(b) Statement-I and Statement-II are correct (a) Only one
and Statement-II is not the correct (b) Only two
(c) Only three
explanation for Statement-I
(d) All four
(c) Statement-I is correct but Statement-II is
65. Consider the following statements:
incorrect
Statement-I: Ocean acidification is
(d) Statement-I is incorrect but Statement-II detrimental to coral reefs.
Statement-II: An increase in acidification
is correct
slows the rate at which calcium carbonate is
generated.
Which one of the following is correct in
62. The Innovative Finance Facility for Climate
respect of the above statements?
in Asia and the Pacific (IF-CAP) which is (a) Both Statement-I and Statement-II are
correct and Statement-I is the correct
recently seen in news, is an initiative of
explanation for Statement-II
(a) Asian Development Bank (ADB) (b) Both Statement-I and Statement-II are
correct and Statement-I is not the correct
(b) Economic and Social Commission for
explanation for Statement-II
Asia and the Pacific (ESCAP) (c) Statement-I is correct but Statement-II is
incorrect
(c) Global Environment Facility (GEF)
(d) Statement-I is incorrect but Statement-II
(d) New Industrial development bank is correct
13 www.visionias.in ©Vision IAS

FREE BY KING R QUEEN P [ऋषभ राजपूत]


66. Consider the following statements in the 69. Consider the following pairs with reference
context of the Mahila Samman Savings to Sustainable Development Goals (SDGs):
Certificate: SDG Number Goal
1. Any woman can open a Mahila Samman 1. SDG 3 : Gender Equality
account that can be opened for herself or 2. SDG 9 : Access to Clean Energy
on behalf of a minor girl. 3. SDG 13 : Sustainable Cities and
2. The minimum investment amount is Rs Communities
5,000 and the maximum investment is 4. SDG 16 : Peace Justice and Strong
Rs 10 lakhs. Institutions
3. There is tax exemption available on the How many pairs given above are correctly
interest earned from the scheme. matched?
4. The account holder may withdraw up to (a) Only one Pair
40% of the balance after the first year (b) Only two Pairs
from the date of account opening. (c) Only three Pairs
How many of the statements given above are (d) All four Pairs
correct?
(a) Only one 70. Consider the following statements about the
(b) Only two Biodiversity Heritage Site:
(c) Only three 1. The Central Government in consultation
(d) All four with local bodies may notify the official
gazette, of areas of biodiversity
67. Which of the following mechanisms have importance as Biodiversity Heritage
been formulated under the Kyoto Protocol?
Sites (BHS) under the Biological
1. Clean Development Mechanism
Diversity Act of 2002.
2. Joint Implementation
2. NallurTamarind Grove in Bengaluru,
3. Intended Nationally Determined
Karnataka, was designated as India's
Contributions
first Biodiversity Heritage Site.
4. International Emission Trading
Which of the statements given above is/are
Select the correct answer using the code
correct?
given below.
(a) 1 only
(a) 1 and 2 only
(b) 2 only
(b) 1, 2, 3 and 4
(c) Both 1 and 2
(c) 1, 2 and 4 only
(d) Neither 1 nor 2
(d) 3 and 4 only

71. Consider the following types of ecological


68. Consider the following factors:
pyramids:
1. Natality
1. Pyramids of number
2. Mortality
2. Pyramid of biomass
3. Immigration
3. Pyramids of energy
How many of the above factors contribute to
How many of the pyramids given above can
an increase in the population density of a
never be inverted in shape?
species in a region?
(a) Only one (a) Only one
(b) Only two (b) Only two
(c) All three (c) All three
(d) None (d) None
14 www.visionias.in ©Vision IAS

FREE BY KING R QUEEN P [ऋषभ राजपूत]


72. Consider the following nutrient cycles: 75. Consider the following statements regarding
1. Nitrogen cycle Trade Records Analysis of Flora and Fauna
in International Commerce (TRAFFIC):
2. Sulfur cycle
1. It is a wildlife trade monitoring network
3. Carbon cycle established jointly by the World Wide
4. Phosphorus cycle Fund for Nature (WWF) and the
How many of the above are gaseous nutrient International Union for Conservation of
Nature (IUCN).
cycles?
2. It is concerned with the monitoring of
(a) Only one both the legal as well as illegal trade in
(b) Only two wildlife across the world.
Which of the statements given above is/are
(c) Only three
correct?
(d) All four
(a) 1 only
(b) 2 only
73. Consider the following statements: (c) Both 1 and 2
(d) Neither 1 nor 2
1. The Earth is currently believed to be
undergoing the sixth mass extinction. 76. The term 'Goldilocks economy' was recently
2. The current species extinction rates are seen in the news. It best describes which of
estimated to be several times faster than the following scenarios in the economy?
(a) It is a highly inflationary state, wherein
in pre-human times.
the prices of commodities rises up
Which of the statements given above is/are exponentially.
correct? (b) It is a state wherein, the economy is not
(a) 1 only expanding or contracting by a large
margin.
(b) 2 only
(c) It is a state of economy where there is
(c) Both 1 and 2 growth and employment opportunities.
(d) Neither 1 nor 2 (d) It is a state of economy wherein there is
high foreign investment pull-out.

74. Consider the following:


77. Which of the following statements best
1. Carbon dioxide describes the Chemical Oxygen Demand of a
2. Hydrofluorocarbons water body?
(a) It is the amount of oxygen
3. Methane
microorganisms required to break down
4. Nitrous oxide
organic materials by the microorganism.
Which of the gases given above are (b) It is the amount of oxygen required to
greenhouse gases? break down the organic and inorganic
material via oxidation.
(a) 1, 2, 3 and 4
(c) It is a measure of dissolved oxygen in
(b) 1, 3 and 4 only the water.
(c) 1, 2 and 3 only (d) It is the measure of the turbidity of the
(d) 2 and 4 only water.
15 www.visionias.in ©Vision IAS

FREE BY KING R QUEEN P [ऋषभ राजपूत]


78. Consider the following statements with 81. Which of the following statements best
respect to wildlife sanctuaries: describes the term ‘spring shock’?
1. It is a protected area where rare and
(a) It is the sudden improvement in
endangered species are encouraged to
breed in human-controlled environments vegetation in polar regions in summer
with restricted settings. due to abnormal increase in
2. Human activities like harvesting or
photosynthesis.
timber collection of minor forest
products and private ownership rights (b) It is the increase in daily average
are allowed in these sanctuaries. temperature due to El nino.
Which of the statements given above is/are
(c) It is the sudden growth of algae in the
correct?
(a) 1 only northern Pacific ocean in the spring
(b) 2 only season.
(c) Both 1 and 2 (d) It is the increase in acidification due to
(d) Neither 1 nor 2
melting of snow and ice over lakes.
79. Consider the following pairs:
Type of Measure of 82. Which of the following regulatory bodies in
Biodiversity
India is responsible for formulating
1. Alpha : Species richness in a
diversity particular ecosystem standards and guidelines to control noise
2. Beta : Total biodiversity within pollution?
diversity an entire region
(a) Centre for Environment Education
3. Gamma : Change in amount of
diversity species between the (b) Pollution Control Committees of
ecosystems. respective states
How many of the above pairs are correctly (c) Central Pollution Control Board (CPCB)
matched?
(d) National Green Tribunal (NGT)
(a) Only one
(b) Only two
(c) All three 83. Consider the following statements about
(d) None
photochemical smog:

80. With reference to the radioactive pollution, 1. It is an oxidizing smog.


consider the following statements: 2. It is made up of secondary pollutants
1. Radioactive wastes can be in gaseous,
like ozone.
liquid, or solid form and can vary in
their level of radioactivity. 3. It can be formed even in the absence of
2. Radioactive wastes can remain sunlight.
radioactive for a few hours to hundreds Which of the statements given above is/are
of thousands of years.
Which of the statements given above is/are correct?
correct? (a) 1 only
(a) 1 only (b) 2 only
(b) 2 only
(c) 1 and 2 only
(c) Both 1 and 2
(d) Neither 1 nor 2 (d) 2 and 3 only
16 www.visionias.in ©Vision IAS

FREE BY KING R QUEEN P [ऋषभ राजपूत]


84. Consider the following pairs: 87. Which of the following constitute the
Bilateral Exercise Participating principles of radioactive waste management
country with followed in India?
India 1. Protection of Human Health and
1. Dharma Guardian : Japan Environment
2. Samudra Shakti : France 2. Concern for Future Generations
3. Al Mohed Al Hindi : Yemen 3. Establishing Legal Framework
How many of the above pairs are correctly 4. Waste Minimisation, Management
matched? Interdependency and Safety of Facilities
(a) Only one Select the correct answer using the code
(b) Only two given below.
(c) All three (a) 1 and 2 only
(d) None (b) 2, 3 and 4 only
(c) 1 and 3 only
85. With reference to Dhruv advanced light (d) 1, 2, 3 and 4
helicopter, consider the following
statements:
88. Consider the following trees:
1. It is designed and developed
1. Bamboo (Bambusa vulgaris)
indigenously by Defence Research and
2. Mulberry (Morus alba)
Development Organisation (DRDO).
3. Ebony (Diospyros ebenum)
2. It is capable of operating in all-weather
How many of the above are deciduous trees?
conditions.
(a) Only one
3. It is certified for both civil and military
roles. (b) Only two
How many of the above statements are (c) Only three
correct? (d) None
(a) Only one
(b) Only two 89. In the context of ecological conservation
(c) All three Davrai, Devgudi and Devrahati are:
(d) None (a) Sacred groves
(b) Water harvesting methods
86. In the context of sustainable development, (c) Methods of organic farming
consider the following statements regarding (d) Reservoir of medicinal plants
the Malthusian theory of population:
1. It states that while the human population 90. Consider the following biodiversity
grows in a geometric progression, conservation sites:
subsistence can only grow in an 1. Sacred groves
arithmetic progression. 2. Botanical gardens
2. The theory was proposed for the first 3. Zoological parks
time in the Limits to Growth report by
4. Wildlife safari parks
the Club of Rome in 1972.
How many of the above are examples of ex-
Which of the statements given above is/are
situ conservation approaches?
correct?
(a) Only one
(a) 1 only
(b) Only two
(b) 2 only
(c) Only three
(c) Both 1 and 2
(d) Neither 1 nor 2 (d) All four
17 www.visionias.in ©Vision IAS

FREE BY KING R QUEEN P [ऋषभ राजपूत]


91. Which of the following are major sources of 95. Consider the following statements about the
soil pollutants in India? Integration of Sikkim:
1. Agricultural waste 1. Sikkim was ruled by the Namgyal
2. Fertilisers and pesticides dynasty before it become a part of India.
3. Industrial waste 2. A plebiscite on abolishing the monarchy
4. Electronic waste was held in Sikkim in 1975.
Select the correct answer using the code 3. The 36th Constitutional Amendment Act
given below. was passed in Parliament to recognize
(a) 1 and 2 only Sikkim as a state.
(b) 2 and 4 only How many of the above statements are
(c) 3 and 4 only correct?
(d) 1, 2, 3 and 4 (a) Only one
(b) Only two
92. 'Gum Arabic' recently seen in the news for (c) All three
its use in the fizzy drinks and candy industry (d) None
is derived from
(a) Baobab tree
96. With reference to productivity in the
(b) Desert Date tree
ecosystem, consider the following
(c) Acacia tree
statements:
(d) Almond tree
1. The gross primary productivity of an
ecosystem is the rate of production of
93. Which of the following tribes are involved in
organic matter during photosynthesis.
the conservation of tigers in Nagarjunasagar
2. The secondary productivity is the
Srisailam Tiger Reserve (NSTR)?
available biomass for consumption to
(a) Maldhari
herbivores and decomposers.
(b) Bugun
Which of the statements given above is/are
(c) Chenchu
correct?
(d) Gond
(a) 1 only
(b) 2 only
94. In how many of the following examples of
(c) Both 1 and 2
interactions among the species, one
(d) Neither 1 nor 2
interacting species is definitely harmed?
1. Cuscuta growing on hedge plants
2. Cattle egret and grazing cattle in close 97. Consider the following organisms:

association 1. Escherichia coli

3. Interaction between clown fish and sea 2. Enterococcus faecalis

anemone 3. Salmonella typhi


Select the correct answer using the code Which of the organisms given above are
given below. present in contaminated water?
(a) Only one (a) 1, 2 and 3
(b) Only two (b) 1 and 2 only
(c) All three (c) 1 and 3 only
(d) None (d) 2 and 3 only
18 www.visionias.in ©Vision IAS

FREE BY KING R QUEEN P [ऋषभ राजपूत]


98. Which of the following statements is correct 100. Consider the following statements regarding
with reference to ‘Agenda 21’? biosphere reserves:
(a) It is a UN resolution for the protection of 1. Each biosphere reserve is traditionally
21 pacific island states from sea-level organized into five interrelated zones.
rise. 2. Human interference is banned
(b) It is a global effort by G20 countries and completely in the transition zone.
the UN to contribute towards carbon 3. Biosphere reserves are helpful in
sequestration. promoting scientific research and
(c) It is a blueprint developed in 1992 for a education.
set of actions to be taken to achieve Which of the statements given above is/are
sustainability in the 21st century. correct?
(d) It is a list of 21 banned Persistent (a) 1 and 2 only
Organic Pollutants under the Stockholm (b) 2 and 3 only
Declaration. (c) 3 only
(d) 1 and 3 only
99. With reference to the rules regarding
'recusal', consider the following statements:
1. It is the act of abstaining from
participation in an official action due to
a conflict of interest of the judges.
2. Judges (Protection) Act, 1985 has
provisions regarding the practice of
judicial recusals.
Which of the statements given above is/are
correct?
(a) 1 only
(b) 2 only
(c) Both 1 and 2
(d) Neither 1 nor 2

Copyright © by Vision IAS


All rights are reserved. No part of this document may be reproduced, stored in a retrieval system or
transmitted in any form or by any means, electronic, mechanical, photocopying, recording or otherwise,
without prior permission of Vision IAS.
19 www.visionias.in ©Vision IAS

FREE BY KING R QUEEN P [ऋषभ राजपूत]


VISIONIAS
www.visionias.in
ANSWERS & EXPLANATIONS
GENERAL STUDIES (P) TEST – 4133 (2024)

Q 1.C
• Solid wastes should be properly disposed by composting, land filling, thermal processes; and by re-
cycling and re- use.
• Composting is the process of degrading decomposable solid waste in a properly dug pit, called as
compost pit. Composting process involves both the aerobic and anaerobic processes. Hence, statement 1
is correct.
o In aerobic composting the biodegradable wastes are filled in the compost pit which is left open from
one or more sides.
o On the other hand, in anaerobic composting the biodegradable wastes are allowed to degrade in
sealed compost pits. The compost prepared through any one of these processes is very good manure
for crops.
• Land filling is just the process of dumping wastes or other substances in low land areas and covering the
same with soil. This is the age old practice of leveling low areas. But, in modern times waste materials are
buried under soil after compacting them properly in a scientific manner. The provision of release of gases
from the decomposing materials of the land fill is also made adequately. Hence, statement 2 is correct.
• In thermal process, the dried solid waste is safely burnt in closed compartments with proper provision of
treatment of harmful gases, and utilization of heat energy evolved during the process. It is called as
incineration. Hence, statement 3 is correct.
• Environmentalists do not favour this process of solid waste treatment in view of fear of air pollution,
rather they advocate for re-cycling and re-use for the disposal of non- biodegradable substances.

Q 2.A
• The term “acid rain” refers to the precipitation of acid in the form of rain. It usually occurs when
atmospheric pollutants such as oxides of nitrogen and sulphur, interact with rainwater and fall as a
result. Hence statement 2 is correct.
• These pollutants reach the atmosphere from vehicular emissions, industrial outlets, etc.
• The Taj Mahal's white marble deteriorates due to this acid rain. Since acid rain also includes
suspended materials such as ash, soot, and gases. SPM, or suspended particulate matter, these adhere to
the monument's surface, turning it pale and yellow (plaque). Hence statement 3 is correct.
• Acid rains can occur anywhere with "rain and pollution". These are not necessarily limited to tropical
areas. Hence statement 1 is not correct.

Q 3.C
• Recent Context: A five-judge Constitution Bench of the Supreme Court held that it can directly grant a
divorce to couples on irretrievable breakdown of marriage under Article 142 of the Constitution.
• What is the current procedure for divorce under the Hindu Marriage Act?
o The Hindu Marriage Act, of 1955 lays down the law for divorce which applies to Hindus, Buddhists,
Jains, and Sikhs. Hence, statement 1 is not correct.
o Divorce by mutual consent is provided under Section 13B of the Act.
o Filing: Both parties to the marriage must together file a petition to the district court on the ground that
they have been living separately for a period of one year or more and have mutually agreed on a
divorce.
o The parties must move a second motion before the court between 6-18 months after the date of the
presentation of the first petition.

1 www.visionias.in ©Vision IAS

FREE BY KING R QUEEN P [ऋषभ राजपूत]


o Six-month period: The mandatory six-month wait is intended to give the parties time to withdraw
their plea. Hence, statement 2 is correct.
o Condition for mutual divorce: A petition for divorce by mutual consent can be moved only after a
year of the marriage. Hence, statement 3 is correct.
o In case of ‘exceptional hardship to the petitioner or of exceptional depravity on the part of the
respondent’, the petition can be moved earlier under Section 14 of HMA.
o A waiver of the six-month waiting period under Section 13B (2) can be sought in an exemption
application filed before the family court.

Q 4.B
• Recent Context: The World Bank Group is implementing a new corporate flagship, B-READY, to
measure the business and investment climates in 180 economies worldwide annually.
• Business Ready (B-Ready) Project
o Objective: It assess the business and investment climate in up to 180 economies in a transparent and
fair way
o Criteria: Business Entry, Business Location, Utility Services, Labor, Financial Services,
International Trade, Taxation, Dispute Resolution, Market Competition, and Business
Insolvency
o Approach: Reflects a more balanced and transparent approach towards evaluating a country’s
business and investment climate than earlier approach.
o Initially it Covers 54 economies in Asia, Latin America, Europe, the Middle East and Sub-Saharan
Africa, in the first year and expected to scale up to 180 economies by 2025-26.
• Hence option (b) is the correct answer.

Q 5.A
• Recent Context: Supreme Court Upholds Laws Allowing Jallikattu, Kambala & Bull-Cart Racing In
Tamil Nadu, Karnataka & Maharashtra.
• It is a traditional buffalo race in paddy fields filled with slush and mud which generally takes place in
coastal Karnataka (Udupi and Dakshina Kannada) from November to March. Hence, statement 1 is
correct.
• Traditionally, it is sponsored by local Tuluva landlords and households in the coastal districts. Tuluva
people are an ethnic group native to Southern India. They are native speakers of the Tulu
language. Hence, statement 2 is not correct.
• During the race, the racers try to bring the buffaloes under control by holding their reins tight and
whipping them.
• In its traditional form, Kambala was non-competitive and buffalo pairs raced one after another in paddy
fields.
• It was also observed as thanksgiving to gods for protecting the animals from diseases.

Q 6.C
• Nitrogen pollution is the term used to describe the harm that excess nitrogen and nitrogen
compounds, such as nitrous oxides, nitrogen oxide, and ammonia, do to the environment, wildlife,
and human health.
• It is usually caused due to various reasons as emissions from chemical fertilizers, livestock-rearing,
manure-making, and burning of fossil fuels. The overuse of nitrogen fertilizer leads to nitrogen losses
from the soil through volatilization, denitrification, leaching to groundwater, and surface runoff and
erosion. Hence statement 2 is correct.
• Such excess nitrogen from the fertilizers (in the form of agricultural runoff), can reach the water
bodies. In this, the nitrogen is taken up by algae which then grows abundantly to
cause eutrophication. Hence, statement 3 is correct.
• The maximum limit of nitrate in drinking water is 50 ppm. Excess nitrate in drinking water can cause
diseases such as methemoglobinemia (‘blue baby’ syndrome). Hence, statement 1 is correct.
o Infant methemoglobinemia is also called “blue baby syndrome.” It is a condition where a baby's skin
turns blue. This happens when there is not enough oxygen in the blood. Parents should immediately
contact a physician if the baby's skin is unusually bluish in color.
Q 7.B
• Bioremediation: The application of biotic agents like microorganisms in the correction and recovery of
environmental damage is called as bioremediation. The removal of oil spilled on sea water by the help
of bacteria is one example of bioremediation.
2 www.visionias.in ©Vision IAS

FREE BY KING R QUEEN P [ऋषभ राजपूत]


• Bioaccumulation: Increase in concentration of a substance within a trophic level in certain tissues of
organisms' bodies due to absorption from food and the environment is called bioaccumulation.
• Biomagnification: Increase in concentration of a substance in the bodies of consumers as one moves up
the food chain is called biomagnification. For example, when chemicals or pesticides are let into rivers or
lakes they are consumed by aquatic organisms like fish, which in turn are consumed by large birds,
animals or humans.
• Biopiracy: The practice of commercial exploitation of biochemicals or genetic materials which occur
naturally. It is the unethical or unlawful appropriation or commercial exploitation of biological materials
• Hence, option (b) is the correct answer.

Q 8.B
• The Environment Protection Act of 1986 is a significant legislation in India that focuses on the
prevention, control, and abatement of environmental pollution.
• EPA empower the Government to:
o Plan and execute a nation-wide programme for the prevention, control and abatement of
environmental pollution. Lay down standards for the quality of the environment in its various aspects
like emission or discharge of environmental pollutants from various sources.
• The Central government has notified Solid Waste Management Rules 2016, Hazardous and Other
Wastes (Management & Transboundary Movement) Rules 2016, Plastic Waste Management Rules
2016, E-Waste (Management) Rule 2016 and Bio-Medical Waste Management Rules, 2018 under
Environment (Protection) Act, 1986. Hence option (b) is the correct answer.

Q 9.A
• Recent Context: The Supreme Court has held that the government, when entering into a contract under
the President’s name, cannot claim immunity from the legal provisions of that contract under Article 299
of the Constitution.
o What is Article 299 of the Indian Constitution?
✓ Article 298 grants the Centre and the state governments the power to carry on trade or business,
acquire, hold, and dispose of property, and make contracts for any purpose, while Article 299
delineates the manner in which these contracts will be concluded.
✓ Article 299 provides that “all contracts made in the exercise of the executive power of the
Union or of a State shall be expressed to be made by the President or by the Governor of the
State” and that all such contracts and “assurances of property made in the exercise of that power
shall be executed” on behalf of the President or the Governor by persons in a manner as directed
and authorized by them. Hence option (a) is the correct answer.
✓ Further, the phrase ‘expressed to be made and executed’ under Article 299 (1) means that there
must be a deed or contract in writing and that it should be executed by a person duly authorized
by the President of the Governor on their behalf.
o Essential Requirements for Government Contracts under Article 299: 1966 ruling in ‘K.P.
Chowdhry v. State of Madhya Pradesh. And Others’, laid down essential requirements for
government contracts under Article 299. Three conditions to be met before a binding contract against
the government could arise, namely:
✓ the contract must be expressed to be made by the Governor or the Governor-General;
✓ it must be executed in writing, and
✓ the execution should be by such persons and in such a manner as the Governor or the Governor-
General might direct or authorize.

Q 10.C
• The 1972 United Nations Conference on the Human Environment in Stockholm was the first world
conference to make the environment a major issue. The participants adopted a series of principles for
sound management of the environment including the Stockholm Declaration and Action Plan for the
Human Environment and several resolutions. The Stockholm Declaration, which contained 26
principles, placed environmental issues at the forefront of international concerns and marked the start of a
dialogue between industrialized and developing countries on the link between economic growth, the
pollution of the air, water, and oceans and the well-being of people around the world.
• In 1987 the Brundtland Report, also known as Our Common Future, alerted the world to the
urgency of making progress toward economic development that could be sustained without
depleting natural resources or harming the environment. The Brundtland Report focused primarily on

3 www.visionias.in ©Vision IAS

FREE BY KING R QUEEN P [ऋषभ राजपूत]


the needs and interests of humans and was concerned with securing global equity for future generations by
redistributing resources towards poorer nations to encourage their economic growth.
• The United Nations Conference on Environment and Development (UNCED), also known as the
'Earth Summit', was held in Rio de Janeiro, Brazil, from 3-14 June 1992. This global conference,
held on the occasion of the 20th anniversary of the first Human Environment Conference in Stockholm,
Sweden, in 1972, brought together political leaders, diplomats, scientists, representatives of the media and
non-governmental organizations (NGOs) from 179 countries for a massive effort to focus on the impact of
human socio-economic activities on the environment.
• The Kyoto Protocol was adopted on 11 December 1997. Owing to a complex ratification process, it
entered into force on 16 February 2005. Currently, there are 192 Parties to the Kyoto Protocol. It
operationalizes the United Nations Framework Convention on Climate Change by committing
industrialized countries and economies in transition to limit and reduce greenhouse gas (GHG) emissions
in accordance with agreed individual targets. The Convention itself only asks those countries to adopt
policies and measures on mitigation and to report periodically.
• Hence, option (c) is the correct answer.

Q 11.C
• The microbial action on solid wastes breaks it into simpler form. Some of the waste matter is converted
into gases (mostly carbon dioxide, CO2) and the rest becomes simple to be taken in by plants.
• Aerobic bacteria decompose solid wastes in air and produce carbon dioxide. They absorb nitrogen, carbon
and phosphorus from the decomposed wastes. Anaerobic decomposition occurs in the absence of
oxygen. Anaerobic bacteria give out enzyme which breaks down the wastes and produce Methane
or Marsh Gas (CH4). The sulphur content is converted into sulphur dioxide (SO2). Hence, statement 1
is correct.
• The nutrients like nitrogen, phosphorus and carbon etc. are then taken in by anaerobic bacteria. Very
much heat (upto 80°C) is produced due to anaerobic decomposition. It kills seeds of weeds and
germs present in the waste. The solid waste, for this type of decomposition has to be covered in a pit
properly to cut off oxygen supply from the air. Hence, statement 2 is correct.

Q 12.B
• Recent Context: The Ministry of Home Affairs in India has directed central law enforcement agencies,
including the CBI, NIA, and the Narcotics Control Bureau (NCB), to access and share their records with
the national fingerprints database, NAFIS (National Automated Fingerprint Identification System).
• About NAFIS:
o This database, known as NAFIS, is a web-based application that serves as a central repository of
criminal fingerprint data collected from various states and Union territories. Hence, statement 1 is
correct.
o It aims to enhance the ability of these agencies to identify criminals and expedite investigations by
leveraging the comprehensive fingerprint data available in the database.
o It is conceptualized by National Crime Records Bureau (NCRB) in 2022. It is managed by the
Central Fingerprint Bureau (CFPB) at NCRB. Hence, statement 3 is not correct.
o A unique 10-digit National Fingerprint Number (NFN) is assigned to each person arrested for a
crime. Hence, statement 2 is correct.
o The NFN is used for the person’s lifetime, and different crimes registered under different FIRs are
linked to the same NFN in the Crime and Criminal Tracking Network & Systems (CCTNS) database
o Fingerprinting History: First began automating the fingerprint database in 1992 with FACTS 1.0,
recommended by the National Police Commission. FACTS 5.0 was upgraded in 2007 and later
replaced by NAFIS in 2022

Q 13.D
• The biological wealth of our planet has been declining rapidly and the accusing finger is clearly pointing
to human activities. The colonisation of tropical Pacific Islands by humans is said to have led to the
extinction of more than 2,000 species of native birds. The IUCN Red List (2004) documents the extinction
of 784 species (including 338 vertebrates, 359 invertebrates and 87 plants) in the last 500 years.
• Some examples of recent extinctions include the dodo (Mauritius), quagga (Africa), thylacine
(Australia), Steller’s Sea Cow (Russia) and three subspecies (Bali, Javan, Caspian) of tiger. Hence
option (d) is the correct answer.
• Careful analysis of records shows that extinctions across taxa are not random; some groups like
amphibians appear to be more vulnerable to extinction. Adding to the grim scenario of extinctions is the
4 www.visionias.in ©Vision IAS

FREE BY KING R QUEEN P [ऋषभ राजपूत]


fact that more than 15,500 species worldwide are facing the threat of extinction. Presently, 12 per cent of
all bird species, 23 per cent of all mammal species, 32 per cent of all amphibian species and 31per cent of
all gymnosperm species in the world face the threat of extinction.

Q 14.C
• Project Hangul is a conservation and protection project for the critically endangered Kashmir Stag
or Hangul. In the 1970s, the Jammu and Kashmir government prepared a project to protect Hangul
and its habitat with the help of the IUCN and the World Wildlife Fund (WWF). Hangul is commonly
referred to as Kashmir Stag. Project Hangul was the name given to this ambitious project for the
conservation and protection of Kashmir stags. As a result, by 1980, the population of this species had
increased to 340. According to the IUCN Red List, Hangul is a Critically Endangered species.
• Project Elephant was launched by the Government of India in the year 1992 as a Centrally
Sponsored Scheme with the following objectives:
o To protect elephants, their habitat & corridors.
o To address issues of man-animal conflict.
o The Welfare of captive elephants
• The Ministry of Environment, Forest and Climate Change launched the “Asiatic Lion Conservation
Project” in the year 2004.
o It will be funded from the Centrally Sponsored Scheme- Development of Wildlife Habitat (CSS-
DWH) with the contributing ratio being 60:40 of Central and State share.
o Focus: To cause habitat improvement, scientific interventions, disease control and veterinary care
supplemented with adequate eco-development works for the fringe population in order to ensure a
stable and viable Lion population in the country.
• Project Snow Leopard was launched in 2009 to promote an inclusive and participatory approach to
conserving snow leopards and their habitat.
o The strategy document stresses knowledge-based, landscape-level and participatory management of
wildlife in the region.
o Each of the range states will identify one landscape under the project.
• Hence, option (c) is the correct answer.

Q 15.C
• Irreparable computers and other electronic goods are known as electronic wastes (e-wastes). E-
wastes are burried in landfills or incinerated.
• Over half of the e-wastes generated in the developed world are exported to developing countries,
mainly to China, India and Pakistan, where metals like copper, iron, silicon, nickel and gold are recovered
during recycling process.
• Unlike developed countries, which have specifically built facilities for recycling of e-wastes, recycling in
developing countries often involves manual participation thus exposing workers to toxic substances
present in e-wastes.
• E-waste-connected health risks may result from direct contact with harmful materials such as lead,
cadmium, chromium, brominated flame retardants or polychlorinated biphenyls (PCBs), from
inhalation of toxic fumes, as well as from accumulation of chemicals in soil, water and food. In addition
to its hazardous components, being processed, e-waste can give rise to a number of toxic by-products
likely to affect human health. Furthermore, recycling activities such as dismantling of electrical
equipment may potentially bear an increased risk of injury.
• Hence option (c) is the correct answer.

Q 16.B
• Eutrophication is the natural aging of a lake by nutrient enrichment of its water. Hence statement 1
is not correct.
o In a young lake, the water is cold and clear, supporting little life. With time, streams draining into the
lake introduce nutrients such as nitrogen and phosphorus, which encourage the growth of aquatic
organisms.
o As the lake’s fertility increases, plant and animal life burgeons, and organic remains begin to be
deposited on the lake bottom.
o Over the centuries, as silt and organic debris pile up, the lake grows shallower and warmer, with
warm-water organisms supplanting those that thrive in a cold environment.
o Marsh plants take root in the shallows and begin to fill in the original lake basin.
o Eventually, the lake gives way to large masses of floating plants (bog), finally converting into land.
5 www.visionias.in ©Vision IAS

FREE BY KING R QUEEN P [ऋषभ राजपूत]


o Depending on the climate, size of the lake, and other factors, the natural aging of a lake may span
thousands of years.
• However, pollutants from man’s activities like effluents from industries and homes can radically
accelerate the aging process. This phenomenon has been called Cultural or Accelerated
Eutrophication.
o During the past century, lakes in many parts of the earth have been severely eutrophied by sewage and
agricultural and industrial wastes.
o The prime contaminants are nitrates and phosphates, which act as plant nutrients. They
overstimulate the growth of algae, causing unsightly scum and unpleasant odors, and robbing the
water of dissolved oxygen vital to other aquatic life. Hence statement 2 is correct.
o At the same time, other pollutants flowing into a lake may poison whole populations of fish, whose
decomposing remains further deplete the water’s dissolved oxygen content. In such a fashion, a lake
can literally choke to death.

Q 17.B
• A National Park is an area protected and preserved by law to protect and preserve flora and fauna within
its boundary. Grazing of cattle and removal of any wildlife from its habitat is strictly prohibited in a
national park and all rights are reserved with the government.
• A Sanctuary is a protected area where wild animals and birds are kept and encouraged to increase their
population. In wildlife sanctuaries, rare and endangered species are encouraged to breed in human-
controlled environments with restricted settings.
• Differences between a National Park and Wildlife Sanctuary:
o A National Park is established by the government in view of the protection of all the flora and fauna
of certain areas. But a Wildlife sanctuary is meant for the protection, breeding and propagation of
individuals of an endangered species.
o The boundaries of a National Park are usually well-marked and circumscribed whereas those of
sanctuaries are often not well-defined and human activities are permitted to a specified limit.
Hence statement 2 is correct.
o A National Park is established under the provisions of Section 35, section 38(2), and Section 66(3) of
the Wild Life (Protection) Act of 1972. On the other hand, a Wildlife Sanctuary is an area that is
established under the provisions of Section 26-A, Section 38(1), and Section 66(3) of this Act.
o An area can not be notified as a National Park if it is a reserve Forest. However, this is not the case
with a Wildlife Sanctuary in which an area can directly be notified as a Wildlife Sanctuary even if it is
a reserve forest.
o National parks are given a greater degree of protection, with human activity greatly
restricted. Sanctuaries have relatively less restrictions and are open to people, without the
requirement of official permission. Hence statement 1 is correct.
• In spite of some differences, National Parks and Wildlife Sanctuaries have basic similarities.
o The Wildlife (Protection) Act (WPA) of 1972 provided for the declaration of National Parks by the
State Government in addition to the declaration of wildlife sanctuaries. The Central Government may
also declare, Wild Life Sanctuary and National Park under certain conditions. Hence statement 3 is
not correct.
o It can be said that both the National Park and the Wildlife Sanctuary are protected areas, and no biotic
interference is allowed in both of these areas.

Q 18.A
• Recent studies have been published stating that termites emit methane. But the extent of their risk
to global warming is uncertain.
• What are Termites?
o These are small insects, who live in the mounds as colonies of 60,000-200,000, are not just skilled
architects and engineers but also a surprising source of methane, a greenhouse gas more potent
than carbon dioxide.
o Termites are known to wreak havoc on agriculture, forests and construction, due to their affinity for
plants and wood. However, only 10-15 per cent of 3,000 termite species identified globally are
categorised as pests.
• Methane Emissions
o Termites are counted among natural sources of methane like wetlands, wild animals, livestock
and geological features like volcanoes, and are estimated to account for 1-3 per cent of emissions
globally. Hence statement-I is correct.
6 www.visionias.in ©Vision IAS

FREE BY KING R QUEEN P [ऋषभ राजपूत]


o In 2008-17, the world emitted 576 Tg (1 Tg or teragram is equal to 1012 g) of methane per year, of
which termites contributed 9 Tg, says the Global Carbon Project, in its “Global Methane Budget”
published in 2020. Other estimates put the emissions at 2-15 Tg per year.
• Reason for Methane Emissions
o In natural ecosystems, they feed on and recycle the nutrients present in dead and decaying plant and
animal matter. It is this cellulose-rich diet that causes their emissions.
o Methanogenic microorganisms that live in the gut of termites break down the cellulose entering
the body and release methane. Hence statement-II is correct and the correct explanation for
Statement-I.

Q 19.A
• Ionizing radiations: Electromagnetic radiations such as short wavelength ultra violet radiations (UV),
X-rays and gamma rays and energetic particles produced in nuclear processes, electrically charged
particles like alpha and beta particles produced in radioactive decay and neutrons produced in nuclear
fission, are highly damaging to living organisms. Hence, statement 1 is not correct.
• Electrically charged particles produced in the nuclear processes can have sufficient energy to knock
electrons out of the atoms or molecules of the medium, thereby producing ions. The ionizing
radiations cause damage to biological systems and are, therefore are pollutants. Hence, statement 2 is
correct.
• A gamma ray passing through a cell, can ionise the water molecules near the DNA. These ions
might react with the DNA causing it to break. They can also cause chemical changes by breaking
the chemical bonds, which can damage living tissues.Hence, statement 3 is correct.

Q 20.B
• The gradual and fairly predictable change in the species composition of a given area is called ecological
succession. Hence statement 1 is not correct.
• During succession, some species colonize an area and their population becomes more numerous whereas
populations of other species decline and even disappear. The entire sequence of communities that
successively change in a given area is called sere(s). The individual transitional communities are termed
seral stages or seral communities.
• Ecological succession takes place in two kinds i.e. Primary Succession and Secondary succession.
• Primary succession occurs in essentially lifeless areas and regions (created or exposed for the first
time) in which the soil is incapable of sustaining life as a result of such factors as lava flows, newly
formed sand dunes, or rocks left from a retreating glacier. The species that invade a bare area are called
pioneer species. The pioneer species in secondary succession are plants such as grasses, birch trees,
and fireweed. The first organisms to appear in areas of primary succession are often mosses or

7 www.visionias.in ©Vision IAS

FREE BY KING R QUEEN P [ऋषभ राजपूत]


lichens. These organisms are known as pioneer species because they are the first species present.
Hence statement 2 is correct.
• Secondary succession occurs in areas where a community that previously existed has been removed;
it is typified by smaller-scale disturbances that do not eliminate all life and nutrients from the
environment.
• All succession whether taking place in water or on land, proceeds to a similar climax community the
mesic.
• Hydrarch succession and xerarch succession are the successions of plants. Hydrarch succession takes
place in wet areas and the successional series progress from hydric to the mesic conditions. As
against this, xerarch succession takes place in dry areas, and the series progress from xeric to mesic
conditions. Hence, both hydrarch and xerarch successions lead to medium water conditions (mesic) –
neither too dry (xeric) nor too wet (hydric).
• At any time during primary or secondary succession, natural or human-induced disturbances (fire,
deforestation, etc.), can convert a particular seral stage of succession to an earlier stage. Also, such
disturbances create new conditions that encourage some species and discourage or eliminate other
species.

Q 21.D
• Recent Context: RBI has approved the First Loss Default Guarantee (FLDG) programme, which
allows fintechs to partner with banks and NBFCs.
o It is a safety-net arrangement among banks, non-banking finance companies (NBFCs) and
lending service providers (LSPs), whereby the LSP guarantees to compensate the regulated
entities (banks, NBFCs etc) for loss due to default up to a certain threshold of the loan portfolio.
Since losses only to a certain threshold are covered under this arrangement, it’s called as first
loss default guarantee or FLDG. Hence statements 1 and 2 are correct.
o This move is expected to strengthen the digital lending ecosystem and is seen as positive for data-tech
NBFCs and fintechs.
• RBI Guidelines regarding FLDG
o In guidelines issued in 2022, the RBI had indicated it was not in favour of such arrangements since
they could encourage lenders to take on undue risk.
o However, recently after extensive consultations with various stakeholders, the RBI allowed FLDG
proposing regulatory framework with the objective of maintaining a balance between innovation and
prudent risk management.
o New guidelines:
✓ The LSP-providing default-loss-guarantee must be incorporated as a company under the
Companies Act, 2013. Hence statement 3 is correct.
✓ Banks and NBFCs should ensure that the total amount of DLG cover on any outstanding
portfolio does not exceed 5% of the amount of that loan portfolio.

Q 22.C
• Recent Context: Quantum physics proposes a new way to study biology. The results of this
quantum biology study could revolutionise our understanding of how life works.
• Quantum physics is the study of matter and energy at the most fundamental level. It aims to
uncover the properties and behaviors of the very building blocks of nature. While many quantum
experiments examine very small objects, such as electrons and photons, quantum phenomena are all
around us, acting on every scale. Hence statement 1 is correct.
• Quantum effects only manifest at very small length and mass scales, or when temperatures
approach absolute zero. This is because quantum objects like atoms and molecules lose their
“quantumness” when they uncontrollably interact with each other and their environment. Hence
statement 2 is correct.
o Quantum physics describes matter and energy as quantum wavefunctions, which sometimes act
like waves and sometimes act like particles, but are actually more complicated entities than just
waves or particles. In reality, every object in the universe (from atoms to stars) operates according to
quantum physics.
o A "quantum effect" is an effect that is not properly predicted by classical physics, but is properly
predicted by quantum theory. Classical physics describes matter as composed of little, solid particles.
Therefore, anytime we get the pieces of matter to act like waves, we are demonstrating a quantum
effect. (Classical waves such as sound and sea waves don't count as quantum because the motion is a

8 www.visionias.in ©Vision IAS

FREE BY KING R QUEEN P [ऋषभ राजपूत]


wave, but the pieces are still little solid balls. In order to be a quantum effect, the particle itself must
be acting like a wave.)
o Quantum mechanics (or physics) differs from classical physics in that energy, momentum, angular
momentum, and other quantities of a boundsystem are restricted to discrete values (quantization);
objects have characteristics of both particles and waves (wave–particle duality); and there are limits to
how accurately the value of a physical quantity can be predicted prior to its measurement, given a
complete set of initial conditions (the uncertainty principle).
o A quantum phenomenon where particles can exist in multiple states simultaneously until measured
or observed, in contrast to classical physics where objects have definite properties is
called Superposition.
• Below are some examples of macroscopic quantum effects.
o Superconductivity: When a conducting material is cooled enough, its conduction electrons spread
out into large-scale coherent wave states. These coherent wave states are able to flow past impurities
and atoms without being perturbed, so that a material with zero electrical resistance results.
Superconductivity leads to interesting macroscopic effects such as quantum levitation (the Meissner
effect).
o Superfluidity: When certain materials are cooled enough, their atoms can spread out into coherent
wave states that resist surface tension, allowing the material to flow like a liquid with zero viscosity.
o Bose Einstein Condensates: When certain materials are cooled enough, their atoms spread out
completely into a single, giant, coherent wave state. A macroscopic chunk of matter that has
condensed in this way acts like a wave and exhibits wave properties such as interference.

Q 23.A
• Most herbicides are toxic to mammals but are not as persistent as organo-chlorides. These chemicals
decompose in a few months. Like organo-chlorides, these too become concentrated in the food web.
Some herbicides cause birth defects. Studies show that cornfields sprayed with herbicides are more prone
to insect attack and plant disease than fields that are weeded manually.
• Pesticides and herbicides represent only a very small portion of widespread chemical pollution. A large
number of other compounds that are used regularly in chemical and industrial processes for manufacturing
activities are finally released in the atmosphere in one or other form. Hence, option (a) is the correct
answer.

Q 24.D
• A higher-than-normal concentration of Ozone molecules called the Ozone layer, is found in
Stratosphere. It acts as a shield absorbing ultraviolet radiation from the sun. Depletion of the Ozone
layer or Ozone hole leads to more UV rays reaching the lower atmosphere.
• Since ozone prevents high-intensity ultraviolet radiation from reaching the surface of the Earth and
causing stratospheric warming, it can be assumed that the formation of the ozone hole changes the total
radiation budget of the Earth. This is, indeed, the case. However, ozone depletion and the formation of
the polar ozone holes don’t lead to a further warming of the troposphere, but to a slight cooling.
• The absorption of ultraviolet radiation by ozone molecules causes warming in the stratosphere.
Some of this heat emitted in the stratosphere is transferred to the troposphere causing slight
tropospheric warming as well. This warming gets lessened due to the formation of an ozone hole.
• In the lower stratosphere, ozone can still act as a greenhouse gas and absorb infrared radiation coming
from the Earth's surface. So absorption of both ultraviolet and infrared radiation by ozone leads to a
warming of the upper troposphere. If ozone levels decrease, the upper troposphere will, therefore,
get cooler.
• The high levels of UV rays cause non-melanoma skin cancer. Additionally, it plays a major role in
malignant melanoma development. UV is also linked to cataracts.
• Plant growth as well as its physiological and developmental process are all affected negatively.
These include the way plants form, the timing of development and growth, the distribution of plant
nutrients and metabolism, etc. These changes can have important implications for plant competitive
balance, animals feeding on these plants, plant diseases, and biogeochemical cycles.
• Increased surface UV leads to increased tropospheric ozone. Ground-level ozone is generally
recognized to be a health risk, as ozone is toxic due to its strong oxidant properties. The risks are
particularly high for young children, the elderly, and those with asthma or other respiratory difficulties.
Ozone at ground level is produced mainly by the action of UV radiation on combustion gases from
vehicle exhausts.
• Hence, option (d) is the correct answer.
9 www.visionias.in ©Vision IAS

FREE BY KING R QUEEN P [ऋषभ राजपूत]


Q 25.C
• An ecotone is a zone of junction or a transition area between two biomes (diverse ecosystems).
Ecotone is the zone where two communities meet and integrate. For e.g. the mangrove forests represent an
ecotone between marine and terrestrial ecosystems.
• The edge effect refers to the changes in population or community structures that occur at the
boundary of two habitats (ecotone). Sometimes the number of species and the population density of some
of the species in the ecotone is much greater than either community. This is called the edge effect. The
organisms which occur primarily or most abundantly in this zone are known as edge species. Hence,
statement option (a) is not correct.
• Ecocline is a zone of gradual but continuous change from one ecosystem to another when there is no
sharp boundary between the two in terms of species composition. Ecocline occurs across the
environmental gradient (gradual change in abiotic factors such as altitude, temperature (thermocline),
salinity (halocline), depth, etc.). Hence, option (c) is the correct answer.
• A niche refers to the unique functional role and position of a species in its habitat or ecosystem. The
functional characteristic of a species in its habitat is referred to as a “niche” in that common habitat. In
nature, many species occupy the same habitat, but they perform different functions: Habitat niche –
where it lives, food niche – what it eats or decomposes & what species it competes with, Reproductive
niche – how and when it reproduces, Physical & chemical niche – temperature, land shape, land slope,
humidity & another requirement. Hence, option (b) is not correct.
• Biome is a large regional unit characterized by a major vegetation type and associated fauna found in
a specific climate zone. The biome includes all associated developing and modified communities
occurring within the same climatic region, e.g., forest biomes, grassland and savanna biomes, desert
biome, etc. Hence, option (d) is not correct.

Q 26.C
• Conservation International was a pioneer in defining and promoting the concept of hotspots. In 1989,
just one year after scientist Norman Myers wrote the paper that introduced the hotspots concept,
Conservation International adopted the idea of protecting them as the guiding principle of their
investments.
• Hot spots are regions with very high levels of species richness and a high degree of endemism (that
is, species confined to that region and not found anywhere else) that are under constant threat.
• To qualify as a biodiversity hotspot, a region must meet two strict criteria:
o It must have at least 1,500 vascular plants as endemics — which is to say, it must have a high
percentage of plant life found nowhere else on the planet. A hotspot, in other words, is irreplaceable.
o It must have 30% or less of its original natural vegetation. In other words, it must be threatened.
• The mere presence of invasive species is not a criteria to identify biodiversity hotspots.
• Hence option (c) is the correct answer.

Q 27.A
• Ozone is a gas composed of three atoms of oxygen. It occurs both in the Earth’s upper atmosphere
and at ground level. Depending on its location in the atmosphere, it can be good or bad.
• Stratospheric ozone is formed naturally through the interaction of solar ultraviolet (UV) radiation
with molecular oxygen (O2). Here, it forms the "ozone layer," which is approximately 6-30 miles above
the Earth's surface. This layer shields us from the harmful UV radiation emanating from the sun. Hence,
statement 2 is correct.
• In the troposphere, ozone is formed when heat and sunlight cause chemical reactions between oxides
of nitrogen (NOX) and Volatile Organic Compounds (VOC), which are also known as Hydrocarbons.
This reaction can occur both near the ground and high in the atmosphere. This ground Ozone is
the “bad” ozone and is a secondary air pollutant. It damages crops, trees, and other vegetation and
caused severe health problems. Hence statement 3 is not correct.
• In both these processes, Ozone is formed naturally. However, it can also be artificially produced, so that
it can be used for water treatment. Ozone generators can create ozone artificially by means of extremely
high voltages or by means of UV light. Both methods involve the decomposition of the oxygen
molecule. Hence statement 1 is not correct.

Q 28.A
• Recent Context: The World Meteorological Congress has approved a new greenhouse gas (GHG)
monitoring initiative in a landmark decision.

10 www.visionias.in ©Vision IAS

FREE BY KING R QUEEN P [ऋषभ राजपूत]


o The initiative supports urgent action to reduce heat-trapping gases, which are fuelling temperature
increases.
• Need for GHG Watch
o Many of the existing international and national activities dealing with greenhouse gases are supported
mainly by the research community.
o At present, there is no comprehensive, timely international exchange of surface and space-based
greenhouse gas observations or modelling products.
o The new global GHG watch will fill critical information gaps and provide an integrated and
operational framework.
o The framework will bring all space-based and surface-based observing systems, as well as modelling
and data assimilation capabilities, under one roof.
• Features:
o The GHG watch will consist of four main components:
✓ A comprehensive, sustained, global set of surface-based and satellite-based observations
of carbon dioxide (CO2), methane (CH4) and nitrous oxide (N2O) concentrations.
✓ Prior estimates of the GHG emissions based on activity data and process-based models;
✓ A set of global high-resolution Earth System models representing GHG cycles;
✓ Associated with the models, data assimilation systems that optimally combine the observations
with model calculations to generate products of higher accuracy.
• Hence option (a) is the correct answer.

Q 29.C
• Polar regions are characterized by the presence of sub-zero temperatures which makes it difficult for a
range of animals to survive in such harsh conditions. An overwhelming majority (99 percent) of animals
and nearly all plants cannot maintain a constant internal environment in adverse conditions. Heat loss or
heat gain is a function of surface area. Since small animals have a larger surface area relative to their
volume, they tend to lose body heat very fast when it is cold outside; then they have to expend much
energy to generate body heat through metabolism. This is the main reason why very small animals are
rarely found in polar regions. Hence statement 1 is correct and statement 2 is not correct.
• Mammals from colder climates generally have shorter ears and limbs to minimize heat loss. This is called
Allen’s Rule. In the polar seas, aquatic mammals like seals have a thick layer of fat (blubber) below their
skin that acts as an insulator and reduces the loss of body heat.

Q 30.A
• Recent context: The Chief Justice of India (CJI) DY Chandrachud condemned ‘forum shopping’.
o Forum shopping is the practice of choosing the court which is likely to provide the most
favorable outcome. Hence option (a) is the correct answer.
o Litigants or lawyers attempt to deliberately move their case to a particular judge or Court where they
think the judgment could be more favorable.
o In 2017 ruling of ‘The Union of India & Ors. Vs. Cipla Ltd.’ case, the court condemned forum
shopping.
o It laid down a ‘functional test’ to be adopted to determine whether a litigant is indulging in forum
shopping or not.
o Test: If there is any functional similarity in the proceedings between one court and another or there is
some sort of subterfuge on the part of a litigant, then it is considered forum shopping.
o Prevention: Most common law countries use the ‘forum non-conveniens’ principle to prevent forum
shopping.
o It is the discretionary power of a court to refuse to exercise its jurisdiction over a matter where
another court, or forum, may more conveniently hear a case.
o Using this power, the court can dismiss a case in the interests of justice and the parties while
allocating it to the appropriate bench.

Q 31.C
• The two main factors taken into account when measuring diversity are richness and evenness. Species
richness and species evenness are two components important in measuring the biological diversity
of an ecosystem. Both measures describe the species living in a particular area.
o Species richness: It is the number of species within a defined region. Generally, species richness
counts the number of species in a particular ecosystem. However, it does not describe the abundance

11 www.visionias.in ©Vision IAS

FREE BY KING R QUEEN P [ऋषभ राजपूत]


of the species in that particular ecosystem. It’s possible to measure the species richness through either
sampling or a census. Hence statement 2 is correct.
o In addition, species richness can be divided into three components:
✓ α-Diversity- Alpha diversity refers to diversity within a particular area, community or ecosystem,
and is measured by counting the number of taxa (usually species) within the ecosystem.
✓ β-Diversity- Beta diversity is species diversity between ecosystems; this involves comparing the
number of taxa that are unique to each of the ecosystems.
✓ Y -Diversity- Gamma diversity is a measurement of the overall diversity for different ecosystems
within a region.
o Furthermore, species richness does not describe the distribution of the species within a particular
geological area. It only describes the number of species in the above particular geological area.
o Species Evenness: It is the number of species and the relative abundance of species in a particular
community. There are several indices to describe species evenness and the two most common
measures of evenness are the Shannon index (H) and the Simpson index (D). Hence statement 1 is
correct.
✓ The Shannon index (H) is a measure of the information content of a community rather than that of
a particular species. On the other hand, the Simpson index (D) measures the dominance of a
multispecies community and can be thought of as the probability that two individuals selected
from a community will be of the same species.

Q 32.B
• Pair 1 is not correctly matched: Kaziranga National Park is located on the bank of the river
Brahmaputra in Assam. It is famous for one Horned Rhinoceros. Besides rhinos, other animals protected
in the park are swamp deer, bison, tiger, leopard, hoolock gibbon, wild buffaloes, pythons, monitor
lizards, elephants, etc. Principal plants found in the park are tall elephant grass, Sal trees, and different
types of bushes. This national park was notified in the year 1974, and its area is 42, 996 hectares.
• Pair 2 is correctly matched: Ranthambore National Park is located in the Rajasthan state of India.
Constituted in 1980, this park is spread in an area of 39, 200 hectares. The principal wildlife protected in
this park is crocodile, nilgai, gazelle, sambhar, etc.
• Pair 3 is correctly matched: Kanha National Park is a Tiger Reserve. It is located in Madhya Pradesh.
Some important animals kept in this park are –leopards, langurs, mongooses, Cats, hyenas, porcupines,
etc. Sal and bamboos are the principal trees in this park besides a wide variety of other plants. This
national park was notified in the year 1955 and its area is 94, 000 hectares.
• Pair 4 is not correctly matched: Simlipal National Park is located in the Mayurbhanj district of Orissa
state of India. This National Park comprises dense Sal forest due to which this park has been chosen for
the Project Tiger. The fauna of this national park includes tigers, elephants, deer, pea foul, talking mainas,
chital, sambhar, panther, gaur, hyenas, and sloth beer. Notified in the year 1978, this national park is
spread over an area of 135,500 hectares.

Q 33.D
• Water pollution is the contamination of water sources by substances that make the water unusable
for drinking, cooking, cleaning, swimming, and other activities. Pollutants include chemicals, trash,
bacteria, parasites, chemicals, etc.
• The processes that contribute to water pollution include-
o Agricultural runoff- It includes contaminants such as fertilizers, pesticides, herbicides, etc.
o Open Defecation- It can cause the release of harmful pathogens in the water body which then
contribute to diseases like Typhoid, etc.
o Discharge of Industrial Effluents- These consist of toxic chemicals, microfibres, and even oil
and petroleum.
o Floods- Floods are destructive in nature and they tend to carry all the material in their way
including plastics, mud, etc.
o Discharge of coolant from Nuclear reactors- The coolant so used tends to be radioactive and can
thus result in radioactivity in the water body and aquatic life.
• Hence, option (d) is the correct answer.

Q 34.A
• Biotic Potential: The maximum rate at which a population can increase when resources are unlimited and
environmental conditions are ideal is termed the population's biotic potential.
• Each species will have a different biotic potential due to variations in:
12 www.visionias.in ©Vision IAS

FREE BY KING R QUEEN P [ऋषभ राजपूत]


o the species' reproductive span (how long an individual is capable of reproducing)o the frequency of
reproduction (how often an individual can reproduce)
o "litter size" (how many offspring are born each time)o survival rate (how many offspring survive to
reproductive age) Carrying Capacity
• For a given region, carrying capacity is the maximum number of individuals of a given species that an
area's resources can sustain indefinitely without significantly depleting or degrading those resources.
• Hence, option (a) is the correct answer.

Q 35.A
• One of the main problems affecting coastal waters is the high levels of nitrogen and phosphorous-based
pollutants entering the water. These pollutants come mainly from human activities. Excessive discharge of
nutrients into coastal water results in accelerated phytoplankton growth. Eutrophication is defined as
‘enhanced plankton growth due to excess supply of nutrients. Large growths of phytoplankton are known
as blooms and these large blooms can have undesirable effects.
• Biological oxygen demand (BOD) is the amount of dissolved oxygen needed (i.e., demanded) by
aerobic biological organisms to break down organic material present in a given water sample at a
certain temperature over a specific time period.
• The common lake or stream contains small amounts of oxygen in the form of dissolved oxygen
(DO). Dissolved oxygen is a crucial component of natural water bodies, maintaining the aquatic life
and quality aesthetic of streams and lakes. So, BOD, indirectly, shows the extent of eutrophication
in the water body. Hence, statement 1 is correct.
• The decay of organic matter in water is measured as biochemical oxygen demand. Environmental stresses
and other human-induced factors can lessen the amount of dissolved oxygen in a water body, however.
• Biological oxygen demand is essentially a measure of the amount of oxygen required to remove
waste organic matter from water in the process of decomposition by aerobic bacteria. Hence, the
higher BOD means the presence of higher organic pollution and not inorganic pollution. Hence,
statement 2 is not correct.

Q 36.B
• According to the IUCN, the total number of plant and animal species described so far is slightly more than
1.5 million, but we have no clear idea of how many species are yet to be discovered and described.
• Some interesting aspects about the earth’s biodiversity based on the currently available species
inventories:
o More than 70 per cent of all the species recorded are animals, while plants (including algae, fungi,
bryophytes, gymnosperms and angiosperms) comprise no more than 22 per cent of the total. Hence
statement 1 is not correct.
o Among animals, insects are the most species-rich taxonomic group, making up more than 70 per
cent of the total. That means, out of every 10 animals on this planet, 7 are insects. Hence statement
2 is correct.
o The number of fungi species in the world is more than the combined total of the species of fishes,
amphibians, reptiles and mammals.

13 www.visionias.in ©Vision IAS

FREE BY KING R QUEEN P [ऋषभ राजपूत]


• It should be noted that these estimates do not give any figures for prokaryotes. Biologists are not sure
about how many prokaryotic species there might be. The problem is that conventional taxonomic methods
are not suitable for identifying microbial species and many species are simply not culturable under
laboratory conditions. If we accept biochemical or molecular criteria for delineating species for this group,
then their diversity alone might run into millions.

Q 37.D
• The accelerated rates of species extinctions that the world is facing now are largely due to human
activities. There are four major causes (‘ The Evil Quartet’ is the sobriquet used to describe them).
o Habitat loss and fragmentation: This is the most important cause driving animals and plants to
extinction. The most dramatic examples of habitat loss come from tropical rainforests. Once covering
more than 14 percent of the earth’s land surface, these rainforests now cover no more than 6 percent.
They are being destroyed fast. Besides total loss, the degradation of many habitats by pollution also
threatens the survival of many species. When large habitats are broken up into small fragments due to
various human activities, mammals and birds requiring large territories, and certain animals with
migratory habits are badly affected, leading to population declines.
o Over-exploitation: Humans have always depended on nature for food and shelter, but when ‘need’
turns to ‘greed’, it leads to the over-exploitation of natural resources. Many species extinctions in the
last 500 years (Steller’s sea cow, passenger pigeon) were due to overexploitation by humans.
Presently many marine fish populations around the world are over-harvested, endangering the
continued existence of some commercially important species.
o Alien species invasions: When alien species are introduced unintentionally or deliberately for
whatever purpose, some of them turn invasive, and cause the decline or extinction of indigenous
species. The Nile perch introduced into Lake Victoria in East Africa led eventually to the extinction of
an ecologically unique assemblage of more than 200 species of cichlid fish in the lake. One must be
familiar with the environmental damage caused and the threat posed to native species by invasive
weed species like carrot grass (Parthenium), Lantana and water hyacinth (Eicchornia). The recent
illegal introduction of the African catfish Clarias gariepinus for aquaculture purposes is posing a
threat to the indigenous catfishes in our rivers.
o Co-extinctions: When a species becomes extinct, the plant and animal species associated with it in an
obligatory way also become extinct. When a host fish species becomes extinct, its unique assemblage
of parasites also meets the same fate. Another example is the case of a coevolved plant-pollinator
mutualism where the extinction of one invariably leads to the extinction of the other.
o Hence option (d) is the correct answer.

Q 38.C
• The diversity of plants and animals is not uniform throughout the world but shows a rather uneven
distribution. For many groups of animals or plants, there are interesting patterns in diversity, the most
well-known being the latitudinal gradient in diversity.
• In general, species diversity decreases as we move away from the equator towards the poles. With
very few exceptions, the tropics (latitudinal range of 23.5° N to 23.5° S) harbour more species than
temperate or polar areas. Hence statement 1 is correct.
• Colombia located near the equator has nearly 1,400 species of birds while New York at 41° N has 105
species and Greenland at 71° N only 56 species. A forest in a tropical region like Ecuador has up to 10
times as many species of vascular plants as a forest of equal area in a temperate region like the Midwest of
the USA.
• Ecologists and evolutionary biologists have proposed various hypotheses on why tropics might account
for their greater biological diversity. ; some important ones are
o Speciation is generally a function of time, unlike temperate regions subjected to frequent glaciations
in the past, tropical latitudes have remained relatively undisturbed for millions of years and thus, had
a long evolutionary time for species diversification.
o Tropical environments, unlike temperate ones, are less seasonal, relatively more constant, and
predictable. Such constant environments promote niche specialization and lead to greater
species diversity. Hence statement 2 is not correct.
o There is more solar energy available in the tropics, which contributes to higher productivity; this in
turn might contribute indirectly to greater diversity.

14 www.visionias.in ©Vision IAS

FREE BY KING R QUEEN P [ऋषभ राजपूत]


Q 39.C
• Recent Context: The Rehabilitation Council of India (RCI) spearheads transformation in education with
a national workshop on inclusive learning.
o The Rehabilitation Council of India (RCI) is a statutory body under the Ministry of Social Justice
and Empowerment. Hence, statement 1 is correct.
o It was established in 1986 as a society and became a statutory body in 1993 by the Rehabilitation
Council of India Act of 1992.
o Its main mandate is to standardize, regulate, and monitor training programs in the field of
special education and disability.
o RCI is also responsible for maintaining the central rehabilitation register (CRR) and promoting
research in this domain.
o The Council also prescribes minimum standards of education and training for 16 categories of
professionals and personnel allocated to RCI. Hence, statement 2 is correct.
o The RCI plans to initiate various national programs that promote flexible education, experiential
learning, practical skill-based education, and ultimately, inclusive education.

Q 40.C
• Great Indian Bustard (hereafter GIB) is one of the rarest birds in the world.
• It’s among the heaviest birds with a horizontal body and long bare legs giving it an ostrich-like
appearance.
• Habitat: The species inhabits open habitats (short grasslands, open scrub, and rain-fed agriculture) and
breeds in traditionally selected grasslands, where males display to attract females. It avoids irrigated areas.
• Great Indian Bustard is an indicator species for grassland habitats and its gradual disappearance from such
environments shows their deterioration. Once the species is lost, there will be no other species to replace
it, and that will destabilise the ecosystem of the grassland and affect critical biodiversities, as well as
blackbucks and wolves, who share their habitat with the GIB.
• Excessive hunting in past and current levels of habitat loss, compounded with very slow life-history traits,
has caused their decline.
• It is endemic to the Indian Sub-continent, found in central India, western India and eastern
Pakistan. Hence, statement 1 is correct.
• The largest population occurs in the Thar Desert, Rajasthan. Other populations occur in Kachchh
(Gujarat), Solapur and Chandrapur (Maharashtra), Kurnool (Andhra Pradesh), and Bellary (Karnataka).
• Bustard Species Found in India: Great Indian Bustard, the Lesser Florican, and the Bengal Florican;
Houbara also belongs to the Bustard family but it's a migratory species.
• Great Indian Bustard is Rajasthan's state bird. The state government has started "Project Godawan"
for its conservation at Desert National Park (DNP) in Jaisalmer. Hence, statement 3 is correct.
• It’s one of the Species for The Recovery Programme under the Integrated Development of Wildlife
Habitats of the Ministry of Environment and Forests. Hence, statement 2 is correct.
• Conservation status of Great Indian Bustard
▪ IUCN Red List: Critically Endangered Wildlife Protection Act: Schedule I and
▪ CITES: Appendix I. Conservation of Migratory Species or Bonn Convention: Appendix I

Q 41.D
• On 1 January 2016, the 17 Sustainable Development Goals (SDGs) of the 2030 Agenda for Sustainable
Development — adopted by world leaders in September 2015 at an historic UN Summit — officially
came into force. Over the next fifteen years, with these new Goals that universally apply to all, countries
will mobilize efforts to end all forms of poverty, fight inequalities and tackle climate change, while
ensuring that no one is left behind.
• While the SDGs are not legally binding, governments are expected to take ownership and establish
national frameworks for the achievement of the 17 Goals. Countries have the primary responsibility for
follow-up and review of the progress made in implementing the Goals, which will require quality,
accessible, and timely data collection.
• The 1992 Rio Declaration on Environment and Development defines the rights of the people to be
involved in the development of their economies, and the responsibilities of human beings to
safeguard the common environment. The declaration builds upon the basic ideas concerning the
attitudes of individuals and nations towards the environment and development, first identified at the
United Nations Conference on the Human Environment (1972). Hence, statement 1 is not correct.
• NITI Aayog has the twin mandate to oversee the adoption and monitoring of the SDGs in the
country, and also promote competitive and cooperative federalism among States and UTs. The index
15 www.visionias.in ©Vision IAS

FREE BY KING R QUEEN P [ऋषभ राजपूत]


represents the articulation of the comprehensive nature of the Global Goals under the 2030 Agenda while
being attuned to the national priorities. The modular nature of the index has become a policy tool and a
ready reckoner for gauging the progress of States and UTs on the expansive nature of the Goals, including
health, education, gender, economic growth, institutions, climate change, and environment. Hence,
statement 2 is not correct.
• The SDG India Index and Dashboard is a crucial tool in India's SDG monitoring efforts. Designed and
developed by NITI Aayog, the Index measures the progress at the national and sub-national levels in our
journey towards meeting the Global Goals and targets. It has also been successful as an advocacy tool
to propagate the messages of sustainability, resilience, and partnerships. IMF has no role in
releasing the Index but it collaborates with the NITI Aayog to estimate the financial cost of
achieving the SDG. Hence, statement 3 is not correct.

Q 42.C
• Recent Context: More than 30 large lakes in India have recorded a drying trend from 1992 to 2020, a
new analysis published in journal Science revealed.
o Of them,16 are the major lakes of southern India. Some of these include Mettur, Krishnarajasagar,
Nagarjuna Sagar and Idamalayar.
• Mettur Lake (Stanley Reservoir):
o Location: Mettur, Tamil Nadu. It was constructed in 1934 and serves as a major source of water for
irrigation and drinking purposes in the region. The reservoir also plays a vital role in generating
hydroelectric power. Hence pair 1 is correctly matched.
• Krishnarajasagar Lake:
o Location: Mysore District, Karnataka. It is built across the River Kaveri and serves as a major water
source for both irrigation and drinking purposes in the region. Hence pair 2 is correctly matched.
• Nagarjuna Sagar Lake:
o Location: Nalgonda District, Telangana. It is formed by the construction of the Nagarjuna Sagar Dam
across the River Krishna. The reservoir is one of the largest man-made lakes in the world and
serves multiple purposes such as irrigation, hydroelectric power generation, and drinking water
supply. Hence pair 3 is not correctly matched
• Idamalayar Lake:
o Location: Ernakulam District, Kerala. It is formed by the Idamalayar Dam, which is constructed
across the Idamalayar River. The reservoir serves as a major water source for the city of
Kochi and is also used for hydroelectric power generation. Hence pair 4 is correctly matched.

Q 43.C
• The Indian experience with Environmental Impact Assessment began over 20 years back. It started in
1976-77 when the Planning Commission asked the Department of Science and Technology to examine the
river-valley projects from an environmental angle.
• Till 1994, environmental clearance from the Central Government was an administrative decision
and lacked legislative support.
• On 27 January 1994, the Union Ministry of Environment and Forests (MEF), Government of India, under
the Environmental (Protection) Act 1986, promulgated an EIA notification making Environmental
Clearance (EC) mandatory for expansion or modernisation of any activity or for setting up new
projects listed in Schedule 1 of the notification. Since then there have been 12 amendments made in the
EIA notification of 1994. Hence, statement 1 is correct.
• Issues with the Indian System
o Screening
✓ Even though some of the industrial set ups do not require EIA as per the statutory norms, they
might involve certain technological processes which could be harmful to the environment, as a
result of which such enlisted industries could have potential impacts on the environment and on
public health.
✓ Exempting industries from the EIA requirements based on the investment value of specific
projects is not acceptable. There are no specific studies conducted till now which
demonstrate that environmental impacts are always inconsequential for projects under a
given value. It is a well established fact that the small scale industries are contributing more
pollution with respect to the major industry. Hence, statement 2 is correct.

16 www.visionias.in ©Vision IAS

FREE BY KING R QUEEN P [ऋषभ राजपूत]


Q 44.C
• Recently, the Allahabad High Court ordered a “scientific survey”, including carbon dating, of a
“Shivling” said to have been found at the Gyanvapi mosque complex in Varanasi.
• Carbon dating is a widely-used method to establish the age of organic materials, things that were once
living. The dating method is based on the fact that Carbon-14 (C-14), an isotope of carbon with an
atomic mass of 14, is radioactive, and decays at a well-known rate. Hence, statement 2 is not correct.
• While C-12 is stable, the radioactive C-14 reduces to one-half of itself in about 5,730 years — known
as its ‘half-life’.
• Though extremely effective, carbon dating cannot be used to determine the age of non-living things
like rocks, for example. Hence, statement 1 is correct.
• Also, the age of things that are more than 40,000-50,000 years old cannot be arrived at through carbon
dating. This is because, after 8-10 cycles of half-lives, the amount of C-14 becomes almost very small and
is almost undetectable.

Q 45.A
• Recently, the Supreme Court has slammed the Kerala government for doing “virtually nothing” for
Endosulfan pesticide exposure victims.
• Endosulfan is an organochlorine insecticide considered to be carcinogen, neurotoxin and genotoxin. It
is linked to a slew of grave medical conditions, such as neurotoxicity, physical deformities, poisoning, etc.
• It was used widely on crops like cashew, cotton, tea, paddy, fruits and others until 2011, when SC banned
its production and distribution.
• Hence, option (a) is the correct answer.

Q 46.C
• Decomposers break down complex organic matter into inorganic substances like carbon dioxide, water,
and nutrients, and the process is called decomposition. Decomposition is largely an oxygen-requiring
process. Warm and moist environments favor decomposition whereas low temperatures and anaerobiosis
inhibit decomposition resulting in the build-up of organic materials. Hence, statement 1 is correct.
• The rate of decomposition is controlled by the chemical composition of detritus and climatic
factors. In a particular climatic condition, the decomposition rate is slower if detritus is rich in lignin
and chitin, and quicker if detritus is rich in nitrogen and water-soluble substances like sugars.
Temperature and soil moisture are the most important climatic factors that regulate decomposition
through their effects on the activities of soil microbes.
• Decomposition involves a series of processes including fragmentation, leaching, catabolism,
humification, and mineralization which act simultaneously on the detritus matter. Detritivores (e.g.,
earthworms) break down detritus into smaller particles. This process is called fragmentation. By the
process of leaching, water-soluble inorganic nutrients go down into the soil horizon and get precipitated as
unavailable salts. Bacterial and fungal enzymes degrade detritus into simpler inorganic substances.
This process is called catabolism. Humification and mineralization occur during decomposition in the
soil. Humification leads to the accumulation of a dark-colored amorphous substance called humus. The
humus is further degraded by some microbes and the release of inorganic nutrients occurs by the
process known as mineralization. Catabolism is essentially the breakdown of complex molecules in
living organisms to form simpler ones, together with the release of energy. Hence, statement 2 is correct.

Q 47.D
• Biogeographic zones represent the major species
groupings. In addition, each of these ten zones
indicates a distinctive set of physical, climatic and
historical conditions. The Himalayas and Gangetic
Plains are examples of two adjacent but obviously
extremely different zones. India has been divided
into ten biogeographic zones.
• The Western Ghats represent one of the major
tropical evergreen forest regions in India. In the
west, the zone is bound by the coast and in the east,
it shares a boundary with the Deccan peninsular
zone. The tropical evergreen forests occupy about
one-third of the total area of this zone.

17 www.visionias.in ©Vision IAS

FREE BY KING R QUEEN P [ऋषभ राजपूत]


• In recent years, a large chunk of the forest cover has been lost and this zone is now of great Student Notes:
conservation concern, more so because of its exceptional biological richness. About two-thirds of India’s
endemic plants are confined to this region. However, the potential of many of these species is yet to be
tapped. Besides harbouring diverse biological communities, the forests in this zone also play an important
role in maintaining the hydrological cycle.
• The well-known species found exclusively in the Western Ghats include the following:
o Flora: The various major vegetation types are tropical evergreen forests, moist deciduous forests, dry
deciduous forests, scrub jungles, sholas, and savannas including high-rainfall savannas, peat bogs, and
Myristica swamps. Four thousand species of flowering plants are known from the Western Ghats.
o Fauna:
✓ Primates – Nilgiri Langur and Lion-tailed Macaque
✓ Squirrels – Several subspecies of Ratufa indica with separate forms in Maharashtra, Mysore,
Malabar, and Tamil Nadu Ghats. The Grizzled Squirrel is restricted to two localities in the
drier Tamil Nadu forest.
✓ Carnivores – Malabar Civet in southern evergreen forests, Rusty-spotted Cat in northern
deciduous forests.
✓ Ungulates – Nilgiri Tahr in Nilgiris to Agastyamalai Montane grassland.
✓ Hornbills – Malabar Grey Hornbill
• Hence option (d) is the correct answer.

Q 48.B
• Recent Context: The Centre has taken control of security in Manipur (by allegedly invoking Article 355),
deploying 12 companies of the Border Security Force (BSF) and airlifting anti-riot vehicles to the
northeastern state.
o Definition: Article 355 empowers the Union government to protect every state in India against
external aggression and internal disturbances.
o Article 356 Vs Article 355: Article 356 empowers the President to impose President’s Rule in a state
in case of a failure or breakdown of constitutional machinery, while Article 355 empowers the Union
government:
✓ To protect every state in India against external aggression and internal disturbances
✓ To issue directions to any state to ensure compliance with the Union’s laws and regulations.
Hence only statements 1 and 2 are correct.
o It is part of Part XVIII of the Indian Constitution, titled “Emergency Provisions”. It is based on the
principle of “duty to protect” enshrined in the Constitution.
o Restrictions: Directions can only be given when there is a failure of the state machinery to comply
with or give effect to any Union law or regulation; should be of an urgent nature and may not extend
beyond the necessary period for remedying the failure; state government should be given an
opportunity to submit its views before issuance.
o Its duration is not specified in the Constitution.
o It can be withdrawn by the Union government when the situation is normalized or the state
government requests it to do so.
o Circumstances of imposition: Failure of the state to comply with the Union’s directions; threat to the
security of India; threat to unity and integrity of India due to violent activities by any group or
organization; request for assistance from the Union to maintain public order when the situation in the
state cannot be controlled by the state’s own forces.
o Scope of judicial review: The satisfaction of the President in invoking Article 355 is subject to
judicial review and can be challenged in court if it violates any fundamental rights or constitutional
provisions.

Q 49.D
• Various sources produce various different types of pollutants.
• Vehicular exhaust can release Ozone due to the combustion of Fossil Fuels and improper cleaning.
Hence pair 1 is correctly matched.
• Plutonium is a byproduct of a fission nuclear reaction. Hence pair 2 is correctly matched.
• Mercury is used in the dental industry for amalgam and in thermometers. Hence pair 3 is correctly
matched.
• Cadmium is one of the major pollutants generated from the Electronics and Electric industry. Hence
pair 4 is correctly matched.

18 www.visionias.in ©Vision IAS

FREE BY KING R QUEEN P [ऋषभ राजपूत]


Q 50.A
• Recently Government of India exercised the Green Shoe option in Coal India Offer For Sale (OFS)
after 346% oversubscription.
• What is Green Shoe Option?
o A green shoe option Is nothing but a clause contained in the underwriting agreement of an Initial
Public Offer (IPO).
✓ An initial public offering IPO or stock launch is a public offering in which shares of a
company are sold to institutional investors and usually also to retail investors.
✓ Under a green shoe option, the issuing company has the option to allocate additional equity
shares up to a specified amount.
o This option permits the underwriters to buy up to an additional 15% of the shares at the offer price if
public demand for the shares exceeds expectations and the share trades above its offering price. -
Green shoe option is also known as an over-allotment provision. - This option is primarily used at the
time of IPO or listing of any stock to ensure a successful opening price.
o From the investor's point of view, an IPO with a green shoe option ensures that after listing the share
price will not fall below its offer price.
o The green shoe option acts as a price stabilising mechanism.
• Hence option (a) is the correct answer.

Q 51.A
• Bioprospecting is the exploration of plant and animal species for the utilization of their genetic
resources in pharmaceutical and biochemical industries and in the production of a wide array of
commercially viable products. Hence option (a) is the correct answer.
• It led to novel treatments for malaria, tuberculosis, HIV/AIDS, and cancer, highlighting yet again the
interdependence between ecosystems and human health and wellbeing.
• Bioprospecting is based on a sustainable approach to bringing economic and social benefits to often poor
communities who otherwise would resort to unsustainable use of land, consumption of environmental
resources or other negative coping mechanisms such as economic migration.
• Additional information:
o Biopiracy is defined as the unauthorized appropriation of knowledge and genetic resources of farming
and indigenous communities by individuals or institutions seeking exclusive monopoly control
through patents or intellectual property.
o Biosparging is an in situ remediation technique that supplies oxygen and nutrients (if needed) to
contaminated soils to promote aerobic biodegradation of contaminants (i.e. petroleum hydrocarbons)
by indigenous microorganisms.
o A biopile is a type of ex-situ treatment that uses biological processes to transform pollutants into low-
toxic byproducts. It is often used to lower petroleum component concentrations in soils by utilizing
the bioremediation process. Biopiles are a type of remediation system that is used for a short period.

Q 52.D
• The steel industry is one of the major contributors to air pollution in India. The pollutants released by the
steel industry can have serious health and environmental impacts. Some of the important pollutants
released by the steel industry in India are:
o Oxides of sulfur: The burning of coal and coke in the steel industry releases oxides of sulfur into the
air. These pollutants can cause respiratory problems and acid rain.
o Particulate Matter: The combustion of coal releases Particulate Matter including PM 10, PM 5, and
PM 2.5.
o Oxides of nitrogen: The high-temperature processes used in the steel industry pollutants can
contribute to smog and respiratory problems.
o Carbon monoxide: The incomplete burning of coal and coke in the steel industry can release carbon
monoxide into the air. This pollutant is toxic and can cause headaches, dizziness, and even death.
o Carbon dioxide: The steel industry is a major contributor to greenhouse gas emissions, releasing
large amounts of carbon dioxide into the atmosphere. This pollutant contributes to climate change and
global warming.
• All of the above pollutants are released by the steel industry in India, making it a major contributor to air
pollution and climate change. It thus needs to adopt cleaner production processes and reduce its emissions
to protect public health and the environment.
• Hence option (d) is the correct answer.

19 www.visionias.in ©Vision IAS

FREE BY KING R QUEEN P [ऋषभ राजपूत]


Q 53.B
• National Adaptation Fund on Climate Change (NAFCC) was launched in 2015 with an initial outlay
of Rs. 350 crores to meet the cost of adaptation to climate change for the State and Union
Territories of India that are particularly vulnerable to the adverse effects of climate change. Hence
statement 1 is not correct.
• The overall aim of the fund is to support concrete adaptation activities which are not covered under
ongoing activities through the schemes of the State and National Governments that reduce the adverse
effects of climate change facing communities, sectors, and states. The Scheme has been taken as
Central Sector Scheme with National Bank for Agriculture and Rural Development (NABARD) as
the National Implementing Entity (NIE). Hence statement 2 is correct.
• Objectives of NAFCC include:
o Funding concrete adaptation projects/programs aligned with the relevant Missions under National
Action Plan on Climate Change (NAPCC) and the State Action on Climate Change (SAPCCs) in
agriculture, horticulture, agro-forestry, environment, allied activities, water, forestry, urban, coastal
and low-lying system, disaster management, human health, marine system, tourism, habitat sector and
other rural livelihood sectors to address climate change related issues.
o Preparing and updating climate scenarios, assessing vulnerability, and climate impact assessment
o Capacity building of various stakeholders on climate change adaptation and project cycle
management and developing a knowledge networkMainstreaming the approaches/ learnings from
project/program implementation through knowledge Management

Q 54.A
• Recent Context: Researchers at the Pune-based Agharkar Research Institute (ARI) have discovered a
new species of single-cell algae, known as diatoms, from the northern Western Ghats — and named it
after a veteran geo-archaeologist from the city, the late Professor S N Rajaguru.
• About Diatoms:
o Diatoms can be found in freshwater and brackish water, and are commonly traced in lakes, rivers,
streams, rivulets and the sea.
o Similar to plants and trees that produce oxygen during the process called photosynthesis, diatoms,
too, supply oxygen to the atmosphere.
• About Professor S N Rajaguru.
o Rajaguru was a former faculty member of Deccan College and known for his attempts to link
biological and archaeological remains during excavations.
o It was through Rajaguru’s extensive studies that researchers today have a clear time-frame of
India’s paleo-environment.
• Hence option (a) is the correct answer.

Q 55.A
• Although ozone depletion is occurring widely in the stratosphere, the depletion is particularly marked
over the Antarctic region. This has resulted in the formation of a large area of the thinned ozone layer,
commonly called the ozone hole. It is only under certain meteorological conditions that ozone holes form.
The conditions required to form the ozone hole are:
o Cold temperatures during the polar winter
o Ice cloud formation
o Special meteorological conditions to form the polar vortex
o Followed by the polar sunrise in the spring
• The ozone hole occurs during the Antarctic spring, from September to early December, as strong westerly
winds start to circulate around the continent and create an atmospheric container. Within this polar vortex,
over 50% of the lower stratospheric ozone is destroyed during the Antarctic spring.
• Reactions that take place on polar stratospheric clouds (PSCs) dramatically enhance ozone
depletion. PSCs form more readily in the extreme cold of the Arctic and Antarctic stratosphere.
Sunlight-less polar winters contribute to a decrease in temperature and the polar vortex traps and
chills air. These low temperatures form cloud particles. These clouds provide surfaces for chemical
reactions whose products will, in the spring lead to ozone Destruction. Hence, statement 1 is correct.
• The role of sunlight in ozone depletion is the reason why the Antarctic ozone depletion is greatest
during spring. During winter, even though PSCs are at their most abundant, there is no light over the
pole to drive chemical reactions. During the spring, however, the sun comes out, providing energy to drive
photochemical reactions and melt the polar stratospheric clouds, releasing considerable ClO, which drives
the whole mechanism. Warming temperatures near the end of spring break up the vortex around mid-
20 www.visionias.in ©Vision IAS

FREE BY KING R QUEEN P [ऋषभ राजपूत]


December. As warm, ozone and NO2-rich air flows in from lower latitudes, the PSCs are destroyed, the
enhanced ozone depletion process shuts down, and the ozone hole closes. Hence, statement 2 is not
correct.

Q 56.C
• The grazing food chain begins with autotrophs. In this, energy and nutrients move from plants to the
herbivores consuming them, and to the carnivores or omnivores preying upon the herbivores. The
detritus food chain (DFC) begins with dead organic matter. Hence, statement 1 is not correct.
• In an aquatic ecosystem, Grazing Food Chain (GFC) is the major conduit for energy flow. As against
this, in a terrestrial ecosystem, a much larger fraction of energy flows through the detritus food chain than
through the GFC. The Detritus food chain may be connected with the grazing food chain at some
levels: some of the organisms of DFC are prey to the GFC animals. Hence, statement 2 is not
correct.
• In a natural ecosystem, some animals like cockroaches, crows, etc., are omnivores. An omnivore is an
animal that has the ability to eat and survive on both plant and animal matter. Thus, Omnivores can
occupy two trophic levels in the food chain first as primary consumer feeding on grass/plants and as
carnivores feeding on insects.
• The detritus food chain (DFC) is made up of decomposers which are heterotrophic organisms,
mainly fungi, and bacteria. They meet their energy and nutrient requirements by degrading dead organic
matter or detritus. These are also known as saprotrophs (sapro: to decompose). Decomposers secrete
digestive enzymes that break down dead and waste materials into simple, inorganic materials, which are
subsequently absorbed by them. Hence, statement 3 is correct.

Q 57.C
• The main source of halogen atoms in the stratosphere that break ozone molecules is the photo-
dissociation of man-made halocarbon refrigerants, solvents, propellants, and foam-blowing agents
(CFCs, HCFCs, freons, halons) – popularly called as ODS (Ozone Depleting Substance). Apart from
these, Carbon tetrachloride, Methyl Bromide, and Methyl Chloroform are also potent ozone-
depleting substances.
• These compounds are transported into the stratosphere by winds after being emitted at the surface. UV
rays act on them releasing Chlorine atoms. Cl degrades ozone releasing molecular oxygen, with these
atoms acting merely as catalysts. Thus Cl atoms are not consumed in the reaction. Hence, whatever CFCs
are added to the stratosphere, they have permanent and continuing effects on Ozone levels.
• Hydrofluorocarbons, though included in the Montreal Protocol for curbing Ozone-depleting
substances, are not Ozone-depleting substances. It is a potent greenhouse gas. Hence, option (c) is
the correct answer.

Q 58.D
• Carbon sequestration – the practice of removing carbon from the atmosphere and storing it – is one
of the many approaches being taken to tackle climate change.
• It can be done in the following ways:
o Afforestation: Plants remove carbon dioxide from the air naturally, and trees are especially good at
storing CO2 removed from the atmosphere by photosynthesis. Expanding, restoring and managing
tree cover to encourage more carbon uptake can leverage the power of photosynthesis, converting
carbon dioxide in the air into carbon stored in wood and soils.
o Wetland Restoration: Wetlands are vital natural assets, capable of taking up atmospheric carbon and
restricting subsequent carbon loss to facilitate long-term storage. They can be deliberately managed to
provide a natural solution to mitigate climate change, as well as to help offset direct losses of
wetlands from various land-use changes and natural drivers.
o Sustainable Agriculture: Soils naturally sequester carbon, but agricultural soils are running a big
deficit due to frequent plowing and erosion from farming and grazing, all of which release stored
carbon.
o Growing Seaweed: Kelp is a type of seaweed that grows in the ocean. globally, seaweeds (including
kelp) are thought to sequester nearly 200 million tonnes of carbon dioxide every year
o Biochar: Biochar produced by pyrolysis of biowaste. It can be used as a landfill and increase soil
fertility.
o Subterranean injection: It involves injecting CO2 into depleted oil and gas reservoirs and other
geological features, or into the deep ocean.

21 www.visionias.in ©Vision IAS

FREE BY KING R QUEEN P [ऋषभ राजपूत]


o Iron Fertilization: Iron fertilization of Oceans encourages the growth of plankton and thus helps in
capturing CO2.
o Hence, option (d) is the correct answer.

Q 59.B
• Recently, the Ministry of Minority Affairs (MoMA) has approved a grant under Pradhan Mantri Jan
Vikas Karyakram (PMJVK) to support the upgradation of Unani Medicine facilities at Hyderabad,
Chennai, Lucknow, Silchar, and Bengaluru.
• Pradhan Mantri Jan Vikas Karyakram (PMJVK) is a Centrally Sponsored Scheme (CSS). Hence,
statement 1 is not correct.
• It is being implemented by the Ministry of Minority Affairs, with the objective to develop infrastructure
projects in identified areas, for the socio-economic development of said areas for the minority
communities. The priority sectors under PMJVK are education, health, skill development, women-
centric projects etc. Hence statement 2 is correct.
• Projects under PMJVK are implemented and managed by the concerned State/ UT Government.
Hence, statement 3 is correct.
• The proposals under PMJVK are sent by the States/Union Territories (UTs) as per demand for
infrastructure in the identified areas, which are considered and approved by the Empowered Committee
(EC) of PMJVK, after due consultation with the concerned Central Ministries.

Q 60.A
• The Central Pollution Control Board (CPCB) is responsible for monitoring India’s Air and Water
Quality and pollution-related issues. It is a statutory organization under the Ministry of
Environment Forest and Climate Change. Hence, statement 1 is correct.
• The Board was established in September 1974 under the Water (Prevention and Control Pollution)
Act 1974 (Not Air Act). Hence, statement 2 is not correct.
• Later on, it was entrusted with the functions and powers of the Air (Prevention and Control
Pollution) Act 1981. Hence it also aims to enhance air quality and prevent or control air pollution apart
from the prevention and control of water pollution in India.

Q 61.C
• Recent Context: In recent years, debt-for-climate swaps have gained popularity among low- and middle-
income countries as a debt-relief tool. Multilateral development banks and organizations like the UNDP
have been promoting this approach to ease the burden of debt for these nations.
• Debt-for-climate swaps are a financial approach that helps alleviate this problem by freeing up
funds for climate investments. They are a form of debt relief that transforms debt into a grant
committed to undertaking climate-related investments. Hence statement-I is correct.
o Both commercial and official bilateral debts can be included in such swaps, with the latter allowing
for redirected debt service payments towards projects mutually agreed upon by both parties, such as
those related to climate action.
• Debt-for-climate swaps originated from debt-for-nature swaps, which were introduced in the 1980s
to promote biodiversity conservation and tropical forest protection in exchange for debt relief.
o Bolivia and Conservation International executed the first debt-for-nature swap in 1987. Debt-for-
climate swaps became a more extensive concept in the 2000s, encompassing not only nature
conservation but also climate change mitigation and adaptation.
o In 2006, Germany and Indonesia implemented the first debt-for-climate swap, with the latter
pledging to reduce greenhouse gas emissions through REDD+ (Reducing Emissions from
Deforestation and Forest Degradation) initiatives in return for debt relief.
• Debt-for-climate swaps provide benefits for both creditors and debtors. Creditors can advance their
development cooperation and climate finance goals, improve their chances of debt recovery, and
strengthen their diplomatic ties with debtor nations
• Steps in Debt-for-climate swaps:
o Suppose a developing country is indebted to a bilateral creditor that wants to offer debt relief and
encourage the developing country to pursue climate-friendly policies or projects.
o The creditor agrees that the developing country no longer needs to service its debt.
o In return, the developing country agrees with the lender either (i) to spend the money that would have
been spent on debt service on climate-friendly projects or (ii) to adopt climate-friendly policies.

22 www.visionias.in ©Vision IAS

FREE BY KING R QUEEN P [ऋषभ राजपूत]


• Benefits of such agreements.
o International organisations and multilateral development banks exercise multiple types of measures to
help highly vulnerable countries survive financial catastrophes caused by climate change impacts.
o However, in the past decade, debt-for-climate swaps has grown relatively popular among low- and
middle-income countries.
o They offer an innovative way to make climate investments while creating the much-needed fiscal
space.
o At the same time, they support climate investment by committing a country to swing their spending
from debt service to an agreed public investment. Hence statement II is not correct.

Q 62.A
• The Asian Development Bank unveiled a new flagship initiative called Innovative Finance Facility for
Climate in Asia and the Pacific (IF-CAP). Hence option (a) is the correct answer.
o The IF-CAP is a multi-donor facility that provides guarantees for parts of the Bank’s sovereign
portfolios to enable it to free up capital and increase loans for climate investment.
• Features of Innovative Finance Facility:
o IF-CAP's initial partners are Denmark, Japan, the Republic of Korea, Sweden, the United Kingdom,
and the United States.
o The reduced risk exposure created by the guarantees will allow ADB to free up capital to
accelerate new loans for climate projects.
o With a model of ‘$1 in, $5 out’, the initial ambition of $3 billion in guarantees could create up to $15
billion in new loans for much-needed climate projects across Asia and the Pacific.
o A leveraged guarantee mechanism for climate finance has never before been adopted by a
multilateral development bank.

Q 63.B
• The National Green Tribunal has been established under the National Green Tribunal Act 2010 for
effective and expeditious disposal of cases relating to environmental protection and conservation of
forests and other natural resources including enforcement of any legal right relating to the environment
and giving relief and compensation for damages to persons and property and for matters connected
therewith or incidental thereto. Hence, statement 1 is not correct.
• The Tribunal is mandated to make and endeavour for disposal of applications or appeals finally within 6
months of the filing of the same.
• NGT has not been vested with powers to hear any matter relating to the Wildlife (Protection) Act, 1972,
the Indian Forest Act, 1927 and various laws enacted by States relating to forests, tree preservation, etc.
• The NGT is not bound by the procedure laid down under the Code of Civil Procedure, 1908, but
shall be guided by principles of natural justice. Hence, statement 2 is correct.
• Further, NGT is also not bound by the rules of evidence as enshrined in the Indian Evidence Act, 1872.
Thus, it will be relatively easier (as opposed to approaching a court) for conservation groups to present
facts and issues before the NGT, including pointing out technical flaws in a project or proposing
alternatives that could minimize environmental damage but which have not been considered.
• While passing Orders/decisions/awards, the NGT will apply the principles of sustainable development, the
precautionary principle and the polluter pays principles.
• However, it must be noted that if the NGT holds that a claim is false, it can impose costs including lost
benefits due to any interim injunction.

Q 64.C
• Initially, nuclear energy was hailed as a non-polluting way for generating electricity. Later on, it was
realised that the use of nuclear energy has two very serious inherent problems. The first is accidental
leakage, as occurred in the Three Mile Island and Chernobyl incidents and the second is safe disposal
of radioactive wastes.
• Three Mile Island, Pennsylvania, United States
o Loss of coolant and partial core meltdown due to operator errors and technical flaws. There was a
small release of radioactive gases.
• Chernobyl, Kiev Oblast, Ukraininan SSR, Soviet Union A flawed reactor design and inadequate safety
procedures led to a power surge that damaged the fuel rods of reactor no. 4 of the Chernobyl power plant.
This caused an explosion and meltdown, necessitating the evacuation of 300,000 people and dispersing
radioactive material across Europe.

23 www.visionias.in ©Vision IAS

FREE BY KING R QUEEN P [ऋषभ राजपूत]


• Fukushima, Japan The Fukushima nuclear disaster was triggered by a tsunami that flooded and
damaged the 3 active reactors at the Fukushima Daiichi nuclear power plant, drowning two workers. Loss
of backup electrical power led to overheating, meltdowns, and evacuations.
• Deepwater Horizon oil spill, also called Gulf of Mexico oil spill
o It is largest marine oil spill in history, caused by an April 20, 2010, explosion on the Deepwater
Horizon oil rig—located in the Gulf of Mexico, approximately 41 miles (66 km) off the coast of
Louisiana—and its subsequent sinking.
• Hence, option (c) is the correct answer.

Q 65.A
• Increased carbon dioxide in the atmosphere is one of the recognized causes of our changing climate
and it is also problematic for coral reefs. The ocean absorbs approximately one-third of the
atmosphere’s excess carbon dioxide, resulting in a more acidic ocean. In order for a coral reef to grow, it
must produce limestone (or calcium carbonate) at a rate that is faster than the reef is being eroded. Ocean
acidification slows the rate at which coral reefs generate calcium carbonate, thus slowing the growth
of coral skeletons.
• Climate change can cause sea level rise; changes in the frequency, intensity, and distribution of tropical
storms; and altered ocean circulation. All of these impacts can have negative consequences for the health
and diversity of reefs around the world. Hence, statement-I and statement II are correct. Also,
statement II is the correct explanation for statement-I.
• A 20 percent increase above current carbon dioxide levels, which could occur within the next two
decades, could significantly reduce the ability of corals to build their skeletons and some could become
functionally extinct within this timeframe.
• In real terms, this does not just mean corals grow more slowly, but also that they will be less able to
overcome typical pressures. Tropical coral reefs are constantly engaging in a battle to grow. Many
reef dwellers actually break apart pieces of the corals’ skeletons, either to feed upon or to create homes.

Q 66.B
• Recently, a new small savings scheme called Mahila Samman Savings Certificate was launched
specifically for female investors to promote investment among women. Accounts opened under this
scheme will be single-holder accounts that can be opened at the Post Office or any registered bank.
• Any woman can open a Mahila Samman account that can be opened for herself or on behalf of a minor
girl. Hence, statement 1 is correct.
• An account can be opened with a one-time deposit of a minimum of Rs. 1,000 and any sum in multiples
of Rs. 100 in multiple accounts. However, the aggregate deposit amount across all accounts cannot
exceed Rs. 2 lakhs for an individual. Hence, statement 2 is not correct.
• The Government has not mentioned any tax benefits for the Mahila Samman Savings Certificate.
Hence, statement 3 is not correct.
• An interest of 7.5% p.a. will be paid on deposits under this scheme. The interest will be compounded
quarterly and credited to the account.
• An account holder can make a partial withdrawal from the account of up to 40% of the eligible
balance after the expiry of one year from the date of account opening. Hence, statement 4 is correct.

Q 67.C
• The Kyoto Protocol is an agreement made under the UNFCCC. Countries that ratify this protocol
commit to reduce their emissions of carbon dioxide and five other greenhouse gases (Methane,
Nitrous Oxide, Sulphur Hexafluoride, Hydrofluorocarbons and Perfluorocarbons), or engage in
emissions trading if they maintain or increase emissions of these gases.
• Under the protocol, Governments are separated into two general categories: countries among the
developed nations, referred to as Annex 1 countries (who have accepted GHG emission reduction
obligations and must submit an annual greenhouse gas inventory); and countries among developing or
least developed nations, referred to as Non-Annex 1 countries (who have no GHG emission reduction
obligations but may participate in the Clean Development Mechanism).
• The Kyoto Protocol introduced three mechanisms in order to achieve its goals: the Clean
Development Mechanism (CDM), Joint Implementation (JI), and Emissions Trading (ET).
o The CDM allows countries with commitments under the Kyoto Protocol to invest in emission-
reduction projects in developing countries. These projects can involve, for example, a rural
electrification project using solar panels or the installation of more energy-efficient boilers. Hence
option 1 is correct.
24 www.visionias.in ©Vision IAS

FREE BY KING R QUEEN P [ऋषभ राजपूत]


o JI mechanism allows a country with a Kyoto Protocol emission reduction target to invest in a project
to reduce emissions in any other country with a commitment (as opposed to a developing country).
Hence option 2 is correct.
o The Emissions Trading (The International Emission Trading) scheme under the Kyoto Protocol set up
a platform where carbon units, or units generated by projects registered under the JI or the CDM, or
from removals through forestry activities, can be exchanged, i.e. sold and purchased, according to a
country’s needs. The scheme made carbon a commodity and created a carbon market. Hence option 4
is correct.
• Nationally determined contributions (NDCs) are at the heart of the Paris Agreement and the
achievement of its long-term goals. NDCs embody efforts by each country to reduce national
emissions and adapt to the impacts of climate change. Article 4 of the Paris Agreement requires
each party to prepare, communicate and maintain successive nationally determined contributions
(NDCs) that it intends to achieve. Parties shall pursue domestic mitigation measures, with the aim of
achieving the objectives of such contributions. Hence, INDCs are not the market mechanisms under the
Kyoto Protocol. Hence option 3 is not correct.

Q 68.B
• The size of a population for any species is not a static parameter. It keeps changing in time, depending on
various factors including food availability, predation pressure, and adverse weather. The density of a
population in a given habitat during a given period fluctuates due to changes in four basic processes, two
of which (natality and immigration) contribute to an increase in population density and two
(mortality and emigration) to a decrease.
• Natality refers to the number of births during a given period in the population that are added to the
initial density. Hence option 1 is correct.
• Mortality is the number of deaths in the population during a given period. Hence option 2 is not
correct.
• Immigration is the number of individuals of the same species that have come into the habitat from
elsewhere during the time period under consideration. Hence option 3 is correct.
• Emigration is the number of individuals in the population who left the habitat and went elsewhere during
the time period under consideration.

Q 69.A
• In September 2015, the heads of nations agreed to set the world on a path toward sustainable development
through the adoption of the 2030 Agenda for Sustainable Development. This agenda includes 17
Sustainable Development Goals, or SDGs, which set out quantitative objectives across the social,
economic, and environmental dimensions of sustainable development all to be achieved by 2030.
• The 17 SDGs are integrated—they recognize that action in one area will affect outcomes in others, and
that development must balance social, economic, and environmental sustainability. There are 169 targets
under the 17 goals.
• Image showing 17 SDGs

• Thus, only pair 4 is correctly matched. Hence option (a) is the correct answer.

25 www.visionias.in ©Vision IAS

FREE BY KING R QUEEN P [ऋषभ राजपूत]


Q 70.B
• “Biodiversity Heritage Sites” (BHS) are well-defined areas that are unique, ecologically fragile
ecosystems - terrestrial, coastal and inland waters and, marine having rich biodiversity comprising of any
one or more of the following components:
o the richness of wild as well as domesticated species or intra-specific categories,
o high endemism,
o presence of rare and threatened species, keystone species, species of evolutionary significance, wild
ancestors of domestic/cultivated species or their varieties,
o past pre-eminence of biological components represented by fossil beds and having significant cultural,
ethical, or aesthetic values are important for the maintenance of cultural diversity, with or without a
long history of human association with them.
• Under Biological Diversity Act, 2002 (BDA) the State Government in consultation with local bodies
may notify the official gazette, of areas of biodiversity importance as Biodiversity Heritage Sites
(BHS). Hence, statement 1 is not correct.
• State Government in consultation with the Central Government may frame rules for the management and
conservation of BHS.
• State Governments shall frame schemes for compensating or rehabilitating any person or section of people
economically affected by such notification.
• Nallur Tamarind Grove in Bengaluru, Karnataka was the first Biodiversity Heritage Site of India,
declared in 2007. Hence, statement 2 is correct.

Q 71.A
• An ecological pyramid is a graphical representation of the relationship between different organisms in an
ecosystem.
• Types of Ecological Pyramids:
• The pyramid of Numbers is the graphic representation of the number of individuals per unit area of
various trophic levels.
o Upright Pyramid of Numbers: In an upright pyramid of numbers, the number of individuals
decreases from the lower level to the higher level. This type of pyramid is usually found in the
grassland ecosystem and the pond ecosystem. The grass in a grassland ecosystem occupies the lowest
trophic level because of its abundance.
o Inverted Pyramid of Numbers: Here, the number of individuals increases from the lower level to
the higher trophic level. For example, the tree ecosystem. Thus, the Pyramid of Numbers can be
inverted and Upright. Hence option 1 is not correct.
• Pyramid of Biomass as the name suggests shows the amount of biomass (living or organic matter
present in an organism) present per unit area at each trophic level.
o Upright Pyramid of Biomass: Ecosystems found on land mostly have pyramids of biomass with
large bases of primary producers with smaller trophic levels perched on top, hence the upright
pyramid of biomass.
o Inverted Pyramid of Biomass: A reverse pyramidal structure is found in most aquatic ecosystems.
Here, the pyramid of biomass may assume an inverted pattern. This is because, in a water body, the
producers are tiny phytoplankton that grow and reproduce rapidly. In this condition, the pyramid of
biomass has a small base, with the producer biomass at the base providing support to consumer
biomass of large weight. Hence, it assumes an inverted shape. The pyramid of Biomass can be
inverted and Upright. Hence, option 2 is not correct.
• The pyramid of Energy is a graphical structure representing the flow of energy through each trophic
level of a food chain over a fixed part of the natural environment. An energy pyramid represents the
amount of energy at each trophic level and the loss of energy at each is transferred to another trophic
level.
• The energy pyramid, sometimes called the trophic pyramid or ecological pyramid, is used in quantifying
the energy transfer from one organism to another along the food chain. Energy decreases as one moves
through the trophic levels from the bottom to the top of the pyramid. Thus, the energy pyramid is
always upright. Hence, option 3 is correct.

Q 72.B
• The movement of nutrient elements through the various components of an ecosystem is called
nutrient cycling. Another name for nutrient cycling is biogeochemical cycles (bio: living organism, geo:
rocks, air, water). Nutrient cycles are of two types: (a) gaseous and (b) sedimentary.

26 www.visionias.in ©Vision IAS

FREE BY KING R QUEEN P [ऋषभ राजपूत]


• The reservoir for the gaseous type of nutrient cycle (e.g., nitrogen, carbon cycle) exists in the
atmosphere and for the sedimentary cycle (e.g., sulphur and phosphorus cycle), the reservoir is
located in Earth’s crust. Environmental factors, e.g., soil, moisture, pH, temperature, etc., regulate the
rate of release of nutrients into the atmosphere. The function of the reservoir is to meet the deficit which
occurs due to an imbalance in the rate of influx and efflux. Hence, option (b) is the correct answer.
• Carbon Cycle: Carbon enters the living world in the form of carbon dioxide through the process
of photosynthesis as carbohydrates. These organic compounds (food) are then passed from the
producers to the consumers (herbivores & carnivores). This carbon is finally returned to the surrounding
medium by the process of respiration or decomposition of plants and animals by the decomposers.
• Nitrogen Cycle: Nitrogen is present in the atmosphere in an elemental form and as such it cannot be
utilized by living organisms. This elemental form of nitrogen is converted into a combined state with
elements such as H, C, and O by certain bacteria so that it can be readily used by plants. Nitrogen is
continuously expelled into the air by the action of microorganisms such as denitrifying bacteria and
finally returned to the cycle through the action of lightening and electrification.
• Sulphur Cycle: Sulphur occurs in all living matter as a component of certain amino acids. It is
abundant in the soil in proteins and, through a series of microbial transformations, ends up as sulphates
usable by plants. Sulphur-containing proteins are degraded into their constituent amino acids by the action
of a variety of soil organisms. The sulphur of the amino acids is converted to hydrogen sulphide (H2S) by
another series of soil microbes. In the presence of oxygen, H2S is converted to sulphur and then to
sulphate by sulphur bacteria. Eventually, the sulphate becomes H2S.
• Phosphorus cycle: Phosphorus is a major constituent of biological membranes, nucleic acids and
cellular energy transfer systems. Many animals also need large quantities of this element to make shells,
bones and teeth. The natural reservoir of phosphorus is rock, which contains phosphorus in the form of
phosphates. When rocks are weathered, minute amounts of these phosphates dissolve in soil solution and
are absorbed by the roots of the plants. Herbivores and other animals obtain this element from plants. The
waste products and the dead organisms are decomposed by phosphate-solubilising bacteria releasing
phosphorus. Unlike the carbon cycle, there is no respiratory release of phosphorus into the atmosphere.

Q 73.C
• A mass extinction is a short period of geological time in which a high percentage of biodiversity, or
distinct species—bacteria, fungi, plants, mammals, birds, reptiles, amphibians, fish, invertebrates—dies
out. In this definition, it’s important to note that, in geological time, a ‘short’ period can span thousands or
even millions of years.
• The planet has experienced five previous mass extinction events, the last one occurring 65.5 million years
ago which wiped out the dinosaurs from existence. Experts now believe we are in the midst of a sixth
mass extinction. Hence statement 1 is correct.
• The ‘Sixth Extinction’ is presently in progress different from the previous episodes. The difference is in
the rates; the current species extinction rates are estimated to be 100 to 1,000 times faster than in
pre-human times and our activities are responsible for the faster rates. Ecologists warn that if the
present trends continue, nearly half of all the species on Earth might be wiped out within the next 100
years. Hence statement 2 is correct.
• Unlike previous extinction events caused by natural phenomena, the sixth mass extinction is driven by
human activity, primarily (though not limited to) the unsustainable use of land, water and energy use, and
climate change.

Q 74.A
• Greenhouse Gases (GHGs) are the gases that lead to the warming of the Earth. Even though these
are naturally present in the Earth's atmosphere, their quantity is increasing exponentially due to
the burning of Fossil Fuels, Crop/Biomass Residue, etc.
• The following are examples of GHGs-
o Carbon Monoxide- Carbon monoxide (CO) is a colorless, odorless, and tasteless gas.- It is formed
as a result of incomplete combustion of carbon-containing materials.
o Methane- Methane (CH4) is a potent greenhouse gas that contributes to global warming.- Methane
is produced by the anaerobic decomposition of organic matter.
o Carbon Dioxide: Carbon dioxide (CO2) is a colorless, odorless, and tasteless gas.- It is formed as a
natural outcome of combustion of carbon-containing materials.
o Hydrofluorocarbons: Potent greenhouse gases (GHG) that have global warming potentials that range
from hundreds to thousands of times that of carbon dioxide.

27 www.visionias.in ©Vision IAS

FREE BY KING R QUEEN P [ऋषभ राजपूत]


o Nitrous oxide molecules stay in the atmosphere for an average of 121 years before being removed by
a sink or destroyed through chemical reactions.
• Hence option (a) is the correct answer.

Q 75.C
International Organizations for Control of Wildlife Trafficking:
• TRAFFIC: Trade Records Analysis of Flora and Fauna in International Commerce (TRAFFIC)
established in 1976, is a wildlife trade monitoring network and a joint programme of WWF and IUCN.
Hence statement 1 is correct.
o It works closely with the National and State Governments and various agencies to help study, monitor
and influence action to curb illegal wildlife trade and bring wildlife trade within sustainable levels.
• CITES (the Washington Convention): CITES signed in 1973 provides a mechanism to regulate the
trade in wildlife. Under its guidance, governments all over the world have taken steps to prevent this
illegal trade and bring it under control.
o The CITES is concerned with the international co-operation to control only the illegal trade in
endangered species whereas the TRAFFIC is concerned with the monitoring of both the legal as
well as the illegal trade in wildlife across the world. Hence statement 2 is correct.

Q 76.B
• Recent Context: In Monetary Policy Committee's (MPC) latest policy review the RBI decided to
maintain the status quo. In other words, it changed nothing. This gave the economists to think
whether India has reached its Goldilocks Scenario.
• What Is a Goldilocks Economy?
o A Goldilocks economy is not too hot or too cold but just right ie. describing situations that are "just
right" amid two extremes —to steal a line from the popular children's story Goldilocks and the Three
Bears.
o The term describes an ideal state for an economic system. In this perfect state, there is full
employment, economic stability, and stable growth. The economy is not expanding or contracting
by a large margin. Hence statement (b) is the correct answer.
o A Goldilocks state is also ideal for investing because as companies grow and generate positive
earnings growth, stocks perform well.
o Goldilocks economies are temporary in nature, as seen by the boom and bust cycles.
• Inflation in Goldilocks Economy:
o A Goldilocks economy has steady economic growth, preventing a recession, but not so much growth
that inflation rises by too much.
o The inflation in such a scenario will neither be too hot (implying high inflation) nor too cold
(referring to faltering GDP growth).
o It implies idle inflation i.e. not too low nor too high.

Q 77.B
• The amount of pollution in a water body can be quantified by the amount of oxygen required to
decompose the pollutants. For this, two parameters are considered- The Biological Oxygen Demand and
the Chemical Oxygen Demand.
• Chemical oxygen demand or COD is the amount of oxygen required to break down organic and
inorganic material via oxidation. Hence option (b) is the correct answer.
• It is frequently stated as the mass of oxygen consumed over the volume of the solution, expressed in
milligrams per liter (mg/L), in SI units. Quantifying the number of oxidizable contaminants present in
surface water (such as lakes and rivers) or wastewater is the most typical application of COD. Chemical
Oxygen Demand is helpful for assessing the quality of water by providing a metric to assess how an
effluent will affect the receiving body.

Q 78.C
• Wildlife Sanctuary: A Sanctuary is a protected area where wild animals and birds are kept and
encouraged to increase their population. Presently, there are more than 551 sanctuaries in India covering a
total area of 1,19,775.80 sq km.
• In wildlife sanctuaries, rare and endangered species are encouraged to breed in human controlled
environments with restricted settings. This is called as captive breeding. This is a successful technique
to increase the populations of rare and endangered species of animals. Hence statement 1 is correct.
28 www.visionias.in ©Vision IAS

FREE BY KING R QUEEN P [ऋषभ राजपूत]


• Human activities like harvesting or timber collection of minor forest products and private
ownership rights are allowed. Hence statement 2 is correct.The examples are:1. Periyar (Kerala)2.
Ranipur (Uttar Pradesh)3. Chilka Lake (Orissa)4. Sariska (Rajasthan).
• National Park, Wildlife Sanctuary and Biosphere Reserve - a comparison
National Park Wildlife Sanctuary Biosphere Reserve
1. It is associated with the habitat of It is species oriented pitcher It takes into consideration the
wild animal species like rhino, tiger, plant, Great Indian Bustard entire ecosystem
etc.
2. Its boundaries are marked by Its boundaries are not Its boundaries are marked by
legislation sacrosanct. legislation
3. Disturbance only limited to buffer Limited disturbance Disturbance only limited to
zone. buffer zone.
4. Tourism is allowed Tourism is allowed Tourism is generally not
allowed
5. Scientific management is lacking Scientific management is lacking Scientifically managed
6. No attention is paid to gene pool No attention is paid to gene pool Attention is paid to gene pool
conservation conservation conservation

Q 79.A
• Measuring the diversity of a species generally incorporates estimates of "richness". Also referred to as
alpha-diversity, species richness is a common way of measuring biodiversity and involves counting the
number of individuals - or even families - in a given area. Hence, pair 1 is correctly matched.
• At the ecosystem-level, measures of biodiversity are often used to compare two ecosystems or to
determine changes over time in a given region. Describing changes in biodiversity within or between
ecosystems is called beta-diversity. Measures of beta-diversity indicate the difference in species richness
between two different habitats or within a single community at different points in time. The resulting
number indicates to researchers whether there is any overlap in the species found in each group. Hence,
pair 2 is not correctly matched.
• Gamma-diversity, on the other hand, estimates the total biodiversity within an entire region. Hence, pair
3 is not correctly matched.

Q 80.C
• Radioactive Pollution
o Radioactive wastes are generated during various operations of the nuclear fuel cycle. Mining, nuclear
power generation, and various processes in industry, defence, medicine and scientific research
produce by-products that include radioactive wastes. They cause Radioactive Pollution and expose
people living nearby to radiation hazards.
o Radioactive waste can be in gas, liquid or solid form, and its level of radioactivity can vary.
Hence, statement 1 is correct.
o The waste can remain radioactive for a few hours or several months or even hundreds of
thousands of years. Depending on the level and nature of radioactivity, radioactive wastes can be
classified as exempt waste, Low & Intermediate level waste and High-Level Waste. Hence,
statement 2 is correct.

Q 81.D
• Acidification of a water body is a phenomenon where there is a decrease in the pH level of the water body
which is harmful to the biodiversity residing in it. Ocean acidification refers to a reduction in the pH of
the ocean over an extended period of time, caused primarily by the uptake of carbon dioxide (CO2) from
the atmosphere.
• A sudden increase in the level of acidity of surface waters (lakes, streams, and rivers) in mid‐
latitude areas, caused by the melting in spring of snow that has accumulated through the winter,
and stored dry fallout of acidic precipitation is called spring shock. An acid shock can cause
significant damage to freshwater species and habitats. Also known as the acid surge.
• Hence, option (d) is the correct answer.

29 www.visionias.in ©Vision IAS

FREE BY KING R QUEEN P [ऋषभ राजपूत]


Q 82.C
• Regulations for Noise Pollution
o The Central Pollution Control Board (CPCB) has laid down the permissible noise levels in India
for different areas. Noise pollution rules have defined the acceptable level of noise in different zones
for both daytime and night time.
o In industrial areas, the permissible limit is 75 dB for daytime and 70 dB at night.
o In commercial areas, it is 65 dB and 55 dB, while in residential areas it is 55 dB and 45 dBduring
daytime and night respectively.
o Additionally, state governments have declared ‘silent zones’ which includes areas that lie within 100
meters of the premises of schools, colleges, hospitals and courts. The permissible noise limit in this
zone is 50 dB during the day and 40 dB during the night.
o Hence, option (c) is the correct answer.

Q 83.C
• Smog is a combination of smoke and fog (smoky fog) caused by the burning of large amounts of coal,
vehicular emission, and industrial fumes. It usually contains particulates like smoke, sulfur dioxide,
nitrogen dioxide, and other components.
• At least two distinct types of smog are recognized: Sulfurous smog and Photochemical Smog.
o The Sulfurous smog or “London smog,” results from a high concentration of sulfur oxides in the air
caused by the use of sulfur-bearing fossil fuels, particularly coal (Coal was the mains source of power
in London during the 19th century, the effects of which started becoming evident in the early
twentieth century).
o The Photochemical smog or “Los Angeles smog”
✓ It is sometimes called Oxidizing smog as it contains high concentrations of oxidizing agents like
Ozone, HNO3 whereas Classical Smog is called Reducing as it contains high concentrations of
sulphur dioxide, which is a reducing agent. Hence, statement 1 is correct.
✓ It occurs most prominently in urban areas with anthropogenic air pollutants,
including ozone, nitric acid, and organic compounds. Such pollutants are the large numbers of
automobiles. These are "Secondary pollutants" which occur as a result of the reaction between
primary pollutants. Hence, statement 2 is correct.
✓ [Primary pollutants: It is any pollutant emitted directly from a source]
✓ It is caused by the action of solar ultraviolet radiation on polluted air. Hence, statement 3 is
not correct.

Q 84.A
• Recently, The sea phase of bilateral exercise 'Al Mohed Al Hindi 23', between the Indian Navy and
Royal Saudi Naval Force (RSNF) was held from 23 - 25 May 23 off Al Jubail, Saudi Arabia. Hence
pair 3 is not correctly matched.
• The fourth edition of the Indo-Indonesia bilateral Exercise Samudra Shakti-23 concluded in the South
China Sea in May 2023. Hence pair 2 is not correctly matched.
• The 4th edition of a joint military exercise, “Ex Dharma Guardian ”, between India and Japan was
conducted at Camp Imazu in Shiga province, Japan in February 2023. Hence pair 1 is correctly
matched.

Q 85.B
• After a spate of helicopter crashes in recent times, a government regulatory body has called for
a safety upgrade of the Dhruv Helicopter.
• The Hindustan Aeronautics Limited's (HAL) indigenously designed and developed Advanced Light
Helicopter (ALH-DHRUV) is a twin-engine, multi-role, multi-mission new generation helicopter in
the 5.5-ton weight class. Hence statement 1 is not correct.
• Features of Dhruv:
o Capable of operating in all weather conditions with a high degree of reliability & survivability.
Hence statement 2 is correct.
o Powered with twin Shakti engines for exceptional high-altitude performance.
✓ Twin Shakti engines are 12 % Higher power than TM 333 2B2 engines; have dual centrifugal
compressor assembly and single crystal blades.
o Equipped with glass cockpit and advanced avionics for enhanced mission effectiveness including
night flying capability.

30 www.visionias.in ©Vision IAS

FREE BY KING R QUEEN P [ऋषभ राजपूत]


o Dhruv is incorporated with state of art technologies such as Hingeless Interchangeable Main Rotor
Blades, Bearingless Tail Rotor Blades, Anti resonance vibration isolation system,s and redundancies
built in for critical systems.
o Dhruv is an ideal platform for operating at various altitudes from sea level to high altitudes of the
Himalayas as well as in Desert and saline atmospheric conditions at extreme temperature ranges.
o Dhruv has evolved from basic utility version to a weaponized platform called Rudra in the 5.8-ton
class with the fitment of Mission and Weapon systems.
o Weapon fitted on Rudra comprises of 20 mm Turret Gun, 70 mm Rocket, Air to Air Missile, and
Anti-Tank Guided Missiles.
o It is incorporated with other systems such as Digital Moving Map On Board Inert Gas Generation
System etc.,
• It is a multi-role, multi-mission new generation helicopter certified for both civil and military roles.
Hence statement 3 is correct.

Q 86.A
• Thomas Robert Malthus was an influential British economist who is best known for his theory on
population growth, outlined in his 1798 book ‘An Essay on the Principle of Population’.
• In it, Malthus argued that populations inevitably expand until they outgrow their available food
supply, causing the population growth to be reversed by disease, famine, war, or calamity. He states
that while the human population grows in a geometric progression, subsistence can only grow in an
arithmetic progression. Hence, statement 1 is correct.
• ‘Limits to Growth’ was published was commission by the Club of Rome. Published 1972 – The
message of this book still holds today: The earth’s interlocking resources – the global system of
nature in which we all live – probably cannot support present rates of economic and population
growth much beyond the year 2100, if that long, even with advanced technology. An international
team of researchers at the Massachusetts Institute of Technology began a study of the implications of
continued worldwide growth. They examined the five basic factors that determine and, in their
interactions, ultimately limit growth on this planet-population increase, agricultural production,
nonrenewable resource depletion, industrial output, and pollution generation. The Malthusian theory was
developed in the 19th century. Hence, statement 2 is not correct.

Q 87.D
• Principles and Philosophy of Radioactive Waste Management followed in India.
• The Atomic Energy Regulatory Board (AERB) of India calls for reprocessing of the spent fuel and then
disposing the waste to a repository. In managing the radio-active wastes, given principles are followed:
o Principle 1: Protection of Human Health and Environment- Radioactive waste shall be managed
in such a way as to provide an acceptable level of protection for human health and the environment.
o Principle 2: Concern for Future Generations- Radioactive waste shall be managed in such a way
that it will not impose undue burden on future generations and its predicted impact on the health of
future generations will not be greater than relevant levels of impact that are acceptable today.
o Principle 3: Establishing Legal Framework- Radioactive waste shall be managed within an
appropriate legal framework including clear allocation of responsibilities and provision for
independent regulatory functions.
o Principle 4: Waste Minimisation, Management Interdependency and Safety of Facilities-
Generation of radioactive waste shall be kept to the minimum practicable. Interdependency among all
steps in radioactive waste generation and management shall be taken into account. The safety of
facilities for radioactive waste management shall be assured during their lifetime.
o Hence, option (d) is the correct answer.

Q 88.B
• Tropical Monsoon Forests are also known as drought-deciduous forests; dry forests; dry- deciduous
forests; tropical deciduous forests. Teak, neem, bamboo, sal, shisham, sandalwood, khair, and mulberry
are some of the important species.
• Tropical Rain Forest Biome
o High temperatures and abundant rainfall support a luxuriant tropical rainforest.
o The equatorial vegetation comprises a multitude of evergreen trees, e.g. mahogany, ebony, dyewoods
etc.
• Hence, option (b) is the correct answer.

31 www.visionias.in ©Vision IAS

FREE BY KING R QUEEN P [ऋषभ राजपूत]


Q 89.A
• Maharashtra has about 4,000 such groves, scattered in the wildlife regions. Bhimashankar wildlife
sanctuary is located in the northern Western Ghats in Pune and Thane districts include about 14 sacred
groves, including a large grove surrounding a Shiva temple, which is one of the twelve Jyotirlingams in
India.
• The groves in Maharashtra are called deorais or devrais and are concentrated in Pune, Ratnagiri,
Raigad and Kolhapur. A rare variety of biodiversity prevails in these areas.
• The different names for Devrai are used in different parts of the country, such as Pavithra Vana,
Kovikaadugal, Jahera, Devgudi, Banis, Haritan, Sarna, Lai Umang, Kovil Kadu, etc. Etymologically the
word “Devrai” is derived from two words; that is Dev (which means God in Marathi) and rai (which
means forest in Marathi).
• These Devrais have been entirely or partially restricted from any human interference, which provided
habitat for rare species of animals, arboreal birds, mammals and vegetation.
• Due to limited resource extraction from such Devrais, they prove to be rich resources for fruit-bearing
trees, small water bodies, habitats, and other factors responsible for biodiversity enrichment.
• Some essential functions such as maintaining local ecological balance, conserving watersheds for birds
and animals, preserving bio-resources, sustainable use of resources, etc., which are crucial steps for
maintaining forests, are frequently followed here in this traditional system.
• Due to the abundance of diversity in species of trees, Devrais often preserve certain plant species which
may have medicinal purposes for the communities or tribes. Such plants possess medicinal properties and
can also have commercial value if studied properly and exploited.
• Hence, option (a) is the correct answer.

Q 90.C
• Biodiversity conservation: When we conserve and protect the whole ecosystem, its biodiversity at all
levels is protected - we save the entire forest to save the tiger. This approach is called in situ (on-site)
conservation. However, when there are situations where an animal or plant is endangered or threatened
and needs urgent measures to save it from extinction, ex-situ (off-site) conservation is the desirable
approach.
• In-situ conservation is the on-site conservation of genetic resources in natural populations of plant or
animal species. In India, ecologically unique and biodiversity-rich regions are legally protected as
biosphere reserves, national parks, sanctuaries, reserved forests, protected forests, and nature reserves.
India has also a history of religious and cultural traditions that emphasized the protection of nature. In
many cultures, tracts of forest were set aside, and all the trees and wildlife within were venerated and
given total protection. In Meghalaya, the sacred groves are the last refuge for a large number of rare and
threatened plants. Sacred groves are a fine example of in-situ conservation.
• Ex situ Conservation–In this approach, threatened animals and plants are taken out from their natural
habitat and placed in special settings where they can be protected and given special care. Zoological
parks, botanical gardens, and wildlife safari parks serve this purpose. In recent years ex-situ
conservation has advanced beyond keeping threatened species in enclosures. Now gametes of threatened
species can be preserved in viable and fertile conditions for long periods using cryopreservation
techniques, eggs can be fertilized in vitro, and plants can be propagated using tissue culture methods.
Seeds of different genetic strains of commercially important plants can be kept for long periods in seed
banks.
• Hence option (c) is the correct answer.

Q 91.D
• Municipal Solid Waste: The urban India generates 62 million tonnes of municipal solid waste per
annum. Only 43 million tonnes (MT) of the waste are collected, 11.9 MT is treated and 31 MT is dumped
in landfill sites. Plastic bags made from low density polyethylene (LDPE) are virtually indestructible and
create colossal environmental hazard. The discarded bags block drains and sewage systems. Leftover
food, vegetable waste etc. on which cows and dogs feed may die due to the choking by plastic bags.
Burning of solid waste release highly toxic and poisonous gases like carbon monoxide, carbon dioxide,
phosgene, dioxins and other poisonous chlorinated compounds.
• Agricultural Waste: India is estimated to produce around 620 Million tonne per annum of agricultural
wastes, 43 per cent of which is animal dung and slaughter wastes. Agriculture and livestock activities
pollute soil through excessive application of pesticides and fertilizers, the use of untreated wastewater for
irrigation, and the use of manure and sewage sludge with high antibiotic, antimicrobial-resistant bacteria
and heavy metal content.
32 www.visionias.in ©Vision IAS

FREE BY KING R QUEEN P [ऋषभ राजपूत]


• Industrial hazardous waste: India produces approximately 51.1 MMT (million metric tonnes) of waste
annually, with around 7.46 MMT of hazardous waste. Approximately 3.41 MMT (46%) is landfilled, 0.69
MMT (9%) is incinerated, and 3.35 MMT (45%) is recycled. Gujarat is the highest producer of hazardous
wastes in India. Industrial wastes include chemical residues, metallic and nuclear wastes. Large number of
industrial chemicals, dyes, acids, etc. find their way into the soil and are known to be carcinogenic.
• Bio-Medical waste: As per Assocham, the total quantity of medical waste generated in India is 550
tonnes per day (TPD) and by 2022, it is likely to increase to 775.5 TPD.
• Electronic Waste: India generates about 1.85 million tonnes per annum of e-waste and ranks fifth in the
world among top e-waste producing countries. E-waste can be toxic, is not biodegradable and accumulates
in the environment, in the soil, air, water and living things.
• Hence, option (d) is the correct answer.

Q 92.C
• Recently, Sudan’s eruption into conflict has left international consumer goods makers racing to
shore up supplies of gum arabic.
• 'Gum Arabic' is a natural gum derived from the hardened sap of two species of the Acacia tree –
Senegalia Senegal and Vachellia seyal. The gum is harvested commercially from wild trees, mostly
in Sudan (80%) and throughout the Sahel (from Senegal to Somalia).
• In their manufacturing process, food and drink companies use a spray-dried version of the gum that is
powder-like. While cosmetics and printing manufacturers may be able to use substitutes, there is no
alternative to gum arabic in fizzy drinks, where it prevents ingredients from separating.
• Hence, option (c) is the correct answer.

Q 93.C
• Bishnoi Tribe of Rajasthan: Bishnois consider trees as sacred and protect the entire ecosystem including
animals and birds that exists in their villages. The tribe has organized their own Tiger Force which is a
brigade of youth actively pursuing wildlife protection.
• Chenchu Tribe of Andhra Pradesh: They are involved in tiger conservation at Nagarjunasagar
Srisailam Tiger Reserve (NSTR). The tribe has been coexisting with tigers and wild animals for a long
without disturbing the ecological balance, which ensures enough water and fodder for the herbivores.
• Maldhari Tribe in Junagadh (Gujarat): The success of lion conservation in the Gir forest area is due to
the peaceful coexistence of tribes with lions.
• Bugun Tribe of Arunachal Pradesh: The tribe using Community-led conservation initiatives and
traditional knowledge helped to protect the critically endangered bird Bugun Liocichla. For its efforts,
Singchung Bugun Community Reserve won the India Biodiversity Award 2018.
• Nyishi tribe of Arunachal Pradesh in conserving hornbills in the Pakke/Pakhui Tiger Reserve. Recently,
the government of Arunachal Pradesh declared the Pakke Paga Hornbill Festival (PPHF)–the state’s only
conservation festival, as a ‘state festival’.
• Hence, option (c) is the correct answer.

Q 94.A
• Parasitism involves a parasite living in or on another living species called the host. Parasitism is
beneficial to one species (parasite) and harmful to the other species (host). The parasite gets its
nourishment and often shelter from its host.
• Parasites that feed on the external surface of the host organism are called ectoparasites. The most familiar
examples of this group are the lice on humans and ticks on dogs. Cuscuta, a parasitic plant that is
commonly found growing on hedge plants, has lost its chlorophyll and leaves in the course of
evolution. It derives its nutrition from the host plant which it parasitizes. Hence option 1 is correct.
• Endoparasites are those that live inside the host body at different sites (liver, kidney, lungs, red blood
cells, etc.). Brood parasitism in birds is an example of parasitism in which the parasitic bird lays its eggs
in the nest of its host and lets the host incubate them.
• Commensalism is the interaction in which one species benefits and the other is neither harmed nor
benefited.
• The cattle egret and grazing cattle in close association, is a classic example of commensalism. The
egrets always forage close to where the cattle are grazing because the cattle, as they move, stir up and
flush out insects from the vegetation that otherwise might be difficult for the egrets to find and
catch. Hence option 2 is not correct.
• Mutualism confers benefits on both interacting species. Lichens represent an intimate mutualistic
relationship between a fungus and photosynthesizing algae or cyanobacteria. Similarly, the mycorrhizae
33 www.visionias.in ©Vision IAS

FREE BY KING R QUEEN P [ऋषभ राजपूत]


are associations between fungi and the roots of higher plants. Another example of Mutualism is
a symbiotic relationship between an anemone and a clownfish the anemone provides the clownfish
with protection and shelter, while the clownfish provides the anemone nutrients in the form of waste while
also scaring. Hence option 3 is not correct.

Q 95.B
• Sikkim Integration
• Why in the news?
o The 22nd Sikkim Day was celebrated on May 16, 2023, recalling the history of the former kingdom’s
integration with India in 1975.
o Sikkim Kingdom:
✓ The kingdom of Sikkim was established in 1642.
✓ Phuntsong Namgyal was the first ruler of Chogyal (king) of Sikkim. Hence, statement 1 is
correct.
✓ Sikkim’s Chogyal dynasty was of Tibetan origin.
✓ Sikkim often saw conflicts over land with Bhutan and Nepal.
✓ The Namgyal dynasty came to an end with the statehood of Sikkim and the position of Chogyal
was abolished.
o Sikkim in the British era:
✓ The British saw Sikkim as a buffer state against China and against Nepal, with whom they fought
in the Anglo-Gorkha War of 1814-16.
✓ In the Anglo-Gorkha War, the British helped Sikkim secure a number of territories that Nepal had
previously captured.
✓ The monarchy of the Namgyal dynasty was maintained for the next 333 years, until its integration
with India in 1975.
✓ The Treaty of Tumlong in 1861 gave the British control over Sikkim but the Chogyals continued
to hold onto power.
▪ Post-Independence:
✓ After India’s independence, princely states had the option to accede to India or Pakistan.
✓ In 1950, the Indo-Sikkim Treaty was signed, making Sikkim an Indian protectorate.
✓ India controlled Sikkim’s defense, external affairs, and strategic communications.
✓ In 1974, a new constitution for Sikkim was adopted, which restricted the role of the monarch to a
titular post.
✓ A referendum was held in Sikkim in 1975 and the majority of people voted in favor of joining
India. Hence, statement 2 is not correct.
✓ The 36th Constitutional Amendment Act was passed in Parliament to recognize Sikkim as a state
in the Union of India. Hence, statement 3 is correct.
✓ With the assent of then President Fakhruddin Ali Ahmed, Sikkim became the 22nd Indian state on
May 16, 1975.

Q 96.A
• Primary production is defined as the amount of biomass or organic matter produced per unit area over a
time period by plants during photosynthesis. It is expressed in terms of weight (gm–2 ) or energy (kcal m–
2). The gross primary productivity (GPP) of an ecosystem is the rate of production of organic
matter during photosynthesis. Hence, statement 1 is correct.
• A considerable amount of GPP is utilized by plants in respiration. Gross primary productivity minus
respiration losses (R), is the net primary productivity (NPP). Net primary productivity is the available
biomass for the consumption of heterotrophs (herbivores and decomposers). Secondary productivity
is defined as the rate of formation of new organic matter by consumers. Hence, statement 2 is not
correct.
• Primary productivity varies in different types of ecosystems. The annual net primary productivity of
the whole biosphere is approximately 170 billion tons (dry weight) of organic matter. Of this, despite
occupying about 70 percent of the surface, the productivity of the oceans are only 55 billion tons (only 32
%). The rest of the 115 billion tons (67% of the total) is on land.

Q 97.A
• Water pollutants can be of multiple types including- Biological, Chemical, Thermal, etc.
• The Biological pollutants include pathogens such as Escherichia coli, Enterococcus faecalis (formely
known as Streptococcus faecalis), Salmonella typhi, etc. Hence, option (a) is the correct answer.
34 www.visionias.in ©Vision IAS

FREE BY KING R QUEEN P [ऋषभ राजपूत]


• Pathogens include bacteria and other organisms that enter the water from domestic sewage and animal
excreta.
• Human excreta contains bacteria such as Escherichia coli and Streptococcus faecalis which cause
gastrointestinal diseases.

Q 98.C
• Agenda 21, established at the 1992 United Nations Conference on Environment and Development,
or “Earth Summit”, in Rio de Janeiro, Brazil, is the blueprint for sustainability in the 21st century.
Agenda 21 is a commitment to sustainable development, which was agreed by many of the world’s
governments. Nations that have pledged to take part in Agenda 21 are monitored by the International
Commission on Sustainable Development and are encouraged to promote Agenda 21 at the local and
regional levels within their own countries. Agenda 21 addresses the development of societies and
economies by focusing on the conservation and preservation of our environments and natural resources.
Hence, option (c) is the correct answer.
• Agenda 21, the Rio Declaration on Environment and Development, and the Statement of Principles for the
Sustainable Management of Forests were adopted by more than 178 Governments at the United Nations
Conference on Environment and Development (UNCED) held in Rio de Janeiro, Brazil, 3 to 14 June
1992.
• The Commission on Sustainable Development (CSD) was created in December 1992 to ensure effective
follow-up of UNCED and to monitor and report on the implementation of the agreements at the local,
national, regional, and international levels. It was agreed that a five-year review of Earth Summit progress
would be made in 1997 by the United Nations General Assembly meeting in a special session.
• The full implementation of Agenda 21, the Programme for Further Implementation of Agenda 21, and the
Commitments to the Rio principles were strongly reaffirmed at the World Summit on Sustainable
Development (WSSD) held in Johannesburg, South Africa from 26 August to 4 September 2002.

Q 99.A
• Recent Context: Calling it an attempt to indulge in ‘bench hunting’, a former Supreme Court judge
refused to recuse himself from hearing a plea.
• What is recusal? It is the act of declining to take part in an official activity, such as a court case because
the administrative or court authority presiding over it has a conflict of interest. Hence, statement 1 is
correct.
o This conflict of interest can arise in many ways: from holding shares in a litigant company to having a
prior or personal association with a party.
o The practice of judicial recusals stems from:
✓ The cardinal principles of due process of law
✓ Nemo judex in sua causa – no person shall be a judge in his own case.
• Why do judges recuse? To prevent the perception that the judge was biased while deciding a case.
• What is the procedure for recusal?
o Although several SC judgments have dealt with the issue, India has no codified rules governing
recusals. Hence, statement 2 is not correct.
o There are two kinds of recusals: automatic (where a judge himself withdraws from the case) or when a
party raises a plea for recusal.
o The decision to recuse rests solely on the conscience and discretion of the judge and no party can
compel a judge to withdraw from a case.
o If a judge recuses himself, the case is listed before the Chief Justice for allotment to an alternate
Bench.

Q 100.C
• A specified area in which multiple use of land is allowed by dividing it into different zones and each zone,
which remains specified for a particular activity, is called a Biosphere Reserve.
• A number of biosphere reserves have been established by United Nations Educational Scientific and
Cultural Organisation (UNESCO) under its Man And Biosphere Programme (MAB)-1986 in different
countries. The biosphere reserves have international networks.
• Each of the biosphere reserves has been divided into three zones. Hence statement 1 is not correct.
o Core Zone, where human interference is banned completely
o Buffer Zones, where human interference is allowed up to a limited extent.
o Manipulated Zone or Transition Zones, where humans are free to perform their activities.
Hence statement 2 is not correct.
35 www.visionias.in ©Vision IAS

FREE BY KING R QUEEN P [ऋषभ राजपूत]


• The biosphere reserves are planned, managed and protected through joint efforts of the government, non-
governmental organizations and the local people. India has declared 18 areas as biosphere reserves. These
areas are aimed at
o Conservation of biodiversity (species, ecosystem, and landscapes).
o Development of economic and human infrastructures.
o Promotion of education, information – exchange and research pertaining to conservation and
development.
• The Biosphere Reserves have the following functions:
o The biosphere reserves are helpful in the conservation of ecosystems, species and other resources.
o The biosphere reserves are helpful in the promotion of economic development.
o Biosphere reserves are helpful in promoting scientific research and education. Hence statement
3 is correct.

Copyright © by Vision IAS (www.pdfnotes.co)


All rights are reserved. No part of this document may be reproduced, stored in a retrieval system or
transmitted in any form or by any means, electronic, mechanical, photocopying, recording or otherwise,
without prior permission of Vision IAS.
36 www.visionias.in ©Vision IAS

FREE BY KING R QUEEN P [ऋषभ राजपूत]


VISIONIAS
www.visionias.in

Test Booklet Series

TEST BOOKLET

GENERAL STUDIES (P) 2024 – Test – 4134


C
Time Allowed: Two Hours Maximum Marks: 200

INSTRUCTIONS

1. IMMEDIATELY AFTER THE COMMENCEMENT OF THE EXAMINATION, YOU SHOULD CHECK THAT THIS BOOKLET
DOES NOT HAVE ANY UNPRINTED OR TURN OR MISSING PAGES OR ITEMS, ETC. IF SO, GET IT REPLACED BY A
COMPLETE TEST BOOKLET.

2. ENCODE CLEARLY THE TEST BOOKLET SERIES A, B, C OR D AS THE CASE MAY BE IN THE APPROPRIATE PLACE IN
THE ANSWER SHEET.

3. You have to enter your Roll Number on the Test Booklet in the Box
provided alongside. Do NOT write anything else on the Test Booklet.

4. This Test Booklet contains 100 items (Questions). Each item is printed in English. Each item comprises four
responses (answers). You will select the response which you want to mark on the Answer Sheet. In case you
feel that there is more than one correct response with you consider the best. In any case, choose ONLY ONE
response for each item.

5. You have to mark all your responses ONLY on the separate Answer Sheet provided. See direction in the
answers sheet.

6. All items carry equal marks. Attempt all items. Your total marks will depend only on the number of correct
responses marked by you in the answer sheet. For every incorrect response 1/3rdof the allotted marks will be
deducted.

7. Before you proceed to mark in the Answer sheet the response to various items in the Test booklet, you have to
fill in some particulars in the answer sheets as per instruction sent to you with your Admission Certificate.

8. After you have completed filling in all responses on the answer sheet and the examination has concluded, you
should hand over to Invigilator only the answer sheet. You are permitted to take away with you the Test
Booklet.

9. Sheet for rough work are appended in the Test Booklet at the end.

DO NOT OPEN THIS BOOKLET UNTIL YOU ARE ASKED TO DO SO


1 www.visionias.in ©Vision IAS

FREE BY KING R QUEEN P [ऋषभ राजपूत]


1. Consider the following statements: 4. Consider the following statements about the
1. Article 1 states India as Bharat which is Indian Judiciary:
a 'Union of States and Union Territories'.
1. The strength of judges in the Supreme
2. The expression of 'Union of India' is
Court can be increased by Parliament.
wider in extent than the expression
'Territory of India'. 2. The strength of judges in the High
Which of the statements given above Courts can be increased by the President.
is/are not correct? Which of the statements given above is/are
(a) 1 only correct?
(b) 2 only
(a) 1 only
(c) Both 1 and 2
(d) Neither 1 nor 2 (b) 2 only
(c) Both 1 and 2
2. Consider the following Directive Principles (d) Neither 1 nor 2
of State Policy (DPSPs):
1. To minimize inequalities in income,
5. In the context of various political systems
status, facilities, and opportunities
prevailing in the world, consider the
2. To promote equal justice and to provide
free legal aid to the poor following countries:
3. To secure opportunities for the healthy 1. China
development of children 2. Vietnam
How many of the above DPSPs were added 3. Cuba
through the 42nd Amendment Act, 1976?
4. Argentina
(a) Only one
(b) Only two How many of the above countries are
(c) All three currently governed by One Party System?
(d) None (a) Only one
(b) Only two
3. Consider the following statements about the
(c) Only three
fundamental rights under Article 21 of the
(d) All four
Indian Constitution:
1. If it is violated by the State, there is an
explicit provision in the Constitution for 6. Consider the following statements regarding
grant of compensation by the State. the Fundamental Rights:
2. The protection under it can be available 1. Constitutional remedies are not available
only against arbitrary executive action
if all the rights are violated by the
and not against arbitrary legislative
action. private individuals.
3. In the Maneka case 1978, the Supreme 2. Some of the Fundamental Rights are not
Court of India recognized the expression self-executory.
‘due process of law’ under Article 21. Which of the statements given above is/are
How many statements given above are
correct?
correct?
(a) 1 only
(a) Only one
(b) Only two (b) 2 only
(c) All three (c) Both 1 and 2
(d) None (d) Neither 1 and 2
2 www.visionias.in ©Vision IAS

FREE BY KING R QUEEN P [ऋषभ राजपूत]


7. With reference to the Constitution of India, 10. Consider the following statements about the
consider the following statements: Sessions Judge:
1. The Indian Constitution seeks to impart
1. The District judge, when hearing a
more flexibility to a written Federal
Constitution than the Constitution of the criminal case, is known as the Sessions
United States. Judge.
2. Some provisions in the Constitution of 2. The Sessions Judge can sentence a
India can only be amended with the
person to life imprisonment but cannot
ratification of two-thirds of the State
award a death sentence.
legislatures with a special majority.
Which of the statements given above is/are Which of the statements given above is/are
correct? correct?
(a) 1 only (a) 1 only
(b) 2 only
(b) 2 only
(c) Both 1 and 2
(d) Neither 1 nor 2 (c) Both 1 and 2
(d) Neither 1 nor 2
8. The Sapru Report of 1945 holds an
important place in the Constitutional history
11. Consider the following pairs:
of India for shaping the key provisions of the
Constitutional Important changes
Constitution because it emphasized the need
for Amendment
1. Inclusion of provisions for the protection 1. 1st : Made the right to
of minority rights. Amendment property a legal right
2. Inclusion of non-justiciable Directive
Act, 1951 from the fundamental
Principles of State Policy
3. Inclusion of non-justiciable Fundamental right
duties 2. 52nd : A member disqualified
Select the correct answer using the code Amendment for defection cannot be
given below.
Act, 1985 a minister
(a) 1 only
(b) 2 and 3 only 3. 77th : Provided reservation in
(c) 1 and 2 only Amendment promotions to SCs and
(d) 1 and 3 only Act, 1995 STs
4. 91st : Stipulated the upper
9. Consider the following statements regarding
Amendment limit for the number of
Theocracy:
1. Religious principles often guide political Act, 2003 Council of Ministers at
decision-making. both central and state
2. State generally supports and promotes a levels
particular religion.
How many pairs given above are correctly
3. Freedom of Religion is guaranteed.
How many of the above statements are matched?
correct? (a) Only one
(a) Only one (b) Only two
(b) Only two
(c) Only three
(c) All three
(d) All four
(d) None
3 www.visionias.in ©Vision IAS

FREE BY KING R QUEEN P [ऋषभ राजपूत]


12. Which political system prioritizes the 15. Which of the following statements is not
protection of individual rights and limits the correct about the remuneration of Judges in
power of government through a written
India?
constitution?
(a) Benevolent Despotism (a) The salaries of the Supreme Court
(b) Autocracy judges are charged on the Consolidated
(c) Constitutionalism Fund of India.
(d) Plutocracy
(b) The pensions of the Supreme Court
13. Consider the following pairs: judges are charged on the Consolidated
Military Exercise India and Associated Fund of India.
Nations
(c) The salaries of the High Court judges are
1.TARKASH : United States of
charged on the Consolidated Fund of the
America
2. Exercise Desert : Iran respective State.
Flag (d) The pensions of the High Court judges
are charged on the Consolidated Fund of
3. Exercise : Egypt
Cyclone-I the respective State.

How many of the above pairs are correctly


16. The "libertarian" political ideology
matched?
(a) Only one advocates for:
(b) Only two (a) strong government control over the
(c) All three
economy and individual freedoms.
(d) None
(b) limited government intervention and
14. With reference to personalities associated maximum individual liberties.
with the making Constitution of India,
(c) social equality and wealth redistribution.
consider the following pairs:
Personality Their Role in (d) a mix of conservative and liberal
Constitution Making policies.
1. H.V.R. : Chief Draftsman of
Iyengar the Constitution
17. Consider the following words/phrases:
2. Prem Behari : Calligrapher of the
Narain Raizada Constitution 1. Universal Adult Franchise
3. Sir B.N. Rau : Legal advisor to the 2. Right to Work
Constituent Assembly
3. Union Cabinet
4. S.N. : Secretary to the
How many of the above words/phrases
Mukerjee Constituent
Assembly do not find mention in the Constitution of
How many pairs given above are correctly India?
matched?
(a) Only one
(a) Only one pair
(b) Only two pairs (b) Only two
(c) Only three pairs (c) All three
(d) All four pairs (d) None
4 www.visionias.in ©Vision IAS

FREE BY KING R QUEEN P [ऋषभ राजपूत]


18. Consider the following activities: 21. Recently, India’s payment system Unified
1. Rainwater harvesting Payments Interface (UPI) and its equivalent
2. Organic farming
network PayNow, were integrated to enable
3. Sawmills
4. Commercial use of wood faster remittances between the two countries

How many of the above-mentioned activities at a competitive rate.


are permitted in the eco-sensitive zones PayNow is a payment system of which of the
(ESZ)??
following nations?
(a) Only one
(a) Nepal
(b) Only two
(c) Only three (b) Singapore
(d) Only four (c) Bhutan
(d) U.A.E
19. Consider the following statements:
1. Millet International Initiative for
Research and Awareness (MIIRA) is an 22. Which of the following are features of the
initiative of the United Nations launched federal government?
on the sidelines of declaring 2023 as the 1. Written Constitution
International Year of Millets.
2. Independent Judiciary
2. MIIRA secretariat will be in Delhi.
Which of the statements given above is/are 3. Dual sovereignty

correct? Select the correct answer using the code


(a) 1 only given below.
(b) 2 only
(a) 1 and 2 only
(c) Both 1 and 2
(b) 2 and 3 only
(d) Neither 1 nor 2
(c) 3 only
20. Which of the following statements is correct (d) 1, 2 and 3
about Directive Principles of State Policy
(DPSP)?
23. With reference to Channapatna toys,
(a) The idea of DPSP is borrowed from
Soviet Union Constitution (erstwhile consider the following statements:
Russia). 1. They are a particular form of toys that
(b) They resemble the ‘Instrument of are manufactured in Karnataka.
Instructions’ enumerated in the
2. The artisans use ivory wood for making
Government of India Act, 1935.
toys.
(c) They are legally enforceable by the
courts for their violation. Which of the statements given above is/are
(d) In Champakam Dorairajan case (1951), correct?
Supreme Court held that the
(a) 1 only
Fundamental Rights can not be amended
(b) 2 only
by the Parliament by enacting the
Constitutional Amendment Acts for the (c) Both 1 and 2
implementation of the DPSPs. (d) Neither 1 nor 2
5 www.visionias.in ©Vision IAS

FREE BY KING R QUEEN P [ऋषभ राजपूत]


24. Gross Domestic Climate Risk Report, which 26. Consider the following statements:

is released by Cross Dependency Initiative Statement-I: Additional Surveillance


(XDI) measures
Mechanism (ASM) on securities is levied
(a) The effect on the Gross Domestic
jointly by SEBI and Exchanges with the
Product of a country due to climate
change. intention to protect investors from market

(b) The risk posed to built environment volatility and unusual changes in share price.

across the world due to extreme weather Statement-II: The shortlisting of securities
events.
under ASM is regarded as an adverse action
(c) The percentage of Budget allocation to
against the concerned company/entity.
its Gross Domestic Product(GDP) by

countries in mitigation and adaptation to Which one of the following is correct in

climate change. respect of the above statements?


(d) The risk posed by built environment (a) Both Statement-I and Statement-II are
across the world to natural environment.
correct and Statement-II is the correct

explanation for Statement-I.


25. Consider the following pairs :

Form of Feature (b) Both Statement-I and Statement-II are

government correct and Statement-II is not the

1. Oligarchy : government by a correct explanation for Statement-I.


small group of very
(c) Statement-I is correct but Statement-II is
powerful people
incorrect.
2. Totalitarianism : seeks to subordinate

all aspects of (d) Statement-I is incorrect but Statement-II

individual life to the is correct.

authority

3. Anarchy : absence of
27. Virovory Halteria, which is recently seen in
government
news is a/an
How many of the pairs given above is/are

correctly matched? (a) Cryptocurrency

(a) Only one pair (b) Virus eating organism


(b) Only two pairs (c) Antivirus Software
(c) All three pairs
(d) Non-Fungible Token
(d) None
6 www.visionias.in ©Vision IAS

FREE BY KING R QUEEN P [ऋषभ राजपूत]


28. With reference to provisions of the 30. Consider the following statements:
Statement-I: The creation of All India
Government of India Act 1935, consider the
Services needs a constitutional amendment
following statements: with half of the states ratifying the
amendment.
1. As per the Act, every bill passed by the
Statement-II: All India Services play a
Central Legislature would be subjected significant role in liaison, cooperation, and
coordination in federal relations between the
to veto by the British Crown.
center and states.
2. The Provincial legislatures were Which one of the following is correct in
respect of the above statements?
conferred jurisdiction over the Police
(a) Both Statement-I and Statement-II are
system. correct. and Statement-II is the correct
explanation for Statement-I
3. It removed the distinction between
(b) Both Statement-I and Statement-II
transferred and reserved subjects in the correct and Statement-II is not the·
correct explanation for Statement-I
provinces and the whole administration (c) Statement-I is correct but Statement-II is
was entrusted with the ministers incorrect
(d) Statement-I is incorrect but Statement-II
responsible to the legislature. is correct
How many of the above statements are
31. How many of the following constitutional
correct? amendments given below are related to
(a) Only one changes in the first schedule?
1. 7th amendment
(b) Only two 2. 12th amendment
(c) All three 3. 35th amendment
4. 100th amendment
(d) None Select the correct answer using the code
given below.
(a) Only one
29. Consider the following: (b) Only two
1. Population control (c) Only three
(d) All four
2. Preserving natural environment
32. In the federal system set up by the
3. Voting percentage
constitution of India, which of the following
How many of the above may depend on the subjects are under the union list?
1. Citizenship
successful implementation of fundamental
2. Insurance
duties enshrined in the Indian constitution? 3. Census
Select the correct answer using the code
(a) Only one
given below.
(b) Only two (a) 1 only
(b) 2 and 3 only
(c) All three
(c) 1 and 3 only
(d) None (d) 1, 2 and 3
7 www.visionias.in ©Vision IAS

FREE BY KING R QUEEN P [ऋषभ राजपूत]


33. Shumang Leela is a festival associated with 37. Consider the following statement regarding
which of the following states? the amendment procedure for the
(a) Manipur
constitution of India:
(b) Assam
1. The expression “total membership”
(c) Odisha
(d) West Bengal means the total number of members
comprising the House irrespective of
34. UPNEXT India 2023 Event, which is whether there are vacancies or absentees
recently seen in news is an initiative of
on any account
(a) Agricultural and Processed Food
Products Export Development Authority 2. The expression “present and voting”,

(APEDA) means members who vote for “ayes”, or


(b) Apparel Export Promotion Council “noes” or any abstention from voting by
(AEPC)
a member.
(c) Marine Products Export Development
Which of the statements given above is/are
Authority (MPEDA)
(d) Khadi and Village Industries correct?

Commission (KVIC) (a) 1 only

(b) 2 only
35. In which of the following cases, the Supreme
(c) Both 1 and 2
Court of India held that the Preamble is an
(d) Neither 1 nor 2
integral part of the Constitution?
1. Berubari Union case, 1960
2. Keshavananda Bharati case,1973 38. Which of the following statements is correct
3. LIC of India case, 1995 about the judicial review of the Indian
Select the correct answer using the code
judiciary?
given below.
(a) 2 only (a) The Constitution defines the term

(b) 2 and 3 only judicial review as the power of the


(c) 1 and 2 only Supreme Court to examine the actions of
(d) 1 and 3 only
the government and determine whether

they are consistent with the Constitution.


36. In a proportional representation electoral
system, how are seats in the legislative body (b) Only the Supreme Court of India is a

allocated? court of review.


(a) Based on a winner-takes-all system (c) The scope of Judicial review in India is
(b) According to the proportion of votes
narrower than that of the USA.
received by political parties.
(d) The laws included in the 9th Schedule of
(c) Determined by a panel of judges
(d) Allocated based on the wealth or social the Indian Constitution are exempt from
status of candidates judicial review.
8 www.visionias.in ©Vision IAS

FREE BY KING R QUEEN P [ऋषभ राजपूत]


39. With reference to Basavanna, consider the 42. Which of the following is/are the
following statements : applications of "Quasicrystals", which is
1. He was a social reformer during the often seen in the news?
reign of the Kalachuri-dynasty king 1. Non-stick frying pans
Bijjala I. 2. Needles for acupuncture
2. He presided over the Sharana movement 3. Razor blades
which attracted people from all castes. Select the correct answer using the code
3. He introduced a new public institution given below.
Anubhava Mantapa to discuss spiritual (a) Only one
(b) Only two
questions of life.
(c) All three
How many of the above statements are
(d) None
correct?
(a) Only one
43. Consider the following statements :
(b) Only two
Statement-I: There have been no
(c) All three
referendums in India since independence.
(d) None
Statement-II: Constitution of India does not
have a specific provision for or against the
40. Which of the following countries have a referendum.
constitutional monarchy? Which one of the following is correct in
1. Canada respect of the above statements?
2. Thailand (a) Both Statement-I and Statement-II are
3. Norway correct and Statement-II is the correct
Select the correct answer using the code explanation for Statement-I
given below. (b) Both Statement-I and Statement-II are
(a) 2 only correct and Statement-II is not the
(b) 1 and 3 only correct explanation for Statement-I
(c) 1 and 2 only (c) Statement-I is correct but Statement-II is
(d) 1, 2 and 3 incorrect
(d) Statement-I is incorrect but Statement-II
41. The Article 20 of the Constitution of India is correct
talks about the right against self-
incrimination. In this context, right against 44. Consider the following statements regarding
self-incrimination does not extend to which Atmanirbhar Clean Plant Programme, which
is recently launched in 2023 Budget :
of the following activities?
1. It is a programme to establish waste-
1. Obtaining DNA from the accused
water treatment plants to check river
2. Voice Samples of the accused
pollution.
3. Compulsory exhibition of the body of
2. The programme will be anchored by the
the accused
National Horticulture Board which in
4. Compulsory production of the material
turn will set up Clean Plant Centres
objects
across the country.
Select the correct answer using the code
Which of the statements given above is/are
given below. correct?
(a) 1, 2 and 3 only (a) 1 only
(b) 1 and 4 only (b) 2 only
(c) 2, 3, and 4 only (c) Both 1 and 2
(d) 1, 2, 3, and 4 (d) Neither 1 nor 2
9 www.visionias.in ©Vision IAS

FREE BY KING R QUEEN P [ऋषभ राजपूत]


45. Apart from the Directives included in Part 47. Consider the following pairs:

IV, there are some other Directives Wildlife Sanctuary State


contained in other Parts of the Constitution.
1. Shendurney Wildlife : Kerala
In this context, which of the following
Sanctuary
Directives is/are not contained in the Part IV
2. Bankapura Wolf Wildlife : Karnataka
of the constitution?
Sanctuary
1. Claims of Scheduled Casts and

Scheduled Tribes to services 3. Kaimur Wildlife : Tamil Nadu

2. To provide adequate facilities for Sanctuary

instruction in the mother-tongue at the How many of the above pairs are correctly

primary stage of education to children matched?


belonging to linguistic minority groups (a) Only one pair
3. Development and promotion of the (b) Only two pairs
Hindi Language
(c) All three pairs
Select the correct answer using the code
(d) None of the pairs
given below.
(a) 2 only
48. With reference to the Unlawful Activities
(b) 2 and 3 only
(Prevention) Act, 1967, consider the
(c) 1 and 3 only
following statements:
(d) 1, 2 and 3
1. Under this act both Indian and foreign

46. Consider the following statements regarding nationals can be charged.

the 'Schengen Zone', which is often seen in 2. It is applicable only to offenses

the news: committed inside Indian territory.


1. The area is named after a small village 3. Investigation of cases can be conducted
in Luxembourg. by officers of the rank of Superintendant
2. Both European Union (EU) and non-EU
of Police (SP) or above.
members can be part of 'Schengen Zone'.
How many of the above statements are
Which of the statements given above is/are
correct?
correct?
(a) Only one
(a) 1 only
(b) Only two
(b) 2 only

(c) Both 1 and 2 (c) All three

(d) Neither 1 nor 2 (d) None


10 www.visionias.in ©Vision IAS

FREE BY KING R QUEEN P [ऋषभ राजपूत]


49. With reference to the Constitution of India, 51. Consider the following statements with
respect to the term ‘Justice’ in Preamble:
consider the following statements:
1. The combination of social, economic,
1. The original Constitution adopted on and political justice denotes what is
November 26, 1949, contained Seven known as ‘distributive justice’.
2. Economic justice denotes the non-
fundamental rights.
discrimination between people based on
2. The objectives resolution was economic factors.
Which of the statements given above is/are
unanimously adopted by the Constituent
correct?
Assembly on January 22, 1947. (a) 1 only
3. The Constitution Assembly of India was (b) 2 only
(c) Both 1 and 2
dissolved with the commencement of the
(d) Neither 1 nor 2
Constitution of India.
52. Consider the following statements about the
How many of the statements given above are
High Court of India:
correct? 1. The Judges of the High Courts are
(a) Only one appointed by the Governor.
2. The Judges of the High Courts can only
(b) Only two
be removed by the President on the
(c) All three recommendation of the Parliament.
3. An eminent jurist can be eligible for
(d) None
appointment as a Judge of a High Court.
Which of the statements given above is/are
50. With reference to the act for the Good correct?
(a) 2 only
Government of India, consider the following
(b) 3 only
statements: (c) 1 and 2 only
(d) 1, 2 and 3
1. It ended the system of double
Government by abolishing the Board of 53. Consider the following statements regarding
Control and Court of Directors. fundamental duties under Article 51A:
1. They are inspired by the Constitution of
2. It did not provide separation of functions
the United States of America.
and all the authority for governance of 2. They serve as an aid in the interpretation
of the Constitution.
India was vested in the Governor-
3. If any law is intended to give effect to
General in Council. fundamental duties, such a law can be
Which of the statements given above is/are considered reasonable in relation to
Article 14.
correct?
How many of the statements given above are
(a) 1 only correct?
(a) Only one
(b) 2 only
(b) Only two
(c) Both 1 and 2 (c) All three
(d) Neither 1 nor 2 (d) None
11 www.visionias.in ©Vision IAS

FREE BY KING R QUEEN P [ऋषभ राजपूत]


54. Consider the following statements: 56. Consider the following statements regarding
Statement-I: The 100th Constitutional the Global Partnership for Financial
Inclusion (GPFI):
Amendment was passed to enforce the
1. It is the main implementing organization
exchange of territories between India and
for the International Monetary Fund's
Bangladesh. Financial Inclusion Action Plan.
Statement-II: To resolve such disputes, 2. India and Italy will be the new co-chairs
Article 3 of the Indian Constitution provides of the GPFI for three years from 2024.
for a constitutional amendment under Article Which of the statements given above is/are
correct?
368.
(a) 1 only
Which one of the following is correct in (b) 2 only
respect of the above statements? (c) Both 1 and 2
(a) Both Statement-I and Statement-II are (d) Neither 1 nor 2
correct. and Statement-II is the correct
57. Ultraviolet Transient Astronomy Satellite
explanation for Statement-I
(ULTRASAT), a space telescope mission
(b) Both Statement I and Statement-II that is an ultraviolet observatory with a large
correct and Statement-II is not the· field of view is developed by
correct explanation for Statement-I (a) National Aeronautics and Space
(c) Statement-I is correct but Statement-II is Administration (NASA)
(b) Israeil Space Agency (ISA)
incorrect
(c) Indian Space Research Organisation
(d) Statement-I is incorrect but Statement-II
(ISRO)
is correct (d) The Japan Aerospace Exploration
Agency (JAXA)
55. With reference to British and Indian
58. Consider the following statements:
Parliamentary systems, consider the
Statement-I: W20 (Women 20) is an official
following statements:
engagement group under the G20 which was
1. British parliamentary system is based on an initiative of India established during its
the doctrine of the sovereignty of presidency.
Parliament. Statement-II: Its primary objective is to
ensure that gender considerations are
2. Unlike Britain, in India, a person who is
mainstreamed into G20 discussions.
not a member of Parliament can also be
Which one of the following is correct in
appointed as minister. respect of the above statements?
3. Unlike Britain, in India, ministers are (a) Both Statement-I and Statement-II are
required to countersign the official acts correct and Statement-II is the correct
of the Head of the State. explanation for Statement-I
(b) Both Statement-I and Statement-II are
How many statements given above are
correct and Statement-II is not the
correct? correct explanation for statement-I
(a) Only one (c) Statement-I is correct but Statement-II is
(b) Only two incorrect
(c) All three (d) Statement-I is incorrect but Statement-II
is correct
(d) None
12 www.visionias.in ©Vision IAS

FREE BY KING R QUEEN P [ऋषभ राजपूत]


59. Consider the following statements regarding 62. Consider the following countries :
Munich Security Conference : 1. Russia
1. It was an initiative of Germany in 2022 2. Kazakhstan
in reaction to Russian Invasion of
3. Armenia
Ukraine.
4. Uzbekistan
2. The conference is only focused on
5. Belarus
military issues.
Which of the statements given above is/are How many of the above-mentioned countries
correct? are members of the Collective Security
(a) 1 only Treaty Organisation (CSTO)?
(b) 2 only (a) Only two
(c) Both 1 and 2 (b) Only three
(d) Neither 1 nor 2 (c) Only four
(d) All five
60. Consider the following statements :
Statement-I: In USA a person is not only a
citizen of the country but also of the 63. Consider the following statements regarding
particular state to which he belongs. National Population Register:
Statement-II: All federal countries have a 1. It is mandatory for every usual resident
system of double citizenship. of India to register in the NPR.
Which one of the following is correct in 2. It includes both Indian citizens as well as
respect of the above statements? foreign citizens.
(a) Both Statement-I and Statement-II are
Which of the statements given above is/are
correct and Statement-II is the correct
correct?
explanation for Statement-I
(a) 1 only
(b) Both Statement-I and Statement-II are
correct and Statement-II is not the (b) 2 only
correct explanation for Statement-I (c) Both 1 and 2
(c) Statement-I is correct but Statement-II is (d) Neither 1 nor 2
incorrect
(d) Statement-I is incorrect but Statement-II 64. Consider the following statements regarding
is correct
Overseas Citizen of India Cardholder:
1. He shall not be entitled for registration
61. Consider the following statements regarding
as a voter.
the citizenship provisions in the Indian
constitution: 2. He shall not be entitled to the right to
1. Part III of the Indian Constitution deals equality of opportunity in matters of
with the citizenship aspect in India. public employment.
2. Constitution identifies the persons who 3. He shall be eligible to nominated as a
became citizens of India at its member of Rajya Sabha.
commencement i.e. on January 26, 1950. How many of the statements given above are
Which of the statements given above is/are
correct?
correct?
(a) Only one
(a) 1 only
(b) 2 only (b) Only two
(c) Both 1 and 2 (c) Only three
(d) Neither 1 nor 2 (d) None
13 www.visionias.in ©Vision IAS

FREE BY KING R QUEEN P [ऋषभ राजपूत]


65. Consider the following countries: 68. If a court violates the principles of natural
1. Indonesia justice, then it is the most suitable writ to be
granted by the superior courts. It is not
2. Brazil
available against legislative bodies and
3. Mexico private individuals or bodies.
How many of the countries given above Which of the following writs has been
described by the passage given above?
have a presidential form of government?
(a) Quo Warranto
(a) Only one (b) Certiorari
(b) Only two (c) Habeas Corpus
(c) All three (d) Mandamus

(d) None
69. Consider the following statements:
Statement-I: Preamble was enacted by the
66. In the context of Citizenship rights in India, Constituent Assembly after the rest of the
Constitution was already enacted.
consider the following statements regarding
Statement-II: The Preamble inserted in the
Assam Accord: Constitution at end was to ensure that it was
1. It states that all persons who came to in conformity with the Constitution as
adopted by the Constituent Assembly.
Assam prior to 1-1-1966 will be
Which one of the following is correct in
deported to their home country. respect of the above statements?
2. It was signed during the prime (a) Both Statement-I and Statement-II are
ministership of Indira Gandhi. correct and Statement-II is the correct
explanation for Statement-I
Which of the statements given above is/are
(b) Both Statement-I and Statement-II ate
correct? correct and Statement-II is not the
(a) 1 only correct explanation for Statement-I
(c) Statement-I is correct but Statement-II is
(b) 2 only
incorrect
(c) Both 1 and 2 (d) Statement-I is incorrect but Statement-II
(d) Neither 1 nor 2 is correct

70. Which of the following statements is correct


67. What is the primary objective of a direct
about the appointment of Judges in High
democracy? Courts?
(a) To maximize individual liberties and (a) Ad-hoc judges during the lack of
Quorum can be appointed by the
rights
President of India.
(b) To establish a system of checks and (b) Retired judges can be appointed
balances temporarily by the President of India.
(c) To concentrate power in the hands of a (c) Additional judges can be appointed by
the President of India.
small group of elites
(d) Acting judges can be appointed by the
(d) To involve citizens directly in decision- Chief Justice of the particular High
making through popular votes Court.
14 www.visionias.in ©Vision IAS

FREE BY KING R QUEEN P [ऋषभ राजपूत]


71. Consider the following statements with 73. Consider the following statements:

reference to the Article 16 of Indian Statement-I: Norovirus is a viral infection

Constitution: that affects mainly the Central Nervous


1. It prohibits discrimination both by the system in children below 6 years.
State and private individuals. Statement-II: The virus is capable of
2. Parliament can not prescribe residence surviving low temperatures and is highly
as a condition for employment or contagious.
appointment in a state or union territory. Which one of the following is correct in
3. The Court can not issue the mandamus respect of the above statements?
to provide reservation or promotion.
(a) Both Statement-I and Statement-II are
How many statements given above are
correct and Statement-II is the correct
correct?
explanation for Statement-I
(a) Only one
(b) Both Statement-I and Statement-II are
(b) Only two
correct and Statement-II is not the
(c) All three
correct explanation for Statement-I
(d) None
(c) Statement-I is correct but Statement-II is

incorrect
72. Consider the following statements:
(d) Statement-I is incorrect but Statement-II
1. Charaideo Moidams are known as the
is correct
‘Pyramids of Assam' because it contains

sacred burial grounds of Ahom kings


74. In the context of representative democracies
and queens.
in the world, the term gerrymandering refers
2. Charaideo city was the original capital
to
of the Ahom Kings.
(a) practice of allowing citizens to vote on
3. Charaideo city was built by Chaolung
proposed changes to district boundaries.
Sukhapa who was the founder of the
(b) system of appointing judges to oversee
Ahom dynasty.
the redistricting process.
How many of the above statements are
(c) manipulating district boundaries to gain
correct?

(a) Only one an unfair political advantage.

(b) Only two (d) legal and ethical practice used to

(c) All three maintain the integrity of district

(d) None boundaries.


15 www.visionias.in ©Vision IAS

FREE BY KING R QUEEN P [ऋषभ राजपूत]


75. Consider the following pairs: 77. Consider the following statements:
Alternate Definition
Statement-I: Each state of India has its own
Dispute
Resolution High Court.
(ADR)
Statement-II: The Constitution of India
Mechanism
1. : It is a non-binding procedure states that there shall be a High Court for
Arbitration in which an impartial third each state.
party assists the parties to a
dispute in reaching a Which one of the following is correct in
mutually satisfactory and respect of the above statements?
agreed-upon settlement of the
dispute. (a) Both Statement-I and Statement-II are

2. : It is a binding procedure in correct and Statement-II is the correct


Conciliation which an impartial third party
explanation for Statement-I

t
bo
assists the parties to a dispute

4_
in reaching a mutually (b) Both Statement-I and Statement-II are

02
satisfactory agreement on a
correct and Statement-II is not the

s2
settlement of the dispute.

rie
3. : It is a non-binding procedure correct explanation for statement-I
se
st
Negotiation of dispute resolution without (c) Statement-I is correct but Statement-II is
Te

the intervention of any third


@

party. incorrect
=
st

How many of the above pairs are correctly (d) Statement-I is incorrect but Statement-II
Te

matched?
ng

(a) Only one pair is correct


hi

(b) Only two pairs


ac
co

(c) All three pairs


78. Which of the following best describes the
ll

(d) None
ra

term "Communication Hotline" in


fo

76. Consider the following pairs with regard to


re

International relations?
he

the Office of Registrar General of India


m

(RGI): (a) Restriction or blockade of Internet data


D

1. It is mandatory to secure the approval of


flow between two countries.
the Office of the RGI for the inclusion of
any community in the Scheduled Tribe (b) The communication channel was opened
list.
by United Nations in disputed or war-
2. It comes under the Ministry of Home
Affairs. torn regions.
3. It gives estimates of fertility and
(c) Open and Interoperable communication
mortality using the Sample Registration
System. Network access agreement between two
How many of the above statements are
countries.
correct?
(a) Only one (d) A special and direct phone line between
(b) Only two the heads of government in different
(c) All three
(d) None countries.
16 www.visionias.in ©Vision IAS

FREE BY KING R QUEEN P [ऋषभ राजपूत]


79. Consider the following provisions in the 82. With reference to the Indian Independence
Constitution of India: Act 1947, consider the following statements:
1. Fundamental Rights 1. This act introduced an interim
2. Directive Principles of State Policy Constitution to be in effect until the day
3. Fundamental Duties of independence.
4. Fifth Schedule 2. As per this act, Constituent Assembly
How many of the above provisions of the was made a fully sovereign legislative
Constitution provide for the formation/ body.
promotion of Co-operative Societies? Which of the statements given above is/are
(a) Only one correct?
(b) Only two (a) 1 only
(c) Only three (b) 2 only
(d) All Four (c) Both 1 and 2
(d) Neither 1 nor 2
80. Jaipur Declaration, which is recently seen in
news is associated with which of the 83. Which of the following countries have a
following objectives? written constitution?
(a) To achieve responsible use of artificial 1. Israel
intelligence in the military 2. USA
(b) To end AIDS in children by 2030 3. France
(c) To achieve long-term goal of safety and Select the correct answer using the code
security of Railway organizations given below.
(d) To achieve the conservation and (a) 1 and 2 only
restoration of one trillion trees within (b) 3 only
this decade (c) 2 and 3 only
(d) 1, 2 and 3
81. Which of the following statements about the
seat of the Supreme Court is not correct? 84. Which of the statements given below is not
(a) The Constitution declares Delhi as the correct regarding the procedure for an
seat of the Supreme Court. amendment to the Constitution?
(b) The Constitution authorizes the (a) A private member can propose an
President of India to appoint any other amendment bill to the constitution.
place as the seat of the Supreme Court. (b) The introduction of a constituion
(c) The provision of declaration of the any amendment bill does not require prior
other place as the seat of the Supreme permission of the president.
Court is only optional and not (c) The constitutional amendment bill can
compulsory. be discussed in a joint sitting of two
(d) No court can give any direction either to houses for discussion.
the President or to the Chief Justice to (d) The President must give assent to
appoint any other place as a seat of the constitutional amendment bill passed by
Supreme Court. the parliament.
17 www.visionias.in ©Vision IAS

FREE BY KING R QUEEN P [ऋषभ राजपूत]


85. Consider the following Directive Principles 88. Amrit Dharohar Scheme which is announced
contained in the Part IV of the Indian in the 2023 Union Budget aims to
Constitution: (a) Facilitate the transition towards a low
1. Equal justice and free legal aid Carbon Economy.
2. Organisation of village panchayats (b) Promote alternative fertilizers and
3. Living wage for the workers balanced use of chemical fertilizers.
4. Participation of workers in the (c) Establish 500 new 'Waste to Wealth'
management of the industries Plants
How many of the above given Directive (d) Promote values of local communities in
Principles are based on the Gandhian conserving Wetland Ecosystem.
ideology?
(a) Only one 89. In the context of Citizenship in India, which
(b) Only two of the following are the modes of acquiring
(c) Only three citizenship as prescribed by 1955
(d) All four Citizenship Act?
1. Birth
86. Consider the following statements with 2. Descent
reference to the FIR and Charge Sheet: 3. Registration
1. FIR is a preliminary report while the 4. Naturalisation
charge sheet is a final report. 5. Incorporation of Territory
2. FIR is prepared by the investigation Select the correct answer using the code
officer while the charge sheet is given below.
(a) 1, 2, 3 and 4 only
prepared by the victim of the crime.
(b) 1, 4 and 5 only
3. FIR is filed in the police station while a
(c) 1, 2, 3 and 5 only
charge sheet is filed in court.
(d) 1, 2, 3, 4 and 5
How many of the above statements are
correct?
90. Consider the following Fundamental Rights
(a) Only one
mentioned in the Indian Constitution:
(b) Only two
1. Equality before law and equal protection
(c) All three
of laws (Article 14)
(d) None
2. Protection in respect of conviction for
offences (Article 20)
87. Consider the following statements with
3. Right to elementary education (Article
respect to 'Quo-Warranto' writ:
21A)
1. It is issued by the court to enquire into
4. Freedom to manage religious affairs
the legality of claim of a person to a
(Article 26)
public office.
5. Right of minorities to establish and
2. It can be sought by any interested person
administer educational institutions
and not necessarily by the aggrieved
(Article 30)
person. How many of the above given Fundamental
Which of the statements given above is/are Rights are available to both citizens of India
correct? and foreigners?
(a) 1 only (a) Only two
(b) 2 only (b) Only three
(c) Both 1 and 2 (c) Only four
(d) Neither 1 nor 2 (d) All five
18 www.visionias.in ©Vision IAS

FREE BY KING R QUEEN P [ऋषभ राजपूत]


91. Consider the following statements: 93. Consider the following statements regarding

Statement-I: Viceroy Canning nominated Agriculture and Processed Food Products

three Indians to the central council–the Raja Export Development Authority (APEDA):

of Benaras, the Maharaja of Patiala, and Sir 1. It is an apex organization under the

Dinkar Rao. Ministry of Commerce and Industry.

Statement-II: The Indian Councils Act 1861 2. Most agricultural products along with

provided for the first time the association of marine products are covered under

Indians with the executive councils. APEDA.

Which one of the following is correct in Which of the statements given above is/are
correct?
respect of the above statements?
(a) 1 only
(a) Both Statement-I and Statement-II are
(b) 2 only
correct and Statement-II is the correct
(c) Both 1 and 2
explanation for Statement-I
(d) Neither 1 nor 2
(b) Both Statement-I and Statement-II are

correct and Statement-II is not the


94. Which of the following are generally
correct explanation for Statement-I
considered duties of citizenship?
(c) Statement-I is correct but Statement-II is
1. Paying taxes
incorrect
2. Defending the country
(d) Statement-I is incorrect but Statement-II
3. Owning property
is correct
Select the correct answer using the code
given below.
92. Consider the following constitutional
(a) 1 only
features:
(b) 1 and 2 only
1. Doctrine of Sovereignty of Parliament
(c) 2 and 3 only
2. Presence of a nominal Executive
(d) 1, 2 and 3
3. Dissolution of the lower house of the

Parliament 95. According to Dr. B.R. Ambedkar which of


How many of the above are the essential the following is a ‘novel feature’ of the
features of the Parliamentary government as Indian Constitution?
provided under the Constitution of India? (a) Right to Constitutional Remedies
(a) Only one (b) Provisions of National Emergency
(b) Only two (c) Directive Principles of State Policy
(c) All three (d) Indestructible Nature of Indian
(d) None Federation
19 www.visionias.in ©Vision IAS

FREE BY KING R QUEEN P [ऋषभ राजपूत]


96. With reference to the Citizenship Act 1955, 98. Consider the following statements about the

in which of the following cases the contempt of court:


1. Wilful disobedience of any judgment of
citizenship of a person can be deprived by
the court is a criminal contempt.
the government? 2. The consent of the Attorney General of
1. If a citizen has, within five years after India is mandatory when a private

registration or naturalization, been citizen wants to initiate a case of


contempt of court against a person.
imprisoned in any country for two years.
3. At present, there is no separate legal law
2. If a citizen has been ordinarily residing dealing with contempt of court.
out of India for seven years How many of the above statements are
correct?
continuously.
(a) Only one
3. If a citizen has shown disloyalty to the
(b) Only two
Constitution of India. (c) All three
Select the correct answer using the code (d) None

given below.
99. Consider the following statements:
(a) 1 and 3 only
Statement-I: The Fazl Ali Commission
(b) 2 and 3 only accepted the theory of ‘one language-one
(c) 1 only state’ in principle.
Statement-II: The Commission viewed that
(d) 1, 2 and 3
the unity and integrity of India was a prime
consideration.
97. Project 39A, recently seen in the news, is Which one of the following is correct in
associated with: respect of the above statements?
(a) Both Statement-I and Statement-II are
(a) Indian Navy’s 30 years plan for
correct. and Statement-II is the correct
indigenous nuclear submarine
explanation for Statement-I
construction. (b) Both Statement-I and Statement-II

(b) criminal reforms advocacy group. correct and Statement-II is not the·
correct explanation for Statement-I
(c) a target of doubling India’s nuclear
(c) Statement-I is correct but Statement-II is
capacity by 2039.
incorrect
(d) an aim to eliminate lymphatic filariasis (d) Statement-I is incorrect but Statement-II

by 2039 is correct

20 www.visionias.in ©Vision IAS

FREE BY KING R QUEEN P [ऋषभ राजपूत]


100. Consider the following personalities:
1. Sucheta Kriplani
2. Usha Mehta
3. Captain Lakshmi Sahgal
4. Begum Aizaz Rasul
5. Annie Mascarene
How many of the above leaders were part of
the Constituent Assembly of India?
(a) Only two
(b) Only three
(c) Only four
(d) All five

Copyright © by Vision IAS


All rights are reserved. No part of this document may be reproduced, stored in a retrieval system or
transmitted in any form or by any means, electronic, mechanical, photocopying, recording or otherwise,
without prior permission of Vision IAS.
21 www.visionias.in ©Vision IAS

FREE BY KING R QUEEN P [ऋषभ राजपूत]


VISIONIAS
www.visionias.in

Test Booklet Series

TEST BOOKLET

GENERAL STUDIES (P) 2024 – Test – 4135


C
Time Allowed: Two Hours Maximum Marks: 200

INSTRUCTIONS

1. IMMEDIATELY AFTER THE COMMENCEMENT OF THE EXAMINATION, YOU SHOULD CHECK THAT THIS BOOKLET
DOES NOT HAVE ANY UNPRINTED OR TURN OR MISSING PAGES OR ITEMS, ETC. IF SO, GET IT REPLACED BY A
COMPLETE TEST BOOKLET.

2. ENCODE CLEARLY THE TEST BOOKLET SERIES A, B, C OR D AS THE CASE MAY BE IN THE APPROPRIATE PLACE IN
THE ANSWER SHEET.

3. You have to enter your Roll Number on the Test Booklet in the Box
provided alongside. Do NOT write anything else on the Test Booklet.

4. This Test Booklet contains 100 items (Questions). Each item is printed in English. Each item comprises four
responses (answers). You will select the response which you want to mark on the Answer Sheet. In case you
feel that there is more than one correct response with you consider the best. In any case, choose ONLY ONE
response for each item.

5. You have to mark all your responses ONLY on the separate Answer Sheet provided. See direction in the
answers sheet.

6. All items carry equal marks. Attempt all items. Your total marks will depend only on the number of correct
responses marked by you in the answer sheet. For every incorrect response 1/3rdof the allotted marks will be
deducted.

7. Before you proceed to mark in the Answer sheet the response to various items in the Test booklet, you have to
fill in some particulars in the answer sheets as per instruction sent to you with your Admission Certificate.

8. After you have completed filling in all responses on the answer sheet and the examination has concluded, you
should hand over to Invigilator only the answer sheet. You are permitted to take away with you the Test
Booklet.

9. Sheet for rough work are appended in the Test Booklet at the end.

DO NOT OPEN THIS BOOKLET UNTIL YOU ARE ASKED TO DO SO


1 www.visionias.in ©Vision IAS

FREE BY KING R QUEEN P [ऋषभ राजपूत]


1. Consider the following states: 4. With reference to the Indian Parliament,
1. Madhya Pradesh which of the following statements best
2. Bihar describes the term 'recess'?
3. Chhattisgarh
(a) It is the period between the prorogation
4. Odisha
of Parliament and its reassembly in a
The Constitution of India provides the
appointment of a tribal welfare minister for new session.
how many of the above states? (b) It is the period between the adjournment
(a) Only one of Parliament and its reassembly on a
(b) Only two new day.
(c) Only three (c) It is the period between the winter
(d) All four
session and the budget session of the
Parliament.
2. In the context of legal jurisprudence in India,
the Doctrine of Promissory Estoppel is a: (d) It is the gap of six months between two
(a) legal principle that gives conditional sessions of the Parliament.
immunity to non-cognizable crimes.
(b) legal concept associated with the 5. Consider the following statements regarding
contract laws.
Depleted Uranium:
(c) principle in environmental law that
1. It is a by-product of the process of
encourages companies to adopt
creating enriched uranium.
sustainable practices voluntarily.
(d) principle in federalism that grants 2. It is highly radioactive and is capable of
exclusive powers to the federal generating a sustained nuclear reaction.
government. Which of the statements given above is/are
correct?
3. Consider the following statements:
(a) 1 only
Statement I: The Speaker can not vote in the
(b) 2 only
first instance but he can exercise a casting
(c) Both 1 and 2
vote in the case of equality of votes.
Statement II: The absence of a vote in the (d) Neither 1 nor 2
first instance will make the position of the
Speaker impartial. 6. Consider the following pairs:
Which one of the following is correct in Volcano in News Country
respect of the above statements?
1. Mount Merapi : Kenya
(a) Both Statement I and Statement II are
2. Tanaga : Indonesia
correct, and Statement II is the correct
explanation for Statement I. 3. Shiveluch : Colombia
(b) Both Statement I and Statement II are How many of the above given pairs are
correct, and Statement II is not the correctly matched?
correct explanation for Statement I. (a) Only one pair
(c) Statement I is correct, but Statement II is (b) Only two pairs
incorrect.
(c) All three pairs
(d) Statement I is incorrect, but Statement II
(d) None of the pairs
is correct.
2 www.visionias.in ©Vision IAS

FREE BY KING R QUEEN P [ऋषभ राजपूत]


7. Consider the following: 9. Consider the following statements with

1. Framing draft legislation, and rules and respect to the joint sitting of the Parliament:
regulations 1. Those amendments that have caused
2. Assisting the ministers in policy-making final disagreement between the Houses
3. Budgeting and control of expenditure will not be proposed during the joint
4. Maintaining contact with the
sitting.
Government of India and other state
2. Its provision is applicable to ordinary
governments
bills or financial bills only and not to
How many of the above functions are
money bills or Constitutional
performed by the State Secretariat?
amendment bills.
(a) Only one
3. It is borrowed from the Constitution of
(b) Only two
Australia.
(c) Only three
How many of the above statements are
(d) All four
correct?

8. Consider the following statements in respect (a) Only one

of the discretionary power of the President (b) Only two

of India: (c) All three

1. He can appoint a Prime Minister when (d) None

no party has a clear majority in the Lok

Sabha. 10. Consider the following statements regarding


2. He can dismiss the council of ministers trade of biofuels in India:
when it cannot prove the confidence in 1. Export of biofuels is banned from
the Lok Sabha. Special Economic Zones (SEZs).
3. He can dissolve the Lok Sabha if the
2. Both import and export of biofuels
council of ministers has lost its majority
requires a license.
in the Lok Sabha.
Which of the statements given above is/are
How many of the above statements are
correct?
correct?
(a) 1 only
(a) Only one
(b) 2 only
(b) Only two
(c) Both 1 and 2
(c) All three
(d) Neither 1 nor 2
(d) None
3 www.visionias.in ©Vision IAS

FREE BY KING R QUEEN P [ऋषभ राजपूत]


11. The Parliament enjoys territorial jurisdiction 14. Consider the following pairs:
to make laws for the whole or any part of the Product with GI Tag Associated State
territory of India but the President is 1. Sundarja mango : Madhya Pradesh
empowered to direct that an act of
2. Basohli painting : Uttarakhand
Parliament does not apply to a tribal area
3. Nagri Dubraj rice : Assam
(autonomous district) in certain states.
The President has such power in respect of How many of the pairs given above are
which of the following states? correctly matched?
1. Assam (a) Only one
2. Meghalaya (b) Only two
3. Tripura
(c) All three
4. Mizoram
(d) None
Select the correct answer using the code
given below.
(a) 1, 2 and 3 only 15. Operation Interflex recently seen in the news
(b) 2, 3 and 4 only aims to:
(c) 1 and 4 only (a) provide military aid and support to
(d) 1, 2, 3 and 4 Ukraine.
(b) prevent illegal trade of endangered
12. In the context of defence sector in India, the
turtles.
PALM 400 recently seen in news is a/an:
(a) attack drone (c) prevent volatility in the supply of
(b) military exercise between India and Five semiconductor chips.
Central Asian countries (d) prevent illegal migration across the
(c) border security system to prevent illegal Mediterranean sea.
migration
(d) prototype humanoid soldier
16. Consider the following statements regarding
the election of the Vice-President:
13. Consider the following statements regarding
the removal of the Vice-President: 1. The original Constitution provided that
1. He can be removed by a resolution the vice president would be elected by
passed by a majority of all the then the two Houses of Parliament assembled
members of the Rajya Sabha and agreed at a joint meeting.
to by the Lok Sabha.
2. The Election Commission, in
2. This resolution can be introduced only in
consultation with the Central
the Rajya Sabha and not in the Lok
Government, appoints the Secretary-
Sabha by giving at least 14 days advance
notice. General of Lok Sabha and Rajya Sabha,
3. No ground has been mentioned in the by rotation, as the Returning Officer.
Constitution for his removal. Which of the statements given above is/are
How many of the above statements are correct?
correct?
(a) 1 only
(a) Only one
(b) 2 only
(b) Only two
(c) All three (c) Both 1 and 2
(d) None (d) Neither 1 nor 2
4 www.visionias.in ©Vision IAS

FREE BY KING R QUEEN P [ऋषभ राजपूत]


17. With reference to the Comptroller and 19. With reference to Department-Related

Auditor-General (CAG) of India, consider Standing Committees (DRSCs) in the

the following statements: Parliament of India, consider the following

1. CAG is appointed by the President by statements:

warrant under his hand and seal and 1. DRSCs under the purview of the Rajya

provided with tenure of 6 years or 65 Sabha have members only from the

years of age, whichever is earlier. Rajya Sabha.

2. He is eligible to hold any office after 2. They can propose cut motions related to

retirement, either under the Government the 'demand for grants' of the respective

ministry or department.
of India or of any state, with prior
Which of the statements given above is/are
permission from the President.
correct?
Which of the statements given above is/are
(a) 1 only
correct?
(b) 2 only
(a) 1 only
(c) Both 1 and 2
(b) 2 only
(d) Neither 1 nor 2
(c) Both 1 and 2

(d) Neither 1 nor 2


20. Consider the following statements regarding

the Comprehensive and Progressive


18. Consider the following statements:
Agreement for Trans-Pacific Partnership
1. The property of the central government
(CPTPP):
is exempted from all taxes levied by the
1. USA is not a member of this trade
state or any authority within the state.
agreement.
2. The property and income of state
2. Its combined GDP is more than that of
governments and local governments
the Regional Comprehensive Economic
situated within a state are exempted
Partnership (RCEP).
from union taxation.
3. It entered into force in 2023.
Which of the statements given above is/are
How many of the statements given above are
correct?
correct?
(a) 1 only (a) Only one
(b) 2 only (b) Only two
(c) Both 1 and 2 (c) All three
(d) Neither 1 nor 2 (d) None
5 www.visionias.in ©Vision IAS

FREE BY KING R QUEEN P [ऋषभ राजपूत]


21. Article 252 of the Indian Constitution deals 24. Consider the following statements regarding
with Parliament's power to legislate on the the powers of the Indian President:
subject of state list when states make a 1. He has the right to address either House
or their joint sitting, at any time, and to
request. In this context, consider the
require the attendance of members for
following statements:
this purpose.
1. A law so enacted applies only to those 2. He has the right to send messages to
states which have passed the resolutions. either House of Parliament regarding
2. Such a law made by parliament can only any pending Bill and the House must
be amended in parliament. consider the message with all convenient
despatch.
Which of the statements given above is/are
Which of the statements given above is/are
correct?
correct?
(a) 1 only (a) 1 only
(b) 2 only (b) 2 only
(c) Both 1 and 2 (c) Both 1 and 2
(d) Neither 1 nor 2 (d) Neither 1 nor 2

25. Consider the following:


22. Consider the following statements regarding
1. Bengaluru
National Generic Document Registration 2. Delhi
System: 3. Hyderabad
1. It was initiated by the Ministry of How many of the places given above
Electronics and Information experience Zero Shadow Day every year?
(a) One only
Technology.
(b) Two only
2. It enables the citizens to buy land online.
(c) All three
Which of the statements given above is/are (d) None
correct?
(a) 1 only 26. With reference to the office of Chief
(b) 2 only Secretary of a state, consider the following
statements:
(c) Both 1 and 2
1. The Chief Secretary is the head of the
(d) Neither 1 nor 2
General Administration department
whose political head is the Chief
23. LIQUID Tree or LIQUID 3, recently seen in Minister herself/himself.
the news, is used to: 2. The scope of his duties and powers in
(a) reduce greenhouse gas emissions and the state administration is narrower than
the Cabinet Secretary in the Central
improve air quality
Government.
(b) trap solar energy more efficiently than
Which of the statements given above is/are
the conventional photovoltaic correct?
technology (a) 1 only
(c) prevent forest fires (b) 2 only
(d) purify metallic pollutants from drinking (c) Both 1 and 2
(d) Neither 1 nor 2
water
6 www.visionias.in ©Vision IAS

FREE BY KING R QUEEN P [ऋषभ राजपूत]


27. Consider the following statements regarding 30. With regards to the Ministry of Information
Central Vigilance Commission (CVC): and Broadcasting, consider the following
1. It is an independent body which is only statements:
responsible to the Parliament.
1. Under the Information Technology
2. It annually presents its report to the
Union Government. Rules 2021, the ministry has extended its
3. The commission was given statutory regulatory powers over internet content
status in the year 1964. as well.
How many of the above statements are 2. Prasar Bharati is the statutory and
correct?
autonomous body under this ministry.
(a) Only one
Which of the statements given above is/are
(b) Only two
(c) All three correct?
(d) None (a) 1 only
(b) 2 only
28. Consider the following statements: (c) Both 1 and 2
1. Article 53 confers all executive powers
(d) Neither 1 nor 2
in President and empowers him to
exercise these powers directly by
himself or through officers subordinate 31. Consider the following executive offices:
to him. 1. President
2. Article 77 holds that all executive 2. Attorney General of India
powers of the Union government shall 3. Council of Ministers
be exercised in the name of the
How many of the above offices enjoy
President.
Parliamentary privileges?
Which of the statements given above is/are
correct? (a) Only one
(a) 1 only (b) Only two
(b) 2 only (c) Only three
(c) Both 1 and 2 (d) None
(d) Neither 1 nor 2

29. Consider the following statements about 32. Consider the following statements regarding
Solicitor General of India (SGI): the International Criminal Court:
1. Solicitor General is the second most 1. It has no power to arrest sitting heads of
senior law officer in India after Attorney state or bring them to trial.
General.
2. It is established under the United
2. The office of the Solicitor General is not
Nations Charter.
mentioned in the Constitution of India.
3. SGI is appointed by the Prime Minister 3. Its headquarters is located in Geneva,
on the advice of the Attorney General of Switzerland.
India. How many of the statements given above are
How many of the statements given above are correct?
correct?
(a) Only one
(a) Only one
(b) Only two
(b) Only two
(c) All three (c) All three
(d) None (d) None
7 www.visionias.in ©Vision IAS

FREE BY KING R QUEEN P [ऋषभ राजपूत]


33. In the context of space, which of the 35. With reference to the state council of

following statements best describes the ministers, consider the following statements:
1. The Governor can dismiss a Council of
"Overview Effect"?
Ministers at his/her pleasure on his/her
(a) It is the physical strain experienced by subjective estimate of the strength of the
astronauts during spacewalks. Chief Minister in the State Assembly.
2. A minister who is a member of one
(b) It is the cognitive shift experienced by
House of the state legislature has the
astronauts when viewing Earth from
right to take part in the proceedings and
space vote in the other House.

(c) It is the rapid increase in technological Which of the statements given above is/are
correct?
advancements in space exploration
(a) 1 only
(d) It is the apparent disappearance of moon (b) 2 only
as observed the astronauts from space (c) Both 1 and 2
(d) Neither 1 nor 2

34. With reference to the Prime Minister's Office


36. Consider the following states in India:
(PMO), consider the following statements: 1. Mizoram

1. It is mainly responsible for functions 2. Sikkim


3. Arunachal Pradesh
involving the Prime Minister in his
4. Nagaland
capacity as the head of the cabinet. How many of the above are outside the
2. The PMO is administratively headed by purview of the 73rd amendment act,1992?
(a) Only one
the Principal Secretary.
(b) Only two
3. The PMO enjoys neither a Constitutional
(c) Only three
nor a Departmental status under the (d) All four

Government of India.
37. Consider the following statements regarding
4. The National Authority Chemical
online gaming regulations in India:
Weapons Conventions Functions under 1. The term "Online games" has not been
the PMO. defined in any rules.
2. Online games that involve any kind of
How many of the statements given above are
gambling are prohibited.
correct?
Which of the statements given above is/are
(a) Only one correct?

(b) Only two (a) 1 only


(b) 2 only
(c) Only three
(c) Both 1 and 2
(d) All Four (d) Neither 1 nor 2
8 www.visionias.in ©Vision IAS

FREE BY KING R QUEEN P [ऋषभ राजपूत]


38. With reference to the office of the whip in 40. Consider the following statements:
the Indian Parliament, consider the following 1. The 24th Constitutional Amendment Act
of 1971 made it obligatory for the
statements:
President to give his assent to a
1. It is mentioned in the Rules of the constitutional amendment bill.
House. 2. The 42nd Constitutional Amendment
2. Every political party, whether ruling or Act of 1976 made the President bound
by the advice of the council of ministers
opposition has its own whip in the
headed by the prime minister.
Parliament. Which of the statements given above is/are
3. The violation of the directives of the correct?
whip could lead to suspension of the (a) 1 only
(b) 2 only
membership of the house.
(c) Both 1 and 2
How many of the statements given above are (d) Neither 1 nor 2
correct?
(a) Only one 41. Consider the following statements regarding
Cabinet Committees:
(b) Only two
1. They were originally mentioned in the
(c) All three Constitution.
(d) None 2. They are of two types- standing and ad
hoc.
3. The non-Cabinet Ministers are not
39. With reference to the office of the Prime
debarred from their membership.
Minister of India, consider the following How many of the statements given above are
statements: correct?
1. The Constitution of India does not (a) Only one
(b) Only two
require that a person must prove his
(c) All three
majority in the Lok Sabha before he is (d) None
appointed as the Prime Minister.
2. The serving Prime Minister is immune 42. Consider the following statements:
1. Goods & Services Tax Council (GST
from any inquiry by the Lokpal for
Council) is a constitutional body for
allegations of corruption. making recommendations to the Union
3. The Prime Minister creates different and State Governments on issues related
ministries for the transaction of business to Goods and Service Tax.
2. The recommendations of the council are
of the Government.
not binding on the union and state
How many of the statements given above are governments.
correct? Which of the statements given above is/are
(a) Only one correct?
(a) 1 only
(b) Only two
(b) 2 only
(c) Only three (c) Both 1 and 2
(d) None (d) Neither 1 nor 2
9 www.visionias.in ©Vision IAS

FREE BY KING R QUEEN P [ऋषभ राजपूत]


43. Consider the following statements with 46. Consider the following statements regarding
reference to Committee on Public
the International Fund for Agricultural
Undertakings:
1. It was created in 1964 on the Development (IFAD):
recommendation of the John Mathai 1. It is a specialized agency of the United
Committee.
Nations.
2. The chairman of the committee is
appointed from the members who are 2. India is not a member of IFAD.
drawn from the Lok Sabha only.
3. IFAD was an outcome of the first World
Which of the statements given above is/are
correct? Food Conference in 1974.
(a) 1 only How many of the statements given above are
(b) 2 only
correct?
(c) Both 1 and 2
(d) Neither 1 nor 2 (a) Only one

(b) Only two


44. Consider the following statements regarding
the Multi-Party Interim Appeal Arbitration (c) All three
Arrangement (MPIA) often seen in the news: (d) None
1. It is an alternative system for resolving
WTO disputes.
2. India is a member of this arrangement. 47. Which of the following best describes
Which of the statements given above is/are
"Competitive Federalism"?
correct?
(a) 1 only (a) It profess a vertical relationship between
(b) 2 only the States and the Central government
(c) Both 1 and 2
while states compete among
(d) Neither 1 nor 2
themselves.
45. With reference to the “Rules of Procedure
(b) It profess equality of all states and
and Conduct of Business in the Lok Sabha”,
consider the following statements: central government should handhold
1. The 'head' of the Lok Sabha is the backward states.
chairman of its recommending
(c) It profess a horizontal relationship
committee.
2. The Rules recommended requiring the between the states and Central
approval of the house.
Government where each compete for
Which of the statements given above is/are
correct? limited resources.
(a) 1 only (d) It profess a vertical relationship between
(b) 2 only
all the states competing among
(c) Both 1 and 2
(d) Neither 1 nor 2 themselves.
10 www.visionias.in ©Vision IAS

FREE BY KING R QUEEN P [ऋषभ राजपूत]


48. With reference to the double membership in 50. In the context of climate change, the
the Indian Parliament, consider the following "Esbjerg Declaration" is related to
(a) Renewable energy projects in North Sea
statements:
(b) preventing access to Russian merchant
1. If a person is chosen for membership in
vessels through Kiel canal.
both Houses of Parliament, his seat in (c) European Union's energy self
the Lok Sabha becomes vacant by sufficiency by 2030
default. (d) tax evasion by Multi National
2. If a member is elected to the Union Companies

Parliament and a State Legislature, his


51. Which of the following organizations has
seat in Parliament becomes vacant if he
recently launched Tropospheric Emissions:
does not resign his seat in the state Monitoring of Pollution (TEMPO)?
legislature within six months. (a) National Aeronautics and Space
Which of the statements given above is/are Administration
(b) European Space Agency
correct?
(c) World Meteorological Organization
(a) 1 only
(d) International Civil Aviation
(b) 2 only Organization
(c) Both 1 and 2
(d) Neither 1 nor 2 52. Consider the following statements regarding
gravity batteries often seen in the news:
1. They can harness and store energy from
49. Consider the following statements regarding
gravitational waves.
the duties of the Prime Minister for
2. Abandoned and decommissioned mines
furnishing information to the President: could be turned into gravity batteries.
1. All decisions of the Council of Ministers Which of the statements given above is/are
relating to the administration of the correct?
affairs of the Union and proposals for (a) 1 only
(b) 2 only
legislation.
(c) Both 1 and 2
2. To furnish such information relating to (d) Neither 1 nor 2
the administration of the affairs of the
Union and proposals for legislation as 53. Consider the following statements regarding
the President may call for. the Commission for Agriculture Costs and
Prices:
3. To submit for the consideration of the
1. It was established as an advisory body
Council of Ministers any matter on
under the Ministry of Finance.
which a decision has been taken by a 2. It is an expert body that recommends the
Minister but which has not been Minimum Support Price of the notified
considered by the Council. Kharif and Rabi crops to the Cabinet
How many of the statements given above are Committee on Economic Affairs.
Which of the statements given above is/are
correct?
correct?
(a) Only one (a) 1 only
(b) Only two (b) 2 only
(c) All three (c) Both 1 and 2
(d) None (d) Neither 1 nor 2
11 www.visionias.in ©Vision IAS

FREE BY KING R QUEEN P [ऋषभ राजपूत]


54. Consider the following statements: 57. In the context of genetics, chimera is a/an:
1. For the enlargement of the jurisdiction (a) genetic disorder resulting in the
of the state public service commission, abnormal development of limbs in an
the state legislative assembly and organism.
council share equal powers. (b) organism whose cells are derived from
2. When an ordinary bill, which has two or more zygotes.
originated in the state legislative council (c) type of genetic engineering technique
and was sent to the assembly, is rejected used to create hybrid organisms.
by the assembly, the bill is sent to the (d) genetic mutation that causes an organism
council for reconsideration. to have an unusually long lifespan.
3. The state legislative council has no
effective say in the ratification of a 58. Consider the following statements:
constitutional amendment bill. Statement-I: Six Fundamental Rights under
How many of the statements given above are Article 19 can be suspended only when the
correct? National Emergency is declared on the
(a) Only one ground of war or external aggression and not
(b) Only two on the ground of armed rebellion.
(c) All three Statement-II: Article 359 of the Indian
(d) None Constitution deals with the suspension of the
Fundamental Rights guaranteed by Article
55. Consider the following statements: 19.
1. The public bill is introduced by a Which one of the following is correct in
minister whereas a private bill can be respect of the above statements?
introduced by any member of the (a) Both Statements-I and Statement-II are
parliament other than the minister. correct and Statement- II is the correct
2. Introduction of public bills require seven explanation for Statement-I
days’ notice, unlike private bill which (b) Both Statement-I and Statement-II are
needs at least a fifteen days notice. correct and Statement II is not the
Which of the statements given above is/are correct explanation for Statement-I
correct? (c) Statement-I is correct but Statement II is
(a) 1 only incorrect
(b) 2 only (d) Statement-I is incorrect but Statement-II
(c) Both 1 and 2 is correct
(d) Neither 1 nor 2
59. With reference to the Union Executive,
56. In the context of medical science, which of consider the following statements:
the following statements about Proton Beam 1. The Cabinet Secretary is the head of
Therapy is correct? Civil Services in India.
(a) It uses X-Rays to target cancer cells. 2. The Cabinet Secretariat functions
(b) It is a type of chemotherapy. directly under the Prime Minister.
(c) It has a higher risk of side effects Which of the statements given above is/are
compared to traditional radiation not correct?
therapy. (a) 1 only
(d) It delivers a more precise dose of (b) 2 only
radiation compared to conventional (c) Both 1 and 2
radiation therapy. (d) Neither 1 nor 2
12 www.visionias.in ©Vision IAS

FREE BY KING R QUEEN P [ऋषभ राजपूत]


60. Consider the following statements regarding 63. Mahajan Commission, which is recently
Commission for Conservation of Antarctic seen in the news is related to
Marine Living Resources: (a) Resolve the border dispute in
Maharashtra, Karnataka and Kerala.
1. Global Environment Facility acts as the
(b) Resolve bifurcation issues between
principal financial arm of the
Telangana and Andhra Pradesh
commission. (c) Recommend Krishna and Godavari
2. India is not a member of this water sharing agreement between
commission. Maharashtra, Karnataka, Telangana and
Which of the statements given above is/are Andhra Pradesh.
(d) Recommend and resolve issues arising
correct?
from the Ken-Betwa Link Project
(a) 1 only
(KBLP).
(b) 2 only
(c) Both 1 and 2 64. With reference to the composition of Lok
(d) Neither 1 nor 2 Sabha, consider the following statements:
1. Each state is allotted a number of seats
61. In the context of Space voyages, consider the in such a manner that the ratio between
that number and its population is the
following statements regarding JUICE
same for all states.
mission:
2. Each state is divided into territorial
1. It aims to study Mars and its two moons constituencies in such a manner that the
Phobos and Deimos. ratio between the population of each
2. It has been launched by NASA. constituency and the number of seats
Which of the statements given above is/are allotted to it is the same throughout the
state.
correct?
Which of the statements given above is/are
(a) 1 only
correct?
(b) 2 only (a) 1 only
(c) Both 1 and 2 (b) 2 only
(d) Neither 1 nor 2 (c) Both 1 and 2
(d) Neither 1 nor 2
62. Consider the following statements regarding
65. Which of the following statements
the Consolidated Fund of India:
accurately describes the Arm's Length
1. It is a fund to which all receipts of the
Principle (ALP) in international taxation?
government are credited, and all (a) It ensures that transactions between
payments are debited. related parties are priced based on their
2. It is at the disposal of the President, to actual costs.
meet unforeseen expenditures pending (b) It aims to maximize tax revenue for the
host country.
its authorisation by the Parliament.
(c) It requires transactions between related
Which of the statements given above is/are
parties to be priced as if they were
correct? conducted by independent and unrelated
(a) 1 only entities
(b) 2 only (d) It allows taxpayers to deduct a portion of
(c) Both 1 and 2 their income for various business
(d) Neither 1 nor 2 expenses
13 www.visionias.in ©Vision IAS

FREE BY KING R QUEEN P [ऋषभ राजपूत]


66. Consider the following statements: 68. In the context of climate change, the Global
Statement-I: With respect to Indians residing Greenhouse Gas Monitoring Infrastructure
(GGMI) was launched by:
in any part of the world, Parliament has
(a) United Nations Framework Convention
exclusive power to make laws regarding any
on Climate Change
matter in the Union list and shared right in (b) World Meteorological Organization
respect of both the State list and the (c) United Nations Environment Programme
Concurrent list. (d) Climate Action Network
Statement-II: With respect to the concurrent
69. With reference to India, consider the
list, laws made earlier or later by parliament
following statements:
prevail over laws made by state legislatures.
1. A person must have attained the age of
Which one of the following is correct in 25 years to be eligible for being a
respect of the above statements? member of a Municipality.
(a) Both Statement-I and Statement-II are 2. The State Election Commission has the
correct and Statement-II is the correct power of superintendence, direction, and
control over the conduct of all elections
explanation for Statement-I
in the Municipalities.
(b) Both Statement-I and Statement-II are
Which of the statements given above is/are
correct and Statement-II is not the correct?
correct explanation for Statement-I (a) 1 only
(c) Statement-I is correct but Statement-II is (b) 2 only
incorrect (c) Both 1 and 2
(d) Neither 1 nor 2
(d) Statement-I is incorrect but Statement-II
is correct
70. Consider the following pairs:
Department Function
67. With reference to the oath and affirmation of 1. Department : Preparation of the
members in the Indian Parliament, consider of Economic Union Budget
the following statements: Affairs
2. Department : Implementation of the
1. Third schedule of the constitution
of Expenditure recommendations of
contains the oath of members along with
the Finance
the President of the country. Commission
2. A person is liable to a penalty of ₹500 3. Department : Control production
for each day he sits or votes as a member of Revenue and disposal of opium
in a house before taking and subscribing 4. Department : Promotion of Financial
of Financial inclusion
to the prescribed oath or affirmation.
Services
Which of the statements given above is/are
How many of the pairs given above are
correct? correctly matched?
(a) 1 only (a) Only one
(b) 2 only (b) Only two
(c) Both 1 and 2 (c) Only three
(d) All four
(d) Neither 1 nor 2
14 www.visionias.in ©Vision IAS

FREE BY KING R QUEEN P [ऋषभ राजपूत]


71. The Minister of State in the Central 74. Consider the following statements regarding
Government is
Enemy Property in India:
1. Also a cabinet minister
2. A part of the Central Council of 1. The Ministry of Home Affairs is
Ministers
responsible for the overall management
3. Appointed by the Prime Minister
How many statements given above are and preservation of enemy property.
correct?
2. It has been monetised by the government
(a) Only one
(b) Only two as part of National Monetisation
(c) All three
Pipeline.
(d) None
Which of the statements given above is/are
72. With reference to the National Capital
correct?
Territory of Delhi (NCT), consider the
following statements: (a) 1 only
1. Article 239AA conferred special status
(b) 2 only
on Delhi following the recommendations
of the S Balakrishnan Committee. (c) Both 1 and 2
2. The parliament cannot make laws with
(d) Neither 1 nor 2
respect to subjects in state list for
National Capital territory of Delhi.
Which of the statements given above is/are
75. Doha Programme of Action recently seen in
correct?
(a) 1 only the news aims to:

(b) 2 only (a) help Least Developed Countries in


(c) Both 1 and 2
(d) Neither 1 nor 2 achieving UN mandated Sustainable

Development Goals (SDGs).


73. Which of the following countries are the
members of Colombo Security Conclave? (b) reverse the desertification in West Asia

1. Sri Lanka by 30 percent by 2030.


2. India
3. Seychelles (c) increase mangrove cover by 50 percent

Select the correct answer using the code by 2030 in Tropical countries.
given below.
(d) balance the use of fossil fuels and
(a) 1 only
(b) 1 and 2 only renewable energy to achieve a just
(c) 2 and 3 only
energy transition in West Asia.
(d) 1, 2 and 3
15 www.visionias.in ©Vision IAS

FREE BY KING R QUEEN P [ऋषभ राजपूत]


76. With reference to a financial emergency 79. Consider the following statements regarding
under Article 360 of the Indian constitution, duties levied by the union but collected and
consider the following statements: appropriated by the states:
1. The proclamation of financial 1. Such duties are collected by the
emergency must be approved by both Government of India and are leviable in
Houses of Parliament within one month any union territory.
of its issue. 2. The proceeds from such duties levied
2. The Financial Emergency, once within any state shall not form part of
approved by both the Houses of the Consolidated Fund of India.
Parliament continues indefinitely till it is
Which of the statements given above is/are
revoked.
correct?
3. President can issue directions for the
(a) 1 only
reduction of salaries and allowances of
(b) 2 only
any class of persons serving in
(c) Both 1 and 2
connection with affairs of the Union
(d) Neither 1 nor 2
except the Judges of the Supreme Court
and High Courts.
80. In India, which of the following
How many of the above statements are
correct? Constitutional Amendments inserted the
(a) Only one phrase 'armed rebellion', replacing the
(b) Only two original phrase 'internal disturbance' as one
(c) All three of the grounds for the proclamation of
(d) None emergency under Article 352?
(a) 24th Amendment
77. Which of the following articles states that (b) 38th Amendment
the ministers shall be appointed by the (c) 42nd Amendment
President on the advice of the Prime (d) 44th Amendment
Minister?
(a) Article 72 81. With reference to the office of the Chief
(b) Article 74 Minister, consider the following statements:
(c) Article 75 1. The term of the Chief Minister is not
(d) Article 78 fixed by the Constitution.
2. The salary and allowances of the Chief
78. Consider the following statements regarding Minister are determined by Governor.
the Economic Advisory Council to the Prime 3. Before the Chief Minister enters his
Minister:
office he/she swears an oath to uphold
1. It is chaired by Cabinet Secretary.
the sovereignty and integrity of India.
2. It can suo motu analyze any economic
4. He advises the governor with regard to
issue and advise the Prime Minister.
the removal of members of the State
3. It is attached to the Prime Minister’s
Public Service Commission.
Office.
How many statements given above are
How many of the statements given above are
correct?
correct?
(a) Only one
(a) Only one
(b) Only two (b) Only two
(c) All three (c) Only three
(d) None (d) All Four
16 www.visionias.in ©Vision IAS

FREE BY KING R QUEEN P [ऋषभ राजपूत]


82. Consider the following statements: 84. With reference to Panchayats (Extension to
Statement-I: Reservation of seats in the Scheduled Areas) Act, 1996, consider the
panchayats (both members and chairpersons) following statements:
for the scheduled caste is not applicable to 1. It extends the Part IX of the Indian
the state of Arunachal Pradesh. Constitution to the Sixth Schedule.
Statement-II: 83rd Constitutional 2. All-State Legislation on Panchayats
Amendment Act of 2000 amended Article shall be in conformity with the
243-M as Arunachal Pradesh is inhabited
customary law, social and religious
fully by indigenous tribal people and there
practices, and traditional management
are no scheduled castes.
practices of community resources.
Which one of the following is correct in
3. It is the duty of the Governor of the State
respect of the above statements?
to safeguard and preserve the traditions
(a) Both Statement-I and Statement-II are
and customs of people and their cultural
correct and Statement-II is the correct
identity.
explanation for Statement-I
Which of the statements given above is/are
(b) Both Statement-I and Statement-II are
correct and Statement-II is not the correct?

correct explanation for Statement-I (a) 1 and 2 only

(c) Statement-I is correct but Statement-II is (b) 2 only


incorrect (c) 3 only
(d) Statement-I is incorrect but Statement-II (d) 1, 2 and 3
is correct
85. With reference to the Advocate general for
83. With reference to Question Hour, consider the State, consider the following statements:
the following statements: 1. The Constitution does not contain the
1. It is an Indian innovation mentioned in procedure and grounds for the removal
the rules of procedure. Advocate General for the State.
2. It starts immediately after the zero hour 2. An Indian Citizen who has held a
and lasts until the agenda for the day. judicial office for ten years is eligible for
3. The list of starred, unstarred, short-
the appointment of Advocate General for
notice questions and questions to private
the State.
members are printed in white, green,
3. The remuneration of the advocate
yellow colour and light pink
general is determined by the governor.
respectively.
How many statements given above are
How many of the above statements are not
correct?
correct?
(a) Only one
(a) Only one
(b) Only two
(b) Only two
(c) All three (c) All three

(d) None (d) None


17 www.visionias.in ©Vision IAS

FREE BY KING R QUEEN P [ऋषभ राजपूत]


86. With reference to the post of Deputy Chief 88. Consider the following statements about the
Minister (CM) in a state, consider the Conduct of Business of the Government of
following statements: India:
1. It is not mentioned in the Constitution of 1. All the executive orders of the Union are
India. executed in the name of the President.
2. It is equivalent to the rank of a cabinet 2. The business of the Government of India
minister in the state and the perks that a allocated to the Cabinet Secretariat shall
cabinet minister is entitled to. be deemed to have been, allotted to the
3. The Deputy CM has to take approval Prime Minister.
from the Chief Minister for any Which of the statements given above is/are
expenditure over and above the allocated correct?
budget. (a) 1 only
How many statements given above are (b) 2 only
correct? (c) Both 1 and 2
(a) Only one (d) Neither 1 nor 2
(b) Only two
(c) All three 89. The term "Candida auris" was recently seen
(d) None in the news. It is a:
(a) Frog
87. Consider the following statements with (b) Fungus
respect to the Money bill: (c) Bird
1. It contains provisions only dealing with (d) Virus
the declaration of any expenditure
charged on the Consolidated Fund of 90. Consider the following statements:
India or increasing the amount of any 1. A municipality reconstituted after

such expenditure. premature dissolution does not enjoy the

2. The bill is not to be deemed to be a full period of five years but remains in

money bill by reason only that it office only for the remainder of the

provides for the imposition of fines or period.

other pecuniary penalties. 2. A municipality must be given a

3. The president cannot return the bill for reasonable opportunity of being heard

reconsideration by the Houses. before its dissolution.

How many of the above statements are Which of the statements given above is/are

correct? correct?

(a) Only one (a) 1 only

(b) Only two (b) 2 only

(c) All three (c) Both 1 and 2

(d) None (d) Neither 1 nor 2


18 www.visionias.in ©Vision IAS

FREE BY KING R QUEEN P [ऋषभ राजपूत]


91. With reference to the defection in the Indian 93. With reference to the President's rule under
Article 356 of the Indian constitution,
Parliament, consider the following
consider the following statements:
statements: 1. The presidential proclamation imposing
1. A nominated member of a House President's Rule is subject to judicial
review.
becomes disqualified for being a
2. President's proclamation to revoke
member of the House if he joins any President's rule does not require
political party after the expiry of six parliamentary approval.
3. President's rule was imposed for the first
months from the date on which he takes time in Kerala in 1959.
his seat in the House. How many of the above statements are
correct?
2. Supreme Court in the I R Coelho case
(a) Only one
declared that the decision of the (b) Only two
presiding officer concerning the (c) All three
(d) None
defection is subject to judicial review.

Which of the statements given above is/are 94. With respect to the District Planning
Committee, consider the following
correct?
statements:
(a) 1 only 1. All members are elected members of the
(b) 2 only district-level panchayat and the
municipalities in the district.
(c) Both 1 and 2
2. The rural and urban populations are
(d) Neither 1 nor 2 represented in equal proportion.
Which of the statements given above is/are
correct?
92. Consider the following subjects in the (a) 1 only
seventh schedule: (b) 2 only
(c) Both 1 and 2
1. Education
(d) Neither 1 nor 2
2. Weights and Measures

3. Forest Rights 95. Consider the following:


1. NITI Aayog
4. Marriage
2. National Development Council
5. Succession 3. National Integration Council
4. Inter-State Council
How many of the above are covered under
5. National Water Resources Council.
Concurrent List? In which of the above bodies/ institutions
(a) Only two Prime Minister act as a Chairman?
(a) 1, 2 and 3 only
(b) Only three
(b) 2, 3 and 4 only
(c) Only four (c) 4 and 5 only
(d) All five (d) 1. 2, 3, 4 and 5
19 www.visionias.in ©Vision IAS

FREE BY KING R QUEEN P [ऋषभ राजपूत]


96. Consider the following pairs: 99. Consider the following statements:
Special provisions with Statement-I: The Doctrine of Colourable
Article Legislation has been used in the cases to
respect to state
decide questions of capacity/ competency to
1. Manipur : 371A
enact a law.
2. Assam : 371B
Statement-II: When there is a question of
3. Nagaland : 371C
determining whether a particular law relates
How many of the above pairs are correctly
matched? to a particular subject the court looks to the
(a) Only one substance and true nature of the matter.
(b) Only two Which one of the following is correct in
(c) All three respect of the above statements?
(d) None (a) Both Statement-I and Statement-II are
correct and Statement-II is the correct
97. Consider the following statements regarding explanation for Statement-I
the Stand Up India scheme: (b) Both Statement-I and Statement-II are
1. It seeks to promote entrepreneurship
correct and Statement-II is not the
among Scheduled Castes (SCs)/Schedule
correct explanation for Statement-I
Tribes and Other Backward Classes.
(c) Statement-I is correct but Statement-II is
2. Loans under the scheme are available for
only green field projects. incorrect
3. It has an overdraft facility for working (d) Statement-I is incorrect but Statement-II
capital up to 1 lakh. is correct
How many of the statements given above are
correct? 100. Consider the following statements regarding
(a) Only one the delegation of functions between central
(b) Only two and state governments:
(c) All three
1. With the consent of the state
(d) None
government, parliament can delegate
both executive and legislative functions
98. In the context of Indian federalism,
"Doctrine of Territorial Nexus" is primarily to the states.
concerned with which of the following? 2. The governor of a state may, with the
(a) Determining the scope of the President's consent of the Central government,
rule in a State. entrust to that government any of the
(b) Defining the relationship between the executive functions of the state.
Union and the Union Territories. Which of the statements given above is/are
(c) Determining the scope of state correct?
legislatures extra-territorial power to
(a) 1 only
enact laws.
(b) 2 only
(d) Defining the relationship between the
(c) Both 1 and 2
Union and the states.
(d) Neither 1 nor 2

Copyright © by Vision IAS


All rights are reserved. No part of this document may be reproduced, stored in a retrieval system or
transmitted in any form or by any means, electronic, mechanical, photocopying, recording or otherwise,
without prior permission of Vision IAS.
20 www.visionias.in ©Vision IAS

FREE BY KING R QUEEN P [ऋषभ राजपूत]


VISIONIAS
www.visionias.in
ANSWERS & EXPLANATIONS
GENERAL STUDIES (P) TEST – 4135 (2024)

Q 1.C
• The chief minister is appointed by the governor. The other ministers are appointed by the governor on the
advice of the chief minister. This means that the governor can appoint only those persons as ministers who
are recommended by the chief minister.
• However, the Constitution provides for the appointment of tribal welfare ministers in Chhattisgarh,
Jharkhand, Madhya Pradesh and Odisha. Originally, this provision was applicable to Bihar, Madhya
Pradesh and Odisha.
• The 94th Amendment Act of 2006 freed Bihar from the obligation of having a tribal welfare minister
as there are no Scheduled Areas in Bihar now and the fraction of the population of the Scheduled Tribes is
very small. The same Amendment also extended the above provision to the newly formed states of
Chhattisgarh and Jharkhand.
• Hence, option (c) is the correct answer.

Q 2.B
• Recent context : In a hearing related to Agnipath scheme, plaintiff (party moving court in a civil action)
cited doctrine of promissory estoppel.
• Promissory estoppel is a concept developed in contractual laws.
o It is invoked by a plaintiff to ensure execution of a contract or seek compensation for failure to
perform the contract.
• In Chhaganlal Keshavalal Mehta v. Patel Narandas Haribhai case (1981), Supreme Court listed out
a checklist for when the doctrine can be applied.
• The Doctrine of Estoppel is a legal principle used to prevent a person from asserting a claim or
right that contradicts their previous actions or statements if it would be unfair to allow them to do
so. It is based on the concept of equity and fairness in legal proceedings.
• There are several types of estoppel, but the common thread among them is the idea that a person should
not be allowed to act in a way that leads another person to believe certain facts or promises, only to later
deny those facts or break those promises to the detriment of the other party.
• The two main types of estoppel are:
o Promissory Estoppel: As mentioned earlier, this is when a person makes a promise to another
party, and the second party relies on that promise to their detriment. In such cases, the person
who made the promise may be stopped from breaking it, even if there is no formal contract.
o Equitable Estoppel: This type of estoppel arises when one person (the "estoppelor") induces another
person (the "estoppel") to believe certain facts or to act in a particular way, and the estoppel
reasonably relies on that belief or action to their detriment. The estoppelor is then precluded from
denying the truth of those facts or from acting inconsistently with the inducement.
• Hence option (b) is the correct answer.

Q 3.A
• The Speaker will not vote in the first instance but shall have and exercise a casting vote in the case
of equality of votes. Hence Statement I is correct.
• The absence of a vote in the first instance will make the position of the Speaker impartial as in
England, and the casting vote is given to him only to resolve a deadlock. Statement II is correct and is
the correct explanation of Statement I.
• The Speaker will have the final power to maintain order within the House of the People and to interpret its
Rules of Procedure.
1 www.visionias.in ©Vision IAS

FREE BY KING R QUEEN P [ऋषभ राजपूत]


• In the absence of a quorum, it will be the duty of the Speaker to adjourn the House or to suspend the
meeting until there is a quorum.

Q 4.A
• The President from time to time summons each House of Parliament to meet. But, the maximum gap
between two sessions of Parliament cannot be more than six months.
• A ‘session’ of Parliament is the period spanning between the first sitting of a House and its
prorogation (or dissolution in the case of the Lok Sabha). During a session, the House meets every day to
transact business. A session of Parliament consists of many meetings.
• Each meeting of the day consists of two sittings, that is, a morning sitting from 11 am to 1 pm and a post-
lunch sitting from 2 pm to 6 pm.
• A sitting of Parliament can be terminated by adjournment or adjournment sine die or prorogation
or dissolution (in the case of the Lok Sabha).
• An adjournment suspends the work in a sitting for a specified time, which may be hours, days or
weeks. The presiding officer (Speaker or Chairman) declares the House adjourned sine die when the
business of a session is completed. Within the next few days, the President issues a notification for the
prorogation of the session. However, the President can also prorogue the House while in session.
• The period spanning between the prorogation of a House and its reassembly in a new session is
called ‘recess’.
• Hence option (a) is the correct answer.

Q 5.A
• Recent context: Recently, the British government said that it would provide Ukraine with armour-
piercing rounds containing depleted uranium.
• In order to produce fuel for certain types of nuclear reactors and nuclear weapons, uranium has to be
"enriched" in the U-235 isotope, which is responsible for nuclear fission.
o During the enrichment process, the fraction of U-235 is increased from its natural level (0.72% by
mass) to between 2% and 94% by mass.
o The by-product uranium mixture (after the enriched uranium is removed) has reduced
concentrations of U-235 and U-234. This by-product of the enrichment process is known as
depleted uranium (DU). Hence statement 1 is correct.
• In comparison to enriched uranium, depleted uranium is much less radioactive and is incapable of
generating a nuclear reaction. Hence statement 2 is not correct.
o The official definition of depleted uranium given by the US Nuclear Regulatory Commission (NRC)
is uranium in which the percentage fraction by weight of U-235 is less than 0.711%.
o Typically, the percentage concentration by weight of the uranium isotopes in DU used for military
purposes is: U-238: 99.8%; U-235: 0.2%; and U-234: 0.001%.
o However, due to its high density — it’s more dense than lead — depleted uranium is widely used in
weapons as it can easily penetrate armour plating.
• Apart from the US, Britain, Russia, China, France and Pakistan produce uranium weapons, which are not
classified as nuclear weapons, as per the International Coalition to Ban Uranium Weapons.
• Even though depleted uranium munitions aren’t considered nuclear weapons, experts suggest that such
weapons must be used with caution because they emit low levels of radiation and can cause severe
diseases.
Q 6.D
• Indonesia’s Mount Merapi, one of the world’s most active volcanoes, erupted late on March 17, 2023
and continued to spew hot ash and other volcanic material.
o It was Merapi’s most powerful eruption since 1930 when about 1,300 people were killed.
• Scientists at the Alaska Volcano Observatory recently warned that it's possible there could be an eruption
at Tanaga Volcano after earthquakes were recorded beneath the volcano's summit.
o Tanaga volcano is the second largest volcano of the central Aleutians in Alaska, USA. It is the
central and highest of 3 young stratovolcanoes oriented along a roughly E-W line at the NW tip of
Tanaga Island.
• One of Russia’s most active volcanoes erupted recently shooting a vast cloud of ash far up into the sky
and smothering villages in drifts of grey volcanic dust, triggering an aviation warning around Russia’s far
eastern Kamchatka Peninsula.
o Shiveluch is one of the largest and most active volcanoes in the Kamchatka peninsula of Russia
having erupted at least 60 times in the past 10,000 years.
• Hence option (d) is the correct answer.
2 www.visionias.in ©Vision IAS

FREE BY KING R QUEEN P [ऋषभ राजपूत]


Q 7.D
• The three components of government at the state level are (i) the Minister; (ii) the Secretary; and (iii) the
Executive Head (the last one in most cases is called the Director, although other nomenclatures are also
used to refer to the executive head). The Minister and the Secretary together constitute the Secretariat,
whereas the office of the Executive Head is designated as the Directorate.
• The State Secretariat, as the top layer of the state administration, is primarily meant to assist the state
government in policy-making and in discharging its legislative functions.
• The main functions of the State Secretariat are as follows:
o Assisting the ministers in policy-making, modifying policies from time to time, and discharging
their legislative responsibilities;
o Framing draft legislation, and rules and regulations;
o Coordinating policies and programs, supervising and controlling their execution, and reviewing of the
results; iv)
o Budgeting and control of expenditure;
o Maintaining contact with the Government of India and other state governments
o Overseeing the smooth and efficient running of the administrative machinery, and initiating measures
to develop greater personnel and organizational competence. Hence, option (d) is the correct
answer.
• The administrative philosophy to which the secretariat system owes its existence is that policy-making
must be kept separate from policy execution.

Q 8.C
• As per Article 74 of the Constitution, there shall be a council of ministers with the Prime Minister as the
head to aid and advise the President, who shall, in the exercise of his functions, act under the advice of the
council of ministers.
• Though the President has no constitutional discretion, he has some situational discretion. In other words,
the President can act at his discretion (that is, without the advice of the ministers) under the following
situations:
o Appointment of Prime Minister when no party has a clear majority in the Lok Sabha or when
the Prime Minister in office dies suddenly and there is no obvious successor. Hence statement 1
is correct.
o Dismissal of the Council of Ministers when it cannot prove the confidence of the Lok Sabha.
Hence statement 2 is correct.
o Dissolution of the Lok Sabha if the council of ministers has lost its majority. Hence statement 3
is correct.

Q 9.B
• Joint sitting is an extraordinary machinery provided by the Constitution to resolve a deadlock
between the two Houses over the passage of a bill.
• A deadlock is deemed to have taken place under any one of the following three situations after a bill has
been passed by one House and transmitted to the other House:
o if the bill is rejected by the other House;
o if the Houses have finally disagreed as to the amendments to be made in the bill; or
o if more than six months have elapsed from the date of the receipt of the bill by the other House
without the bill being passed by it.
• In the above three situations, the president can summon both Houses to meet in a joint sitting for the
purpose of deliberating and voting on the bill.
• It must be noted here that the provision of joint sitting is applicable to ordinary bills or financial bills
only and not to money bills or Constitutional amendment bills. Hence, statement 2 is correct.
• If the bill (under dispute) has already lapsed due to the dissolution of the Lok Sabha, no joint sitting can
be summoned. But, the joint sitting can be held if the Lok Sabha is dissolved after the President has
notified his intention to summon such a sitting (as the bill does not lapse in this case).
• After the President notifies his intention to summon a joint sitting of the two Houses, none of the Houses
can proceed further with the bill. The Speaker of Lok Sabha presides over a joint sitting of the two
Houses and the Deputy Speaker, in his absence. If the Deputy Speaker is also absent from a joint
sitting, the Deputy Chairman of the Rajya Sabha presides. If he is also absent, such other person as
may be determined by the members present at the joint sitting presides over the meeting.
• The quorum to constitute a joint sitting is one-tenth of the total number of members of the two
Houses.
3 www.visionias.in ©Vision IAS

FREE BY KING R QUEEN P [ऋषभ राजपूत]


• The joint sitting is governed by the Rules of Procedure of Lok Sabha and not of Rajya Sabha.
• The Constitution has specified that at a joint sitting, new amendments to the bill cannot be proposed
except in two cases:
o those amendments that have caused final disagreement between the Houses; and Hence, statement 1
is not correct.
o those amendments that might have become necessary due to the delay in the passage of the bill.
• The joint sitting of the two houses of Parliament has been taken from the Australian Constitution. Hence,
statement 3 is correct.

Q 10.B
• Recently, the Directorate General of Foreign Trade (DGFT) amended Biofuels’ Export Policy.
• DGFT has modified the 2018 notification, allowing export of biofuel from Special Economic Zones
(SEZs) and Export-Oriented Units (EOUs) for both fuel and non-fuel purposes without any
restrictions if biofuel is produced using imported feed stock. Hence statement 1 is not correct.
• Biofuel imports and exports both require a license. Hence statement 2 is correct.
• Biofuel, renewable form of energy, is any hydrocarbon fuel that is produced from organic matter in a
short period of time. This is in contrast with fossil fuels.
o Types of Biofuel: Bio ethanol, Biodiesel, Compressed Bio-Gas (CBG), Bio-hydrogen etc.
• Government policies to promote Biofuels
o National Policy on Biofuels, 2018 for reaching 20% ethanol-blending and 5% biodiesel-blending by
2030.
o Pradhan Mantri JI-VAN (Jaiv Indhan- Vatavaran Anukool fasal awashesh Nivaran) Yojana, 2019.
o GOBAR (Galvanizing Organic Bio-Agro Resources) DHAN scheme, 2018.
o Sustainable Alternative Towards Affordable Transportation (SATAT) to establish an ecosystem for
CBG production.

Q 11.B
• Exceptions to Parliament's territorial jurisdiction: The Constitution places certain restrictions on the
plenary territorial jurisdiction of the Parliament. In other words, the laws of Parliament are not applicable
in the following:
o Union Territories
✓ The President can make regulations for the peace, progress, and good government of the four
Union Territories—the Andaman and Nicobar Islands, Lakshadweep, Dadra and Nagar Haveli,
and Daman and Diu.
✓ A regulation so made has the same force and effect as an act of Parliament. It may also repeal or
amend any act of Parliament in relation to these union territories.
o Scheduled area's in the state
✓ The governor is empowered to direct that an act of Parliament does not apply to a scheduled area
in the state or apply with specified modifications and exceptions.
o Tribal areas in the state
✓ The Governor of Assam may likewise direct that an act of Parliament does not apply to a tribal
area (autonomous district) in the state or apply with specified modifications and exceptions.
✓ The President enjoys the same power with respect to tribal areas (autonomous districts)
in Meghalaya, Tripura, and Mizoram.
• Hence option (b) is the correct answer.

Q 12.A
• Recent context: Indian Army is going to conduct PALM 400 attack drone tests in Pokhran. The
PALM 400 - acronym for 'Precision Attack Loitering Munition' is a long-range, high-precision
loitering munition system.
• PALM 400 is an armed Remotely Piloted Vehicle (RPV) built by AVision Systems.
o It is a joint venture between Israeli firm UVision Air Ltd and Hyderabad-based Aditya Precitech
Private Ltd.
• It is a high precision loitering system launched from a canister which can loiter for up to 120 minutes.
o Speed: 50-140 knots (90-260 km per hour), Height: 3,000-4,000 feet above ground.
o It picks out its target and fires an armorpenetrating projectile from top - where its armor protection is
the least.
o It uses complex navigational techniques that enable it to function even in environments where the
GPS has been disabled.
4 www.visionias.in ©Vision IAS

FREE BY KING R QUEEN P [ऋषभ राजपूत]


o It can loiter above a target in stealth mode, collect information, and immediately strike at time-
sensitive threats.
o It uses electro-optical/infrared stabilized cameras to detect and engage targets both during the day and
at night.
• Hence option (a) is the correct answer.

Q 13.C
• The Vice-President holds office for a term of five years from the date on which he enters his office.
However, he can resign from his office at any time by addressing the resignation letter to the President.
He can also be removed from the office before completion of his term.
• A formal impeachment is not required for his removal. He can be removed by a resolution passed by a
majority of all the then members of the Rajya Sabha and agreed to by the Lok Sabha. Hence
statement 1 is correct. This means that this resolution should be passed in the Rajya Sabha by an
effective majority and in the Lok Sabha by a simple majority.
• The effective majority in India is only a type of special majority and not a separate one. Further, this
resolution can be introduced only in the Rajya Sabha and not in the Lok Sabha. Hence statement 2
is correct.
• But, no such resolution can be moved unless at least 14 days advance notice has been given.
Notably, no ground has been mentioned in the Constitution for his removal. Hence statement 3 is
correct.
Q 14.A
• Recently, Sundarja mango, Manamadurai pottery, Nagri Dubraj rice have received GI tags.
• Geographical Indications (GIs) are one of the forms of intellectual property that authenticate the origins,
quality, distinct characteristics, and reputation of a product attributable to a particular geographical region.
• The Geographical Indications of Goods (Registration and Protection) Act, 1999 (GI Act) is an Act of the
Parliament of India for the protection of geographical indications in India. India, as a member of the
World Trade Organization (WTO), enacted the Act to comply with the Agreement on Trade-Related
Aspects of Intellectual Property Rights.
• Sundarja mango
o The Sundarja mango is found in abundance in Govindgarh town of Rewa district, Madhya
Pradesh. The Sundarja mango has a wide popularity and even a stamp has been issued in the name of
it. This mango is called pride of the Vindya and it is fibre free fruit. Hence pair 1 is correctly
matched.
• Basohli Painting' - popular for its miniature art style from Kathua district of Jammu has obtained
the Geographical Indication (GI) Tag.
o Basohli, a simple town in the Jammu region, was the cradle of Pahari paintings. It gave birth to a
unique style of miniature paintings that witnessed a fusion of mythology and traditional folk art.
o It was under Sangram Pal (1635-1673) and later Kirpal Pal (1678-1693) that Basohli paintings
flourished.
o Vaishnavism was adopted under Sangram Pal and hence the early paintings, specifically the
Rasmanjari series, depict Krishna as the protagonist.
o The characteristic features of these paintings were the use of bright and bold colours like red, yellow,
and blue in the borders, as well as for the generally flat background.
o The other distinguishing part was the facial features- a prominent nose and lotus-shaped eyes.
o Depiction of flora was not unusual in Basohli paintings. The artists’ favourite was the bright red
flowers of rhododendron. Hence pair 2 is not correctly matched.
• Chhattisgarh's Nagri Dubraj rice variety got a geographical indication tag.
o After Jeeraphool rice in 2019, Dubraj is the second rice brand from Chattisgarh to get the GI tag.
o The women's self-help group “Maa Durga Swasahayata Samuh” of Nagri in Dhamtari district has
been harvesting Dubraj and has applied for the GI tag. Hence pair 3 is not correctly matched.
Q 15.A
• Operation Interflex is a part of the United Kingdom’s (UK) commitment of £2.3 billion for military
aid and support to Ukraine.
• Operation Interflex aims to teach weapon handling, range activity, marksmanship, fieldcraft basics, field
tactics, battle casualty drills, counter explosives, the laws of armed conflict, first aid, and cyber security
awareness.
• Under it, the UK and other countries including Canada, New Zealand, Australia, Norway, Denmark,
Finland, Sweden, Lithuania, and Netherlands are training Ukraine’s military.
• Hence option (a) is the correct answer.
5 www.visionias.in ©Vision IAS

FREE BY KING R QUEEN P [ऋषभ राजपूत]


Q 16.C
• The Vice-President is elected not directly by the people but by the method of indirect election. He is
elected by the members of an electoral college consisting of the members of both Houses of Parliament.
• The original Constitution provided that the vice president would be elected by the two Houses of
Parliament assembled at a joint meeting. This cumbersome procedure was done away by the 11th
Constitutional Amendment Act of 1961. Hence statement 1 is correct.
• The Election Commission, in consultation with the Central Government, appoints the Secretary-
General of Lok Sabha and Rajya Sabha, by rotation, as the Returning Officer. The Commission also
decides to appoint Assistant Returning Officers in Parliament House (Lok Sabha) to assist Returning
Officers. Hence statement 2 is correct.

Q 17.A
• After independence, Article 148 of the 1949 Indian Constitution provided for the establishment of a
Comptroller and Auditor General to be appointed by the President of India.
• In 1971 the central government enacted the Comptroller and Auditor General (Duties, Powers, and
Conditions of Service) Act, 1971. The act made CAG responsible for both accounting and auditing duties
for central and state governments.
• Article 148 broadly deals with the CAG appointment, oath and conditions of service.
• Article 149 deals with the Duties and Powers of the Comptroller and Auditor-General of India.
• Article 150 says that the accounts of the Union and of the States shall be kept in such form as the
President may, on the advice of the CAG, prescribe.
• CAG is appointed by the President by warrant under his hand and seal and provided with tenure of
6 years or 65 years of age, whichever is earlier. Hence, statement 1 is correct.
• CAG can be removed by the President only in accordance with the procedure mentioned in the
Constitution which is the manner same as the removal of a Supreme Court Judge.
• He is ineligible to hold any office, either under the Government of India or of any state, once he
retires/resigns as a CAG. Hence, statement 2 is not correct.

Q 18.A
• Exemption of Central Property from State Taxation:
o The property of the Centre is exempted from all taxes imposed by a state or any authority within a
state like municipalities, district boards, panchayats and so on. Hence statement 1 is correct.
o But, the Parliament is empowered to remove this ban.
o The corporations or the companies created by the Central government are not immune from state
taxation or local taxation. The reason is that a corporation or a company is a separate legal entity.
• Exemption of State Property or Income from Central Taxation
o The property and income of a state is exempted from Central taxation. Such income may be
derived from sovereign functions or commercial functions.
o But the Centre can tax the commercial operations of a state if Parliament so provides.
o Notably, the property and income of local authorities situated within a state are not exempted
from the Central taxation. Hence statement 2 is not correct.
o Similarly, the property or income of corporations and companies owned by a state can be taxed
by the Centre.

Q 19.D
• There are a total of 24 department-related standing committees, of which eight are with the Rajya Sabha
and the remaining 16 with the Lok Sabha. The eight Rajya Sabha committees consist of 244 members
from both Houses, with 78 from Rajya Sabha and 166 from Lok Sabha. The standing committee system
was introduced in 1993 with the panels drawing members from all political parties. Hence statement 1 is
not correct.
• Rules relating to the department-related parliamentary standing committees, state that:
o Each of the Standing Committees shall have the following functions, namely:-
✓ to consider the Demands for Grants of the related Ministries/Departments and report thereon. The
report shall not suggest anything of the nature of cut motions;
✓ to examine Bills, pertaining to the related Ministries/Departments, referred to the Committee by
the Chairman or the Speaker, as the case may be, and report thereon;
✓ to consider the annual reports of the Ministries/Departments and report thereon; and
✓ to consider national basic long-term policy documents presented to the Houses, if referred to the
Committee by the Chairman or the Speaker, as the case may be, and report thereon.
6 www.visionias.in ©Vision IAS

FREE BY KING R QUEEN P [ऋषभ राजपूत]


✓ Provided that the Standing Committees shall not consider matters of the day-to-day administration
of the related Ministries/Departments.
✓ Hence statement 2 is not correct.

Q 20.A
• Recently, the United Kingdom agreed to join CPTPP. It will become the first new member and the first
country in Europe to join the CPTPP.
• It was signed in March 2018 and came into force in December 2018. Hence statement 3 is not
correct.
o It succeeded the Trans-Pacific Partnership (TPP) after the United States withdrew from the TPP in
2017.
• It is a Free Trade Agreement (FTA) signed between 11 nations.
• Member countries: Australia, Brunei, Canada, Chile, Japan, Malaysia, Singapore, Vietnam, Peru,
Mexico, and New Zealand.
o USA is thus not a member of CPTPP. Hence statement 1 is correct.

• Hence statement 2 is not correct.

Q 21.C
• Article 252- Parliament's power to legislate on the subject of state list when states make a
request: Any Act so passed by Parliament may be amended or repealed by an Act of Parliament
passed or adopted in like manner but shall not, as respects any State to which it applies, be
amended or repealed by an Act of the Legislature of that State
o When the legislatures of two or more states pass resolutions requesting the Parliament to enact laws
on a matter in the State List, then the Parliament can make laws for regulating that matter.
o A law so enacted applies only to those states which have passed the resolutions. However, any
other state may adopt it afterward by passing a resolution to that effect in its legislature. Hence
statement 1 is correct.
o Such a law can be amended or repealed only by the Parliament and not by the legislatures of the
concerned states. Hence statement 2 is correct.
• The effect of passing a resolution:
o The effect of passing a resolution under the above provision is that,
✓ The Parliament becomes entitled to legislate with respect to a matter for which it has no power to
make a law.
✓ On the other hand, the state legislature ceases to have the power to make a law with respect to
that matter.
✓ The resolution operates as abdication or surrender of the power of the state legislature with
respect to that matter and it is placed entirely in the hands of Parliament which alone can then
legislate with respect to it.
7 www.visionias.in ©Vision IAS

FREE BY KING R QUEEN P [ऋषभ राजपूत]


✓ Some examples of laws passed under the above provision are Prize Competition Act, 1955;
Wild Life (Protection) Act, 1972; Water (Prevention and Control of Pollution) Act, 1974;
Urban Land (Ceiling and Regulation) Act, 1976; and Transplantation of Human Organs
Act, 1994.

Q 22.B
• Recent context: 28 States/UTs have adopted the National Generic Document Registration System
(NGDRS) for Land Records and 26 States/UTs have adopted the Unique Land Parcel Identification
Number (ULPIN) or Bhu-Aadhar.
• About National Generic Document Registration System (NGDRS)
o Application: NGDRS is a common, generic, and configurable application developed for document
registration departments across the country.
o Initiated by: Department of Land Resources (DoLR), Ministry of Rural Development. Hence
statement 1 is not correct.
o State-specific: NGDRS facilitates states to create state-specific instances and configure the software
as per requirements.
o Citizen services: Offering a complete user interface for property and document registration, the
application enables citizens to proceed with land buying online. Hence statement 2 is correct.
o Objectives:
✓ Enabling the idea of One Nation One Software
✓ Citizen empowerment by enabling property valuation (including auto calculation of duty) and
online document submission
✓ Single platform for all stakeholders in the registration process
• ULPIN: ULPIN is part of the Digital India Land Records Modernization Programme (DILRMP) and is a
14-digit identification number accorded to a land parcel.
o It was developed by National Informatics Centre (NIC).

Q 23.A
• Recently, Belgrade (Serbia) has introduced liquid tree, an urban photo-bioreactor, to combat air
pollution.
• LIQUID 3 contains water and uses microalgae to reduce greenhouse gas emissions and improve air
quality by binding carbon dioxide and producing pure oxygen through photosynthesis.
• LIQUID 3 is more efficient than trees, replacing two 10- year-old trees or 200 square meters of lawn.
• A photobioreactor (PBR) can be defined as an enclosed, illuminated culture vessel designed to control
biomass production through adjustment of the operating parameters.
• Hence option (a) is the correct answer.

Q 24.C
• Besides the right to address a joint sitting of both Houses at the commencement of the first session, the
President shall also have the right to address either House or their joint sitting, at any time, and to
require the attendance of members for this purpose [ Art. 86(1 ) ]. This right is no doubt borrowed
from the English Constitution, but there it is not exercised by the Crown except on ceremonial
occasions. Hence statement 1 is correct. Apart from the right to address, the Indian President shall
have the right to send messages to either House of Parliament either in regard to any pending Bill
or to other matters, and the House must then consider the message “ with all convenient despatch [
Art. 86(2)]. Hence statement 2 is correct.
• The Indian President shall have the power to send messages not only on legislative matters but also
otherwise’. Since the head of the Indian Executive is represented in Parliament by his Ministers, the
power given to the President to send messages regarding legislation may appear to be superfluous, unless
the President has the freedom to send messages differing from the Ministerial policy, in which case again
it will open a door for friction between the President and the Cabinet.

Q 25.B
• Recent context: Bengaluru sees Zero Shadow Day.
• A Zero Shadow Day is a specific day when the Sun is positioned in such a way that an upright object,
such as a vertical pole or a person standing, does not cast a visible shadow. This phenomenon occurs in
certain geographic locations.
• For every point on Earth between the Tropic of Cancer and the Tropic of Capricorn, there are two
Zero Shadow Days a year because of the tilt of the Earth's axis and its revolution around the Sun.
8 www.visionias.in ©Vision IAS

FREE BY KING R QUEEN P [ऋषभ राजपूत]


o The Earth's axis is tilted at an angle of approximately 23.5 degrees with respect to its orbital plane.
This tilt causes the angle of the Sun's rays to change throughout the year as the Earth revolves around
the Sun.
o ZSD occurs when the sun’s declination (earth’s axis results in a daily variation of the angle between
the earth–sun line and the earth’s equatorial plane) becomes equal to the latitude of the location and
the sun’s rays will fall exactly vertical relative to an object on the ground leading to no shadow of that
object.
• Delhi is located beyond the Tropic of Cancer at 28 degrees latitude and hence does not experience
zero shadow day.
• The latitude of Hyderabad is 17.3850° N and it was seen on May 9 at 12.12 p.m. and on August 3,
when the shadow of any vertical object will disappear.
• Hence option (b) is the correct answer.

Q 26.A
• The Chief Secretary is the chief advisor to the Chief Minister and Secretary to the State Cabinet. S/he is
the head of the General Administration department whose political head is the Chief Minister
herself/himself. The Chief Secretary is also the Head of the Civil Services in the State. S/he is the main
channel of communication between the State Government and the Central and other State governments.
Hence, statement 1 is correct.
• The office of the Chief Secretary is an institution unique to the states; it is without a parallel in the
administrative landscape of the entire country. The Chief Secretary’s office has, for instance, no
parallel in the Central Government. The work s/he performs in relation to the State Government is, at
the Union level, shared by three high-ranking functionaries of more or less an equal status, i.e., Cabinet
Secretary, Home Secretary and Finance Secretary, This is a vivid reflection on the wide scope of the
duties and powers of the Chief Secretary. Hence, statement 2 is not correct.

Q 27.A
• Central Vigilance Commission is an apex Indian governmental body created in 1964 to address
governmental corruption. In 2003, the Parliament enacted a law conferring statutory status on the CVC.
• The CVC was set up by the Government in February 1964 on the recommendations of the Committee on
Prevention of Corruption, headed by Shri K. Santhanam. In 2003, the Parliament enacted CVC Act
conferring statutory status on the CVC. Hence, statement 3 is not correct.
• The CVC is not controlled by any Ministry/Department. It is an independent body which is only
responsible to the Parliament. Hence, statement 1 is correct.
• Its annual report gives the details of the work done by the commission and points to systemic failures
which lead to corruption in government departments. It presents this report annually to the
President. The President then places this report before each House of Parliament. Hence, statement 2 is
not correct.

Q 28.C
• Article 53 deals with the executive powers of the President of India. The powers of the President are
broadly divided into two types, namely, ordinary and emergency powers. The executive powers of the
Union are vested in the President. Article 53 confers all executive powers in him and empowers him
to exercise these powers directly by himself or through officers subordinate to him. Hence statement
1 is correct.
• Article 75 requires the Prime Minister to communicate to the President regarding all decisions of the
Union Council of Ministers.
• Article 77 holds that all executive powers of the Union government shall be exercised in the name of
the President. Hence statement 2 is correct.
• The Constitution prescribes an indirect election through an electoral college (composed of the elected
members of Parliament and the elected members of the state legislative assemblies) on the basis of
proportional representation and bymeans of a single transferable vote.All doubts and disputes in
connection with election of the President are inquired into and decided by the Supreme Court whose
decision is final. Anti-defection law is not applicable in presidential election; thus, electors are not
bound to vote along party line.

9 www.visionias.in ©Vision IAS

FREE BY KING R QUEEN P [ऋषभ राजपूत]


Q 29.B
• Solicitor-General is the second highest law officer of the Government of India after the Attorney
General (AG) of India. He/She assists the AG in the fulfillment of his/her official responsibilities. Hence,
statement 1 is correct.
• It should be noted here that only the office of the AG is created by the Constitution. In other words,
Article 76 does not mention the Solicitor general and additional solicitor general. Hence, statement 2 is
correct.
• The SGI is appointed by the President on the recommendations of the Appointments Committee of
the Cabinet. The SGI generally has a tenure of 3 years and is eligible for reappointment. Hence,
statement 3 is not correct.

Q 30.C
• Prasar Bharati is a statutory autonomous body established under the Prasar Bharati Act and came into
existence on 23.11. 1997. It is the Public Service Broadcaster of the country. The Ministry is responsible
for the administration of Prasar Bharati, the broadcasting arm of the Indian Government. Hence,
statement 2 is correct.
• The Central Board of Film Certification is the other important statutory body under this ministry being
responsible for the regulation of motion pictures broadcast in India.
• Until the Information Technology Rules 2021, the Ministry of Information and Broadcasting had the
power to regulate content in all sectors — TV channels, magazines, newspapers and movies in theatres
and on TV, and radio — with the exception of the internet.
• The Information Technology (Intermediary Guidelines and Digital Media Ethics Code) Rules, 2021, were
promulgated in February 2021, and they extended their regulatory powers over internet content,
particularly on digital news platforms and Over the Top ("OTT") platforms. Hence, statement 1 is
correct.

Q 31.B
• The Union executive consists of the President, the Vice President, the Prime Minister, the council of
ministers, and the attorney general of India.
• Parliamentary privileges are special rights, immunities, and exemptions enjoyed by the two Houses of
Parliament, their committees, and their members. The Constitution has also extended parliamentary
privileges to those persons who are entitled to speak and take part in the proceedings of a House of
Parliament or any of its committees. These include the attorney general of India and Union ministers.
• Every minister and the attorney general of India have the right to speak and take part in the
proceedings of either House, any joint sitting of both the Houses, and any committee of Parliament of
which he is a member, without being entitled to vote. Hence options 2 and 3 are correct.
• The parliamentary privileges do not extend to the President who is also an integral part of the
Parliament. Article 361 of the Constitution provides for privileges for the President. Hence option 1 is
not correct.

Q 32.A
• Recent context: International Criminal Court (ICC) issued arrest warrant for Vladimir Putin for war
crimes in Ukraine.
• ICC stated that Putin bears individual criminal responsibility for the abduction and deportation of
Ukrainian children since Russia’s full-scale invasion began in February last year.
o However, ICC has no power to arrest sitting heads of state or bring them to trial. Hence
statement 1 is correct.
• War crimes are serious violations of humanitarian laws during a conflict (derived from 1949 Geneva
Conventions).
• War crimes include torture, mutilation, corporal punishment, hostage taking and acts of terrorism.

10 www.visionias.in ©Vision IAS

FREE BY KING R QUEEN P [ऋषभ राजपूत]


• Hence statement 2 is not correct.
• Its Headquarters is located at The Hague in the Netherlands. Hence statement 3 is not correct.
11 www.visionias.in ©Vision IAS

FREE BY KING R QUEEN P [ऋषभ राजपूत]


Q 33.B
• Recent context: Christina Koch, the American astronaut who spent 328 days aloft on her first trip to
International Space Station, spoke about the overview effect.
• The "Overview Effect" is a term used to describe the cognitive shift experienced by astronauts
when viewing the Earth from space. This transformative experience often leads to a profound
change in perspective, values, and attitudes towards the planet and its inhabitants.
• It highlights the interconnectedness of humanity and the fragility of the Earth's ecosystem.
• The Overview Effect has been reported by numerous astronauts and is considered an essential aspect of
human space exploration.
• Hence option (b) is the correct answer.

Q 34.A
• The Prime Minister's Office (PMO) is a staff agency meant for providing secretarial assistance and crucial
advice to the Prime Minister. The Prime Minister's Office is headed politically by Prime Minister
and administratively by the Principal Secretary. Additionally, it consists of one or two b Additional
Secretaries, three to five Joint Secretaries, a number of Directors Deputy Secretaries, and
Undersecretaries. There are also other officers like Officers on Special Duty; Private secretaries, and so
on. These officers are supported by regular office establishments. Hence, statement 2 is correct.
• The major functions of PMO are:
o Acting as the ‘think-tank’ of the Prime Minister.
o Faster decision-making: It helps in faster decision-making as it involves experienced and powerful
decision-makers. Assisting the Prime Minister in respect of his overall responsibilities as head of the
government like Maintaining liaison with central ministries/departments and the state governments.
o It acts as the residual legatee of the Central Government, that is, it deals with all such subjects which
are not allotted to any ministry/department.
o However, is not concerned with the responsibilities of the Prime Minister as the chairman of the
Union Cabinet. Hence, statement 1 is not correct.
• PMO is an extra-constitutional body that has no mention in the Indian Constitution. However, it was
given the status of a department under the Government of India Allocation of Business Rules, 1961.
However, Unlike Ministries/Departments, no administrative or quasi–judicial decisions are generally
taken in the Prime Minister’s Office which may affect a particular person or class of persons. Hence,
statement 3 is not correct.
• National Authority Chemical Weapons Conventions has been established under the Chemical Weapons
Convention Act, 2000 for implementing the provisions of the Convention on the Prohibition of the
Development, Production, Stockpiling, and Use of Chemical Weapons and on their Destruction, signed on
behalf of the Government of India at Paris on the 14th day of January 1993. NACWC is an office in the
Cabinet Secretariat, Government of India. Hence, statement 4 is not correct.

Q 35.D
• Article 164 states that the Council of Ministers is collectively responsible to the legislative assembly of
the state. This means that all the ministers own joint responsibility to the legislative assembly for all their
acts of omission and commission. They work as a team and swim or sink together.
• Article 164 contains the principle of individual responsibility. It states that the ministers hold office at
the pleasure of the governor. This means that the governor can remove a minister at a time when the
council of ministers enjoys the confidence of the legislative assembly.
• Thus, the Governor cannot dismiss a Council of Ministers at his pleasure on his/her subjective
estimate of the strength of the Chief Minister in the State Assembly at any time because it is for the
legislative Assembly to enforce collective responsibility. Hence, statement 1 is not correct.
• A person who is not a member of either House of the state legislature can also be appointed as a
minister. But, within six months, he must become a member (either by election or by nomination) of
either House of the state legislature, otherwise, he ceases to be a minister. A minister who is a member of
one House of the state legislature has the right to speak and take part in the proceedings of the other
House. But, he can vote only in the House of which he is a member. Hence, statement 2 is not
correct.

12 www.visionias.in ©Vision IAS

FREE BY KING R QUEEN P [ऋषभ राजपूत]


Q 36.B
• 73rd Amendment Act of 1992 has added a new Part-IX to the Constitution of India. This part is entitled
‘The Panchayats’ and consists of provisions from Articles 243 to 243 O.
• The provisions of this Part are applicable to the Union territories. But, the President may direct that they
would apply to a Union territory subject to such exceptions and modifications as he may specify.
• The act does not apply to the states of Nagaland, Meghalaya and Mizoram and certain other areas.
• These areas include
o the scheduled areas and the tribal areas in the states;
o the hill areas of Manipur for which district councils exist; and
o the Darjeeling district of West Bengal for which Darjeeling Gorkha Hill Council exists.
• However, the Parliament may extend the provisions of this Part to the scheduled areas and tribal
areas subject to such exceptions and modifications as it may specify. Under this provision, the Parliament
enacted the “Provisions of the Panchayats (Extension to the Scheduled Areas Act”, 1996, popularly
known as the PESA Act or the Extension Act.
• Hence, option (b) is the correct answer.

Q 37.B
• Recent context: The Ministry of Electronics and Information Technology (MeiTY) has amended the
Information Technology (Intermediary Guidelines and Digital Media Ethics Code) Rules (IT Rules)
2021.
o The aim of these amendments is to enforce greater due diligence by online gaming and social media
intermediaries in respect of online games & fake or false misleading information related to
Government business.
• Key Features of the Rules on Online Gaming
o Clear definitions:
✓ “Online games” mean a game that is offered on the Internet and is accessible by a user
through a computer resource or an intermediary. Hence statement 1 is not correct
✓ “Online gaming intermediary (OGI)” means any intermediary that enables the users of its
computer resource to access one or more online games.
o Role of Intermediaries: To make a reasonable effort to not host, publish or share any online game that
can cause the user harm, or that has not been verified as a permissible online game by an online
gaming Self-Regulatory Body/Bodies (SRBs) designated by the Central Government.
✓ The intermediary will also have to ensure that no advertisement or surrogate advertisement or
promotion of an online game that is not a permissible online game, is hosted on its platform.
o Prohibition: Online games that involve any kind of gambling (including ads) will be prohibited.
Hence statement 2 is correct.
o Additional Obligations on OGI: The amended rules cast additional obligations on OGI in relation
to online games involving real money. These include:
✓ The displaying of a mark of verification by the self-regulatory body on such games
✓ Informing their users of the policy for withdrawal or refund of the deposit
✓ Obtaining the KYC details of the users
✓ Not giving credit or enabling financing by third parties to the users
o Multiple SRBs: The MeiTY may notify multiple SRBs, for the purposes of verifying an Online
game as a permissible one. An SRB should fulfil the following criteria:
✓ Company registered under Section 8 (Not-for-Profit entity) of the Companies Act 2013.
✓ Representative of the online gaming industry, promoting online games in a responsible manner.
✓ Incorporates provisions related to grievance redressal, arm’s length principle, disclosure and
reporting and clear criteria for membership.
o Authority of SRBs: The SRB may categorise any Game as a permissible game if it is satisfied
that:
✓ the online game does not involve wagering on any outcome
✓ the OGI and the game comply with the rules and the requirements under law for being competent
to enter into a contract (currently at 18 years), and the OGI and the game complies with the
framework made by the SRB regarding safeguards

Q 38.B
• The office of ‘whip’ is mentioned neither in the Constitution of India nor in the Rules of the House
nor in a Parliamentary Statute. It is based on the conventions of the parliamentary government. Hence,
statement 1 is not correct.
13 www.visionias.in ©Vision IAS

FREE BY KING R QUEEN P [ऋषभ राजपूत]


• Every political party, whether ruling or Opposition has its own whip in the Parliament. Hence,
statement 2 is correct.
• He is appointed by the political party to serve as an assistant floor leader. He is charged with the
responsibility of ensuring the attendance of his party members in large numbers and securing their support
in favor of or against a particular issue.
• He regulates and monitors their behavior in the Parliament. The members are supposed to follow the
directives given by the whip. Otherwise, disciplinary action can be taken.
• The Tenth Schedule - popularly known as the Anti-Defection Act - was included in the Constitution
via the 52nd Amendment Act, of 1985.
o It sets the provisions for disqualification of elected members on the grounds of defection to
another political party.
o A member of a House belonging to any political party becomes disqualified from being a member of
the House.
✓ if he voluntarily gives up his membership in such political party; or
✓ if he votes or abstains from voting in such House contrary to any direction issued by the
whip of his political party without obtaining prior permission of such party and such act has
not been condoned by the party within 15 days. Hence, statement 3 is correct.

Q 39.A
• The Constitution does not contain any specific procedure for the selection and appointment of the Prime
Minister. Article 75 says only that the Prime Minister shall be appointed by the president. In accordance
with the conventions of the parliamentary system of government, the President has to appoint the leader of
the majority party in the Lok Sabha as the Prime Minister. In 1980, the Delhi High Court held that the
Constitution does not require that a person must prove his majority in the Lok Sabha before he is
appointed as the Prime Minister. The President may first appoint him the Prime Minister and then ask
him to prove his majority in the Lok Sabha within a reasonable period. Hence, statement 1 is correct.
• The allegations of corruption against a sitting Prime Minister can be inquired by the Lokpal. The
jurisdiction of Lokpal includes the Prime Minister, Ministers, members of Parliament, Groups A, B, C,
and D officers, and officials of the Central Government. The jurisdiction of the Lokpal included the Prime
Minister except on allegations of corruption relating to international relations, security, public order,
atomic energy, and space. Hence, statement 2 is not correct.
• The Government of India (Allocation of Business) Rules, 1961 are made by the President of India under
Article 77 of the Constitution for the allocation of business of the Government of India. The
Ministries/Departments of the Government are created by the President on the advice of the Prime
Minister under these Rules. Each of the Ministries is assigned to a Minister by the President on the advice
of the Prime Minister. Hence, statement 3 is not correct.

Q 40.C
• A bill passed by the Parliament can become an act only if it receives the assent of the President. When
such a bill is presented to the President for his assent, he has three alternatives (under Article 111 of the
Constitution):
o He may give his assent to the bill, or
o He may withhold his assent to the bill, or
o He may return the bill (if it is not a Money bill) for reconsideration by the Parliament. However, if the
bill is passed again by the Parliament with or without amendments and again presented to the
President, the President must give his assent to the bill.
• It should be noted here that the President has no veto power in respect of a constitutional
amendment bill. The 24th Constitutional Amendment Act of 1971 made it obligatory for the
President to give his assent to a constitutional amendment bill. Hence statement 1 is correct.
• The 42nd Constitutional Amendment Act of 1976 (enacted by the made the President bound by the
advice of the council of ministers headed by the prime minister. Hence statement 2 is correct.
• The 44th Constitutional Amendment Act of 1978 authorized the President to require the council of
ministers to reconsider such advice either generally or otherwise. However, he ‘shall’ act in accordance
with the advice tendered after such reconsideration. In other words, the President may return a matter once
for reconsideration by his ministers, but the reconsidered advice shall be binding.

14 www.visionias.in ©Vision IAS

FREE BY KING R QUEEN P [ऋषभ राजपूत]


Q 41.B
• Cabinet Committees are extra-constitutional in emergence. They are not mentioned in the Constitution.
However, the Rules of Business provide for their establishment. Hence, statement 1 is not correct.
• They are of two types- standing and ad-hoc. The former are of permanent nature while the latter are of
temporary nature. The ad-hoc committees are constituted from time to time to deal with special problems.
They are disbanded after their task is completed. Hence, statement 2 is correct.
• They are mostly headed by the Prime Minister. Sometimes other Cabinet members, particularly the Home
Minister or the Finance Minister, also act as their Chairman.
• They usually include only Cabinet ministers. However, the non-cabinet ministers are not debarred from
their membership. Hence, statement 3 is correct.

Q 42.C
• The GST Council is a constitutional body responsible for making recommendations on issues
related to the implementation of the Goods and Services Tax (GST) in India. Hence statement 1 is
correct.
o As per Article 279A (1) of the amended Constitution, the GST Council was constituted by the
President.
o Members:
✓ The members of the Council include the Union Finance Minister (Chairperson), theUnion
Minister of State (Finance) from the Centre.
✓ Each state can nominate a minister in-charge of finance or taxation or any other minister as a
member.
• Recommendations are not binding:
o Last year the Supreme Court in the case Union of India and Anr versus M/s Mohit Minerals
Through Director ruled that recommendations of the Goods and Services Tax (GST) Council only
have persuasive value, and cannot be binding on the Centre and states. Hence statement 2 is
correct.

Q 43.B
Committee on Public Undertakings
• This committee was created in 1964 on the recommendation of the Krishna Menon Committee. The
first Estimates Committee in the post-independence era was constituted in 1950 on the recommendation of
John Mathai, the then finance minister. Hence statement 1 is not correct.
• Originally, it had 15 members (10 from the Lok Sabha and 5 from the Rajya Sabha). However, in 1974,
its membership was raised to 22 (15 from the Lok Sabha and 7 from the Rajya Sabha).
• The members of this committee are elected by the Parliament every year from amongst its own members
according to the principle of proportional representation by means of a single transferable vote. Thus, all
parties get due representation in it.
• The term of office of the members is one year. A minister cannot be elected as a member of the
committee.
• The chairman of the committee is appointed by the Speaker from amongst its members who are drawn
from the Lok Sabha only. Thus, the members of the committee who are from the Rajya Sabha cannot
be appointed as the chairman. Hence statement 2 is correct.

Q 44.A
• Recent context: Japan became the latest country to join Multi-Party Interim Appeal Arbitration
Arrangement (MPIA).
• MPIA (formed in 2020) is an alternative system for resolving WTO disputes that are appealed by a
Member in the absence of a functioning WTO Appellate Body. Hence statement 1 is correct.
o WTO members can resort to MPIA under Article 25 of WTO Dispute Settlement Understanding.
• Any member can join MPIA by notifying the Dispute Settlement Body (composed of representatives of
all WTO Members).
o India not a member yet. Hence statement 2 is not correct.
o In a dispute between members, MPIA will supersede the previous appeal processes and also apply to
future disputes between members.
• WTO’s dispute settlement process functioning is impacted since 2018 in the backdrop of USA blocking
appointments to Appellate Body.

15 www.visionias.in ©Vision IAS

FREE BY KING R QUEEN P [ऋषभ राजपूत]


• There are two main ways to settle a dispute once a complaint has been filed in WTO:
o Parties find a mutually agreed solution, particularly during the phase of bilateral consultations.
o Through adjudication, including the subsequent implementation of the panel and Appellate Body
reports.

Q 45.C
• The Constituent Assembly (Legislative) Rules of Procedure and Conduct of Business came into force
immediately before the commencement of the Constitution of India was modified and adopted by the
Speaker of Lok Sabha in the exercise of the powers conferred on the Speaker by Article 118(2) of the
Constitution and published under the title “Rules of Procedure and Conduct of Business in the House of
the People” in the Gazette of India Extraordinary dated the 17th April 1952.
• Those Rules were amended by the Speaker from time to time on the recommendations of the Rules
Committee of the House until September 1954. In September 1954, the Rules Committee decided that
their recommendations should be approved by the House before amendments were given effect. Hence
statement 2 is correct.
• The Speaker is the head of the Lok Sabha and its representative. S/he is also the ex-officio chairman of
some Lok Sabha committees like the Rules Committee, the Business Advisory Committee, and the
General Purposes Committee. Hence statement 1 is correct.

Q 46.B
• Recent context: IFAD and Japan announced the launch of the Enhanced Linkages between Private Sector
and Small-scale Producers initiative (ELPS).
o Its aim is to make small-scale producers and local food systems around the world more resilient and
sustainable together with the enhanced engagement of private sector companies.
• The International Fund for Agricultural Development (IFAD), a specialized agency of the United
Nations, was one of the major outcomes of the 1974 World Food Conference. Hence statements 1
and 3 are correct.
o The conference was organized by the United Nations in response to the food crises of the early 1970s,
when global food shortages were causing widespread famine and malnutrition, primarily in the
Sahelian countries of Africa.
• Its aim is to transform rural economies and food systems by making them more inclusive, productive,
resilient and sustainable.
• Its membership is open to any State that is a member of the United Nations, any of its specialized agencies
or the International Atomic Energy Agency.
o India is a member. Hence statement 2 is not correct.
• Nearly 100 members voluntarily support the Fund through financial contributions.
• IFAD is headed by a President elected for a four-year term.
• India has received funding from IFAD for projects in rural development, tribal development, women's
empowerment and micro-finance.
o Since 1979, it has financed more than 27 projects.
o India has also contributed to IFAD resources.

Q 47.A
• Competitive Federalism professes a vertical relationship between the States and the Central
government while states compete among themselves. Hence option (a) is the correct answer.
• Cooperative Federalism signifies a horizontal relationship between the Centre and the State. This
means the two entities cooperate and tackle shared issues to serve the larger public interest.
• Cooperative and Competitive Federalism in India:
o Cooperative and Competitive Federalism, though seem to be at loggerheads, aims towards the
common objective of economic growth and welfare of the nation in an equal and equitable manner.
o When our Constitution was first created, it only comprised the notion of cooperative federalism
through mechanisms like the Inter-state council, Zonal council, and 7th schedule.
o But in the course of time, as States competed to attract capital and Investment to facilitate and
stimulate economic activity and improve administrative efficiency, the need for Competitive
capitalism was founded.
o It ensures minimum waste and maximum resource usage by stimulating healthy competition among
states in creating physical and social infrastructure.
o NITI Aayog’s Index approach, whereby it releases sector-specific indices like School Education
Quality Index, Sustainable Development Goals Index, State Health Index, India Innovation Index,
16 www.visionias.in ©Vision IAS

FREE BY KING R QUEEN P [ऋषभ राजपूत]


Composite Water Management Index, and Export Preparedness Index, has heralded a new wave of
healthy competition among states.

Q 48.D
• A "double membership" in Indian polity refers to a situation where an individual holds
membership in more than one political party or organization simultaneously.
• In some cases, individuals who are found to be holding double membership may be expelled from their
political party or organization or may be disqualified from holding public office. This is also known as
"dual membership."
• Thus, the Representation of People Act (1951) provides for the following:
o If a person is elected to both Houses of Parliament, he must intimate within 10 days in which House
he desires to serve. In default of such intimation, his seat in the Rajya Sabha becomes vacant. Hence,
statement 1 is not correct.
o If a sitting member of one House is also elected to the other House, his seat in the first House
becomes vacant.
o If a person is elected to two seats in a House, he should exercise his option for one. Otherwise, both
seats become vacant.
o Similarly, a person cannot be a member of both the Parliament and the state legislature at the same
time. If a person is so elected, his seat in Parliament becomes vacant if he does not resign his seat in
the state legislature within 14 days. Hence, statement 2 is not correct.

Q 49.C
• Article 78 defines the duties of the Prime Minister as respecting the furnishing of information to the
President: It shall be the duty of the Prime Minister—
o to communicate to the President all decisions of the Council of Ministers relating to the administration
of the affairs of the Union and proposals for legislation. Hence statement 1 is correct.
o to furnish such information relating to the administration of the affairs of the Union and proposals for
legislation as the President may call for. Hence statement 2 is correct.
o if the President so requires, to submit for the consideration of the Council of Ministers any matter on
which a decision has been taken by a Minister but which has not been considered by the Council.
Hence statement 3 is correct.

Q 50.A
• Recent context: European nations participated in Second North Sea Summit in Ostend, Belgium. North
Sea Summit aims at making the North Sea the largest powerhouse of Europe by 2050 by taking its
offshore wind capacity to 120 GW by 2030 and 300 GW by 2050.
• 7 European Union (EU) countries (Belgium, Denmark, France, Germany, Ireland, Luxembourg,
Netherlands), and 2 Non-EU countries Norway and United Kingdom participated in this second summit.
• At the end of first summit in Denmark in 2022, Belgium, Denmark, Germany and Netherlands
signed the Esbjerg Declaration.
o Esbjerg Declaration aimed at making the North Sea into the “Green Power Plant of Europe”,
an offshore renewable energy system.
o Declaration set the targets for offshore wind energy capacity of 150 GW by 2030 and 20 GW green
hydrogen production capacity by 2030.
• Wind energy in India
o India stands 4th globally in wind power capacity as per REN21 Renewables 2022 Global Status
Report.
o As of February 2023, India’s total installed wind power capacity is approximately 42 GW.
• Hence option (a) is the correct answer.

Q 51.A
• Recently, NASA launched device called Tropospheric Emissions: Monitoring of Pollution (TEMPO)
to monitor air pollution from space.
• TEMPO will allow scientists to monitor air pollutants and their emission sources and air quality
across greater North America on an hourly basis during daytime.
• The TEMPO instrument is a UV-visible spectrometer, and will be the first ever space-based
instrument to monitor air pollutants hourly across the North American continent during daytime.
It will collect high-resolution measurements of ozone, nitrogen dioxide and other pollutants.
o Situated in geostationary orbit 35,786 kilometers above the equator.
17 www.visionias.in ©Vision IAS

FREE BY KING R QUEEN P [ऋषभ राजपूत]


o Among the pollutants tracked by TEMPO will be nitrogen dioxide, produced from the combustion of
fossil fuels, formaldehyde and ozone.
o It will be able to measure atmospheric pollution down to a spatial resolution of 10 square kilometers
or neighborhood level.
• Hence option (a) is the correct answer.

Q 52.B
• Recent context: Two massive gravity batteries are nearing completion in the US and China.
• Gravity batteries are an emerging concept in energy storage technology.
• Gravity batteries, also known as gravitational energy storage systems, are a type of potential energy
storage system that store energy in the form of gravitational potential energy.
o These systems work by raising heavy objects to a higher elevation when excess energy is available
(during periods of low demand or high renewable energy generation).
o Then, when electricity demand increases or renewable energy supply decreases, the stored potential
energy can be released by lowering the heavy objects, converting the potential energy back into
electrical energy using generators.
o Thus they do not harness energy from gravitational waves. Hence statement 1 is not correct.
• There are different designs for gravity batteries, but one common approach involves lifting heavy masses,
such as large blocks or containers of materials, using electrical energy when it is abundant. Then, when
energy is needed, these heavy masses are lowered, driving generators to produce electricity.
• According to scientists at the International Institute for Applied Systems Analysis (IIASA), abandoned
mines could provide us a way to store renewable energy by dropping containers of sand or heavy
weights down mine shafts. Thus they can be turned into giant gravity batteries. Hence statement 2
is correct.
o They claim that turning decommissioned mines into vast “gravity batteries” could provide up to 70
terawatts of energy storage. This is enough to match the entire world’s daily electricity consumption.

Q 53.B
• Commission for Agricultural Costs and Prices (CACP) is a decentralized agency of the Government of
India.
• It was established in 1965 as the Agricultural Prices Commission and was given its present name in 1985.
It is an advisory body, attached to the Ministry of Agriculture & Farmers Welfare, Government of
India. Hence, statement 1 is not correct.
• The CACP is an expert body that recommends the MSPs of the notified Kharif and Rabi crops to the
Cabinet Committee on Economic Affairs (CCEA). Hence, statement 2 is correct.
• The Commission was established to recommend Minimum Support Prices (MSPs), to motivate cultivators
and farmers to adopt the latest technology in order to optimize the use of resources and increase
productivity.

Q 54.B
• The constitutional position of the council (as compared with the assembly) can be studied from two
angles:
o Spheres where the council is equal to the assembly.
o Spheres where the council is unequal to assembly.
• Equal with Assembly: In the following matters, the powers and status of the council are broadly equal to
that of the assembly:
o Enlargement of the jurisdiction of the state public service commission. Hence, statement 1 is
correct.
o Introduction and passage of ordinary bills. However, in case of disagreement between the two Houses,
the will of the assembly prevails over that of the council.
o Approval of ordinances issued by the Governor; etc.
• Unequal with Assembly: In the following matters, the powers and status of the council are unequal to
that of the assembly:
o When an ordinary bill, which has originated in the council and was sent to the assembly, is
rejected by the assembly, the bill ends and becomes dead. Hence, statement 2 is not correct.
o The council does not participate in the election of the president of India and representatives of the
state in the Rajya Sabha.

18 www.visionias.in ©Vision IAS

FREE BY KING R QUEEN P [ऋषभ राजपूत]


o The council has no effective say in the ratification of a constitutional amendment bill. In this
respect also, the will of the assembly prevails over that of the council. Hence, statement 3 is
correct.
o A Money Bill can be introduced only in the assembly and not in the council.
o The council cannot amend or reject a money bill. It should return the bill to the assembly within 14
days, either with recommendations or without recommendations; etc.
• From the above, it is clear that the position of the council vis-a-vis the assembly is much weaker than the
position of the Rajya Sabha vis-a-vis the Lok Sabha. The Rajya Sabha has equal powers with the Lok
Sabha in all spheres except financial matters and with regard to the control over the Government. On the
other hand, the council is subordinate to the assembly in all respects.

Q 55.A
• The differences between the Public Bill and Private Bill are as stated below:
Public Bill Private Bill
1. It is introduced in the Parliament by a minister. It is introduced by any member of Parliament other
than a minister.
2. It reflects the policies of the government. It reflects the stand of the opposition party on a public
matter.
3. It has a greater chance to be approved by the Parliament. It has a lesser chance to be approved by the Parliament.
4. Its rejection by the house amounts to the expression of Its rejection by the house has no implications on the
want of parliamentary confidence in the government and parliamentary confidence in the government or its
may lead to its resignation. resignation.
5. Its introduction in the house requires seven days' Its introduction in the house requires one month's
notice notice.
6. It is drafted by the concerned department in consultation Its drafting is the responsibility of the member
with the law department. concerned.

Q 56.D
• Proton therapy is a type of radiation therapy that uses protons rather than x-rays. It painlessly
delivers radiation to treat some types of cancer.
• In general, proton therapy has fewer side effects than other types of radiation therapy. This is
because it can be targeted more directly at the tumor and does less damage to other tissue.
o A proton is a positively charged particle. At high energy, protons can destroy cancer cells.
• Proton beam therapy allows doctors to deliver a higher dose of radiation directly to the tumor while
sparing surrounding healthy tissues and organs from unnecessary radiation exposure.
o This precision helps reduce the risk of damage to nearby healthy tissues and can be particularly
beneficial for treating certain types of tumors located near sensitive organs or critical
structures.
• Hence option (d) is the correct answer.

Q 57.B
• Recent context: Researchers just discovered a method of animal reproduction previously unknown to
science, reporting that a biological law is being “broken” by a species of ant.
o The acid-spraying ant has a bizarre form of reproduction unseen until now in the animal kingdom, one
that may have allowed it to become one of the world’s worst invasive species, devastating many of
the delicate ecosystems it encounters.
• A male yellow crazy ant represents what scientists call a chimera, or a single animal with two sets of
genetically distinct cells.
o Thus a chimera is an organism whose cells are derived from two or more zygotes
o Scientists have found a handful of chimeras in nature, both in humans and other species, sometimes
resulting in people or creatures with two different colored eyes.
o But most chimerism usually stems from rare developmental incidents, such as when one embryo
absorbs another in the womb.
• Hence option (b) is the correct answer.

19 www.visionias.in ©Vision IAS

FREE BY KING R QUEEN P [ऋषभ राजपूत]


Q 58.C
• A proclamation of National Emergency under Article 352 of the Indian Constitution has drastic and
wide-ranging effects on the political system of India. Articles 358 and 359 describe the effect of a
National Emergency on Fundamental Rights.
• Article 358 deals with the suspension of the Fundamental Rights guaranteed by Article 19, while Article
359 deals with the suspension of other Fundamental Rights (except those guaranteed by Articles 20
and 21). Hence, Statement II is not correct.
• According to Article 358, when a proclamation of national emergency is made, the six Fundamental
Rights under Article 19 are automatically suspended. No separate order for their suspension is required.
• However, The 44th Amendment Act of 1978 restricted the scope of Article 358 as the six Fundamental
Rights under Article 19 can be suspended only when the National Emergency is declared on the
ground of war or external aggression and not on the ground of armed rebellion. Hence, statement I
is correct.
• Hence option (c) is the correct answer.

Q 59.D
• The Cabinet Secretariat functions directly under the Prime Minister. The business allocated to the
Cabinet Secretariat under the Government of India (Allocation of Business) Rules, 1961 includes (i)
Secretarial assistance to the Cabinet and Cabinet Committees; and (ii) Rules of Business. Hence,
statement 2 is correct.
• The Cabinet Secretariat is responsible for the administration of the Government of India (Transaction of
Business) Rules, 1961 and Government of India (Allocation of Business) Rules, 1961 facilitating smooth
transaction of business in Ministries/ Departments.
• The administrative head of the Secretariat is the Cabinet Secretary who is also the ex-officio Chairman of
the Civil Services Board. The Cabinet Secretary is the head of Civil Services in India. Hence,
statement 1 is correct.

Q 60.D
• Recent context: Kochi was the host of international working group meetings of the Commission for the
Conservation of Antarctic Marine Living Resources.
• The Commission for the Conservation of Antarctic Marine Living Resources (CCAMLR) was established
by international convention in 1982 with the objective of conserving Antarctic marine life.
o This was in response to increasing commercial interest in Antarctic krill resources, a keystone
component of the Antarctic ecosystem and a history of over-exploitation of several other marine
resources in the Southern Ocean.
• Being responsible for the conservation of Antarctic marine ecosystems, CCAMLR practises an
ecosystem-based management approach.
o This does not exclude harvesting as long as such harvesting is carried out in a sustainable manner and
takes account of the effects of fishing on other components of the ecosystem.
• CCAMLR is an international commission with 27 Members, and a further 10 countries have acceded to
the Convention. Based on the best available scientific information, the Commission agrees a set of
conservation measures that determine the use of marine living resources in the Antarctic.
o India is a member of the commission.
• There is no such arrangement for the Global Environment Facility as the principal financial arm of
the commission.
• Hence option (d) is the correct answer.

Q 61.D
• Recently, European Space Agency (ESA) has launched JUICE from Europe's spaceport French
Guiana on an eight-year-long voyage to Jupiter and its moons through Ariane-5.
• Spacecraft will complete fly-bys of Venus, Earth, and the Earth-Moon system to arrive at its destination in
2031.
o It is the first large-class mission in ESA's Cosmic Vision 2015-2025 programme.
o For the first time that the ESA has sent a spacecraft beyond the asteroid belt.
o It is a product of "global" cooperation between 23 countries, academic institutions, and private
companies.

20 www.visionias.in ©Vision IAS

FREE BY KING R QUEEN P [ऋषभ राजपूत]


• About JUICE Mission
o Objectives: It will make detailed observations about Jupiter and its three large ocean-bearing moons –
Ganymede, Callisto and Europa. o It will inspect and analyse the moons' weather, magnetic field,
gravitational pull and other elements.
o Time Period: It will spend at least three years studying the gas giant and its moons.
o Spacecraft: It has a record 85 square metres of solar panels, which stretch out to the size of a
basketball court.
o It will collect as much energy as possible near Jupiter, where sunlight is 25 times weaker than on
Earth.
o Payload: It includes GALA (GAnymede Laser Altimeter), MAJIS (Moons and Jupiter Imaging
Spectrometer), UVS (UV imaging Spectrograph) etc.
• Hence option (d) is the correct answer.

Q 62.A
• Consolidated Fund of India (Article 266): It is a fund to which all receipts are credited and all
payments are debited; Hence, statement 1 is correct.
o all revenues received by the Government of India;
o all loans raised by the Government by the issue of treasury bills, loans or ways and means of
advances; and
o all money received by the government in repayment of loans forms the Consolidated Fund of India.
o All the legally authorised payments on behalf of the Government of India are made out of this fund.
• No money out of this fund can be appropriated (issued or drawn) except in accordance with
parliamentary law. Hence statement 2 is not correct.

Q 63.A
• Recent Context: Supreme Court has taken up Maharashtra’s plea for a hearing on the border row with
Karnataka, the issue that has been simmering since the States Reorganisation Act of 1956.
• Genesis of the dispute
o The erstwhile Bombay Presidency, a multilingual province, included the present-day Karnataka
districts of Vijayapura, Belagavi, Dharwad and Uttara-Kannada.
o In 1948, the Belgaum municipality requested that the district, having a predominantly Marathi-
speaking population, be incorporated into the proposed Maharashtra state.
o However, the States Reorganisation Act of 1956, which divided states on linguistic and administrative
lines, made Belgaum and 10 talukas of Bombay State a part of the then Mysore State (which was
renamed Karnataka in 1973).
• Mahajan Commission
o In 1966, the Centre set up the Mahajan Commission to resolve the border dispute in
Maharashtra, Karnataka and Kerala. Hence option (a) is the correct answer.
o The Commission recommended that 264 villages be transferred to Maharashtra and that Belagavi
(Belgaum) and 247 villages remain with Karnataka.
o Maharashtra rejected the report, and in 2004, moved the Supreme Court.
o In 2022, the supreme court has started hearing the issue.
o Belagavi is currently part of Karnataka.

Q 64.C
• For the purpose of holding direct elections to the Lok Sabha, each state is divided into territorial
constituencies. In this respect, the Constitution makes the following two provisions:
o Each state is allotted a number of seats in the Lok Sabha in such a manner that the ratio
between that number and its population is the same for all states. This provision does not apply to
a state having a population of less than six million. Hence, statement 1 is correct.
o Each state is divided into territorial constituencies in such a manner that the ratio between the
population of each constituency and the number of seats allotted to it is the same throughout the
state. Hence, statement 2 is correct.

Q 65.C
• Recent context: Supreme court altered the approach to resolving transfer pricing disputes in India.
• Transfer pricing is an accounting practice that allows for the establishment of prices for the goods and
services exchanged between, divisions, subsidiaries or affiliates that are part of the larger enterprise.
• Usually, companies use transfer pricing to reduce the overall tax burden of the parent company.
21 www.visionias.in ©Vision IAS

FREE BY KING R QUEEN P [ऋषभ राजपूत]


• This is done by charging a higher price to subsidiaries in high-tax countries (reducing profit) while
charging a lower price (increasing profits) for subsidiaries in low-tax countries.
• About Arm’s Length Principle (ALP)
• ALP was agreed upon by all OECD member countries and adopted as an objective guideline for use by
multinational companies and tax administrations in international taxation.
• It means that the price a company pays to purchase goods or services from a related company entity
should be the same as if the two entities were unrelated (known as arm’s length price).
o There should be no price adjustment or special conditions for the transaction simply because the
parties are related legal entities.
• Its objective is to avoid the erosion of the tax base or the transfer of profits to low tax jurisdictions.
• In India, Transfer Pricing and ALP provisions are contained in Chapter X of Income-tax Act, 1961 and
arm's length price is determined by the Assessing Officer.
• Hence option (c) is the correct answer.

Q 66.D
• Territorial limits of the legislative powers
o Article 245 of the Constitution defines the territorial limits of the legislative powers vested in the
Centre and the states.
o Parliament legislative power
✓ The Parliament can make laws for the whole or any part of the territory of India.
✓ The territory of India includes the states, the union territories, and any other area for the time
being included in the territory of India.
o State legislative power
✓ A state legislature can make laws for the whole or any part of the state.
✓ The laws made by a state legislature are not applicable outside the state, except when there is
a sufficient nexus between the state and the object.
o Extra-territorial legislative power
✓ The Parliament alone can make ‘extra-territorial legislation’.
✓ It implies state legislatures do not have extra-territorial legislation power.
✓ Thus, the laws of the Parliament are also applicable to Indian citizens and their property in any
part of the world. Hence statement I is not correct.
• Inconsistency between laws made by Parliament and laws made by the Legislatures of States
o Article 254(1) of the constitution which deals with Inconsistency between laws made by Parliament
and laws made by the Legislatures of States says that:
✓ with respect to matters enumerated in the Concurrent List, the law made by Parliament,
whether passed before or after the law made by the Legislature of such State, the existing
law, shall prevail and the law made by the Legislature of the State shall, to the extent of the
repugnancy, be void. Hence statement II is correct.

Q 67.B
• Oath and Affirmation
o Every member of either House of Parliament, before taking his seat in the House, has to make
and subscribe to an oath or affirmation given in the third schedule before the President or some
person appointed by him for this purpose. In his oath or affirmation, a member of Parliament
swears:
✓ to bear true faith and allegiance to the Constitution of India;
✓ to uphold the sovereignty and integrity of India; and
✓ to faithfully discharge the duty upon which he is about to enter.
o The oath of the president is under Article 60 and not a part of the third schedule. Hence,
statement 1 is not correct.
• Unless a member takes the oath, he cannot vote and participate in the proceedings of the House and does
not become eligible for parliamentary privileges and immunities.
• A person is liable to a penalty of ₹500 for each day he sits or votes as a member in a House in the
following conditions:
o Before taking and subscribing to the prescribed oath or affirmation; or Hence, statement 2 is
correct.
o When he knows that he is not qualified or that he is disqualified for its membership; or
o When he knows that he is prohibited from sitting or voting in the House by virtue of any
parliamentary law.
22 www.visionias.in ©Vision IAS

FREE BY KING R QUEEN P [ऋषभ राजपूत]


Q 68.B
• Recently, the World Meteorological Organization (WMO) launched the Global Greenhouse Gas
Monitoring Infrastructure (GGMI).
• About GGMI
o It aims to provide better ways of measuring planet-warming pollution and help inform policy
choices.
o The new platform will integrate space-based and surface-based observing systems, and seek to clarify
uncertainties about where greenhouse gas emissions end up.
o Global coordination efforts of the type that is needed for the development of these infrastructure has
proved successful in weather prediction and climate monitoring.
✓ It is embodied by WMO’s 60-year-old World Weather Watch and its acclaimed Global
Atmosphere Watch.
o The initiative aims to leverage WMO's experience in coordinating global collaboration in weather
prediction, climate analysis, and greenhouse gas monitoring.
o GGMI will build on and expand WMO’s longstanding activities in GHG monitoring, implemented as
part of the Global Atmosphere Watch (GAW) and the Integrated Global Greenhouse Gas Information
System (IG3IS).
✓ GAW focuses on building a single coordinated global understanding of atmospheric composition,
its change, and helps to improve the understanding of interactions between the atmosphere, the
oceans and the biosphere.
✓ IG3IS is an observation-based information system for determining trends and distributions of
greenhouse gases (GHGs) in the atmosphere and the ways in which they are consistent or not with
efforts to reduce GHG emissions.
• Hence option (b) is the correct answer.

Q 69.B
• 74th Amendment Act of 1992 added a new Part IX-A to the Constitution of India. This part is
entitled ‘The Municipalities’ and consists of provisions from Articles 243-P to 243-ZG.
• Article 243V lays down that all persons who are qualified to be chosen to the State legislature shall be
qualified for being a member of a Municipality.
• However, the constitution in the same article specifically provided that persons who have attained the age
of 21 years will be eligible to be a member of the Municipality. While the constitutional requirement is
that for election to the State legislature of a state, a person must have attained the age of 25 years (Article
173). Hence, statement 1 is not correct.
• The superintendence, direction, and control of the preparation of electoral rolls and the conduct of all
elections to the municipalities shall be vested in the State Election Commission appointed under
Article 243 K of the Indian Constitution. The state legislature may make provisions with respect to all
matters relating to elections to the municipalities. Hence, statement 2 is correct.
Q 70.D
• The Finance Ministry consists of five departments, namely,
• The Department of Economic Affairs
o It is the nodal agency of the government to formulate and monitor the country’s economic policies
and programmes having a bearing on domestic and international aspects of economic management.
o A principal responsibility of this Department is the preparation and presentation of the Union
Budget to the Parliament and the budget for the state governments under the President’s Rule and
union territory administrations. Hence pair 1 is correctly matched.
o Other functions include formulation and monitoring of macroeconomic policies, including issues
relating to fiscal policy and public finance, inflation, public debt management and the functioning of
the capital market including stock exchanges; production of banknotes and coins of various
denominations, postal stationery, postal stamps; and cadre management, career planning and training
of the Indian Economic Service.
• The Department of Expenditure
o It is the nodal Department for overseeing the public financial management system in the central
government and matters connected with state finances.
o It is responsible for the implementation of the recommendations of the Finance Commission and
Central Pay Commission, monitoring of audit comments/ observations, and preparation of central
government accounts. Hence pair 2 is correctly matched.
o It further assists central ministries/ departments in controlling the costs and prices of public services,
reviewing systems and procedures to optimize outputs and outcomes of public expenditure.
23 www.visionias.in ©Vision IAS

FREE BY KING R QUEEN P [ऋषभ राजपूत]


• The Department of Revenue
o It exercises control in respect of revenue matters relating to Direct and Indirect Union taxes through
two statutory boards namely, the Central Board of Direct Taxes (CBDT) and the Central Board
of Excise and Customs (CBEC).
o The Department is also entrusted with the administration and enforcement of regulatory measures
provided in the enactments concerning Central Sales tax, Stamp duties and other relevant fiscal
statutes.
o Control over production and disposal of opium and its products is vested in this Department. Hence
pair 3 is correctly matched.
• The Department of Financial Services (DFS)
o It is mainly responsible for policy issues relating to Public Sector Banks (PSBs) and Financial
Institutions including their functioning, appointment of Chairman, Managing Director and Chief
Executive Officers (MD & CEOs), Executive Directors (EDs), Chairman cum Managing Directors
(CMDs), legislative matters, international banking relations. Appointment of Governor/Deputy
Governor of Reserve Bank of India matters relating to National Bank for Agriculture and Rural
Development (NABARD), Agriculture Finance Corporation, Co-operative Banks, Regional Rural
Banks (RRBs), and rural/agriculture Credit.
o The Department also administers the financial inclusion programme of the government, social
security schemes, and other targeted schemes aimed at facilitating flow of credit, matters relating to
the insurance sector and performance of public sector insurance companies, and administration of
various Insurance Acts. Matters relating to the Insurance Regulatory and Development Authority of
India (IRDAI) and matters relating to pension reforms including the New Pension System (NPS),
etc. Hence pair 4 is correctly matched.
• Department of Investment and Public Asset Management
o It was set up as a separate Department in 1999 and was later renamed as Ministry of Disinvestment in
2001. From 2004, the Department of Disinvestment is one of the departments under the Ministry of
Finance.
o The Department of Disinvestment has been renamed as Department of Investment and Public Asset
Management (DIPAM) from 2016.
o The mandate of the Department includes all matters related to management of Central Government
investments in equity including disinvestment of equity in Central Public Sector Undertakings.
Decisions on the recommendations of administrative ministries, NITI Aayog, etc., for disinvestment
including strategic disinvestment.
o All matters related to Independent External Monitor(s) for disinvestment and public asset
management. Financial policy in regard to the utilization of the proceeds of disinvestment channelized
into the National Investment Fund.
Q 71.A
• Article 75 of the Indian Constitution provides for the appointment, tenure, responsibility, qualification,
oath, and salaries and allowances of the ministers. As per Article 75, the Prime Minister shall be
appointed by the President and the other Ministers shall be appointed by the President on the advice of the
Prime Minister.
• The total number of ministers, including the Prime Minister, in the Council of Ministers shall not
exceed 15% of the total strength of the Lok Sabha. This provision was added by the 91st Amendment Act
of 2003.
• The Council of Ministers consists of three categories of ministers, namely, cabinet ministers, ministers of
state, and deputy ministers. At the top of all these ministers stands the Prime Minister. Thus, the Minister
of State is a part of the Central Council of Ministers. Hence option 2 is correct.
• The Cabinet Ministers head the important ministries of the Central government like home, defense,
finance, external affairs, and so forth.
• Ministers of State can either be given independent charge of ministries/ departments or can be attached to
cabinet ministers. Hence, option 1 is not correct.
• As the Ministers of State are a part of the Central Council of Ministers and they are appointed by the
President on the recommendations of the Prime Minister. Hence, option 3 is not correct.
Q 72.A
• What is Article 239AA of the Constitution?
o Article 239AA is inserted into the Constitution by the 69th Amendment Act, 1991.
o It conferred special status on Delhi following the recommendations of the S Balakrishnan
Committee that was set up in 1987 to look into Delhi’s demands for statehood. Hence statement 1 is
correct.
24 www.visionias.in ©Vision IAS

FREE BY KING R QUEEN P [ऋषभ राजपूत]


• Important Provisions of Article 239AA:
o According to its provisions, the NCT of Delhi will have an administrator and a Legislative Assembly.
o Subject to the provisions of the Constitution, the Legislative Assembly, “shall have the power to make
laws for the whole or any part of the NCT with respect to any of the matters in the State List or
Concurrent List in so far as any such matter is applicable to Union territories,” except on the
subjects of Police, Public order, and Land.
• Supreme court observation on Legislative power of NCT
o The 2018 ruling said that with the introduction of Article 239AA in the Constitution, Parliament
envisaged a “representative form of Government” for Delhi while seeking to provide a directly
elected Legislative Assembly with legislative powers over matters within the State List and the
Concurrent List, barring those exempted.
o It also sought to mandate the Lieutenant Governor to act on the aid and advice of the Council of
Ministers, except when he decides to refer the matter to the President for a final decision.
o Taking a closer look at Article 239AA(3), the court said that it revealed Parliament’s power to
make laws for Delhi, for matters given in the State and Concurrent List. Hence statement 2 is
not correct.
o At the same time, the Legislative Assembly of Delhi also has the power to make laws over all those
subjects which figure in the Concurrent List and all, but three excluded subjects, in the State List.
• Supreme court observation on Executive power of NCT
o The supreme court clarified that the Centre has exclusive executive power with respect to Delhi over
its police, land, and public order.
o In respect of other matters, the executive power is to be exercised by the Government of NCT of
Delhi.
o This, however, is subject to the proviso to Article 239AA(4) of the Constitution.

Q 73.B
• Recent context: Indian Coast Guard Region (North East) is conducting fourth edition of Table top
exercise (simulated emergency) under Colombo Security Conclave (CSC).
• CSC was formed in 2011 as a trilateral maritime security grouping of India, Sri Lanka and
Maldives.
o Later Mauritius joined as fourth member and Bangladesh & Seychelles participate as observer
countries.
• Conclave underlines regional cooperation and shared security objectives concerning all littoral nations in
the Indian Ocean Region (IOR).
• It aims to make maritime security, marine pollution response and maritime search & rescue priorities for
the region.
• Hence option (b) is the correct answer.

Q 74.D
• Recently, the Union Home Ministry has started the process for eviction and sale of enemy properties.
• Enemy property refers to property or assets held or managed on behalf of an enemy subject or an
enemy company. o It also refers to assets left behind by individuals who migrated to Pakistan or
China and are no longer citizens of India.
o These assets include land, buildings, shares held in companies, jewellery of the citizens of enemy
countries.
o The Central government took over properties of citizens of China (in 1962) and Pakistan (in 1965 and
1971) under the Defence of India Acts.
• To administer these properties, the Enemy Property Act was enacted in 1968.
o The Enemy Property (Amendment and Validation) Act, 2017 had amended the Enemy Property Act,
1968, to bar successors of those who migrated to Pakistan and China during partition from any claim
over the properties left behind in India.
o The law empowered the Custodian of Enemy Property of India (CEPI) to manage and preserve
the enemy properties. Hence statement 1 is not correct.
o Enemy properties valued between ₹1 crore and ₹100 crore will be disposed of by the CEPI through an
electronic auction or otherwise, as may be decided by the central government and at the rate
determined by the Enemy Property Disposal Committee.
• In case of the enemy properties valued below Rs 1 crore, the custodian shall offer for purchase to the
occupant first and if offer of purchase is refused by the occupant, then the enemy property shall be
disposed of in accordance with the procedure specified.
25 www.visionias.in ©Vision IAS

FREE BY KING R QUEEN P [ऋषभ राजपूत]


o CEPI worked under the aegis of Ministry of Home Affairs since 2007.
• According to a Home ministry notification, the guidelines for disposal of the enemy properties have been
changed under which the process for eviction of enemy properties now shall be initiated with the help of
the District Magistrate or Deputy Commissioner concerned before the sale of properties.
o There are a total of 12,611 establishments called enemy property, estimated to be worth over ₹1 lakh
crore, in the country.
o Out of the 12,611 properties vested with the Custodian of Enemy Property of India (CEPI), a total of
12,485 were related to Pakistani nationals and 126 to Chinese citizens.
o The highest number of enemy properties were found in Uttar Pradesh, followed by West Bengal,
Delhi.
o None of the 12,611 immovable enemy property has been monetised so far by the government.
Hence statement 2 is not correct.

Q 75.A
• Recently, The Fifth United Nations Conference on the Least Developed Countries (LDC5) concluded
with adoption of the ‘Doha Political Declaration’ by the world leaders.
• Political Declaration welcomed Doha Programme of Action (DPoA) for Least Developed Countries
(LDCs) that was adopted at first part of Conference, held in 2022 in New York.
• Through the Declaration, representatives recognized that despite some positive results, progress fell short
of goals set out in the Istanbul Programme of Action (IPoA) for LDCs for the Decade 2011– 2020.
o The IPoA recognizes that LDCs represent an enormous human and natural resource potential for
world economic growth, welfare and prosperity.
• About Doha Programme of Action (DPoA)
o DPoA manifests a new generation of renewed commitments between the LDCs and their development
partners, including the private sector, civil society, and governments at all levels.
o It is a 10-year plan (2022-2031) to put world’s 46 most vulnerable countries back on track to
achieving the UNmandated Sustainable Development Goals (SDGs).
• There are currently 46 countries (Africa 33, Asia 9, Caribbean 1 and Pacific 3) on the list of LDCs which
is reviewed every three years by the Committee for Development (CDP), a subsidiary body of the United
Nations Economic and Social Council.
• Hence option (a) is the correct answer.

Q 76.A
• Under Article 360 of the Indian Constitution, if the President is satisfied that a situation has arisen
whereby the financial stability or credit of India or of any part of the territory thereof is threatened, he
may by a Proclamation make a declaration to that effect.
• A proclamation declaring a financial emergency must be approved by both Houses of Parliament within
two months from the date of its issue. Hence, statement 1 is not correct.
• Once approved by both Houses of Parliament, the Financial Emergency continues indefinitely till it is
revoked. This implies two things:
o there is no maximum period prescribed for its operation, and
o repeated parliamentary approval is not required for its continuation. Hence, statement 2 is correct.
• During Financial Emergency, The President may issue directions for the reduction of salaries and
allowances of (a) all or any class of persons serving the Union; and (b) the judges of the Supreme Court
and the high courts. Hence, statement 3 is not correct.

Q 77.C
• As per Article 75 of the Constitution the Prime Minister shall be appointed by the President and the other
Ministers shall be appointed by the President on the advice of the Prime Minister. Hence option (c) is the
correct answer.
• The ministers shall hold office at the pleasure of the President. As the ministers hold office during the
pleasure of the President they shall be liable to dismissal even when they have the confidence of the
Legislature. But since the Prime Minister’s advice will be available in the matters of dismissing other
ministers individually, it may be expected that this power of the President will virtually be a power of the
Prime Minister against his colleagues to get rid of the undesirable colleague even when the even where
that Minister may still possess the confidence of the majority in the House of the People. Usually, the
Prime Minister exercises this power by asking an undesirable colleague to resign.

26 www.visionias.in ©Vision IAS

FREE BY KING R QUEEN P [ऋषभ राजपूत]


Q 78.A
• Recent context: An analysis report of the working of the Seventh Schedule was released by the Economic
Advisory Council to the Prime Minister (EAC-PM
• Economic Advisory Council to the Prime Minister (EAC-PM) is an independent body constituted to
give advice on economic and related issues to the Government of India, specifically to the Prime
Minister. It is not an attached body of the Prime Minister’s Office.
o At present, the chairman of EAC-PM is: Dr. Bibek Debroy. Therefore it is not chaired by
cabinet secretary. Hence statements 1 and 3 are not correct.
• The Terms of Reference of EAC-PM include analyzing any issue, economic or otherwise, referred to it by
the Prime Minister and advising him thereon, addressing issues of macroeconomic importance and
presenting views thereon to the Prime Minister.
• These could be either suo-motu or on reference from the Prime Minister or anyone else. They also
include attending to any other task as may be desired by the Prime Minister from time to time.
Hence statement 2 is correct.

Q 79.C
• Power to levy, collect and appropriate a tax:
o The Constitution draws a distinction between the power to levy and collect a tax and the power to
appropriate the proceeds of the tax so levied and collected.
o For example, the income-tax is levied and collected by the Centre but its proceeds are distributed
between the Centre and the states.
• Taxes Levied by the Centre but Collected and Appropriated by the States (Article 268):
o This category includes the following taxes and duties:
✓ Stamp duties on bills of exchange, cheques, promissory notes, policies of insurance, transfer
of shares and others.
✓ Excise duties on medicinal and toilet preparations containing alcohol and narcotics.
o In the case of these duties leviable within any Union territory, they are collected by the Government
of India. Hence statement 1 is correct.
o In other cases, these duties are collected by the States within which such duties are respectively
leviable.
o The proceeds of these duties levied within any state do not form a part of the Consolidated Fund
of India, but are assigned to that state i.e they become part of Consolidated Fund of respective state
government. Hence statement 2 is correct.

Q 80.D
• Under Article 352 of the Indian Constitution, the president can declare an emergency when the security
of India or a part of it is threatened by war, external aggression, or armed rebellion. This is
popularly known as a 'National Emergency'. However, the Constitution employs the expression
'proclamation of emergency' to denote an emergency of this type.
• The phrase 'armed rebellion' was inserted by the 44th Amendment Act of 1978, replacing the original
phrase 'internal disturbance'.
• Other Key provisions of the 44th Amendment Act of 1978:
o Fundamental Rights: The 44th Amendment Act restored the right to property as a legal right, but it
was removed as a fundamental right. This meant that property rights were still protected, but they
could no longer be enforced through a direct constitutional remedy.
o Right to Life: The amendment added "personal liberty" to Article 21 of the Constitution, making it
more comprehensive.
o Abolition of Privy Purses: The amendment abolished the privy purse and special privileges granted to
the former princely states after Independence.
o Anti-Defection Law: The Tenth Schedule was added to the Constitution by the 44th Amendment Act,
which aimed to curb political defections
• Further, it made President to declare a national emergency only on the written recommendation of the
cabinet.
• Hence, option (d) is the correct answer.

Q 81.B
• The term of the Chief Minister is not fixed and he holds office during the pleasure of the governor.
However, this does not mean that the governor can dismiss him at any time. He cannot be dismissed by
the governor as long as he enjoys majority support in the legislative assembly. But, if he loses the
27 www.visionias.in ©Vision IAS

FREE BY KING R QUEEN P [ऋषभ राजपूत]


confidence of the assembly, he must resign or the governor can dismiss him. Hence, statement 1 is
correct.
• The salary and allowances of the Chief Minister are determined by the state legislature. In addition
to the salary and allowances, which are payable to a member of the state legislature, he gets a sumptuary
allowance, free accommodation, traveling allowance, medical facilities, etc. Hence, statement 2 is not
correct.
• Before the Chief Minister enters his office, the governor administers to him the oaths of office and
secrecy. In his oath of office, the Chief Minister swears:
o to bear true faith and allegiance to the Constitution of India,
o to uphold the sovereignty and integrity of India,
o to faithfully and conscientiously discharge the duties of his office
o to do right to all manner of people in accordance with the Constitution and the law, without fear or
favor, affection or ill will. Hence, statement 3 is correct.
• He advises the governor with regard to the appointment of important officials like the advocate general,
chairman and members of the state public service commission, state election commissioner, and so on.
However, the members of the State Public Service Commission are removed by the President. Hence,
statement 4 is not correct.

Q 82.A
• The 73rd Constitutional Amendment Act provides for the reservation of seats for scheduled castes
and scheduled tribes in every panchayat (i.e., at all three levels) in proportion of their population to
the total population in the panchayat area. Further, the state legislature shall provide for the
reservation of offices of chairperson in the panchayat at the village or any other level for the SCs and
STs.
• It must be noted here that the above provision relating to the reservation of seats in panchayats (both
members and chairpersons) for the scheduled castes is not applicable to the state of Arunachal
Pradesh. This is because the state is inhabited fully by indigenous tribal people and there are no
scheduled castes. This provision was added later by the 83rd Constitutional Amendment Act of 2000.
• Hence, option (a) is the correct answer.

Q 83.C
• Question Hour
o The first hour of every parliamentary sitting is slotted for this. During this time, the members
ask questions and the ministers usually give answers. The questions are of three kinds, namely,
starred, unstarred, and short notice.
✓ A starred question (distinguished by an asterisk) requires an oral answer and hence
supplementary questions can follow.
✓ An unstarred question, on the other hand, requires a written answer and hence, supplementary
questions cannot follow.
✓ A short notice question is one that is asked by giving a notice of less than ten days. It is
answered orally.
✓ In addition to the ministers, the questions can also be asked to the private members. Thus, a
question may be addressed to a private member if the subject matter of the question relates to
some Bill, resolution, or other matter connected with the business of the House for which that
member is responsible. The procedure in regard to such questions is the same as that followed in
the case of questions addressed to a minister.
o The list of starred, unstarred, short notice questions and questions to private members are
printed in green, white, light pink and yellow color, respectively, to distinguish them from one
another. Hence, statement 3 is not correct.
o The "Question Hour" is not an Indian innovation, it is a common practice present in the
parliamentary systems of several other nations such as U.K., Canada, Australia etc. Hence
statement 1 is not correct.
• Zero Hour
o Unlike the question hour, the zero hour is not mentioned in the Rules of Procedure.
o Thus it is an informal device available to the members of the Parliament to raise matters
without any prior notice.
o The zero hour starts immediately after the question hour and lasts until the agenda for the day
(ie, regular business of the House) is taken up. In other words, the time gap between the question
hour and the agenda is known as zero hour. Hence statement 2 is not correct.
28 www.visionias.in ©Vision IAS

FREE BY KING R QUEEN P [ऋषभ राजपूत]


o It is an Indian innovation in the field of parliamentary procedures and has been in existence since
1962.

Q 84.B
• Recently, a National Conference was organized to commemorate the 25th year of enactment of the
Panchayats (Extension to the Scheduled Areas) Act or PESA Act, 1996.
• Article 243M exempts the Fifth Schedule areas from Part IX (Provision of Panchayati Raj) of the
Constitution but the Parliament is empowered to extend its provisions to the Scheduled and Tribal Areas
by law without it being considered as an amendment to the Constitution.
• Based on the recommendations of the Dileep Singh Bhuria Committee, PESA Act was enacted in 1996
for tribal empowerment and to bring them into the mainstream.
• PESA Act is called a ‘Constitution within the Constitution’ as it extends the Part IX of the Constitution
to the Fifth Schedule areas of 10 States under clause (1) of the Article 244 with certain modifications
and exceptions. Hence, statement 1 is not correct.
o Most of the North eastern states under Sixth Schedule Areas (where autonomous councils exist) are
not covered by PESA, as these states have their own Autonomous councils for governance.
o The 10 states: Andhra Pradesh, Chhattisgarh, Gujarat, Himachal Pradesh, Jharkhand, Madhya
Pradesh, Maharashtra, Orissa, Rajasthan and Telangana.
• Promoting institutions of Local Self Governance and participatory democracy, all the State Panchayati Raj
Acts for Fifth Schedule areas have the following salient features:
o All State Legislation on Panchayats shall be in conformity with the customary law, social and
religious practices and traditional management practices of community resources. Hence, statement 2
is correct.
o Every village has a separate Gram Sabha consisting of persons whose names are included in the
electoral rolls for the Panchayat at the village level.
o Every Gram Sabha to safeguard and preserve the traditions and customs of people, their
cultural identity, community resources, and the customary mode of dispute resolution. Hence,
statement 3 is not correct.
o Every panchayat to have a reservation of seats in proportion to the community population (minimum
of 50%) with the Chairperson of Panchayats at all levels to be reserved for STs.
o Roles and Responsibilities of Gram Sabhas: To approve all development works in the village, identify
beneficiaries and issue certificates of utilization of funds.

Q 85.C
• The advocate general is appointed by the governor. He must be a person who is qualified to be appointed
a judge of a high court. In other words, he must be a citizen of India and must have held a judicial office
for ten years or been an advocate of a high court for ten years.
• The term of office of the advocate general is not fixed by the Constitution. Further, the Constitution does
not contain the procedure and grounds for his removal. He holds office at the pleasure of the
governor. This means that he may be removed by the governor at any time. Hence statement 1 is correct.
• The advocate general is appointed by the governor. He must be a person who is qualified to be appointed
a judge of a high court. In other words, he must be a citizen of India and must have held a judicial office
for ten years or been an advocate of a high court for ten years. Hence statement 2 is correct.
• The remuneration of the advocate general is not fixed by the Constitution. He receives such
remuneration as the governor may determine. Hence statement 3 is correct.

Q 86.C
• The office of the Deputy Chief Minister (CM) is not mentioned in the Constitution of India. Article
163 says that there shall be a Council of Ministers with the Chief Minister as the head to aid and advise
the Governor in the exercise of his functions, except in so far as he is required to exercise his functions in
his discretion. Unlike the posts of chief minister and cabinet ministers, that of a deputy chief minister is
not a constitutional post. Hence statement 1 is correct.
• The post of Deputy CM is equivalent to the rank of a cabinet minister (in the state) and enjoys the
pay and perks that a cabinet minister is entitled to. Hence statement 2 is correct.
• The deputy CM does not enjoy any specific financial power. He also holds the same financial powers as
other cabinet ministers. Even for his own portfolio, the Deputy CM has to take approval of the chief
minister for any expenditure over and above the allocated budget. Hence, statement 3 is correct.

29 www.visionias.in ©Vision IAS

FREE BY KING R QUEEN P [ऋषभ राजपूत]


Q 87.C
• Money Bills: Article 110 of the Constitution deals with the definition of money bills.
o It states that a bill is deemed to be a money bill if it contains ‘only’ provisions dealing with all or
any of the following matters:
✓ The imposition, abolition, remission, alteration or regulation of any tax;
✓ The regulation of the borrowing of money by the Union government;
✓ The custody of the Consolidated Fund of India or the contingency fund of India, the payment of
moneys into or the withdrawal of money from any such fund;
✓ The appropriation of money out of the Consolidated Fund of India;
✓ Declaration of any expenditure charged on the Consolidated Fund of India or increasing the
amount of any such expenditure; Hence, statement 1 is correct.
✓ The receipt of money on account of the Consolidated Fund of India or the public account of India
or the custody or issue of such money, or the audit of the accounts of the Union or of a state; or
✓ Any matter incidental to any of the matters specified above.
o However, a bill is not to be deemed to be a money bill by reason only that it provides for:
✓ the imposition of fines or other pecuniary penalties, or Hence, statement 2 is correct.
✓ the demand or payment of fees for licenses or fees for services rendered; or
✓ the imposition, abolition, remission, alteration or regulation of any tax by any local authority or
body for local purposes.
o If any question arises whether a bill is a money bill or not, the decision of the Speaker of the Lok
Sabha is final.
o His decision in this regard cannot be questioned in any court of law or in the either House of
Parliament or even the President.
o When a money bill is transmitted to the Rajya Sabha for recommendation and presented to the
president for assent, the Speaker endorses it as a money bill.
o Finally, when a money bill is presented to the president, he may either give his assent to the bill
or withhold his assent to the bill but cannot return the bill for reconsideration of the Houses.
Hence, statement 3 is correct.
o Normally, the president gives his assent to a money bill as it is introduced in the Parliament with his
prior permission.

Q 88.C
• Article 77: Conduct of Business of the Government of India:
o All executive action of the Government of India shall be expressed to be taken in the name of
the President. Hence, statement 1 is correct.
o Orders and other instruments made and executed in the name of the President shall be authenticated in
such manner as may be specified in rules to be made by the President.
o The President shall make rules for the more convenient transaction of the business of the Government
of India and for the allocation among Ministers of the said business.
• In exercise of the powers conferred by clause (3) of above Article 77 of the Constitution and in
supersession of all previous rules and orders on the subject the President has made the Rules for the
Allocation of the Business of the Government of India, 1961:
o The Ministries/Departments of the Government are created by the President on the advice of the
Prime Minister under these Rules.
o The Ministries/Departments of the Government are diminished or merged by the President on the
advice of the Prime Minister.
• The Cabinet Secretariat is under the direct charge of the Prime Minister. The administrative head of the
Secretariat is the Cabinet Secretary who is also the ex-officio Chairman of the Civil Services Board.
• The business of the Government of India allocated to the Cabinet Secretariat is and, shall always be
deemed to have been, allotted to the Prime Minister. Hence statement 2 is correct.

Q 89.B
• Recent context: A drug-resistant and potentially deadly fungus has been spreading rapidly through U.S.
healthcare facilities.
• The fungus, a type of yeast called Candida auris, or C. auris, can cause severe illness in people with
weakened immune systems. The number of people diagnosed with infections.
• It has been causing severe illness in hospitalized patients and in some patients, this yeast can enter the
bloodstream and spread throughout the body, causing serious invasive infections. This yeast often does
not respond to commonly used antifungal drugs, making infections difficult to treat. Patients who have
30 www.visionias.in ©Vision IAS

FREE BY KING R QUEEN P [ऋषभ राजपूत]


been hospitalized in a healthcare facility a long time, have a central venous catheter, or other lines or tubes
entering their body, or have previously received antibiotics or antifungal medications, appear to be at
highest risk of infection with this yeast.
• It was first identified in Japan in 2009.
• Symptoms: Fever and chills that don’t go away after treatment with antibiotics.
• Mortality rate: Estimated to be between 30-60%.
• Hence option (b) is the correct answer.
Q 90.C
• Article 243U of the constitution:
• It provides for a five-year term of office for every municipality. However, it can be dissolved before the
completion of its term.
• A municipality constituted upon the dissolution of a municipality before the expiration of its
duration shall continue only for the remainder of the period for which the dissolved municipality
would have continued had it not been so dissolved. In other words, a municipality reconstituted after
premature dissolution does not enjoy the full period of five years but remains in office only for the
remainder of the period. Hence, statement 1 is correct.
• Article 243U prescribes that before dissolution a reasonable opportunity of being heard must be
given to the municipality. Hence, statement 2 is correct.
• A person shall be disqualified for being chosen as or for being a member of a municipality if he is so
disqualified
o under any law for the time being in force for the purposes of elections to the legislature of the state
concerned; or
o under any law made by the state legislature. Further, all questions of disqualifications shall be
referred to such authority as the state legislature determines.
Q 91.A
• The Tenth Schedule contains the following provisions with respect to the disqualification of
members of Parliament and the state legislatures on the ground of defection:
• Disqualification
o Members of Political Parties: A member of a House belonging to any political party becomes
disqualified for being a member of the House, (a) if he voluntarily gives up his membership of such
political party; or (b) if he votes or abstains from voting in such House contrary to any direction
issued by his political party without obtaining prior permission of such party and such act has not
been condoned by the party within 15 days. From the above provision, it is clear that a member
elected on a party ticket should continue in the party and obey the party directions.
o Independent Members: An independent member of a House (elected without being set up as a
candidate by any political party) becomes disqualified to remain a member of the House if he joins
any political party after such an election.
o Nominated Members: A nominated member of a House becomes disqualified for being a
member of the House if he joins any political party after the expiry of six months from the date
on which he takes his seat in the House. This means that he may join any political party within six
months of taking his seat in the House without inviting this disqualification. Hence, statement 1 is
correct.
o Exceptions: The above disqualification on the ground of defection does not apply in the
following two cases:
✓ If a member goes out of his party as a result of a merger of the party with another party. A merger
takes place when two-thirds of the members of the party have agreed to such a merger.
✓ If a member, after being elected as the presiding officer of the House, voluntarily gives up the
membership of his party or rejoins it after he ceases to hold that office. This exemption has been
provided in view of the dignity and impartiality of this office. It must be noted here that the
provision of the Tenth Schedule pertaining to the exemption from disqualification in case of a
split by one-third of members of the legislature party has been deleted by the 91st Amendment
Act of 2003. It means that the defectors have no more protection on grounds of splits.
o Any question regarding disqualification arising out of defection is to be decided by the presiding
officer of the House. Originally, the act provided that the decision of the presiding officer is final and
cannot be questioned in any court. However, in the Kihoto Hollohan case (1993), the Supreme
Court declared this provision as unconstitutional on the ground that it seeks to take away the
jurisdiction of the Supreme Court and the high courts. It held that the presiding officer while
deciding a question under the Tenth Schedule, functions as a tribunal. Hence, statement 2 is not
correct.
31 www.visionias.in ©Vision IAS

FREE BY KING R QUEEN P [ऋषभ राजपूत]


Q 92.D
• Seventh Schedule:
o The Constitution provides for a three-fold distribution of legislative subjects between the Centre and
the states, viz., List-I (the Union List), List-II (the State List) and List-III (the Concurrent List) in the
Seventh Schedule:
• List-I (the Union List):
o The Parliament has exclusive powers to make laws with respect to any of the matters enumerated in
the Union List.
o This list has at present 100 subjects (originally 97 subjects) like defence, banking, foreign affairs,
currency, atomic energy, insurance, communication, inter-state trade and commerce, census,
audit and so on.
• List-II (the State List):
o The state legislature has “in normal circumstances” exclusive powers to make laws with respect to
any of the matters enumerated in the State List.
o This has at present 61 subjects (originally 62 subjects) like public order, police, public health and
sanitation, agriculture, prisons, local government, fisheries, markets, theaters, gambling and so
on.
• List-III (the Concurrent List):
o Both, the Parliament and state legislature can make laws with respect to any of the matters
enumerated in the Concurrent List.
o This list has at present 52 subjects (originally 47 subjects) like criminal law and procedure, civil
procedure, marriage and divorce, succession, population control and family planning, electricity,
labour welfare, economic and social planning, drugs, newspapers, books and printing press, and
others.
o The 42nd Amendment Act of 1976 transferred five subjects to Concurrent List from State List,
that is, (a) Education b) forests, (c) Weights and Measures, (d) protection of wild animals and
birds, and (e) administration of justice; constitution and organisation of all courts except the
Supreme Court and the high courts. Hence option d is correct.
• The power to make laws with respect to residuary subjects (i.e., the matters which are not
enumerated in any of the three lists) is vested in the Parliament.
Q 93.B
• Article 356 of the Constitution of India is based on Section 93 of the Government of India Act, 1935.
• According to Article 356, President's Rule can be imposed on any state of India on the grounds of the
failure of the constitutional machinery.
• The Supreme Court in the S.R.Bommai vs Union of India case held that Presidential proclamation under
Article 356 is not absolute and the power conferred by Article 356 on the President is conditioned power.
The Supreme Court held that the presidential proclamation is not immune from judicial review. Hence,
statement 1 is correct.
• A proclamation of the President’s Rule may be revoked by the President at any time by a subsequent
proclamation. Such a proclamation does not require parliamentary approval. Hence, statement 2 is
correct.
• Article 356 was used for the first time in 1954. It has also been used in the state of Patiala and East
Punjab States Union (PEPSU) and then during Vimochana samara to dismiss the democratically elected
Communist state government of Kerala on 31 July 1959. Hence, statement 3 is not correct.

Q 94.D
• According to the 74th Constitutional Amendment Act of 1992, every state shall constitute at the district
level, a district planning committee to consolidate the plans prepared by panchayats and municipalities in
the district, and to prepare a draft development plan for the district as a whole.
• The act lays down that four-fifths (not all) of the members of a district planning committee should
be elected by the elected members of the district panchayat and municipalities in the district from
amongst themselves.
• The representation of these members in the committee should be in proportion to the ratio between the
rural and urban populations in the district (not equal). Hence both statements are not correct.

Q 95.D
• Prime Minister is the real executive authority. The Constitution does not contain any specific procedure
for the selection and appointment of the Prime Minister.
• Article 75 says only that the Prime Minister shall be appointed by the President.
32 www.visionias.in ©Vision IAS

FREE BY KING R QUEEN P [ऋषभ राजपूत]


• Before the Prime Minister enters upon his office, the President administers to him the oaths of office and
secrecy.
• The term of the Prime Minister is not fixed and he holds office during the pleasure of the President. So
long as the Prime Minister enjoys the majority in the Lok Sabha, he cannot be dismissed by the President.
• He is the chairman of:
o NITI Aayog,
o National Development Council,
o National Integration Council,
o Inter-State Council and
o National Water Resources Council.
• Hence option (d) is the correct answer.

Q 96.A
• Special provision for states
o Articles 371 to 371-J in Part XXI of the constitution contain special provisions for twelve
states which are Maharashtra, Gujarat, Nagaland, Assam, Manipur, Andhra Pradesh, Telangana,
Sikkim, Mizoram, Arunachal Pradesh, Goa and Karnataka.
o Original constitution did not contain any special provisions for these states. They have been
incorporated by the various subsequent amendments made in the context of reorganisation of the
states or conferment of statehood on the Union Territories.
o The intention behind them is to meet the aspirations of the people of backward regions of the
states or to protect the cultural and economic interests of the tribal people of the states or to
deal with the disturbed law and order condition in some parts of the states or to protect the
interests of the local people of the states.
• Special provision for state and associated article in the constitution

• Hence option (a) is the correct answer.

Q 97.A
• Recent context: Recently, the Prime Minister acknowledged Stand-Up India’s role in empowering the
SC/ST communities, women, and job creation at the completion of 7 years of the Scheme.
• It was launched in 2016 under the Ministry of Finance, Government of India.
33 www.visionias.in ©Vision IAS

FREE BY KING R QUEEN P [ऋषभ राजपूत]


o This scheme has been extended up to 2025.
• It seeks to promote entrepreneurship among Scheduled Castes (SCs)/Schedule Tribes (STs) and
Women. (Not OBCs). Hence statement 1 is not correct.
• It covers all branches of Scheduled Commercial Banks (bank loans of ₹10- 100 lakh to at least one SC/ST
and one-woman borrower per branch).
• Applicants can draw up to ₹10 lakhs as working capital by an Overdraft. Hence statement 3 is not
correct.
• The government does not allocate funds for loans under the Stand-Up India Scheme. It can be accessed in
three potential ways:
o Directly at the branch
o Through Stand-Up India Portal
o Through the Lead District Manager (LDM)
• Loans under the scheme is available for only green field project. Green field signifies, in this context,
the first time venture of the beneficiary in the manufacturing or services or trading sector. Hence
statement 2 is correct.
o In case of non-individual enterprises, 51% of the shareholding and controlling stake should be held by
either SC/ST and/or Women Entrepreneur.

Q 98.C
• Concept of Doctrine of Territorial Nexus
o According to the Doctrine of territorial nexus, laws enacted by state legislatures apply only
within the state and not elsewhere unless there is a proper relationship between the object and
the state.
o Article 245 of the Indian Constitution serves as the foundation for its power.
o In this doctrine of territorial nexus, the term "territorial" simply means "related to territory, region, or
location," and the term "nexus" simply means "related to people, things, or events."
o According to the doctrine of territorial nexus, the Indian parliament has the authority to enact
laws pertaining to Indian territory, extraterritorial territory, or causes that have an impact
there. Hence option (c) is the correct answer.
o States are subject to the Territorial Nexus Doctrine, which is frequently used in situations
involving tax regulations.
o According to this idea, it's critical for governments and objects to have a sufficient geographical
nexus.
o States collect taxes on individuals, things, and things located inside their borders, but according to the
doctrine of territorial nexus, states are also permitted to charge taxes on territory outside their borders
if there is a sufficient and legal nexus. This theory governs taxation on non-Indian citizens.
o Example:
✓ The State of Bihar taxed all the sales of Tata Iron and Steel company inside and outside the state.
Since the goods are manufactured in the state of Bihar the Supreme Court felt there is a clear case
of territorial nexus between the state and Tata Iron and Steel Company which can be taxed by
law.

Q 99.B
• The Doctrine of Colourable Legislation:
o The Doctrine of Colourable Legislation is derived from a latin maxim which means "What cannot be
done directly, should also not be done indirectly".
o This doctrine has been used in the cases to decide questions of capacity/ competency to enact a
law. Hence statement 1 is correct.
o It says that when a lawmaking body violates its given power and enact after something in an indirect
way which it can't do in a direct way, such law will be void.
• The Doctrine of Pith and substance:
o Pith denotes true nature or essence of something and Substance means the most important or essential
part of something.
o This doctrine says that when there is a question of determining whether a particular law relates
to a particular subject the court looks to the substance and true nature of the matter. Hence
statement II is correct
o The doctrine pulls out the true nature and character of the matter to bifurcate it into its
appropriate list.

34 www.visionias.in ©Vision IAS

FREE BY KING R QUEEN P [ऋषभ राजपूत]


• Comparison of two Doctrines:
o The Doctrine of Colourable Legislation comes to decide whether the authority is having the power to
legislate or not and the doctrine of pith and substance comes into application when subject matter
between 2 lists seems to be conflicting.
o The doctrine of Pith and substance pulls out the true nature and character of the matter to
bifurcate it into its appropriate list not the Doctrine of Colourable legislation.Hence statement
II is not the correct explanation for statement I.
o The Doctrine of Pith and substance is applied when legislation made by of the legislatures is
challenged or trespassed by other legislatures and Doctrine of colourable legislation is applied to
decide questions of capacity/ competency to enact a law.
• Hence option b is correct.

Q 100.B
• Distribution of legislative powers:
o The distribution of legislative powers between the Centre and the states is rigid. Consequently,
the Centre cannot delegate its legislative powers to the states and a single state cannot request the
Parliament to make a law on a state subject. Hence statement 1 is not correct.
• Distribution of executive power with consent:
o The distribution of executive power in general follows the distribution of legislative powers. But, such
a rigid division in the executive sphere may lead to occasional conflicts between the two.
o Hence, the Constitution provides for inter-government delegation of executive functions in order to
mitigate rigidity and avoid a situation of deadlock.
o Accordingly, the President may, with the consent of the state government, entrust to that
government any of the executive functions of the Centre.
o Conversely, the governor of a state may, with the consent of the Central government, entrust to
that government any of the executive functions of the state. Hence statement 2 is correct. The
legislative functions can be delegated by state legislature not the Governor.
o This mutual delegation of administrative functions may be conditional or unconditional.
• Distribution of executive power without consent:
o The Constitution also makes a provision for the entrustment of the executive functions of the Centre
to a state without the consent of that state. But, in this case, the delegation is by the Parliament and
not by the president.
o Notably, State legislature cannot delegate the executive functions without the consent of the
Centre.

Copyright © by Vision IAS


All rights are reserved. No part of this document may be reproduced, stored in a retrieval system or
transmitted in any form or by any means, electronic, mechanical, photocopying, recording or otherwise,
without prior permission of Vision IAS.

35 www.visionias.in ©Vision IAS

FREE BY KING R QUEEN P [ऋषभ राजपूत]


VISIONIAS
www.visionias.in

Test Booklet Series

TEST BOOKLET

GENERAL STUDIES (P) 2024 – Test – 4136


C
Time Allowed: Two Hours Maximum Marks: 200

INSTRUCTIONS

1. IMMEDIATELY AFTER THE COMMENCEMENT OF THE EXAMINATION, YOU SHOULD CHECK THAT THIS BOOKLET
DOES NOT HAVE ANY UNPRINTED OR TURN OR MISSING PAGES OR ITEMS, ETC. IF SO, GET IT REPLACED BY A
COMPLETE TEST BOOKLET.

2. ENCODE CLEARLY THE TEST BOOKLET SERIES A, B, C OR D AS THE CASE MAY BE IN THE APPROPRIATE PLACE IN
THE ANSWER SHEET.

3. You have to enter your Roll Number on the Test Booklet in the Box
provided alongside. Do NOT write anything else on the Test Booklet.

4. This Test Booklet contains 100 items (Questions). Each item is printed in English. Each item comprises four
responses (answers). You will select the response which you want to mark on the Answer Sheet. In case you
feel that there is more than one correct response with you consider the best. In any case, choose ONLY ONE
response for each item.

5. You have to mark all your responses ONLY on the separate Answer Sheet provided. See direction in the
answers sheet.

6. All items carry equal marks. Attempt all items. Your total marks will depend only on the number of correct
responses marked by you in the answer sheet. For every incorrect response 1/3rdof the allotted marks will be
deducted.

7. Before you proceed to mark in the Answer sheet the response to various items in the Test booklet, you have to
fill in some particulars in the answer sheets as per instruction sent to you with your Admission Certificate.

8. After you have completed filling in all responses on the answer sheet and the examination has concluded, you
should hand over to Invigilator only the answer sheet. You are permitted to take away with you the Test
Booklet.

9. Sheet for rough work are appended in the Test Booklet at the end.

DO NOT OPEN THIS BOOKLET UNTIL YOU ARE ASKED TO DO SO


1 www.visionias.in ©Vision IAS

FREE BY KING R QUEEN P [ऋषभ राजपूत]


1. With reference to India, consider the 4. Consider the following pairs:
following statements: Craft State
1. Zardozi : Uttar Pradesh
1. The Constitution of India empowers the
2. Kamdani : Odisha
Election Commission of India to allot
3. Gotta-patti : Rajasthan
symbols to parties. 4. Tilla : West Bengal
2. In case of a split in the political parties, How many of the above pairs are correctly
the High court decide on the dispute matched?
related to party symbols. (a) Only one
(b) Only two
Which of the statements given above is/are
(c) Only three
correct?
(d) All four
(a) 1 only
(b) 2 only 5. Consider the following statements regarding
(c) Both 1 and 2 the Finance Commission:
(d) Neither 1 nor 2 1. The constitution provides for the
composition of the Finance Commission
with a Chairman and four other
2. Consider the following statements with members.
respect to the Ram Prasad Bismil: 2. The president determines the
1. He was one of the founding members of qualifications for the appointment of the
the Hindustan Republican Association. chairman and members of the
commission.
2. He started a revolutionary organization
3. The commission recommends measures
named Matrivedi.
needed to augment the consolidated fund
3. He wrote a famous poem Mainpuri ki of a state to supplement the resources of
Pratigya. panchayats and Municipalities.
How many of the above statements are How many of the above statements are
correct? correct?
(a) Only one
(a) Only one
(b) Only two
(b) Only two (c) All three
(c) All three (d) None
(d) None
6. With reference to international collaboration
in the field of artificial intelligence (AI),
3. Consider the following regions:
consider the following statements:
1. Western Ghats
1. Global Partnership on AI is a group of
2. Andaman and Nicobar Islands 25 member countries of which India is
3. North-East India also a member.
4. Terai regions 2. Hiroshima AI Process is an annual
In how many of the above-mentioned summit hosted by G-20 countries.
Which of the statements given above is/are
regions Myristica swamps are found?
correct?
(a) Only one (a) 1 only
(b) Only two (b) 2 only
(c) Only three (c) Both 1 and 2
(d) All four (d) Neither 1 nor 2
2 www.visionias.in ©Vision IAS

FREE BY KING R QUEEN P [ऋषभ राजपूत]


7. Consider the following statements regarding 9. Consider the following statements:
the Centrally Sponsored Scheme (CSSs): 1. A default bail is granted because of the
1. These are extended by the Union default of the investigating agency for
Government to states under Article 282 not completing the investigation on time.
of the Indian Constitution. 2. A Magistrate can order an accused
2. On the recommendation of B. K. person to be detained in the custody of
Chaturvedi committee CSSs were the police for 15 days.
restructured to avoid overlapping of 3. The Magistrate can authorize the
schemes. detention of the accused person in
3. After the enactment of the Appropriation
judicial custody for a maximum period
Bill, the budgets allocated for a scheme
of 60 days.
is directly released by the Union Finance
How many of the above statements are
Ministry to the Consolidated Fund of
correct?
States.
(a) Only one
Which of the statements given above is/are
(b) Only two
correct?
(c) All three
(a) 2 and 3 only
(d) None
(b) 1 and 3 only
(c) 1 and 2 only
10. Consider the following statements:
(d) 2 only
Statement-I: The first communist ministry in
India came into being in Kerela in 1957.
8. Consider the following statements:
Statement-II: The Congress party was the
1. The number of recognized parties keeps
sole national party until 1977.
changing based on their performance in
Which one of the following is correct in
the general elections.
respect of the above statements?
2. In the first general elections, there were
only three national parties. (a) Both Statement-I and Statement-II are

3. Every national party is allotted a symbol correct and Statement-II is the correct

exclusively reserved for its use explanation for Statement-I

throughout the country. (b) Both Statement-I and Statement-II are

Which of the statements given above is/are correct and Statement-II is not the

correct? correct explanation for Statement-I

(a) 1, 2 and 3 (c) Statement-I is correct but Statement-II is

(b) 1 and 3 only incorrect


(c) 2 only (d) Statement-I is incorrect but Statement-II
(d) 2 and 3 only is correct
3 www.visionias.in ©Vision IAS

FREE BY KING R QUEEN P [ऋषभ राजपूत]


11. Consider the following statement with 13. It is also known as Artsakh and is a
respect to Tribunals: landlocked mountainous area in the South
1. The original Constitution did not contain Caucasus. It was claimed by both Azerbaijan
provisions with respect to tribunals. and Armenia after the fall of the Russian
2. It consists of only two Articles–Article Empire in 1917 and has remained a point of
323 A dealing with administrative tension ever since. The territory is
tribunals and Article 323 B dealing with internationally recognised as part of oil-rich
tribunals for other matters. Azerbaijan, but its inhabitants are
Which of the statements given above is/are predominantly ethnic Armenians and have
correct? their own government which has enjoyed
(a) 1 only close links to the government in
(b) 2 only neighbouring Armenia but has not been
(c) Both 1 and 2 officially recognised by it or other U.N.
(d) Neither 1 nor 2 Which of the following places is being
described in the above-given passage?
12. Consider the following statements regarding, (a) Panjsheer
the Protection of Children from Sexual (b) Kahramanmaras
Offences (POCSO) Act, 2012: (c) Kherson
1. The Act deals with sexual offenses (d) Nagorno-Karabakh
committed against children below the
age of 16 years. 14. In relation to how many of the following
2. The Act provides for the establishment matters can the Parliament and state
of special courts to expedite the trial of legislatures establish tribunals under Article
such offenses. 323 B?
3. The Act empowers the Commission for 1. Industrial and labor
Protection of Child Rights at the 2. Land reforms
national/state level for monitoring the 3. Elections to Parliament and state
implementation of the provisions of this legislatures
Act. 4. Taxation
How many of the above statements are Select the correct answer using the code
correct? given below.
(a) Only one (a) Only two
(b) Only two (b) Only three
(c) All three (c) All four
(d) None (d) None
4 www.visionias.in ©Vision IAS

FREE BY KING R QUEEN P [ऋषभ राजपूत]


15. Consider the following statements regarding 17. Consider the following statement with

Inter-State Council: respect to Tribunals:

1. It was established as a permanent body 1. The number of Tribunals was reduced


from 26 to 19 under the provisions of
through the amendment in the
The Finance Act, of 2017.
constitution.
2. The Income Tax Appellate Tribunal was
2. The Home Minister acts as the ex-officio
established as the first Tribunal in India.
chairman of the council.
3. The power to make rules to provide for
3. The Constitution provides that its
the qualifications, appointments, and
decision cannot be challenged in the
removal of chairpersons and members of
Supreme court or in any other court. the tribunals lies with the respective
How many of the above statements are state governments.
correct? How many of the statements given above

(a) Only one is/are correct?

(b) Only two (a) Only one

(c) All three (b) Only two


(c) All three
(d) None
(d) None

16. Consider the following statements about


18. With reference to the Sevottam framework,
elections in India:
which of the following statements is/are
1. As per the Constitution of India, one
correct?
candidate can contest from a maximum 1. It is an assessment improvement
of two constituencies. framework for public service delivery.
2. If a candidate elected from two 2. It was designed by the Department of

constituencies fails to resign from one of Administrative Reforms & Public

the seats within 14 days, then both his Grievances (DARPG) in 2006.

seats shall fall vacant. 3. Kerala was the first state to implement
the model.
Which of the statements given above is/are
Select the correct answer using the code
correct?
given below.
(a) 1 only
(a) 1 only
(b) 2 only
(b) 1 and 2 only
(c) Both 1 and 2
(c) 2 and 3 only
(d) Neither 1 nor 2
(d) 1, 2 and 3
5 www.visionias.in ©Vision IAS

FREE BY KING R QUEEN P [ऋषभ राजपूत]


19. Consider the following statements: 22. Consider the following statements regarding
1. The Election Commission of India has the Scheduled Castes and Scheduled Tribes
the power to deregister a political party
(Prevention of Atrocities) Act, 1989 :
if it violates the provisions of the
Representation of the People’s Act, 1. The scope of the Act to punish the
1951. public servants extends to even the
2. So far, no political party has been members of a scheduled caste or
derecognized.
scheduled tribe if they wilfully neglect
Which of the statements given above is/are
correct? their duty/duties to be performed under
(a) 1 only the act.
(b) 2 only
2. The Act has provisions for the
(c) Both 1 and 2
establishment of Special Courts for the
(d) Neither 1 nor 2
speedy trial of offenses under this Act.
20. The Directorate of Enforcement (ED) in 3. The Act has provisions related to
India plays a role in the investigation of
providing relief and rehabilitation of
offence of money laundering and violations
victims.
under which of the following acts?
1. Prevention of Money Laundering Act, How many of the above statements are
2002 correct?
2. Conservation of Foreign Exchange and
(a) Only one
Prevention of Smuggling Activities Act,
1974 (b) Only two
3. Fugitive Economic Offenders Act, 2018 (c) All three
4. The Foreign Exchange Management (d) None
Act, 1999 (FEMA)
Select the correct answer using the code
given below. 23. Consider the following statements with
(a) 1, 2 and 3 only regard to the electoral bonds in India:
(b) 3 and 4 only 1. Electoral bonds can be given to any
(c) 1 and 4 only
registered political party in India.
(d) 1, 2, 3 and 4
2. It is a banking instrument resembling
21. Consider the following: promissory notes.
1. Dharma Vira Commission 3. These are zero-coupon bonds that do not
2. Ribeiro Committee
pay any interest.
3. Gore Committee
4. Strachey Commission How many of the above statements are
How many of the above are related to police correct?
reforms?
(a) Only one
(a) Only one
(b) Only two
(b) Only two
(c) Only three (c) All three
(d) All four (d) None
6 www.visionias.in ©Vision IAS

FREE BY KING R QUEEN P [ऋषभ राजपूत]


24. Consider the following statements about 26. With reference to Titan, a submersible
District Development Coordination and operated by OceanGate, consider the
following statements:
Monitoring Committees (DISHA):
1. It is the only crewed submersible in the
1. It has been constituted by the Ministry of world that can take 5 people as deep as
Panchayati Raj. 4,000 meters.
2. It is guided by using the GPS navigation
2. It aims to ensure the optimisation of
system.
public funds, promote synergy and
Which of the statements given above is/are
convergence between different layers of correct?
governance and review intra-State (a) 1 only
(b) 2 only
disparities for greater time-bound
(c) Both 1 and 2
impact. (d) Neither 1 nor 2
3. The Chairperson of the DISHA should
be a Member of Parliament (Lok Sabha) 27. With reference to the Da Vinci glow
phenomenon, consider the following
elected from the district, nominated by
statements:
the nodal Ministry. 1. It is a phenomenon that can happen
Which of the statements given above is/are around sunset when a crescent moon is
correct? on the horizon.
2. In this phenomenon the outline of a full
(a) 1 only
moon is not visible.
(b) 2 and 3 only Which of the statements given above is/are
(c) 1 and 3 only correct?
(a) 1 only
(d) 1, 2 and 3
(b) 2 only
(c) Both 1 and 2
25. With reference to Stirling and Fuel cell air- (d) Neither 1 nor 2
independent propulsion (AIP), consider the
28. Consider the following statements regarding
following statements:
Hemis Monastery:
1. Stirling AIP is a simple system that uses 1. Hemis Monastery (the largest monastic
diesel fuel and liquid oxygen. institution in Ladakh) is situated on the
2. Fuel cell AIP generates power through western banks of the Indus River.
2. It belongs to Hinayana Buddhism.
the reverse electrolysis of oxygen and
3. It is also famous for the Hemis Festival
hydrogen. which is celebrated in honor of
Which of the statements given above is/are Padmasambhava.
Which of the statements given above are
correct?
correct?
(a) 1 only
(a) 1 and 2 only
(b) 2 only (b) 2 and 3 only
(c) Both 1 and 2 (c) 1 and 3 only
(d) 1, 2 and 3
(d) Neither 1 nor 2
7 www.visionias.in ©Vision IAS

FREE BY KING R QUEEN P [ऋषभ राजपूत]


29. Consider the following: 31. The Global Slavery Index is released by
1. Constitute a State Security Commission. which of the following organizations?
(a) ECLT foundation
2. Prescribe a minimum tenure of two
(b) World Vision
years to the police officers on (c) Walk Free Foundation
operational duties. (d) International Labour Organization

3. Set up a Police Establishment Board at


32. Consider the following statements with
the state level. respect to cooperative societies:
4. Constitute Police Complaints Authorities 1. Only state legislatures can make laws
related to cooperatives of any kind.
at the state and district level for looking
2. The Seventh Schedule of the Indian
into complaints against police officers. Constitution provides for the state
How many of the above constitute a segment legislature to legislate on cooperative
of the directives issued by the Supreme societies.
Which one of the following is correct in
Court as a part of Prakash Singh's judgment?
respect of the above statements?
(a) Only one (a) Both Statement-I and Statement-II are
(b) Only two correct and Statement-II is the correct
explanation for Statement-I
(c) Only three
(b) Both Statement-I and Statement-II are
(d) All four correct and Statement-II is not the
correct explanation for Statement for
Statement-I
30. Consider the following statements:
(c) Statement-I is correct but Statement-II is
Statement I: During the general elections in incorrect
India, a candidate who secures the highest (d) Statement-I is incorrect but Statement-II
number of votes in that constituency is is correct

declared elected.
33. Consider the following statements regarding
Statement II: The First Past the Post system Mahatma Gandhi National Rural
is followed in the general elections. Employment Guarantee Act, 2005
(MGNREGA):
Which one of the following is correct in
1. It guarantees 100 days of unskilled work
respect of the above statements? per year for every rural household.
(a) Both Statement-I and Statement-II are 2. It covers all the districts in the country
except those with more than 50 percent
correct and Statement-II is the correct
urban population.
explanation for Statement-I 3. If work is not provided within 30 days
(b) Both Statement-I and Statement-II are from when it is demanded, the worker
has to be given a daily unemployment
correct and Statement-II is not the
allowance.
correct explanation for Statement-I How many of the above statements are
(c) Statement-I is correct but Statement-II is correct?
incorrect (a) Only one
(b) Only two
(d) Statement-I is incorrect but Statement-II
(c) All three
is correct (d) None
8 www.visionias.in ©Vision IAS

FREE BY KING R QUEEN P [ऋषभ राजपूत]


34. Consider the following statements with 36. With respect to the voting rights in India,
who among the following can vote through
reference to star campaigners in elections in
Electronically Transmitted Postal Ballot
India: Papers (ETPB)?
1. The Election Commission of India has 1. Members of armed forces
2. Those employed under the Government
the power to revoke the status of a star
of India, in a post outside India.
campaigner of a party. 3. Non-Resident Indians (NRIs)
2. As per revised guidelines, the list of the Select the correct answer using the code
given below.
star campaigners has to be
(a) 1 only
communicated to the returning officer (b) 1 and 2 only
within five days from the date of (c) 2 and 3 only
(d) 1, 2 and 3
notification of an election.
Which of the statements given above is/are 37. Consider the following statements:
correct? 1. Statutory bail is a right to bail that
accrues when police fail to complete the
(a) 1 only investigation within a specified period in
(b) 2 only respect of a person in judicial custody.
2. Regular bail is granted to an accused
(c) Both 1 and 2
person who is in custody and is usually
(d) Neither 1 nor 2 granted on the basis of surety or personal
bond.
Which of the statements given above is/are
35. Consider the following statements regarding
correct?
Forest Rights Act, 2006: (a) 1 only
1. This Act recognizes only the rights to (b) 2 only
(c) Both 1 and 2
hold and live in the forest land but not
(d) Neither 1 nor 2
ownership of land.
2. The rights under the act can only be 38. Consider the following statements with
reference to the National Commission to
claimed by members or communities of
Review the Working of the Constitution:
Scheduled Tribes. 1. It was set up through an act during the
thirteenth Lok Sabha in 2000 under M.
3. The Gram Panchayat is the authority to
N. Venkatachaliah.
initiate the process for determining the 2. The commission decided to redefine and
nature and extent of Forest Rights. restructure the basic structure of the
Indian Constitution while suggesting
How many of the above statements are
reforms in the Constitution.
correct? Which of the statements given above is/are
(a) Only one correct?
(a) 1 only
(b) Only two
(b) 2 only
(c) All three (c) Both 1 and 2
(d) None (d) Neither 1 nor 2
9 www.visionias.in ©Vision IAS

FREE BY KING R QUEEN P [ऋषभ राजपूत]


39. With reference to the official languages, 42. ‘E-Government Development Index' is
which of the amendment did not add a new released by which of the following?
language/s to the eighth schedule? (a) World Justice Project
(a) 21st amendment Act, 1967 (b) United Nations Department of Economic
(b) 42nd amendment Act, 1976 and Social Affairs
(c) 71st amendment Act, 1992 (c) World Economic Forum
(d) 92nd amendment Act, 2003 (d) Amnesty International

40. Consider the following statements with 43. With reference to the lightweight payment
regard to the Chiral Bose-liquid state: system, consider the following statements:
1. It is a new phase discovered by physicist 1. It will operate on complex wired
in which electrons can even synchronize
networks backed by advanced IT
their movements.
infrastructure.
2. In this state Electrons can be frozen into
2. It can be used for critical transactions
predictable patterns.
during catastrophic events.
Which of the statements given above is/are
3. It can ensure near-zero downtime of the
correct?
payment and settlement system in the
(a) 1 only
country.
(b) 2 only
How many of the above statements are
(c) Both 1 and 2
correct?
(d) Neither 1 nor 2
(a) Only one
(b) Only two
41. Consider the following statements about the
(c) All three
social audit in India:
(d) None
1. Social Audit is the examination and
assessment of a programme/scheme
44. Recently thousands of dead fish washed up
conducted with the active involvement
of people and comparing official records on multiple beaches in southeast Texas due

with actual ground realities. to a phenomenon called, “fish kill”. Which

2. The Mahatma Gandhi National Rural of the following can be the possible reasons

Employment Guarantee Act, 2005 for this phenomenon?

(MGNREGA) was the first law to 1. Low levels of dissolved oxygen


mandate social audit as a statutory 2. Calm Seas
requirement. 3. Cloudy weather
Which of the statements given above is/are Select the correct answer using the code
correct? given below.
(a) 1 only (a) 1 and 2 only
(b) 2 only (b) 2 and 3 only
(c) Both 1 and 2 (c) 1 and 3 only
(d) Neither 1 nor 2 (d) 1, 2 and 3
10 www.visionias.in ©Vision IAS

FREE BY KING R QUEEN P [ऋषभ राजपूत]


45. Consider the following statements with 48. Which of the following countries make up
respect to tobacco cultivation: the East and Horn of Africa and the Great
1. It can be grown both in tropical and Lakes (EHAGL) region?
temperate regions. 1. Democratic Republic of the Congo
2. It is sensitive to waterlogging. 2. Burundi
3. Curing is a process used to achieve a 3. Rwanda
4. Kenya
quality of a specific tobacco type.
Select the correct answer using the code
How many of the above statements are
given below.
correct?
(a) 1, 2, and 3 only
(a) Only one
(b) 2 and 3 only
(b) Only two
(c) 3 and 4 only
(c) All three (d) 1, 2, 3 and 4
(d) None
49. Consider the following statements regarding
46. With reference to the Comptroller and abortion laws in India:
Auditor-General of India (CAG), consider 1. The recent amendment in the Medical
the following statements: Termination of Pregnancy Act (MTPA),
1. He is represented by the Finance 1971 has legalised abortion up to 20
Minister in the Parliament. weeks in case of substantial foetal
2. He is ineligible to hold any office once abnormalities.
he retires/resigns. 2. Marital rape is recognised as one of the
Which of the statements given above is/are conditions for abortion in India.
Which of the statements given above is/are
correct?
correct?
(a) 1 only
(a) 1 only
(b) 2 only
(b) 2 only
(c) Both 1 and 2
(c) Both 1 and 2
(d) Neither 1 nor 2 (d) Neither 1 nor 2

47. Consider the following statements: 50. With reference to Special Officer for
1. In Lok Sabha elections, the None of the Linguistic Minorities, consider the following
Above or NOTA option was first used in statements:
2014. 1. The Constitution originally did not make
2. In Lok Sabha elections 2019, the Voter- any provision with respect to the Special
verified Paper Audit Trails (VVPATs) Officer for Linguistic Minorities.
were first time used in all the 2. His qualifications and tenure are not
constituencies. specified in the Constitution.
3. In India for the first time the Electronic 3. The office is designated as the
Commissioner for Linguistic Minorities
Voting Machines (EVMs) were used in
and falls under the Ministry of Home
the general election in Kerala in 1982.
Affairs.
Which of the statements given above is/are
How many of the above statements are
correct?
correct?
(a) 3 only
(a) Only one
(b) 1 and 2 only (b) Only two
(c) 1 only (c) All three
(d) 1, 2 and 3 (d) None
11 www.visionias.in ©Vision IAS

FREE BY KING R QUEEN P [ऋषभ राजपूत]


51. Consider the following statements regarding 53. Consider the following statements regarding
the Right to Fair Compensation and
the Central Sector Schemes:
Transparency in Land Acquisition,
1. These are fully funded by the Union
Rehabilitation, and Resettlement Act, of
2013: Government and implemented by state
1. The Act requires the consent of at least government machinery.
50% of affected families for acquiring
2. These are formulated in subjects from
land for public-private partnership (PPP)
projects. the State List to encourage States to

2. Social Impact Assessment Study is prioritise areas that require more


mandatory under the act to acquire land
attention.
for public-private partnership (PPP)
projects under any circumstances. Which of the statements given above is/are

3. State governments can amend the act correct?


after receiving the Presidential assent.
(a) 1 only
How many of the above statements are
correct? (b) 2 only

(a) Only one (c) Both 1 and 2


(b) Only two
(d) Neither 1 nor 2
(c) All three
(d) None
54. Arrange the following in the correct
52. With reference to the difference between the chronological sequence:
Indian Comptroller and Auditor-General of
1. Lowering of voting age from 21 to 18.
India (CAG) and his British counterpart,
consider the following statements: 2. Introduction and usage of Electronic
1. Indian CAG audits the accounts after the Voting Machines (EVM).
expenditure is done but in the UK
3. First usage of Voter-verifiable paper
money cannot be drawn without the
approval of the CAG. audit trail (VVPAT).

2. In India, CAG is not a member of the 4. Adoption of Model Code of Conduct.


parliament while in Britain CAG is a
Select the correct answer using the code
member of house of the Commons.
Which of the statements given above is given below.

correct? (a) 1-4-2-3


(a) 1 only
(b) 4-2-1-3
(b) 2 only
(c) 4-1-2-3
(c) Both 1 and 2
(d) Neither 1 nor 2 (d) 1-2-4-3
12 www.visionias.in ©Vision IAS

FREE BY KING R QUEEN P [ऋषभ राजपूत]


55. How many of the following is/are not the 57. Consider the following statements regarding,

recommendations of the National the National Assessment and Accreditation

Commission to Review the Working of the Council (NAAC):

Constitution: 1. It is an autonomous body under the

1. Increase in the scope of protection from University Grants Commission (UGC)

judicial review afforded by Article 31B responsible for assessing the quality of

2. A new directive principle on population higher educational institutions in India.

control 2. Accreditation by NAAC is mandatory to

3. The new fundamental duty of vote at be recognized as an educational

elections institution in India.

4. Removal of the domicile requirement to 3. Currently, only institutes that are at least

contest Rajya Sabha elections 6 years old or from where at least two

Select the correct answer using the code batches of students have graduated can

given below: apply for accreditation.

(a) Only one Which of the statements given above are

(b) Only two correct?

(c) Only three (a) 1 and 2 only

(d) All four (b) 1 and 3 only

(c) 2 and 3 only

56. Consider the following statements about the (d) 1, 2 and 3

Right to Information Act, 2005:

1. Application should be submitted in only 58. Consider the following statements:

English and Hindi. 1. Non-Governmental Organizations in

2. Application can be handwritten or typed. India receiving foreign funds cannot use

3. In the normal course, information to an more than 50 per cent of such funds for

applicant is to be supplied within 90 administrative purposes.

days from the receipt of the application 2. Public servants are prohibited from

by the public authority. receiving foreign funds in India.

Which of the statements given above are not Which of the statements given above is/are

correct? correct?

(a) 1 and 2 only (a) 1 only

(b) 2 and 3 only (b) 2 only

(c) 1 and 3 only (c) Both 1 and 2

(d) 1, 2 and 3 (d) Neither 1 nor 2


13 www.visionias.in ©Vision IAS

FREE BY KING R QUEEN P [ऋषभ राजपूत]


59. The Consumer Protection Act was enacted in 61. Consider the following statements:

1986 with the objective of providing better Statement-I: In India, only the election
tribunals are authorized to hear election
protection of consumer interests. Under the
petitions.
Act, which of the following rights is enjoyed Statement-II: The 19th Amendment of the
by a consumer? Indian Constitution (1966) provided the

1. Right to Information High Court with the power to hear election


petitions.
2. Right-to-consumer education
Which one of the following is correct in
3. Right to representation respect of the above statements?
4. Right to safety (a) Both Statement-I and Statement-II are
correct and Statement-II is the correct
Select the correct answer using the code
explanation for Statement-I
given below.
(b) Both Statement-I and Statement-II are
(a) 1, 2, 3 and 4 correct and Statement-II is not the
(b) 2 and 4 only correct explanation for Statement-I
(c) Statement-I is correct but Statement-II is
(c) 1 and 3 only
incorrect
(d) 2 and 3 only
(d) Statement-I is incorrect but Statement-II
is correct
60. Consider the following statements regarding
62. India after independence adopted the First
the removal of the chairman of the Union
Past The Post system (FPTP). Which among
Public Service Commission (UPSC):
the following is not the reason behind its
1. The President can remove the chairman adoption?
of the UPSC on the grounds of (a) Proportional Representation is a
complicated system which may work in
'misbehavior'.
a small country but difficult to work in
2. The Constitution has not defined the
large country like India.
term 'misbehavior' in the context of the (b) The entire election system of FPTP is

removal of the chairman. extremely simple to understand even for


common voters.
Which of the statements given above is/are
(c) FPTP is a constituency based system and
correct?
representative is responsible for one
(a) 1 only locality.

(b) 2 only (d) In FPTP Voters are often asked to


choose a party and the representatives
(c) Both 1 and 2
are elected on the basis of party lists.
(d) Neither 1 nor 2
14 www.visionias.in ©Vision IAS

FREE BY KING R QUEEN P [ऋषभ राजपूत]


63. Consider the following statements in the 65. With reference to National Commission for
context of the role of a Whip: Scheduled Castes (NCSC), consider the

1. A whip is an official of a political party following statements:


1. It consists of a chairperson and other
whose task is to ensure party discipline
members who are appointed by the
in the legislature.
President by warrant under his hand and
2. Only national and state parties can have
seal.
their own whip in the Parliament.
2. It reviews and monitors the
3. A political party has a constitutional implementation of the reservation
right to issue a whip to its legislators. policy.
How many of the statements given above are Which of the statements given above is/are
correct? correct?

(a) Only one (a) 1 only


(b) 2 only
(b) Only two
(c) Both 1 and 2
(c) All three
(d) Neither 1 nor 2
(d) None

66. The National Disaster Management


64. With reference to the Central Vigilance Authority (NDMA) has been constituted
Commission, consider the following under the Disaster Management Act 2005.
statements: The chairman of the NDMA is

Statement - I - It was established under the (a) Minister of Environment


(b) Prime Minister
Prevention of Corruption Act, of 1988.
(c) Home Minister
Statement II - The Committee on Prevention
(d) Home Secretary
of Corruption, headed by Shri K. Santhanam
had recommended its formation.
67. Consider the following statements:
Which one of the following is correct in 1. The Benami Transactions (Prohibition)
respect of the above statements? Act, 1988 is one of the initiatives by the
(a) Both Statement-I and Statement-II are Government of India to curb the
correct and Statement-II is the correct incidences of corruption in the country.

explanation for Statement-I. 2. There is no global initiative regarding


corruption that legally binds the
(b) Both Statement-I and Statement-II are
signatories.
correct and Statement-II is not the
Which of the statements given above is/are
correct explanation for Statement-I.
correct?
(c) Statement-I is correct but Statement-II is
(a) 1 only
incorrect. (b) 2 only
(d) Statement-I is incorrect but Statement-II (c) Both 1 and 2
is correct (d) Neither 1 nor 2
15 www.visionias.in ©Vision IAS

FREE BY KING R QUEEN P [ऋषभ राजपूत]


68. Consider the following statements with 70. According to the Antiquities and Art

respect to cooperative societies: Treasures Act, of 1972, consider the

1. Number of directors on the board of a following statements:


1. An “antiquity” is an article or object that
cooperative society shall not exceed
is at least 500 years old.
twenty-one.
2. If it is a manuscript or record of any
2. Twenty-seven percent of the total seats
scientific, historical, literary, or aesthetic
on the board of every cooperative have
value, it should be at least 75 years old.
to be reserved for backward classes. 3. There is no need for compulsory
Which of the statements given above is/are registration of notified categories of

correct? antiquities if they are 500 years old.

(a) 1 only Which of the statements given above is/are


correct?
(b) 2 only
(a) 1 and 2 only
(c) Both 1 and 2
(b) 1, 2 and 3
(d) Neither 1 nor 2
(c) 2 and 3 only
(d) 2 only
69. With reference to Scheduled Tribes (STs),

consider the following statements: 71. Which of the following statements is/are
1. The Constitution does not define the correct with regard to the Vaikom

criteria for recognition of STs. Satyagraha?

2. According to Article 342 of the 1. It was a movement against


untouchability and caste discrimination
Constitution, STs are the tribes declared
which became violent in the later stages.
by the Parliament through a public
2. The movement resulted in the passage of
notification.
the Temple Entry Proclamation Act
3. Punjab and Haryana are the only states
(1936).
which do not have any STs. 3. Periyar E.V. Ramaswamy provided
How many of the above statements are leadership to the Vaikom Satyagraha.

correct? Select the correct answer using the code

(a) Only one given below.


(a) 1 and 2 only
(b) Only· two
(b) 2 and 3 only
(c) All three
(c) 2 only
(d) None
(d) 1, 2 and 3
16 www.visionias.in ©Vision IAS

FREE BY KING R QUEEN P [ऋषभ राजपूत]


72. Consider the following statements: 74. Consider the following pairs:
Wildlife Sanctuary State
1. Should have a Miniratna Category-I
1. Debrigarh Wildlife : Odisha
status Sanctuary
2. Should be listed on the Indian stock 2. Chinnar Wildlife : Tamil Nadu
Sanctuary
exchange with minimum prescribed
3. Amchang Wildlife : Manipur
public shareholding under SEBI Sanctuary
regulations Which of the pairs given above is/are
correctly matched?
3. Should have an average annual turnover
(a) 1, 2 and 3
of more than Rs. 15,000 crore during the (b) 1 and 3 only
last 3 years (c) 1 only
(d) 2 and 3 only
4. Should have a significant global
presence 75. Consider the following statements regarding
How many of the above are the criteria for the Hindu Succession Act, 1956:
1. It applies to everyone who is not a
allocating a Maharatna status to a Central Muslim, Christian, Parsi, or Jew by
Public Sector Enterprise? religion.
2. Women from Schedule Tribe are
(a) Only one
excluded to be recognized as legal heirs
(b) Only two even after the amendment in 2005.
(c) Only three 3. The law applies to ancestral property
and to intestate succession in personal
(d) All four
property, where succession happens as
per law and as well as through a will.
73. With reference to the Attorney General for Which of the statements given above are
correct?
India, consider the following statements:
(a) 1 and 2 only
1. He is the highest law officer in the (b) 1 and 3 only
country. (c) 2 and 3 only
(d) 1, 2 and 3
2. He is not debarred from private legal
practice. 76. Which of the following committees are
associated with the cooperative movement in
3. He should not defend the accused in
India?
criminal cases without the permission of 1. Edward Law Committee
the Government of India. 2. Maclagen Committee
3. Gadgil Committee
How many of the above statements are
4. Gorwala Committee
correct? Select the correct answer from the code
(a) Only one given below.
(a) 1 and 2 only
(b) Only· two
(b) 2, 3 and 4 only
(c) All three (c) 1, 2, 3 and 4
(d) None (d) 1 and 4 only
17 www.visionias.in ©Vision IAS

FREE BY KING R QUEEN P [ऋषभ राजपूत]


77. With reference to the alphabet ‘T’ in the 80. Hexaconazole, Fenpyroximate,
abbreviation ChatGPT, consider the Propiconazole and Quinalphos, terms
following statements: sometimes seen in the news are:
1. It stands for 'translator'. (a) pesticides in tea plantations.
2. Its main task is to translate a sentence (b) antibiotics prescribed for Tuberculosis.
from one language to another. (c) food additives used as adulterants.
Which of the statements given above is/are (d) newly discovered anti-cancer drugs
correct?
(a) 1 only
81. Consider the following statements with
(b) 2 only
reference to the official language of the
(c) Both 1 and 2
states:
(d) Neither 1 nor 2
1. Article 343 (1) of the Constitution
provides that Hindi in Devanagari script
78. Consider the following statements regarding
and English shall be the official
the Right to Education Act, 2009:
1. It provides free and compulsory languages of the Union

education to all children between the 2. Article 345 declares that the state
ages of 6 and 14 years. legislature can declare only one
2. All private schools must keep 25% of language as an official language.
seats reserved for children belonging to Which of the statements given above is/are
economically weaker sections of society. correct?
Which of the statements given above is/are (a) 1 only
correct? (b) 2 only
(a) 1 only (c) Both 1 and 2
(b) 2 only (d) Neither 1 nor 2
(c) Both 1 and 2
(d) Neither 1 nor 2 82. With reference to the National Commission
for Backward Classes (NCBC), consider the
79. Consider the following statements regarding following statements:
Consumer Protection Rights Act, 2019:
1. The 102nd Constitution Amendment Act
1. E-commerce entities are not covered
added Article 338 B and changed NCBC
under this act.
from a statutory to a constitutional body.
2. There will be no fee for filing cases for
2. It presents an annual report to the
settlement by consumers up to Rs. 5
President.
lakh.
3. The Commission, while investigating
3. Compensation can be claimed against
any matter or enquiring into any
product manufacturer or service provider
under the concept of product liability but complaint, has all the powers of a civil

not against product seller. court.


How many of the above statements are not Which of the statements given above is/are
correct? correct?
(a) Only one (a) 1 and 2 only
(b) Only two (b) 3 only
(c) All three (c) 1 and 3 only
(d) None (d) 1, 2 and 3
18 www.visionias.in ©Vision IAS

FREE BY KING R QUEEN P [ऋषभ राजपूत]


83. Systematic Voters’ Education and Electoral 86. With reference to the rights, liabilities, and
Participation Program (SVEEP) is the properties of the Union and the states,
flagship program of which of the following? consider the following statements:
(a) NITI Aayog 1. All rights and obligations of an Indian
(b) Ministry of Information and
princely state before the commencement
Broadcasting
of the constitution became the exclusive
(c) Ministry of Education
rights and obligations of the Union
(d) Election Commission of India
Government.
2. While the sea wealth in the territorial
84. He was an Indian painter, born in Kerala,
best known for uniting Hindu mythological waters is the property of the bordering

subject matter with European realist states, the sea wealth in the exclusive

historicist painting style. He was the first economic zones will be the property of
Indian artist to use oil paints, which were the Union only.
brought to India by the Britishers. His 3. Any property can be acquired by the
painting ‘Hamsa Damayanti’, which depicts Parliament as well as the state
the story of King Nala and Damayanti from governments without a constitutional
Mahabharata won the first prize at Vienna
obligation to pay compensation.
Exhibition in 1873. British government
How many of the above statements are
awarded him with Kaiser-i-Hind Gold Medal
correct?
in 1904.
(a) Only one
Who among the following is being described
(b) Only two
in the above-given passage?
(a) Nandlal Bose (c) All three

(b) Jaimini Roy (d) None

(c) Abanindranath Tagore


(d) Raja Ravi Varma 87. ‘Zoonomia Project’ sometimes seen in the
news is associated with:
85. It is a west-flowing river that originates in (a) a genome project to protect and restore
Kaprada taluka and flows through Gujarat. endangered species in North America.
Recently it was reported that fisherfolk (b) the project to prevent zoonotic diseases
living on the banks of this river are in
by expanding and improving natural
distress as chemicals from Vapi industries
areas and landscapes.
destroy river catch.
(c) to build capacities of African countries
Which of the following rivers is being
to improve the surveillance of important
described in the passage given above?
zoonotic diseases.
(a) Kolak
(b) Kaveri (d) a project to discover the genomic basis

(c) Tapi of shared and specialized traits in


(d) Narmada mammals.
19 www.visionias.in ©Vision IAS

FREE BY KING R QUEEN P [ऋषभ राजपूत]


88. Consider the following: 90. Consider the following statements regarding
1. e-Samiksha is a real-time online system the social audits:
for monitoring and follow-up action on 1. It is the audit of a government scheme
the decisions taken by the central jointly by the government and the
government with respect to the people.
2. It ensures true democracy by direct
implementation of important
participation.
Government programmes/projects.
3. It is a tool for empowerment of the
2. The Centralized Public Grievance
stakeholders which involves auditing of
Redress and Monitoring System
financial accounts.
(CPGRAMS) provides the facility to How many statements given above are
lodge a grievance online from any correct?
geographical location. (a) Only one
3. Itat e-dwar aims at assessing the States, (b) Only two
UTs and Central Ministries on the (c) All three
efficiency of e-Governance service (d) None
delivery.
How many of the above statements correctly 91. Consider the following statements regarding
describe government initiatives in order to electoral bonds:
1. Electoral bonds are purchased
improve governance and public service
anonymously by donors and are valid for
delivery?
90 days from the date of issue.
(a) Only one
2. The bonds are issued by SBI in
(b) Only two
denominations of Rs 1,000 only.
(c) Only three 3. All political parties are eligible to
(d) None receive electoral bonds.
How many of the above statements are
89. In the context of electoral funding in India, correct?
consider the following statements: (a) Only one
1. Political parties are not required to (b) Only two
reveal the name of individuals or (c) Only three
organizations giving less than Rs. (d) None
20,000.
92. Consider the following statements regarding
2. There is no limit on election expenditure
NATGRID:
by the political parties.
1. It was proposed after the 2008 Mumbai
3. Companies Act of 2013 prevents
terror attacks.
companies from donating more than 7.5 2. Its database will be available to
percent of their average net profit to a prominent federal agencies including the
political party. CBI, ED and NIA.
How many of the statements given above are Which of the statements given above is/are
correct? correct?
(a) Only one (a) 1 only
(b) Only two (b) 2 only
(c) All three (c) Both 1 and 2
(d) None (d) Neither 1 nor 2
20 www.visionias.in ©Vision IAS

FREE BY KING R QUEEN P [ऋषभ राजपूत]


93. With reference to Joint Public Service 95. Consider the following statements, in the

Commission (JPSC), consider the following context of the National Legal Services
Authority of India (NALSA):
statements:
1. Article 39 A of the Constitution of India
1. It can be created by President on the
provides for free legal aid to the poor
request of state legislatures concerned.
and weaker sections of the society, to
2. Its members holds office for a term of
promote justice on the basis of equal
six years or until they attain the age of opportunity.

62 years, whichever is earlier. 2. The Executive Chairman of the authority

3. It presents its annual report to the is the Chief Justice of India.


3. A woman is entitled to free legal aid
President.
irrespective of her income or financial
How many of the statement given above are
status.
not correct?
How many of the above statements are
(a) Only one correct?
(b) Only two (a) Only one

(c) All three (b) Only two

(d) None (c) All three


(d) None

94. Consider the following statements in respect


96. Consider the following statements regarding
of the Civil Society 20 (C20):
the Right to Information Act, of 2005:
1. It is a platform established in 2013 for 1. Private organizations that are involved in
civil society organizations to engage performing public duties or providing

with the Group of 20 (G20). public services can be considered

2. Recently, the C-20 India summit was "public authorities" under the Act.
2. It mandates that information sought by
held in Nagpur.
citizens must be provided within 15
Which of the statements given above is/are
working days.
correct?
Which of the statements given above is/are
(a) 1 only correct?
(b) 2 only (a) 1 only

(c) Both 1 and 2 (b) 2 only


(c) Both 1 and 2
(d) Neither 1 and 2
(d) Neither 1 nor 2
21 www.visionias.in ©Vision IAS

FREE BY KING R QUEEN P [ऋषभ राजपूत]


97. Consider the following pairs: (a) The Central Information Commission is
Port Country a constitutional body.
1. Sabang : Singapore (b) The Right to Information Act contains
2. Ekatha : Maldives
provisions for creating understanding
3. Chittagong : Myanmar
among the citizens towards this right.
How many of the above pairs are correctly
matched? (c) The Official Secrets Act, 1923 was
(a) Only one pair repealed by the Right to Information
(b) Only two pairs Act, 2005.
(c) All three pairs (d) The CIC submits annual report to the
(d) None of the pairs
President to be placed before both
Houses of the Parliament.
98. Consider the following pairs:
Act Provisions
1. The : Conduct of elections and 100. Consider the following statements regarding
Representation the qualifications and National Food Security Act, 2013 (NFSA):
of People Act, disqualifications for 1. Antyodaya Anna Yojana (AAY)
1950 membership of in households under NFSA are entitled to 5
Parliament and state kg of food grains per person per month.
legislatures.
2. Pregnant women and lactating mothers
2. The : Allocation of seats of the
are entitled to free nutritious meals
Representation Parliament and state
of People Act, legislatures. under the NFSA.
1951 3. Cash maternity benefit of a minimum of
3. The : Readjustment of seats and Rs. 6,000 to partly compensate for the
Delimitation the reservation of wage loss during the period of
Commission territorial constituencies. pregnancy is provided under the act.
Act, 1952
How many of the above statements are
4. Presidential : Conduct of Presidential
correct?
and Vice- and vice-presidential
(a) Only one
Presidential elections and mechanisms
Election Act for the settlement of any (b) Only two
1952 dispute arising out of such (c) All three
elections. (d) None
How many of the above pairs are correctly
matched?
(a) Only one
(b) Only two
(c) Only three
(d) All four

99. In the
Copyright © context ofIAS
by Vision the Right to Information in
All rights arewhich
India, reserved.
of theNofollowing
part of statements
this document
is may be reproduced, stored in a retrieval system or
transmitted in any form or by any means, electronic, mechanical, photocopying, recording or otherwise,
withoutcorrect?
prior permission of Vision IAS.
22 www.visionias.in ©Vision IAS

FREE BY KING R QUEEN P [ऋषभ राजपूत]


VISIONIAS
www.visionias.in
ANSWERS & EXPLANATIONS
GENERAL STUDIES (P) TEST – 4136 (2024)

Q 1.D
• The Election Symbols (Reservation and Allotment) Order, 1968 empowers the Election Commission
of India (ECI) to recognise political parties and allot symbols. The Paragraph 15 of the Order allows ECI
to decide disputes among rival groups or factions of a recognised political party staking claim to its name
and symbol. Hence, statement 1 is not correct.
• Paragraph 15 of the Election Symbols (Reservation and Allotment) Order, 1968 says the ECI is the
only authority to decide issues over claims of Party symbol in case of split. Even the Supreme Court
in Sadiq Ali and another vs. ECI case (1971) had upheld its validity. However, in case of splits in
registered but unrecognised parties, the ECI usually advises the warring factions to resolve their
differences internally or to approach the court. Hence, statement 2 is not correct.

Q 2.C
• Ram Prasad Bismil
o Context: The 126th birth anniversary of Ram Prasad Bismil, a revered symbol of patriotism and
Hindu-Muslim unity, was commemorated recently
o He joined the Arya Samaj and became a prolific writer and poet.
o He wrote patriotic verses in Hindi and Urdu under pen names like ‘Agyat’, ‘Ram’, and the one that is
most known ‘Bismil’.
o He wrote the poem Mera Janm (My Birth), venting out his anger over the death sentence handed out
to Arya Samaj missionary Bhai Parmanand.
o The Mainpuri Conspiracy, 1918
✓ Bismil started a revolutionary organization called Matrivedi (The Altar of the Motherland)
and joined forces with fellow revolutionary Genda Lal Dixit. Hence statement 2 is correct.
✓ Dixit was well-connected with the dacoits of the state and wanted to utilize them in the armed
struggle against the British.
✓ In 1918, Bismil wrote the famous poem, Mainpuri ki Pratigya. Hence statement 3 is correct.
✓ In order to collect funds for their parties, he carried out at least 3 instances of looting at
government offices in the Mainpuri district.
o Hindustan Republican Association, 1923
✓ Ashfaqullah Khan and Bismil found the Hindustan Republican Association (HRA) in
1923. Hence statement 1 is correct.
✓ Aim: Winning freedom for the country through an armed revolution.
✓ HSRA published a manifesto titled “The Revolutionary” in 1925.
✓ It was renamed the Hindustan Socialist Republican Army (HSRA), in 1928.
o Kakori Conspiracy, 1925
✓ In August 1925, an armed robbery took place on board the Kakori Express which was going from
Shahjahanpur to Lucknow. It was carrying money that had been collected at various railway
stations and was to be deposited in Lucknow.
✓ In this planned robbery, carried out to fund the activities of the HSRA, Bismil, Khan and over 10
other revolutionaries stopped the train and fled with the cash they found in it.
✓ Within a month of the robbery, many members of the HSRA were arrested.
✓ In September 1926, Bismil was arrested. After 18 month's trial, Bismil was sentenced to death.
The sentence was carried out on December 19, 1927.
✓ Ghulami Mita Do was his famous poem that denotes he was not willing to negotiate or beg for
his country’s freedom, if the British did not accede, he was willing to take it by force.
1 www.visionias.in ©Vision IAS

FREE BY KING R QUEEN P [ऋषभ राजपूत]


Q 3.B
• Myristica swamps
o Context: Climate change and human intervention threaten the Myristica swamps of Kerala.
o Myristica swamps are freshwater swamps predominated by members of the Myristicaceae family.
o The evergreen trees have evolved to live in the waterlogged conditions of the swamps.
o They have two types of roots, knee roots (pneumatophores) and stilt roots.
o They have evolved over millions of years and are comprised of old-growth trees.
o Location: In India, these unique habitats occur in the Western Ghats and a smaller distribution
exists in the Andaman and Nicobar Islands. Hence option (b) is the correct answer.
o Myristica swamps are seen next to rivers and help in retaining water and act as a sponge, ensuring
perennial water availability.
o Important Species: Gymnocranthera canarica and Myristica fatua — belonging to the primitive
Myristicaceae family.

Q 4.B
• Zardosi Embroidery
o Context: Zardosi artisans made two lakh car hangers designed like a crown for King Charles III’s
coronation.
o Zardozi comes from two Persian words - zar or zarin meaning ‘gold’, and dozi meaning ‘sewing’.
o It is a type of heavy and elaborate metal embroidery on a silk, satin, or velvet fabric base.
o Designs are often created using gold and silver threads and can incorporate pearls, beads, and precious
stones.
o Other names
✓ Zardozi: Bhopal, Delhi, Hyderabad, and Uttar Pradesh (Lucknow). Hence pair 1 is correctly
matched.
✓ Kamdani or Badla: Lucknow. Hence pair 2 is not correctly matched.
✓ Danka and gotta-patti: Rajasthan. Hence pair 3 is correctly matched.
✓ Tilla: Jammu & Kashmir and parts of western India. Hence pair 4 is not correctly matched.
o Historically, it was used to embellish the attire of the royals. It is found in Iran, Azerbaijan, Iraq,
Kuwait, Syria, Turkey, Central Asia, India, Pakistan, and Bangladesh.
o Main Centers in India: Lucknow, Farrukhabad, Chennai, Bhopal, etc.
o Zardosi embroidery has been in existence in India since the time of the Rig Veda. The Delhi Sultanate
era further boosted this craft. It attained its summit in the 17th century, under the patronage of Mughal
Emperor Akbar.
o In 2013, the Geographical Indication Registry (GIR) accorded Geographical Indication (GI)
registration to the Lucknow Zardozi.

Q 5.B
• Finance Commission
o Article 280(1) provides that the President shall at the expiration of every fifth year or at such earlier
time as the President considers necessary by order constitute a Finance Commission.
✓ It also provides that the commission shall consist of a Chairman and four other
members to be appointed by the President. Hence statement 1 correct.
o Article 280(2) provides power to the Parliament to determine the qualifications which shall be
requisite for appointment as members of the Commission and the manner in which they shall be
selected. Hence statement 2 is not correct.
o Article 280(3) provided that it shall be duty of commission to make certain recommendations to the
president. They include
✓ the distribution between the Union and the States of the net proceeds of taxes
✓ the principles which should govern the grants- in-aid of the revenues of the States
✓ the measures needed to augment the Consolidated Fund of a State to supplement the resources
of the Municipalities and panchayats in the State on the basis of the recommendations made by
the Finance Commission of the State. Hence statement 3 is correct.
Q 6.A
• Hiroshima AI Process (HAP)
o Context: The annual Group of Seven (G7) Summit, hosted by Japan, took place in Hiroshima and
initiated the Hiroshima AI process (HAP)
o HAP is an effort by G7 countries to determine a way forward to regulate artificial intelligence
(AI). Hence statement 2 is not correct.
2 www.visionias.in ©Vision IAS

FREE BY KING R QUEEN P [ऋषभ राजपूत]


o Hiroshima AI process (HAP) made a plan for ministers from the respective countries to meet by the
end of the year to establish common rules for promoting trustworthy AI.
o HAP will work in cooperation with the OECD and Global Partnership on Artificial Intelligence
(GPAI) and will discuss generative AI by the end of this year.
• Global Partnership on AI (GPAI)
o It is an international initiative to support responsible and human-centric development and use of
Artificial Intelligence (AI).
o Aim: To bridge the gap between theory and practice on AI by supporting cutting-edge research and
applied activities on AI-related priorities.
o It is built around a shared commitment to the OECD Recommendation on Artificial Intelligence.
o GPAI is a group of 25 member countries. Hence statement 1 is correct.
o India joined the group as a founding member in 2020.
o It brings together engaged minds and experts from science, industry, civil society, governments,
international organizations, and academia to foster international cooperation.
• G7
o It is an intergovernmental economic group consisting of 7 largest IMF-advanced economies such as
Canada, France, Germany, Italy, Japan, the UK, and the US.
o EU is also represented within the G7 as an invitee.
o These countries are the 7 major advanced economies as reported by the IMF.
o Formerly called G8 with Russia in it, but due to the Crimean crisis, Russia was ejected from the
group.
o It has no binding impact on policy, and all decisions and commitments made at G7 meetings need to
be ratified independently by governing bodies of member states.
o India’s membership: India is not a member of G7.

Q 7.B
• Centrally Sponsored Schemes are different from Central Sector Schemes in the sense that Central Sector
Schemes are implemented by the Center directly while Centrally Sponsored Schemes are implemented by
states.
• CSS are extended by the Union Government to States under Article 282 of the Constitution. The main
cover items are listed in the state's list. Hence statement 1 is correct.
• Under Centrally Sponsored Scheme (CSS) a certain percentage of the funding is borne by the States in the
ratio of 50:50, 70:30, 75:25, or 90:10 and the implementation is by the State Governments.
• Centrally Sponsored Schemes are formulated in subjects from the State List to encourage States to
prioritize in areas that require more attention. Funds are routed either through consolidated funds of States
or are transferred directly to State/ District Level Autonomous Bodies/Implementing Agencies.
• As per the Baijal Committee Report, April 1987, CSS have been defined as the schemes which are funded
directly by Central Ministries/Departments and implemented by States or their agencies, irrespective of
their pattern of financing, unless they fall under the Centre's sphere of responsibility i.e., the Union List.
• Conceptually both CSS and Additional Central Assistance (ACA) Schemes have been passed by the
Central Government to the State governments. The difference between the two has arisen because of the
historical evolution and the way these are being budgeted and controlled and the release of funds takes
place. In the case of CSS, the budgets are allocated under the ministries concerned themselves, and
the entire process of release is also done by them. Hence statement 3 is not correct.
• In 2014, the central government approved the Planning Commission’s proposal to merge the 147
Centrally Sponsored Schemes (CSS) and bring it down to 66 across various sectors for effective
implementation and monitoring of the 12th Five-Year Plan. The merger proposal was in line with the
recommendations of the Chaturvedi committee, which had suggested bringing down CSS to avoid
overlap. Hence statement 2 is correct.
o In BE 2014-2015 after the restructuring of centrally-sponsored schemes (CSSs), existing CSSs have
been restructured and plan schemes under which central assistance is provided to states/UTs have
been classified as central assistance to state/UT Plans. Central assistance to states/UTs under Plan
expenditure is reflective of this.
Q 8.B
• The Election Commission registers political parties for the purpose of elections and grants them
recognition as national or state parties on the basis of their poll performance. The other parties are
simply declared as registered-unrecognized parties. The recognition granted by the Commission to the
parties determines their right to certain privileges like allocation of the party symbols, provision of time
for political broadcasts on the state-owned television and radio stations and access to electoral rolls.
3 www.visionias.in ©Vision IAS

FREE BY KING R QUEEN P [ऋषभ राजपूत]


• Every national party is allotted a symbol exclusively reserved for its use throughout the
country. Similarly, every state party is allotted a symbol exclusively reserved for its use in the state or
states in which it is so recognized. Hence statement 3 is correct.
o A registered-unrecognized party, on the other hand, can select a symbol from a list of free symbols. In
other words, the Commission specifies certain symbols as ‘reserved symbols’ which are meant for the
candidates set up by the recognized parties and others as ‘free symbols’ which are meant for other
candidates.
The number of recognised parties keeps on changing on the basis of their performance in the
general elections. Hence statement 1 is correct.

• Hence statement 2 is not correct.

Q 9.B
• Why in the news?
o The Supreme Court directed lower courts to decide pending default bail applications without relying
on its own judgment of the Ritu Chhabaria case.
o What is default bail?
✓ Default bail: The right to statutory bail, often known as default bail or compulsive bail, is granted
because of the default of the investigating agency in not completing the investigation within the
specified time. Hence, statement 1 is correct.
✓ CrPC: Under Section 167(2) of CrPC, a Magistrate can order an accused person to be detained in
the custody of the police for 15 days. Beyond 15 days, the Magistrate can authorize the detention
of the accused person in judicial custody (jail) if necessary. Hence, statement 2 is correct.
✓ The maximum period of detention: However, the accused cannot be detained for more than
o 90 days (when an authority is investigating an offense punishable with death, life
imprisonment, or imprisonment for at least 10 years). Hence, statement 3 is not correct.
o 60 days (when the authority is investigating any other offense)
✓ Right, to bail: After the period of 90/60 days, if the investigation has not been completed and the
charge sheet has not been filed, the accused person has the right to be released on bail.
✓ Judicial custody: the default bail prevents the misuse of judicial custody by the investigation
agency.
✓ Article 21: The court in multiple judgments has held that the default bail flows from the Article
21 of the Constitution which guarantees the right to life and personal liberty.

Q 10.C
• There have been multiple communist governments throughout the World including in Russia, China,
Vietnam, etc. Yet, the first communist ministry ever to come to office through “democratic
elections”, was the ministry headed by E.M.S. Namboodiripad in Kerala (India) in 1957. Hence
Statement I is correct.
• There were other national parties as well before 1977 and during 1977 elections like Communist Party of
India, Communist Party of india (Marxist), Bhartiya Jan Sangh (1971), Janata Party (1977), Indian
National Congress (O), etc. The 1977 elections saw rise of opposition political parties such as the
Congress (O), Bharatiya Jana Sangh, Bharatiya Lok Dal as well as defectors from the Indian National
Congress joined to form the Janata party, which won a sweeping majority in the Indian
Parliament. Hence, Statement II is not correct.
4 www.visionias.in ©Vision IAS

FREE BY KING R QUEEN P [ऋषभ राजपूत]


Q 11.C
• The original Constitution did not contain provisions with respect to tribunals. The 42nd
Amendment Act of 1976 added a new Part XIV-A to the Constitution. Hence, statement 1 is correct.
• This part is entitled ‘Tribunals’ and consists of only two Articles–Article 323 A dealing with
administrative tribunals and Article 323 B dealing with tribunals for other matters. Hence, statement 2 is
correct.

Q 12.B
• The Protection of Children from Sexual Offences (POCSO) Act, 2012 :
o The Protection of Children from Sexual Offences (POCSO) Act, 2012 deals with sexual offences
against persons below 18 years of age, who are deemed as children. Hence statement 1 is not
correct.
o The Act for the first time, defines “penetrative sexual assault”, “sexual assault” and “sexual
harassment”.
o The offence is considered graver if it is committed by a police officer, public servant, any member of
the staff at a remand home, protection or observation home, jail, hospital or educational institution, or
by a member of the armed or security forces.
• The act empowers the National Commission for Protection of Child Rights (NCPCR) and State
Commission for Protection of Child Rights for monitoring the implementation of the provisions of
this Act in such manner as may be prescribed. Hence statement 3 is correct.
• In discharge of its duties NCPCR has been taking up the matter with regard to implementation of the
POCSO Act in respect of following aspects:
o Designation of Special Courts. Hence statement 2 is correct.
o Appointment of Special Public Prosecutors.
o Designation and implementation of modules for training of various stakeholders.
o Steps taken for spreading the awareness on the provisions of the POCSO Act.
o Setting up of child Welfare Committees (CWCs), District Child Protection Units (DCPUs) and
Special Juvenile Police Units (SJPUs).

Q 13.D
• Context: A feud over the disputed Nagorno-Karabakh region has simmered between Armenia and
Azerbaijan for 30 years.
• What is Nagorno-Karabakh?
• Nagorno-Karabakh, known as Artsakh by Armenians, is a landlocked mountainous area in the
South Caucasus. Hence option (d) is the correct answer.
• It was claimed by both Azerbaijan and Armenia after the fall of the Russian Empire in 1917 and has
remained a point of tension ever since. The territory is internationally recognized as part of oil-rich
Azerbaijan, but its inhabitants are predominantly ethnic Armenians and have their own government which
has enjoyed close links to the government in neighboring Armenia but has not been officially recognized
by it or other U.N. member states.
• Armenians, who are Christians, claim a long historical dominance in the area, dating back to several
centuries before Christ. Azerbaijan, whose inhabitants are mostly Muslim, links its historical identity to
the territory too. It accuses the Armenians of driving out Azeris who lived nearby in the 1990s. It wants to
gain full control over the enclave, suggesting ethnic Armenians take Azeri passports or leave.

5 www.visionias.in ©Vision IAS

FREE BY KING R QUEEN P [ऋषभ राजपूत]


Q 14.C
• Article 323 A provides for the adjudication or trial by administrative tribunals of disputes and
complaints with respect to recruitment and conditions of service of persons appointed to public
services and posts in connection with the affairs of the Union or of any State or of any local or other
authority within the territory of India or under the control of the Government of India or of any
corporation owned or controlled by the Government.
• Under Article 323 B, the Parliament and the state legislatures are authorized to provide for the
establishment of tribunals for the adjudication of disputes relating to the following matters:
o Taxation
o Foreign exchange, import, and export
o Industrial and labor
o Land reforms
o Ceiling on urban property
o Elections to Parliament and state legislatures
o Foodstuffs
o Rent and tenancy rights
• Hence option (c) is the correct answer.

Q 15.D
• Inter-State Council:
o It is a mechanism that was constituted “to support Centre-State and Inter-State coordination and
cooperation in India”.
o The Inter-State Council was established under Article 263 of the Constitution, which states that the
President may constitute such a body if a need is felt for it.
o The Council is basically meant to serve as a forum for discussions among various governments.
o In 1988, the Sarkaria Commission suggested the Council should exist as a permanent body, and in
1990 it came into existence through a Presidential Order. Hence statement 1 is not correct.
• Functions of the Council
o The main functions of the Council are inquiring into and advising on disputes between
states, investigating and discussing subjects in which two states or states and the Union have a
common interest, and making recommendations for the better coordination of policy and action.
Its recommendations are only advisory in nature, Hence statement 3 is not correct.
• Composition of Council:
o The Prime Minister is the chairman of the Council, whose members include the Chief Ministers of
all states and UTs with legislative assemblies, and Administrators of other UTs. Six Ministers of
Cabinet rank in the Centre’s Council of Ministers, nominated by the Prime Minister, are also its
members. Hence statement 2 is not correct.

Q 16.B
• Recent Context: The Chief Election Commissioner had asked the Ministry of Law and Justice
to limit the number of seats from which a candidate can contest to just one. In this regard, the Supreme
Court has refused to set aside a provision (Section 33 (7)) of the Representation of People Act and has
left it to the wisdom of the Parliament.
• As per Section 33(7) of the RPA (Representation of the People Act), 1951 (and not the
Constitution), one candidate can contest from a maximum of two constituencies. Hence, statement 1
is not correct.
• More constituencies were allowed until 1996, when the RPA was amended to set the cap at two
constituencies. Thus, in the past, candidates have contested from three Constituencies as well. For
example: Shri Devi Lal, deputy chief minister of Haryana contested from three Lok Sabha seats.
• According to the Indian Constitution, an individual cannot simultaneously be a member of either
House of Parliament (or a state legislature), or both Parliament and a state legislature, or represent
more than one seat in a House.
o If a candidate is elected from two constituencies, he/she has to resign one seat within 14 days of
the declaration of the result. If failed to do so, both his seats shall fall vacant. Hence, statement 2 is
correct.
o Additionally: The members of state legislatures elected to the Lok Sabha must resign their seats
within 14 days from the date of the publication of the result in the Gazette of India, failing which the
seats in Lok Sabha shall automatically fall vacant.

6 www.visionias.in ©Vision IAS

FREE BY KING R QUEEN P [ऋषभ राजपूत]


Q 17.B
• The Income Tax Appellate Tribunal was established as the first Tribunal in India in the year
1941. The objective was to reduce the workload of courts, expedite adjudication of disputes, and build
expertise on tax matters within the Tribunal. Hence, statement 2 is correct.
• The Finance Act, of 2017 reorganised the tribunal system by merging tribunals based on functional
similarity. The number of Tribunals was reduced from 26 to 19. Hence, statement 1 is correct.
• It delegated powers to the central government to make Rules to provide for the qualifications,
appointments, removal, and conditions of service for chairpersons and members of these tribunals. Hence,
statement 3 is not correct.

Q 18.B
• The Department of Administrative Reforms & Public Grievances (DARPG), Ministry of Personnel,
Public Grievances and Pensions has taken steps to put in place Sevottam Compliant Citizen’s Charter and
Grievance Redress Mechanism. The Sevottam framework was designed by DARPG in 2006 as an
assessment improvement framework for public service delivery. Hence statements 1 and 2 are
correct.
• Sevottam literally is the combination of the Hindi words ‘SEWA + UTTAM’, meaning uttam sewa i.e.
excellence in services. The Sevottam model was developed with expert support after studying
international best practices, stakeholder consultations and field validity. It has basically three modules -
Citizen Charter, Public Grievance Redress Mechanism and Service Delivery Capability.
• The Seven Steps to Sevottam are - Define all services which a department provides and identify clients;
Set standards and norms for each service; Develop the capability to meet the set standards; Perform to
achieve the standards; Monitor performance against the set standards; Evaluate the impact through an
independent mechanism and Continuous improvement based on monitoring and evaluation results.
• As the State Governments provide a very large number of pro-poor services, the Quality Management
System Sevottam was piloted in four States in four different sectors, namely – (i) Water supply and
Sanitation, (ii) Women and Child Development for Integrated Child Development Services through
Anganwadi Centres, (iii) Food Supplies and Consumer Affairs, and (iv) Public Health and Family Welfare
through Primary Health Centres. These states are Himachal Pradesh, Karnataka, Madhya Pradesh
and Odisha. Hence, statement 3 is not correct.

Q 19.D
• Deregistration means the cancellation of the registration of a political party. In India, the Election
Commission (ECI) is not empowered to de-register parties. Hence statement 1 is not correct.
• The ECI can only derecognize a political party. Derecognition means the withdrawal of the
recognition of a political party by the ECI. Such parties are known as registered unrecognized parties.
• ECI derecognizes a political party if it violates either:
o the Indian Constitution or
o the Representation of the People Act, 1951.
• Recently, the ECI derecognized the TMC, NCP and CPI as national parties. Hence statement 2 is
not correct.
• The grounds for such derecognition (as per ECI) include:
o If the party has failed to secure at least 6% of the total votes polled in the general election to the Lok
Sabha or the legislative assembly of the state concerned, and if it fails to have at least 4 MPs elected
in the last LS polls (also, if it doesn't win 1 seat in the LS from the same state),
o If it has failed to win at least 2% of the total seats in the LS from at least 3 states
o If it has failed to secure 8% of the total valid votes polled in the state at a General Election to the LS
from the state or to the State LA.
o If the party has failed to submit its audited accounts to the ECI on time.
o If the party has failed to hold its organizational elections on time.

Q 20.D
• The Directorate of Enforcement (ED) has been given the responsibility to enforce the provisions of the
Prevention of Money Laundering Act, 2002 by conducting investigation to trace the assets derived from
proceeds of crime, to attach the property provisionally and to ensure prosecution of the offenders and
confiscation of the property by the Special court.
• Under the Foreign Exchange Management Act, 1999 (FEMA), ED has been given the responsibility to
conduct investigations into suspected contraventions of foreign exchange laws and regulations, and to
adjudicate and impose penalties on those convicted to have contravened the law.
7 www.visionias.in ©Vision IAS

FREE BY KING R QUEEN P [ऋषभ राजपूत]


• Under The Fugitive Economic Offenders Act, 2018 (FEOA), the ED is mandated to attach the
properties of fugitive economic offenders who have escaped from India warranting arrest and provide for
confiscating their properties to the Central Government.
• Under the Conservation of Foreign Exchange and Prevention of Smuggling Activities Act, 1974
(COFEPOSA), the ED is empowered to sponsor cases of preventive detention with regard to the
contraventions of FEMA.
• Hence, option (d) is the correct answer.

Q 21.C
• Dharma Vira Commission: The Government of India appointed a National Police Commission in 1977,
as it felt that “far-reaching changes have taken place in the country” since independence but “there has
been no comprehensive review at the national level of the police system after independence despite
radical changes in the political, social and economic situation in the country”.
• The Commission even drafted a model Police Bill which could be enacted. Its recommendations,
however, received no more than a cosmetic treatment at the hands of the Government of India.
• Other Committees: Apart from the National Police Commission, several other bodies were constituted
from time to time to go into the question of police reforms. These were:
o Gore Committee on Police Training (1971-73)
o Ribeiro Committee on Police Reforms (1998)
o Padmanabhaiah Committee on Police Reforms (2000)
o Group of Ministers on National Security (2000-01)
o Malimath Committee on Reforms of Criminal Justice System (2001-3)
• The Gore Committee was constituted to review the state of police training in the country and suggest
improvements. The Ribeiro Committee was set up by the Supreme Court while it was deliberating over
the Public Interest Litigation filed for police reforms; the Court wanted the Committee to examine if the
National Police Commission’s recommendations, which formed the core of the PIL, were still relevant or
that any modifications were called for.
• The Padmanabhaiah Committee examined the requirements of policing in the new millennium. The
Group of Ministers examined the reports of various Committees which were set up in the wake of
Pakistan’s aggression in Kargil, including the one dealing with internal security, and suggested
comprehensive measures to strengthen the internal and external security apparatus. The Malimath
Committee made far-reaching recommendations to reform the criminal justice system.
• Hence, 1, 2, and 3 are related to police reforms.
• Richard Strachey Commission of 1880 was created to develop a general strategy and principles to
deal with famines. It was the first Famine commission and was constituted during the period of
Lord Lytton.
• Hence, 4 is not related to police reforms.
• Hence, option (c) is the correct answer.

Q 22.B
• Scheduled Castes and Scheduled Tribes (Prevention of Atrocities) Act, 1989: The main objectivesof
the act includes:
o To prevent the commission of offences of atrocities against the members of the Scheduled Castes and
the Scheduled Tribes
o To provide for Special Courts and the Exclusive Special Courts for the trial of such offences.
Hence statement 2 is correct.
o To provide relief and rehabilitation of the victims of such offences and for matters connected
therewith or incidental thereto. Hence statement 3 is correct.
• Punishment for neglect of duties by Public servant:
o Whoever, being a public servant but not being a member of a Scheduled Caste or a Scheduled
Tribe, wilfully neglects his duties required to be performed by him under this Act shall be punishable
with imprisonment for a term minimum of six months but which may extend to one year. Hence
statement 1 is not correct.
o The duties of public servant referred in the act include:
✓ To read out to an informant the information given orally, and reduced to writing by the officer in
charge of the police station, before taking the signature of the informant;
✓ To record the statement of the victims or witnesses;
✓ To conduct the investigation and file charge sheet in the Special Court or the Exclusive Special
Court within a period of sixty days.
8 www.visionias.in ©Vision IAS

FREE BY KING R QUEEN P [ऋषभ राजपूत]


o Provided that the charges in this regard against the public servant shall be booked on the
recommendation of an administrative enquiry.
• The cognizance in respect of any neglect of duty referred to above duties by a public servant( be it
member of a scheduled caste ot a scheduled tribe) shall be taken by the Special Court or the Exclusive
Special Court and shall give direction for penal proceedings against such public servant.

Q 23.B
• In order to curb black money and other malpractices, the electoral bond scheme was introduced
in 2017. It will allow donors to pay political parties using banks as intermediaries.
• It is a banking instrument resembling promissory notes. Hence, statement 2 is correct.
• Although called bonds, these do not bear any interest. They can be given directly to the political party
of choice, which can then encash them. Hence, statement 3 is correct.
• The electoral bonds are a bearer's instrument and do not carry the name of the payee. These are
available for purchase by any citizen of India. These can be bought for any value, in multiples of Rs
1,000, Rs 10,000, Rs 1 lakh, Rs 10 lakh, or Rs 1 crore. Their purchase is anonymous, and they are valid
for 15 days from the date of issue.
• The electoral bonds can only be given to a registered political party that has secured at least 1% of
the vote in the last Lok Sabha or state assembly elections (not all parties). Hence, statement 1 is not
correct.
• As per a report by the SBI, for national parties, there was a 743% increase in donations from electoral
bonds between FY 2017–18 and FY 2021–22, while for corporate donations, this increase is only 48%. In
this regard, the total donations declared by the BJP are more than three times the total donations declared
by all other national parties, the report said.

Q 24.B
• District Development Coordination and Monitoring Committees (DISHA), have been constituted by
the Ministry of Rural Development at district levels under the Chairpersonship of concerned Member of
Parliament to monitor proper implementation of key Central Schemes/ Programmes and to ensure better
coordination at all levels of elected representatives for efficient and time-bound development of
districts. Hence, statement 1 is not correct.
• Similarly, State Development Coordination and Monitoring Committees have also been constituted under
the chairpersonship of the concerned Chief Minister/Administrator to attend to the matters which need to
be resolved at the highest level in the State/UT.
• DISHA is an important step towards participatory governance with the mantra of ‘Minimum Government,
Maximum Governance’ and to attain the global agenda for 2030 including SDGs based on inclusiveness
and universality- ‘Leave No One Behind’.
• The main objective of DISHA is to ensure the quality of expenditure - optimization of public funds;
monitor programme implementation; promote synergy and convergence between different layers of
governance and review intra- State disparities for greater time-bound impact. Hence, statement 2 is
correct.
• The Chairperson of the DISHA should be a Member of Parliament (Lok Sabha) elected from the
district, nominated by the Ministry of Rural Development. The other Members of Parliament (Lok Sabha)
representing the district should be designated as Co-Chairpersons. Hence, statement 3 is correct.
• The Member Secretary is the District Collector/Deputy Commissioner/ the CEO Zilla Parishad or a Senior
ADM to be the Member Secretary for a particular meeting to ensure that meetings of the DISHA are held
as scheduled.
• The other Members of the Committee are members of the State Legislative Assembly elected from the
district, one representative of the State, all Mayors/ Chairpersons of Municipalities including one woman
and five elected heads of Gram Panchayat including two women, Chairperson and CEO of the Zilla
Panchayat and Chairpersons of block panchayat, Head of the Autonomous District Council in districts
having Schedule VI Areas and Project Director of DRDA.
• Moreover, one member from a reputed NGO, SC, ST and women, lead bank officer of the district, senior
superintendent of the postal department and District level nodal functionaries of all programmes that will
be under the purview of Disha.

Q 25.C
• Fuel cell AIP Vs Stirling AIP
o Context: The German shipbuilders signed an agreement with India’s Mazagon Dock Limited (MDL)
to cooperate in building 6 state-of-the-art submarines for the Indian Navy.
9 www.visionias.in ©Vision IAS

FREE BY KING R QUEEN P [ऋषभ राजपूत]


o Conventional diesel-electric submarines: They can operate underwater silently for up to 48 hours.
o AIP-driven submarines: They remain underwater for up to 2 weeks before they must surface for
battery charging.
o Fuel cell AIP
✓ Fuel cell-based AIP generates power through the reverse electrolysis of oxygen and
hydrogen. Hence statement 2 is correct.
✓ This process does not need air but requires the storage of highly inflammable hydrogen on board.
✓ Fuel cell AIP systems are associated with German submarines of Class 212A and Class 214 and
will power the Project 75-I submarines too.
✓ Other fuel cell AIPs are being developed in other countries, such as Russia (alkaline fuel cells)
and India (phosphoric-acid fuel cells).
✓ Class 212A submarines are 3 times larger than previous German Navy Class 206 submarines.
✓ In addition, fuel cell-based AIP systems require meticulous crew training and dedicated
infrastructure.
✓ A key problem in fuel cell development was to master hydrogen storage.
o Stirling AIP
✓ The 2nd type of AIP, based on the Stirling engine, is the first modern combat-ready system.
✓ It is a simple system that uses diesel fuel (typical for the submarine) and liquid oxygen,
discharging the plant overboard at small and medium depths. Hence statement 1 is correct.
✓ Low-power Stirling engines are much quieter than the main diesel generators of submarines,
providing considerable tactical gains.
✓ It took less than 15 years for the creation of this system from concept to implementation.
✓ Although this system lacks stealth, it has proved relatively good in small areas, shallow depths,
complex hydrology, and heavy traffic, which require a submarine’s acoustic signature to be
minimized.
✓ The Stirling engine is compact enough for small submarines.

Q 26.A
• Submersible vs Submarine
• Context: The Titan, one of the crewed submersibles operated by OceanGate, went missing in the area of
the Titanic wreck in the North Atlantic.
o Titan: Titan is the only crewed submersible in the world that can take 5 people as deep as 4,000
meters enabling it to reach almost 50% of the world’s oceans. It is made of carbon fiber and titanium.
The submersible was part of an 8-day journey conducted by OceanGate Expeditions to reach the
Titanic wreck site.
o Submersible
✓ A submersible is a small boat or other craft, designed especially for research and
exploration. Hence statement 1 is correct.
✓ It does not function as an autonomous craft and needs a mother ship that can launch and
recover it.
✓ The submersibles have a top speed of 3 knots.
✓ It can’t stay underwater for as long compared to submarines.
✓ Since there is no GPS underwater, the submersible is only guided by text messages from the
surface ship. The pilot steers the sub using a video game controller but if that fails, a hard-wired
system can control the propellers. Hence statement 2 is not correct.
o Submarines
✓ A submarine is a watercraft capable of independent operation underwater. It refers to
any naval vessel that is capable of propelling itself beneath the water as well as on the
water’s surface.
✓ When the submarine is to dive, water is filled in water tanks and it is made heavier.
✓ Since the average density of a submarine becomes greater than the density of seawater, it sinks.
✓ To make the submarine rise to the surface of the water, water tanks are emptied.

Q 27.A
• Da Vinci glow
o Context: The experts predicted that a ‘da Vinci glow’ would be visible around the new moon on May
19, 2023.
o The da Vinci glow is a phenomenon in which the crescent moon is on the horizon, but the outline of a
full moon is visible. Hence, statement 2 is not correct.
10 www.visionias.in ©Vision IAS

FREE BY KING R QUEEN P [ऋषभ राजपूत]


o The phenomenon usually happens close to sunset. Hence, statement 1 is correct.
o On some days, this happens after the sun sets, and the sky is dark but not completely dark.
o This glow is also called earthshine since it occurs due to light reflected from the earth and not the sun.
o The earthshine is the sunlight first reflected by the earth’s surface, and then it’s reflected by the moon
before it reaches our eyes.
o Since it is reflected twice, it is dimmer than the visible part of the moon and thus appears as a faint
glow, called the da Vinci glow. Earth's clouds are the primary source of this earthshine.
o According to NASA, earthshine makes night on the moon 50 times brighter than a full-moon night on
the Earth.
o Leonardo da Vinci (1452-1519) was the first person to describe the phenomenon in the 16th century
and it was named after him. Before da Vinci’s theory, the ‘ashen glow’ of the moon was often called
the ‘old moon in the new moon’s arms’.

Q 28.C
• Recent Context-Delegates of the G20 summit visited Hemis and Thiksey Monasteries.
• Hemis Monastery is the largest monastic institution in Ladakh. It is situated on the western banks
of the Indus River. Hence statement 1 is correct.
• It belongs to the Dragon Order of Mahayana Buddhism (or the Drupka Lineage). Hence statement
2 is not correct.
• Hemis has more than 200 branch monasteries in the Himalayas.
• The monastery was first established in the 11th century and later re-established in the 17th century by the
Ladakhi King Sengge Namgyal.
• It is also famous for its 2-day religious ceremony known as the Hemis Festival. This festival is
celebrated in honour of Padmasambhava which is observed here in Hemis Monastery. It is the
biggest and very richly endowed monastery. Hence statement 3 is correct.

Q 29.D
• On September 22, 2006, the Supreme Court passed a judgment in a Writ Petition filed by former DGP
Prakash Singh and others on several issues concerning police reforms.
• The main directions issued by the Supreme Court are:
o Constitute a State Security Commission on any of the models recommended by the National
Human Right Commission, the Ribeiro Committee or the Sorabjee Committee.
o Select the DGP of the state from amongst three senior-most officers of the department impaneled for
promotion to that rank by the UPSC and once selected, provide him a minimum tenure of at least two
years irrespective of his date of superannuation.
o Prescribe a minimum tenure of two years to the police officers on operational duties.
o Separate investigating police from law & order police, starting with towns/urban areas having a
population of ten lakhs or more, and gradually extending to smaller towns/urban areas also.
o Set up a Police Establishment Board at the state level for inter alia deciding all transfers,
postings, promotions and other service-related matters of officers of and below the rank of
Deputy Superintendent of Police.
o Constitute Police Complaints Authorities at the state and district level for looking into
complaints against police officers.
o The Supreme Court also directed the central government to set up a National Security Commission at
the Union Level to prepare a panel for being placed before the appropriate Appointing Authority, for
the selection and placement of chiefs of the Central Police Organisations (CPOs), who should also be
11 www.visionias.in ©Vision IAS

FREE BY KING R QUEEN P [ऋषभ राजपूत]


given a minimum tenure of two years, with additional mandate to review from time to time measures
to upgrade the effectiveness of these forces, improve the service conditions of its personnel, ensure
that there is proper coordination between them and that the forces are generally utilized for the
purposes they were raised and make recommendations in that behalf.
• Therefore, all four form a part of the judgement.
• In July 2018, the Supreme Court clarified that:
o All states shall send their proposals in anticipation of the vacancies to the UPSC, well in time at least
three months prior to the date of retirement of the incumbent on the post of Director General of
Police.
o The state shall immediately appoint one of the persons from the panel prepared by the UPSC.
o None of the states shall ever conceive of the idea of appointing any person on the post of DGP on
acting basis for there is no concept of acting Director General of Police.
• Hence, option (d) is the correct answer.

Q 30.A
• In India, general elections are held on the basis of the Universal Adult Franchise. Herein, all adult
citizens of India are allowed to participate. E.g., Lok Sabha Elections or State Assembly elections. For
such general elections, the First Past the Post system is followed. Hence Statement II is correct.
• In this kind of voting system, the candidate with the most votes in a constituency is declared
elected. Hence Statement I is correct.
• Further, since India follows the FPTP system, the candidate securing the most votes in a general
election is declared elected. Hence, Statement II is the correct explanation of Statement I.

Q 31.C
• Global Slavery Index 2023
o Context: G20 countries including India are fuelling modern slavery, says a new report. The Global
Slavery Index 2023 shows the world’s 20 richest countries account for more than half the estimated
50 million people living in modern slavery.
o Among the G20 nations, India tops the list with 11 million people working as forced laborers,
followed by China, Russia, Indonesia, Turkey, and the U.S.
o The Index is published by the Australia-based human Rights Group Walk Free Foundation. Hence
option (c) is the correct answer.
o It includes data on three key variables:
✓ Prevalence of modern slavery in each country
✓ Vulnerability
✓ Government responses to modern slavery
o NOTE: Modern slavery refers to situations of exploitation that a person cannot leave because of
threats, violence, coercion, and abuse of power or deception.

Q 32.D
• State List - 7th Schedule
o Entry 32 - Corporation, other than those specified in List I, and universities; unincorporated
trading, literacy, scientific, religious, and other societies, and associations; co-operative societies.
Hence, statement II is correct.
• The 97th Amendment Act was challenged in several High Courts across the country. The Gujarat High
Court was pleased to rule that the aforementioned Act was unconstitutional in Rajendra N. Shah v. Union
of India. The petitioner in the aforementioned case, among other things, contested the constitutionality of
the 97th Amendment because ‘Co-operative Societies’ are solely a matter for state legislatures to pass
laws. In other words, only the State Legislature has the power to pass legislation governing Cooperative
societies. The petitioner based his argument on Entry 32 in List II of Schedule VII.
• With Parliament having control over Multi-State Co-operative Societies and State legislatures responsible
for enacting laws governing “other Co-operative Societies,” the scheme governing Multi-State Co-
operative Societies differs from the scheme governing “Other Co-operative Societies.” Thus in the case of
multi-state cooperatives (with aims and objectives not confined to one state), the Centre or the Union can
make laws. Hence Statement I is not correct.
• There is no question, in the Court’s opinion, that Multi-State Co-operative Societies would not be covered
by Article 246(3) or Entry 32 List 2 of the 7th Schedule which is specific to only Co-operative Societies
and not Multi-State Co-operative Societies.

12 www.visionias.in ©Vision IAS

FREE BY KING R QUEEN P [ऋषभ राजपूत]


Q 33.A
• Mahatma Gandhi National Rural Employment Guarantee Act (MGNREGA):
o MGNREGA is one of the largest work guarantee programmes in the world launched in 2005 by
the Ministry of Rural Development.
o The primary objective of MGNREGS is to provide 100 days of unskilled work per year for every
rural household that wants it, covering all districts in the country except those with a 100%
urban population. Hence statement 1 is correct and statement 2 is not correct.
• Demand-Driven Scheme:
o If work is not provided within 15 days from when it is demanded, the worker has to be given a
daily unemployment allowance. Hence statement 3 is not correct.
o Additionally, the wages of unskilled workers also have to be paid within 15 days and in case of a
delay, the Centre has to compensate them.
o Wages must be paid according to the statutory minimum wages specified for agricultural labourers in
the state under the Minimum Wages Act, 1948.
o At least one-third of beneficiaries have to be women.
o The act mandates Gram sabhas to recommend the works that are to be undertaken and at least 50% of
the works must be executed by them.

Q 34.D
• The Supreme Court stayed the Election Commission’s order of revoking the star campaigner status of a
person in the recently concluded Madhya Pradesh state assembly elections, observing the poll body “has
no power” to decide who is a star campaigner. Hence statement 1 is not correct.
• A recognized political party can have forty (40) star campaigners and an unrecognized (but registered)
political party can have 20. As per the revised guidelines due to the pandemic, the maximum limit on the
number of star campaigners for recognized National/State political parties shall be 30 in place of 40, and
for unrecognized registered political parties it shall be 15 in place of 20 during the period of the
pandemic.
• The list of the star campaigners has to be communicated to the Chief Electoral Officer and Election
Commission within a week from the date of notification of an election. As per the revised guidelines due
to the pandemic, the period of submission of the list of star campaigners is extended from 7 days to 10
days from the date of notification. Political parties, which have already submitted a list of star
campaigners shall resubmit a revised list within the stipulated period. Hence, statement 2 is not correct.

Q 35.D
• Forest Rights Act, 2006 :
o The act recognize and vest the forest rights and occupation in Forest land in forest Dwelling
Scheduled Tribes (FDST) and Other Traditional Forest Dwellers (OTFD)who have been residing in
such forests for generations.
o It seeks to rectify colonial injustice to the FDST and OTFD who are integral to the very survival and
sustainability of the forest ecosystem.
• The act identifies four types of rights:
o Title rights
✓ It gives recognised scheduled tribes and Other Traditional Forest Dwellers the right to
ownership to land farmed by tribals or forest dwellers subject to a maximum of 4 hectares.
Hence statement 1 is not correct.
✓ Ownership is only for land that is actually being cultivated by the concerned family and no new
lands will be granted.
o Use rights:
✓ The rights of the dwellers extend to extracting Minor Forest Produce, grazing areas, to pastoralist
routes, etc.
o Relief and development rights:
✓ The rehabilitation rights in case of illegal eviction or forced displacement and to provision of
basic amenities. The rights subject to restrictions for forest protection.
o Forest management rights:
✓ It includes the right to protect, regenerate or conserve or manage any community forest resource
which they have been traditionally protecting and conserving for sustainable use.
• Who can claim these Rights?
o Members or community of the Scheduled Tribes who primarily reside in and who depend on the
forests or forest lands for bona fide livelihood needs.
13 www.visionias.in ©Vision IAS

FREE BY KING R QUEEN P [ऋषभ राजपूत]


o It can also be claimed by any member or community who has for at least three generations (75
years) prior to the 13th day of December, 2005 primarily resided in forests land for bona fide
livelihood needs. Hence statement 2 is not correct.
• The Gram Sabha (not Gram Panchayat) is the authority to initiate the process for determining the
nature and extent of Individual Forest Rights (IFR) or Community Forest Rights (CFR) or both that may
be given to forest Dwelling Scheduled Tribes (FDST) and Other Traditional Forest Dwellers
(OTFD). Hence statement 3 is not correct.
o The Gram Panchayat consists of ward members and Sarpanch.
o The Gram Sabha consists of every person whose name is registered in the voter list of the village.
o The members of the Gram Panchayat are directly elected by the members of the Gram Sabha.
o The members of Gram Sabha are not elected.

Q 36.B
• Recently, Election Commission (EC) proposed to permit Non-Resident Indians (NRIs) to cast their votes
from overseas through postal ballots.
• EC informed the government that it is ready to extend the Electronically Transmitted Postal Ballot System
(ETPBS) to voters abroad for elections in 2021 in Assam, West Bengal, Kerala, Tamil Nadu, and
Puducherry.
• To extend ETPBS to overseas voters, the government only needs to amend the Conduct of Election Rules
1961. It doesn’t require Parliament’s nod.
• Electronically Transmitted Postal Ballot System
o It is a type of voting whereby Electronically Transmitted Postal Ballot Papers (ETPB) are distributed
to electors and returned by post.
o Under ETPBS, the postal ballot is dispatched electronically and returned via ordinary mail and
it is currently only available to service voters like
✓ members of the Armed Forces
✓ people employed by the government outside India
✓ people under preventive detention etc.
• The government has not yet given nod to this proposal by Election Commission.
• But the procedure of voting if granted approval is as follows.
o Any NRI interested in voting through the postal ballot will have to inform the Returning Officer (RO)
not later than five days after the notification of the election.
o On receiving such information, the RO will dispatch the ballot paper electronically.
o NRI voters will mark their preference on the printout and send it back along with a declaration
attested by an officer appointed by the diplomatic or consular representative of India in the country
where the NRI is resident.
• NRIs or an Overseas Elector is “a person who is a citizen of India, absent from the country owing to
employment, education etc, has not acquired citizenship of any other country and are otherwise eligible to
be registered as a voter in the address mentioned in your passport.
• According to estimates of the Ministry of External Affairs, there are about 3.10 crore NRIs living in
different countries across the world. In the last Lok Sabha elections, roughly 25,000 of them flew to India
to vote.
• Hence, option (b) is the correct answer.

Q 37.C
• Context: The Supreme Court has ruled that an accused person’s fundamental right (under Article 21) to
receive default bail cannot be violated by probe agencies by filing supplementary charge sheets in cases
where the investigation is yet to be completed. The violation of such a right directly attracts consideration
under Article 32 of the Constitution.
• Bail is the release of an accused person from custody, on the undertaking that they will appear in court for
their trial.
• Legal Basis-Bail in India is governed by the Code of Criminal Procedure (CrPC), which provides for the
granting of bail by police and courts.
• Statutory Bail- Statutory bail is a right to bail that accrues when police fail to complete the
investigation within a specified period in respect of a person in judicial custody. It is enshrined in
the CrPC and is available for most offences. Hence statenebt 1 is correct.
• The time limit for statutory bail-60 days to complete the investigation and file a final report (in most
cases). 90- or 180-day limit for some cases.

14 www.visionias.in ©Vision IAS

FREE BY KING R QUEEN P [ऋषभ राजपूत]


• Regular Bail-Regular bail is granted to an accused person who is in custody and is usually granted
on the basis of surety or personal bond. Hence statenebt 2 is correct.
• Anticipatory Bail- Anticipatory bail is granted before arrest and is meant to protect an accused person
from arrest.
• Conditions for Bail- Bail may be granted with conditions, such as surrendering of passport, attending
court hearings regularly, not contacting witnesses, etc.
• Eligibility- Bail eligibility depends on several factors, including the nature of the crime, severity of the
offence, likelihood of fleeing from justice, past criminal record, and the strength of evidence against the
accused.

Q 38.D
• The National Commission to Review the Working of the Constitution (NCRWC) was set up by a
resolution of the Government of India in 2000. The 11-member Commission was headed by M.N.
Venkatachaliah, the former Chief Justice of India. It submitted its report in 2002. Hence, statement 1 is
not correct.
• Even before the appointment of this Commission in 2000, certain attempts were already made to
review the working of the Constitution. The commission itself summarized the previous attempts
such as a total of eighty-five amendments till the formation of the commission, the living and
organic nature of the constitution, various commissions to review the center-state relations, etc. A
document titled ‘A Fresh Look at Our Constitution - Some Suggestions’ surfaced at the time of the 1975
All India Congress committee session and was circulated but, after the recommendations contained in it
had drawn stringent criticism from diverse quarters, it was not pursued. Hence, statement 2 is not
correct.
• According to the terms of reference, the commission was required to examine, in the light of the
experience of the past fifty years, as to how far the existing provisions of the Constitution are capable of
responding to the needs of an efficient, smooth and effective system of governance and socio-economic
development of modern India and to recommend changes, if any. The terms of reference clearly
specified that the commission should recommend changes that are required to be made in the
Constitution within the framework of parliamentary democracy and without interfering with the
‘basic structure’ or ‘basic features’ of the Constitution.

Q 39.B
• The Eighth Schedule to the Constitution of India lists the official languages of India. Although there are
hundreds of languages spoken across the country, the eighth schedule recognizes a total of 22 languages
as the official languages. While 14 of these languages were included in the Constitution at first, the rest of
the languages were added to the Constitution through subsequent amendments.
• Sindhi was added by the 21st Amendment Act of 1967; Konkani, Manipuri and Nepali were added
by the 71st Amendment Act of 1992; and Bodo, Dongri, Maithili, and Santhali were added by the
92nd Amendment Act of 2003. 42nd Amendment did not add any new official language. Hence,
option (b) is the correct answer.

Q 40.C
• Chiral Bose-liquid State
• Context: Recent research shows that the chiral bose-liquid state may be an entirely new state of matter.
o States of matter: There are 3 classical states of matter in day-to-day life- solid, liquid, and gas. But,
exotic or quantum states of matter, such as plasma, time crystals, and Bose-Einstein
condensate also exist at the atomic scale, and at extremely low temperatures approaching absolute
zero.
o Chiral Bose-liquid state:
✓ The chiral Bose-liquid state is a new phase discovered by physicists in a frustrated quantum
system, where infinite possibilities result from the interaction of particles.
✓ The team developed a bilayer semiconductor device or a frustration.
✓ The top layer of the device was made to be electron-rich, in which the electrons can freely move
about.
✓ The bottom layer only had holes, which are slots an electron can occupy.
✓ Then the two layers are brought extremely close together.
✓ The machine is then triggered to create a local imbalance resulting in electrons not having enough
holes to fill (similar to the game of musical chairs).
✓ This kicks off the novel state called the chiral bose-liquid state.
15 www.visionias.in ©Vision IAS

FREE BY KING R QUEEN P [ऋषभ राजपूत]


o Significance: In this state,
✓ Electrons can be frozen into predictable patterns. Hence statement 2 is correct.
✓ Electrons can be made resilient to changes in spin (a defining characteristic of subatomic
particles)
✓ Electrons can even synchronize their movements. Hence statement 1 is correct.

Q 41.C
• Social Audit is the examination and assessment of a programme/scheme conducted with the active
involvement of people and comparing official records with actual ground realities. Social Audit is a
powerful tool for social transformation, community participation and government accountability. Hence,
statement 1 is correct.
• The Mahatma Gandhi National Rural Employment Guarantee Act (MGNREGA) was the first law
to mandate social audit as a statutory requirement. MGNREGA Audit of Schemes Rules was passed
by the Central Government, in consultation with the Comptroller and Auditor General (C&AG) in
2011. Under MGNREGA scheme rules, apart from the ongoing process of social audit, it has been
mandated to convene Social Audit Forum once every six months, which will serve as an institutional
forum where people can conduct details public audits of all MGNREGA works that have been
carried out in their area in the preceding six months. Hence statement 2 is correct.
• Section 17 of the MGNREGA has mandated a Social audit of all Works executed under the
MGNREGA. Social Audit is different from Financial Audit. Financial audits involve inspecting and
assessing documents related to financial transactions in an organization to provide a true picture of its
profits, losses and financial stability. Social audits focus on the performance of a programme in fulfilling
its intended social objectives and ethical vision through consultation with a range of stakeholders
including social programme beneficiaries, community members, and government officials and verifying
the information obtained with documents and physical evidence. Thus social audits examine and assess
the social impact of specific programmes and policies.
• The process of Social Audit combines people's participation and monitoring with the requirements of the
audit discipline. It is necessary to promote people's participation in the audit along with support provided
by an independent social audit organization that facilitates the process.
• The Audit of Scheme Rules, 2011 were prepared by MoRD in consultation with the Comptroller and
Auditor General (CAG) of India. These rules are also called the Mahatma Gandhi National Rural
Employment Guarantee Audit of Schemes Rules, 2011. These rules define the process of social audit and
the responsibilities of the Social Audit Unit (SAU), state government and the field functionaries of
MGNREGA, to be followed across the country.
• These rules also emphasize the role of the SAU, its prerequisites, the process of social audit and the
responsibilities of designated officials.
• Rule 4 of Audit of Schemes Rules, 2011 stipulates that each State Government shall identify or
establish an independent organization, “Social Audit Unit” (SAU) to facilitate the conduct of the
Social Audit of MGNREGS works.

Q 42.B
• The United Nations Department of Economic and Social Affairs (UN DESA) has been publishing the
EGDI and survey report since 2001 biennially.
• The Survey is the only global report that assesses the e-government development status of all United
Nations Member States. The assessment measures the e-government performance of countries relative to
one another, as opposed to being an absolute measurement.
• India ranked 105 in the E-Government Development Index and 61 in E-Participation Index, down
from the 2020 rank of 100 and 29 respectively. Denmark ranked 1 in the E-Government Development
Index while Japan ranked 1 in E-Participation Index.
• Hence, option (b) is the correct answer.

Q 43.B
• Lightweight Payment and Settlement System (LPSS)
• Context: RBI is planning to use the lightweight payment system for emergencies and as an alternative to
UPI, NEFT, and RTGS.
o Details
✓ The proposed Lightweight and Portable Payment System (LPSS) will be independent of
conventional technologies and can be operated from anywhere by a bare minimum staff,
according to the central bank.
16 www.visionias.in ©Vision IAS

FREE BY KING R QUEEN P [ऋषभ राजपूत]


✓ It can be used for critical transactions during catastrophic events like natural calamities and war.
o Need for the lightweight payment system
✓ Existing conventional payment systems like RTGS (Real Time Gross Settlement), NEFT
(National Electronic Funds Transfer), and UPI (Unified Payments Interface) are designed to
handle large volumes while ensuring sustained availability. These systems are dependent on
complex wired networks backed by advanced IT infrastructure.
✓ However, catastrophic events like natural calamities and war have the potential to render
these payment systems temporarily unavailable by disrupting the underlying information and
communication infrastructure. Hence statement 2 is correct
✓ Therefore, it is prudent to be prepared to face such extreme and volatile situations.
✓ Keeping this objective in mind, RBI has conceptualized the LPSS that will be independent of
conventional technologies and can be operated from anywhere by a bare minimum of staff.
o Features of LPSS
✓ It will operate on minimalistic hardware and software and would be made active only on a
need basis. Hence statement 1 is not correct.
✓ The system can ensure near-zero downtime of the payment and settlement system in the
country and keep the liquidity pipeline of the economy intact by facilitating the uninterrupted
functioning of essential payment services like bulk payments, interbank payments, and provision
of cash to participant institutions. Hence statement 3 is correct.
Q 44.D
• Fish Kill
o Context: Recently thousands of dead fish washed up on multiple beaches in southeast Texas, USA
due to a phenomenon called, “fish kill”.
o The fish kill is the sudden and unexpected death of many fish or other aquatic animals over a short
period and mostly within a particular area.
o Causes of fish kill
✓ Low levels of dissolved oxygen: When sea surface temperatures rise, it becomes difficult for the
fish to breathe as oxygen dissolves easily in colder water compared to warmer water.
✓ When many fishes get trapped in shallow water, they get warmer more quickly, leading to
suffocation.
✓ This causes fish to act more erratically, which in turn, further depletes the oxygen from the water,
ultimately leading to the death of the fish.
✓ Calm Seas: Oxygen enters the seawater by mixing with wind and waves. Very calm seas cause
the depletion of dissolved oxygen and cause fish to kill.
✓ Overcast weather: It refers to more than 95% of cloud coverage. Photosynthesis is driven by
sunlight and it slows down on cloudy days, resulting in decreased dissolved oxygen
concentration. Hence option (d) is the correct answer.
✓ Other reasons: Alteration in natural water chemistry, biological changes, rising temperatures of
oceans, chemical pollution, or miscellaneous human activity.

Q 45.C
• Tobacco Cultivation
o Context: Farmers’ body questions WHO recommendations on substituting tobacco cultivation with
alternative crops.
o Nicotiana tabacum and N. rustica are the two most commonly cultivated for producing commercial
tobacco.
o Origin: The primary center of origin of N. tabacum is South America and that of N. rustica is Peru. In
India, it is introduced by the Portuguese in the 17th century.
o Area and production: N. tabacum is widely cultivated in most countries of the world while N.
Rustica is restricted to India, Russia, and a few other Asiatic countries.
o Top production and productivity in India: Gujarat.
o Other producers: Andhra Pradesh followed by Gujarat, Karnataka, UP, Bihar, etc.
o Climate: Mean temperature - 20° to 27°C.
o Rainfall: Not exceeding 1200 mm during the season.
o It is tropical in origin but successfully grown in temperate also. Hence statement 1 is correct.
o It is sensitive to waterlogging. Hence statement 2 is correct.
o Curing: It is a carefully controlled process used to achieve the texture, color, and overall quality of a
specific tobacco type. Hence statement 3 is correct.
o China is the largest tobacco producer in the world followed by India.
17 www.visionias.in ©Vision IAS

FREE BY KING R QUEEN P [ऋषभ राजपूत]


o Tobacco Board
✓ It was constituted as a statutory body under the Tobacco Board Act, of 1975.
✓ Headquarters: Guntur, Andhra Pradesh.
✓ Ministry: Ministry of Commerce and Industry.
o WHO Framework Convention on Tobacco Control (FCTC)
✓ It is the first global public health treaty on tobacco.
✓ It is an evidence-based treaty that reaffirms the right of all people to the highest standard of
health.
✓ India is a party to the convention.

Q 46.B
• The Comptroller and Auditor-General of India (CAG) is an independent authority under the
Constitution of India and is the head of the Indian audit & account department and chief Guardian of
the Public purse.
• It is the institution through which the accountability of the government and other public authorities (all
those who spend public funds) to Parliament and State Legislatures and through them to the people is
ensured.
• CAG is appointed by the President by warrant under his hand and seal and provided with tenure of
6 years or 65 years of age, whichever is earlier.
• He can be removed by the President only in accordance with the procedure mentioned in the
Constitution which is the manner same as the removal of a Supreme Court Judge.
• He is ineligible to hold any office, either under the Government of India or of any state, once he
retires/resigns as a CAG and no minister can represent the CAG in Parliament. Hence statement 1 is not
correct and statement 2 is correct.

Q 47.D
• The Electronic Voting Machine (EVM), the replacement of the ballot box is mainstay in the electoral
process. First conceived in 1977 in the Election Commission.
o First time use of EVMs occurred in the general election in Kerala in May, 1982; however, the
absence of a specific law prescribing its use led to the Supreme Court striking down that
election. Hence statement 3 is correct.
o Subsequently, in 1989, the Parliament amended the Representation of the People Act, 1951 to create a
provision for the use of EVMs in the elections (chapter 3). A general consensus on its introduction
could be reached only in 1998 and these were used in 25 Legislative Assembly constituencies spread
across three states of Madhya Pradesh, Rajasthan and Delhi.
• Voter Verifiable Paper Audit Trail (VVPAT) is an independent system attached with the Electronic
Voting Machines that allows the voters to verify that their votes are cast as intended. When a vote is cast,
a slip is printed on the VVPAT printer containing the serial number, name and symbol of the candidate
and remains exposed through a transparent window for 7 seconds. Thereafter, this printed slip
automatically gets cut and falls in sealed drop box of the VVPAT.
o In the 2014 Lok Sabha polls, VVPATs were used in eight constituencies and in the 2019 Lok Sabha
polls, VVPATs were used in all the constituencies. Hence statement 2 is correct.
• NOTA, or "None of the Above", is the option which enables the voter to officially register a vote of
rejection for all candidates who are contesting. If a voter chooses to press NOTA it indicates that the voter
has not chosen to vote for any of the party.
• On 27 September 2013, the Supreme court of India ruled that the right to register a "none of the above"
vote in elections should apply, while ordering the Election Commission to provide a button for the same
in the electronic voting machines.
• The ECI introduced a particular symbol for ‘None of the Above’ option to allow the voters to exercise
NOTA. This symbol appears in the last panel on all Electronic Voting Machines (EVMs).
• In Lok Sabha elections, the None of the Above or NOTA option was first used in 2014. Hence statement
1 is correct.

Q 48.A
• East and Horn of Africa and the Great Lakes (EHAGL) region
• Horn of Africa
o It is a region in eastern Africa.
o Countries included: Djibouti, Eritrea, Ethiopia, and Somalia.
o Coastlines included: the Red Sea, the Gulf of Aden, and the Indian Ocean.
18 www.visionias.in ©Vision IAS

FREE BY KING R QUEEN P [ऋषभ राजपूत]


o Rivers flowing in the region: Blue Nile, White Nile, and Dawa River.
o Lakes present in the region: Lake Tana, Lake Turkana.
o Ogaden desert: It is situated between the Somalia-Ethiopia border and the Ethiopian Eastern
Highlands.
o The Horn of Africa, almost equidistant from the Equator and the Tropic of Cancer is an arid region.
• Great Lakes Region
o The African Great Lakes are a series of lakes constituting part of the Rift Valley lakes in and around
the East African Rift.
o They include Lake Victoria, the 2nd largest freshwater lake in the world in terms of surface area.
o Lake Tanganyika is the world's second-largest in volume as well as the second deepest.
o Lists of the African Great Lakes - Lake Victoria, Lake Tanganyika, Lake Malawi, Lake Turkana,
Lake Albert, Lake Kivu, and Lake Edward.
o The 4 countries that make up the Great Lakes region are the Democratic Republic of the Congo
(D.R.C.), Burundi, Rwanda, and Uganda.Hence option (a) is the correct answer.

Q 49.B
• The 2021 amendment to the Medical Termination of Pregnancy Act(MTPA), 1971:
o The 2021 amendment to the MTP Act extends the time limit for termination of pregnancy from
20 to 24 weeks. This extension is provided only to certain categories of women as prescribed under
the Rules. Hence statement 1 is not correct.
o The 2021 Rules list out the women eligible for termination of pregnancy up to 24 weeks and this
includes
✓ women who are survivors of sexual assault/rape/incest,
✓ minors, women with disabilities,
✓ women withsubstantial foetal abnormalities,
✓ women in disaster or emergencies and women who have had a change of marital status during the
pregnancy (widowhood and divorce).
o Thus, technically, single or unmarried women would not get the benefit of the extension of the 24-
week window for abortion under the act.
o But,by giving a broad and purposive interpretation to Rule 3B, the Supreme Court in its judgement
held that the Rule could not be limited only to married women. Hence now all women
irrespective of marriage can avail abortion rights up to 24 weeks of pregnancy.

• Marital rape as a condition for abortion up to 24 weeks:


o The Supreme Court has addressed the sexual violence within marriage and recognised marital rape
as a condition for abortion up to 24 weeks. Hence statement 2 is correct.
o Since the 2021 Rules allowing abortions up to 24 weeks include women who are survivors of sexual
assault or rape, the Supreme Court held that for this purpose, it would include marital rape.
o While this would not amount to recognition of marital rape or the removal of the exception under the
Indian Penal Code, that it is only a matter of time till our courts strike down the marital rape
exception. This judgment takes some steps towards that goal.

Q 50.B
• Originally, the Constitution of India did not make any provision with respect to the Special Officer
for Linguistic Minorities. Hence statement 1 is correct.
• Later, the States Reorganisation Commission (1953–55) made a recommendation in this regard.
• Accordingly, the Seventh Constitutional Amendment Act of 1956 inserted a new Article 350- B in
Part XVII of the Constitution.
• The Constitution does not specify the qualifications, tenure, salaries and allowances, service
conditions, and procedure for removal of the Special Officer for Linguistic Minorities. Hence
statement 2 is correct.
• In pursuance of the provision of Article 350-B of the Constitution, the office of the Special Officer for
Linguistic Minorities was created in 1957. He is designated as the Commissioner for Linguistic
Minorities.
• At the Central level, the Commissioner falls under the Ministry of Minority Affairs. Hence, he
submits the annual reports or other reports to the President through the Union Minority Affairs Minister.
Hence statement 3 is not correct.

19 www.visionias.in ©Vision IAS

FREE BY KING R QUEEN P [ऋषभ राजपूत]


Q 51.A
• Right to Fair Compensation and Transparency in Land Acquisition, Rehabilitation, and
Resettlement Act, 2013:
o The provisions of this Act relating to land acquisition, consent, compensation, rehabilitation and
resettlement, shall also apply, when the appropriate Government acquires land for the following
purposes, namely:
✓ for public private partnership projects, where the ownership of the land continues to vest with the
Government,
✓ for private companies for public purpose.
• Provided that in the case of acquisition for—
o private companies, the prior consent of at least eighty per cent, of those affected families.
o public private partnership projects, the prior consent of at least seventy per cent. of those affected
families. Hence statement 1 is not correct.
• Preparation of Social Impact Assessment study:
o Whenever the appropriate Government intends to acquire land for a public purpose, it shall consult
the concerned Panchayat, Municipality or Municipal Corporation, as the case may be, at village level
or ward level, in the affected area and carry out a Social Impact Assessment study in consultation with
them, in such manner and from such date as may be specified by such Government by notification.
• Exemption from Social Impact Assessment:
o Where land is proposed to be acquired invoking the urgency provisions under section 40, the
appropriate Government may exempt undertaking of the Social Impact Assessment study.
Hence statement 2 is not correct.
• Acquisition and requisitioning of property under Concurrent list.
o Though land is a state subject, "acquisition and requisitioning of property" is in the concurrent
list. Hence state governments can amend the act with the assent from president. Hence
statement 3 is correct.
o Recently, the Odisha government has proposed to amend the Right to Fair Compensation and
Transparency in Land Acquisition, Rehabilitation and Resettlement Act, 2013 to ease land acquisition
process for various industrial and infrastructure projects.
o Odisha will be the fourth state to amend the legislation as Gujarat (2016), Maharashtra (2018) and
Karnataka (2019) have already made the amendment and received the assent of the President of India.
Q 52.C
• How is the CAG in India Different from the CAG in Britain?
o CAG of India is only performing the role of an Auditor General and not of a Comptroller but in
Britain it has the power of both Comptroller as well as Auditor General.
o In India, the CAG audits the accounts after the expenditure is committed i.e., ex post facto. In the UK
no money can be drawn from the public exchequer without the approval of the CAG. Hence
statement 1 is correct.
o In India, CAG is not a member of the parliament while in Britain; CAG is a member of house of the
Commons. Hence statement 2 is correct.
Q 53.D
• In India’s developmental plan exercise, we have two types of schemes viz; central sector and centrally
sponsored scheme. The nomenclature is derived from the pattern of funding and the modality for
implementation.
• While the Union government fully funds the central sector schemes, centrally sponsored schemes are
jointly funded by the Centre and states.
• Under Central sector schemes, it is 100% funded by the Union government and implemented by the
Central Government machinery. Hence statement 1 is not correct.
• Central sector schemes are mainly formulated on subjects from the Union List. Hence statement 2 is not
correct.
• In addition, the Central Ministries also implement some schemes directly in States/UTs which are called
Central Sector Schemes but resources under these Schemes are not generally transferred to States.

Q 54.C
• Voter verifiable paper audit trail (VVPAT) or verified paper record (VPR) is a method of providing
feedback to voters using a ballotless voting system.
o A VVPAT is intended as an independent verification system for voting machines designed to allow
voters to verify that their vote was cast correctly, to detect possible election fraud or malfunction, and
to provide a means to audit the stored electronic results.
20 www.visionias.in ©Vision IAS

FREE BY KING R QUEEN P [ऋषभ राजपूत]


o It contains the name of the candidate (for whom the vote has been cast) and the symbol of the
party/individual candidate.
o While it has gained in use in the United States compared with ballotless voting systems without it, it
looks unlikely to overtake hand-marked ballots.
o Voter-verifiable paper audit trail was first used in an election in India in September 2013 in
Noksen (Assembly Constituency) in Nagaland.
• An electronic voting machine is a voting machine based on electronics.
o First-time use of EVMs occurred in the general election in Kerala in May 1982; however, the absence
of a specific law prescribing its use led to the Supreme Court striking down that election.
o Subsequently, in 1989, the Parliament amended the Representation of the People Act, of 1951 to
create a provision for the use of EVMs in the elections. A consensus on its introduction could be
reached only in 1998 and these were used in 25 Legislative Assembly constituencies spread
across three states of Madhya Pradesh, Rajasthan, and Delhi.
• The Sixty-first Amendment of the Constitution of India, officially known as The Constitution (Sixty-
first Amendment) Act, 1988, lowered the voting age of elections to the Lok Sabha and the Legislative
Assemblies of States from 21 years to 18 years.
• The existence of the Code dates back to the 1960 Kerala State Legislative Assembly Elections when the
State Administration for the first time, issued a set of instructions, regulating the conduct of the
participating political institutions. The Code was then taken into national cognizance during the 1962 Lok
Sabha Elections and the State Legislative Assembly elections. It was widely circulated to all the
recognized states and national political parties. As a result, the Code was voluntarily embraced and
accepted in furtherance of securing a free and fair elections regime.
• Hence, option (c) is the correct answer.

Q 55.B
• The National Commission to Review the Working of the Constitution, in al, made 249 recommendations.
Of them, 58 recommendations involve amendments to the Constitution, 86 involve legislative measures
and the remaining 105 recommendations could be accomplished through executive action. Some of the
important recommendations are given below:
• On Fundamental Rights:
o The scope of the prohibition against discrimination (under Articles 15 and 16) should be extended to
include ‘ethnic or social origin, political or other opinions, property or birth’.
o The freedom of speech and expression (under Article 19) should be expanded to include explicitly
‘the freedom of the press and other media, the freedom to hold opinions and to seek, receive and
impart information and ideas’.
o The protection from judicial review afforded by Article 31-B to the Acts and Regulations
specified in the Ninth Schedule should be restricted to only those which relate to (i) agrarian
reforms, (ii) reservations, and (iii) the implementation of Directive Principles specified in clause
(b) or (c) of Article 39. Thus option 1 is not a recommendation.
• On Directive Principles
o The heading of Part-IV of the Constitution should be amended to read as ‘Directive Principles of State
Policy and Action
o A new Directive Principle on control of the population should be added. Thus option 2 was a
recommendation.
o An independent National Education Commission should be set up every five years.
• On Fundamental Duties
o The recommendations of the Justice Verma Committee on the operationalization of Fundamental
Duties should be implemented at the earliest.
o Duty to vote at elections, actively participate in the democratic process of governance, and pay
taxes. Thus option 3 was a recommendation.
o To foster a spirit of family values and responsible parenthood in the matter of education, physical and
moral well-being of children
• On Parliament and State Legislatures
o The privileges of legislators should be defined and delimited for the free and independent functioning
of Parliament and state legislatures.
o The domiciliary requirement for eligibility to contest elections to Rajya Sabha from the
concerned state should be maintained. This is essential to ensure the federal character of the
Rajya Sabha. Thus option 4 is not a recommendation.

21 www.visionias.in ©Vision IAS

FREE BY KING R QUEEN P [ऋषभ राजपूत]


o The Election Commission should be empowered to identify and declare the various offices under the
central and state governments to be ‘offices of profit’ for the purposes of being chosen, and for being,
a member of the appropriate legislature. Hence, option (b) is the correct answer.
Q 56.C
• Article 19 (1) of the Indian Constitution guarantees us the right to freedom of speech and expression and
as recognized by the Supreme Court this also implies a full right to information. The Right to Information
Act, 2005 (“the Act”) has established the necessary practical regime of the right to information. Right to
information can empower citizens to take charge by participating in decision-making and by challenging
corrupt and arbitrary actions at all levels. With access to government records, citizens can evaluate and
determine whether the government they have elected is delivering the results that are expected.
• Application should be submitted in English, Hindi or in the official language of the area in which
the application is made. Hence, statement 1 is not correct.
• In the normal course, information to an applicant is to be supplied within 30 days from the receipt of the
application by the public authority. Hence, statement 3 is not correct.
• If the information sought concerns the life or liberty of a person, it shall be supplied within 48 hours.
• In case the application is sent through the Assistant Public Information Officer or it is sent to the wrong
public authority, five days shall be added to the period of thirty days or 48 hours, as the case may be.
• Application can be handwritten or typed. Hence, statement 2 is correct.

Q 57.B
• The National Assessment and Accreditation Council (NAAC), is an autonomous body under the
University Grants Commission (UGC) responsible for assessing the quality of higher educational
institutions in India. It was established in 1994 and awards grades ranging from A++ to C based on
parameters such as curriculum, faculty, infrastructure, research, and financial well-being. Hence
statement 1 is correct.
• Accreditation by NAAC is voluntary, and only institutes that are at least 6 years old or from where at
least two batches of students have graduated can apply for accreditation, which is valid for 5 years.
Hence statement 2 is not correct and 3 is correct.
• Efforts to expedite accreditation include the UGC launching a scheme named 'Paramarsh' in 2019
to mentor institutes aspiring to get accredited, NAAC exploring the possibility of issuing Provisional
Accreditation for Colleges (PAC) to one-year-old institutes, and the National Education Policy (2020)
setting an ambitious target of getting all higher educational institutes to obtain the highest level of
accreditation over the next 15 years.
• However, India's higher education system faces several challenges, including limited access, gender
inequality, and employability issues. The use of digital technology can help make education more
accessible, cost-effective, and efficient.

Q 58.B
• Foreign Contribution Regulation Act regulates the acceptance and utilisation of foreign contribution
by individuals, associations and companies. Foreign contribution is the donation or transfer of any
currency, security or article (of beyond a specified value) by a foreign source.
• Under the Act, any person/entity/NGO receiving foreign contribution must use it only for the purpose for
which the contribution is received. Further, they must not use more than 20% (amended in 2020) of the
contribution for meeting administrative expenses. Hence, statement 1 is not correct.
• Under the Act, certain persons are prohibited to accept any foreign contribution. These include: public
servants (as defined under the Indian Penal Code), election candidates, editor or publisher of a
newspaper, judges, government servants, members of any legislature, and political parties, among
others. Hence, statement 2 is correct.

Q 59.A
• The Consumer Protection Act was enacted in 1986 with the objective of providing better protection of
consumers’ interests. The Act provides for effective safeguards to consumers against various types of
exploitation and unfair dealings.
• The Act enshrines six rights of consumers, namely,
o Right to Safety: It is the right to be protected against the marketing of goods and services which are
hazardous to life and property.
o Right to Information: It is the right of consumers to be informed about the quality, quantity,
potency, purity, standard and price of goods or services, with a view to protecting the consumer
against unfair trade practices.
22 www.visionias.in ©Vision IAS

FREE BY KING R QUEEN P [ऋषभ राजपूत]


o Right to Choose: The right to choose can be made meaningful by ensuring access to a variety of
goods and services at competitive prices.
o Right to Represent: It is the right to be heard and to be assured that consumers’ interests will receive
due consideration at appropriate forums.
o Right to Redressal: It is a right to seek redressal against unfair trade practices or restrictive trade
practices or unscrupulous exploitation of consumers.
o Right to Consumer Education: The right to consumer education is a right which ensures that
consumers are informed about the practices prevalent in the market and the remedies available to
them.
• Hence option (a) is the correct answer.

Q 60.A
• The President can remove the chairman or any other member of UPSC from office under the following
circumstances:
o If he is adjudged insolvent (that is, has gone bankrupt);
o If he engages, during his term of office, in any paid employment outside the duties of his office; or
o If he is, in the opinion of the president, unfit to continue in office by reason of infirmity of mind or
body.
o The president can also remove the chairman or any other member of UPSC for misbehaviour
but the President has to refer the matter to the Supreme Court for an enquiry. If the Supreme
Court, after the enquiry, upholds the cause of removal and advises so, the president can remove the
chairman or a member.
o Under the provisions of the Constitution, the advice tendered by the Supreme Court in this
regard is binding on the President. Hence statement 1 is correct.
• Defining the term ‘misbehaviour’ in this context, the Constitution states that the chairman or any
other member of the UPSC is deemed to be guilty of misbehaviour if he is concerned or interested in
any contract or agreement made by the Government of India or the government of a state, or participates
in any way in the profit of such contract or agreement or in any benefit therefrom otherwise than as a
member and in common with other members of an incorporated company.
• Hence statement 2 is not correct.

Q 61.D
• An election petition refers to the procedure for challenging the result of a Parliamentary election. To
hear these petitions, prior to the 19th Amendment, India had a mechanism of Election Tribunals. The
decision of such Election Tribunals would be final. Hence Statement I is not correct.
• The Nineteenth Amendment of the Constitution of India, officially known as The Constitution
(Nineteenth Amendment) Act, 1966, abolished Election Tribunals in India and enabled the trial of
election petitions by High Courts. Hence Statement II is correct.
Q 62.D
• Under the First Past the Post (FPTP) system, a candidate who gets one vote more than other candidates is
declared as the winner. In proportional representation, the number of seats won by a party or group of
candidates is proportionate to the number of votes received.
• Proportional representation (PR) is a complicated system which may work in a small country but would
be difficult to work in a sub-continental country like India. Hence option (a) is correct.
• The reason for the popularity and success of the FPTP system is its simplicity. The entire election system
is extremely simple to understand even for common voters who may have no specialized knowledge about
politics and elections. Hence option (b) is correct.
• There is also a clear choice presented to the voters at the time of elections. Voters have to simply endorse
a candidate or a party while voting. Depending on the nature of actual politics, voters may either give
greater importance to the party or to the candidate or balance the two.
• The FPTP system offers voters a choice not simply between parties but specific candidates. In other
electoral systems, especially PR systems, voters are often asked to choose a party and the representatives
are elected on the basis of party lists. As a result, there is no one representative who represents and is
responsible for one locality. In a constituency-based system like the FPTP, the voters know who their own
representative is and can hold him or her accountable. Hence option (c) is correct.
• More importantly, the makers of our Constitution also felt that a PR-based election may not be suitable for
giving a stable government in a parliamentary system. This system requires that the executive has a
majority in the legislature. The PR system may not produce a clear majority because seats in the
legislature would be divided on the basis of the share of votes. Hence option (d) is not correct.
23 www.visionias.in ©Vision IAS

FREE BY KING R QUEEN P [ऋषभ राजपूत]


Q 63.B
• Context: A five-judge Bench led by Chief Justice of India (CJI) D Y Chandrachud is hearing petitions
filed in the wake of last year’s political crisis in Maharashtra.
• About the Role of a Whip:
o A whip is an official of a political party whose task is to ensure party discipline in the legislature.
Hence statement 1 is correct.
o Whips are the party’s “enforcers”.
o The whip ensures that the members of the political party vote according to the party, rather than
according to their own individual ideology or the will of their donors or constituents.
o Every political party, whether ruling or Opposition has its own whip in the Parliament. He is
appointed by the political party and serves as an assistant floor leader. Hence statement 2 is not
correct.
o Under the Tenth Schedule (anti-defection law) a political party has a constitutional right to issue a
whip to its legislators. Hence statement 3 is correct.
o Kihoto Hollohan vs Zachillhu case, 1992
✓ The Supreme Court held that the application of the Tenth Schedule is limited to a vote on
a “motion of confidence” or “no-confidence” in the government or where the motion under
consideration relates to a matter which was an integral policy and program of the political party.
✓ Paragraph 2(1)(b) provides for a lawmaker’s disqualification “if he votes or abstains from voting
in such House contrary to any direction.
o Defiance of Whip
✓ A legislator may face disqualification proceedings if they disobey the whip of the party unless
the number of lawmakers defying the whip is 2/3rds of the party’s strength in the house.
✓ This disqualification is decided by the Speaker/Chairman of the house.

Q 64.D
• The Central Vigilance Commission was set up by the Government in February 1964 on the
recommendations of the Committee on Prevention of Corruption, headed by Shri K. Santhanam, to advise
and guide Central Government agencies in the field of vigilance. It was set up under the Prevention of
Corruption Act, of 1988. Hence statement I is not correct and statement II is correct.
• The CVC is not controlled by any Ministry/Department. It is an independent body that is only responsible
for the Parliament. The Commission shall consist of A Central Vigilance Commissioner - Chairperson;
• Not more than two Vigilance Commissioners - Members; The Central Vigilance Commissioner shall hold
office for a term of four years from the date on which he enters upon his office or till he attains the age of
sixty-five years, whichever is earlier.
• The Central Vigilance Commissioner, on ceasing to hold the office, shall be ineligible for reappointment
in the Commission or for further employment under the Central or state government.
• The CVC is not an investigating agency. The CVC either gets the investigation done through the CBI or
through the Departmental Chief Vigilance Officers. Secondly, the CVC orders investigation into cases of
officials of Central Government Departments/Companies/Organisations only.

Q 65.C
• Recently, the chairperson of the National Commission for Scheduled Castes (NCSC) resigned citing
personal reasons.
• National Commission for Scheduled Castes
o The National Commission for Scheduled Castes (NCSC) is a constitutional organization in India
that seeks to protect the interests of scheduled castes.
o It aims to safeguard the SC people from prejudice and exploitation while also offering facilities to
improve the SC community which is addressed under Article 338 of India's constitution.
o Its duties include investigating and monitoring all matters relating to safeguards provided for them,
inquiring into specific complaints, and participating and advising on the planning process of their
socio-economic development etc.
• It consists of a chairperson, a vice-chairperson, and three other members. They are appointed by
the President by warrant under his hand and seal. Hence statement 1 is correct.
• Functions:
o Monitoring and investigating all issues concerning the safeguards provided for the SCs under
the constitution.
o Enquiring into complaints relating to the deprivation of the rights and safeguards of the SCs
but its recommendations are non-binding in nature. Hence statement 2 is correct.
24 www.visionias.in ©Vision IAS

FREE BY KING R QUEEN P [ऋषभ राजपूत]


o Taking part in and advising the central or state governments with respect to the planning of the socio-
economic development of the SCs.
o Regular reporting to the President of the country on the implementation of these safeguards.
o Recommending steps to be taken to further the socio-economic development and other welfare
activities of the SCs.
o Any other function with respect to the welfare, protection, development, and advancement of the SC
community.
• Wings of NCSC

Q 66.B
• The National Disaster Management Authority has been constituted under the Disaster Management Act
2005, with the Prime Minister of India as its Chairman; a Vice Chairman with the status of Cabinet
Minister, and eight members with the status of Ministers of State. NDMA, as the apex body, is mandated
to lay down the policies, plans and guidelines for Disaster Management to ensure timely and effective
response to disasters.
• Hence option (b) is the correct answer.

Q 67.A
• Corruption refers to misusing public power for personal gain. It can be done by an elected politician, civil
servant, journalist, administrator of a school, or anyone in authority.
• India has the highest rate of bribery and use of personal links to access public services in Asia, according
to a survey released by global civil society Transparency International.
• India is in the 85th position among 180 countries in the Corruption Perception Index, 2021.
• Public servants in India can be penalized for corruption under the Indian Penal Code (IPC), 1860, and the
Prevention of Corruption Act, 1988. The Benami Transactions (Prohibition) Act, of 1988 prohibits
Benami transactions. The Prevention of Money Laundering Act, of 2002 penalizes public servants for the
offense of money laundering.
• The Benami Transactions (Prohibition) Act, 1988:
o The Act prohibits any benami transaction (purchase of property in the false name of another person
who does not pay for the property) except when a person purchases property in his wife’s or
unmarried daughter’s name.
o Any person who enters into a benami transaction shall be punished with imprisonment of up to three
years and/or a fine.All properties that are held to be benami can be acquired by a prescribed authority
and no money shall be paid for such acquisition.
• Hence, statement 1 is correct.
• The United Nations Convention Against Corruption is the only legally binding international anti-
corruption multilateral treaty. Negotiated by member states of the United Nations it has been adopted by
the UN General Assembly in October 2003 and entered into force in December 2005.
• Hence, statement 2 is not correct.
25 www.visionias.in ©Vision IAS

FREE BY KING R QUEEN P [ऋषभ राजपूत]


Q 68.A
• Number and Term of Members of the Board and its Office Bearers
o The board shall consist of such number of directors as may be provided by the state legislature. But,
the maximum number of directors of a cooperative society shall not exceed twenty-one. Hence,
statement 1 is correct.
o The state legislature shall provide for the reservation of one seat for the Scheduled Castes or the
Scheduled Tribes and two seats for women on the board of every cooperative society having members
from such a category of persons. There is no provision for the reservation of seats for the backward
classes. Hence, statement 2 is not correct.
o The term of office of elected members of the board and its office bearers shall be five years from the
date of the election.
o The state legislature shall make provisions for the co-option of persons having experience in the field
of banking, management, finance or specialization in any other related field, as members of the board.
But, the number of such co-opted members shall not exceed two (in addition to twenty-one directors).
Further, the co-opted members shall not have the right to vote in any election of the co-operative
society or be eligible to be elected as office bearers of the board.
o The functional directors of a cooperative society shall also be the members of the board and such
members shall be excluded for the purpose of counting the total number of directors (that is, twenty-
one).
Q 69.B
• As per Census-1931, Schedule tribes are termed as "backward tribes” living in the "Excluded"
and "Partially Excluded" areas. The Government of India Act of 1935 called for the first time for
representatives of "backward tribes" in provincial assemblies.
• The Constitution does not define the criteria for recognition of Scheduled Tribes and hence the
definition contained in the 1931 Census was used in the initial years after independence.
• However, Article 366(25) of the Constitution only provides a process to define Scheduled Tribes:
“Scheduled Tribes means such tribes or tribal communities or parts of or groups within such tribes or
tribal communities as are deemed under Article 342 to be Scheduled Tribes for the purposes of this
Constitution.” Hence statement 1 is correct.
• 342(1): The President may with respect to any State or Union Territory, and where it is a State,
after consultation with the Governor, by public notification, specify the tribes or tribal communities
or part of or groups within tribes or tribal communities as Scheduled Tribe in relation to that State
or Union Territory. Hence statement 2 is not correct.
• There are over 700 tribes that have been notified and the largest number of tribal communities (62) are
found in Odisha.
• No Tribe was identified in Haryana, Punjab, Chandigarh, Delhi, and Pondicherry. Hence statement
3 is correct
Few Related Committees
• The Lokur Committee (1965) was set up to look into criteria for defining Schedule Tribes. The
Committee recommended 5 criteria for identification, namely, primitive traits, distinct culture,
geographical isolation, shyness of contact with the community at large, and backwardness.
• Bhuria Commission (2002-2004) focused on a wide range of issues from the 5th Schedule to tribal
land and forests, health and education, the working of Panchayats, and the status of tribal women.
• A High-Level Committee (HLC) in 2013, under the chairmanship of Prof. Virginius Xaxa was
constituted to study the 5 critical issues related to tribal communities : (1) livelihood and employment,
(2) education, (3) health, (4) involuntary displacement and migration, (5) and legal and constitutional
matters.

Q 70.C
• Recently it was reported in the news that 486 ASI antiquities have been missing since 1947.
• The Antiquities and Art Treasures Act of 1972 defines an "antiquity" as an object or article that is at
least 100 years old and represents science, art, crafts, customs, religion, literature, or anything of
historical interest. Hence statement 1 is not correct.
• An "art treasure" is a human work of art, other than antiquity, that is declared to be a treasure by
the Centre for its artistic value after the artist's death. If it is a manuscript or record of any scientific,
historical, literary, or aesthetic value, it should be at least 75 years old. Hence statement 2 is correct.
• India's rich cultural heritage, bureaucratic apathy, and poor implementation of antiquities protection law
have made India a fertile ground for the smuggling of antiques for sale in the international market, which
26 www.visionias.in ©Vision IAS

FREE BY KING R QUEEN P [ऋषभ राजपूत]


poses a risk to national security. Poor restoration and conservation of antiquities in museums, the
lack of an integrated database of existing and stolen artifacts, and the absence of any centralized
information are some of the major concerns.
• Preventive measures include the compulsory registration of notified categories of antiquities,
regulating the export trade in antiquities and art treasures, and preventing smuggling and
fraudulent dealings in antiques are taken under the Antiquities and Art Treasures Act. Hence statement
3 is not correct.
• Section 3 of the Act prohibits the export of antiquity by anyone other than the Centre or its
agencies.

Q 71.B
• Recent Context-As the year 2024 marks the centenary of the Vaikom Satyagraha, the Chief Minister of
Kerala and Tamil Nadu jointly inaugurated the centenary celebrations.
• What is Vaikom Satyagraha? Background: The princely state of Travancore had a feudal, militaristic,
and ruthless system of custom-ridden government, some of the most rigid, refined and ruthless social
norms and customs were seen in Travancore. Lower castes like the Ezhavas and Pulayas were considered
polluting and various rules were in place to distance them from upper castes. These included a prohibition,
not just on temple entry, but even on walking on the roads surrounding temples.
• Beginning of Satyagraha:
• On March 30, 1924, the Satyagrahis walked in procession toward the forbidden public roads. They were
stopped 50 yards away from the place where a board cautioning the oppressed communities against
walking on the roads (surrounding the Vaikom Mahadeva temple), was placed. Dressed in khadi and
wearing khadi caps, Govinda Panikkar (Nair), Bahuleyan (Ezhava) and Kunjappu (Pulaya), defied the
prohibitionary orders. The police stopped them. In protest, the three men sat on the road and were
arrested. Then on, every day, three volunteers from three different communities were sent to walk
on the prohibited roads. Within a week, the leaders of the movement were all arrested. The
movement was non-violent in nature. Hence statement 1 is not correct.
• Contribution of Leaders: In 1923, Madhavan presented the issue as a resolution at the Kakinada meeting
of the All India Congress Committee. Subsequently, it was taken up by the Congress Untouchability
Committee formed by the Kerala Pradesh Congress Committee in January 1924. Periyar E.V.
Ramaswamy was a great social reformer. In 1924, he took an active part in the Vaikam Satyagraha and
provided leadership to the satyagraha.
• E.V. Ramasamy Periyar led the famous Vaikam Sathya Graha in 1924, he was given the title of
'Vaikam Hero' for his leadership. The other associated leaders were T. K. Madhavan, K. Kelappan
K. P. Kesava Menon, George Joseph. Mahatma Gandhi is also associted with vaikom satyagraha
and he himself visited vaikom in 1925. Hence statement 3 is correct.
• In 1936, the historic Temple Entry Proclamation was signed by the Maharaja of Travancore which
removed the age-old ban on the entry of temples. Hence statement 2 is correct.

Q 72.B
• Criteria for grant of Maharatna status to CPSEs: The CPSEs meeting the following criteria are
eligible to be considered for grant of Maharatna status.
o Having Navratna status
o Listed on the Indian stock exchange with minimum prescribed public shareholding under SEBI
regulations
o An average annual turnover of more than Rs. 25,000 crores during the last 3 years
o An average annual net worth of more than Rs. 15,000 crores during the last 3 years
o An average annual net profit after tax of more than Rs. 5,000 crores during the last 3 years
o Should have significant global presence/international operations.
o Hence option (b) is the correct answer.
• Criteria for grant of Navratna status to CPSEs: The CPSEs which are Miniratna I, Schedule ‘A’ and
have obtained ‘excellent’ or ‘very good’ MOU rating in three of the last five years and have a composite
score of 60 or above in following six selected performance indicators are eligible to be considered for
grant of Navratna status.
• Maximum Weight: 100
o Net Profit to Net worth : 25
o Manpower Cost to total Cost of Production or Cost of Services : 15
o PBDIT to Capital employed : 15
o PBIT to Turnover : 15
27 www.visionias.in ©Vision IAS

FREE BY KING R QUEEN P [ऋषभ राजपूत]


o Earning Per Share : 10
o Inter Sectoral Performance : 20
• Criteria for grant of Miniratna status to CPSEs:
o Miniratna Category-I status: The CPSEs which have made a profit in the last three years
continuously, with pre-tax profit is Rs.30 crores or more in at least one of the three years, and have a
positive net worth are eligible to be considered for grant of Miniratna-I status.
o Miniratna Category-II status: The CPSEs which have made a profit for the last three years
continuously and have a positive net worth are eligible to be considered for grant of Miniratna-II
status.
o Miniratna CPSEs should have not defaulted in the repayment of loans/interest payments on any loans
due to the Government.
o Miniratna CPSEs shall not depend upon budgetary support or Government guarantees.

Q 73.C
• The Attorney General (AG) of India is a part of the Union Executive. AG is the highest law officer
in the country. Article 76 of the Constitution provides for the office of AG of India. Hence statement
1 is correct.
• AG is appointed by the President on the advice of the government. S/he must be a person who is qualified
to be appointed a judge of the Supreme Court, i.e. s/he must be a citizen of India and must have been a
judge of some high court for five years or an advocate of some high court for ten years or an eminent
jurist, in the opinion of the President.
• Term of the Office: It is not fixed by the Constitution.
• Removal: Procedures and grounds for the removal of AG are not stated in the Constitution so he
holds office at the pleasure of the President (may be removed by the President at any time).
• Duties and Functions:
o To give advice to the Government of India (GoI) upon such legal matters, which are referred to
her/him by the President.
o To perform such other duties of a legal character that are assigned to her/him by the President.
o To appear on behalf of the GoI in all cases in the Supreme Court or in any case in any High Court in
which the GoI is concerned.
o To represent the GoI in any reference made by the President to the Supreme Court under Article 143
(Power of the President to consult the Supreme Court) of the Constitution.
o He should not defend accused persons in criminal prosecutions without the permission of the
Government of India. Hence statement 3 is correct.
• Rights and Limitations:
o S/he has the right to speak and to take part in the proceedings of both the Houses of Parliament or
their joint sitting and any committee of the Parliament of which s/he may be named a member, but
without a right to vote.
o S/he enjoys all the privileges and immunities that are available to a member of Parliament.
o S/he does not fall in the category of government servants and is not debarred from private legal
practice. Hence statement 2 is correct.

Q 74.C
• Debrigarh Wildlife Sanctuary:
o Context: As per recent reports, the Debrigarh Wildlife Sanctuary of Odisha recorded a high prey
density. Hence option (c) is the correct answer.
o Following the pre-monsoon sign survey in the Debrigarh Wildlife Sanctuary, an herbivore or prey
density of 46 animals per sq. km was recorded in the sanctuary.
o Moreover, for the first time in the last many decades, a tiger has also been sighted in Debrigarh during
Census.
o About Debrigarh Wildlife Sanctuary:-
o Location: Bargarh district of Odisha, near Hirakud dam.
o Hirakud dam: it is on the Mahanadi River.
o
o Biodiversity:
o Flora: Dry deciduous forests
o Fauna: Leopards, deer, sambar, elephants, gaur, wild boar, and a variety of birds and other creatures
can be found in the Debrigarh sanctuary’s vast and dense woodlands.

28 www.visionias.in ©Vision IAS

FREE BY KING R QUEEN P [ऋषभ राजपूत]


o It is also known for easy wildlife sightings, including Indian bison, sambhar, wild boars, peacocks,
etc.
o The four-horned antelope or Chousingha is one of the endangered animals that also inhabits this
sanctuary.
o It is an Eco-sensitive Zone.
• Chinnar Wildlife Sanctuary
o Citizen: The tribal settlements in Chinnar Wildlife Sanctuary are reviving the cultivation of millets
and endemic crops through the Punarjeevanam scheme.
o Chinnar Wildlife Sanctuary in Idukki district of Kerala comes under Munnar Wildlife Division.
o Chinnar Wildlife Sanctuary is home to the Great Grizzled Squirrel of India.
o There are 11 tribal settlements comprising Muduvan and Hill Pulaya communities under the Wildlife
Division.
o Punarjeevanam (revival or resurrection) scheme was launched in 2016 by Kerala Forest and Wildlife
Department.
o The scheme was launched to revive farming of millets and endemic crops in the tribal settlements
under Chinnar Wildlife Sanctuary.
o It has revived 34 varieties of ragi, in addition to multiple kinds of beans, millets, amaranthus, maize,
and pumpkin.
• Amchang Wildlife Sanctuary
o Context: Recently, Indian Army generated a unique ecosystem for peaceful co-existence with wild
o elephants in Amchang Wildlife Sanctuary, Assam.
o About Amchang Wildlife Sanctuary:
o The Amchang Wildlife Sanctuary is located on the eastern fringe of Guwahati, Assam.
o It comprises three Reserve forests:
o Khanapara, amchang, and South Amchang
o It stretches from the Brahmaputra River in the north to the hilly forests of Meghalaya in
o the south, forming a continuous forest belt through Meghalaya's Maradakdola Reserve
o Forests.
o It was declared a wildlife sanctuary in 2004 by the government of Assam.
o Flora: Khasi Hill Sal Forests, East Himalayan Mixed Deciduous Forest, Eastern Alluvial
o Secondary Semi-Evergreen Forests and East Himalayan Sal Forests.
o Fauna: It is home to Mammals (Flying Fox, Assamese Macaque, Slow Loris, etc.), Birds
o (Lesser and Greater Adjutant, White-backed Vulture, Slender-billed Vulture), Reptiles (Python,
Monitor Lizard, Indian Cobra, etc.).
o Tree yellow butterflies: found at the Amchang wildlife sanctuary, which are indigenous to Thailand,
Malaysia, Singapore, and northeast India.

Q 75.A
• Recently, a Member of Parliament in India has urged the government to include women from the
Scheduled Tribe community in the inheritance rights provisions of the Hindu Succession
Amendment Act, 2005. Currently, Section 2(2) of the Act excludes these women, which results in the
denial of their equal rights to inherit their father's or Hindu Undivided Family properties. Hence
statement 2 is correct.
• The M.P. argues that this exclusion is discriminatory based on gender and against the Indian
Constitution's Article 14, which advocates for equality before the law. The Hindu Succession Act, of
1956, only recognized male descendants as legal heirs, but the 2005 amendment granted daughters
equal rights to inherit their father's or HUF properties.
• The Hindu Succession Act lays down a comprehensive system of laws for cases relating to conditions
like intestate (dying without a will), or unwilled succession, among Hindus, Buddhists, Jains, and
Sikhs including followers of Brahmo Samaj and Arya Samaj. Hence statement 1 is correct.
• However, tribal women were considered excluded from the scope of the Act, as section 2 (2) of the Act
states that the provisions will not apply to the Scheduled Tribes unless the central government directs
otherwise “by notification”.
• The Hindu Succession Act of 2005 brought about significant changes to the Act by giving daughters
equal rights to ancestral property, which was previously denied to them.
• The amendment also provides for the devolution of a woman's property on her death, in the same way
as a man's property devolves, and for the first time recognizes the rights of widows in a deceased
husband's property. The amendment has been hailed as a significant step towards gender equality in
India and has helped to empower women by giving them greater control over their property and financial
29 www.visionias.in ©Vision IAS

FREE BY KING R QUEEN P [ऋषभ राजपूत]


assets. The law applies to ancestral property and to intestate succession in personal property,
where succession happens as per law and not through a will. In the case of Hindus, the laws relating to
the testamentary succession( through a Will) are applicable as per the Indian Succession Act, 1925. Hence
statement 3 is not correct.
• It's important to note that the 2005 amendment only applies to Hindus, Buddhists, Sikhs, and Jains. Other
religions have their own personal laws that govern inheritance rights.

Q 76.C
• In 1901 the Famine Commission recommended the establishment of Rural Agricultural Banks through the
establishment of Mutual Credit Associations, and such steps were taken by the Government of North
Western provinces and Oudh. The underlying idea of a number of persons combining together was the
voluntary creation of a new and valuable security. A strong association competent to offer guarantees and
advantages of lending to groups instead of individuals were major advantages. The Commission also
suggested the principles underlying Agricultural Banks.
• Cooperative Credit Societies Act, 1904 - The First Incorporation Taking cognizance of these
developments and to provide a legal basis for cooperative societies, the Edward Law Committee with
Mr. Nicholson as one of the members was appointed by the Government to examine and recommend a
course of action. The Cooperative Societies Bill, based on the recommendations of this Committee, was
enacted on 25th March 1904. As its name suggests, the Cooperative Credit Societies Act was restricted to
credit cooperatives. Hence option 1 is correct.
• Maclagen Committee on Cooperation (1914) - The Banking Crisis and the First World War both
affected the growth of cooperatives. Although member deposits in cooperatives increased sharply, the war
affected the export and prices of cash crops adversely, resulting in increased over-dues of loans of primary
agricultural societies. Hence option 2 is correct.
• In 1944, the Gadgil Committee recommended compulsory adjustment of debts and setting up of
Agricultural Credit Corporations, wherever cooperative agencies were not strong enough. Hence, option
3 is correct.
• A major watershed initiative at this time was the appointment by the Government of the Gorwala
Committee, popularly known as the All India Rural Credit Survey Committee. The Committee was
appointed in 1951 and submitted its report in 1954. It observed that large parts of the country were not
covered by cooperatives and in such areas where it had been covered, a large segment of the agricultural
population remained outside its membership. Hence, option 4 is correct.

Q 77.B
• Transformer in ChatGPT
• Context: In recent times, Machine Learning (ML) is experiencing a transformative shift with the rise of
transformer models.
o The capital ‘T’ in ChatGPT stands for ‘transformer’. Hence, statement 1 is not correct.
o Transformers are tasked with translating a sentence from one language to another, similar to what
Google Translate does when converting from, say, English to Hindi. Hence, statement 2 is correct.
o A transformer is a two-part neural network. The first part is an ‘encoder’ that ingests the input
sentence in the source language (e.g. English); the second is a ‘decoder’ that generates the translated
sentence in the target language (Hindi).
o The encoder converts each word in the source sentence to an abstract numerical form that captures the
meaning of the word within the context of the sentence and stores it in a memory bank.
o Just like a person would write or speak, the decoder generates one word at a time referring to what has
been generated so far and looking back at the memory bank to find the appropriate word.
o Both these processes use a mechanism called ‘attention’.
o A key improvement over previous methods is the ability of a transformer to translate long sentences
or paragraphs correctly.

Q 78.C
• The Right of Children to Free and Compulsory Education (RTE) Act, 2009:
o The Constitution (Eighty-sixth Amendment) Act, 2002 inserted Article 21-A in the Constitution
of India to provide free and compulsory education of all children in the age group of six to fourteen
years as a Fundamental Right.
o Consequently The Right of Children to Free and Compulsory Education (RTE) Act, 2009, which
represents the legislation envisaged under Article 21-A, was passed in the parliament. Hence
statement 1 is correct.
30 www.visionias.in ©Vision IAS

FREE BY KING R QUEEN P [ऋषभ राजपूत]


• The RTE Act provides for the following:
o Right of children to free and compulsory education till completion of elementary education in a
neighbourhood school.
o It makes provisions for a non-admitted child to be admitted to an age appropriate class.
o It lays down the norms and standards relating inter alia to Pupil Teacher Ratios (PTRs), buildings
and infrastructure, school-working days, teacher-working hours.
o It provides for appointment of appropriately trained teachers, i.e. teachers with the requisite entry and
academic qualifications.
o It prohibits (a) physical punishment and mental harassment; (b) screening procedures for
admission of children; (c) capitation fee; (d) private tuition by teachers and (e) running of
schools without recognition.
o All private schools must keep 25% of seats reserved for children belonging to weaker sections of
society. Hence statement 2 is correct.

Q 79.A
• The Consumer Protection Act, 2019:
o Act will empower consumers and help them in protecting their rights through its various notified
Rules and provisions like Consumer Protection Councils, Consumer Disputes Redressal
Commissions, Mediation, Product Liability and punishment for manufacture or sale of products
containing adulterant / spurious goods.
• Provisions and rules regarding E-Commerce:
o The act recognises e-commerce platforms and provides following rules;
o Under this act every e-commerce entity is required to provide information relating to return, refund,
exchange, warranty and guarantee, delivery and shipment, modes of payment, grievance
redressal mechanism, payment methods, security of payment methods, charge-back options, etc.
o They need to provide information regarding country of origin which are necessary for enabling the
consumer to make an informed decision at the pre-purchase stage on its platform.
o The e-commerce platforms have to acknowledge the receipt of any consumer complaint within
forty-eight hours and redress the complaint within one month from the date of receipt under this
Act. Hence statement 1 is not correct.
• Product liability:
o New Act introduces the concept of product liability and brings within its scope, the product
manufacturer, product service provider and product seller, for any claim for compensation.
Hence statement 3 is not correct.
o The Act provides for punishment by a competent court for manufacture or sale of adulterant/spurious
goods.
o The court may, in case of first conviction, suspend any licence issued to the person for a period of up
to two years, and in case of second or subsequent conviction, cancel the licence.
• Fee for filing cases:
o As per the Consumer Disputes Redressal Commission Rules, there will be no fee for filing cases
upto Rs. 5 lakh. Hence statement 2 is correct.
o There are provisions for filing complaints electronically, credit of amount due to unidentifiable
consumers to Consumer Welfare Fund (CWF).

Q 80.D
• Recently, The Food Safety and Standards Authority of India (FSSAI) has operationalised the
maximum residue limit (MRL) for five pesticides used in tea plantations. The five pesticides are
emamectin benzoate, fenpyroximate, hexaconazole, propiconazole, and quinalphos.
o Emamectin Benzoate is widely used in controlling lepidopterous pests (larvae and insects including
butterflies, moths, and skippers)
o Hexaconazole is a systemic conazole (imidazole) fungicide used for the control of many seed-borne
and soil-borne diseases and fungi.
o Fenpyroximate and Propiconazole are pesticides with activity against various mites as well as a few
whiteflies and psyllids.
o Quinalphos is an organophosphorus insecticide and acaricide with contact and stomach action.
• Hence, option(d) is the correct answer.

31 www.visionias.in ©Vision IAS

FREE BY KING R QUEEN P [ऋषभ राजपूत]


Q 81.D
• Article 345 states that, subject to the provisions of Articles 346 and 347, the legislature of a state may by
law adopt any one or more of the languages in use in the State or Hindi as the language or
languages to be used for all or any of the official purposes of that State provided that, until the Legislature
of the State otherwise provides by law, the English language shall continue to be used for those official
purposes within the State for which it was being used immediately before the commencement of this
Constitution. Hence statement 2 is not correct.
• Hence, the ‘in use’ criteria do not impede any state to not to declare Hindi as the official language.
This is complemented by Article 347 under which the president may direct the states to recognize a
language as an official language if a substantial population demands so to be used all over the states
or in some parts.
• Under this provision, most of the states have adopted the major regional language as their official
language. For example, Andhra Pradesh has adopted Telugu, Kerala– Malayalam, Assam–Assamese,
West Bengal–Bengali, Odisha–Odia. The nine northern states of Himachal
• Pradesh, Uttar Pradesh, Uttarakhand, Madhya Pradesh, Chhattisgarh, Bihar, Jharkhand, Haryana, and
Rajasthan have adopted Hindi. Gujarat has adopted Hindi in addition to Gujarati.
• Article 343 (1) of the Constitution provides that Hindi in Devanagari script shall be the official
language of the Union. Hence statement 1 is not correct.
• Article 343 (3) empowered the Parliament to provide by law for the continued use of English for official
purposes even after January 25, 1965.

Q 82.D
• National Commission for Backward Classes(NCBC)
o The Supreme Court ordered the government to establish a permanent commission to consider,
examine, and recommend the inclusion and exclusion of various Backward Classes for the purposes
of benefits and protection in the Indra Sawhney case of 1992.
o The National Commission for Backward Classes Act of 1993 was passed in response to these
directives, and the NCBC was established.
o Later, the 102nd Amendment Act of 2018 conferred a constitutional status on the Commission.
o The amendment inserted a new Article 338-B in the constitution. Hence, the Commission ceased
to be a statutory body and became a constitutional body. Hence statement 1 is correct.
• The functions of the Commission are the following:
o To investigate and monitor all matters relating to the constitutional and other legal
safeguards for the socially and educationally backward classes.
o To inquire into specific complaints with respect to the deprivation of rights and safeguards of
the socially and educationally backward classes.
o To participate and advise on the socio-economic development of the socially and educationally
backward classes and to evaluate the progress of their development under the Union or a state.
o To present to the President, annually and at such other times as it may deem fit, reports upon
the working of those safeguards. Hence statement 2 is correct.
• The Commission, while investigating any matter or enquiring into any complaint, has all the powers
of a civil court trying a suit and in particular in respect of the following matters:
o Summoning and enforcing the attendance of any person from any part of India and examining
him on oath. Hence statement 3 is correct.
o Requiring the discovery and production of any document and receiving evidence on affidavits.
o Requisitioning any public record from any court or office
o Issuing summons for the examination of witnesses and documents and any other matter which
the President may determine

Q 83.D
• The Systematic Voters’ Education and Electoral Participation Program (SVEEP) is the flagship
program of the Election Commission of India for:
o voter education,
o spreading voter awareness, and
o promoting voter literacy in India.
• It is a multi-intervention program that reaches out through different modes and media to educate
citizens, electors, and voters about the electoral process in order to increase their awareness and promote
their informed participation.

32 www.visionias.in ©Vision IAS

FREE BY KING R QUEEN P [ऋषभ राजपूत]


• The objectives envisaged for SVEEP as per the strategy document for 2022–25 are:
o Increase voter turnout to 75% in Lok Sabha elections in 2024 by:
✓ Purifying the electoral roll of every polling booth
✓ Bridging gender gap in enrolment and turnout
✓ Ensuring inclusion of all non-voters and marginalized sections through targeted interventions,
technological solutions and policy changes
✓ Addressing urban and youth apathy in electoral participation
✓ Turning around all low turnout constituencies and polling stations
o To enhance the quality of electoral participation in terms of informed and ethical voting through
continuous electoral and democracy education
• Hence option (d) is the correct answer.

Q 84.D
• April 29 is the birth anniversary of the famed Indian painter Raja Ravi Varma (1848-1906).
• Key contributions:
o He is remembered for giving Indians their Western, classical representations of Hindu gods and
goddesses. Through his printing press, Varma’s humanised depiction of Hindu pantheon travelled
beyond the surfaces of costly canvases, and into the prayer and living rooms of working-class
homes.He achieved this by making affordable lithographs, which were accessible even to the poor.
o Ravi Varma first started a press in Mumbai and later shifted it to a place near Lonavala. Awards he
won:
o His 1873 painting, Nair Lady Adorning Her Hair, won Varma prestigious awards including
Governor’s Gold Medal when it was presented in the Madras Presidency, and Certificate of Merit at
an exhibition in Vienna.
o In 1904, the British colonial government awarded Varma with the Kaiser-i-Hind Gold Medal. In
2013, a crater on the planet Mercury was named in his honour.
• His prominent works include: A Family of Beggars, A Lady Playing Swarbat, Arjuna and Subhadra,
Damayanti Talking to a Swan, Jatayu (a bird devotee of Lord Rama), Lady Lost in Thought, Shakuntala.
• Criticisms:
• Varma is often criticized for being too showy in his paintings. His paintings are also condemned for
overshadowing traditional Indian art forms, especially the ones depicting Hindu gods and goddesses. His
approach is said to lack the dynamism of expression seen in traditional paintings. Critics have also
criticized him for modelling goddesses after prostitutes, saying that his representation of deities have
reduced them to the level of mortals.

33 www.visionias.in ©Vision IAS

FREE BY KING R QUEEN P [ऋषभ राजपूत]


Q 85.A
• River Kolak
o Context: Kolak fisherfolk living along the banks of River Kolak is in distress as chemicals from Vapi
industries destroy river catch.
o Kolak River is a river in Gujarat in western India.
o The west-flowing river originates in Kaprada taluka. Kolak River flows into the Arabian Sea near
Udwada.
o It is also connected to the Madhuban reservoir of the Damanganga River.
o It flows through the north side of Vapi city in Gujarat. Kolak village on the bank of River Kolak in
Gujarat’s Valsad district.
o Hence option (a) is the correct answer.

Q 86.D
• All property and assets that were vested in the Dominion of India or a province or an Indian princely state,
before the commencement of the present Constitution, became vested in the
• Union or the corresponding state. Similarly, all rights, liabilities and obligations of the government
of the dominion of India or a province or an Indian state would now be the rights, liabilities and
obligations of the Government of India or the corresponding state. Hence, statement 1 is not
correct.
• All lands, minerals, and other things of value under the waters of the ocean within the territorial
waters of India, the continental shelf of India, and the exclusive economic zone of India vests in the
Union. Hence, a state near the ocean cannot claim jurisdiction over these things. India’s territorial waters
extend to a distance of 12 nautical miles from the appropriate baseline. Similarly, India’s exclusive
economic zone extends upto 200 nautical miles. Hence, statement 2 is not correct.
• The Parliament as well as the state legislatures are empowered to make laws for the compulsory
acquisition and requisitioning of private property by the governments. Further, the 44th Amendment Act
(1978) has also abolished the constitutional obligation to pay compensation in this regard except in two
cases:
o when the government acquires the property of a minority educational institution; and
o when the government acquires the land held by a person under his personal cultivation and the
land is within the statutory ceiling limits. Hence, statement 3 is not correct.

Q 87.D
• Zoonomia Project
o The ambitious genome project, Zoonomia, shows how humans fit with other mammals. Hence
option (d) is the correct answer.
o The Zoonomia Project is an international collaboration to discover the genomic basis of shared and
specialized traits in mammals.
o Zoonomia project compares the genomes of 240 mammal species as well as people to trace
evolutionary changes spanning 100 million years.
o It aims at pinpointing genetic traits widely shared and those more uniquely human.
o Key Findings:
✓ The project identified parts of the genome functionally important in people and other mammals.
✓ It showed how certain mutations can cause disease.
✓ The project revealed the genetics of uncommon mammalian traits like hibernation and showed
how the sense of smell varies widely.
o Some mammals have a very keen sense of smell like the African savanna elephant while others have
almost none like whales and dolphins.
o Humans have an average sense of smell.

Q 88.B
• Centralized Public Grievance Redress and Monitoring System (CPGRAMS): It is an online web-
enabled system developed by the National Informatics Centre (Ministry of Electronics & IT [MeitY]), in
association with the Directorate of Public Grievances (DPG) and Department of Administrative Reforms
and Public Grievances (DARPG). Hence, statement 2 is correct.
• The CPGRAMS provides the facility to lodge a grievance online from any geographical location. It
enables the citizen to track online the grievance being followed up with the Departments concerned and
also enables DARPG to monitor the grievance.

34 www.visionias.in ©Vision IAS

FREE BY KING R QUEEN P [ऋषभ राजपूत]


• e-Samiksha: A real-time online system for monitoring and follow-up action on the decisions taken by the
Government at the Apex level in respect of the implementation of important Government programs/
projects. Hence, statement 1 is correct.
• Itat e dwar: It is an e-filing portal of the Income Tax Appellate Tribunal. Hence, statement 3 is not
correct.
o It will enable the online filing of Appeals, Applications, documents, etc. by various parties.
o It would not only result in the economization of the use of paper, and savings in costs but also
rationalization of the fixation of cases leading to quicker disposal of cases.
o It seeks to bridge the divide between the digital haves and the digital have-nots leading to digital
inclusion achieved by technology.
• Hence, option (b) is the correct answer.

Q 89.B
• Recent context: According to a recent report by the Association of Democratic Reform (ADR), donations
from unknown sources comprised more than half the funding received by eight national political parties.
• Political parties are not required to reveal the name of individuals or organizations giving less than
Rs. 20,000 nor those who donated via Electoral Bonds. Hence statement 1 is correct.
• No limit on political parties: Under Section 77 of Representation of Peoples Act, 1951 and Conduct of
Election Rules, 1961, there are limits on election expenditure only for candidates, such a limit is absent
in the case of political party expenditure. Hence statement 2 is correct.
• Contributions received by the parties are also 100% exempted from income tax.
• In the 2017, finance bill, the government included an amendment to the Companies Act of 2013 to
do away with the 7.5% of net profits limit set on donations and the requirement for a company to
disclose the name of political parties to which they donate. Hence statement 3 is correct.

Q 90.B
• Social Audit is the audit of a scheme jointly by the Government and the people, especially by those who
are affected by the scheme or its beneficiaries. Hence statement 1 is correct.
• Social audit is different from financial audits which involve inspecting and assessing documents related to
financial transactions in an organization to provide a true picture of its profits, losses and financial
stability. Hence statement 3 is not correct.
• Social audits, citizen feedback mechanisms, and sustainability reporting are some of the approaches being
used by governments to evaluate their performance and ensure that they are meeting the needs of their
citizens.
• It is a powerful tool that helps government bodies and civil society organisations to promote
accountability, transparency, and good governance. It ensures true democracy by direct participation.
Hence statement 2 is correct.
• It enhances mutual understanding and the legitimacy of government policies and programs. Moreover,
Social Audit can help to identify areas of inefficiency and waste, and to promote best practices in resource
allocation and management.
• It is a process of evaluating and assessing the impact of social and economic policies and programs. It
involves collecting and analysing information from various stakeholders to understand the social,
environmental, and economic effects of policies and programs.
• By involving stakeholders in the process, social auditing can also help to build trust and improve
communication between the government, businesses, and communities.

Q 91.D
• An electoral bond is like a promissory note that can be bought by any Indian citizen or company
incorporated in India from select branches of the State Bank of India. The citizen or corporate can then
donate the same to any eligible political party of his/her choice.
• The bonds under the Scheme shall be available for purchase for a period of 10 days each in the months of
January, April, July, and October, as may be specified by the Central Government.
• Only the political parties registered under Section 29A of the Representation of the People Act, 1951, and
have secured not less than 1% of the votes polled in the last general election to the House of the People or
the Legislative Assembly, are eligible to receive electoral bonds. Hence, statement 3 is not correct.
• State Bank of India is authorized to issue and encash these bonds.
• Electoral bonds are purchased anonymously by donors and are valid for 15 days from the date of
issue. Hence, statement 1 is not correct.
35 www.visionias.in ©Vision IAS

FREE BY KING R QUEEN P [ऋषभ राजपूत]


• As debt instruments, these can be bought by donors from a bank, and the political party can then encash
them.
• These can be redeemed only by an eligible party by depositing the same in its designated account
maintained with a bank.
• The bonds are issued by SBI in denominations of Rs 1,000, Rs 10,000, Rs 1 lakh, Rs 10 lakh, and Rs 1
crore. Hence, statement 2 is not correct.
• The bonds are available for purchase by any citizen of India for a period of ten days each in the months of
January, April, July, and October as may be specified by the Central Government.

Q 92.C
• NATGRID, attached office of Ministry of Home Affairs (MHA), is the integrated intelligence grid
that connects databases of core security agencies. It was proposed after 2008 Mumbai terror attacks
and approved in 2010. Hence statement 1 is correct.
• It is conceptualised as a seamless and secure database for information on terrorists, economic crimes and
similar incidents.
• It will help to track suspects with real-time data and access to classified information like immigration,
banking, individual taxpayers, air and train travels.
• NATGRID database will be available to prominent federal agencies including CBI, ED, IB, DRI
(Directorate of Revenue
Intelligence), NIA etc. Hence statement 2 is correct.
• NATGRID faced opposition on account of possible violations of privacy and leakage of confidential
personal data.

Q 93.B
• Statement 1 is not correct: The Constitution makes a provision for the establishment of a Joint State
Public Service Commission (JSPSC) for two or more states. While the UPSC and the SPSC are created
directly by the Constitution, a JSPSC can be created by an act of Parliament on the request of the state
legislatures concerned.
• Statement 2 is correct: The chairman and members of a JSPSC are appointed by the president. They hold
office for a term of six years or until they attain the age of 62 years, whichever is earlier.
• Statement 3 is not correct: A JPSC presents its annual performance report to each of the concerned state
governors. Each governor places the report before the state legislature.

Q 94.A
• Civil Society 20 (C20), launched in 2013 as an official G20 Engagement Group, provides a platform
for the Civil Society Organisations to bring forth non-governmental perspectives on the issues being
taken up by the official G20. It gives them a forum to reflect on the primary & common
concerns affecting the world and promote social & economic development. Hence, statement 1 is
correct.
36 www.visionias.in ©Vision IAS

FREE BY KING R QUEEN P [ऋषभ राजपूत]


• India is holding the presidency of G20 this year and C20 Summit took place in Jaipur, Rajasthan on 30-
31 July 2023. Hence, statement 2 is not correct.

Q 95.B
• Article 39 A of the Constitution of India provides for free legal aid to the poor and weaker sections of the
society, to promote justice on the basis of equal opportunity. Article 14 and Article 22 (1), obligate the
State to ensure equality before the law. Hence, statement 1 is correct.
• The National Legal Services Authority (NALSA) has been constituted under the Legal Services
Authorities Act, 1987 to provide free Legal Services to the weaker sections of the society and to organize
Lok Adalats for amicable settlement of disputes.
• The Patron-in-chief is the Chief Justice of India.
• The Executive Chairman of the Authority is the second senior-most judge of the SC. Hence, statement 2
is not correct.
• At the state level, the State Legal Services Authority has been constituted to give effect to the policies of
NALSA at the state level, and also to conduct Lok Adalats in the states. NALSA provides funds for the
State Legal Services Authority for the implementation of various legal aids and programmes.
• At the district level also, the District Legal Services Authority has been established.
• Taluk Legal Services Committees are also constituted for each of the Taluk or Mandal or for a group of
Taluk or Mandals to coordinate the activities of legal services in the Taluk and to organise Lok Adalats.
Every Taluk Legal Services Committee is headed by a senior Civil Judge operating within the jurisdiction
of the Committee who is its ex-officio Chairman.
• The sections of the society as enlisted under Section 12 of the Legal Services Authorities Act are entitled
to free legal services, they are :
o A member of a Scheduled Caste or Scheduled Tribe;
o A victim of trafficking in human beings or begar as referred to in Article 23 of the Constitution;
o A woman or a child;
o A mentally ill or otherwise disabled person;
o A person under circumstances of undeserved want such as being a victim of a mass disaster, ethnic
violence, caste atrocity, flood, drought, earthquake or industrial disaster; or
o An industrial workman; or
o In custody, including custody in a protective home within the meaning of clause (g) of Section 2 of
the Immoral Traffic (Prevention) Act, 1956(104 of 1956); or in a juvenile home within the meaning of
clause(j) of Section 2 of the Juvenile Justice Act, 1986 (53 of 1986); or in a psychiatric hospital or
psychiatric nursing home within the meaning of clause (g) of Section 2 of the Mental Health Act,
1987(14 of 1987);or
o a person in receipt of annual income less than the amount mentioned in the following schedule (or any
other higher amount as may be prescribed by the State Government), if the case is before a Court
other than the Supreme Court, and less than Rs 5 Lakh, if the case is before the Supreme Court.
o A woman is entitled to free legal aid irrespective of her income or financial status. A woman is
eligible to apply for free legal aid by virtue of Section 12(c) of the Legal Services Authorities Act,
1987. Hence, statement 3 is correct.
o Senior citizens’ eligibility for free legal aid depends on the Rules framed by the respective State
Governments in this regard.
o A child is eligible for free legal aid till the age of majority i.e. 18 years. This is effectuated by Section
12 (c) of the Legal Services Authorities Act, 1987.

Q 96.A
• Right to Information Act, 2005:
o The RTI Act is a powerful tool that allows Indian citizens to access information held by public
authorities at the central and state government levels.
o The Right to Information Act has significantly contributed to promoting transparency, accountability,
and citizen participation in India's governance system.
o It enables citizens to exercise their right to know and hold public authorities accountable for their
actions and decisions.
• Private organizations as Public Authorities:
o According to a significant ruling by the Central Information Commission (CIC) in 2008, private
organizations that are involved in performing public duties, providing public services, or
utilizing public funds can be considered "public authorities" under the RTI Act. Hence
statement 1 is correct.
37 www.visionias.in ©Vision IAS

FREE BY KING R QUEEN P [ऋषभ राजपूत]


o This means that they are also subject to the obligations and requirements of the Act, including the
disclosure of information upon request. Enacting the RTI Act in India was a landmark decision for
several reasons:
• Disposal of request:
o In the normal course, information to an applicant shall be supplied within 30 days from the
receipt of the application by the public authority. If the information sought concerns the life or
liberty of a person, it shall be supplied within 48 hours. Hence statement 2 is not correct.
• Failure to dispose of Information:
o The applicant can appeal to the first appellate authority who is an officer senior in rank to the Public
Information Officer;
✓ If the applicant is not supplied information within the prescribed time of thirty days or 48
hours, as the case may be,
✓ If he is not satisfied with the information furnished to him.
o Such an appeal should be filed within a period of thirty days from the date on which the limit of 30
days of supply of information is expired or from the date on which the information or decision of the
Public Information Officer is received.
Q 97.A
• Sabang Port
o Sabang Port is located in the Aceh Province of Indonesia. It is roughly 700 km from the Andaman
and Nicobar Islands. Sabang is at the entrance of the Strait of Malacca. Malacca Straits is a narrow
stretch of the sea between Indonesia and Malaysia. The successful development of this vital port
would allow India easier access to the Malacca Straits. Hence pair 1 is not correctly matched.
• Ekatha Harbour
o ‘Ekatha Harbour’ is a harbour for the Coast Guard of the Maldives National Defence Force (MNDF).
It is located at Sifavaru in Uthuru Thila Falhu (UTF) atoll. The UTF Harbour Project is among the
most significant Indian grant-in-aid projects in the Maldives. At UTF atoll India is assisting in the
construction of a naval dockyard. Hence pair 2 is correctly matched.
• Chittagong Port
o The Chittagong or Chhatogram port is one of the important seaports of Bangladesh, located in the
region of Chittagong hill tracts. It is the main seaport of Bangladesh and is located on the bank of the
Karnaphuli River. The port of Chittagong was known as Shetgang as early as the 4th century BC, and
it received ships from the Middle East and China. Hence pair 3 is not correctly matched.
o Almost 90% of Bangladesh's exports and imports are now handled by the Chittagong Port. In the
colonial era, the Chittagong Port was one of the largest ports of eastern India through which cargo
was carried to the borders of Myanmar through railways and roadways.

Q 98.B
• The Parliament, under Article 327, enacted certain provisions, namely:
o Representation of People Act, 1950: The Act, provides for allocation of seats and delimitation of
constituencies of the Parliament and state legislatures, officers related to conduct of elections,
preparation of electoral rolls, and manner of filling seats in the Council of States allotted to Union
Territories. Hence, pair 1 is not correctly matched.
o The Representation of People Act, 1951: It provides for the conduct of elections of the Houses of
Parliament and to the House or Houses of the Legislature of each State, the qualifications and
disqualifications for membership of those Houses, the corrupt practices and other offences at or in
connection with such elections and the decision of doubts and disputes arising out of or in connection
with such elections. Hence, pair 2 is not correctly matched.
o The Delimitation Commission Act, 1952: It provides for the readjustment of seats, the
delimitation and reservation of territorial constituencies, and other related matters. Hence, pair 3
is correctly matched.
o The Presidential and Vice-Presidential Election Act of 1952: It provides for the conduct of
Presidential and vice presidential elections and mechanisms for the settlement of any dispute
arising out of such elections. Hence, pair 4 is correctly matched.

Q 99.B
• Section 26 of the RTI Act, 2005 states that the appropriate government may develop and organize
educational programmes to advance the understanding of the public towards the exercise of rights under
the Act. Hence option (b) is correct.

38 www.visionias.in ©Vision IAS

FREE BY KING R QUEEN P [ऋषभ राजपूत]


• Central Information Commission has been set up under the RTI Act, 2005, and hence, is a statutory
body. Hence option (a) is not correct.
• Official Secrets Act, 1923 is still in force, but the RTI Act contains provisions whereby in case of
inconsistency, the RTI Act is to prevail. Hence option (c) is not correct.
• The annual reports of CIC are submitted to the Central Government which places them before the Houses
of Parliament. Similarly, the State Information Commissions submit reports to the State Government
which are then placed before the State Legislature. Hence option (d) is not correct.

Q 100.B
• National Food Security Act, (NFSA) 2013 :
o The Act legally entitles upto 75% of the rural population and 50% of the urban population to
receive subsidized foodgrains under Targeted Public Distribution System.
o About two thirds of the population therefore is covered under the Act to receive highly subsidized
foodgrains.
o As a step towards women empowerment, the eldest woman of the household of age 18 years or above
is mandated to be the head of the household for the purpose of issuing of ration cards under the Act.
• Life-cycle approach:
o One of the guiding principles of the Act is its life-cycle approach wherein special provisions have
been made for pregnant women and lactating mothers and children in the age group of 6 months
to 14 years by entitling them to receive nutritious meal free of cost. Hence statement 2 is correct.
o The act provides them nutritious meal free of cost through a widespread network of Integrated Child
Development Services (ICDS) centres, called Anganwadi Centres under ICDS scheme and also
through schools under Mid-Day Meal (MDM) scheme.
o Higher nutritional norms have been prescribed for malnourished children upto 6 years of age.
o Pregnant women and lactating mothers are further entitled to receive cash maternity benefit of
not less than Rs. 6,000 to partly compensate for the wage loss during the period of pregnancy
and also to supplement nutrition. Hence statement 3 is correct.
• Coverage and Entitlement under the NFSA:
o Beneficiaries under the National Food Security Act fall into two categories: Priority Households
(PHH) and Antyodaya Anna Yojana (AAY) households.
o While AAY households, which constitute poorest of the poor are entitled to 35 kg of foodgrains
per family per month, irrespective of size of family and priority households are entitled to 5 kg per
person per month depending on the members of family. Hence statement 1 is not correct.

Copyright © by Vision IAS


All rights are reserved. No part of this document may be reproduced, stored in a retrieval system or
transmitted in any form or by any means, electronic, mechanical, photocopying, recording or otherwise,
without prior permission of Vision IAS.
39 www.visionias.in ©Vision IAS

FREE BY KING R QUEEN P [ऋषभ राजपूत]


VISIONIAS
www.visionias.in

Test Booklet Series

TEST BOOKLET

GENERAL STUDIES (P) 2024 – Test – 4138


C
Time Allowed: Two Hours Maximum Marks: 200

INSTRUCTIONS

1. IMMEDIATELY AFTER THE COMMENCEMENT OF THE EXAMINATION, YOU SHOULD CHECK THAT THIS BOOKLET
DOES NOT HAVE ANY UNPRINTED OR TURN OR MISSING PAGES OR ITEMS, ETC. IF SO, GET IT REPLACED BY A
COMPLETE TEST BOOKLET.

2. ENCODE CLEARLY THE TEST BOOKLET SERIES A, B, C OR D AS THE CASE MAY BE IN THE APPROPRIATE PLACE IN
THE ANSWER SHEET.

3. You have to enter your Roll Number on the Test Booklet in the Box
provided alongside. Do NOT write anything else on the Test Booklet.

4. This Test Booklet contains 100 items (Questions). Each item is printed in English. Each item comprises four
responses (answers). You will select the response which you want to mark on the Answer Sheet. In case you
feel that there is more than one correct response with you consider the best. In any case, choose ONLY ONE
response for each item.

5. You have to mark all your responses ONLY on the separate Answer Sheet provided. See direction in the
answers sheet.

6. All items carry equal marks. Attempt all items. Your total marks will depend only on the number of correct
responses marked by you in the answer sheet. For every incorrect response 1/3rdof the allotted marks will be
deducted.

7. Before you proceed to mark in the Answer sheet the response to various items in the Test booklet, you have to
fill in some particulars in the answer sheets as per instruction sent to you with your Admission Certificate.

8. After you have completed filling in all responses on the answer sheet and the examination has concluded, you
should hand over to Invigilator only the answer sheet. You are permitted to take away with you the Test
Booklet.

9. Sheet for rough work are appended in the Test Booklet at the end.

DO NOT OPEN THIS BOOKLET UNTIL YOU ARE ASKED TO DO SO


1 www.visionias.in ©Vision IAS

FREE BY KING R QUEEN P [ऋषभ राजपूत]


1. Slavery in India was abolished through 4. Consider the following statements:
which of the following legislations? Statement-I: The General Service Enlistment
(a) Queen's Proclamation of 1858 Act by Lord Canning's government caused
(b) Act V of 1843 resentment among sepoy recruits to the
(c) Charter Act of 1833 Bengal Army.

(d) Charter Act of 1853 Statement-II: The General Service


Enlistment Act removed the privilege of free

2. Consider the following statements regarding postal services enjoyed by the Indian

the Deoband school of Islamic theology: sepoys.


Which one of the following is correct in
1. It began as a revivalist movement with
respect of the above statements?
the aim of a moral and religious
(a) Both Statement-I and Statement-II are
regeneration of the Muslim community.
correct and Statement-II is the correct
2. It opposed the formation of the Indian
explanation for Statement-I
National Congress which it considered
(b) Both Statement-I and Statement-II are
to be a majoritarian party.
correct and Statement-II is not the
Which of the statements given above is/are
correct explanation for Statement-I
correct?
(c) Statement-I is correct but Statement-II is
(a) 1 only
incorrect
(b) 2 only
(d) Statement-I is incorrect but Statement-II
(c) Both 1 and 2
is correct
(d) Neither 1 nor 2

5. Which of the following events occurred


3. Consider the following statements with
during the office of Lord Curzon?
respect to the Treaty of Allahabad of 1765: 1. Setting up of Raleigh Commission to
1. Shah Alam II was to pay Rs 50 lakh to inquire into the conditions and prospects
the East India Company as war of universities in India.
indemnity. 2. Younghusband’s mission to Tibet to
2. Shuja-ud-Daula agreed to surrender check on increasing Russian influence.
Allahabad and Kara to the Britishers; 3. Setting up of Durand Commission to
3. Robert Clive annexed Awadh to expand define the Durand Line between India
the empire of British rule. and Afghanistan
How many of the statements given above are Select the correct answer using the code
correct? given below.
(a) Only one (a) 1 and 2 only
(b) Only two (b) 2 and 3 only
(c) Only three (c) 1 and 3 only

(d) None (d) 1, 2 and 3


2 www.visionias.in ©Vision IAS

FREE BY KING R QUEEN P [ऋषभ राजपूत]


6. Consider the following pairs: 9. Consider the following statements regarding
GI Product State Ramkrishna Paramhansa:
1. He sought salvation through traditional
1. Matty Banana : Kerala
ways of renunciation, meditation and
2. Mushkabudji Rice : Jammu and bhakti.
Kashmir 2. He recognized the fundamental oneness
3. Mankurad Mango : Maharashtra of all religions.
3. He influenced the understanding of
How many of the above pairs are correctly
Keshub Chandra Sen who in turn
matched? inculcated some of his ideas into
(a) Only one Brahmoism.
(b) Only two How many of the statements given above are
(c) All three correct?
(a) Only one
(d) None
(b) Only two
(c) All three
7. Which of the following statements is correct (d) None
regarding the Indian Councils Act, 1861?
(a) It created a legislative demarcation 10. Consider the following statements:
1. Genomics explores the complete genetic
between the central legislative council
information of a single organism only.
and provincial legislative councils. 2. Metagenomics explores a mixture of
(b) It gave the Viceroy the power to issue DNA from multiple organisms.
ordinances in case of emergency. Which of the statements given above is/are
(c) It simplified the job of the Secretary of correct?
(a) 1 only
state by creating a 15-member council of
(b) 2 only
India to assist him. (c) Both 1 and 2
(d) It rejected the legislative devolution of (d) Neither 1 nor 2
financial decentralization.
11. Consider the following statements:
1. It superseded the Indian League founded
8. Consider the following statements regarding
by Sisir Kumar Ghosh.
Pabna Agrarian Leagues: 2. It protested against the reduction of the
1. These were set up by the Britishers to age limit in 1877 for candidates of the
prevent the peasants from acquiring Indian Civil Service examination.
3. The membership fee was kept low in
occupancy rights under Act X of 1859.
order to attract the poorer sections to the
2. The method adopted by the leagues was
association.
to fight cases in the courts against the 4. Surendranath Banerjee and Ananda
occupancy rights. Mohan Bose were its prominent leaders.
Which of the statements given above is/are Which of the following organizations has
been described by the statements given
correct?
above?
(a) 1 only
(a) Indian National Congress
(b) 2 only (b) The Bombay Presidency Association
(c) Both 1 and 2 (c) The East India Association
(d) Neither 1 nor 2 (d) The Indian Association of Calcutta
3 www.visionias.in ©Vision IAS

FREE BY KING R QUEEN P [ऋषभ राजपूत]


12. Consider the following pairs: 15. Consider the following statements regarding
Political Association Member the device 'Nabhmitra' which was recently
1. United Patriotic : Syed Ahmad seen in the news:
Association Khan 1. It seeks to provide safety for the
fishermen who venture out into the sea.
2. Madras Mahajan : Panapakkam
2. It has been developed by the National
Sabha Anandacharlu
Institute of Ocean Technology.
3. The Bombay : Kashinath
Which of the statements given above is/are
Presidency Trimbak Telang correct?
Association (a) 1 only
How many of the above pairs are correctly (b) 2 only
matched? (c) Both 1 and 2
(a) Only one (d) Neither 1 nor 2
(b) Only two
(c) All three 16. North India's first River Rejuvenation
Project Devika is nearing completion. In this
(d) None
context, consider the following statements
regarding river Devika:
13. Consider the following statements with
1. It originates from the Mansarovar Lake.
respect to Haidar Ali of the Mysore:
2. It is a tributary of the Ravi River.
1. The First Anglo-Mysore War resulted in Which of the statements given above is/are
a thorough defeat of Haidar Ali. correct?
2. The military requirements of Haidar (a) 1 only
Ali's army were fulfilled by the French. (b) 2 only
3. He signed the Treaty of Mangalore when (c) Both 1 and 2
the second Anglo-Mysore war ended (d) Neither 1 nor 2
without any positive outcome.
17. 'Belem Declaration’ sometimes seen in the
How many of the statements given above are
news is associated with the:
correct?
(a) Freedom of the press
(a) Only one
(b) Women rights
(b) Only two (c) Neglected Tropical Diseases
(c) All three (d) Amazon rainforests
(d) None
18. Consider the following statements regarding
14. Consider the following personalities the Wahabi Movement:
regarding the Sikh reform movement: 1. Its leaders were sentenced for life
1. Baba Ram Singh imprisonment in Malda and Rajmahal
trials of 1870.
2. Baba Dayal Das
2. In the wake of 1857 revolt, it turned into
3. Baba Darbar Singh
armed resistance against British.
How many of the above given personalities
Which of the statements given above is/are
are related to Namdhari movement? correct?
(a) Only one (a) 1 only
(b) Only two (b) 2 only
(c) All three (c) Both 1 and 2
(d) None (d) Neither 1 nor 2
4 www.visionias.in ©Vision IAS

FREE BY KING R QUEEN P [ऋषभ राजपूत]


19. Consider the following statements: 21. Consider the following:
Statement-I: The famous triangular trade that 1. Atrocities under the Permanent

the East India Company carried out involved settlement system of land revenue.
2. Raising the land revenue by 50 percent.
Britain, India, and China.
3. Passage of Deccan Agriculturists Relief
Statement-II: India was involved in the
Act (1879)
triangular trade as the goods produced in the
How many of the above are the reasons
West (Britain) did not find a market in behind the Deccan riots?
China. (a) Only one
Which one of the following is correct in (b) Only two

respect of the above statements? (c) All three


(d) None
(a) Both Statement-I and Statement-II are
correct and Statement-II is the correct
22. Consider the following statements regarding
explanation for Statement-I
French governor Joseph Francis Dupleix:
(b) Both Statement-I and Statement-II are 1. He is often credited as the originator of
correct and Statement-II is not the the practice of a subsidiary alliance in
correct explanation for Statement for India.

Statement-I 2. His first appointment in India was as


Director-General of French colonies.
(c) Statement-I is correct but Statement-II is
3. During the war of succession for the
incorrect
throne of Hyderabad, he supported
(d) Statement-I is incorrect but Statement-II
Muzaffar Jung.
is correct How many of the statements given above are
correct?
20. Consider the following pairs: (a) Only one

Published Reformer (b) Only two


(c) All three
work
(d) None
1. Tuhfat-ul : Hasrat Mohani
Muwahhidin
23. Consider the following pairs:
2. Shatpatre : Gopal Hari Deshmukh Hydropower project State
3. Tahzib al- : Sir Syed Ahmed Khan 1. Shanan : Himachal Pradesh
akhlaq 2. Mekedatu : Tamil Nadu
How many of the pairs given above are 3. Kalasa Banduri : Goa
How many of the above pairs are correctly
correctly matched?
matched?
(a) Only one
(a) Only one
(b) Only two
(b) Only two
(c) All three (c) Only three
(d) None (d) None
5 www.visionias.in ©Vision IAS

FREE BY KING R QUEEN P [ऋषभ राजपूत]


24. Kharwar rebellion in Bihar in 1870s was a 28. Consider the following statements regarding
protest against: the Theosophical Movement:
(a) revenue settlement activities. 1. Colonel M.S. Olcott of the U.S. Army
(b) forced labour. laid the foundation of the movement in
(c) Hindu-Muslim conflicts. the United States in 1875.
(d) oppression of the moneylenders. 2. The Theosophical Society rejected the
Hindu beliefs in reincarnation and
25. Consider the following statements: karma.
1. Oort cloud is a giant spherical shell that 3. It drew inspiration from the philosophy
mostly surrounds the Sun. of the Samkhya, Yoga and Vedanta
2. Kuiper Belt is the collection of millions schools of thought.
of icy objects. How many of the statements given above are
Which of the statements given above is/are correct?
correct? (a) Only one
(a) 1 only (b) Only two
(b) 2 only (c) All three
(c) Both 1 and 2 (d) None
(d) Neither 1 nor 2
29. Consider the following statements regarding
26. Consider the following statements regarding the Act for Better Government for India,
the Finger Minutiae Record – Finger Image 1858:
Record (FMR-FIR) modality: 1. It transferred the power to govern India
1. It is an Artificial Intelligence technology from the East India Company to the
used to prevent fraud in the Aadhaar- Viceroy.
enabled Payment System. 2. Under the act, India was to be governed
2. This technology helps detect cloned through a portfolio system which was
fingerprints, ensuring that only real answerable to the Viceroy.
fingerprints are used for verification. Which of the statements given above is/are
Which of the statements given above is/are not correct?
correct? (a) 1 only
(a) 1 only (b) 2 only
(b) 2 only (c) Both 1 and 2
(c) Both 1 and 2 (d) Neither 1 nor 2
(d) Neither 1 nor 2
30. Which of the following statements is correct
27. Consider the following statements regarding regarding the Brahmo Samaj?
Namda Art: (a) Rammohan Roy founded the Brahmo
1. It involves making a rug or carpet which Samaj which was later renamed as
is primarily made of pure sheep wool. Brahmo Sabha.
2. It originated and is prevalent in the state (b) There was no place for priesthood in the
of Sikkim. Samaj nor sacrifices of any kind were
Which of the statements given above is/are allowed.
correct? (c) Readings of Upanishads and Vedas were
(a) 1 only prohibited in the Brahmo Samaj.
(b) 2 only (d) In 1866, Keshab Chandra Sen left the
(c) Both 1 and 2 Brahmo Samaj and formed Adi Brahmo
(d) Neither 1 nor 2 Samaj.
6 www.visionias.in ©Vision IAS

FREE BY KING R QUEEN P [ऋषभ राजपूत]


31. Who among the following was the first 34. Consider the following statements regarding
Indian journalist to be imprisoned for the the Bio-Trace Minerals Project:
contempt of court by the British? 1. It aims to enhance livestock productivity
(a) Sisir Kumar Ghosh and optimize feed and fodder resources.
(b) Bal Gangadar Tilak 2. The project introduces a proprietary
(c) Surendranath Banerjee
nutrient that facilitates chelation
(d) G. Subramania Aiyar
reactions under milder conditions.
3. MinBioZen is a product that addresses
32. With reference to the legislations passed for
the need for bio trace minerals in
improvement in conditions of women during
the British period, consider the following livestock.
statements: How many of the above statements are
1. The regulation of 1829 declared the correct?
practice of sati illegal and punishable by (a) Only one
criminal courts as culpable homicide. (b) Only two
2. The Bengal regulations of 1795 and (c) All three
1804 declared infanticide illegal and (d) None
equivalent to murder.
3. The Age of Consent Act 1891 forbade 35. With reference to Indian history, which of
the marriage of girls below the age of
the following happened earliest?
12.
(a) Balshastri Jambhekar started the
How many of the above statements are
newspaper named ‘Darpan’.
correct?
(b) Dayal Singh College was opened in
(a) Only one
(b) Only two Lahore to promote the ideas of Brahmo
(c) All three Samaj.
(d) None (c) Karsondas Mulji who started the Satya
Prakash in Gujarati to advocate widow
33. With reference to the economic critique of remarriage.
the British rule by the early nationalists, (d) Age of Consent Act was passed because
consider the following statements: of the efforts of Behramji Malabari.
1. They clearly understood that British
colonialism was the main obstacle to
36. With reference to the Bethune School,
India’s economic development.
consider the following statements:
2. They saw modern Industries as a major
1. It was first founded in Madras
force to unite the diverse people of
presidency in 1849.
India.
3. They saw foreign capital as a welcome 2. Ishwar Chandra Vidyasagar was
addition to the efforts of associated with the school as its
Industrialization of India. Secretary.
How many of the statements given above are Which of the statements given above is/are
correct? correct?
(a) Only one (a) 1 only
(b) Only two (b) 2 only
(c) All three (c) Both 1 and 2
(d) None (d) Neither 1 nor 2
7 www.visionias.in ©Vision IAS

FREE BY KING R QUEEN P [ऋषभ राजपूत]


37. Consider the following pairs: 39. With reference to the 'PM Vishwakarma'

Island in News Location scheme consider the following statements:


1. It aims to support traditional artisans and
1. Triton Island : Mediterranean Sea
craftspeople in rural and urban India.
2. Maui Island : Pacific Ocean 2. It is launched by the Ministry of Micro,
3. Timor Island : Atlantic Ocean Small & Medium Enterprises.
3. It will provide total credit support of
How many of the pairs given above are
Rs.3 lakhs at a concessional interest rate
correctly matched?
of 5%.
(a) Only one How many of the above statements are
(b) Only two correct?
(a) Only one
(c) All three
(b) Only two
(d) None (c) All three
(d) None
38. Consider the following statements with
40. Consider the following statements regarding
respect to the labour legislation during
the Singh Sabha movement:
British rule: 1. The Singh Sabha movement took
1. The first-ever demand for regulation of forward the reform activities of the
Brahmo Samaj in Punjab.
the condition of workers in factories in
2. The Sabha established a network of
India came from the Lancashire textile
Khalsa schools throughout Punjab.
capitalist lobby. Which of the statements given above is/are
2. Indian Factory Act, of 1881 reduced the correct?
(a) 1 only
maximum working hours for children to
(b) 2 only
7 hours a day. (c) Both 1 and 2
3. Labour Laws covered all the factories (d) Neither 1 nor 2
along with British-owned tea and coffee
41. Arrange the following kingdoms/states in the
plantations.
chronological order of their capture by the
4. Despite having provisions against child use of the infamous Doctrine of Lapse.
labour moderates did not support the 1. Udaipur

Factory Acts of 1881. 2. Satara


3. Jhansi
How many of the statements given above are
4. Sambalpur
correct? Select the correct answer using the code
(a) Only one given below.
(a) 2-1-3-4
(b) Only two
(b) 2-4-1-3
(c) Only three
(c) 3-2-4-1
(d) All four (d) 3-1-2-4
8 www.visionias.in ©Vision IAS

FREE BY KING R QUEEN P [ऋषभ राजपूत]


42. Singhpo's tribal uprising took place in which 46. With reference to Indian history, the
of the following states of British India? ‘Bibighar massacre’ is related to:
(a) Punjab (a) The death of Britishers put in prison by
(b) Bombay the Bengal Nawab
(c) Assam (b) The destruction of Delhi carried out by
(d) Madras
Nadir Shah
(c) Killngs of English occupants near
43. Consider the following statements:
Kanpur by the sepoys of Nanasaheb
Statement-I: Lord Canning was made the
Peshwa
first Viceroy of India due to the change in
(d) The arson of Police station in Chauri
the power-structure under the act of 1858.
Statement-II: The Act of 1858, transferred Chaura in 1922

power from the East India Company to the


British Crown. 47. Consider the following statements in respect
Which one of the following is correct with of the Consultative Committees:
respect to the above statements? 1. They are constituted on an yearly basis.
(a) Both Statement-I and Statement-II are 2. The maximum membership of a
correct, and Statement-II is the correct Consultative Committee should be
explanation for Statement-I. limited to forty.
(b) Both Statement-I and Statement-II are 3. There is the requirement for the presence
correct, and Statement-II is not the
of a minimum number of Members to
correct explanation for Statement-I.
constitute the quorum for holding
(c) Statement-I correct, but Statement-II is
meetings.
incorrect.
How many of the statements given above are
(d) Statement-I is incorrect, but Statement-II
is correct. correct?
(a) Only one
44. Who among the following has published (b) Only two
'Trutiya Ratna' (1855) which is based on the (c) All three
exploitation of shudras? (d) None
(a) Jyotibha Phule
(b) Sri Narayana Guru 48. Consider the following committees
(c) B.R. Ambedkar constituted during British Rule in India:
(d) C.N. Annadurai 1. General Committee of Public Instruction
2. Hunter Commission
45. Under which of the following legislations,
3. Saddler Committee
the East India Company’s territories in India
How many of the above committees had a
were termed ‘British Possessions’ for the
bearing on education?
first time?
(a) Only one
(a) Charter Act of 1813
(b) Charter Act of 1853 (b) Only two

(c) Pitt’s India Act of 1784 (c) All three


(d) Regulating Act of 1773 (d) None
9 www.visionias.in ©Vision IAS

FREE BY KING R QUEEN P [ऋषभ राजपूत]


49. Consider the following pairs: 52. The Limitation Law passed by the British in
Centre of 1857 Suppressed by
1859 aimed to
Revolt British Officer
1. Delhi : James Outram (a) prevent the accumulation of interests
2. Benaras : Hugh Rose over loans taken by the ryots.
3. Kanpur : John Nicholson
(b) fix the quantity of crop produce to be
How many of the above pairs are correctly
matched? shared by sharecroppers with jotedars.
(a) Only one (c) limit the size of landholdings for which
(b) Only two
(c) All three zamindari rights were granted to
(d) None landlords.

(d) fix the amount of raw cotton to be sold


50. With reference to Trans Lunar Injection,
consider the following statements: by ryots in the open markets.
1. It is used to launch a spacecraft from
Earth's orbit to place it on a track beyond
53. Consider the following statements with
the Moon.
2. It is performed when the spacecraft is at respect to the Treaty of Salbai:
the closest point to Earth.
1. The Treaty of Salbai was signed to
3. It takes place before the spacecraft
enters the lunar orbit. conclude the first Anglo-Maratha war.
How many of the statements given above are 2. As per the treaty, Salsette was to go into
correct?
the possession of the Marathas.
(a) Only one
(b) Only two 3. The treaty was signed between Warren
(c) All three Hastings and Mahadaji Shinde.
(d) None
How many of the statements given above
51. Born in 1845 in Bombay, this particular is/are correct?
leader came under Dadabhai Naoroji’s
(a) Only one
influence while studying law in London
during the 1860s. He was one of the (b) Only two
founders of the Bombay Presidency (c) Only three
Association as also the Indian National
(d) None
Congress. He along with G.K. Gokhale, and
D.A. Khare organized the first-ever walk-out
in Indian legislative history to oppose a
54. Who among the following started the journal
Bombay legislative Council Bill.
Which of the following leaders is described 'Asiatick Researches'?
in the passage given above? (a) William Carey
(a) Dinshaw Wacha
(b) Charles Wilkins
(b) R.M. Sayani
(c) Bhau Daji Lad (c) Colin Mackenzie
(d) Pherozeshah Mehta (d) William Jones
10 www.visionias.in ©Vision IAS

FREE BY KING R QUEEN P [ऋषभ राजपूत]


55. Consider the following statements regarding 57. Consider the following statements regarding
the Charter Act of 1833: the Indian Universities Act of 1904:
1. The act provided for the appointment of 1. It was introduced by Lord Curzon to
improve the condition of university
a Law Commission to study, collect and
education in India.
codify various rules and regulations
2. It was passed on the recommendations
present in India. of Raleigh commission.
2. The Act added one member to the 3. The Act introduced governmental
Executive Council of the Governor control over universities.
General as a Law Member with the right How many of the statements given above are
correct?
to vote.
(a) Only one
3. The Act had an anti-discrimination
(b) Only two
clause which became a cause for (c) All three
political agitation in India. (d) None
How many of the statements given above are
correct? 58. Consider the following pairs:
(a) Only one Battle Fought between
1. Battle of : British and Portuguese
(b) Only two
Bedara
(c) All three
2. Battle of : British and French
(d) None Wandiwash
3. Battle of St. : French and Dutch
56. Consider the following statements: Thome
Statement-I: The British, by and large, were How many pairs given above are correctly
matched?
not in support of the jhum cultivation
(a) Only one
practiced by the tribals.
(b) Only two
Statement-II: Verrier Elwin, a missionary (c) All three
who came to India in 1927, argued that Jhum (d) None
cultivation should be banned.
Which one of the following is correct with 59. Consider the following statements with
respect to the above statements? respect to Sindh:
1. In 1758, an English factory was built at
(a) Both Statement-I and Statement-II are
Thatta, owing to a parwana given by the
correct, and Statement-II is the correct
Kallora prince, Ghulam Shah.
explanation for Statement-I. 2. In 1783, the Talpuras, under the
(b) Both Statement-I and Statement-II are leadership of Mir Fatah Ali Khan,
correct, and Statement-II is not the established a complete hold over Sindh.
correct explanation for Statement-I. Which of the statements given above is/are
correct?
(c) Statement-I correct, but Statement-II is
(a) 1 only
incorrect.
(b) 2 only
(d) Statement-I is incorrect, but Statement-II (c) Both 1 and 2
is correct. (d) Neither 1 nor 2
11 www.visionias.in ©Vision IAS

FREE BY KING R QUEEN P [ऋषभ राजपूत]


60. Consider the following statements with 63. Consider the following statements regarding
respect to the Mughal Empire: the Regulating Act of 1773:
1. The act was brought in because the King
1. Khafi Khan gave the title of Shah-i-
of England wanted to control the
Bekhabar to Bahadur Shah.
operations of the company.
2. Farrukhsiyar followed a policy of 2. Under the act, the Governor-General had
religious tolerance by abolishing Jaziya an overriding power over his council.
and the pilgrimage tax. 3. The Act also provided for the
establishment of a Supreme Court of
3. The Battle of Karnal was fought during
Justice at Calcutta to give justice to
the rule of Muhammad Shah. Europeans, their employees, and the
How many of the statements given above are citizens of Calcutta.
correct? How many of the statements given above are
correct?
(a) Only one
(a) Only one
(b) Only two
(b) Only two
(c) All three (c) All three
(d) None (d) None

64. Tarabai Shinde, a woman educated at home


61. Consider the following statements regarding
at Poona, published a book, criticising that in
Paika Rebellion of 1817: a new colonial society men enjoyed all the
1. It was associated with the present-day rights, opportunities and benefits of change,
states of Bihar and Bengal. while women were blamed for all the evils
and were still bound by the old strictures
2. It began as a protest against heavy land
of pativrata (duty to husband). Which of the
taxes imposed by British colonial rulers.
following is the title of that book?
3. It was led by tribal leaders who sought (a) The History of Doing
to establish an independent tribal (b) Ratanbai
kingdom. (c) Stripurushtulna
(d) Deep Nirban
How many of the above statements are
correct? 65. With reference to Graphene-Aurora
(a) Only one Program, consider the following statements:
(b) Only two 1. It has been launched by the Ministry of
Electronics & Information Technology.
(c) All three
2. It aims to fill the gap between graphene
(d) None
research and commercialization by
providing facilities to startups and
62. Which tribal leader was associated with the industries.
"Hul Rebellion" in Chotanagpur in 1856? Which of the statements given above is/are
correct?
(a) Rani Gaidinliu
(a) 1 only
(b) Birsa Munda (b) 2 only
(c) Tantia Bhil (c) Both 1 and 2
(d) Sidhu Murmu (d) Neither 1 nor 2
12 www.visionias.in ©Vision IAS

FREE BY KING R QUEEN P [ऋषभ राजपूत]


66. With reference to French colonial activities 69. With reference to the South Indian Liberal
in India, which of the following is correct? Federation, consider the following
(a) Treaty of Ryswick concluded in statements:
September 1697 resulted in the loss of
1. It was the first non-Brahamanical
Pondicherry for France.
organisation founded by T.M. Nair.
(b) The Spanish succession war gave the
2. It was renamed as Sree Narayana
French upper hand against the
Portuguese. Dharma Paripalana Yogam by
(c) Balasore and Qasim Bazar were Ramaswami Naicker.
important trading centres of the French. Which of the statements given above is/are
(d) Nawab Alivardi Khan gave the French correct?
permission to establish a township near (a) 1 only
Calcutta for the first time.
(b) 2 only
(c) Both 1 and 2
67. With reference to the working class
movements, consider the following (d) Neither 1 nor 2
statements:
1. Indian Working class got complete 70. Consider the following pairs:
support from native capitalist classes to Places in news Country
raise voice against exploitation by 1. Lachin : Argentina
foreign employers.
2. Jeddah : Saudi Arabia
2. First Factory Act 1881 prohibited the
3. Kuril Island : Canada
employment of children under the age of
How many of the above pairs are correctly
16.
Which of the statements given above is/are matched?
correct? (a) Only one
(a) 1 only (b) Only two
(b) 2 only (c) All three
(c) Both 1 and 2 (d) None
(d) Neither 1 nor 2

71. With respect to Indian history, the Pledge


68. Consider the following statements with
reference to the WorldCoin Project: Movement relates to:
1. It aims to create a new form of digital (a) the agitation against the Cunningham
identity that can verify that its holder is a circular which pledged to ban the
real human being. student political activities.
2. It will be used to generate a World ID by (b) the series of pledges taken during the
scanning a person's eyes using a device
Swadeshi movement to oppose western
called orb.
textile.
Which of the statements given above is/are
(c) a movement launched against child
correct?
(a) 1 only marriage by Indian Social Conference.
(b) 2 only (d) anti-liquor movement began during the
(c) Both 1 and 2 constructive period of Gandhian
(d) Neither 1 nor 2 movement.
13 www.visionias.in ©Vision IAS

FREE BY KING R QUEEN P [ऋषभ राजपूत]


72. With reference to Sahitya Akademi Awards, 75. Consider the following statements regarding
consider the following statements: the Swami Dayanand Saraswati:
1. This award is given for literary works 1. His motto was ‘Go back to the Vedas’.
2. He accepted the doctrine of karma, but
only in the languages mentioned in the
rejected the theory of niyati (destiny).
eighth schedule of the Indian
3. He rejected that man's soul is merely a
constitution. part of God and held that God, soul and
2. This is the highest literary award given matter (prakriti) were distinct and eternal
by the Government of India. entities.
Which of the statements given above is/are How many of the statements given above are
correct? correct?
(a) Only one
(a) 1 only
(b) Only two
(b) 2 only
(c) All three
(c) Both 1 and 2 (d) None
(d) Neither 1 nor 2
76. Consider the following statements about
73. In the context of British India, which of the Lord Wellesley:
following statements is correct about the 1. He was responsible for the establishment
of Fort William College at Calcutta.
policy of masterly inactivity?
2. He participated in the Battle of
(a) It referred to the focus of the first few
Seringapatnam as a Colonel.
Governor Generals that was limited to 3. He served as governor of Madras and
the region of Bengal. later also served as Governor General of
(b) It was the lack of disinterest of the Bengal.
British Empire in solving the How many of the above statements are
correct?
developmental challenges of the north-
(a) Only one
east India.
(b) Only two
(c) It referred to the aggressive expansion (c) All three
policy followed by Lord Cornwallis (d) None
towards the state of Mysore.
(d) It was a foreign policy followed by John 77. Consider the following statements with
Lawrence to deal with the Afghan issue. reference to Portuguese activities in India:
1. Portuguese sent Jesuit priests to Shah
Jahan’s court and were able to obtain his
74. Consider the following pairs:
favor and support.
Places in News Country 2. They made money through both trade
1. Chashma : Pakistan and slave capture in India.
2. Arkadag : Kazakhstan 3. One of the reasons for the decline of the
3. Vladivostok : Ukraine Portuguese in India was their
How many of the above pairs are correctly involvement in piracy.
How many of the statements given above are
matched?
correct?
(a) Only one pair
(a) Only one
(b) Only two pairs (b) Only two
(c) All three pairs (c) All three
(d) None of the pairs (d) None
14 www.visionias.in ©Vision IAS

FREE BY KING R QUEEN P [ऋषभ राजपूत]


78. Recently seen in the news "Havana 81. Which of the following is not the
Syndrome" is a/an: achievement of the moderate phase of the
(a) genetic disorder. Indian National Congress?
(b) chronic respiratory illness. (a) They exposed the basic exploitative
(c) overpopulation of space with objects and character of the British rule.
debris. (b) They widened their democratic base
(d) problem of feeling ill after hearing through increase in scope of their
strange sounds. demands.
(c) They trained people in political work
79. Consider the following statements in the
through popularization of modern ideas.
context of Composite Quantum Material:
(d) They created a feeling of a common
1. A composite quantum material combines
nation to rally against a common enemy.
two different quantum properties in one
substance.
82. Consider the following Governor Generals
2. Rashba splitting is a quantum
of India:
phenomenon where electrons separate
1. Lord Auckland
due to interactions with an electric field.
2. Lord Metcalfe
3. Spintronics is a technology that exploits
3. Lord Bentinck
the intrinsic spin of electrons alongside
their charge for innovative electronic Arrange them chronologically in the order of

devices. their terms.

How many of the statements given above are (a) 1-2-3


correct? (b) 3-2-1
(a) Only one (c) 1-3-2
(b) Only two (d) 2-3-1
(c) All three
(d) None 83. In the context of tribal movements in India,
the term 'diku' refers to:
80. 'Consider the following statements regarding (a) non-tribal foreigners who often held
'Operation Duck Hunt' which was recently positions of authority and exploited
seen in the news: tribal communities.
1. It is aimed at preventing wildlife (b) the traditional system stipulating an
trafficking. equal sharing of the net produce of the
2. It was led by International Union for land by the jenmi, the Kanamdar, and
Conservation of Nature (IUCN).
the cultivator.
Which of the statements given above is/are
(c) the traditional tribal leadership structure
correct?
in which the eldest member of the tribe
(a) 1 only
held the highest authority.
(b) 2 only
(d) a land document given by the
(c) Both 1 and 2
government to provide cultivation rights.
(d) Neither 1 nor 2
15 www.visionias.in ©Vision IAS

FREE BY KING R QUEEN P [ऋषभ राजपूत]


84. Consider the following statements: 87. With reference to the Sree Narayana Dharma
1. In the nineteenth century, Christian Paripalana Yogam (SNDP), consider the
missionaries in India gave support for
following statements:
practical education among people.
1. It was established in Madras by Sree
2. The christian missionaries set up a
Narayana Guru Swamy.
mission at Serampore to control the
activities of the East India company. 2. This organiization built a number of

Which of the statements given above is/are temples that were opened only to the
correct? Ezhava caste to counter Brahmanical
(a) 1 only domination.
(b) 2 only
Which of the statements given above is/are
(c) Both 1 and 2
not correct?
(d) Neither 1 nor 2
(a) 1 only

85. Consider the following statements in respect (b) 2 only


of the Urea Gold: (c) Both 1 and 2
1. It is a kind of urea that is fortified with (d) Neither 1 nor 2
sulphur.
2. It ensures a gradual release of Nitrogen.
88. It was the first organized strike by any
Which of the statements given above is/are
section of the working class. The demands
correct?
(a) 1 only related to wages, hours of work and other
(b) 2 only conditions of service were raised. Almost all
(c) Both 1 and 2 nationalist newspapers came out fully in
(d) Neither 1 nor 2 support of the strike, with Tilak’s
newspapers Mahratta and Kesari
86. Consider the following statements regarding
campaigning for it for months. Public
the Moplahs (or Mappilas) uprising:
1. The Moplahs (or Mappilas) were the meetings and fund collections in aid of the

descendants of Arab traders who had strikers were organized in Bombay and
settled in the Malabar region. Bengal by prominent nationalists like
2. The Moplahs' main targets were the Pherozeshah Mehta, D.E. Wacha and
Britishers who deprived them of their
Surendranath Tagore.
customary right to cultivate the land.
Which of the following is being described in
Which of the statements given above is/are
the passage given above?
correct?
(a) 1 only (a) Ahmedabad mill strike
(b) 2 only (b) Great Indian Peninsular Railway strike
(c) Both 1 and 2 (c) Bombay textile mill strike
(d) Neither 1 nor 2 (d) Calcutta Jute mill strike
16 www.visionias.in ©Vision IAS

FREE BY KING R QUEEN P [ऋषभ राजपूत]


89. Consider the following statements regarding 92. Consider the following pairs:
the Indigo revolt (1859-60): Maratha Confederacy Region
1. The reason for the revolt was the 1. Peshwa : Nagpur
forceful growing of indigo without any 2. Scindia : Gwalior
increase in wages. 3. Gaekwad : Indore
2. The Christian missionaries supported the
4. Holkar : Baroda
Britishers and used religion to force
5. Bhonsle : Pune
peasants to grow indigo.
How many of the pairs given above are
3. Indigo Commission set up to enquire
correctly matched?
into the excesses during the revolt
favoured the peasants. (a) Only one
How many of the statements given above are (b) Only three
correct? (c) Only five
(a) Only one (d) None
(b) Only two
(c) All three 93. With reference to the widow remarriage in
(d) None India, consider the following statements:
1. Hindu Widows' Remarriage act 1856
90. With reference to Indian History, William was passed by British government to
Adam’s reports are related to:
suppress demands of legalising widow
(a) Decentralization under Government of
remarriages in India.
India Act, 1935
2. In Bengal, widow remarriage reform
(b) Secret Service reports on the
achieved huge success.
underground activities during Quit India
movement Which of the statements given above is/are
(c) Reports on education in British correct?
provinces (a) 1 only
(d) Reports on economic relations between (b) 2 only
Britain and its colonies in India (c) Both 1 and 2
(d) Neither 1 nor 2
91. With reference to the policy development of
education in India under East India 94. With reference to Einstein Cross, consider
Company, consider the following
the following statements:
statements:
1. It is located in the constellation Pegasus.
1. Utilitarians preferred English as a
2. The cross-like pattern is created by the
medium of education as more suited to
four bright images of the quasar around
the demands of the administration.
2. Orientalists thought Indian learning to be the lensing galaxy.
substandard and hence favored the 3. It is explained with the help of Einstein's
English language over vernaculars. theory of the photoelectric effect.
Which of the statements given above is/are How many of the statements given above are
correct? correct?
(a) 1 only (a) Only one
(b) 2 only (b) Only two
(c) Both 1 and 2 (c) All three
(d) Neither 1 nor 2 (d) None
17 www.visionias.in ©Vision IAS

FREE BY KING R QUEEN P [ऋषभ राजपूत]


95. Who among the following started the Satya 99. Consider the following bird species:
Prakash weekly to advocate widow 1. Bugun Liocichla
remarriage in India?
2. Himalayan Quail
(a) Dhondo Keshav Karve
3. Jerdon’s Courser
(b) Jagganath Shankar Seth
(c) Vishnu Shastri Pandit How many of the above birds are classified
(d) Karsondas Mulji as critically endangered?
(a) Only one
96. The compulsory begar system was
(b) Only two
abolished, and the Tenancy Act of 1903 was
(c) All three
introduced as a result of which of the
following movement? (d) None
(a) Birsa Munda Revolt
(b) Tana Bhagat Movement 100. He was an important leader of the 1857
(c) Bastar Revolt
revolt. Educated in Hyderabad, he became a
(d) The Santhal Rebellions
preacher when young. In 1856, he was seen
97. Nizam-ul-Mulk Asaf Jah founded the moving from village to village preaching
independent state of Hyderabad during the religious war against the British and urging
reign of which of the Mughal emperors? people to rebel. He was popularly called
(a) Muhammad Shah
Danka Shah. He was elected by the mutinous
(b) Farrukh Siyar
22nd Native Infantry as their leader. He
(c) Ahmad Shah Bahadur
(d) Jahandar Shah fought in the famous Battle of Chinhat in
which the British forces under Henry
98. Consider the following statements with Lawrence were defeated.
respect to the Anglo-Sikh Wars:
Who among the following is best described
1. The Battle of Gujrat was an important
by the passage given above?
battle fought as a part of the first Anglo-
Sikh war. (a) Ahmadullah Shah
2. Under the Treaty of Lahore, Daleep (b) Vasudev Balwant Phadke
Singh was recognized as the ruler with (c) Birjis Qader
Rani Jindan as regent.
(d) Bakht Khan
3. The second Anglo-Sikh war took place
during the Governor-Generalship of
Lord Canning.
How many of the statements given above are
correct?
(a) Only one
(b) Only two
(c) All three
(d) None

Copyright © by Vision IAS


All rights are reserved. No part of this document may be reproduced, stored in a retrieval system or
transmitted in any form or by any means, electronic, mechanical, photocopying, recording or otherwise,
without prior permission of Vision IAS.

18 www.visionias.in ©Vision IAS

FREE BY KING R QUEEN P [ऋषभ राजपूत]


VISIONIAS
www.visionias.in

Test Booklet Series

TEST BOOKLET

GENERAL STUDIES (P) 2024 – Test – 4139


C
Time Allowed: Two Hours Maximum Marks: 200

INSTRUCTIONS

1. IMMEDIATELY AFTER THE COMMENCEMENT OF THE EXAMINATION, YOU SHOULD CHECK THAT THIS BOOKLET
DOES NOT HAVE ANY UNPRINTED OR TURN OR MISSING PAGES OR ITEMS, ETC. IF SO, GET IT REPLACED BY A
COMPLETE TEST BOOKLET.

2. ENCODE CLEARLY THE TEST BOOKLET SERIES A, B, C OR D AS THE CASE MAY BE IN THE APPROPRIATE PLACE IN
THE ANSWER SHEET.

3. You have to enter your Roll Number on the Test Booklet in the Box
provided alongside. Do NOT write anything else on the Test Booklet.

4. This Test Booklet contains 100 items (Questions). Each item is printed in English & Hindi. Each item comprises
four responses (answers). You will select the response which you want to mark on the Answer Sheet. In case
you feel that there is more than one correct response with you consider the best. In any case, choose ONLY
ONE response for each item.

5. You have to mark all your responses ONLY on the separate Answer Sheet provided. See direction in the
answers sheet.

6. All items carry equal marks. Attempt all items. Your total marks will depend only on the number of correct
responses marked by you in the answer sheet. For every incorrect response 1/3rdof the allotted marks will be
deducted.

7. Before you proceed to mark in the Answer sheet the response to various items in the Test booklet, you have to
fill in some particulars in the answer sheets as per instruction sent to you with your Admission Certificate.

8. After you have completed filling in all responses on the answer sheet and the examination has concluded, you
should hand over to Invigilator only the answer sheet. You are permitted to take away with you the Test
Booklet.

9. Sheet for rough work are appended in the Test Booklet at the end.

DO NOT OPEN THIS BOOKLET UNTIL YOU ARE ASKED TO DO SO


1 www.visionias.in ©Vision IAS

FREE BY KING R QUEEN P [ऋषभ राजपूत]


1. Consider the following statements regarding 3. Consider the following statements regarding

the All India Muslim League: Gandhian years in Africa:


1. It was formed during the tenure of 1. He organized the Passive Resistance
Viceroy Curzon.
Association against the restrictions
2. Nawab Mohsin-uI-Mulk and Aga Khan
placed on Indian immigration.
were its founding members.
2. To sustain the movement in Africa, both
3. Mohammad Ali Jinnah joined the
the Indian National Congress and the
Muslim League after the abrupt end of
Muslim League supported with funds.
the Non-Cooperation movement.
3. Lord Hardinge, the Viceroy of India,
How many of the statements given above are

correct? actively participated in the negotiations

(a) Only one between Gandhiji and the government of

(b) Only two South Africa.

(c) All three How many of the statements mentioned


(d) None above are correct?

(a) Only one


2. With respect to the Communal Award as
(b) Only two
announced in 1932 by British Prime
(c) All three
Minister, Ramsay MacDonald, consider the
(d) None
following statements:
1. It was based on the findings of the
4. In the context of power sector in India, the
Indian Franchise Committee or Lothian
term 'market coupling' refers to a:
Committee.

2. It granted separate electorates to (a) process where discoms have the freedom

Muslims, Europeans, Sikhs, Indian to issue green bonds in capital markets.

Christians, Anglo-Indians and women. (b) method to discover a uniform market

3. It accorded the status of minority to the clearing price from all the power
depressed classes. exchanges in the country.
Which of the statements given above is/are (c) process of integrating grids of solar
correct?
energy plants with thermal power plants.
(a) 1 and 2 only
(d) method of increasing plant load capacity
(b) 2 only
utilisation factor of thermal power
(c) 1 and 3 only
plants.
(d) 1, 2 and 3
2 www.visionias.in ©Vision IAS

FREE BY KING R QUEEN P [ऋषभ राजपूत]


5. Consider the following statements regarding 8. With reference to the Jatiya Sarkar, consider
the Swarajists: the following statements:
1. They functioned as a group within the 1. It came into existence in Satara,
framework of the national Congress.
Maharashtra as a parallel government.
2. C.R. Das was elected as the Mayor of
2. It was organized under Y.B. Chavan and
Calcutta.
Nana Patil.
3. Vithalbhai Patel was elected as President
3. It organized an armed Vidyut Vahini
of the Central Legislative Assembly.
4. In 1923 elections, they got a clear during the Quit India movement.

majority in the Central Provinces and How many of the above statements are
were the largest party in Bengal. correct?
How many statements given above are (a) Only one
correct?
(b) Only two
(a) Only one
(c) All three
(b) Only two
(d) None
(c) Only three
(d) All four
9. Consider the following statements regarding

6. With reference to Gandhi's early career and the Tilak's Home Rule movement:

activism, which of the following happened 1. Lokmanya Tilak’s Home Rule League
the earliest? worked in Maharashtra, Karnataka, and
(a) Setting up of the Natal Indian Congress
Central Provinces with a special focus
(b) Starting a paper called Indian Opinion
on Bombay city.
(c) Establishing the Phoenix Settlement
2. Tilak proposed the formation of
(d) Forming a Satyagraha Sabha
linguistic states during the Home Rule

7. With reference to Indian history, the Shore campaign.

Committee was formed to: 3. C.P. Ramaswamy Aiyar and George

(a) study the constitutional reforms post Arundale worked for Tilak’s League
1892. How many statements given above are
(b) fight for the rights of the passengers of
correct?
Komagata Maru.
(a) Only one
(c) create a public opinion around he round
(b) Only two
table conferences.
(c) All three
(d) negotiate and discuss with the princely
states. (d) None
3 www.visionias.in ©Vision IAS

FREE BY KING R QUEEN P [ऋषभ राजपूत]


10. With reference to the tribal movements in 12. Consider the following statements regarding

the North-East India during the British the Morley-Minto reforms:

period, consider the following statements: 1. Indians were allowed to participate in

1. Kukis’ Revolt was aimed against British the election of various legislative

policies of recruiting labor during the councils.

2. An Indian was to be appointed a member


First World War.
of the Governor-General’s Executive
2. The Zeliangrong movement was started
Council.
against the failure of the British to
3. It introduced bicameralism in the Indian
protect them during the Kuki violence in
legislative structure.
1917-19.
4. It included two Indians in the Secretary
Which of the statements given above is/are
of State’s India Council for the first
correct?
time.
(a) 1 only
How many of the statements given above are
(b) 2 only
correct?
(c) Both 1 and 2 (a) Only one
(d) Neither 1 nor 2 (b) Only two

(c) Only three


11. Consider the following statements about the (d) All four

Cabinet Mission Plan:

1. Only one Indian member was a part of 13. Which of the following are not the causes

the Mission. behind the Bengal famine, in 1943?

2. It rejected the demand for a separate 1. Devastating cyclone in Midnapur.

"Pakistan". 2. The refugee influx from Burma into

Chittagong.
3. It provided for direct elections with a
3. The stoppage of imported rice from
limited franchise.
Burma.
How many of the above statements are
Select the correct answer using the code
correct?
given below.
(a) Only one
(a) 1 and 2 only
(b) Only two
(b) 2 and 3 only
(c) All three
(c) 1 and 3 only
(d) None
(d) 1, 2 and 3
4 www.visionias.in ©Vision IAS

FREE BY KING R QUEEN P [ऋषभ राजपूत]


14. Consider the following statements: 17. Consider the following :
1. It was brought to eliminate the spread of Festival Associated State
socialism through the deporting of
1. Athachamayam : Karnataka
undesirable foreigners including
Britishers. Festival
2. Motilal Nehru called it ‘the Slavery of 2. Aadi Perukku : Jharkhand
India, Bill No.1’. 3. Bonderam : Goa
3. Capitalists like Purshottam Thakurdas
Festival
and G.D. Birla opposed the bill.
How many of the pairs given above are
Which of the following bills has been
described by the statements given above? correctly matched?
(a) The Rowlatt Bills (a) Only one
(b) Trade Dispute Bill (b) Only two
(c) Public Safety Bill
(c) All three
(d) Inclusion of Sedition provisions in the
IPC (d) None

15. Dark Patterns are unethical user interface/ 18. Consider the following pairs:
experience (UI/UX interactions), designed to
Published work Author
mislead or trick users. In this context,
consider the following examples: 1. Gandhi vs. Lenin : S.A. Dange
1. Basket sneaking 2. Navayug : Ghulam Hussain
2. Confirm shaming 3. Inquilab : Muzaffar Ahmed
3. Nagging How many pairs given above are correctly
4. Bait and switch
matched?
How many of the above-mentioned types of
dark patterns have been identified by the (a) Only one
Ministry of Consumer Affairs, Food and (b) Only two
Public Distribution? (c) All three
(a) Only one
(d) None
(b) Only two
(c) Only three
(d) All four 19. The term "Debt-for-nature swaps" was
recently seen in the news. In this context,
16. In the context of biodiversity of India,
consider the following countries :
consider the following statements regarding
1. Ecuador
Salamanders :
1. They belong to class of reptiles. 2. Gabon
2. Some of the salamanders have the ability 3. India
to re-grow lost limbs. How many of the above countries are
3. They are not naturally found in India
involved in such swaps?
naturally.
How many of the statements given above are (a) Only one
correct? (b) Only two
(a) Only one (c) All three
(b) Only two
(d) None
(c) All three
(d) None
5 www.visionias.in ©Vision IAS

FREE BY KING R QUEEN P [ऋषभ राजपूत]


20. Consider the following statements: 22. In the context of government initiatives

Statement-I: On August 15th, 1947, India regarding skilling people in India, consider
the following statements regarding Project
was declared independent from British
Amber:
colonialism.
1. It is an initiative of the Department for
Statement-II: The Prime Minister of Britain, Promotion of Industry and Internal
Clement Atlee, declared on February 20, Trade.

1947, that the British would quit India after 2. It aims to skill people in traditional craft
making.
transferring power on August 15th, 1947.
Which of the statements given above is/are
Which one of the following is correct in
correct?
respect of the above statements? (a) 1 only
(a) Both Statement-I and Statement-II are (b) 2 only

correct, and Statement-II is the correct (c) Both 1 and 2


(d) Neither 1 nor 2
explanation for Statement-I.

(b) Both Statement-I and Statement-II are


23. During the Civil Disobedience Movement,
correct, and Statement-II is not the some areas saw the Anti-Chowkidari tax
correct explanation for Statement-I. campaigns. In this context, which of the

(c) Statement-I is correct, but Statement-II following statements is not correct regarding
the campaign?
is incorrect.
(a) Chowkidars acted as spies for the
(d) Statement-I is incorrect, but Statement-II
government.
is correct. (b) Chowkidars were directly appointed by
the local police stations.

21. Consider the following natural disasters: (c) The campaigns were carried out in the
regions of Bengal and Bihar.
1. Cloudburst
(d) Rajendra Prasad was involved in the
2. Lightning
Anti-Chowkidara campaign.
3. Pest attack

4. Cold waves 24. During the Indian revolutionary movement,

How many of the above natural disasters are there was large-scale participation of young
women under Surya Sen. In this context,
covered under the State Disaster Response
which of the following revolutionaries
Fund (SDRF)?
conducted a raid in Chittagong alongside
(a) Only one Surya Sen?
(b) Only two (a) Usha Mehta

(c) Only three (b) Santi Ghosh


(c) Pritilata Waddedar
(d) All four
(d) Suniti Chaudhuri
6 www.visionias.in ©Vision IAS

FREE BY KING R QUEEN P [ऋषभ राजपूत]


25. In the context of Cripps' proposals, consider 28. Consider the following statements:
the following: Statement-I: Mahatma Gandhi was hesitant
1. Indian Union with a dominion status to begin a full-fledged civil disobedience or
2. Procedure for the accession of provinces a no-revenue campaign during the Non-
to the Union. Cooperation Movement (NCM).
3. Extension of the separate electorate to Statement-II: During NCM, the masses often
other communities crossed the limits of the Gandhian creed of
How many of the above are the reasons non-violence and Gandhi failed to restrain
behind the rejection of Cripps' proposals by them.
the Muslim league?
Which one of the following is correct in
(a) Only one
respect of the above statements?
(b) Only two
(a) Both Statement-I and Statement-II are
(c) All three
correct and Statement-II is the correct
(d) None
explanation for Statement-I
(b) Both Statement-I and Statement-II are
26. With reference to the Eka movement,
correct and Statement-II is not the
consider the following statements:
correct explanation for Statement-I
1. The grassroots leadership of the
(c) Statement-I is correct Statement-II is
movement came from lower-caste
incorrect
leaders and small zamindars.
(d) Statement-I incorrect but Statement-II is
2. It was a secular movement with no
correct
involvement of any religious symbols.
3. The peasants decided to refuse to pay
29. Consider the following statements regarding
any land rent unless their demands were
met. the Congress Socialist Party (CSP):

How many of the statements given above are 1. The first annual session of the All India

correct? Congress Socialist Party was held in

(a) Only one 1936 alongside the All India Kisan

(b) Only two Sabha.

(c) Only three 2. The Meerut thesis related to


(d) None organizational changes in the Congress
was given by the CSP.
27. Which of the following rebellions happened Which of the statements given above is/are
the earliest? correct?
(a) Bhil Uprisings (Maharashtra) (a) 1 only
(b) Khasi uprisings (b) 2 only
(c) Revolt of Gadkari (c) Both 1 and 2
(d) Kittur uprisings (d) Neither 1 nor 2
7 www.visionias.in ©Vision IAS

FREE BY KING R QUEEN P [ऋषभ राजपूत]


30. Consider the following events: 33. The Government of India Act 1919 was an
1. Establishment of Anti-Non-Cooperation Act of the Parliament of the United
Society Kingdom which was passed to expand the
2. Formation of the Federation of Indian
participation of Indians in the government of
Chambers of Commerce and Industry
India. In this context, which of the following
(FICCI)
statements are correct regarding the changes
3. Formation of the Indian Merchants’
Chamber introduced by the Act?

Arrange the above events in chronological 1. It introduced bicameralism in the


order. provinces.
(a) 1-2-3 2. The legislators could ask questions and
(b) 1-3-2 pass adjournment motions.
(c) 2-1-3
3. It provided for the establishment of a
(d) 3-1-2
Central Public Service Commission.
Select the correct answer using the code
31. How many of the following parties/
organisations extended support in the release given below.

of the Indian National Army (INA) (a) 1 and 2 only


prisoners? (b) 2 and 3 only
1. Rashtriya Swayamsevak Sangh
(c) 1 and 3 only
2. Communist Party
(d) 1, 2 and 3
3. Justice Party
Select the correct answer using the code
given below. 34. Fabianism was a socialist school of thought
(a) Only one in Britain that originated in the 19th century
(b) Only two Britain. Which of the following nationalist
(c) All three leaders are considered to be the followers of
(d) None
Fabianism in economic development?
(a) Jawaharlal Nehru and Annie Besant
32. Which of the following statements best
(b) Mahatma Gandhi and Jawaharlal Nehru
explains 'Karnataka method', used as a
(c) Dadabhai Naoroji and Gopal Krishna
strategy in the Quit India Movement?
(a) It was an action to form parallel Gokhale
governments in isolated pockets. (d) Dr. Ambedkar and Dadabhai Naoroji
(b) It was a method to execute of
underground activities like guerilla 35. ‘Funga campaign’ sometimes seen in the
warfare. news has been launched by:
(c) It was a non-violent action and
(a) UN Biodiversity
constructive program.
(b) IUCN
(d) It s a strategy used by a group of
(c) UNEP
ordinary peasants to carry out sabotage
operations at night. (d) WWF
8 www.visionias.in ©Vision IAS

FREE BY KING R QUEEN P [ऋषभ राजपूत]


36. Consider the following statements regarding 38. Consider the following statements regarding
the Wardha Scheme of Basic Education
the Chamber of Princes, 1921:
(1937) formulated by the Zakir Hussain
1. The Chamber of Princes was formed committee:
1. It advocated free and compulsory
under the policy of the Subordinate education for all children below seven
Union. years of age.
2. It is recommended that teaching be made
2. The chamber of Princes was merely an in mother tongue.
advisory and consultative body. 3. It advocated for the inclusion of
religious and moral education in the
Which of the statements given above is/are curriculum.
How many of the above statements are
correct?
correct?
(a) 1 only (a) Only one
(b) Only two
(b) 2 only
(c) All three
(c) Both 1 and 2 (d) None

(d) Neither 1 nor 2 39. In the context of India, CSIR Prima ET11
was recently seen in the news. It is a/an:
(a) Drone
37. Consider the following passage: (b) Electric Vehicle
(c) Satellite
“He was a wanderer who left home at the
(d) Missile
age of thirteen. He did a stint as an
40. The Kampala declaration recently adopted
indentured laborer in Fiji and eventually
by the African countries seeks to address:
returned to India. He emerged as a leader of (a) Ivory trade
(b) Climate change and migration
the peasants of Avadh and soon
(c) Rare Earth Metal Pollution
demonstrated considerable leadership and (d) Racism faced by Africans

organizational capacities. In 1920, he led a 41. Which of the following historical


few hundred tenants from the Jaunpur and personalities were founding members of the
All India Women’s Conference?
Pratapgarh districts to Allahabad.” 1. Maharani Chimnabai Gaekwad
2. Sarojini Naidu
Which of the following is being referred to
3. Kamla Devi Chattopadhyaya
in the above passage? 4. Lady Dorab Tata
Select the correct answer using the code
(a) Thinguri Singh
given below:
(b) lndra Narain Dwivedi (a) 2 and 3 only
(b) 1, 2 and 4 only
(c) Gauri Shankar Misra
(c) 1, 2, 3 and 4
(d) Baba Ramchandra (d) 3 and 4 only
9 www.visionias.in ©Vision IAS

FREE BY KING R QUEEN P [ऋषभ राजपूत]


42. With reference to Sarojini Naidu, consider 44. Consider the following statements regarding

the following statements: the Lucknow session of the Indian National

1. She led an unarmed crowd to raid the Congress in 1916:


1. Ambika Charan Majumdar was the
Dharasana Salt Works during the Civil
President of the session.
Disobedience Movement.
2. G.K. Gokhale and Pherozeshah Mehta
2. She represented Indian women in the
finally agreed to the inclusion of Tilak in
First Round Table Conference along
the Congress.
with Begum Jahanara Shahnawaz. 3. Annie Besant played an instrumental in
3. She headed the Calcutta session of the the Lucknow Pact between the Congress
Indian National Congress in 1917. and Muslim League.

4. She penned the poem titled “The Feather How many statements given above are
correct?
of Dawn”.
(a) Only one
How many of the statements given above
(b) Only two
is/are correct?
(c) All three
(a) Only one
(d) None
(b) Only two

(c) Only three 45. The 'Kisan Manifesto' was adopted by the
(d) All four Congress at
(a) Lucknow Session, 1916

43. Which of the following events/activities (b) Gaya Session, 1922


(c) Faizpur Session, 1936
were witnessed during the Quit India
(d) Bombay Session, 1934
movement?

1. A major peasant rebellion


46. Consider the following statements regarding
2. Revolts in urban areas marked by
Sickle Cell Disease:
strikes, boycott, and picketing 1. People with this disease have sickle-
3. Terrorist and underground activities shaped white blood cells.
Select the correct answer using the code 2. It can spread through mosquito bite.

given below. Which of the statements given above is/are

(a) 1 and 2 only correct?


(a) 1 only
(b) 2 and 3 only
(b) 2 only
(c) 3 only
(c) Both 1 and 2
(d) 1, 2 and 3
(d) Neither 1 nor 2
10 www.visionias.in ©Vision IAS

FREE BY KING R QUEEN P [ऋषभ राजपूत]


47. During which of the following movements, 51. Who among the following was/were
the National Council of Education was associated with the Kheda Satyagraha
established?
organised by Mahatma Gandhi in 1918?
(a) Non-Cooperation movement
1. Sardar Vallabhbhai Patel
(b) Simon Commission boycott movement
(c) Swadeshi movement 2. Rajendra Prasad
(d) Home Rule movement 3. Indulal Yagnik
Select the correct answer using the code
48. Consider the following national leaders : given below.
1. Nellie Sengupta
(a) Only 1
2. Lala Lajpat Rai
(b) 1 and 3 only
3. Satyendra Prasanna Sinha
4. Tej Bahadur Sapru (c) 2 and 3 only
How many of the above never served as the (d) 1, 2 and 3
president of Indian National congress?
(a) Only one
52. With reference to British rule in India, the
(b) Only two
‘Revolutionary Movement Ordinance’ and
(c) Only three
(d) All four ‘Special Criminal Courts Ordiance’ are
related to:
49. Which of the following statements is not (a) World War I and the Ghadar movement
correct with reference to Aurobindo Ghosh?
(b) Civil Disobedience Movement
(a) His idea of Swaraj was that of an
(c) Revolt of 1857
absolute freedom the British Empire.
(b) He acted as the principal of (d) Quit India movement

Shantiniketan during the Swadeshi


movement. 53. Consider the following statements regarding
(c) He influenced Gandhiji’s principles of the Individual Satyagraha,1940-41:
Satyagraha and passive resistance.
1. Its aim was to give another opportunity
(d) ‘The Life Divine’ and 'The Synthesis of
for the British Government to accept
Yoga’ are his famous works.
Congress’s demands peacefully.
50. With reference to Indian history, December 2. Their demand was the freedom of
22, 1939 is significant because: speech against Indian participation in the
(a) Linlithgow, the viceroy, declared India's
war through an anti-war declaration.
involvement in the World War II
Which of the statements given above is/are
(b) Jinnah asked the muslims to observe the
Day of Deliverance. not correct?
(c) Subhash Cahndra Bose escaped from (a) 1 only
house arrest. (b) 2 only
(d) The August offer was made to assure (c) Both 1 and 2
India's cooperation in war efforts
(d) Neither 1 nor 2
11 www.visionias.in ©Vision IAS

FREE BY KING R QUEEN P [ऋषभ राजपूत]


54. With reference to the Methane-eating 57. Which of the following was the main
bacteria, consider the following statements: objective of the Newspapers Act of 1908?
1. Methanotrophic microorganisms oxidize (a) To promote freedom of the press.
methane to harness energy under oxic (b) To impose strict censorship on all kinds
and anoxic conditions.
of publications.
2. The bacterial strain
(c) To encourage British newspapers to
Methylotuvimicrobium buryatense
counter Indian militant publications and
5GB1C consumes methane.
Which of the statements given above is/are gain support from the educated class.

correct? (d) To provide financial support to Indian


(a) 1 only newspapers.
(b) 2 only
(c) Both 1 and 2 58. Consider the following:
(d) Neither 1 nor 2 1. Motilal Nehru
2. Jawaharlal Nehru
55. Consider the following countries :
3. Subhash Chandra Bose
1. Libya
4. Annie Besant
2. Egypt
5. Mahatma Gandhi
3. Nigeria
4. Mali How many of the above-mentioned were

How many of the above countries share a part of the drafting committee of the Nehru
border with Niger? report?
(a) Only one (a) Only two
(b) Only two (b) Only three
(c) Only three (c) Only four
(d) All four (d) All five

56. With reference to the Freedom Army (Mukti


59. Which of the following were the initiatives
Sena), consider the following statements:
by Dr. BR Ambedkar?
1. It was set up by Subhash Chandra Bose
with the help of the Japanese 1. All India Scheduled Caste Federation

government. 2. Samaj Samta Sangh


2. It consisted of the prisoners of war of 3. Depressed Classes Institution
Indian origin captured by Britain in (Bahishkrit Hitakarini Sabha)
Malaya. 4. All-India Anti-Untouchability League
3. Dresden (Germany) was made the office 5. Self-Respect Movement
of the Freedom Army. Select the correct answer using the code
How many of the above statements are
given below:
correct?
(a) Only one
(a) Only one
(b) Only two
(b) Only two
(c) All three (c) Only three

(d) None (d) All five


12 www.visionias.in ©Vision IAS

FREE BY KING R QUEEN P [ऋषभ राजपूत]


60. Consider the following statements regarding 63. Consider the following statements regarding
Turtle Survival Alliance : Lord Mountbatten:
1. It is an intergovernmental organization. 1. He was the last governor-general of
2. It seeks to protect freshwater turtles as India.
well as tortoises. 2. Originally, he had proposed a plan for
3. It is formed under the aegis of Wetland the balkanization of India.
International. 3. In his 3rd June Plan, he accepted the
How many of the statements given above are principle of the partition of British India.
correct? How many of the above given statements are
(a) Only one correct?
(b) Only two (a) Only one
(c) All three (b) Only two
(d) None (c) All three
(d) None
61. Consider the following pairs:
Newspaper /Journal Founder 64. Consider the following statements about
1. Indian Sociologist : Shyamji Krishna Indian National Congress (INC) sessions:
Varma 1. Subhash Bose was elected twice as the
2. Bande Mataram : Madam Bikaji President of INC.
Kama 2. Gandhiji presided over an INC session
3. Talvar : Taraknath Das only once.
4. Free Hindustan : Virendranath 3. None of the INC presidents were
Chattopadhyaya foreigners.
How many of the above pairs are correctly 4. The first Indian woman President of the
matched? INC was Sarojini Naidu.
(a) Only one How many of the above statements are
(b) Only two correct?
(c) Only three (a) Only one
(d) All four (b) Only two
(c) Only three
62. Consider the following statements regarding (d) Only four
the Round Table Conferences (RTCs):
1. The Gandhi-Irwin Pact was signed after 65. “General Comment No. 26 “is often seen in
the second RTC. the news in the context of:
2. Indian National Congress did not (a) UN Security Council comment on
participate in the first and third RTCs. criticizing Russia's attack on Ukraine.
Which of the statements given above is/are (b) UN guidance which calls for climate
correct? action by States to protect children’s
(a) 1 only rights.
(b) 2 only (c) ILO resolution on family planning
(c) Both 1 and 2 (d) ILO Conventions on the worst forms of
(d) Neither 1 nor 2 child labor.
13 www.visionias.in ©Vision IAS

FREE BY KING R QUEEN P [ऋषभ राजपूत]


66. Consider the following pairs: 68. Consider the following pairs:
Revolutionary Founder
Viceroy Major event
Orgnisation
1. Lord Reading : Chauri Chaura 1. Free India Society : Lala Hardayal
incident 2. United India House : Tarak Nath Das
3. Paris Indian : Madam Bhikaji
2. Lord Willingdon : Quit India
Society Cama
Movement How many pairs given above are correctly
3. Lord Linlithgow : Poona Pact matched?
(a) Only one
How many of the above pairs are correctly (b) Only two
matched? (c) All three
(d) None
(a) Only one

(b) Only two 69. Consider the following events during India's
(c) All three freedom struggle:
1. Launch of Home Rule League by Bal
(d) None
Gangadhar Tilak
2. Death of Lala Lajpat Rai
3. Renouncement of Knighthood by
67. Consider the following statements:
Rabindra Nath Tagore
Statement-I: The Moderates wanted the 1907 4. Gandhiji presiding over the annual
session to be held in Surat. session of Congress
Which one of the following is the correct
Statement-II: It was a general convention
chronological order of their occurrence?
that a leader from the host province was not (a) 1-3-4-2
allowed to preside over the Congress (b) 2-3-1-4
(c) 1-4-3-2
session. (d) 3-1-2-4
Which one of the following is correct in
70. With reference to the Telangana Movement
respect of the above statements?
of the 1940s, consider the following
(a) Both Statement-I and Statement-II are statements:
correct. and Statement-II is the correct 1. The immediate cause of the movement
was the occurrence of the great Indian
explanation for Statement-I
famine of 1943.
(b) Both Statement-I and Statement-II 2. The movement led to the disappearance
of Vethi or forced labor from the
correct and Statement-II is not
villages.
the·correct explanation for Statement-I Which of the statements given above is/are
(c) Statement-I is correct but Statement-II is correct?
(a) 1 only
incorrect
(b) 2 only
(d) Statement-I is incorrect but Statement-II (c) Both 1 and 2
is correct (d) Neither 1 nor 2
14 www.visionias.in ©Vision IAS

FREE BY KING R QUEEN P [ऋषभ राजपूत]


71. In the context of international river bodies, 74. Consider the following statements regarding
the Sulina Channel which was recently seen the All India Trade Union Congress
(AITUC):
in the news is a distributary of:
1. It refused to support Congress
(a) Danube candidates in 1937 elections.
(b) Volga 2. Lala Lajpat Rai was elected as its first
president.
(c) Nile
3. Its formation was opposed and criticised
(d) Amazon
by the Indian National Congress at Gaya
Session, 1922.
72. Consider the following leaders: How many statements given above are
correct?
1. Sachindranath Sanyal
(a) Only one
2. Vishnu Ganesh Pingley (b) Only two
3. Rash Behari Bose (c) All three
4. Kartar Singh Sarabha (d) None

How many of the above leaders were related


75. The programme of this movement included
to the Ghadar movement? within its ambit the surrender of titles and
(a) Only one honors, boycott of government-affiliated
(b) Only two schools and colleges, law courts, and foreign
cloth, and could be extended to include
(c) Only three
resignation from government service. The
(d) All four struggle and spirit of defiance gave rise to
many local movements such as the Awadh
Kisan movement, and the Eka movement.
73. With reference to the Pacific Decadal
Which of the following movements is best
Oscillation (PDO), consider the following
described by the above-given passage?
statements: (a) Non-Cooperation Movement
1. It is a cyclical event that repeats every (b) Civil Disobedience movement
(c) Quit India movement
20-30 years with a ‘cool’ and ‘warm’
(d) Home Rule movement
phase.
2. Positive PDO means cooler west Pacific 76. Consider the following leaders:
Ocean and warmer eastern side. 1. Aurobindo Ghosh
2. C.R. Das
3. ENSO with a positive PDO brings more
3. Barindra Ghosh
rain to India. 4. Ullaskar Dutta
How many of the above statements are How many of the above personalities were
associated with the “Maniktala Bomb Case
correct?
Trial”?
(a) Only one
(a) Only one
(b) Only two (b) Only two
(c) All three (c) Only three
(d) All four
(d) None
15 www.visionias.in ©Vision IAS

FREE BY KING R QUEEN P [ऋषभ राजपूत]


77. Consider the following pairs: 79. Consider the following pairs:
Sportsperson Game
Delhi Durbars Significance/Occasion
1. Shaili Singh : Long jump
1. The first : The title of “Kaiser E- 2. Prannoy H.S : Badminton
3. R Praggnanandhaa : Chess
Delhi Durbar Hind‟ was adopted by
How many of the above pairs are correctly
Queen Victoria. matched?
(a) Only one pair
2. The second : The shifting of capital (b) Only two pairs
Delhi Durbar from Calcutta to Delhi (c) All three pairs
(d) None
was announced

3. The third : Marked the succession 80. With reference to the National Coal Index,
consider the following statements:
Delhi Durbar of George V 1. Only Prices of coal produced within the
How many pairs given above are correctly country are taken into account for
compiling the NCI.
matched? 2. It reflects the change of price level of
coal in a particular month relative to the
(a) Only one
fixed base year.
(b) Only two 3. It is composed of a set of five sub-
indices out of which three for Non-
(c) All three
Coking Coal and two for Coking Coal.
(d) None How many of the above statements are
correct?
(a) Only one
78. Consider the following Statements: (b) Only two
(c) All three
1. Just Energy Transition Partnership aims
(d) None
to accelerate the phasing-out of coal
81. Consider the following statements regarding
dependency.
Indian Press Act, 1910 :
2. Glasgow Financial Alliance for Net Zero 1. It reformed the worst features of Lytton's
Press Act, of 1878.
aims to provide a forum for leading 2. The act transferred the authority to local
financial institutions to transition to a governments for the registration of new
printing presses and the collection of
net-zero global economy. security money.
Which of the statements given above is/are 3. The act provided the provision of appeal
to a special tribunal of the High Court
correct? against orders of forfeiture.
(a) 1 only How many of the above statements are
correct?
(b) 2 only (a) Only one
(b) Only two
(c) Both 1 and 2
(c) All three
(d) Neither 1 nor 2 (d) None

16 www.visionias.in ©Vision IAS

FREE BY KING R QUEEN P [ऋषभ राजपूत]


82. Consider the following statements about 85. Which of the following explicitly declared
events in Indian History: the British government policy of gradual
1. It began on February 18, 1946. realization of responsible government in
2. It saw participation from a wide range of India?

backgrounds, including laborers, (a) Act for the Good government of India
1858
farmers, and others.
(b) Morley Minto Reforms
3. It is often referred to as the Last War of
(c) August Declaration of 1917
Indian Independence.
(d) Delhi Durbar of 1911
Which event is being referred to in the above
statements?
86. With reference to the Red Sand Boa,
(a) Direct Action Day consider the following statements:
(b) The Quit India Movement 1. They are a non-venomous species found
(c) The Royal Indian Navy Mutiny throughout the dry parts of the Indian
(d) The Azad Hind War subcontinent.
2. They are ovoviviparous and nocturnal.
83. With reference to the achievements of 3. They come under the Schedule I of the
Chandrayaan-3, consider the following Wildlife (Protection) Act, 1972.
statements: How many of the statements given above are

1. It is the first time a country has landed correct?


(a) Only one
successfully on the South side of the
(b) Only two
moon.
(c) All three
2. It has conducted in-situ measurements of
(d) None
lunar composition.
3. It has detected sulfur and water on the 87. Consider the following statements regarding
moon's surface for the first time. Dr. Babasaheb Ambedkar:
How many of the above statements are 1. He led the Mahad Satyagraha to publicly
correct? burn the Manusmriti.
(a) Only one 2. He was appointed by the Bombay
(b) Only two Legislative Council to work with the
(c) All three Simon Commission.
(d) None 3. He founded the Independent Labour
Party in 1936 which fought the 1937
elections.
84. ‘Public Tech Platform for Frictionless
4. He was the first president of the All
Credit’ (PTPFC), sometimes seen in the
India Depressed Classes Association.
news, is an initiative of:
How many of the statements given above are
(a) Reserve Bank of India
correct?
(b) Securities and Exchange Board of India
(a) Only one
(c) TransUnion CIBIL Limited (b) Only two
(d) Pension Fund Regulatory and (c) Only three
Development Authority (d) All four
17 www.visionias.in ©Vision IAS

FREE BY KING R QUEEN P [ऋषभ राजपूत]


88. Consider the following statements regarding 91. Consider the following statements:
Government Resolution on Education 1. He had been a key figure in the
formation of the Communist Party of
Policy, 1913:
Mexico in 1919.
1. The resolution was passsed by
2. He formed the Communist Party of India
government due to the efforts of G.K. in Tashkent in 1920.
Gokhale in Legislative council. 3. He was a member of the Indian
2. The Government of India accepted the Constituent Assembly.
principle of compulsory primary 4. He is known as the father of “New
Humanism”.
education for the first time.
The above statements refer to which of the
Which of the statements given above is/are
personalities associated with the Freedom
correct? Struggle of India?
(a) 1 only (a) P. C. Joshi
(b) 2 only (b) Subhash Chandra Bose
(c) Both 1 and 2 (c) Manbendranath Roy
(d) Dr. B. R. Ambedkar
(d) Neither 1 nor 2

92. With reference to the Corporate Debt Market


89. Consider the following statements about the Development Fund (CDMDF), consider the
Bombay Manifesto, 1936: following statements:
1. It was an open indictment of Nehru’s 1. It is a trust fund formed by the
Department of Economic Affairs (DEA).
preaching of socialist ideals.
2. It is intended to provide liquidity support
2. It was an initiative of Indian Chamber of
in the event of a financial crisis.
Commerce. 3. It will buy only investment-grade
Which of the statements given above is/are securities from the secondary markets of
correct? maturity up to five years.
(a) 1 only How many of the above statements are
correct?
(b) 2 only
(a) Only one
(c) Both 1 and 2 (b) Only two
(d) Neither 1 nor 2 (c) All three
(d) None
90. In the context of Indian history, what was
93. Consider the following statements about the
common between C. Rajgopalachari, Baldev
Shimla Conference:
Singh, and Jagjeevan Ram?
1. It was a meeting convened in order to
(a) They were Presidents of the Indian discuss the plan for Indian self-
National Congress government.
(b) They were members of the Interim 2. Lord Wavell was the Viceroy of India at
Government in 1946 that time.
Which of the statements given above is/are
(c) They were members of the Drafting
correct?
Committee in the Constituent Assembly (a) 1 only
(d) They were Representatives of the Indian (b) 2 only
National Congress at the 2nd Round (c) Both 1 and 2
Table Conference (d) Neither 1 nor 2
18 www.visionias.in ©Vision IAS

FREE BY KING R QUEEN P [ऋषभ राजपूत]


94. With reference to women’s participation in 96. Consider the following statements with
reference to the activities of the
the Indian National Movement, consider the
revolutionary nationalists during the 20th
following statements: century:
1. In the beginning, the revolutionaries
1. The Morley Minto reforms for the first believed in group activities and mass
time provided the right to vote for the struggle, while later they shifted to
individual heroic actions.
women in India. 2. Abhinav Bharat was a secret society that
was set up as a response to the limited
2. The first Indian woman to contest for
success of the moderates during the
election was Kamala Devi Swadeshi movement.
Which of the statements given above is/are
Chattopadhyay.
correct?
Which of the statements given above is/are (a) 1 only
(b) 2 only
correct? (c) Both 1 and 2
(a) 1 only (d) Neither 1 nor 2

(b) 2 only 97. In the context of cybersecurity, which of the


following is a common characteristic of
(c) Both 1 and 2
smishing attacks?
(d) Neither 1 nor 2 (a) Attackers use email to deliver malicious
links
(b) Attackers impersonate trusted entities in
95. The call for a boycott of the Indian Statutory text messages.
(c) Attackers gain access to a victim's
Commission of 1927 was endorsed by which
computer through a phone call.
among the following groups? (d) Attackers primarily target social media
accounts.
1. Liberal Federation

2. Indian Industrial and Commercial 98. The Government of India Act, 1935
proposed the establishment of a ‘Federation
Congress of India’. In this context, consider the
following statements:
3. Hindu Mahasabha
1. The Federation of India was intended to
4. Muslim League be a sovereign legislature.
2. To join the Federation, a princely state
Select the correct answer using the code
must execute an Instrument of Accession
given below. in favor of the Crown.
Which of the statements given above is/are
(a) Only one correct?
(b) Only two (a) 1 only
(b) 2 only
(c) Only three (c) Both 1 and 2
(d) Neither 1 nor 2
(d) All four
19 www.visionias.in ©Vision IAS

FREE BY KING R QUEEN P [ऋषभ राजपूत]


99. Which of the following extremist leaders in
1916 wrote in ‘Young India’ to criticize the
Indian National Congress using the “safety
valve theory” and called the Congress ‘a
product of Lord Dufferin’s brain’?
(a) G.S. Khaparde
(b) Bipin Chandra Pal
(c) Vinayak Damodar Savarkar
(d) Lala Lajpat Rai

100. With reference to the Rampa rebellion


during the 20th century period in India,
consider the following statements:
1. It was also known as the Manyam
Rebellion.
2. It started against the oppressive
measures taken under the Madras Forest
Act, of 1882.
Which of the statements given above is/are
correct?
(a) 1 only
(b) 2 only
(c) Both 1 and 2
(d) Neither 1 nor 2

Copyright © by Vision IAS


All rights are reserved. No part of this document may be reproduced, stored in a retrieval system or
transmitted in any form or by any means, electronic, mechanical, photocopying, recording or otherwise,
without prior permission of Vision IAS.

20 www.visionias.in ©Vision IAS

FREE BY KING R QUEEN P [ऋषभ राजपूत]


VISIONIAS
www.visionias.in

Test Booklet Series

TEST BOOKLET

GENERAL STUDIES (P) 2024 – Test – 4140


C
Time Allowed: Two Hours Maximum Marks: 200

INSTRUCTIONS

1. IMMEDIATELY AFTER THE COMMENCEMENT OF THE EXAMINATION, YOU SHOULD CHECK THAT THIS BOOKLET
DOES NOT HAVE ANY UNPRINTED OR TURN OR MISSING PAGES OR ITEMS, ETC. IF SO, GET IT REPLACED BY A
COMPLETE TEST BOOKLET.

2. ENCODE CLEARLY THE TEST BOOKLET SERIES A, B, C OR D AS THE CASE MAY BE IN THE APPROPRIATE PLACE IN
THE ANSWER SHEET.

3. You have to enter your Roll Number on the Test Booklet in the Box
provided alongside. Do NOT write anything else on the Test Booklet.

4. This Test Booklet contains 100 items (Questions). Each item is printed in English. Each item comprises four
responses (answers). You will select the response which you want to mark on the Answer Sheet. In case you
feel that there is more than one correct response with you consider the best. In any case, choose ONLY ONE
response for each item.

5. You have to mark all your responses ONLY on the separate Answer Sheet provided. See direction in the
answers sheet.

6. All items carry equal marks. Attempt all items. Your total marks will depend only on the number of correct
responses marked by you in the answer sheet. For every incorrect response 1/3rdof the allotted marks will be
deducted.

7. Before you proceed to mark in the Answer sheet the response to various items in the Test booklet, you have to
fill in some particulars in the answer sheets as per instruction sent to you with your Admission Certificate.

8. After you have completed filling in all responses on the answer sheet and the examination has concluded, you
should hand over to Invigilator only the answer sheet. You are permitted to take away with you the Test
Booklet.

9. Sheet for rough work are appended in the Test Booklet at the end.

DO NOT OPEN THIS BOOKLET UNTIL YOU ARE ASKED TO DO SO


1 www.visionias.in ©Vision IAS

FREE BY KING R QUEEN P [ऋषभ राजपूत]


1. Consider the following statements: 4. Consider the following statements regarding

1. The Constitution of India gives the the loss of Indian citizenship:

Parliament the power to supplement the 1. Any citizen of India of full age and

provisions of the Constitution by capacity can make a declaration

legislation. renouncing his Indian citizenship.

2. The makers of the Constitution aimed 2. When a person renounces his Indian

for both exhaustiveness and exigencies citizenship, every minor child of that

of the future. person also loses citizenship.

Which of the statements given above is/are 3. The number of citizens renouncing

Indian citizenship has consistently


correct?
increased in the last decade.
(a) 1 only
4. A person voluntarily giving up Indian
(b) 2 only
citizenship is not qualified to apply for
(c) Both 1 and 2
the Overseas Citizen of India (OCI).
(d) Neither 1 nor 2
How many of the statements given above are

correct?
2. Society for the Acquisition of General
(a) Only one
Knowledge, 1838 was established by
(b) Only two
(a) Keshub Chandra Sen
(c) Only three
(b) Sayyid Ahmed Khan
(d) All four
(c) Followers of Henry Vivian Derozio

(d) Debendranath Tagore


5. Consider the following statements regarding
Ahmed Shah Abdali's invasions of India:
3. Shugoshin, recently seen in the news, is
1. He named Shah Alam II as the Mughal
a/an:
Emperor after winning the Third Battle
(a) protein that locks the X chromosome
of Panipat.
into an X shape.
2. He carried with him the Peacock Throne
(b) chromosomes that have centromeres
of Shahjahan while leaving India.
present at the end of the chromosome. Which of the statements given above is/are
(c) chromosomes whose centromere correct?
separates the chromosome into two (a) 1 only
equally sized arms. (b) 2 only
(d) unique structure allows our genetic data (c) Both 1 and 2
to be discreetly kept inside our nuclei. (d) Neither 1 nor 2
2 www.visionias.in ©Vision IAS

FREE BY KING R QUEEN P [ऋषभ राजपूत]


6. Consider the following statements with 8. Consider the following statements regarding

regard to the Gram Nyayalayas: Derozians of the Young Bengal Movement:


1. The Gram Nyayalaya is a court of 1. They posed an intellectual challenge to
Judicial Magistrates of the first class. the religious and social orthodoxy of
2. The Presiding Officer (Nyayadhikari) in Hinduism.
Gram Nyayalaya is appointed by the 2. They did not have total faith in the
Governor in consultation with the High British and rejected English education.
Court. 3. Their professed "atheism" became
3. Nyayadhikaris who preside over Gram famous which was manifested through
Nyayalayas are strictly judicial officers.
wining and dining in forbidden meat.
Which of the statements given above is/are
How many of the above statements are
correct?
correct?
(a) 1 and 2 only
(a) Only one
(b) 2 only
(b) Only two
(c) 1 and 3 only
(c) All three
(d) 1, 2 and 3
(d) None

7. Consider the following statement with


9. Consider the following statements regarding
reference to the Fundamental Duties:
Lord Cross's Act, 1892:
1. Fundamental Duties serve as a warning
1. It has provided for the expansion of the
against anti-national activities.
legislative councils both at the center
2. The Parliament can not provide for the
and the provinces.
imposition of penalty or punishment for
2. It has provided for the election to the
failure to fulfill Fundamental Duties.
councils for the first time in British
3. Fundamental Duties create a feeling that
India.
citizens are not mere spectators but
3. The provisions included both the right to
active participants in the governance of
discussion and vote on the budget.
the state.

Which of the statements given above is/are How many of the above statements are

correct? correct?

(a) 1 and 3 only (a) Only one

(b) 1 and 2 only (b) Only two

(c) 1, 2 and 3 (c) Only three

(d) 2 only (d) None


3 www.visionias.in ©Vision IAS

FREE BY KING R QUEEN P [ऋषभ राजपूत]


10. Consider the following statements regarding 13. Consider the following pairs :
the federal features of the Indian Reform Act Governor-
constitution:
General/Viceroy
1. It consists of a dual polity with the
1. Abolition of Sati : William Bentick
Union government at the center and
states at the periphery. Act,1829
2. Bicameralism strengthens the federal 2. Widow : Lord Dalhousie
feature of the Indian political system. Remarriage
Which of the statements given above is/are
Act,1856
correct?
3. Age of Consent : Lord Lansdowne
(a) 1 only
(b) 2 only Act,1891
(c) Both 1 and 2 How many of the above pairs are correctly
(d) Neither 1 nor 2 matched?
(a) Only one
11. ‘The Congress is tottering to its fall, and one
(b) Only two
of my greatest ambitions while in India is to
assist it to a peaceful demise’. Who said (c) All three
these famous lines? (d) None
(a) Lord Dufferin
(b) Lord Irwin 14. The "Paris Principles" were adopted by the
(c) Lord Curzon
UN in 1993. They primarily deal with:
(d) George Hamilton
(a) Human Rights

12. Consider the following statements: (b) Climate Change


Statement-I: Originally the scope of judicial (c) Pandemics
review power of the Supreme Court in India (d) Gender Equality
is narrower than that of what exists in the
USA.
15. Consider the following pairs:
Statement-II: The American Constitution
provides for ‘due process of law’ against that Educational Established during
of ‘procedure established by law’ contained Institutions the tenure of
in the Indian Constitution (Article 21). Governor-General
Which one of the following is correct in
1. Sanskrit College : Lord Cornwallis
respect of the above statements?
(a) Both Statement-I and Statement-II are 2. Calcutta Madarassa : Lord Wellesley
correct and Statement-II is the correct 3. Fort William : Warren Hastings
explanation for Statement-I college
(b) Both Statement-I and Statement-II are
How many of the above pairs are correctly
correct and Statement-II is not the
matched?
correct explanation for Statement-I
(c) Statement-I is correct Statement-II is (a) Only one
incorrect (b) Only two
(d) Statement-I incorrect but Statement-II is (c) All three
correct
(d) None
4 www.visionias.in ©Vision IAS

FREE BY KING R QUEEN P [ऋषभ राजपूत]


16. Consider the following pairs: 19. Consider the following millets:
Islands in news Country 1. Kodo
1. Macquarie : Brazil 2. Poroso
2. Munroe Thuruthu : India 3. Ragi
3. Siniyah : Japan 4. Barnyard
How many of the above pairs are correctly How many of the above-mentioned millets
matched?
are examples of minor millets?
(a) Only one pair
(a) Only one
(b) Only two pairs
(b) Only two
(c) All three pairs
(c) Only three
(d) None
(d) All four

17. Consider the following statements regarding


the expunging of proceedings in the Indian 20. Consider the following statements:
Parliament: 1. Indian territory can be ceded to a foreign
1. Expunged portions cease to exist in the state only by amending the Constitution
records of Parliament but can be under Article 368.
reported by media houses. 2. The settlement of a boundary dispute
2. The decision to expunge a word or between India and another country does
portion of a speech is made by the not require a constitutional amendment.
Speaker. Which of the statements given above is/are
Which of the statements given above is/are not correct?
correct? (a) 1 only
(a) 1 only (b) 2 only
(b) 2 only
(c) Both 1 and 2
(c) Both 1 and 2
(d) Neither 1 nor 2
(d) Neither 1 nor 2

21. Consider the following pairs:


18. A person is eligible to get a ‘Person of
Maratha Significant events
Indian Origin’ card if:
1. A person at any time held an Indian Leaders

passport 1. Balaji : Secured chauth and


2. A person is a spouse of a citizen of Vishwanath sardeshmukh rights
India from Mughals
3. A person whose great-grandfather or 2. Ragunath Rao : Signed Treaty of
grandfather was born and permanently Bassein with British
resident in India as defined under the 3. Madhav Rao : Restored Shah Allam
Government of India Act 1935 II to Mughal Throne
How many of the conditions given above are How many of the above pairs are correctly
correct? matched?
(a) Only one (a) Only one
(b) Only two
(b) Only two
(c) All three
(c) All three
(d) None
(d) None
5 www.visionias.in ©Vision IAS

FREE BY KING R QUEEN P [ऋषभ राजपूत]


22. In the context of Indian geography, 25. With reference to the East India Company,
consider the following events:
Parkachik, Pindari, Machoi are:
1. The company established its first factory
(a) Mountains in east India in Odisha.
2. The company acquired the island of
(b) Glaciers
Bombay from Portugal.
(c) Rivers 3. The company was given permission to
trade at Hughli
(d) Salt water lakes
4. The company established its first factory
in the south at Masulipatnam.
23. Consider the following British-Indian Acts: Which of the following is the correct
chronological sequence of the above events?
1. Press Act, 1835 (a) 1-2-3-4
2. Licensing Act, 1857 (b) 4-1-2-3
(c) 4-2-3-1
3. Vernacular Press Act, 1878 (d) 4-1-3-2
4. Indian Press Act, 1910
26. Consider the following statements:
Which of the above acts were meant to curb 1. There is no provision in the Indian
the freedom of press in India? constitution for the direct disallowance
(without the intervention of a state
(a) 1, 2, and 3 only
governor) of a state assembly bill by the
(b) 2, 3, and 4 only President.
2. There is no means for a state assembly
(c) 1, 3, and 4 only
to override a Presidential veto on its bills
(d) 1, 2 and 4 only referred to him.
Which of the statements given above is/are
not correct?
24. Consider the following statements regarding (a) 1 only
(b) 2 only
Kuka Movement:
(c) Both 1 and 2
1. It was founded by Ranjit Singh in 1840. (d) Neither 1 nor 2

2. Initially, it was a religious movement but


27. With reference to anti-defection law,
later transformed into a political consider the following statements:
1. The process to disqualify a member on
movement.
the grounds of defection is the same as
3. It advocated for a boycott of English the process followed for disqualification
laws and education. in case the person holds an office of
profit.
How many of the above statements are 2. The presiding officer can take up a
correct? defection case on a suo-motu basis.
Which of the statements given above is/are
(a) Only one correct?
(b) Only two (a) 1 only
(b) 2 only
(c) All three (c) Both 1 and 2
(d) None (d) Neither 1 nor 2
6 www.visionias.in ©Vision IAS

FREE BY KING R QUEEN P [ऋषभ राजपूत]


28. He was an Indian student and one of the first 31. 'India has been bled by the forces of
leaders of the Indian community in North organized capital and is today lying prostrate
America. He started the newspaper called at its feet. Militarism and Imperialism are the
the Free Hindustan and wanted to popularise
twin-children of capitalism; they are one in
the hypocritical nature of the British
three and three in one. Their shadow, their
government.
Who among the following Indian fruit and their bark all are poisonous. It is
Revolutionaries has been described in the only lately that an antidote has been
passage given above? discovered and that antidote is organized
(a) Abdul Mohamed Barakatullah labour.'
(b) Rash Behari Bose
Who among the following has said these
(c) Tarak Nath Das
historic lines given above?
(d) G.D. Kumar
(a) C.R. Das
29. With reference to the Goods and Services (b) Acharya J.B. Kripalani
Tax Council (GST Council), consider the (c) N.M. Joshi
following statements: (d) Lala Lajpat Rai
1. The 100th Amendment Act of 2016
added a new Article 279-A giving the
32. Consider the following statements with
President the authority to appoint a GST
respect to the oath taken by judges of the
Council.
2. One-half of the members constitute the Higher Judiciary:
quorum and the decision is taken by a 1. There is no difference in the oath of a
majority of not less than three-fourths of Supreme court judge or a high court
the members present and voting. judge.
3. Any act of the Council will not become
2. The oath of the judges of high courts and
invalid on any vacancy or defect in the
supreme courts is mentioned in the third
constitution of the Council.
How many of the above statements are schedule of the Constitution of India.
correct? 3. The supreme court or high court judges
(a) Only one do not take an oath to protect and defend
(b) Only two the constitution of India.
(c) All three Which of the statements given above is/are
(d) None
correct?
(a) 1 and 2 only
30. With reference to Rang Ghar, consider the
following statements: (b) 2 only
1. It is a two-storey amphitheatre located in (c) 1 and 3 only
Assam's Sivasagar district. (d) 1, 2 and 3
2. It was built during the reign of Lachit
Barphukan.
33. Which of the following is embodied in the
Which of the statements given above is/are
Preamble to the Constitution of India?
correct?
(a) 1 only (a) Equality of welfare
(b) 2 only (b) Equality of opportunity
(c) Both 1 and 2 (c) Equality of justice
(d) Neither 1 nor 2 (d) Equality of Outcome
7 www.visionias.in ©Vision IAS

FREE BY KING R QUEEN P [ऋषभ राजपूत]


34. In the context of Maratha administration, the 37. Consider the following statements with
terms Mamlatadars and Kamavisdars were respect to the High Courts:
(a) representatives of the Peshwa in the
1. The constitution empowers the
district
(b) village headman parliament and respective state
(c) military officials legislatures to specify the strength of any
(d) intelligent officers high court.
2. The parliament can extend the
35. Consider the following statements about the
jurisdiction of a high court to any union
Indian National Army (INA):
1. The idea of INA was first conceived by territory or exclude the jurisdiction of a
Rash Behari Bose. high court from any union territory.
2. The outbreak of the Quit India Which of the statements given above is/are
Movement in 1942 gave a fillip to the
correct?
INA.
3. In 1943, it set up a Provisional Indian (a) 1 only
Government headquartered in Rangoon (b) 2 only
only. (c) Both 1 and 2
How many of the above statements are
(d) Neither 1 nor 2
correct?
(a) Only one
(b) Only two 38. Consider the following statements:
(c) All three Statement-I: The federal government under
(d) None
the Government of India Act 1935 could not
materialize.
36. Consider the following events:
1. The first parallel government was set up Statement-II: The All India States People
in Ballia under the leadership of Chittu Conference objected to the undemocratic
Pande. governance proposed for the princely states
2. The National Herald and Harijan ceased
in the Government of India Act 1935.
publication for the entire duration of the
struggle. Which one of the following is correct in
3. National flags were forcibly hoisted on respect of the above statements?
public buildings in defiance of the (a) Both Statement-I and Statement-II are
police.
correct and Statement-II is the correct
4. For regular communication, Ram
Manohar Lohia broadcasted on the explanation for Statement-I
Congress radio. (b) Both Statement-I and Statement-II
How many of the above were the correct and Statement-II is not
consequences of the arrest of top congress
the·correct explanation for Statement-I
leaders during the Quit India Movement?
(c) Statement-I is correct but Statement-II is
(a) Only one
(b) Only two incorrect
(c) Only three (d) Statement-I is incorrect but Statement-II
(d) All four is correct
8 www.visionias.in ©Vision IAS

FREE BY KING R QUEEN P [ऋषभ राजपूत]


39. With reference to the Central Council of 42. The National Convention which drafted the
Local Government, consider the following Commonwealth of India Bill (1925) was
statements: chaired by:
1. It was constituted by an Act of the
(a) Jawaharlal Nehru
Parliament.
2. The Union Minister of Housing and (b) Tej Bahadur Sapru
Urban Affairs acts as the Chairman of (c) Motilal Nehru
the Council. (d) G.K. Gokhale
3. It deals with the matters of urban local
government only. 43. Consider the following statements:
How many of the statements given above are
Statement-I: Indian model of government is
not correct?
the parliamentary form of government.
(a) Only one
(b) Only two Statement-II: The Council of Ministers is
(c) All three elected by and responsible to the parliament.
(d) None Which one of the following is correct in
respect of the above statements?
40. Consider the following pairs:
(a) Both Statement-I and Statement-II are
Carnatic Wars Treaty signed
correct and Statement-II is the correct
1. Battle of St. : Treaty of Puducherry
explanation for Statement-I
Thome
2. Battle of : Treaty of Peace of (b) Both Statement-I and Statement-II
Ambur Paris correct and Statement-II is not
3. Battle of : Treaty of Aix-La the·correct explanation for Statement-I
Wandiwash Chapelle (c) Statement-I is correct but Statement-II is
How many of the above pairs are correctly incorrect
matched? (d) Statement-I is incorrect but Statement-II
(a) Only one
is correct
(b) Only two
(c) All three
(d) None 44. Consider the following statements:
1. Raleigh Commission was established to
41. With reference to the recently organized examine the conditions and prospects of
Global Buddhist Summit, consider the primary and secondary education in
following statements:
India.
1. It was the first-ever Global Buddhist
2. The Moderates objected to the Raleigh
Summit organized by the Ministry of
Culture. Commission's suggestions because they
2. It was organized in collaboration with considered them to be retrograde
the International Buddhist measures.
Confederation. Which of the statements given above is/are
Which of the statements given above is/are
correct?
correct?
(a) 1 only
(a) 1 only
(b) 2 only (b) 2 only
(c) Both 1 and 2 (c) Both 1 and 2
(d) Neither 1 nor 2 (d) Neither 1 nor 2
9 www.visionias.in ©Vision IAS

FREE BY KING R QUEEN P [ऋषभ राजपूत]


45. Arrange the following states in the 48. Consider the following statements regarding
decreasing order of the strengths of their Objectives Resolution:
legislative assemblies. 1. Most of the significant provisions of
1. Andhra Pradesh objectives resolution were later accepted
2. Odisha as preamble.
3. Punjab 2. Objectives Resolution did not mention
4. Rajasthan the word “democratic”.
Select the correct answer using the code 3. It was presented by Nehru and seconded
given below. by Sardar Patel.
(a) 3-4-1-2 Which of the statements given above is/are
(b) 2-4-3-1 correct?
(c) 1-3-2-4 (a) 1 only
(d) 4-1-2-3
(b) 1 and 2 only
(c) 2 and 3 only
46. With reference to the borrowed features of
(d) 3 only
Indian Constitution, consider the following
pairs:
49. Consider the following events:
Countries Borrowed Features
1. Dropping of Atomic bomb on Hiroshima
1. Ireland : Procedure for amendment
2. Attlee's declaration
of the Constitution
3. Royal Indian Navy Revolt
2. USA : Post of Vice- President
What is the correct chronological sequence
3. South : Method of election of
of the above events?
Africa president.
(a) 1-2-3
4. France : Suspension of Fundamental
(b) 3-2-1
Rights during Emergency
(c) 2-1-3
(d) 1-3-2
How many of the pairs given above are
correctly matched?
(a) Only one 50. Consider the following:

(b) Only two 1. Self Government

(c) Only three 2. Abolition of Office of Secretary of State

(d) All four 3. Democratic rights for all members of


Indian society.
47. The Jotedars, a new class who had emerged 4. Extension of Permanent Settlement to
with the Permanent settlement Revenue Ryotwari and Mahalwari areas.
system brought by British East India How many of the above given are the
company were demands of moderates to the British?
(a) rich peasants (a) Only one
(b) middlemen (b) Only two
(c) absentee landlords (c) Only three
(d) share-croppers (d) All four
10 www.visionias.in ©Vision IAS

FREE BY KING R QUEEN P [ऋषभ राजपूत]


51. Consider the following statements: 54. Consider the following different types of
Statement-I: Approval voting system reduces water bodies:
the need for pre-poll alliances and seat- 1. A structure where water from ice melt,
sharing arrangements among parties. streams, springs, rain is accumulated
Statement-II: Approval voting system is 2. Water stored by diversion from a stream,
beneficial because it is based on ranked- nala or river
choice voting. 3. Oceans and lagoons
Which one of the following is correct in 4. Swimming pools
respect of the above statements? 5. Pucca open water tanks created only for
(a) Both Statement-I and Statement-II are cattle to drink water.
correct and Statement-II is the correct How many of the above-mentioned are
explanation for Statement-I. included under the definition of ‘Water
(b) Both Statement-I and Statement-II are bodies’ under the First Water Census of
correct and Statement-II is not the India?
correct explanation for Statement-I. (a) Only two
(c) Statement-I is correct but Statement-II is (b) Only three
incorrect. (c) Only four
(d) Statement-I is incorrect but Statement-II (d) Only five
is correct.
55. With reference to the objections raised for C.
52. Consider the following statements: Rajagopalachari Formula,1944, consider the
1. The first woman President of the Indian following statements:
National Congress was Annie Besant. 1. Mohammad Ali Jinnah wanted only the
2. The first English President of the Indian Muslims of North-West and North-East
National Congress was A. O. Hume. to vote in the plebiscite and not the
Which of the statements given above is/are entire population.
correct? 2. Hindu leaders led by Vir Savarkar
(a) 1 only condemned the formula.
(b) 2 only Which of the statements given above is/are
(c) Both 1 and 2 correct?
(d) Neither 1 nor 2 (a) 1 only
(b) 2 only
53. With reference to the Eighth Schedule of the (c) Both 1 and 2
Constitution, consider the following (d) Neither 1 nor 2
statements:
1. Parliament can add a language to the 56. Subhash Chandra Bose was associated with
Eighth Schedule by a simple majority. which of the following organizations?
2. Kannada, Konkani, and Malayalam were 1. Indian Independence League
among the fourteen languages included 2. Forward Bloc
in the Eighth Schedule of the original 3. Mukti Sena
Constitution. 4. Ghadar Party
Which of the statements given above is/are Select the correct answer from the code
correct? given below.
(a) 1 only (a) Only one
(b) 2 only (b) Only two
(c) Both 1 and 2 (c) Only three
(d) Neither 1 nor 2 (d) All four
11 www.visionias.in ©Vision IAS

FREE BY KING R QUEEN P [ऋषभ राजपूत]


57. With reference to Gongadi Shawls, consider 60. The Preamble embraces three dimensions of
the following statements: equality—civic, political and economic,
which is ensured through various Parts of the
1. These are woven by the indigenous
Indian Constitution. In this context, consider
Kuruma and Kuruba pastoralist
the following pairs:
communities. Dimensions of Parts of the
2. These are made using the coarse wool of Equality Constitution
Nalla gorrae sheep. 1. Civic equality : Part III
2. Political equality : Part XV
Which of the statements given above is/are
3. Economic equality : Part IV
correct?
How many of the pairs given above are
(a) 1 only correctly matched?
(b) 2 only (a) Only one
(c) Both 1 and 2 (b) Only two
(c) All three
(d) Neither 1 nor 2
(d) None

58. Consider the following pairs: 61. Consider the following statements regarding
Leaders of Civil Region the Santosh Trophy:
Disobedience Movement 1. It is Asia’s oldest football tournament in
which clubs from across the country
1. C. Rajgopalachari : Tamil Nadu
participate.
2. Imam Sahib : Lucknow 2. This year Mohun Bagan won this trophy.
3. K. Kelappan : Kerala Which of the statements given above is/are
How many pairs given above are correctly correct?
(a) 1 only
matched?
(b) 2 only
(a) Only one pair (c) Both 1 and 2
(b) Only two pairs (d) Neither 1 nor 2
(c) All three pairs
(d) None 62. Which of the following statements is not
correct about the Constituent Assembly?
(a) H.C. Mukherjee and V.T.
59. Consider the following pairs:
Krishnamachari were elected as the
Tribes State Vice-Presidents of the Constituent
1. Tiwa : Assam Assembly.
2. Kattunayakan : Tamil Nadu (b) The idea of a Constituent Assembly was
first proposed by J.L. Nehru.
3. Soliga : Madhya Pradesh
(c) It held its first meeting on 9th December
How many of the pairs given above is/are
1946, and elected Dr Sachchidanand
correctly matched? Sinha as the temporary President of the
(a) Only one Assembly.
(b) Only two (d) In the Assembly, seats allocated to each
British province were divided among the
(c) All three
Muslims, Sikhs and general in
(d) None
proportion to their population.
12 www.visionias.in ©Vision IAS

FREE BY KING R QUEEN P [ऋषभ राजपूत]


63. Consider the following statements regarding 65. Consider the following pairs:
Treaty Signed between
the Indian Independence Act of 1947: 1. Treaty of Sugauli, : Tibet and British
1. It empowered the Constituent 1816
2. Treaty of : Nepal and British
Assemblies to repeal any Act of the Yandaboo, 1826
3. Treaty of Lhasa : Afghanistan and
British Parliament, including the (1904) British
Independence Act itself. How many of the above pairs are correctly
matched?
2. It designated the Governor-General of (a) Only one
(b) Only two
India and the provincial governors as (c) All three
(d) None
constitutional heads of the states.

3. It provided for the partition of India with 66. Consider the following powers of the Indian
the right to secede from the British Parliament:
1. To admit into the Union of India new
Commonwealth. states
How many statements given above are 2. To increase or decrease the area of any
state
correct?
3. To establish new states
(a) Only one 4. To alter the boundaries of any state
5. To alter the name of any state
(b) Only two
How many of the above-given powers of
(c) All three Parliament are mentioned in the Article 3 of
(d) None the Indian Constitution?
(a) Only two
(b) Only three
64. Consider the following statements regarding (c) Only four
(d) All five
Prarthana Samaj:

1. It was established in 1863 by Atmaram 67. Consider the following statements regarding
Pandurang. Hunter Education Commission (1882-83) :
1. It emphasized the use of English as the
2. It confronted Hindu orthodoxy for social medium of instruction for primary
reforms. education.
2. It recommended transferring control of
3. It demanded widow remarriage and an
primary education to district and
increase of the marriage age for both municipal boards.
3. It suggested for the establishment of
males and females.
separate educational institutions for
How many of the above statements are females.
correct? How many of the above statements are
correct?
(a) Only one (a) Only one
(b) Only two (b) Only two
(c) All three
(c) All three
(d) None
(d) None
13 www.visionias.in ©Vision IAS

FREE BY KING R QUEEN P [ऋषभ राजपूत]


68. With reference to urban local bodies in 70. Consider the following statements:

India, consider the following pairs: Statement-I: Presently, no state government


can raise any loan without the consent of the
Urban Local Body Feature
Central Government.
1. Township : It is established Statement-II: Every state in India has some
by a notification outstanding loan in respect of which a
guarantee has been given by the Central
in the government
Government.
gazette. Which one of the following is correct in
2. Town Area : It is created by an respect of the above statements?
(a) Both Statement-I and Statement-II are
Committee Act of
correct and Statement-II is the correct
Parliament.
explanation for Statement-I.
3. Port Trust : It consists of both (b) Both Statement-I and Statement-II are

elected and correct and Statement-II is not the


correct explanation for Statement-I.
nominated
(c) Statement-I is correct but Statement-II is
members. incorrect.
How many of the pairs given above are (d) Statement-I is incorrect but Statement-II
is correct.
correctly matched?

(a) Only one 71. The Right to silence is derived from which
(b) Only two one of the following Constitutional

(c) All three provisions in India?


(a) Article 19
(d) None
(b) Article 20
(c) Article 21
69. With reference to Viscose fiber, consider the (d) Article 22

following statements:
72. Consider the following statements regarding
1. Viscose is a type of rayon. Project Taara :
2. It is a biodegradable fiber and is made 1. It aims to offer internet connectivity via
light beams.
from tree wood pulp.
2. It is an initiative of the International
Which of the statements given above is/are
Telecommunication Union.
correct? Which of the statements given above is/are

(a) 1 only correct?


(a) 1 only
(b) 2 only
(b) 2 only
(c) Both 1 and 2 (c) Both 1 and 2
(d) Neither 1 nor 2 (d) Neither 1 nor 2
14 www.visionias.in ©Vision IAS

FREE BY KING R QUEEN P [ऋषभ राजपूत]


73. Consider the following statements regarding 76. Emergence of titles like Jagath
"The Code of Gentoo Law": Seth and Nagar Seth were given to
1. It was an attempt by Britishers to codify (a) local wazirs
Indian laws. (b) bankers
2. It came during the tenure of Lord (c) army officials
Curzon. (d) judicial officers
Which of the statements given above is/are
77. Consider the following statements regarding
correct?
XPoSat Satellite (X-ray Polarimeter
(a) 1 only
Satellite):
(b) 2 only
1. It is aimed at studying various dynamics
(c) Both 1 and 2
of astronomical sources in extreme
(d) Neither 1 nor 2
conditions.
2. It is the world’s first polarimetry mission
74. Consider the following statements: using X-Ray.
Statement-I: The Praja Mandal movement 3. It is a collaboration between the ISRO
started in India in the 1920s. and the Raman Research Institute.
Statement-II: The Indian National Congress How many of the above statements are
(INC) passed a resolution to start the praja correct?
mandal movement at the Haripura session. (a) Only one
Which one of the following is correct in (b) Only two
respect of the above statements? (c) All three
(a) Both Statement-1 and Statement-II are (d) None
correct and Statement-II is the correct
explanation for Statement-I. 78. Consider the following national leaders:
(b) Both Statement-I and Statement-II are 1. K.T. Telang
2. R.G Bhandarkar
correct and Statement-II is not the
3. Dadoba Pandurang
correct explanation for Statement-I.
How many of the above advocated for the
(c) Statement-I correct but Statement-II is
Orientalist method of social reforms?
incorrect.
(a) Only one
(d) Statement-I is incorrect but Statement-II
(b) Only two
is correct.
(c) All three
(d) None
75. Consider the following:
1. 73rd constitutional amendment 79. Consider the following National Parks:
2. 36th Constitutional amendment 1. Mollem National Park
3. 100th constitutional amendment 2. Periyar National Park
How many of the amendments mentioned 3. Mhadei Wildlife Sanctuary
above are responsible for the expansion of Malabar Tree Toad is found in which of the
the ‘Union of India’? above-mentioned National Parks?
(a) Only one (a) Only one
(b) Only two (b) Only two
(c) All three (c) All three
(d) None (d) None
15 www.visionias.in ©Vision IAS

FREE BY KING R QUEEN P [ऋषभ राजपूत]


80. Consider the following pairs : 83. Consider the following statements regarding
Protected Area State stablecoins:

1. Mukurthi National : Karnataka 1. Their value is pegged or tied to that of


another currency, commodity, or
Park
financial instrument.
2. Sahyadri Tiger : Maharashtra
2. They can use cryptocurrencies as the
Reserve
underlying collateral.
3. Bura chapori : Assam Which of the statements given above is/are
Wildlife Sanctuary correct?
How many of the pairs given above are (a) 1 only
correct? (b) 2 only

(a) Only one (c) Both 1 and 2


(d) Neither 1 nor 2
(b) Only two
(c) All three
84. Consider the following minerals:
(d) None
1. Antimony
2. Beryllium
81. Consider the following statements regarding 3. Zinc
Leadership in Energy and Environmental 4. Cobalt
Design Zero Certification (LEED): 5. Chromium

1. It is an initiative of the International How many of the above-mentioned minerals


are examples of critical minerals according
Energy Agency.
to the Government of India?
2. India ranks first globally in LEED Zero-
(a) Only two
certified green building projects.
(b) Only three
Which of the statements given above is/are (c) Only four
correct? (d) All five
(a) 1 only
(b) 2 only 85. Consider the following pairs:

(c) Both 1 and 2 Peshwa Contemporary


Mughal Emperor
(d) Neither 1 nor 2
1. Baji Rao I : Farrukhsiyar
2. Balaji Baji Rao : Muhammad Shah
82. In the context of volcanism, fumarole is a:
3. Baji Rao II : Shah Alam II
(a) snow capped volcano.
How many of the above pairs are correctly
(b) opening in the earth’s surface that emit
matched?
steam and volcanic gases. (a) Only one
(c) coarse grained plutonic rock. (b) Only two
(d) mantle plume. (c) All three
(d) None
16 www.visionias.in ©Vision IAS

FREE BY KING R QUEEN P [ऋषभ राजपूत]


86. Consider the following statements regarding 89. Consider the following pairs:
Spheroidal Carbonaceous Particles (SCP): Women Organisations Founders
1. It is a component of flyash.
1. Ladies Social : Sarla Devi
2. It has no other anthropogenic or natural
Conference Chaudhurani
source other than fossil fuel combustion.
Which of the statements given above is/are
2. Arya Mahila Samaj : Pandita Ramabai
correct?
Sarasvati
(a) 1 only
(b) 2 only 3. Bharat Stree : Ramabai Ranade

(c) Both 1 and 2 Mahamandal


(d) Neither 1 nor 2
How many of the above pairs are correctly

87. Consider the following pairs: matched?


Place in news Country (a) Only one
1. Mahdia : Guyana (b) Only two
2. Belgorod : Ukraine
(c) All three
3. Dnipro : Russia
(d) None
How many of the pairs given above are
correctly matched?
(a) Only one 90. Consider the following statements regarding
(b) Only two the Hallmark Unique Identification (HUID)
(c) All three number:
(d) None
1. A hallmark is a mark on gold jewellery
which is affixed by an entity recognised
88. Consider the following statements:
1. Congress under the influence of by the Bureau of Indian Standards.

Gandhiji believed that the cause of the 2. HUID number is a unique six-digit
Allies (Britain and France) in war was alphanumeric code given to every piece
just of jewellery.
2. Lack of Hindu- Muslim unity
3. 22K916 means that it is 22-carat gold
3. Indiscipline and lack of cohesion within
and the piece of jewellery has 91.6
the Congress ranks
How many of the above are the possible percent gold.

reasons for not initiating an immediate mass How many of the statements given above are
movement in the wake of World War II by correct?
the Congress? (a) Only one
(a) Only one
(b) Only two
(b) Only two
(c) All three
(c) All three
(d) None (d) None
17 www.visionias.in ©Vision IAS

FREE BY KING R QUEEN P [ऋषभ राजपूत]


91. Which of the following statements is not 94. Consider the following statements regarding
correct regarding the National Register of African Union:
Citizens (NRC)? 1. Only the countries of the African
(a) The first National Register of Citizens continent are the members of African
was compiled in 1951, after the Census Union.
2. Its headquarters is located in Cape
was completed that year.
Town, South Africa.
(b) Anyone who cannot prove that they or
3. The combined population of the member
their ancestors entered in India before
states exceeds 2 billion.
the midnight of March 24, 1971, would
How many of the statements given above are
be declared a foreigner.
correct?
(c) It is a unique exercise in which the onus (a) Only one
to prove citizenship lies with the (b) Only two
citizens. (c) All three
(d) Those who are declared foreigners will (d) None
be deported to Bangladesh under the
repatriation treaty and Assam Accord. 95. Which one of the following was the most
significant feature of the Congress election
92. Consider the following statements with campaign during the Indian General
respect to the ad-hoc judges of the Supreme Elections of 1945?
Court of India: (a) Protest against the misdeeds of the Nazis
in Germany
1. The qualifications for appointment are
(b) Massive turnout of Muslims at election
the same as those of becoming a
meetings
Supreme Court judge under normal
(c) Political mobilization of Indians against
circumstances.
British rule
2. The president determines the need for an
(d) Election campaigning by Gandhi
ad-hoc judge in the Supreme Court as
per the recommendations of the chief 96. Consider the following pairs:
justice of India. Policy towards Governor
Which of the statements given above is/are Afghanistan General/Viceroy
correct? 1. Forward Policy : Lord Lawrence
(a) 1 only 2. Policy of Proud : Lord Lytton
(b) 2 only Reserve
(c) Both 1 and 2 3. Policy of : Lord Curzon
(d) Neither 1 nor 2 Withdrawal and
Concentration
93. Who among the following Portuguese How many of the above pairs are correctly
Governor Generals abolished 'Sati' in Goa? matched?
(a) Francisco De Almeida (a) Only one
(b) Alfonso de Albuquerque (b) Only two

(c) Nino da Cunha (c) All three


(d) None
(d) Vedor da Fazenda
18 www.visionias.in ©Vision IAS

FREE BY KING R QUEEN P [ऋषभ राजपूत]


97. Consider the following statements regarding 100. With reference to the Cabinet Committee on
the impeachment of the President: Parliamentary Affairs, recently seen in the
1. A President can only be impeached for a news, consider the following statements:
‘violation of the Constitution’.
1. It determines the date and duration of
2. The charge of ‘violation of the
parliamentary sessions in India.
Constitution’ can be preferred only after
2. At present, the Prime Minister of India
a signed resolution by 2/3rd of the total
acts as an ex-officio chairperson of the
members of the house pressing the
charge. committee.
Which of the statements given above is/are Which of the statements given above is/are
correct? correct?
(a) 1 only (a) 1 only
(b) 2 only (b) 2 only
(c) Both 1 and 2
(c) Both 1 and 2
(d) Neither 1 nor 2
(d) Neither 1 nor 2
98. The appellate jurisdiction of the High courts
extends to how many of the following?
1. Administrative tribunals
2. National Green Tribunal
3. Armed Forces Tribunal
Select the correct answer using the code
given below.
(a) Only one
(b) Only two
(c) All three
(d) None

99. Who among the following are involved in


the defense of the INA prisoners?
1. K.N. Katju
2. Asaf Ali
3. Jawahar Lal Nehru
4. Tej Bahadur Sapru
Select the correct answer using the code
given below.
(a) Only one
(b) Only two
(c) Only three
(d) All four

Copyright © by Vision IAS


All rights are reserved. No part of this document may be reproduced, stored in a retrieval system or
transmitted in any form or by any means, electronic, mechanical, photocopying, recording or otherwise,
without prior permission of Vision IAS.

19 www.visionias.in ©Vision IAS

FREE BY KING R QUEEN P [ऋषभ राजपूत]


VISIONIAS
www.visionias.in

Test Booklet Series

TEST BOOKLET

GENERAL STUDIES (P) 2024 – Test – 4141


C
Time Allowed: Two Hours Maximum Marks: 200

INSTRUCTIONS

1. IMMEDIATELY AFTER THE COMMENCEMENT OF THE EXAMINATION, YOU SHOULD CHECK THAT THIS BOOKLET
DOES NOT HAVE ANY UNPRINTED OR TURN OR MISSING PAGES OR ITEMS, ETC. IF SO, GET IT REPLACED BY A
COMPLETE TEST BOOKLET.

2. ENCODE CLEARLY THE TEST BOOKLET SERIES A, B, C OR D AS THE CASE MAY BE IN THE APPROPRIATE PLACE IN
THE ANSWER SHEET.

3. You have to enter your Roll Number on the Test Booklet in the Box
provided alongside. Do NOT write anything else on the Test Booklet.

4. This Test Booklet contains 100 items (Questions). Each item is printed in English. Each item comprises four
responses (answers). You will select the response which you want to mark on the Answer Sheet. In case you
feel that there is more than one correct response with you consider the best. In any case, choose ONLY ONE
response for each item.

5. You have to mark all your responses ONLY on the separate Answer Sheet provided. See direction in the
answers sheet.

6. All items carry equal marks. Attempt all items. Your total marks will depend only on the number of correct
responses marked by you in the answer sheet. For every incorrect response 1/3rdof the allotted marks will be
deducted.

7. Before you proceed to mark in the Answer sheet the response to various items in the Test booklet, you have to
fill in some particulars in the answer sheets as per instruction sent to you with your Admission Certificate.

8. After you have completed filling in all responses on the answer sheet and the examination has concluded, you
should hand over to Invigilator only the answer sheet. You are permitted to take away with you the Test
Booklet.

9. Sheet for rough work are appended in the Test Booklet at the end.

DO NOT OPEN THIS BOOKLET UNTIL YOU ARE ASKED TO DO SO


1 www.visionias.in ©Vision IAS

FREE BY KING R QUEEN P [ऋषभ राजपूत]


1. Consider the following statements regarding 4. Consider the following:
the Son River: 1. Clear skies and a rise in temperature
1. It originates near the Amarkantak hills. 2. Oppressive weather
3. Southward movement of a low-pressure
2. Its course covers a total of three states.
trough in the Plains of Ganga
3. Gharials are found in the Son River.
4. Rainfall on the southeastern coast
How many of the statements given above are Which of the following has the
correct? characteristics mentioned above?
(a) Only one (a) Breaks in monsoon
(b) Only two (b) Summer season
(c) An El Nino year
(c) All three
(d) Retreating monsoon
(d) None

5. Consider the following statements:


2. Consider the following statements regarding Statement I: Maximum number of solar
Isotherms: eclipses in a year is 2.
1. The spacing of isotherms depicts the Statement II: Earth's orbit is elliptical in
temperature gradient across a portion of shape.
Which one of the following is correct in
the Earth's surface.
respect of the above statements?
2. Widely spaced isotherms indicate a
(a) Both Statement-I and Statement-II are
small change in temperature over correct and Statement-II is the correct
distance and closely spaced isotherms explanation for Statement-I
indicate large changes in temperature. (b) Both Statement-I and Statement-II are
Which of the statements given above is/are correct and Statement-II is not the
correct explanation for Statement-I
correct?
(c) Statement-I is correct but Statement-II is
(a) 1 only
incorrect
(b) 2 only (d) Statement-I is incorrect but Statement-II
(c) Both 1 and 2 is correct
(d) Neither 1 nor 2
6. With reference to the Defence Acquisition
3. These types of soils constitute about 10 to Council (DAC), consider the following
statements:
40% of the organic matter and also a
1. It was formed post-Kargil war in 2001.
reasonable number of soluble salts. They are
2. The Prime minister is the chairman of
low in phosphate and potash content. They DAC and the defense minister is the
are found in the Almora district of Vice-chairman.
Uttarakhand, and also a few districts of 3. It decides capital acquisitions for all
Kerala. three services i.e. Army, Navy and Air
Force as well as Indian coast guard.
The above description relates to which of
How many of the above statements are
following types of soil?
correct?
(a) Peaty Soils (a) Only one
(b) Saline Soils (b) Only two
(c) Alluvial Soils (c) All three
(d) Arid Soils (d) None
2 www.visionias.in ©Vision IAS

FREE BY KING R QUEEN P [ऋषभ राजपूत]


7. Consider the following statements regarding 10. Consider the following pairs in the context
coral reefs: of rainfall distribution in India:
1. A fringing reef is a coralline platform
Rainfall intensity Region
lying close to the shore extending
1. Extreme rainfall : Arunachal
outwards from the mainland.
2. A barrier reef is separated from the coast Pradesh
by a much wider and deeper channel or 2. Heavy rainfall : Nagaland
lagoon. : West Bengal
3. Moderate Rainfall
3. Atolls are similar to barrier reefs except
How many pairs given above are correctly
that they are circular in shape, enclosing
a shallow lagoon. matched?
How many of the statements given above are (a) Only one
correct? (b) Only two
(a) Only one
(c) All three
(b) Only two
(d) None
(c) All three
(d) None
11. What is the primary driving force behind
8. Consider the following statements: isostatic adjustments in the Earth's crust?
Statement-I: The western disturbances
(a) Tectonic plate movements
arriving in India have either delayed or
(b) Volcanic eruptions
weakened in the past few years.
Statement-II: During December month, sub- (c) Buoyancy and gravitational forces
tropical westerly jet streams have been (d) Earth's magnetic field
shifting northwards.
Which one of the following is correct in
12. Chennai has a hot and humid tropical
respect of the above statements?
climate with temperatures often exceeding
(a) Both Statement-I and Statement-II are
correct and Statement - II is the correct 35°C in summer and milder winters with
explanation for Statement-I temperatures around 20-25°C while
(b) Both Statement-I and Statement-II are Hyderabad experiences hotter summers with
correct and Statement - II is not the
temperatures reaching 40-45°C and slightly
correct explanation for Statement-I
cooler winters with temperatures around 15-
(c) Statement-I is correct but statement - II
is incorrect 25°C.
(d) Statement -I is incorrect but Statement - Which of the following best explains the
II is correct phenomenon?
(a) Chennai is situated at a higher elevation
9. The "Great Oxygenation Event" during the
compared to Hyderabad.
Precambrian Era was a significant geological
event that led to the accumulation of oxygen (b) Longitudinally, Chennai is situated to
in Earth's atmosphere. What was the primary the east of Hyderabad.
source of this oxygen? (c) Chennai experiences maritime climate
(a) Volcanic activity
while Hyderabad has a more continental
(b) Cyanobacteria
climate.
(c) Meteor impacts
(d) Vascular Plants (d) None of the above
3 www.visionias.in ©Vision IAS

FREE BY KING R QUEEN P [ऋषभ राजपूत]


13. With reference to the Himalayan region, 16. Consider the following planets:
consider the following statements: 1. Venus
1. Dun formations are part of the Lesser 2. Mars
Himalayan range. 3. Jupiter
2. The Kanchenjunga Peak is located in the How many of the planets given above take
more time than earth to revolve around the
Shiwalik range.
Sun?
3. In the west, the Great Himalayan range
(a) Only one
ends at Nanga Parbat.
(b) Only two
How many statements given above are
(c) All three
correct? (d) None
(a) Only one
(b) Only two 17. In the context of the Indian monsoon,
(c) All three consider the following statements:
(d) None 1. The monsoon rainfall has a declining
trend with decreasing distance from the
14. Consider the following statements: sea.
1. Clear sky, calm air, high relative 2. The variability of rainfall is among the
highest in the northern part of Jammu
humidity, and cold and long nights are
and Kashmir.
ideal conditions for formation of Dew.
Which of the statements given above is/are
2. For the formation of dew, it is necessary
correct?
that the dew point is below the freezing (a) 1 only
point. (b) 2 only
Which of the statements given above is/are (c) Both 1 and 2
correct? (d) Neither 1 nor 2
(a) 1 only
(b) 2 only 18. The Kaimur Hills are situated to the:
(c) Both 1 and 2 (a) North of Hazaribagh Plataeu.
(d) Neither 1 nor 2 (b) South of Baghelkhand.
(c) East of Bundelkhand.
(d) West of Vindhyan range.
15. "This type of climate is found in the western
coastal region, south of Mumbai. This area
19. With reference to the salinity in water
receives over 300 cm of annual rainfall in
bodies, consider the following statements:
summer from the south-west monsoons." 1. Equatorial waters are highly saline due
According to Koppen's classification, the to high temperatures at the equator.
above mentioned paragraph refers to which 2. Lake Van, one of the most saline water
of the following climatic regions? bodies, is located in Turkey.
(a) Monsoon type with short dry winter Which of the statements given above is/are
season correct?
(b) Monsoon type with dry season in high (a) 1 only
sun period (b) 2 only
(c) Both 1 and 2
(c) Monsoon type with dry winters
(d) Neither 1 nor 2
(d) Tropical Savannah type
4 www.visionias.in ©Vision IAS

FREE BY KING R QUEEN P [ऋषभ राजपूत]


20. Consider the following: 23. Which of the following best explains the
1. Ria coasts general absence of lightning during the
2. Fjord coasts active phase of the monsoon season?
3. Dalmatian coasts (a) Monsoon clouds are mainly stratiform
How many of the above are features of clouds and lightening is mainly
submergence coast? associated with convective clouds.
(a) Only one (b) Monsoon clouds are mainly convective
(b) Only two clouds and lightening is mainly
(c) All three associated with stratiform clouds.

(d) None (c) Interaction of Convective and stratiform


clouds negates the possibility of
lightening.
21. With reference to the Kaveri River, consider
(d) None of the above
the following statements:
1. It rises in Brahmagiri hills.
24. Consider the following mountain ranges:
2. It covers the states of Kerala, Karnataka,
1. Alps
Tamil Nadu, and the Union territory of
2. Andes
Puducherry.
3. Sierra Nevada
3. It thrice forks into two streams and
4. Appalachian Mountains
reunites forming the islands of
How many of the above mountain ranges are
Srirangappattanam, Sivasamudram, and
fold mountains?
Srirangam.
(a) Only one
How many of the above statements are
(b) Only two
correct?
(c) Only three
(a) Only one
(d) All four
(b) Only· two
(c) All three 25. Consider the following statements regarding
(d) None the Intergovernmental Platform on
Biodiversity and Ecosystem Services
22. Consider the following countries: (IPBES):
1. India 1. It is a United Nations climate science
2. USA body.
3. Brazil 2. It was established by the Busan outcome
4. France document.
5. Argentina 3. All member states of the United Nations
How many of the above countries are are eligible for IPBES membership.
founding members of Global Biofuel Which of the statements given above is/are
Alliance? correct?
(a) Only two (a) 1 only
(b) Only three (b) 2 only
(c) Only four (c) 2 and 3 only
(d) All five (d) 1, 2 and 3
5 www.visionias.in ©Vision IAS

FREE BY KING R QUEEN P [ऋषभ राजपूत]


26. Arrange the following hills in India from 29. Storm Daniel, recently seen in the news
North to South: caused floods in
1. Satmala hills (a) Morocco
2. Kaimur hills (b) Libya
3. Pir Panjal Range (c) Brazil

Select the correct answer using the code (d) Argentina

given below.
(a) 2-3-1 30. Consider the following statements:

(b) 1-2-3 1. Long Period Average of rainfall is the


rainfall recorded over a particular region
(c) 1-3-2
for a given interval average over a long
(d) 3-2-1
period like 30 years, 50 years, etc.
2. Long-range forecast of a monsoon is the
27. Consider the following statements regarding
forecast over a period ranging from 15
the trade winds:
years to 25 years.
1. These are westerly winds.
Which of the statements given above is/are
2. They are humid and warmer in the areas
correct?
of their origin.
(a) 1 only
3. Strong trade winds are associated with
(b) 2 only
heavy precipitation.
(c) Both 1 and 2
4. The Southeast Asian monsoon, is a
(d) Neither 1 nor 2
seasonal, moisture-laden trade wind.
How many of the statements given above are
31. Consider the following pairs:
correct?
Moon Description
(a) Only one 1. Supermoon : Occurs when a full
(b) Only two moon coincides with its
(c) Only three apogee point
(d) All four 2. Blood moon : Occurs when the moon
is total lunar eclipse
28. Which of the following factors primarily 3. Blue moon : The second full moon
influences the texture or the size of mineral within a single calendar
grains of igneous rocks? month
(a) The rate of deposition and the rate of How many of the pairs given above are
erosion at a site correctly matched?
(b) Proximity to other rocks of similar (a) Only one
nature (b) Only two
(c) Rate of cooling of the molten material (c) All three

(d) Amount of organic material present (d) None


6 www.visionias.in ©Vision IAS

FREE BY KING R QUEEN P [ऋषभ राजपूत]


32. Consider the following pairs: 36. Classified as ‘Southern Montane Wet
Rivers Tributary Temperate Forest’, these are found in the
1. Wardha : Penganga upper reaches of the Nilgiris, Anamalais,
2. Mahanadi : Seonath Palni hills, Kalakadu, Mundanthurai and
3. Damodar : Barakar Kanyakumari in the states of Tamil Nadu
4. Kaveri : Bhavani and Kerala. The introduction of alien plant
How many of the above pairs are correctly species and frequent fires have threatened
matched? their existence. Recently, the forest type has
(a) Only one been in the news due to the issue of its
(b) Only two classification.
(c) Only three Which of the following is best described by
(d) All four the passage given above?
(a) Sal forests
33. Gujarat Declaration, 2023 which was (b) Silent valley rainforests
recently seen in the news is related to (c) Neelkurinji tracts
(a) Traditional Medicine (d) Shola forests
(b) Green Hydrogen
(c) International Debt 37. The general elevation ranges between 700-
(d) Biofuels 1,000 m above the mean sea level and it
slopes towards the north and northeastern
34. Consider the following statements regarding directions. Most of the tributaries of the river
the troposphere:
Yamuna have their origin In this region. An
1. The temperature in the troposphere
eastern extension is formed by the Rajmahal
changes at a positive lapse rate.
hills, to the south of which lies a large
2. All weather phenomena like cyclones,
reserve of mineral resources in the
rainfall, fog and hailstorms, etc. are
Chotanagpur plateau. In the West, its
confined to this layer.
extension can be seen as far as Jaisalmer.
3. The temperature at the tropopause is
Which of the following is best described in
nearly constant.
the passage given above?
How many statements given above are
(a) Deccan Plataeu
correct?
(b) Central Highlands
(a) Only one
(c) Maikal ranges
(b) Only two
(d) The Eastern Ghats
(c) All three
(d) None
38. Consider the following pairs:
Rivers Origin State
35. Consider the following pairs:
1. Myntdu : Meghalaya
Satellite Planet
2. Barak : Manipur
1. Phobos : Mars
3. Tizu : Nagaland
2. Ganymede : Saturn
3. Titan : Jupiter 4. Gomati : Tripura
How many of the pairs given above are How many of the above pairs are correctly
correctly matched? matched?
(a) Only one (a) Only one
(b) Only two (b) Only two
(c) All three (c) Only three
(d) None (d) All four
7 www.visionias.in ©Vision IAS

FREE BY KING R QUEEN P [ऋषभ राजपूत]


39. Which type of fault results from the 42. Consider the following conditions regarding
horizontal compression of rocks, causing one Delta formation:
block to move upward relative to the other? 1. High sediment load of the river
(a) Normal fault 2. Turbulent waves near the mouth of the

(b) Reverse fault river

(c) Strike-slip fault 3. Shallow coastal waters

(d) Transform fault 4. Multiple distributaries of the river


How many of the above conditions are

40. With reference to ocean currents, consider favorable for delta formation?

the following statements: (a) Only one

1. Labrador current is a cold ocean current (b) Only two


(c) Only three
flowing from the Arctic Ocean south
(d) All four
along the east coast of Canada.
2. Kuroshio Current north-flowing warm
43. If you are traveling westward across the
current off the coast of Japan in the
International Date Line, how does your
Pacific Ocean.
calendar date change?
3. The Gulf Stream is a warm current that
(a) It moves one day ahead.
flows from the Gulf of Mexico to the
(b) It moves one day behind
Atlantic coast of Europe.
(c) It remains the same.
4. Agulhas Current is a warm current found
(d) It skips two days.
along the east coast of Africa.
How many of the statements given above
44. With reference to the Indian Ocean Dipole
is/are correct?
(IOD), consider the following statements:
(a) Only one
1. It is characterized by differences in sea
(b) Only two
surface temperatures (SSTs) between the
(c) Only three
western and eastern Indian Ocean.
(d) All four
2. Negative IOD can lead to increased
rainfall and atmospheric convection over
41. Which of the following protected sites is the eastern Indian Ocean, Indonesia, and
being prepared for the introduction of northern Australia.
Cheetahs as an alternative to the Kuno Which of the statements given above is/are
National Park? not correct?
(a) Kanha Tiger Reserve (a) 1 only
(b) Ratapani Wildlife Sanctuary (b) 2 only
(c) Gandhi Sagar Wildlife Sanctuary (c) Both 1 and 2
(d) Sariska Tiger Reserve (d) Neither 1 nor 2
8 www.visionias.in ©Vision IAS

FREE BY KING R QUEEN P [ऋषभ राजपूत]


45. In the context of Earth's geological history, 48. Arrange the following in decreasing order of
the terms Vaalbara, Rodinia, Pannotia refer the albedo values.
to: 1. Cirrus Clouds
(a) Ancient species of dinosaurs that thrived 2. Sand
during the Paleozoic era. 3. Ice and Snow
(b) Ancient supercontinents that existed at 4. Water
different points in Earth's history. Select the correct answer using the code
(c) Geological epochs associated with mass given below.
extinctions.
(a) 1-3-2-4
(d) Subduction zones responsible for the
(b) 3-1-2-4
formation of mountain ranges.
(c) 3-2-4-1
(d) 2-3-4-1
46. It is one of the oldest forms of weaving in
the world. This style of weaving from
49. The term 'Brandt line' is often seen in the
Nagaland was showcased at 'Crafts Bazaar'
during G20 Summit. Dramatic color news recently. Which of the following
combinations, bold patterns, and striking statements best describes the term 'Brandt
minimalist geometric motifs mark this form. line'?
Most of the patterning is done in geometric (a) It is a line that tracks Earth Overshoot
designs, except for the warrior shawls of the days over the years when humanity
Ao tribe, which have hand-painted central consumes more natural resources than
band depicting tigers, bison, crossed spears, the Earth can regenerate.
and roosters. (b) It is a line showing hypothesized
Which style of weaving is best described by relationship between environmental
the passage given above? quality and economic development of a
(a) Tanchoi weaving nation.
(b) Mekhla Cheddar Weaving (c) It refers to the border separating pre-
(c) Moirang phee weaving 1967 Israel from the erstwhile Palestine.
(d) Loin Loom Weaving (d) It is a curve that divides the world into
the economically rich North and poor
47. Consider the following statements regarding
South.
forces affecting the movement of wind:
1. In the movement of wind the Coriolis
50. Which of the following statements is not
force acts parallel to the direction of the
correct regarding the Sudan type of climate?
pressure gradient force.
(a) It is a transitional type of climate found
2. When there is no friction and the
between the equatorial forests and the
pressure gradient force is balanced by
the Coriolis force, the resultant wind is trade wind hot deserts.
known as the geostrophic wind. (b) It is characterized by an alternate hot,
Which of the statements given above is/are rainy season and cool, dry season.
correct? (c) The annual precipitation is more than
(a) 1 only that of the Tropical Monsoon Climate.
(b) 2 only (d) The extreme diurnal range of
(c) Both 1 and 2 temperature is another characteristic
(d) Neither 1 nor 2 feature of it.
9 www.visionias.in ©Vision IAS

FREE BY KING R QUEEN P [ऋषभ राजपूत]


51. Consider the following statements: 54. Consider the following statements regarding
Statement-I: Soil degradation issues can be temperature inversion:
solved with biogas plants. 1. A long winter night with cloudy skies
Statement-II: Biogas plants replenish organic and moving air is an ideal situation for
material and phosphorous content in the temperature inversion.
soils. 2. This phenomenon commonly occurs in
Which one of the following is correct in hills and mountains.
respect of the above statements? Which of the statements given above is/are
correct?
(a) Both Statement-I and Statement-II are
(a) 1 only
correct. and Statement-II is the correct
(b) 2 only
explanation for Statement-I
(c) Both 1 and 2
(b) Both Statement-I and Statement-II
(d) Neither 1 nor 2
correct and Statement-II is not the
correct explanation for Statement-I
55. In 2023, the United Nations Convention
(c) Statement-I is correct but Statement-II is
against Transnational Organized Crime
incorrect (UNTOC) entered the 20th year of its
(d) Statement-I is incorrect but Statement-II implementation. In this context, consider the
is correct following statements:
1. It is a global legally binding instrument
52. The terms 'ChaSTE' and 'ILSA' often seen in to fight transnational organized crime.
the news recently, are 2. UNTOC has a 'Protocol on Prohibitions
(a) particle accelerators in CERN or Restrictions on the Use of Incendiary
(b) modules of International Space Station Weapons'.
(c) payloads on Chandrayan-3's Vikram 3. Enforcement Directorate is India's
lander national nodal agency for cross-border
(d) supercomputers developed by C-DAC assistance on matters relating to the
Convention.
53. Consider the following statements regarding How many of the above statements are
earthquakes: correct?
(a) Only one
1. Epicenter is the location below the
(b) Only two
earth’s surface where the earthquake
(c) All three
starts and hypocentre is the location
(d) None
directly above it on the surface.
2. Only natural processes like plate
56. With reference to the Orange Pekoe tea
movements and faulting are capable of
recently seen in the news, consider the
causing earthquakes.
following statements:
3. Earthquakes cannot be predicted in 1. Orange pekoe is the grade name for
advance as there are no instruments black tea.
invented for the same. 2. It is cultivated in Darjeeling.
Which of the statements given above is/are Which of the statements given above is/are
correct? correct?
(a) 1 and 2 only (a) 1 only
(b) 1 and 3 only (b) 2 only
(c) 3 only (c) Both 1 and 2
(d) 2 and 3 only (d) Neither 1 nor 2
10 www.visionias.in ©Vision IAS

FREE BY KING R QUEEN P [ऋषभ राजपूत]


57. Consider the following statements regarding 60. The term 'Abraham Accords' is recently seen
the Laurentian type of climate:
in the news in the context of
1. It has cold, dry winters and warm
(a) Armenia-Azerbaijan conflict
summers.
2. It has features of both the maritime and (b) Afghanistan peace-talks between US and
the continental climates. Taliban
3. In the southern hemisphere, this climate
(c) Piracy in the Persian Gulf region
type is absent.
How many statements given above are not (d) Israel-Palestine Conflict
correct?
(a) Only one
61. Consider the following statements:
(b) Only two
(c) All three 1. Excess soil salinity causes uneven,
(d) None stunted growth and poor yields in crops.

2. The osmotic pressure of the soil solution


58. With reference to the Earth's magnetic poles,
consider the following statements: increases as the salt concentration
1. The magnetic poles are stationary and do increases.
not change over time.
Which of the statements given above is/are
2. The magnetic poles coincide exactly
with the geographic poles. correct?
Which of the statements given above is/are (a) 1 only
correct?
(b) 2 only
(a) 1 only
(b) 2 only (c) Both 1 and 2
(c) Both 1 and 2 (d) Neither 1 nor 2
(d) Neither 1 nor 2

59. With reference to tides, consider the 62. Consider the following pairs:
following statements: Plateau Continent
1. Together gravitational pull and the
1. Massif Central : Europe
centrifugal forces are responsible for
creating the two major tidal bulges on 2. Loess Plateau : Asia

the Earth. 3. Katanga : Africa


2. Normally, there is a fifteen-day interval
How many of the pairs given above are
between the spring tides and neap tides.
correctly matched?
Which of the statements given above is/are
correct? (a) Only one
(a) 1 only
(b) Only two
(b) 2 only
(c) All three
(c) Both 1 and 2
(d) Neither 1 nor 2 (d) None
11 www.visionias.in ©Vision IAS

FREE BY KING R QUEEN P [ऋषभ राजपूत]


63. Consider the following statements regarding 66. The first Global Stocktake synthesis report,
the extratropical cyclones and tropical 2023 has been released by
cyclones:
(a) The Intergovernmental Panel on Climate
1. Extratropical cyclones can originate over
the sea but not on land. Change (IPCC)
2. The wind velocity of a tropical cyclone (b) The United Nations Environment
is much higher and more destructive
Programme (UNEP)
than an extratropical cyclone.
3. The extra tropical cyclones move from (c) The United Nations Framework
west to east but tropical cyclones, move Convention on Climate Change
from east to west. (UNFCCC).
Which of the statements given above is/are
(d) The Global Environment Facility (GEF).
correct?
(a) 1 only
(b) 1 and 2 only 67. Consider the following statements:
(c) 2 and 3 only
1. Samvidhan Sadan is a name given to the
(d) 1, 2 and 3
new Parliament House.
64. Consider the following statements regarding 2. The old Parliament House was designed
Electromagnetic Ion Cyclotron (EMIC) by British architects Sir Edwin Lutyens
Waves:
and Herbert Baker.
1. These waves are the discreet
electromagnetic emissions observed in Which of the statements given above is/are
the Earth's magnetosphere. correct?
2. These waves are generated in the (a) 1 only
equatorial latitudes.
(b) 2 only
3. These waves can resonate with charged
particles causing them to move in a (c) Both 1 and 2
cyclical motion. (d) Neither 1 nor 2
Which of the statements given above is/are
correct?
(a) 1 only 68. With reference to World Soil Day, consider
(b) 2 and 3 only the following statements:
(c) 1 and 2 only 1. Due to the efforts of the United Nations,
(d) 1, 2 and 3
it is celebrated every year on December

65. Consider the following statements regarding 5.


the Aphelion position of the Earth: 2. "Soils: Where Food Begins" was the
1. It occurs when the Earth is at its greatest
campaign for World Soil Day 2022.
distance from the Sun.
2. It occurs during the month of July. Which of the statements given above is/are
Which of the statements given above is/are correct?
correct? (a) 1 only
(a) 1 only
(b) 2 only
(b) 2 only
(c) Both 1 and 2 (c) Both 1 and 2
(d) Neither 1 nor 2 (d) Neither 1 nor 2
12 www.visionias.in ©Vision IAS

FREE BY KING R QUEEN P [ऋषभ राजपूत]


69. Recently a magnificent Nataraja sculpture 72. Due to which of the following reasons the
has been installed at 'Bharat Mandapam', the monsoon depressions during the south west
venue of the G20 Leaders' Summit in Delhi. monsoon season generally do not intensify
In this context, consider the following into cyclonic storms?
statements regarding Nataraja sculpture.
(a) Presence of high pressure belt on the
1. The statue is an ashtadhatu (eight-metal
subcontinent
alloy) piece of art, crafted by sculptors
(b) Presence of low pressure belt on the
from Swamimalai in the Thanjavur
Indian ocean
district of Tamil Nadu.
2. The traditional ‘lost-wax’ casting (c) High vertical wind shear
method has been used to create the (d) Lack of vertical wind shear
sculpture.
3. All three temples that inspired the 73. Which of the following factors leads to a
Bharat Mandapam Nataraja statue were reduction in soil organic matter in an open
originally constructed by the Cholas. cycle system?
How many of the statements given above are
1. Low harvest index
correct?
2. Overgrazing
(a) Only one
3. Tillage practices
(b) Only two
Select the correct answer from the code
(c) All three
(d) None given below.
(a) 1 only
70. Which of the following is not a direct effect (b) 1 and 3 only
of the Earth's axial tilt? (c) 2 and 3 only
(a) Changing seasons (d) 1, 2 and 3
(b) Variation in Daylight
(c) Length of the year 74. With reference to the Enhanced
(d) Solar Energy Distribution
Collaborative Autonomous Rover System
(ECARS), consider the following
71. Consider the following statements regarding
statements:
the sub-tropical heat belt:
1. A calm condition with feeble winds is 1. It is a multi-terrain manned ground
created in this high-pressure belt. vehicle for a wide range of military
2. Most of the deserts are present along this situations.
belt, in both hemispheres. 2. It is developed by the Defence Research
3. The corresponding latitudes of the and Development Organisation
subtropical high-pressure belt are called (DRDO).
horse latitudes. Which of the statements given above is/are
How many statements given above are
correct?
correct?
(a) 1 only
(a) Only one
(b) 2 only
(b) Only two
(c) Both 1 and 2
(c) All three
(d) None (d) Neither 1 nor 2
13 www.visionias.in ©Vision IAS

FREE BY KING R QUEEN P [ऋषभ राजपूत]


75. Consider the following statements: 78. Consider the following statements with
1. Earth receives the sun’s radiation in the reference to lakes in India:
1. The Wular Lake is formed due to a
form of short waves.
structural depression.
2. Most of the light received by Earth is
2. Kolleru Lake is formed due to a hollow
scattered light. between deltas.
Which of the statements given above is/are 3. Lakes in Nainital in the Kumaon
correct? Himalaya were formed by faulting.
How many of the statements given above are
(a) 1 only
correct?
(b) 2 only (a) Only one
(c) Both 1 and 2 (b) Only two
(d) Neither 1 nor 2 (c) All three
(d) None

76. Consider the following pairs:


79. Consider the following landforms:
Famous Place State 1. Hoodoo
1. Kufri : Uttarakhand 2. Hanging Valley
2. Chakrata : Himachal Pradesh 3. Spits and Bars
How many of the above landforms are
3. Kalimpong : West Bengal
formed due to erosional activity?
How many of the above pairs are correctly (a) Only one
matched? (b) Only two
(a) Only one pair (c) All three
(b) Only two pairs (d) None

(c) All three pairs


80. Consider the following statements regarding
(d) None of the pairs Southern Oscillation in El Nino and
Southern Oscillation (ENSO):
77. Consider the following statements with 1. Its strength is measured in the Southern
Oscillation Index (SOI) which is
reference to North-East Indian states:
computed from the fluctuations in air-
1. Mizoram shares a boundary with the
pressure differences between Tahiti and
Bay of Bengal and Myanmar. Darwin.
2. Tripura shares boundary with 2. El Niño episodes are associated with
Bangladesh, Meghalaya, Assam and negative values of the SOI, meaning that
the pressure at Tahiti is relatively low
Mizoram.
compared to Darwin.
3. Assam does not share a boundary with 3. When the Walker circulation enters its
Sikkim. El Niño phase, the SOI is strongly
Which of the statements given above is/are negative.
How many statements given above are
correct?
correct?
(a) 1 and 2 only
(a) Only one
(b) 2 only (b) Only two
(c) 2 and 3 only (c) All three
(d) 3 only (d) None
14 www.visionias.in ©Vision IAS

FREE BY KING R QUEEN P [ऋषभ राजपूत]


81. The 'State of the Rhino 2023' report was 84. Consider the following statements regarding
released recently. In this context, consider the International Organisation of Legal
the following statements:
Metrology (OIML) certificates:
1. The 'State of the Rhino' report is
1. An OIML pattern approval certificate is
released annualy by the World Wide
Fund for Nature. mandatory to sell a weight or measure in
2. The greater one-horned rhinos, which the international market.
are native to India are categorised as
2. India is a member of OIML and also a
'endangered' on the IUCN Red List.
certificate issuing authority.
Which of the statements given above is/are
correct? Which of the statements given above is/are

(a) 1 only correct?


(b) 2 only (a) 1 only
(c) Both 1 and 2
(b) 2 only
(d) Neither 1 nor 2
(c) Both 1 and 2

82. Bhoramdeo Wildlife Sanctuary (BWS), (d) Neither 1 nor 2


which has been designated as the Tiger
Reserve recently is located at 85. Consider the following statements:
(a) Chattisgarh
Statement-I: The Norwegian coast at
(b) Madhya Pradesh
latitudes 60° to 70°N is ice-free throughout
(c) Rajasthan
(d) Jharkhand the year.

Statement-II: The North Atlantic Drift is a


83. Consider the following statements with
warm current.
reference to forest cover in India:
Which one of the following is correct in
1. India ranks first globally with respect to
the net gain in average annual forest area respect of the above statements?

between 2010 and 2020. (a) Both Statement-I and Statement-II are
2. Mizoram has the largest forest cover by correct and Statement-II is the correct
area among states.
explanation for Statement-I
3. Madhya Pradesh has the largest forest
(b) Both Statement-I and Statement-II are
cover as a percentage of the total
geographical area among states. correct and Statement-II is not the
How many statements given above are correct explanation for Statement-I
correct? (c) Statement-I is correct but Statement-II is
(a) Only one
incorrect
(b) Only two
(c) All three (d) Statement-I is incorrect but Statement-II

(d) None is correct


15 www.visionias.in ©Vision IAS

FREE BY KING R QUEEN P [ऋषभ राजपूत]


86. Consider the following pairs: 89. In the context of geological time scale,
Discontinuity Between arrange the following in decreasing order.
1. Mohorovicic : Upper and Lower 1. Era
Mantle 2. Eon
2. Gutenberg : Crust and Mantle 3. Period
3. Lehmann : Inner and Outer core 4. Epoch
How many of the pairs given above are Select the correct answer using the code
correctly matched?
given below.
(a) Only one
(a) 1-2-3-4
(b) Only two
(b) 2-1-3-4
(c) All three
(c) 3-1-2-4
(d) None
(d) 1-2-4-3

87. In the context of fluvial erosion, river


90. Consider the following pairs:
capture occurs when:
Places often mentioned Country/Region
(a) Both rivers have similar discharge and
sediment loads. in news

(b) One river has a higher discharge and 1. Scarborough shoal : South China Sea
eroding power than the other. 2. Izmail port : Israel
(c) There are no geological faults or 3. Al Haouz province : Yemen
fractures in the area. How many of the above pairs are correctly
(d) The rivers are located in arid desert matched?
regions. (a) Only one
(b) Only two
88. Consider the following statements:
(c) All three
1. Of the five, the Lagrangian Point 1 (or
(d) None
L-1) of the Earth-Sun system is currently
home to the Solar and Heliospheric
91. In the context of the oceanic distribution of
Observatory Satellite (SOHO).
temperature, consider the following
2. Aditya L-1 is the first space based Indian
statements regarding Thermocline:
mission to study the Sun.
1. It is the rapid increase in temperature
3. Aditya-L1 has seven payloads, all
developed indigenously by ISRO and with an increase in depth.

national research laboratories. 2. It is about 5 kilometers in thickness.

How many of the above statements are Which of the statements given above is/are

correct? correct?
(a) Only one (a) 1 only
(b) Only two (b) 2 only
(c) All three (c) Both 1 and 2
(d) None (d) Neither 1 nor 2
16 www.visionias.in ©Vision IAS

FREE BY KING R QUEEN P [ऋषभ राजपूत]


92. Consider the following pairs: 95. In the context of landform formation, uvalas,
Mangrove Region dolines and poljes are:
1. Pichchavaram : Andhra Pradesh (a) landforms of glaciation.
2. Baratang : West Bengal (b) landforms of wind erosion.
3. Bhitakarnika : Odisha (c) landforms of river erosion.
4. Ratnagiri : Maharashtra (d) landforms of karst topography.
How many pairs given above are correctly
matched? 96. An unconformity is a break in time in an
(a) Only one otherwise continuous rock record. In this
(b) Only two context, the Eparchaean Unconformity
(c) Only three which creates a million years gap in India’s
(d) All four geological history is located in:
(a) Trans Himalayas
93. With reference to the seaweed, consider the (b) Western Ghats
following statements: (c) Eastern Ghats
1. More than 95% of the seaweed (d) Vindhyan Range
harvested for human consumption is
cultivated. 97. Consider the following statements:
2. The seaweeds do not require fertilizer, 1. The sun is vertically overhead at the
pesticides, freshwater and land. equator only on two days each year.
3. India's first multi-purpose seaweed park 2. The longest day in the northern and
will be established in Lakshadweep. southern hemispheres falls on the 22nd
How many of the above statements are of December and 21st of June in summer
correct? solstices respectively.
(a) Only one 3. The Arctic and Antarctic circles mark
(b) Only two the limits of the overhead sun.
(c) All three Which of the statements given above is/are
(d) None not correct?
(a) 1 only
94. Consider the following statements: (b) 1 and 3 only
Statement-I: The off-shore vortices on the (c) 2 and 3 only
west coast bring very heavy rainfall to the (d) 1, 2 and 3
west coast during the monsoon season.
Statement-II: The Votices are formed due to 98. Recently Santiniketan has been designated
the inability of monsoon winds to climb over as the 41st UNESCO World Heritage Site
western ghats. (WHS) of India. In this context, consider the
Which one of the following is correct in following statements:
respect of the above statements? 1. Its design has been predominantly
(a) Both Statement-I and Statement-II are inspired from Greek and Roman
correct and Statement - II is the correct architecture.
explanation for Statement-I 2. It is the only world heritage site in
(b) Both Statement-I and Statement-II are Bengal.
correct and Statement - II is not the Which of the statements given above is/are
correct explanation for Statement-I correct?
(c) Statement-I is correct but statement - II (a) 1 only
is incorrect (b) 2 only
(d) Statement -I is incorrect but Statement - (c) Both 1 and 2
II is correct (d) Neither 1 nor 2
17 www.visionias.in ©Vision IAS

FREE BY KING R QUEEN P [ऋषभ राजपूत]


99. Recently Kokborok language was in the
news. Its widely spoken in which of the
following states/UT?
(a) Ladakh
(b) Odisha
(c) Lakshadweep
(d) Tripura

100. The Government of India has come out with


a new set of National Awards in the field of
Science, Technology and Innovation known
as “Rashtriya Vigyan Puraskar’’. In this
context, consider the following pairs:
1. Vigyan Ratna (VR) award: To recognize
distinguished contributions made in any
field of science and technology (S&T).
2. Vigyan Yuva-Shanti Swarup Bhatnagar
(VY-SSB) award: To recognize young
scientists up to the age of 45 years who
made an exceptional contribution in any
field of S&T.
3. Vigyan Shri (VS) award: To recognize
lifetime achievements in any field of
S&T.
How many of the above pairs are correctly
matched?
(a) Only one
(b) Only two
(c) All three
(d) None

Copyright © by Vision IAS


All rights are reserved. No part of this document may be reproduced, stored in a retrieval system or
transmitted in any form or by any means, electronic, mechanical, photocopying, recording or otherwise,
without prior permission of Vision IAS.

18 www.visionias.in ©Vision IAS

FREE BY KING R QUEEN P [ऋषभ राजपूत]


FREE BY KING R QUEEN P [ऋषभ राजपूत]
FREE BY KING R QUEEN P [ऋषभ राजपूत]
FREE BY KING R QUEEN P [ऋषभ राजपूत]
FREE BY KING R QUEEN P [ऋषभ राजपूत]
FREE BY KING R QUEEN P [ऋषभ राजपूत]
FREE BY KING R QUEEN P [ऋषभ राजपूत]
FREE BY KING R QUEEN P [ऋषभ राजपूत]
FREE BY KING R QUEEN P [ऋषभ राजपूत]
FREE BY KING R QUEEN P [ऋषभ राजपूत]
FREE BY KING R QUEEN P [ऋषभ राजपूत]
FREE BY KING R QUEEN P [ऋषभ राजपूत]
FREE BY KING R QUEEN P [ऋषभ राजपूत]
FREE BY KING R QUEEN P [ऋषभ राजपूत]
FREE BY KING R QUEEN P [ऋषभ राजपूत]
FREE BY KING R QUEEN P [ऋषभ राजपूत]
FREE BY KING R QUEEN P [ऋषभ राजपूत]
FREE BY KING R QUEEN P [ऋषभ राजपूत]
FREE BY KING R QUEEN P [ऋषभ राजपूत]
FREE BY KING R QUEEN P [ऋषभ राजपूत]
FREE BY KING R QUEEN P [ऋषभ राजपूत]
FREE BY KING R QUEEN P [ऋषभ राजपूत]
FREE BY KING R QUEEN P [ऋषभ राजपूत]
FREE BY KING R QUEEN P [ऋषभ राजपूत]
FREE BY KING R QUEEN P [ऋषभ राजपूत]
FREE BY KING R QUEEN P [ऋषभ राजपूत]
FREE BY KING R QUEEN P [ऋषभ राजपूत]
FREE BY KING R QUEEN P [ऋषभ राजपूत]
FREE BY KING R QUEEN P [ऋषभ राजपूत]
FREE BY KING R QUEEN P [ऋषभ राजपूत]
FREE BY KING R QUEEN P [ऋषभ राजपूत]
FREE BY KING R QUEEN P [ऋषभ राजपूत]
FREE BY KING R QUEEN P [ऋषभ राजपूत]
FREE BY KING R QUEEN P [ऋषभ राजपूत]
FREE BY KING R QUEEN P [ऋषभ राजपूत]
FREE BY KING R QUEEN P [ऋषभ राजपूत]
FREE BY KING R QUEEN P [ऋषभ राजपूत]
FREE BY KING R QUEEN P [ऋषभ राजपूत]
VISIONIAS
www.visionias.in

Test Booklet Series

TEST BOOKLET

GENERAL STUDIES (P) 2024 – Test – 4144


C
Time Allowed: Two Hours Maximum Marks: 200

INSTRUCTIONS

1. IMMEDIATELY AFTER THE COMMENCEMENT OF THE EXAMINATION, YOU SHOULD CHECK THAT THIS BOOKLET
DOES NOT HAVE ANY UNPRINTED OR TURN OR MISSING PAGES OR ITEMS, ETC. IF SO, GET IT REPLACED BY A
COMPLETE TEST BOOKLET.

2. ENCODE CLEARLY THE TEST BOOKLET SERIES A, B, C OR D AS THE CASE MAY BE IN THE APPROPRIATE PLACE IN
THE ANSWER SHEET.

3. You have to enter your Roll Number on the Test Booklet in the Box
provided alongside. Do NOT write anything else on the Test Booklet.

4. This Test Booklet contains 100 items (Questions). Each item is printed in English. Each item comprises four
responses (answers). You will select the response which you want to mark on the Answer Sheet. In case you
feel that there is more than one correct response with you consider the best. In any case, choose ONLY ONE
response for each item.

5. You have to mark all your responses ONLY on the separate Answer Sheet provided. See direction in the
answers sheet.

6. All items carry equal marks. Attempt all items. Your total marks will depend only on the number of correct
responses marked by you in the answer sheet. For every incorrect response 1/3rdof the allotted marks will be
deducted.

7. Before you proceed to mark in the Answer sheet the response to various items in the Test booklet, you have to
fill in some particulars in the answer sheets as per instruction sent to you with your Admission Certificate.

8. After you have completed filling in all responses on the answer sheet and the examination has concluded, you
should hand over to Invigilator only the answer sheet. You are permitted to take away with you the Test
Booklet.

9. Sheet for rough work are appended in the Test Booklet at the end.

DO NOT OPEN THIS BOOKLET UNTIL YOU ARE ASKED TO DO SO


1 www.visionias.in ©Vision IAS

FREE BY KING R QUEEN P [ऋषभ राजपूत]


1. Consider the following statements with 4. Consider the following statements regarding
respect to the Green Hydrogen Mission of the Water (Prevention and Control of
India: Pollution) Act,1974:
1. It aims to make India a net exporter of 1. Under this act, a cess is levied and
green hydrogen. collected on water consumed by persons
2. It targets to build capabilities to produce operating and carrying on certain types
at least 5 million metric tonnes (MMT) of industrial activities.
2. In 2022, the government amended rules
of green hydrogen per annum by 2030.
under the act to add regulations on the
3. NITI Aayog will be responsible for the
use of water purification systems.
overall coordination and implementation
Which of the statements given above is/are
of the mission.
correct?
How many of the statements given above are
(a) 1 only
correct?
(b) 2 only
(a) Only one
(c) Both 1 and 2
(b) Only two (d) Neither 1 nor 2
(c) All three
(d) None 5. Consider the following:
1. Biodiversity loss
2. With reference to the carbon credit, consider 2. International waters
the following statements: 3. Chemicals and waste
1. One carbon credit is equal to one metric 4. Climate change
ton of carbon dioxide, or in some 5. Renewable energy
markets, carbon dioxide equivalent How many of the above are included in the
gases. Five Focal Areas of the World Environment
2. It is based on the cap-and-trade model. Facility (WEF)?
Which of the statements given above is/are (a) Only two
correct? (b) Only three
(a) 1 only (c) Only four
(d) All five
(b) 2 only
(c) Both 1 and 2
6. Consider the following statements regarding
(d) Neither 1 nor 2
nuclear energy in India:
1. It contributes less than 1% of the total
3. Which of the following are the sources of
electricity generation in the country.
indoor air pollution?
2. India has over 50 nuclear reactors across
1. Phthalate esters the country which produces around
2. Ozone 10,000 MW of nuclear power.
3. Radon 3. Over two-third of the reactors in India
4. Animal dander are Pressurised Heavy Water Reactors.
Select the correct answer using the code How many of the above statements are
given below. correct?
(a) 1, 2 and 3 only (a) Only one
(b) 2 and 4 only (b) Only two
(c) 1, 3 and 4 only (c) All three
(d) 1, 2, 3 and 4 (d) None
2 www.visionias.in ©Vision IAS

FREE BY KING R QUEEN P [ऋषभ राजपूत]


7. Hydrodynamic trapping, solubility trapping 10. Arrange the following gases according to
and mineral carbonation are associated with: their global warming potential in ascending
(a) In situ bioremediation order.
(b) Carbon Sequestration
1. Nitrous Oxide
(c) Decomposition of solid waste
2. Methane
(d) Shale gas extraction
3. Sulphur hexafluoride
4. Carbon dioxide
8. Which of the following are the components
Select the correct answer using the code
of the Global Solar Facility under
International Solar Alliance (ISA)? given below.

1. Solar Payment Guarantee Fund (a) 2-4-3-1


2. Solar Insurance Fund (b) 4-1-2-3
3. Global Energy Alliance for People and (c) 4-2-1-3
Planet (d) 2-1-4-3
4. Scaling Solar Applications for
Agriculture Use
11. Consider the following statements regarding
Select the correct answer using the code
biochar:
given below.
1. It is a carbon-enriched biomaterial
(a) 1 and 2 only
generated through incineration.
(b) 1, 3 and 4 only
(c) 3 and 4 only 2. It enriches the soil and prevents it from

(d) 1, 2, 3 and 4 leaching pesticides.


3. It is also an excellent carbon sink and
9. Consider the following statements regarding can be used for carbon sequestration.
Resilient and Inclusive Supply-chain Which of the statements given above is/are
Enhancement (RISE):
correct?
1. It is an initiative of the Organisation for
(a) 1 only
Economic Co-operation and
(b) 2 and 3 only
Development.
(c) 3 only
2. It will help emerging markets and
(d) 1, 2 and 3
developing countries (EMDCs) increase
the manufacturing of clean-energy
products. 12. Which of the following greenhouse gases
Which of the statements given above is/are have the highest atmospheric lifetime in
correct? years?
(a) 1 only (a) Methane
(b) 2 only
(b) Nitrous Oxide
(c) Both 1 and 2
(c) Hydrofluorocarbons
(d) Neither 1 nor 2
(d) Chloroflorocarbons
3 www.visionias.in ©Vision IAS

FREE BY KING R QUEEN P [ऋषभ राजपूत]


13. Carbon trading as a concept was initially 16. In the context of the environment, ‘Carbon
introduced under which of the following Leakage’ is best described as:
international agreements/summits? (a) It is the expenses incurred by the nation
(a) Kyoto Protocol, 1997 for carbon emission trading.
(b) Montreal Protocol, 1987 (b) It is an increase in emissions outside a
(c) Earth Summit, 1992 region as a direct result of the policy
(d) Paris Agreement, 2015 measures to cap emissions in the region.
(c) It is the excess of carbon emissions by a
14. Consider the following statements with nation that exceeds prescribed limit even
respect to the Inclusive Conservation after acquiring carbon credits from other
Initiative: nations.
1. It is an initiative to support the
(d) It is a tax evasion mechanism used by
leadership of indigenous peoples and
emitters to avoid paying carbon tax for
local communities in stewarding land,
greenhouse gas emissions.
water and natural resources.
2. It will provide direct financial support to
17. Consider the following statements with
indigenous and locally-led initiatives in
respect to 'Jaivik Pahal' campaign:
Africa, Central and South America, Asia
1. It is the campaign launched by the
and the Pacific region.
automobile industry in India.
3. It is a joint initiative of the United
2. It promotes biofuels produced from
Nations and Conservation International.
renewable biological materials.
How many of the statements given above are
Which of the statements given above is/are
correct?
correct?
(a) Only one
(a) 1 only
(b) Only two
(b) 2 only
(c) All three
(d) None (c) Both 1 and 2
(d) Neither 1 nor 2

15. Consider the following pairs:


GI Tag Product State 18. In the context of space science, consider the

1. Udangudi : Karnataka following statements regarding neutrinos:


Panangarupatti 1. They never interact with any substance.
2. Tangsa Textile : Sikkim 2. They are nearly massless.
3. Khamti Rice : Arunachal Pradesh 3. They do not have any charge.
How many of the pairs given above are How many of the statements given above are
correctly matched? correct?
(a) Only one (a) Only one
(b) Only two (b) Only two
(c) All three (c) All three
(d) None (d) None
4 www.visionias.in ©Vision IAS

FREE BY KING R QUEEN P [ऋषभ राजपूत]


19. Consider the following statements regarding 22. With reference to the global climate finance
the Ramsar Convention of 1971: regime, consider the following statements
1. It is a legally binding intergovernmental
regarding the Least Developed Countries
treaty that provides the framework for
Fund (LDCF):
the conservation of wetlands.
1. It was created by the Marrakesh Accords
2. Lotic water bodies are not covered under
the Ramsar Convention. in the 17th conference of parties in 2011.

3. India has the largest network of Ramsar 2. It aims to address the needs of the 51
wetlands in Asia. LDCs which are particularly vulnerable
How many of the above statements are to the adverse impacts of climate
correct?
change.
(a) Only one
3. It supports the preparation and
(b) Only two
implementation of the National
(c) All three
(d) None Adaptation Programmes of Action.

How many of the statements given above are


20. Consider the following statements: correct?
1. E-waste is categorized as hazardous
(a) Only one
waste under the Basel Convention,1989.
(b) Only two
2. India is the world's largest e-waste
(c) All three
producer as per the Global E-waste
Monitor 2020. (d) None

Which of the statements given above is/are


correct? 23. The India GHG Program is:
(a) 1 only (a) an initiative by the Ministry of Science
(b) 2 only
and Technology to develop climate
(c) Both 1 and 2
change mitigation technologies.
(d) Neither 1 nor 2
(b) an initiative by the Ministry of

21. Recently, which of the following villages Environment, Forest and Climate

has been selected as Best Tourism Village Change to reduce greenhouse gas
by the United Nations World Tourism emissions.
Organization (UNWTO)?
(c) an industry-led voluntary framework to
(a) Dhordo
measure and manage greenhouse gas
(b) Mawlynnong
emissions.
(c) Malana
(d) Odanthurai (d) None of the above.

5 www.visionias.in ©Vision IAS

FREE BY KING R QUEEN P [ऋषभ राजपूत]


24. In the context of Nanophotonic Electron 26. With reference to 'Zero Budget Natural
Accelerator (NEA), recently seen in the Farming', a term often seen in news, which
of the following statements are correct?
news, consider the following statements:
1. It is a method of farming done without
1. It is the world's smallest particle
using any credit, and without spending
accelerator which has been turned on for any money on purchased inputs.
the first time. 2. It aims to eliminate the use of chemical
2. One of the key goals of developing NEA pesticides.
3. It is not practiced in India.
is to exploit the energy released by the
Select the correct answer using the code
use of miniature particle accelerators for
given below.
medicinal purposes. (a) 1 and 2 only
3. The NEA uses light beams focused at (b) 2 and 3 only
the vacuum tube's pillars to magnify (c) 1 and 3 only

energy, resulting in a very low energy (d) 1, 2 and 3

field.
27. The Worldwide Governance Indicators
How many of the statements given above are (WGI) is often seen in the news. It is an
correct? initiative of
(a) Only one (a) Inter-Parliamentary Union
(b) Only two (b) Organisation for Economic Co-operation
and Development
(c) All three
(c) World Bank
(d) None
(d) United Nations

25. Consider the following statements with 28. Consider the following statements regarding
respect to E-waste Rules (Management), the United Nations Environment Assembly
(UNEA):
2022:
1. It was founded as a result of the UN
1. These rules do not apply to waste
Conference on the Human Environment
batteries and radioactive wastes. (Stockholm Conference) in 1972.
2. It defines an ‘e-retailer’ as a company 2. It is the world’s highest-level decision-
which deals in e-waste online. making body with near-universal
membership for matters related to the
3. It defines 'orphaned products’ as those
environment.
produced more than five years ago by a
3. The session of assembly is conducted
branded company. every year at the UNEP headquarters in
How many of the statements given above are Nairobi, Kenya.
correct? How many of the statements given above are
correct?
(a) Only one
(a) Only one
(b) Only two
(b) Only two
(c) All three (c) All three
(d) None (d) None
6 www.visionias.in ©Vision IAS

FREE BY KING R QUEEN P [ऋषभ राजपूत]


29. Consider the following sources of energy: 32. With reference to the 'Ethanol Blending
1. Wind energy programme (EBP) in India', consider the
2. Nuclear energy following statements:
3. Solar energy 1. India has advanced the target of 20
4. Biomass energy percent ethanol blending in petrol from
5. Geothermal energy
2030 to 2025-26.
How many of the above are included in the
2. Department of Food and Public
renewable energy sector in India?
Distribution (DFPD) is the nodal
(a) Only two
(b) Only three department for the promotion of fuel-
(c) Only four grade ethanol-producing distilleries in
(d) All five the country.
Which of the statements given above is/are
30. Consider the following statements with correct?
respect to the Carbon Offsetting and (a) 1 only
Reduction Scheme for International Aviation (b) 2 only
(CORSIA): (c) Both 1 and 2
1. It is the first global market-based
(d) Neither 1 nor 2
measure for any sector that offers a
harmonized way to reduce emissions
33. Consider the following:
from international aviation.
2. Unlike the pilot phase, the participation 1. Increase in Research Octane Number
of states is mandatory in its final phase. 2. Higher flame speed
Which of the statements given above is/are 3. Lesser fuel requirement for the same
correct? engine output
(a) 1 only How many of the above are the benefits of
(b) 2 only ethanol blending?
(c) Both 1 and 2 (a) Only one
(d) Neither 1 nor 2 (b) Only two
(c) All three
31. Consider the following statement with
(d) None
respect to ozone-depleting substances:
1. Montreal Protocol and Kigali
Amendment, both address the issues 34. Which of the following are a part of the
related to ozone-depleting substances. biological treatment of wastewater
2. The Kigali amendment aims to reduce containing hazardous chemicals?
HFCs by 80-85 per cent by the late 1. Neutralisation
2040s. 2. Composting
3. India has successfully met the phase-out 3. Conditioning
targets of all the Ozone Depleting 4. Digestion
Substances as per the Montreal Protocol 5. Lagooning
Schedule.
Select the correct answer using the code
How many of the statements given above are
given below.
correct?
(a) 1 and 5 only
(a) Only one
(b) 2, 3 and 5 only
(b) Only two
(c) All three (c) 2, 3 and 4 only
(d) None (d) 1, 2, 3, 4 and 5
7 www.visionias.in ©Vision IAS

FREE BY KING R QUEEN P [ऋषभ राजपूत]


35. Consider the following National Missions 38. Consider the following statements regarding
being implemented in India: the Convention on International Trade in
1. National Mission on Natural Farming Endangered Species of Wild Fauna and
(NMNF) Flora (CITES):
2. National Mission on Strategic 1. It is a non-binding international
Knowledge for Climate Change agreement between governments to
(NMSKCC) regulate international trade in wild
3. National Mission For Enhanced Energy animals and plants.
Efficiency 2. Its secretariat is administered by the
United Nations Environment Programme
4. National Mission on Sustainable Habitat
(UNEP).
How many of the above are not included in
Which of the statements given above is/are
India's National Action Plan On Climate
correct?
Change (NAPCC)?
(a) 1 only
(a) Only one
(b) 2 only
(b) Only two
(c) Both 1 and 2
(c) Only three
(d) Neither 1 nor 2
(d) None
39. Consider the following statements regarding
36. Consider the following statements regarding the International Whaling Commission
the Global Framework for Climate Services (IWC):
(GFCS): 1. It adopted a global moratorium on
1. It is an initiative of the United Nations commercial whaling due to the
Framework Convention on Climate overexploitation of whale stocks in
Change. 1986.
2. It was established in the year 2004 after 2. Aboriginal subsistence whaling is not
the Indian Ocean Tsunami. regulated by the IWC.
Which of the statements given above is/are Which of the statements given above is/are
correct? correct?
(a) 1 only (a) 1 only
(b) 2 only (b) 2 only
(c) Both 1 and 2 (c) Both 1 and 2
(d) Neither 1 nor 2 (d) Neither 1 nor 2

40. Which of the following countries is the part


37. Consider the following statements regarding
of recently proposed IPMDA Initiative?
Gambusia Fish:
1. Japan
1. It migrates from the ocean to lay eggs in
2. Rusia
freshwater.
3. China
2. It is critically endangered.
4. U.S.A.
3. They are known for their symbiotic
5. India
relationship with the Olive Ridley
6. UK
Turtle. 7. South Korea
How many of the statements given above are Select the correct answer using the code
correct? given below.
(a) Only one (a) 2, 3 and 7 only
(b) Only two (b) 1, 2 and 3 only
(c) All three (c) 1, 4 and 5 only
(d) None (d) 1, 4, 5 and 6 only
8 www.visionias.in ©Vision IAS

FREE BY KING R QUEEN P [ऋषभ राजपूत]


41. Consider the following statements regarding 44. Recently, the Prime Minister launched the
Global Footprint Network: PM-PVTG scheme on the occasion of
1. It was established by the World Summit Janjatiya Gaurav Divas. In this context,
on Sustainable Development in 2002. consider the following statements:
2. It is responsible for hosting and 1. It aims to bring critical infrastructure to
calculating Earth Overshoot Day. the 75 PVTG communities.
Which of the statements given above is/are 2. To ensure coordination it aims to
correct? appoint one nodal officer for each state.
(a) 1 only 3. The Ministry of Rural Development is
(b) 2 only the nodal ministry for overall policy
(c) Both 1 and 2 planning and coordination of programs.
(d) Neither 1 nor 2 How many of the statements given above are
correct?
42. Which of the following statements is correct (a) Only one
regarding National River Conservation Plan (b) Only two
(NRCP) ? (c) All three
(a) It is under the Ministry of Water (d) None
Resources, River Development & Ganga
Rejuvenation. 45. National Air Quality Monitoring Programme
(b) National Lake Conservation Programme (NAMP) determines the status and trends of
is a sub-scheme under the National ambient air quality with respect to which of
River Conservation Programme. the following?
(c) All the states are covered under the 1. Carbon Dioxide
National River Conservation Plan. 2. Sulphur Dioxide
(d) It includes river front development 3. PM10
works and construction of low cost 4. Wind speed and direction
sanitation toilets. 5. Temperature
6. Oxides of Nitrogen
43. Consider the following statements regarding Select the correct answer using the code
the “Loss and Damage Fund”: given below.
1. It aims to provide financial assistance to (a) 2, 3, 4, 5 and 6 only
nations most vulnerable and impacted by (b) 1, 3 and 6 only
the effects of climate change. (c) 1, 2, 3, 4, 5 and 6
2. It was opertionalised at COP-21, UN (d) 1, 2 and 6 only
Climate Change Conference in Paris,
France. 46. The report ‘Global Landscape of Climate
Which of the statements given above is/are Finance 2023’ is released by:
correct? (a) World Bank
(a) 1 only (b) Intergovernmental Panel on Climate
(b) 2 only Change
(c) Both 1 and 2 (c) World Economic Forum
(d) Neither 1 nor 2 (d) Climate Policy Initiative
9 www.visionias.in ©Vision IAS

FREE BY KING R QUEEN P [ऋषभ राजपूत]


47. Consider the following statements regarding 50. Arrange the following in the descending
Project Kusha: order of renewable energy potential of India:
1. It is aimed at developing nuclear
1. Solar energy
submarines for India.
2. It is being jointly developed with the 2. Wind energy
help of Israel. 3. Bioenergy
Which of the statements given above is/are 4. Small Hydro
correct?
Select the correct answer using the code
(a) 1 only
given below.
(b) 2 only
(c) Both 1 and 2 (a) 1-2-4-3
(d) Neither 1 nor 2 (b) 1-2-3-4
(c) 4-1-3-2
48. With reference to the Indian coal, consider
(d) 2-4-1-3
the following statements:
1. India has the fifth largest coal reserves
and is second in coal production after 51. Consider the following statements regarding
China. solar energy sector in India:
2. Gondwana coal makes up more than 95
1. The current installed solar power
% of the total reserves and 99% of the
capacity in the country is more than
production of coal in India.
3. It has high ash and sulphur content but 100GW.
low ash fusion temperature thus making 2. Ground-mounted solar plants has the
India one of the largest importer. largest share followed by roof-top and
How many of the above statements are
hybrid projects respectively.
correct?
(a) Only one Which of the statements given above is/are
(b) Only two correct?
(c) All three (a) 1 only
(d) None
(b) 2 only

49. Recently, the World Health Organisation (c) Both 1 and 2


released the Global TB Report 2023. In this (d) Neither 1 nor 2
context, consider the following statements:
1. Staphylococcus aureus is the primary 52. World Heritage Outlook Report is released
causative agent of tuberculosis.
by:
2. Tuberculosis is an infectious disease that
affects only the lungs. (a) United Nations Educational, Scientific
3. The Mantoux test is used to monitor and Cultural Organization (UNESCO)
blood sugar in Tuberculosis patients. (b) Economic and Social Council
How many of the above statements are
(ECOSOC) of UN
correct?
(a) Only one (c) International Union for the Conservation
(b) Only two of Nature (IUCN)
(c) All three (d) United Nations Environment Programme
(d) None
(UNEP)
10 www.visionias.in ©Vision IAS

FREE BY KING R QUEEN P [ऋषभ राजपूत]


53. Consider the following statements regarding 55. Consider the following statements regarding

the Energy Saving Certificates (ESCerts): the International Tropical Timber

1. These are market-based regulatory Organization:


1. It is a multilateral non-governmental
instruments to reduce energy
organization for promoting the
consumption in energy-intensive
sustainable management and
industries.
conservation of tropical forests.
2. ESCerts trading is regulated by the
2. Countries from tropical areas with
Indian Renewable Energy Development timber trade are only eligible to become
Agency members of ITTO.

3. These certificates are issued under the 3. India is one of its producing members.

Perform, Trade and Achieve (PAT) How many of the above statements are
correct?
scheme.
(a) Only one
Which of the statements given above is/are
(b) Only two
correct?
(c) All three
(a) 1 and 2 only
(d) None
(b) 2 only

(c) 1 and 3 only 56. Consider the following statements, with


(d) 1, 2 and 3 respect to Graded Response Action Plan:
1. It is a set of measures implemented in
Delhi-NCR throughout the year for
54. On which of the following principles do air
controlling air pollution.
pollution control devices work?
2. The Commission for Air Quality
1. Gravity separation
Management (CAQM), is an
2. Cyclonic separation
autonomous body tasked with improving
3. Filtration the air quality in Delhi.
4. Wet Scrubbing 3. It calls for a complete ban on eateries,
5. Electrostatic Precipitation restaurants and hotels only when the Air

Select the correct answer using the code Quality Index reaches 450 level.
How many of the statements given above are
given below.
correct?
(a) 2, 3 and 4 only
(a) Only one
(b) 1 and 5 only
(b) Only two
(c) 1, 2, 3, 4 and 5
(c) All three
(d) 2, 3, 4 and 5 only (d) None
11 www.visionias.in ©Vision IAS

FREE BY KING R QUEEN P [ऋषभ राजपूत]


57. Consider the following statements regarding 60. Consider the following pairs:
the ‘Champions of the Earth Award’ : Generation of Source
1. It is the United Nations’ highest Bio-Fuels

environmental honor. 1. First : Food sources like sugar,


2. It recognizes the contributions of Generation starch or animal fats
individuals as well as organizations. 2. Second : Non-food crops like
3. Recently, this award was given to Generation stems, husks and wood
laureates who are implementing chips
3. Third : From micro-organisms
solutions and policies to eliminate
Generation like algae
plastic pollution.
4. Fourth : Genetically engineered
How many statements given above are
Generation crops
correct? How many pairs given above are correctly
(a) Only one matched?
(b) Only two (a) Only one
(c) All three (b) Only two
(d) None (c) Only three
(d) All four

58. Consider the following statements regarding


61. The biennial State of Worlds Forests Report
the Annapurna Certificate Programme: is published by:
1. It acknowledges restaurants abroad (a) World Environment Facility (WEF)
contributing to India’s cultural cause (b) United Nations Forum on Forests
through cuisines. (UNFF)
(c) World Forest Watch (WFW)
2. It is an initiative of the Indian Council
(d) The Food and Agriculture Organization
for Cultural Relations.
(FAO)
Which of the statements given above is/are
correct? 62. Consider the following statements with
(a) 1 only respect to Bharat Stage VI norms:
(b) 2 only 1. In April 2020 India transitioned from BS

(c) Both 1 and 2 V emission norms to BS-VI norms.


2. Emissions from petrol cars have to
(d) Neither 1 nor 2
decrease more compared to Diesel cars
to reach the norms prescribed in BS-VI.
59. BHARAT TAP initiative and Nirmal Jal 3. BS-VI-compliant fuel has zero levels of
Prayas initiative are under: sulphur content.
(a) Ministry of Jal Shakti. Which of the statements given above is/are
(b) Ministry of Housing and Urban Affairs. not correct?
(a) 3 only
(c) Ministry of Environment, Forest and
(b) 1 and 2 only
Climate Change.
(c) 2 and 3 only
(d) Ministry of Panchayati Raj (d) 1, 2 and 3
12 www.visionias.in ©Vision IAS

FREE BY KING R QUEEN P [ऋषभ राजपूत]


63. Consider the following statements with 66. The Fagradalsfjall system, recently seen in
respect to the short-lived climate pollutants: the news, is associated with?
1. They remain in the atmosphere for a
(a) Irrigation System
much shorter period of time than
methane but their potential to warm the (b) Volcanic system

atmosphere can be many times greater. (c) System to rehabilitate war victim
2. They are responsible for up to 45% of (d) System to attract honey bees
current global warming.
3. Climate & Clean Air Coalition is the
67. Consider the following statements regarding
only international body working to
address this issue. the National Clean Air Program (NCAP):
How many of the statements given above are 1. The NCAP aims to reduce particulate
correct?
matter (PM) concentrations by 20-30%
(a) Only one
in cities by 2030.
(b) Only two
(c) All three 2. It is being implemented by the Ministry
(d) None of Environment, Forest and Climate
Change (MoEFCC).
64. With reference to the disposal of waste, the
Which of the statements given above is/are
term 'Incineration' refers to:
(a) the process in which sedimented water is correct?
subjected to a chemical process to (a) 1 only
extract potable water.
(b) 2 only
(b) a physico-chemical process for
(c) Both 1 and 2
separating suspended and colloidal
impurities from liquids. (d) Neither 1 nor 2
(c) the process in which disinfectants like
chlorine or bleaching powder are added 68. In the context of the economy, consider the
to remove impurities in water.
following statements regarding Project
(d) the process of burning waste at very high
temperatures. Mariana:
1. It is an initiative of the World Bank.
65. Consider the following statements regarding 2. It tests cross-border trading and
White Phosphorous:
settlement of wholesale central bank
1. It ignites in contact with air.
2. It is banned by the Convention on digital currencies.

Certain Conventional Weapons (CCW). Which of the statements given above is/are
Which of the statements given above is/are correct?
correct?
(a) 1 only
(a) 1 only
(b) 2 only
(b) 2 only
(c) Both 1 and 2 (c) Both 1 and 2
(d) Neither 1 nor 2 (d) Neither 1 nor 2
13 www.visionias.in ©Vision IAS

FREE BY KING R QUEEN P [ऋषभ राजपूत]


69. Consider the following statements regarding 72. Consider the following statements regarding
'SATAT' scheme:
election funding in India:
1. The scheme envisages to target
production of 15 MMT (million tons) of 1. An individual can donate as much as he
compressed biogass by 2023. wants to a political party.
2. It aims to encourage entrepreneurs to set
2. A company can donate up to 10% of the
up compressed biogass plants, produce
and supply CBG to Oil Marketing average of the preceding three years'
Companies.
profits.
3. The scheme will help in better use of
municipal solid waste along with 3. There is no legal expenditure limit on
agricultural residue. expenditure by political parties.
How many of the statements given above are
How many of the statements given above are
correct?
(a) Only one correct?
(b) Only two (a) Only one
(c) All three
(d) None (b) Only two

(c) All three


70. Invasive species Specialist Group' (that
(d) None
develops Global Invasive Species Database)
belongs to which one of the following
organizations? 73. With reference to India’s updated
(a) The International Union for
Nationally Determined Contribution (NDC),
Conservation of Nature
(b) The United Nations Environment consider the following statements:
Programme
1. It aims to reduce emissions intensity of
(c) The United Nations World Commission
for Environment and Development. its GDP by 45 percent by 2030, from
(d) The World Wide Fund for Nature 2000 as the base year.

2. It aims to achieve 50 percent of its


71. Consider the following statements regarding
the National Organ and Tissue Transplant energy capacity from renewable based
Organisation (NOTTO):
energy resources by 2030.
1. It operates under the Ministry of Science
and Technology. 3. It targets net zero emissions by 2070.
2. It oversees organ procurement but How many of the above statements are
distribution is left to the states.
correct?
Which of the statements given above is/are
correct? (a) Only one
(a) 1 only
(b) Only· two
(b) 2 only
(c) Both 1 and 2 (c) All three
(d) Neither 1 nor 2 (d) None
14 www.visionias.in ©Vision IAS

FREE BY KING R QUEEN P [ऋषभ राजपूत]


74. Consider the following statements regarding 77. In the context of Indian culture, the term
Just Energy Transition Partnerships (JETPs): Vajra Mushti Kalaga was recently seen in
1. JETPs aim to bridge the gap between the news. It is a
developed and developing nations in (a) Buddhist sect
moving towards clean energy. (b) form of wrestling
2. The first JETP was announced at the (c) folk dance
Sharm el-Sheikh Climate Change
(d) form of puppetry
Conference (COP 27).
3. South Africa, Bangladesh, and
78. Which of the following statements correctly
Venezuela are the first three countries to
describes the "Non-Attainment Cities' under
receive funding under a JETP.
the National Clean Air Programme (NCAP)?
Which of the statements given above is/are
(a) Cities that do not participate in attaining
correct?
National Ambient Air Quality Standards
(a) 1 only
(b) 1 and 2 only (NAAQS) for a time bound period.

(c) 2 and 3 only (b) Cities identified by the Central Pollution


(d) 3 only Control Board (CPCB) who have not
attained 40% to 50% reduction in
75. Consider the following statements regarding particulate matter.
the Convention on Conservation of (c) Cities that do not meet the National
Migratory Species (CMS): Ambient Air Quality Standards
1. CMS is the only global (NAAQS) over a 5-year period for PM
intergovernmental organisation 10 or N02.
established exclusively for the (d) None of the above.
conservation and management of
migratory species.
79. Which of the following activities could best
2. It is an environmental treaty under the
prevent smog?
aegis of the United Nations Environment
1. Fuel up cars during the cooler hours of
Programme.
the day
Which of the statements given above is/are
2. Use of low-volatile organic compound
correct?
paints
(a) 1 only
3. Use of electric appliances instead of gas
(b) 2 only
powered lawnmowers
(c) Both 1 and 2
4. Use of public transportation
(d) Neither 1 nor 2
Select the correct answer using the code
given below.
76. In the context of security, 'Prachand' is a/an
(a) light combat helicopter. (a) 1 only

(b) inter-continental ballistic missile. (b) 1 and 3 only


(c) 2, 3 and 4 only
(c) nuclear submarine.
(d) 1, 2 , 3 and 4
(d) anti-tank guided missile.
15 www.visionias.in ©Vision IAS

FREE BY KING R QUEEN P [ऋषभ राजपूत]


80. Recently, millions of people have been 83. Consider the following statements with
internally displaced due to ongoing conflict respect to Corporate Average Fuel
with the rebel group Mouvement du 23 Mars Efficiency/Economy (CAFE) regulations:
(M23). To which country does this M23
1. They are primarily designed for oil
rebel group belong?
companies to increase fuel efficiency.
(a) Democratic Republic of Congo
(b) Palestine 2. CAFE is the weighted average of

(c) Sudan emissions and fuel economy for the


(d) Syria entire fleet sold in the country.
3. They were first notified in 2017 by the
81. Consider the following pairs: Union Ministry of Petroleum and
Bio- Used for
Natural Gas.
Medical
4. The existing limit for average CO2
Waste
emissions happens to be 100g/km.
Bag
1. Red : Discarded or Expired How many of the statements given above are
medicine correct?
2. Yellow : Soiled Waste (a) Only one
3. Blue : Glassware (b) Only two
4. White : Waste sharps including
(c) Only three
metal
(d) All four
How many pairs given above are correctly
matched?
(a) Only one 84. Consider the following statements with
(b) Only two respect to the Central Pollution Control
(c) Only three Board:
(d) All four 1. It can prohibit the use of any fuel or
appliance that may cause air pollution.
82. Consider the following statements regarding
2. It can inspect any control equipment,
the Atmospheric Waves Experiment :
industrial plant or manufacturing process
1. It is aimed at studying the interactions
between terrestrial and space weather. for the prevention of air pollution.

2. It is an initiative of the World 3. It may establish or recognize a


Meteorological Organization. laboratory for performing its functions
3. It will be launched and mounted on the under the Air (Prevention and Control of
exterior of the International Space Pollution) Act,1981.
Station (ISS).
How many of the statements given above are
How many of the statements given above are
correct?
correct?
(a) Only one
(a) Only one
(b) Only two (b) Only two
(c) All three (c) All three
(d) None (d) None
16 www.visionias.in ©Vision IAS

FREE BY KING R QUEEN P [ऋषभ राजपूत]


85. Consider the following statements regarding 88. Which of the following are the by-products
Global Methane Pledge (GMP): of coal based power plants in India?
1. Oxides of aluminium and calcium
1. Its aim is to collectively reduce methane
2. Arsenic
emissions by at least 30% below 2020 3. Boron
levels by 2030. 4. Chromium
5. Lead
2. It was launched at COP-26 of United
Select the correct answer using the code
Nations Climate Change Conference given below.
(UNFCCC) in Glasgow, Scotland. (a) 1, 3 and 5 only
3. India did not sign it because of its (b) 1, 4 and 5 only
(c) 2, 3, 4 and 5 only
concerns over the trade, farm and (d) 1, 2, 3, 4 and 5
livestock sector in the economy.
How many of the above statements are 89. Consider the following pairs:
Alternative fuel Produced from
correct?
1. Renewable : Cellulose material
(a) Only one Gasoline
(b) Only two 2. Renewable Diesel : Fats and oil
(c) All three 3. Sustainable : Woody biomass
(d) None Aviation
Fuel
How many pairs given above are correctly
86. Consider the following: matched?
1. Extension of existing Airports (a) Only one
(b) Only two
2. Ports and Harbour dealing in Fish
(c) All three
3. Road Highway Toll Plazas (d) None
How many of the above do not require
90. Consider the following statements with
public consultation under Environment
respect to the Rotterdam Convention:
Impact Assessment rules? 1. It is a multilateral treaty to promote
(a) Only one shared responsibilities in relation to the
(b) Only two importation of hazardous chemicals.
2. The movements of hazardous chemicals
(c) All three
listed in Annex III are subject to the
(d) None Prior Informed Consent procedure.
3. Exports are only allowed if the State of
import has consented to the future
87. Which of the following institutions publishes
import of the specific chemical.
the Global Energy Transition Index? 4. It also covers chemicals imported for the
(a) International Energy Agency purpose of research and analysis.
(b) World Economic Forum How many of the statements given above are
correct?
(c) Cornell University, USA
(a) Only one
(d) The Energy and Resources Institute, (b) Only two
India (c) Only three
(d) All four
17 www.visionias.in ©Vision IAS

FREE BY KING R QUEEN P [ऋषभ राजपूत]


91. 'Climate Neutral Now Initiative', an initiative 94. In the context of the electoral process in
of UNFCCC is: India, consider the following statements
regarding the term "ENCORE" :
(a) a certification scheme for non-parties to
1. It is software for complete candidate and
the convention. election management for returning
(b) an initiative to act now in order to officers.
2. It was developed by the National
achieve a climate-neutral world by 2030.
Technical Research Organisation.
(c) a certification scheme based on the
Which of the statements given above is/are
completion of nationally determined correct?
contributions. (a) 1 only
(b) 2 only
(d) an initiative to promote the voluntary
(c) Both 1 and 2
use of carbon market mechanisms.
(d) Neither 1 nor 2

92. Consider the following statements with 95. Consider the following statements regarding
ocean thermal energy in India:
respect to Chemical Oxygen Demand:
1. Ocean thermal energy is produced by
1. COD determines the amount of oxygen harnessing the temperature difference
required for the chemical oxidation of an between cold current and warm current.
organic matter. 2. India’s first ocean thermal energy plant
is being implemented in Kavaratti in
2. It helps differentiate between
Lakshadweep.
biologically oxidizable and biologically 3. The National Institute of Ocean
inert material. Technology (NIOT) is the implementing
agency for the plant.
3. It helps determine the efficiency of
How many statements given above are
treatment plants and pollution loads.
correct?
How many of the statements given above are (a) Only one
correct? (b) Only two
(c) All three
(a) Only one
(d) None
(b) Only two
(c) All three 96. Consider the following countries:
(d) None 1. Russia
2. Malaysia
3. Cambodia
93. The Palermo Convention recently seen in the 4. Indonesia
news deals with How many of the above countries are
(a) transnational organized crime. members of Global Tiger Forum?
(a) Only one
(b) sea level rise.
(b) Only two
(c) land degradation (c) All four
(d) global Financial Architecture. (d) None
18 www.visionias.in ©Vision IAS

FREE BY KING R QUEEN P [ऋषभ राजपूत]


97. Consider the following statements regarding 99. Consider the following pairs:
India's Carbon Credit Trading Scheme, Convention Related Subject
2023: 1. Basel : Transboundary
1. Under the scheme, MoEFCC will issue Convention Movements of
carbon credit certificates to the entities Hazardous Wastes and
that surpass the assigned targets. their Disposal.
2. The Central Electricity Regulatory 2. Hong Kong : Safe and
Commission (CERC) will regulate the Convention Environmentally Sound
trading of carbon credits under the Recycling of Ships.
scheme. 3. Stockhom : Persistent Organic
Which of the statements given above is/are Convention Pollutants
correct? How many pairs given above are correctly
(a) 1 only matched?
(b) 2 only (a) Only one
(c) Both 1 and 2 (b) Only two
(d) Neither 1 nor 2 (c) All three
(d) None
98. Cocoyoc declaration is associated with
(a) sustainable development 100. Climate Smart Agriculture aims to:
(b) ozone layer depletion 1. Adapting and building resilience of
(c) wildlife conservation crops and livestock to climate change
(d) nuclear non-proliferation 2. Sustainably increasing agricultural
productivity
3. Reducing greenhouse gas emissions
Which of the statements given above is/are
correct?
(a) 1 only
(b) 1 and 3 only
(c) 2 and 3 only
(d) 1, 2 and 3

Copyright © by Vision IAS


All rights are reserved. No part of this document may be reproduced, stored in a retrieval system or
transmitted in any form or by any means, electronic, mechanical, photocopying, recording or otherwise,
without prior permission of Vision IAS.
19 www.visionias.in ©Vision IAS

FREE BY KING R QUEEN P [ऋषभ राजपूत]


VISIONIAS
www.visionias.in

Test Booklet Series

TEST BOOKLET

GENERAL STUDIES (P) 2024 – Test – 4145


C
Time Allowed: Two Hours Maximum Marks: 200

INSTRUCTIONS

1. IMMEDIATELY AFTER THE COMMENCEMENT OF THE EXAMINATION, YOU SHOULD CHECK THAT THIS BOOKLET
DOES NOT HAVE ANY UNPRINTED OR TURN OR MISSING PAGES OR ITEMS, ETC. IF SO, GET IT REPLACED BY A
COMPLETE TEST BOOKLET.

2. ENCODE CLEARLY THE TEST BOOKLET SERIES A, B, C OR D AS THE CASE MAY BE IN THE APPROPRIATE PLACE IN
THE ANSWER SHEET.

3. You have to enter your Roll Number on the Test Booklet in the Box
provided alongside. Do NOT write anything else on the Test Booklet.

4. This Test Booklet contains 100 items (Questions). Each item is printed in English. Each item comprises four
responses (answers). You will select the response which you want to mark on the Answer Sheet. In case you
feel that there is more than one correct response with you consider the best. In any case, choose ONLY ONE
response for each item.

5. You have to mark all your responses ONLY on the separate Answer Sheet provided. See direction in the
answers sheet.

6. All items carry equal marks. Attempt all items. Your total marks will depend only on the number of correct
responses marked by you in the answer sheet. For every incorrect response 1/3rdof the allotted marks will be
deducted.

7. Before you proceed to mark in the Answer sheet the response to various items in the Test booklet, you have to
fill in some particulars in the answer sheets as per instruction sent to you with your Admission Certificate.

8. After you have completed filling in all responses on the answer sheet and the examination has concluded, you
should hand over to Invigilator only the answer sheet. You are permitted to take away with you the Test
Booklet.

9. Sheet for rough work are appended in the Test Booklet at the end.

DO NOT OPEN THIS BOOKLET UNTIL YOU ARE ASKED TO DO SO


1 www.visionias.in ©Vision IAS

FREE BY KING R QUEEN P [ऋषभ राजपूत]


1. Consider the following statements regarding 3. Recently, India was re-elected to the Council

the Madden-Julian Oscillation: of the International Maritime Organization


(IMO). In this context, consider the
1. It is an equatorial westward moving
following statements:
disturbance of clouds, rainfall, winds,
1. International Maritime Organization is a
and pressure.
specialized agency of the United
2. It is observed mainly over the Atlantic Nations.

Ocean. 2. India was elected among ten states with

Which of the statements given above is/are the largest interest in providing

international shipping services.


correct?
3. India has the distinction of being a
(a) 1 only
member of the IMO council
(b) 2 only
continuously since 1959.
(c) Both 1 and 2 How many statements given above are

(d) Neither 1 nor 2 correct?

(a) Only one

(b) Only two


2. Which of the following statements best
(c) All three
describes the process of vernalization in
(d) None
ecology?

(a) It is the induction of the flowering 4. Consider the following statements regarding

process of the plant by exposure to the productivity in ecology:


1. Oceans contribute a higher share in
long periods of cold winter or such
annual net primary productivity of the
conditions.
whole biosphere than land.
(b) It is the process by which plants die to
2. Secondary productivity is defined as the
growing in low-light conditions. rate at which consumers convert the
(c) It is the mechanism by which plants organic matter into new biomass.

regulate their water intake during dry Which of the statements given above is/are

correct?
periods.
(a) 1 only
(d) It is a process of asexual reproduction in
(b) 2 only
which plants reproduce through
(c) Both 1 and 2
structures other than flowers and seeds. (d) Neither 1 nor 2
2 www.visionias.in ©Vision IAS

FREE BY KING R QUEEN P [ऋषभ राजपूत]


5. Consider the following statements regarding 8. Consider the following statements regarding
the Thorium mineral used for production of the Ethics Committees of the Parliament
nuclear energy: which was in the news recently:
1. It is extracted from monazite ore. 1. Its origins lie in the report of the
2. It is mainly extracted from Narmada- National Commission to Review the
Tapti river basin. Working of the Constitution.
3. Chevara in Kerala is famous for
2. Any person may directly complain
Thorium deposits.
against a member of parliament to the
Which of the statements given above are
ethics committee.
correct?
Which of the statements given above is/are
(a) 1 and 2 only
correct?
(b) 2 and 3 only
(a) 1 only
(c) 1 and 3 only
(b) 2 only
(d) 1, 2 and 3
(c) Both 1 and 2

6. Consider the following statements regarding (d) Neither 1 nor 2

the Global Partnership on Artificial


Intelligence (GPAI): 9. Consider the following statements regarding
1. It is a multistakeholder initiative to Anthrax disease:
guide the responsible development and 1. Anthrax is caused by gram-positive
use of artificial intelligence bacteria.
2. Its secretariat is hosted by the 2. Humans can catch anthrax when they
Organisation for Economic Co-operation come in contact with infected animals.
and Development. 3. Currently, there is no treatment available
3. The first GPAI summit in 2023 was for Anthrax.
hosted by India. How many statements given above are
How many statements given above are
correct?
correct?
(a) Only one
(a) Only one
(b) Only two
(b) Only two
(c) All three
(c) All three
(d) None
(d) None

10. Which island in the Andaman and Nicobar


7. Which of the following biogeochemical
cycles lacks a significant component in the group is known for its limestone caves and

atmosphere? mud volcanoes?

(a) Carbon Cycle (a) Little Andaman

(b) Phosphorus Cycle (b) Baratang


(c) Nitrogen Cycle (c) Barren Island
(d) Sulphur Cycle (d) Great Nicobar
3 www.visionias.in ©Vision IAS

FREE BY KING R QUEEN P [ऋषभ राजपूत]


11. In geological terms, the concept of Isostasy 14. Arrange the following from North to South:
refers to 1. Gulf of Bothnia
(a) The study of seismic activity and
2. Gulf of California
earthquake patterns.
3. Gulf of Carpentaria
(b) The equilibrium in the Earth's crust,
where lighter masses float on denser Select the correct answer using the code
materials. given below.
(c) The process of erosion and
(a) 1-2-3
sedimentation in river valleys.
(b) 1-3-2
(d) The formation and movement of tectonic
plates beneath the Earth's surface. (c) 2-1-3

(d) 3-2-1
12. Consider the following rivers :
1. Hemavati
15. Which of the following rivers does not
2. Kabani
3. Indravati drain into the Arabian Sea?

How many of the rivers given above are the (a) Zuari
tributaries of the Cauvery River? (b) Sharavati
(a) Only one
(c) Vaigai
(b) Only two
(c) All three (d) Bharathappuzha

(d) None

16. Consider the following statements regarding


13. Radiative forcing is a measure of the
alternative fuels:
influence a factor has in altering the balance
1. Diesohol is an alternative fuel which is a
of incoming and outgoing energy in the
Earth's atmosphere. In this context, consider blend between diesel and ethanol.
the following gases: 2. A comprehensive study of ethanol
1. Carbon dioxide
blended fuel E27 was recently launched
2. Methane
by Hindustan Petroleum Corporation
3. Tropospheric Ozone
4. Sulfate aerosols Limited (HPCL).

How many of the above given gases are Which of the statements given above is/are
considered to be positive radiative forcing in
correct?
climate change?
(a) 1 only
(a) Only one
(b) Only two (b) 2 only

(c) Only three (c) Both 1 and 2


(d) All four (d) Neither 1 nor 2
4 www.visionias.in ©Vision IAS

FREE BY KING R QUEEN P [ऋषभ राजपूत]


17. Per- and polyfluoroalkyl substances (PFAS) 19. With reference to rare earth metals, consider
are also known as forever chemicals. In this the following statements:
1. Rare earths are 15 elements referred to
context, consider the following statements:
in the lanthanide and Actinide series of
1. They are a large, naturally occurring
the periodic table of elements.
chemical family which can be produced 2. India does not have industrial-scale
synthetically. intermediate facilities for the use of rare
2. They are used to make fluoropolymer earth metals in alloys.
3. The Indian resource only contains Light
coatings and products that resist heat,
Rare Earth Elements (LREE) in
oil, stains, grease, and water
extractable quantities.
3. They can be found in human blood too How many of the statements given above are
because of their wide use. correct?
Which of the statements given above is/are (a) Only one

correct? (b) Only two


(c) All three
(a) 1 and 2 only
(d) None
(b) 2 only
(c) 2 and 3 only 20. In the context of the origin and evolution of
(d) 3 only the earth, “The Big Splat” event is associated
with?
(a) The collision between Earth and a
18. The Coalition for Disaster Resilient
Mars-sized body, leading to the
Infrastructure released the first-ever biennial
formation of the Moon.
report to lay down the political and (b) The sudden eruption of large volcanic
economic ground for the need to invest in masses shaping the Earth's surface.
disaster-resilient infrastructure. In this (c) The impact of a comet, causing mass
extinction events.
context, consider the following statements
(d) The gravitational interaction between
regarding the Coalition for Disaster Resilient
Earth and a passing asteroid.
Infrastructure (CDRI):
1. It is a United Nations organization 21. Consider the following statements:
aiming to promote the resilience of new 1. Uttar Pradesh does not have a border
with Chhattisgarh.
and existing infrastructure.
2. Only three Indian states have a border
2. It was launched during the UN Climate
with Pakistan.
Action Summit in 2019. 3. The Tropic of Cancer does not pass
Which of the statements given above is/are through Manipur.
correct? How many of the statements given above are
correct?
(a) 1 only
(a) Only one
(b) 2 only
(b) Only two
(c) Both 1 and 2 (c) All three
(d) Neither 1 nor 2 (d) None
5 www.visionias.in ©Vision IAS

FREE BY KING R QUEEN P [ऋषभ राजपूत]


22. With reference to Lithium resources, 25. With reference to the population and its
consider the following statements: density, consider the following statements:
1. The lithium triangle includes Argentina, 1. Physiological density refers to the ratio
Chile, and Bolivia.
of the total population of an area and the
2. Recently, the Geological Survey of India
net cultivated area.
discovered lithium reserves in the Reasi
district of Jammu and Kashmir. 2. The Agricultural population of an area
3. ONGC Videsh, is responsible for includes the agricultural labourers as
securing lithium blocks overseas. well as their family members.
How many of the statements given above Which of the statements given above is/are
is/are correct?
correct?
(a) Only one
(a) 1 only
(b) Only two
(c) All three (b) 2 only
(d) None (c) Both 1 and 2
(d) Neither 1 nor 2
23. Consider the following statements with
respect to the World Migration Report:
26. Consider the following statements regarding
1. Since 2000, the International
Global Biofuel Alliance (GIBA):
Organization for Migration has been
publishing it every year. 1. It is an India-led initiative to create a
2. World Migration Report 2022 is only favourable ecosystem for promoting
available online. development and deployment of
Which of the following statements is/are biofuels.
correct? 2. It was formally launched on the side-
(a) 1 Only
lines of the G20 Summit in New Delhi.
(b) 2 Only
Which of the statements given above is/are
(c) Both 1 and 2
(d) Neither 1 nor 2 correct?
(a) 1 only
24. Consider the following conventions/ (b) 2 only
protocols: (c) Both 1 and 2
1. United Nations Convention on
(d) Neither 1 nor 2
Biological Diversity
2. Stockholm Convention on Persistent
Organic Pollutants 27. Consider the following lakes in India:
3. United Nations Convention to Combat 1. Pachpadra Lake
Desertification 2. Tso Kar
4. Montreal Protocol 3. Kaliveli Lake
How many of the mentioned above have How many of the above lakes are freshwater
Global Environment Facility as their formal
lakes?
financial mechanisms?
(a) Only one
(a) Only one
(b) Only two (b) Only two
(c) Only three (c) All three
(d) All four (d) None
6 www.visionias.in ©Vision IAS

FREE BY KING R QUEEN P [ऋषभ राजपूत]


28. Consider the following statements regarding 32. The term ‘JT-60SA’, which was recently
Nonylphenol, sometimes seen in the news: seen in the news, is related to:
1. It is a toxic bio-accumulate that is (a) The Japanese space mission to the sun
present in detergents. (b) Genetically modified millet variety
2. It is an endocrine disruptor in humans developed in India
and can create reproductive defects in
(c) Biofuel produced using jute crop
rodents.
(d) Experimental nuclear fusion reactor
3. United Nations Environment Programme
has designated it as a chemical of global
concern. 33. ‘Aboriginal and Torres Strait Islander
4. The Bureau of Indian Standards Voice‘, recently seen in the news, is related
regulates the amount of Nonyphenols in to:
drinking water. (a) a movement in the Nordic countries for
How many statements given above are economic demands
correct? (b) an organization for the protection of
(a) Only one human rights in the states of the east
(b) Only two coast of the USA
(c) Only three
(c) a referendum conducted in Australia to
(d) All four
recognize the first people of Australia
under its constituion
29. Phytoplanktons are primarily found in which
of the following zones of a lake? (d) a movement in Caribbean islands by
(a) Littoral zone aboriginals for democratic rights.
(b) Benthic zone
(c) Limnetic zone 34. Which of the following organizations
(d) Profundal zone recently launched the INFUSE mission to
study a supernova event remnant that took
30. Which of the following is used to describe place 20,000 years ago?
the chemical interactions between plants that (a) European Space Agency
can have either beneficial or harmful effects?
(b) Japan Aerospace Exploration Agency
(a) Mutualism
(c) Indian Space Research Organization
(b) Commensalism
(d) National Aeronautics and Space
(c) Allelopathy
(d) Symbiosis Administration

31. Which of the following could be termed 35. Which of the following best describes the
secondary activities? concept of "Ecotype"?
1. Retail sales (a) A group of organisms that share a
2. Textile similar role in an ecosystem, regardless
3. Shipbuilding of their taxonomic affiliation.
4. Tourism (b) A genetically distinct population of a
5. Energy utilities
species that is adapted to a specific
Select the correct answer using the code
environment.
given below.
(c) A taxonomic classification of organisms
(a) 1 and 4 only
based on their physical characteristics.
(b) 2, 3 and 5 only
(c) 1, 2 and 4 only (d) A measure of the diversity of species
(d) 1, 2, 3, 4 and 5 within a particular ecosystem.
7 www.visionias.in ©Vision IAS

FREE BY KING R QUEEN P [ऋषभ राजपूत]


36. Consider the following statements regarding 38. Consider the following pairs:
Harmful Algal Blooms (HABs): Glaciers Location
1. Algal blooms are toxic in nature because 1. Thwaites Glacier : Antarctica
presence of algae is unhealthy for
aquatic ecosystems.
2. The Steenstrup : Greenland
2. HABs occur only in the freshwater
Glacier
ecosystems.
3. Schlatenkees : Austria
3. They kill fishes in the ecosystem by
causing oxygen depletion. How many pairs given above are correctly

How many of the above statements are matched?


correct? (a) Only one
(a) Only one (b) Only two
(b) Only two
(c) All three
(c) All three
(d) None
(d) None

39. Consider the following statements regarding


37. Consider the following statements regarding
underwater noise emissions: the scheme for “Development of Solar Parks

1. Marine sound pollution is caused by and Ultra Mega Power Projects":

shipping, oil and gas exploration and 1. Under the scheme, the Government of
military sonar. India has currently sanctioned 50 solar
2. Masking is a process where the external
parks across all the states.
noise overlaps with marine species
2. Solar Energy Corporation of India
communication sound which hampers
(SECI) acts as a nodal agency for the
navigational skills of these species.
scheme.
3. India currently has no mechanism to
measure underwater noise emissions. 3. Recently, the targets for the scheme

4. The National policy for Blue Economy were achieved well before the expected

aims to tackle underwater noise deadline of 2026.


pollution. How many statements given above are
How many of the statements given above are
correct?
correct?
(a) Only one
(a) Only one
(b) Only two
(b) Only two
(c) All three
(c) Only three
(d) All four (d) None
8 www.visionias.in ©Vision IAS

FREE BY KING R QUEEN P [ऋषभ राजपूत]


40. Consider the following statements with 43. With reference to the Pacific decadal
respect to Noise Pollution (Regulation and oscillation (PDO), consider the following
Control) Rules, 2000: statements:
1. These rules are prescribed under the 1. PDO is a naturally occurring short - term
purview of the Air (Prevention and phenomenon with each phase lasting
Control of Pollution) Act, 1981. around 6 to 12 months.
2. Noise emanating from industry is 2. During the positive phase, the eastern
North Pacific tends to be warmer than
regulated by the Central Pollution
average, while during the negative
Control Board.
phase, it tends to be cooler than average.
3. Loudspeakers can be used throughout
3. Positive phase of PDO tends to be
the night in closed premises like
associated with more rapid global
community halls and banquet halls.
warming while the negative phase is
How many of the above statements are linked to slow global warming.
correct? 4. During the positive phase of the PDO in
(a) Only one wintertime much of Asia is usually
(b) Only two cooler than normal, with above normal
(c) All three temperatures over India.
(d) None How many of the statements given above are
correct?
41. Which of the following best describes the (a) Only one
concept of "Trophic cascade"? (b) Only two
(a) A sudden and dramatic increase in the (c) Only three
population of a particular species (d) All four
(b) A series of indirect ecological effects
that result from the removal or addition 44. Article 99, a rarely used provision of the
United Nations Charter was invoked by the
of a predator or prey species
UN Secretary General in the context of:
(c) A type of biological control that uses
(a) hostilities in Gaza and Israel.
predators to control pest populations
(b) Russia-Ukraine conflict.
(d) A measure of the efficiency of energy
(c) increasing hold of ISIS in employing
transfer through a food web
youth in terrorism.
(d) China's growing influence in Pakistan.
42. Which of the following bodies have been
formed under Wildlife (Protection) Act, 45. Which of the following best describes
1972? 'Torrefaction technology' ?
1. National Board for Wildlife (a) It refers to conversion of biomass into
2. Central Zoo Authority green hydrogen.
3. National Tiger Conservation Authority (b) It refers to gasification of coal to
4. Wildlife Crime Control Bureau produce a synthetic gas made from
Select the correct answer using the code carbon monoxide and hydrogen.
given below. (c) It refers to conversion of biomass into
(a) 1, 2 and 3 only coal-like pallets.
(b) 1 and 4 only (d) It refers to thermal depolymerization of
(c) 2, 3 and 4 only organic matter in the presence of
nitrogen or absence of oxygen.
(d) 1, 2, 3 and 4
9 www.visionias.in ©Vision IAS

FREE BY KING R QUEEN P [ऋषभ राजपूत]


46. Consider the following: 49. Consider the following statements regarding
1. Dominance of phtoplankton and algae Renewable Purchase Obligations (RPOs)
used in India:
2. Damage to the coral reefs
1. RPOs are mandatory under the
3. Sea bottom hypoxia
Environmental (Protection) Act, 1986.
How many of the above are the impacts of 2. Both captive power producers and
eutrophication? discoms may be considered as obligated
(a) Only one entities.

(b) Only two Which of the statements given above is/are


correct?
(c) All three
(a) 1 only
(d) None
(b) 2 only
(c) Both 1 and 2
47. Consider the following cities: (d) Neither 1 nor 2
1. Lucknow
50. Chief Economists Outlook report is
2. Bhopal
published by
3. Bengaluru
(a) World Economic Forum (WEF)
How many of the above cities are located to
(b) The International Monetary Fund (IMF)
the west of the Standard Meridian of India? (c) The Organization for Economic
(a) Only one Cooperation and Development (OECD)
(b) Only two (d) World Bank

(c) All three


51. Consider the following statements:
(d) None
Statement-1: Alternate stripes of positive
and negative magnetic anomalies are found
48. Consider the following environmental act: on either side of the mid-oceanic ridges.
1. Wild life (Protection) Act, 1972 Statement 2: Earth experiences a temporal

2. The Public Liability Insurance Act, 1991 reversal in the geocentric dipole magnetic
field.
3. The Water (Prevention and Control of
Which one of the following is correct in
Pollution) Cess Act, 1977
respect of the above statements?
4. The Water (Prevention and Control of (a) Both Statements-1 and Statement-2 are
Pollution) Act, 1974 correct and Statement-2 is the correct
How many of the above statutes mentioned explanation for Statement-1
(b) Both Statement-1 and Statement-2 are
in the Schedule I of the National Green
correct and Statement-2 is not the
Tribunal Act, 2010 ?
correct explanation for Statement-1
(a) Only one (c) Statement-1 is correct but Statement-2 is
(b) Only two incorrect
(c) Only three (d) Statement-2 is incorrect but Statement-1
(d) All four is correct
10 www.visionias.in ©Vision IAS

FREE BY KING R QUEEN P [ऋषभ राजपूत]


52. Consider the following factors: 55. Consider the following statements regarding
1. The introduction of a new species to the
Soil Respiration:
ecosystem
2. Changes in environmental conditions, 1. Soil respiration is a measure of the
such as temperature or precipitation
carbon dioxide (CO2) released from soil
3. The removal of a key species from the
ecosystem as a result of decomposition of soil
How many of the above can significantly
impact the stability of a food chain? organic matter (SOM) and plant litter by
(a) Only one soil microbes.
(b) Only two
(c) All three 2. The rate of soil respiration increases
(d) None with increase in ambient temperature

53. Consider the following statements with and decrease with decrease in ambient
respect to the United Nations Population
temperature.
Fund:
1. Its goal is to end child marriage and Which of the statements given above is/are
female genital mutilation by 2030.
correct?
2. Its funding is supported by the United
Nations regular budget. (a) 1 only
3. The UN Economic and Social Council
(b) 2 only
(ECOSOC) establishes its mandate.
How many of the statements given above are (c) Both 1 and 2
correct?
(d) Neither 1 nor 2
(a) Only one
(b) Only two
(c) All three
56. Consider the following pairs:
(d) None
Tribes Region/Country
54. Consider the following statements regarding
the SDG Agrifood Accelerator Programme: 1. Tuareg : North Africa
1. It is an initiative by the World Food 2. Ayoreo : Paraguay
Program and is supported by the SEED
initiative. 3. Kawahiva : Brazil
2. It supports small enterprises from 4. Sentinelese : India
developing countries pioneering
solutions to transform agrifood systems How many pairs given above are correctly
through enhancing environmental
matched?
protections.
Which of the statements given above is/are (a) Only one
correct?
(b) Only two
(a) 1 only
(b) 2 only (c) Only three
(c) Both 1 and 2
(d) All four
(d) Neither 1 nor 2
11 www.visionias.in ©Vision IAS

FREE BY KING R QUEEN P [ऋषभ राजपूत]


57. India has been unanimously elected as a 60. Consider the following factors of migration:
member representing the Asian region in the 1. Lack of employment opportunities
executive committee of the Codex 2. Frequent droughts
Alimentarius Commission (CAC), In this
3. Better access to healthcare
context, onsider the following statements
4. Small size of land holdings
regarding the CAC:
1. It is an international food standard and Which of the above will fall under the
code of practice contributing to safe category of 'push factors' of migration from
international food trade. rural to urban areas?
2. It was created by the Food and (a) 1 and 2 only
Agriculture Organization and the World
(b) 1, 2 and 4 only
Trade Organization.
(c) 3 and 4 only
Which of the statements given above is/are
correct? (d) 1, 2, 3 and 4
(a) 1 only
(b) 2 only 61. Which of the following can be considered as
(c) Both 1 and 2 a factor of an Allogenic succession?
(d) Neither 1 nor 2
(a) The increase in soil fertility due to the
decomposition of organic matter
58. Consider the following statements regarding
tropical dry evergreen forests of India: (b) The competition between different plant
1. They occur predominantly along the species for sunlight.
Gujarat coast. (c) The exponential changes in species
2. The chief characteristics of these forests composition due to competitive
are short-statured trees.
interactions
3. Bamboo is grown extensively in these
(d) The introduction of a new species that
forests.
How many of the above statements are prey on native species.
correct?
(a) Only one 62. Consider the following statements:
(b) Only two 1. Volcanic eruptions can cause the ozone
(c) All three
hole to expand.
(d) None
2. Ozone Depleting Substances (ODS) are

59. Consider the following statements regarding human-made chemicals containing


GWP100: chlorine and bromine.
1. It is an internationally accepted emission 3. Stratospheric aerosol injection is
accounting standard used by countries. scientifically proven to reduce ozone
2. It is suited to measure the impact of
depletion.
short-lived greenhouse gases.
How many statements given above are
Which of the statements given above is/are
correct? correct?
(a) 1 only (a) Only one
(b) 2 only (b) Only two
(c) Both 1 and 2 (c) All three
(d) Neither 1 nor 2
(d) None
12 www.visionias.in ©Vision IAS

FREE BY KING R QUEEN P [ऋषभ राजपूत]


63. Which of the following groups of animals 67. Consider the following:
represents a "Guild in a community"? River Island Associated River
(a) Lions, zebras, and gazelles 1. Bhavani Island : Godavari
(b) Bees, Butterflies, and American 2. Umananda : Brahmaputra
Humming Bird Island
(c) Desert Sparrow, Butterfly, and Ants 3. Nongkhnum : Shyok
(d) Polar Bears, Penguins, and Seals How many of the above are correctly
matched?
64. Consider the following: (a) Only one
1. Inefficient grazing strategies (b) Only two
2. Low turnover rate of phytoplankton (c) All three
3. Long food chains (d) None
How many of the above are reasons for the
Inverted Pyramid Of Biomass in Oceans? 68. Consider the following railway sites:
(a) Only one 1. Darjeeling Himalayan Railway
(b) Only two 2. Nilgiri Mountain Railway
3. Kalka Shimla Railway
(c) All three
4. Chhatrapati Shivaji Terminus, Mumbai
(d) None
5. Matheran Light Railway
6. Kangra Valley Railway
65. Consider the following pairs:
How many of the above sites are UNESCO-
Resources Region
accorded World Heritage Sites?
1. Oil : Sahara Desert (a) Only three
2. Diamonds : Atacama Desert (b) Only four
3. Caliche : Kalahari Desert (c) Only five
(d) All six
Which of the pairs given above is/are
correctly matched?
69. Consider the following statements:
(a) 1 only
Statement-I: Biological oxygen demand
(b) 2 and 3 only
(BOD), is the amount of dissolved oxygen
(c) 1, 2 and 3
needed by aerobic biological organisms to
(d) 3 only
break down organic material present in a
given water body.
66. Consider the following pairs:
Statement-II: The waterbody with low
Volcanic belt Cause for eruption
Biological oxygen demand (BOD) is
1. Circum : Collision of Indian considered highly polluted.
Pacific Belt and Pacific Plate Which one of the following is correct in
2. Mid : Convergence of respect of the above statements?
continental belt Eurasian and (a) Both Statement-I and Statement-II are
African, Indian plate correct and Statement-II is the correct
3. Mid Atlantic : Splitting of Atlantic explanation for Statement-I.
belt plate (b) Both Statement-I and Statement-II are
How many of the above pairs are correctly correct and Statement-II is not the
matched? correct explanation for Statement-I.
(a) Only one (c) Statement-I is correct but Statement-II is
(b) Only two incorrect.
(c) All three (d) Statement-I is incorrect but Statement-II
(d) None is correct.
13 www.visionias.in ©Vision IAS

FREE BY KING R QUEEN P [ऋषभ राजपूत]


70. Consider the following system with 72. Consider the following statements with
reference to the chemical composition of the reference to 'Amaterasu particles' recently
earth: seen in news :
1. The dominant minerals in the 1. They are high energy particles with
composition of the crust are feldspar and energy exceeding 240 EeV.
mica.
2. They are produced by the Large Hadron
2. The upper part of the crust is composed
Collider, which is the most powerful
of heavy silicate matter whereas light
accelerator ever built.
silicate dominates the lower part.
Which of the statements given above is/are
3. The core of the earth is composed of
correct?
silica and magnesium.
(a) 1 only
How many of the above statements are
(b) 2 only
correct?
(c) Both 1 and 2
(a) Only one
(b) Only two (d) Neither 1 nor 2

(c) All three


(d) None 73. Which of the following statements best

describes the roles of Anabolism and


71. Consider the following statements: Catabolism in ecological processes?
1. Quinary activities focus on the creation, (a) Anabolism is the synthesis of complex
re-arrangement and interpretation of new organic compounds from simpler forms,
and existing ideas while Quaternary while catabolism is the breakdown of
activities focus on the collection,
complex organic compounds into
production and dissemination of
simpler forms.
information.
(b) Anabolism refers to the process of
2. The services provided under the Quinary
breakdown, whereas catabolism refers to
sector can not be completely outsourced
the process of growth.
whereas services involved in the
(c) Anabolism involves the conversion of
Quaternary type of economy are
solar energy into chemical energy, while
outsourced in varied forms.
catabolism is the release of stored
Which of the following statements is/are
correct? energy through photosynthesis.

(a) 1 only (d) Anabolism is responsible for energy

(b) 2 only storage in ecosystems, while catabolism

(c) Both 1 and 2 is the release of energy during the


(d) Neither 1 nor 2 production of organic matter.
14 www.visionias.in ©Vision IAS

FREE BY KING R QUEEN P [ऋषभ राजपूत]


74. Which of the following best describes 77. Consider the following statements regarding
"Operation Chakra-2" recently seen in news? the concept of Planetary boundaries:
(a) It aimed to track down cyber criminals 1. It aims to define environmental limits
who indulge in financial crimes and within which humanity can safely
operate.
fraud.
2. The nine planetary boundaries are
(b) It was launched to disrupt, degrade, and
identified and defined by the
dismantle networks, with international
Intergovernmental Panel on Climate
linkages, involved in drug trafficking. Change in 2007.
(c) It is related to fight against the sharing 3. Novel entities such as organic polluters
and dissemination of Child Sexual and microplastics form part of planetary
Abuse Material (CSAM) online. boundaries.
(d) It aimed to reusing and recycling 4. Recently, it was established that all the
materials to minimize the consumption planetary boundaries have been
breached for the first time.
of natural resources and prevent waste
How many statements given above are
generation.
correct?
(a) Only one
75. Consider the following : (b) Only two
1. Competition (c) Only three
2. Predation (d) All four
3. Mutualism
How many of the above can be classified as 78. Which of the following statements is not
Density-Independent Limiting Factors in correct regarding the PM-Vishwakarma
scheme?
Terrestrial Ecosystems?
(a) It is launched by Ministry of Micro,
(a) Only one
Small and Medium Enterprises.
(b) Only two
(b) It recognizes artisans and craftsmen
(c) All three through PM Vishwakarma Certificate.
(d) None (c) Only one member per family can avail
benefits of the scheme.
76. Consider the following services provided by (d) The minimum age of the beneficiary
Ecosystem: should be 14 years.
1. Recreation and Tourism
79. Consider the following statements:
2. Production of Food
1. India is the third largest Electronic waste
3. Carbon Sequestration
producer in the world.
4. Maintenance of Genetic Diversity
2. India processed more than 50% of e-
How many of the above can be classified waste it generated in FY 2021-22.
under "Supporting Services" provided by Which of the statements given above is/are
Ecosystem? correct?
(a) Only one (a) 1 only
(b) Only two (b) 2 only
(c) Only three (c) Both 1 and 2
(d) Neither 1 nor 2
(d) All four
15 www.visionias.in ©Vision IAS

FREE BY KING R QUEEN P [ऋषभ राजपूत]


80. Consider the following landforms: 82. Consider the following statements regarding

1. Stack particulate matter and its impact on health

2. Chasm and the environment:

3. Notch 1. Particulate matter can cause both

4. Chimney atmosphere cooling and warming.


2. Particulate matter can build anti-
5. Shingle
microbial resistance.
6. Spit
3. Plant growth inhibition and yield loss
7. Tombolo
are possible due to metal particulate
How many of the above-mentioned are sea
matter.
waves and coastal landforms?
4. Maternal exposure to particulate air
(a) Only four
pollution during pregnancy may lead to
(b) Only five
abnormally higher newborn birthweight.
(c) Only six
How many statements given above are
(d) All seven correct?
(a) Only one
81. Consider the following statements: (b) Only two
Statement 1: The content of water vapour (c) Only three
increases from the equator to poleward and (d) All four

the content of vapour increases upward.

Statement II: The content of water vapour 83. Consider the following statements regarding

depends on the temperature. the International Classification of Diseases:

Which one of the following is correct in 1. It originated in the post-covid global


efforts to tackle pandemics.
respect of the above statements?
2. It provides critical knowledge on the
(a) Both Statement-1 and Statement-II are
extent, causes and consequences of
correct and Statement-II is the correct
human disease and death worldwide.
explanation for Statement-I
3. It is maintained by the World Health
(b) Both Statement-I and Statement-II are
Organization.
correct and Statement-II is not the
How many statements given above are
correct explanation for Statement-I
correct?
(c) Statement-I is correct but Statement-II is
(a) Only one
incorrect (b) Only two
(d) Statement-I is incorrect but Statement-II (c) All three
is correct (d) None
16 www.visionias.in ©Vision IAS

FREE BY KING R QUEEN P [ऋषभ राजपूत]


84. Consider the following statements regarding 87. Consider the following statements regarding
sand and dust storms create due to climate Equatorial Evergreen Forest:
change: 1. They have buttress roots to support the
1. According to UNCCD, human activities heavy and tall trees.
cause more than 75% of the sand and
2. Vegetation is characterized by distinct
dust storms across the globe.
vertical stratification.
2. In central Asia, shrinking of Aral sea
3. The soil is very rich in nutrients and
and creation of Aralkum desert is one of
humus.
the major sources of sand and dust
storms. How many of the above statements are

Which of the above statements is/are correct?


correct? (a) Only one
(a) 1 only (b) Only two
(b) 2 only (c) All three
(c) Both 1 and 2 (d) None
(d) Neither 1 nor 2
88. Consider the following processes
85. Recently, the State government of Gujarat
1. Sedimentation
banned plantation of this tree. It is a
2. Decomposition
flowering plant belonging to the family of
3. Weathering of rocks
Combretaceae. It is an invasive mangrove
How many of the above processes add
species. It has been used by various public
authorities in India as landscaping for road carbon dioxide to carbon cycle?
medians, along roads, and in public gardens. (a) Only one
This tree is famous for its dark green leaves (b) Only two
color throughout the year and withstands (c) All three
harsh environmental conditions such as high (d) None
and low temperatures.
Which of the following is best described in 89. ISRO launched the ADITYA L-1 Mission to
the passage given above? study the Sun's structure. In this context,
(a) Prosopis juliflora
arrange the various layers of the Sun from
(b) Acacia mangium
inner to outer layers:
(c) Lantana camara
1. Convective Zone
(d) Conocarpus erectus
2. Radiative Zone

86. Mission ‘Operation Storm Makers II’ 3. Photosphere


recently seen in the news is related to: 4. Corona
(a) Israel’s military mission in the Gaza 5. Chromosphere
Strip. Select the correct answer using the code
(b) Humanitarian assistance mission by given below.
India. (a) 3-1-5-4-2
(c) INTERPOL led operation targeting (b) 4-5-3-1-2
human trafficking. (c) 2-1-3-5-4
(d) Military exercise between India-UAE.
(d) 2-3-1-5-4
17 www.visionias.in ©Vision IAS

FREE BY KING R QUEEN P [ऋषभ राजपूत]


90. With reference to the submarine canyons, 93. Consider the following statements regarding
consider the following statements: the Perform Achieve Trade (PAT) scheme:

1. The course of subaerial canyons is 1. It is a mechanism for improvements in


energy efficiency of energy-intensive
usually sinuous, while that of submarine
industries.
canyons is straight.
2. Under the scheme, the central
2. The gradient of continental canyons is government notifies industries as
normally steeper than submarine 'designated consumers' which have to
canyons. follow the energy restrictions.
3. The glacial canyons are more in number 3. If a certain industry does not achieve the

than the non-glacial submarine canyons. energy targets, it has to buy energy-
saving certificates.
How many of the statements given above are
How many of the statements given above are
correct?
correct?
(a) Only one (a) Only one
(b) Only two (b) Only two
(c) All three (c) All three
(d) None (d) None

94. Consider the following statements regarding


91. Consider the following conventions:
the Global Cooling Pledge:
1. Basel Convention
1. It is the initiative of the United Arab
2. Bamako Convention Emirates as the host of CoP 28 of the
3. Waigani Convention UNFCCC.
How many of the conventions mentioned 2. It commits countries to reduce cooling-
above are related to waste management? related emissions by at least 68% by

(a) Only one 2050 compared to 2022 levels.


3. India has signed this pledge.
(b) Only two
How many statements given above are
(c) All three
correct?
(d) None (a) Only one
(b) Only two
92. 'Biotransformation', a new technology, (c) All three
recently seen in news, is related to: (d) None

(a) Production of biodegradable variety of


95. The ‘State of Finance for Nature 2023’
plastic
report was released by:
(b) Cost-effective production of biofuels
(a) Global Environment Facility
(c) Bioremediation technolgy used in metal (b) International Union for Conservation of
waste reduction Nature
(d) Technology to use stem cells in helping (c) United Nations Environment Program
disabled people (d) Conservation International
18 www.visionias.in ©Vision IAS

FREE BY KING R QUEEN P [ऋषभ राजपूत]


96. Consider the following statements with 98. In which Himalayan region does the
reference to the Article 6A of the Citizenship "Zabarwan Range" play a significant role in
Act (1955), recently seen in news: influencing the local climate?
(a) Himachal Pradesh
1. It allows people who entered India
(b) Uttarakhand
between January 1, 1966, and March 25,
(c) Jammu and Kashmir
1971, and have been living in Assam to
(d) Sikkim
register themselves as citizens.
2. The section was inserted in the 99. Which among the following factors
Citizenship Act in 2019. responsible for growing cotton textile mills
3. Assam is the only state to have an away from cotton producing areas?
exclusive cut-off date for citizenship 1. Easy availability of cheap labour
against a common one for other states. 2. Local investment
3. Development of hydroelectricity
Which of the statements given above is/are
Select the correct answer using the code
correct?
given below.
(a) 1 and 2 only
(a) 3 only
(b) 2 only (b) 1 and 3 only
(c) 1 and 3 only (c) 1, 2 and 3
(d) 3 only (d) 2 only

97. Consider the following passage: 100. Consider the following passage :
These soils contain a larger proportion of This plateau roughly forms a triangle based
on the Vindhyan Hills, bounded by the
sodium, potassium, and magnesium, and
Aravali Range in the west and a sharply
thus do not support much vegetative growth.
defined scarp overlooking Bundelkhand in
They occur in arid and semi-arid regions and
the east. It inherits a complex geology;
waterlogged and swampy areas. They lack scarcely any one of the peninsular groups is
nitrogen and calcium. These soils are unrepresented here. This plateau has two
widespread in western Gujarat, deltas of the systems of drainage; one towards the
eastern coast, and in Sunderban areas of Arabian Sea and the other towards the Bay
West Bengal. They are also known as Usara of Bengal. It is composed of extensive lava
flow and is covered with black soil.
soils.
Which of the following plateaus is being
Which of the following soil types is being
described in the passage given above?
described in the passage given above?
(a) Malwa
(a) Peaty Soils (b) Hazaribagh
(b) Saline soils (c) Baghelkahnd
(c) Red and Yellow Soil (d) Chotanagpur
(d) Laterite soils

Copyright © by Vision IAS


All rights are reserved. No part of this document may be reproduced, stored in a retrieval system or
transmitted in any form or by any means, electronic, mechanical, photocopying, recording or otherwise,
without prior permission of Vision IAS.
19 www.visionias.in ©Vision IAS

FREE BY KING R QUEEN P [ऋषभ राजपूत]


FREE BY KING R QUEEN P [ऋषभ राजपूत]
FREE BY KING R QUEEN P [ऋषभ राजपूत]
FREE BY KING R QUEEN P [ऋषभ राजपूत]
FREE BY KING R QUEEN P [ऋषभ राजपूत]
FREE BY KING R QUEEN P [ऋषभ राजपूत]
FREE BY KING R QUEEN P [ऋषभ राजपूत]
FREE BY KING R QUEEN P [ऋषभ राजपूत]
FREE BY KING R QUEEN P [ऋषभ राजपूत]
FREE BY KING R QUEEN P [ऋषभ राजपूत]
FREE BY KING R QUEEN P [ऋषभ राजपूत]
FREE BY KING R QUEEN P [ऋषभ राजपूत]
FREE BY KING R QUEEN P [ऋषभ राजपूत]
FREE BY KING R QUEEN P [ऋषभ राजपूत]
FREE BY KING R QUEEN P [ऋषभ राजपूत]
FREE BY KING R QUEEN P [ऋषभ राजपूत]
FREE BY KING R QUEEN P [ऋषभ राजपूत]
FREE BY KING R QUEEN P [ऋषभ राजपूत]
FREE BY KING R QUEEN P [ऋषभ राजपूत]
FREE BY KING R QUEEN P [ऋषभ राजपूत]
VISIONIAS
www.visionias.in

Test Booklet Series

TEST BOOKLET

GENERAL STUDIES (P) 2024 – Test – 4147


C
Time Allowed: Two Hours Maximum Marks: 200

INSTRUCTIONS

1. IMMEDIATELY AFTER THE COMMENCEMENT OF THE EXAMINATION, YOU SHOULD CHECK THAT THIS BOOKLET
DOES NOT HAVE ANY UNPRINTED OR TURN OR MISSING PAGES OR ITEMS, ETC. IF SO, GET IT REPLACED BY A
COMPLETE TEST BOOKLET.

2. ENCODE CLEARLY THE TEST BOOKLET SERIES A, B, C OR D AS THE CASE MAY BE IN THE APPROPRIATE PLACE IN
THE ANSWER SHEET.

3. You have to enter your Roll Number on the Test Booklet in the Box
provided alongside. Do NOT write anything else on the Test Booklet.

4. This Test Booklet contains 100 items (Questions). Each item is printed in English. Each item comprises four
responses (answers). You will select the response which you want to mark on the Answer Sheet. In case you
feel that there is more than one correct response with you consider the best. In any case, choose ONLY ONE
response for each item.

5. You have to mark all your responses ONLY on the separate Answer Sheet provided. See direction in the
answers sheet.

6. All items carry equal marks. Attempt all items. Your total marks will depend only on the number of correct
responses marked by you in the answer sheet. For every incorrect response 1/3rdof the allotted marks will be
deducted.

7. Before you proceed to mark in the Answer sheet the response to various items in the Test booklet, you have to
fill in some particulars in the answer sheets as per instruction sent to you with your Admission Certificate.

8. After you have completed filling in all responses on the answer sheet and the examination has concluded, you
should hand over to Invigilator only the answer sheet. You are permitted to take away with you the Test
Booklet.

9. Sheet for rough work are appended in the Test Booklet at the end.

DO NOT OPEN THIS BOOKLET UNTIL YOU ARE ASKED TO DO SO


1 www.visionias.in ©Vision IAS

FREE BY KING R QUEEN P [ऋषभ राजपूत]


1. In India, how many of the following can be 3. Consider the following statements regarding
considered as public investment in the Special Economic Zones (SEZ):
agriculture? 1. FDI up to 100% is allowed through the

1. Construction of major irrigation projects automatic route for manufacturing

2. Land and soil improvement programmes activities in SEZs.

3. Interest subvention on institutional 2. Exports and supplies to SEZs are zero-

agricultural credit rated as per the IGST Act.

4. Development of cooperative institutions 3. SEZs are exempted from paying

Select the correct answer using the code Minimum Alternate Tax (MAT).
How many of the statements given above are
given below.
correct?
(a) Only one
(a) Only one
(b) Only two
(b) Only two
(c) Only three
(c) All three
(d) All four
(d) None

2. Consider the following statements regarding


4. Consider the following statements:
the fisheries sector in India during 2011-
Statement-I: The First Five Year Plan in
2021:
India emphasised on investment in heavy
Statement I: Among agricultural
industries in order to achieve
commodities, the highest growth rate is
industrialisation for rapid economic
achieved in the fishery sector.
development.
Statement II: The fisheries sector gets the
Statement-II: The First Five Year Plan in
highest input subsidy and output price
India was based on the Harrod-Domar
support among all agri-commodities.
Model.
Which one of the following is correct in Which one of the following is correct in
respect of the above statements? respect of the above statements?
(a) Both Statement-I and Statement-II are (a) Both Statement-I and Statement-II are
correct and Statement-II is the correct correct and Statement-II is the correct
explanation for Statement-I explanation for Statement-I
(b) Both Statement-I and Statement-II are (b) Both Statement-I and Statement-II are
correct and Statement-II is not the correct and Statement-II is not the
correct explanation for Statement-I correct explanation for Statement-I
(c) Statement-I is correct but Statement-II is (c) Statement-I is correct but Statement-II is
incorrect incorrect
(d) Statement-I is incorrect but Statement-II (d) Statement-I is incorrect but Statement-II

is correct is correct
2 www.visionias.in ©Vision IAS

FREE BY KING R QUEEN P [ऋषभ राजपूत]


5. In the context of India, 'Rejupave' 8. Consider the following statements regarding
technology recently seen in the news is used the e-Shram portal:
to:
1. Any worker who is aged only between
(a) correct genetic disorders
14-40 years, is eligible to register on the
(b) build humanoid robots
(c) construct roads portal.

(d) build earthquake resistant buildings 2. Agricultural labor and landless farmers

are not eligible to register on the portal.


6. With reference to insurance penetration, 3. There are no income criteria for
consider the following statements:
registering on e-Shram as an
1. It is a ratio of premium underwritten in a
unorganized worker.
given year to the Gross Domestic
Product (GDP). How many of the statements given above are

2. Insurance penetration in India has correct?


steadily increased in the last two (a) Only one
decades.
(b) Only two
3. The share of the life insurance business
(c) All three
is higher than the share of the non-life
insurance business in India. (d) None

How many of the above statements are


correct? 9. Consider the following:
(a) Only one
1. Establishment of Food Corporation of
(b) Only two
India
(c) All three
2. Launch of Jawahar Rozgar Yojana
(d) None
3. Establishment of Unit Trust of India

7. Consider the following statements regarding 4. Establishment of National Bank for


Wetland City Accreditation: Agriculture and Rural Development
1. It is an initiative under the Ramsar
5. Launch of National Programme for
Convention.
Family Planning
2. Chennai and Kolkata are the only cities
in India that have received a Wetland How many of the above developments took

City Accreditation. place during the Third Five Year Plan in


Which of the statements given above is/are India?
correct?
(a) Only two
(a) 1 only
(b) Only three
(b) 2 only
(c) Only four
(c) Both 1 and 2
(d) Neither 1 nor 2 (d) All five
3 www.visionias.in ©Vision IAS

FREE BY KING R QUEEN P [ऋषभ राजपूत]


10. Consider the following pairs: 12. Consider the following statements:
Initiative Objective Statement-I: In practice, the sum of all credit
entries is identical to the sum of all debit
1. Bima : It aims to form a women-
entries in balance of payments, and the net
Sugam centric insurance balance of all entries in the statement is zero.
distribution channel. Statement-II: In Balance of Payments,
double entry accounting system is adopted.
2. Bima : It is an online portal for all
Which one of the following is correct in
Vahak insurance-related queries.
respect of the above statements?
3. Bima : It is a social safety net (a) Both Statement-I and Statement-II are
Vistaar product targeting untapped correct and Statement-II is the correct
explanation for Statement-I.
geographies.
(b) Both Statement-I and Statement-II are
How many of the above pairs are correctly correct and Statement-II is not the
matched? correct explanation for Statement-I.
(c) Statement-I is correct but Statement-II is
(a) Only one
incorrect.
(b) Only two
(d) Statement-I is incorrect but Statement-II
(c) All three is correct.
(d) None
13. ‘Investment Facilitation Agreement’,
sometimes seen in news, is being negotiated
11. Consider the following statements regarding under:
the National Incubator Capacity (a) World Bank
Development Program (NICDP): (b) Belt and Road initiative
(c) World Trade Organization
1. The program is a joint initiative of the
(d) International Centre for Settlement of
Ministry of Commerce and the Ministry Investment Disputes
of Corporate Affairs.
14. With reference to the external sector of an
2. Under this program, an incubator need
economy, which of the following correctly
not be an incorporated/registered entity describes the term “Revaluation”?
in India. (a) A market-driven situation when
3. Under this, an incubator should have at domestic currency loses its value in front
of a foreign currency.
least 5 startups incubated in the current
(b) A government initiative in which the
or last financial year. domestic currency loses its value in front
Which of the statements given above is/are of a foreign currency.
(c) The act of raising the value of the
correct?
domestic currency against a foreign
(a) 1 and 3 only
currency.
(b) 3 only (d) A market-driven situation when
(c) 1, 2 and 3 domestic currency increases its value
against the value of a foreign currency.
(d) None
4 www.visionias.in ©Vision IAS

FREE BY KING R QUEEN P [ऋषभ राजपूत]


15. The term "Operation Prosperity Guardian" 18. Consider the following statements regarding
was recently seen in the news. It is aimed at: the Indian agriculture:
(a) preventing illegal fishing in high seas 1. India has remained a net importer of
(b) safe movement of ships in the Red Sea
agri-products since 1991 economic
(c) eradicating tax evasion
reforms.
(d) tackling plastic pollution and generating
2. The livestock sector has been the largest
wealth from it
contributor to agriculture GVA, over the

16. Consider the following statements, with last six years.


reference to the external sector of the Indian Which of the statements given above is/are
economy in 2022-23: correct?
1. The largest trading partners of India are
(a) 1 only
the USA followed by China and the
(b) 2 only
UAE.
(c) Both 1 and 2
2. Engineering Goods, Petroleum Products
(d) Neither 1 nor 2
and Gems & Jewellery were the top
three exported commodities.
Which of the statements given above is/are 19. Consider the following statements regarding
correct? the Foreign Trade Policy (FTP) 2023:
(a) 1 only 1. It aims to increase India’s overall
(b) 2 only
exports to USD 2 trillion by 2030 with
(c) Both 1 and 2
equal contributions from the
(d) Neither 1 nor 2
merchandise and services sectors.

17. Consider the following statements with 2. Battery electric vehicles of all types will

reference to the Inland Vessel Act 2021: be eligible for reduced export obligation
1. It introduced a uniform regulatory requirement under Export Promotion of
framework for inland vessel navigation Capital Goods (EPCG) Scheme.
across the country, instead of separate 3. Dairy sector will be exempted from
rules framed by the States.
maintaining average export Obligation.
2. The state government will maintain an
4. District specific export action plans will
electronic centralized record of data on
be prepared for each district of the
inland vessels.
3. The registration certificate will be valid country.

only in the respective state. How many statements given above are
How many of the above statements are correct?
correct? (a) Only one
(a) Only one (b) Only two
(b) Only two
(c) Only three
(c) All three
(d) All four
(d) None
5 www.visionias.in ©Vision IAS

FREE BY KING R QUEEN P [ऋषभ राजपूत]


20. In the context of foreign trade policy of 23. With reference to the genetically modified
India, which of the following statements best crop variety DMH-11, consider the
describes the merchanting trade?
following statements:
(a) It is a trading strategy that involves
1. It has been developed by the Indian
buying or selling securities based on
their recent strong performance. Agricultural Research Institute.
(b) It involves shipment of goods from one 2. It is a genetically modified variant of
foreign country to another foreign herbicide-tolerant rice.
country through an Indian intermediary
3. Approval of DMH-11 would make it the
but without touching Indian ports.
first GM food crop to be commercially
(c) It is a trade practice where Indian
merchant exporters act as a middlemen cultivated in India.
between foreign exporters and Indian How many of the statements given above are
buyers. correct?
(d) It is a mode of exchange in which
(a) Only one
products are purchased and processed by
(b) Only two
merchants in large quantities and sold to
end customers directly. (c) All three
(d) None
21. Which of the following is/are the major
objectives of India's Agriculture Export
24. In the context of defence sector in India,
Policy 2018?
'Ugram' which was recently seen in the news
1. To provide an institutional mechanism
for pursuing market access is a:
2. To triple India’s share in world agri (a) Submarine
exports (b) Missile
Select the correct answer using the code
(c) Rifle
given below.
(d) Tank
(a) 1 only
(b) 2 only
(c) Both 1 and 2 25. With reference to Eighth Five Year Plan (8th
(d) Neither 1 nor 2 FYP) in India, consider the following
statements:
22. Consider the following statements regarding
1. Indicative Planning was adopted in India
the Public-Private Partnership Appraisal
Committee (PPPAC): during the 8th FYP.
1. It has been set up on the 2. The actual growth rate achieved during
recommendations of the Kelkar the 8th FYP period was higher as
Committee.
compared to targeted growth rate.
2. PPP Projects worth more than 1000
Which of the statements given above is/are
crores are approved by this committee.
Which of the statements given above is/are correct?
correct? (a) 1 only
(a) 1 only (b) 2 only
(b) 2 only
(c) Both 1 and 2
(c) Both 1 and 2
(d) Neither 1 nor 2
(d) Neither 1 nor 2
6 www.visionias.in ©Vision IAS

FREE BY KING R QUEEN P [ऋषभ राजपूत]


26. Consider the following statements: 29. With reference to the Himalayan Wolf,
1. Tarapore Committee is related to capital
consider the following statements:
account convertibility of Indian Rupee.
2. Indian Rupee is fully convertible on 1. It is naturally found in the Trans-
capital account. Himalayan region only.
3. Capital account transactions are
regulated under Foreign Exchange and 2. It is classified as 'Vulnerable' on the
Regulation Act (FERA) in India. IUCN Red List.
How many of the above statements are
3. It is the only species of wolf that is
correct?
(a) Only one found in India.
(b) Only two
Which of the statements given above is/are
(c) All three
(d) None correct?

(a) 1 and 2 only


27. With reference to New Industrial Policy,
1991, consider the following statements: (b) 2 only
1. It established the Foreign Investment (c) 1 and 3 only
Promotion Board (FIPB) to promote
Foreign Direct Investment in India. (d) 1, 2 and 3

2. It reduced the number of industries


reserved for public sector from 17 to
30. Consider the following statements regarding
only two.
Which of the statements given above is/are Coal Controller Organisation:
correct?
1. It is a statutory organization set up under
(a) 1 only
(b) 2 only the Mines and Minerals (Development
(c) Both 1 and 2 and Regulation) Act.
(d) Neither 1 nor 2
2. It acts as the appellate authority in case
28. Which of the following is not correct of dispute between consumers and
regarding Minimum Support Price (MSP)?
owner arising out of declaration of grade
(a) At present more than 20 major
agricultural commodities are covered and size of coal.
under MSP.
Which of the statements given above is/are
(b) MSP has a margin of at least 50 per cent
over the all-India weighted average cost correct?
of production.
(a) 1 only
(c) MSP for pulses and oilseeds is more
than the MSP for paddy. (b) 2 only
(d) MSP is announced by the government (c) Both 1 and 2
based on the recommendations of the
NITI Aayog. (d) Neither 1 nor 2
7 www.visionias.in ©Vision IAS

FREE BY KING R QUEEN P [ऋषभ राजपूत]


31. Consider the following pairs: 33. G20 TechSprint 2023, recently in news, was:
Industrial Policy Goal (a) organised to promote innovative
1. Industrial : It emphasised on solutions aimed at improving cross-
Policy workers getting fair border payments in G20 nations.
Resolution of wages and social
(b) organized to check and control money
1948 security.
laundering using technology in G20
2. Industrial : It emphasised on
countries.
Policy heavy industries and
Resolution of and priority was given (c) organized to encourage the exchange of
1956 to industrial monetary and fiscal information among
development in G20 nations.
backward regions. (d) organized to promote economic
3. Industrial : It introduced the cooperation in areas of technology
Policy Statement Monopolies and
among the countries of G20.
of 1973 Restrictive Trade
Practices (MRTP) Act
34. Which of the following statements is not
trade.
How many of the above pairs are correctly correct regarding the trade agreements?
matched? (a) In a Preferential Trade Agreement
(a) Only one (PTA) there is a positive list of products
(b) Only two on which duty is to be reduced, whereas,
(c) All three
in a Free Trade Agreement (FTA) there
(d) None
is a negative list on which duty is not
reduced or eliminated.
32. In the context of the India Meteorological
Department (IMD), consider the following (b) In a customs union, partner countries
statements: may decide to trade at zero duty among
1. The IMD was established in the pre- themselves, however, they maintain
independence era. common tariffs against the rest of the
2. IMD is exclusively responsible for
world.
earthquake monitoring in India.
(c) The Early Harvest Scheme/Programme
3. The headquarter of IMD is located in
is a mechanism to build greater
Mumbai.
4. IMD has a mandate to warn against confidence amongst trading partners to
severe weather phenomena like tropical prepare them for bigger economic
cyclones, heavy rains and snow, cold engagement.
and heat waves. (d) An FTA covers trade in goods and
How many of the statements given above are services only, whereas, a
correct?
Comprehensive Economic Partnership
(a) Only one
Agreement (CEPA) covers goods,
(b) Only two
(c) Only three services and investment along with other
(d) All four areas including IPR, competition etc.
8 www.visionias.in ©Vision IAS

FREE BY KING R QUEEN P [ऋषभ राजपूत]


35. Consider the following statements regarding 38. Consider the following statements, with
Finance Commission : reference to the Indian aviation market:
1. It is a five member body including the 1. In the financial year 2022, the total air
Chairman. passenger traffic in India reached more
2. Members of the Commission hold office than 500 million passengers.
for a period of five years.
2. To the total airline traffic, the share of
3. It makes recommendations on principles
international traffic is higher than the
that should govern the grants-in-aid to
domestic traffic.
the states by the Centre.
3. India is the world's second-largest air
How many of the statements given above are
passenger market and fourth-largest air
correct?
(a) Only one freight market.

(b) Only two How many of the statements given above are
(c) All three correct?
(d) None (a) Only one
(b) Only two
36. Consider the following pairs : (c) All three
Report Published By (d) None
1. World : International
Employment and Labour
39. Consider the following statements regarding
Social Outlook Organization
Indian Agriculture sector (2020-21):
2. Global Risks : World Economic
1. Agriculture sector contributes more than
Report Forum
60 percent of total workforce.
3. World Economic : International
2. Income per agriculture worker is
Situation and Monetary Fund
Prospects significantly higher than the income of a

How many of the pairs given above are non-agriculture worker.


correctly matched? Which of the statements given above is/are
(a) Only one correct?
(b) Only two (a) 1 only
(c) All three (b) 2 only
(d) None (c) Both 1 and 2
(d) Neither 1 nor 2
37. The report ‘Global Water Monitor Report’,
sometimes appearing in the news, is released
40. In the context of economic planning in India,
by:
the 'Sarvodaya Plan' was drafted by
(a) One Drop Foundation
(a) M.N. Roy
(b) UNESCO World Water Assessment
(b) Acharya Vinobha Bhave
Programme
(c) World Health Organization (c) Jayaprakash Narayan

(d) None of the above (d) Jawahar Lal Nehru


9 www.visionias.in ©Vision IAS

FREE BY KING R QUEEN P [ऋषभ राजपूत]


41. Which of the following statements is not 43. Consider the following statements regarding
correct regarding the Agreement on Sovereign Gold Bond (SGB) :
Agriculture (AoA)?
1. Bonds will be redeemed in gold on
(a) It is a WTO treaty that was negotiated
maturity.
during the Uruguay Round of the
2. The maximum limit of subscription for
General Agreement on Tariffs and Trade
in 1994 at Marrakesh, Morocco. an individual is 4kg gold per year.

(b) Least developed countries do not have to Which of the statements given above is/are
make any commitments to reduce tariffs correct?
or subsidies. (a) 1 only
(c) It prohibits export subsidies on
(b) 2 only
agricultural products unless the subsidies
(c) Both 1 and 2
are specified in a member’s lists of
(d) Neither 1 nor 2
commitments.
(d) The government services such as direct
income support to farmers, assistance to 44. Consider the following statements regarding
help farmers restructure agriculture, and the Micro, Small, and Medium Enterprises
direct payments under regional
Development Act of 2006:
assistance programmes are kept under
1. An enterprise is considered a micro
Blue Box.
enterprise if its turnover is less than

42. Consider the following statements with rupees 5 crores.

respect to the Special Drawing Rights: 2. An enterprise is considered a medium


1. It is an interest-free international reserve enterprise if its investment in plant and
asset, which can be exchanged for freely
machinery is less than rupees 250 crores
usable currencies.
but greater than 50 crores.
2. Only IMF members can participate in
3. Exports are not counted in turnover for
the SDR facility.
3. It is often called paper gold as it is just a any enterprises whether micro, small, or

book entry in the Special Drawing medium.

Account of the IMF. How many of the above statements are


Which of the statements given above is/are correct?
correct?
(a) Only one
(a) 2 and 3 only
(b) Only two
(b) 1, 2 and 3
(c) All three
(c) 1 and 3 only
(d) 2 only (d) None
10 www.visionias.in ©Vision IAS

FREE BY KING R QUEEN P [ऋषभ राजपूत]


45. With reference to the international trade, 47. ‘Green Hydrogen: Enabling Measures
consider the following statements: Roadmap for Adoption in India’ sometimes
seen in the news, is a publication of which of
1. Pegging of currency is when a country
the following organization?
uses a foreign currency in lieu of its
(a) NITI Aayog
currency, mainly due to the former's (b) World Economic Forum
stability. (c) International Transport Forum
2. In the floating exchange rate system, the (d) World Bank

exchange rates are regulated by the


48. Consider the following statements, with
demand and supply of the domestic and
respect to Telecom Technology
foreign currencies in the concerned Development Fund (TTDF):
economy. 1. TTDF aims to fund R&D in urban-
3. Hard peg is an exchange rate policy in specific communication technology
which the usually market sets the applications.
2. Only Indian entities are eligible for
exchange rate, but in some cases, the
support from this fund.
central bank will intervene. 3. Universal Service Obligation Fund
How many of the above statements are (USOF) launched the TTDF Scheme.
correct? How many of the statements given above are
(a) Only one correct?
(a) Only one
(b) Only two
(b) Only two
(c) All three
(c) All three
(d) None (d) None

46. Consider the following statements, with 49. With reference to the operating ratio (OR) of
the railways, consider the following
reference to the Balance of Payments:
statements:
1. Investment income is part of the Current
Statement 1: The higher the ratio, the
account of the Balance of Payments. healthier the finances.
2. The acquisition of non-produced, non- Statement 2: The operating ratio measures
financial assets is part of the capital operating expenses against revenue.
Which one of the following is correct in
account of the Balance of Payments.
respect of the above statements?
3. Rupee Debt Service is shown under the
(a) Both Statement-1 and Statement 2 are
capital account of the Balance of correct and Statement-2 is the correct
Payments. explanation for Statement-1
Which of the statements given above are (b) Both Statement-1 and Statement-2 are
correct and Statement-2 is not the
correct?
correct explanation for Statement-1
(a) 1 only
(c) Statement-1 is correct but Statement-2 is
(b) 2 only incorrect
(c) 1, 2 and 3 (d) Statement-1 is incorrect but Statement-2
(d) 2 and 3 only is correct
11 www.visionias.in ©Vision IAS

FREE BY KING R QUEEN P [ऋषभ राजपूत]


50. The term “current Good Manufacturing 53. In the context of the government's initiatives
Practices”, recently seen in the news, is often to encourage manufacturing in India,
currently which of the following sectors is
used in the context of manufacturing of ?
not covered by the Production Linked
(a) Solar Panels
Incentive Scheme?
(b) Pharmaceutical products
(a) Manufacturing of Medical Devices
(c) Textiles (b) Construction
(d) Electronic item (c) Drones and Drone Components
(d) Specialty Steel

51. Consider the following statements, with


54. Consider the following statements about the
reference to the real effective exchange rate
Thirty Meter Telescope (TMT):
(REER) and nominal effective exchange rate
1. The project is solely funded and
(NEER): managed by the European Southern
1. The NEER is the current price Observatory.
consumers will pay to buy a foreign 2. It will be the world's largest ground-

product using their domestic currency. based X-ray telescope upon completion.
3. The telescope's mirror is designed to be
2. A higher NEER coefficient above 1
30 meters in diameter.
means that the country's currency is
4. The primary location for TMT is the
usually worth less than the imported
south pole to optimize astronomical
currency. observations.
3. An increase in a nation's REER means Which of the statements given above are not
that it is gaining trade competitiveness. correct?

How many of the above statements are (a) 1, 2, and 4 only


(b) 2 and 4 only
correct?
(c) 3 and 4 only
(a) Only one
(d) 1, 2, and 3 only
(b) Only two
(c) All three 55. In the context of economics, which of the
(d) None following is correct with respect to
Gresham's law?
(a) It relates amount of money supply in an
52. The term ‘Operation Amrith’ is sometimes
economy with inflation.
seen in the news in the context of:
(b) It relates the quality of currency (its
(a) supply of oxygen during the COVID-19
intrinsic value) to its circulation.
pandemic (c) It relates the GDP of a country to the
(b) antibiotic abuse level of employment.
(c) human trafficking (d) It relates the stability of a currency to the
polity of the country.
(d) conservation of marine biodiversity
12 www.visionias.in ©Vision IAS

FREE BY KING R QUEEN P [ऋषभ राजपूत]


56. In the context of international trade, India 59. Consider the following sectors:
has recently sought to establish a Bharat 1. Pharmaceuticals (Greenfield)
2. Food processing
Park trade zone in
3. Defence
(a) Kenya
In how many of the above sectors is FDI
(b) UAE permitted 100 percent through the automatic
(c) USA route?
(d) South Africa (a) Only one
(b) Only two
(c) All three
57. With reference to the Companies Act 2013,
(d) None
in which of the following cases do the
provisions of Corporate Social 60. With reference to sugarcane pricing policy in
Responsibility (CSR) apply to a company? India, consider the following statements:
1. Net worth of more than Rs.500 crore 1. The fair and remunerative price (FRP) of
sugarcane is announced by the Cabinet
2. Turnover of more than Rs.1000 crore
Committee on Economic Affairs.
3. Net annual profit of more than Rs.1 2. Recovery of sugar from sugarcane is one
crore of the factors considered for fixing the
Select the correct answer using the code FRP of sugarcane.
given below. 3. The State Advised Price of sugarcane is
determined by the Commission for
(a) 1 only
Agricultural Costs and Prices (CACP) in
(b) 1 and 2 only
consultation with the state governments.
(c) 2 and 3 only Which of the statements given above is/are
(d) 1, 2 and 3 correct?
(a) 1 and 2 only
(b) 2 and 3 only
58. Consider the following statements regarding
(c) 1 only
the 1991 industrial policy:
(d) 1, 2 and 3 only
1. It has abolished industrial licensing in all
the sectors. 61. Consider the following statements regarding
2. It has abolished the Monopolies and the Mines and Minerals (Regulation and
Development) Act or the MMDR Act, 1957:
Restrictive Trade Practices Act.
1. As per this Act, the mineral exploration
3. It has adopted a disinvestment policy for
license will be issued for 10 years.
the restructuring of the public sector in 2. Prospecting and mining of atomic
the country. minerals is reserved for government
How many of the above statements are entities under the Act.
Which of the statements given above is/are
correct?
correct?
(a) Only one
(a) 1 only
(b) Only two (b) 2 only
(c) All three (c) Both 1 and 2
(d) None (d) Neither 1 nor 2
13 www.visionias.in ©Vision IAS

FREE BY KING R QUEEN P [ऋषभ राजपूत]


62. With reference to the financial assistances 64. Consider the following initiatives:
provided by the International Monetary 1. NIDHI
2. SAATHI
Fund, consider the following statements:
3. PRASHAD
1. The Rapid Credit Facility is available to 4. RCS UDAN
all member countries facing an urgent How many of the above are the initiatives
taken to boost the tourism sector?
balance of payments crisis.
(a) Only one
2. The Rapid Financing Instrument is (b) Only two
available only to low-income countries (c) Only three
(d) All four
without any conditionality.
Which of the statements given above is/are 65. Which one of the following commitees is
correct? related to the disinvestment of public sector
enterprises in India after 1991 reforms?
(a) 1 only
(a) Chelliah Committee
(b) 2 only (b) Gadgil-Mukherjee Committee
(c) Both 1 and 2 (c) Suresh Tendulkar Committee
(d) Neither 1 nor 2 (d) C. Rangarajan Committee

66. Consider the following statements with


63. Consider the following statements, with reference to Bubble Baby Syndrome (BBS):
reference to the Social and Development 1. It is a genetic disorder.
2. The only treatment for BBS is a heart
Box (S & D Box) of the World Trade
transplant.
Organisation: 3. Children with BBS are extremely
1. These are “special and differential vulnerable to infectious diseases.
Which of the statements given above is/are
treatment provisions” which give
correct?
developing countries special rights to (a) 1 and 2 only
treat them more favourably. (b) 3 only
(c) 1, 2, and 3
2. The condition to use the S&D box
(d) 1 and 3 only
revolves around human development
issues such as poverty, health support, 67. In the context of labor reforms in India,
consider the following statements:
etc.
1. Code on Wages Act, 2019 stipulates
3. The eligible countries can forward such government to review minimum rates of
subsidies to the extent of less than 10 per wages at an interval not exceeding five
cent of their total agricultural output. years.
2. Currently, the National Floor Level
How many of the above statements are
Minimum Wage is 300 rupees.
correct? Which of the statements given above is/are
(a) Only one correct?
(a) 1 only
(b) Only two
(b) 2 only
(c) All three (c) Both 1 and 2
(d) None (d) Neither 1 nor 2
14 www.visionias.in ©Vision IAS

FREE BY KING R QUEEN P [ऋषभ राजपूत]


68. Consider the following statements regarding 71. Consider the following:
the automobile sector:
1. Increase in the Statutory Liquidity Ratio
1. Currently, the automobile sector
contributes more than 10 percent to (SLR) and Cash Reserve Ratio (CRR)

GDP. 2. Reduction in the number of public sector


2. 100 percent FDI is permitted in the
banks in India
automobile sector.
Which of the statements given above is/are 3. Establishment of an Asset
correct?
Reconstruction Fund
(a) 1 only
4. Regulation of interest rates by
(b) 2 only
(c) Both 1 and 2 Government
(d) Neither 1 nor 2
For the better regulation of banking sector in

69. Consider the following pairs : India, how many of the above were
GI Tag State suggested by Narsimham committee in
1. Kachchhi Kharek : Bihar
1991?
2. Lanjia Saura painting : Odisha
3. Kalonunia Rice : Bengal (a) Only one
How many of the pairs given above are
(b) Only two
correctly matched?
(a) Only one (c) Only three

(b) Only two (d) All four


(c) All three
(d) None
72. Consider the following :
70. Consider the following statements regarding
1. Cement
SBI Green Rupee Term Deposit (SGRTD)
scheme: 2. Petroleum Refinery Products

1. It seeks to mobilize funds to support 3. Automobiles


environment friendly initiatives and
4. Food processing
projects.
2. It is open to resident individuals, non- How many of the above are included in the
individuals, and NRIs. Index of Eight Core Industries (ICI)?
Which of the statements given above is/are
(a) Only one
correct?
(a) 1 only (b) Only two
(b) 2 only
(c) Only three
(c) Both 1 and 2
(d) Neither 1 nor 2 (d) All four
15 www.visionias.in ©Vision IAS

FREE BY KING R QUEEN P [ऋषभ राजपूत]


73. Consider the following statements: 75. Which of the following is not an industrial
Statement-I: A Developing country status corridor being developed by the Government
helps a country to receive preferential of India?
treatment and a longer transition period (a) Amritsar-Kolkata Industrial Corridor
before they are required to fully implement (b) Hyderabad-Warangal Industrial Corridor
World Trade Organization agreement. (c) Delhi-Nagpur Industrial Corridor
Statement-II: World Trade Organization (d) Guwahati-Kolkata Industrial Corridor
(WTO) has set criteria to define ‘developed’
and ‘developing’ countries for providing 76. With reference to Intellectual Property

various benefits. Rights, consider the following pairs:

Which one of the following is correct in Right Area Term of protection


1. Patent : 10 years and renewed
respect of the above statements?
for 10 years on
(a) Both Statement-I and Statement-II are
payment of additional
correct and Statement-II is the correct
fees.
explanation for Statement-I.
2. Geographical : 10 years
(b) Both Statement-I and Statement-II are
Indication
correct and Statement-II is not the
3. Copyrights : Authors’ lifetime + 60
correct explanation for Statement-I.
years for literary
(c) Statement-I is correct but Statement-II is
works
incorrect.
4. Trade Secrets : 100 years
(d) Statement-I is incorrect but Statement-II
How many of the above pairs are correctly
is correct.
matched?
(a) Only one
74. Consider the following statements regarding
(b) Only two
the Agriculture Infrastructure Fund (AIF): (c) Only three
1. It is a short-term debt financing facility (d) All four
for investment in agriculture.
2. Its objective is to mobilise funds for the 77. Consider the following pairs:
creation of post-harvest management Report Released by
infrastructure and community farm 1. The Future of : World Economic
assets. Jobs Report Forum
3. The scheme envisages mobilisation of 2. International Debt : International
Rs. 1 lakh crore by banks and financial Report Monetary Fund
institutions as loans for 2020-21 to 3. World Investment : World Bank
2025-26. Report
How many of the statements given above are How many of the above pairs are correctly
correct? matched?
(a) Only one (a) Only one
(b) Only two (b) Only two
(c) All three (c) All three
(d) None (d) None
16 www.visionias.in ©Vision IAS

FREE BY KING R QUEEN P [ऋषभ राजपूत]


78. Consider the following statements about the 80. Consider the following statements regarding
Shree Kalaram Temple: the peace clause at WTO:
1. It allowed developing countries to
1. The temple is located on the banks of the breach subsidy limits on food crops
river Kali in the state of Maharashtra. subject to certain conditions.
2. India became the first country to invoke
2. This temple derives its name from a
peace clause.
black statue of Lord Rama. 3. It was adopted at WTO's Bali Ministerial
3. The temple is also the site of a landmark Conference in 2013.
How many of the statements given above are
agitation led by Jyotiba Phule
correct?
demanding temple entry rights for dalits. (a) Only one
How many of the statements given above are (b) Only two
(c) All three
correct?
(d) None
(a) Only one
(b) Only two 81. Consider the following funds:
1. Rural Infrastructure Development Fund
(c) All three
2. Long Term Irrigation Fund
(d) None 3. Micro Irrigation Fund
4. Alternative Investment Funds
How many of the above funds are created or
79. With reference to ‘The Export Preparedness
operationalised under NABARD?
Index (EPI) 2022’, consider the following (a) Only one
statements: (b) Only two
(c) Only three
1. It is released by NITI Aayog in
(d) All four
association with Institute for
Competitiveness. 82. Consider the following passage:
Proba-3 is the world’s – first precision
2. It is based on four pillars – Policy,
formation flying mission. In this a pair of
Business Ecosystem, Export Ecosystem satellites will fly together maintaining a
and Export Performance. fixed configuration as a ‘large rigid
structure’ in space to prove formation flying
3. Under this index, the states are classified
technologies and rendezvous experiments. It
into three categories: coastal, is set to study the sun’s faint corona closer to
landlocked, and Union Territories/small the solar rim than has ever before been
achieved.
states.
Which of the following agencies has
Which of the statements given above are launched the above mentioned mission?
correct? (a) National Aeronautics and Space
Administration (NASA)
(a) 1 and 2 only
(b) European Space Agency (ESA)
(b) 2 and 3 only (c) Japan Aerospace Exploration Agency
(c) 1 and 3 only (JAXA)
(d) Roscosmos
(d) 1, 2 and 3
17 www.visionias.in ©Vision IAS

FREE BY KING R QUEEN P [ऋषभ राजपूत]


83. Consider the following statements with 85. Consider the following statements regarding
respect to the Mines and Minerals the National Jute Board:
(Development & Regulation) Amendment 1. It is a statutory body established under
Act, 2023: the National Jute Board Act, 2008.
1. Lithium, Beryllium, and Titanium were 2. The Headquarters of the National Jute
omitted from the list of twelve atomic Board is in Mumbai.
minerals. 3. It is under the administrative control of
2. Central Government to exclusively the Ministry of Agriculture.
auction mineral concessions for Critical How many of the statements given above are
Minerals. correct?
3. Auction premiums and other statutory (a) Only one
payments shall be received by the (b) Only two
Central Government. (c) All three
4. It allows pitting, trenching, and sub- (d) None
surface excavation as part of
reconnaissance, which was prohibited
86. Consider the following statements regarding
earlier.
Financial Intelligence Unit-India:
How many of the above statements are
1. It is a statutory body set up under the
correct?
Fugitive Economic Offenders Act.
(a) Only one
2. It reports directly to the Prime Minister's
(b) Only two
Office.
(c) Only three
Which of the statements given above is/are
(d) All four
correct?
(a) 1 only
84. With reference to Direct to Mobile (D2M)
(b) 2 only
Technology, consider the following
(c) Both 1 and 2
statements:
(d) Neither 1 nor 2
1. It does not require internet connectivity
to stream TV content directly to a
87. Consider the following:
mobile phone.
1. Uniform license for exploration and
2. It is compatible with all types of mobile
production of all forms of hydrocarbon
network generations including 2G and
2. An open acreage policy
3G.
3. Profit sharing model
3. It can only be utilized for pre-recorded
videos. 4. Marketing and pricing freedom for the

4. It is similar to the technology behind an crude oil and natural gas produced

FM radio. How many of the above are the main


Which of the statements given above is/are features of the Hydrocarbon Exploration and
correct? Licensing Policy (HELP)?
(a) 1, 2 and 4 only (a) Only one
(b) 1 and 4 only (b) Only two
(c) 3 and 4 only (c) Only three
(d) 1, 3 and 4 only (d) All four
18 www.visionias.in ©Vision IAS

FREE BY KING R QUEEN P [ऋषभ राजपूत]


88. Consider the following statements regarding 90. Consider the following statements regarding

the One Nation One Ration Card Scheme the changes in the composition of the
agriculture sector during the period 1995-96
(ONORC):
to 2020-21:
1. ONORC system enables intra-state and
1. At present, cereals constitute the largest
inter-state portability of ration cards. commodity group within the agriculture
2. Presently, national portability is enabled sector in terms of value.

in all 36 States/UT, covering 100 per 2. The share of pulses and oilseeds in the
value of total agriculture had increased
cent of the total NFSA population.
over the period.
3. Partial claim of food grains from 3. The share of Fruits and vegetables in the
multiple Fair Price Shops (FPS) is not value of total agriculture had remained
covered under the scheme. stagnant during the period.
How many of the statements given above are
Which of the statements given above are
correct?
correct?
(a) Only one
(a) 1 and 2 only (b) Only two
(b) 2 and 3 only (c) All three

(c) 1 and 3 only (d) None

(d) 1, 2 and 3
91. Poverty Reduction and Growth Trust
(PRGT) and Resilience and Sustainability
89. With reference to the National Infrastructure Trust (RST) are the lending facilities to the
Investment Fund (NIIF), Consider the low-income countries by which one of the

following statements: following?


(a) World Bank
1. NIIF is an Indian government-backed
(b) United Nations Development Program
entity established to provide long-term (c) International Monetary Fund
capital to the infrastructure sector. (d) New Development Bank
2. The Indian government has a 100 %
92. Which of the following can lead to more
stake in NIIF.
patents getting filed in India?
3. The Governing Council of NIIF is
1. Establishment of world class universities
chaired by the Prime Minister of India. 2. Cutting bureaucratic red-tape
How many of the above statements are 3. Fostering an entrepreneurial culture
correct? Select the correct answer using the code
given below.
(a) Only one
(a) 2 only
(b) Only two
(b) 1 and 2 only
(c) All three (c) 1 and 3 only
(d) None (d) 1, 2 and 3
19 www.visionias.in ©Vision IAS

FREE BY KING R QUEEN P [ऋषभ राजपूत]


93. Consider the following statements about 96. Which of the following factors have
Public Distribution System (PDS): contributed to the presence and development
1. It is operated under the joint of a large informal sector in the Indian
responsibility of the Central and the
economy?
State Governments.
1. Slow pace of labour reforms
2. It is intended to make available the
2. Low levels of skill development
entire requirement of foodgrains
distributed to any household in India. 3. Low levels of financial inclusion
Which of the statements given above is/are Select the correct answer using the code
correct? given below.
(a) 1 only (a) 1 and 3 only
(b) 2 only (b) 2 only
(c) Both 1 and 2
(c) 1 and 2 only
(d) Neither 1 nor 2
(d) 1, 2 and 3

94. With reference to International Finance


Corporation, consider the following 97. Which of the following is/are the key
statements: challenges that horticulture sector faces in
1. It is an arm of the International India?
Monetary Fund. 1. Post-harvest losses
2. It offers rupee denominated masala 2. Lack of quality planting material
bonds.
3. Lack of market access for horticulture
3. It gives interest free loans to world's
produce of small farmers
poorest countries.
Which of the statements given above is/are Select the correct answer using the code
correct? given below.
(a) 1 only (a) 1 only
(b) 2 only (b) 1 and 2 only
(c) 2 and 3 only (c) 2 and 3 only
(d) 1, 2 and 3
(d) 1, 2 and 3

95. In the context of agricultural credit, consider


98. Which among the following form basis of
the following statements:
1. Formal sources account for more than 80 international trade?
per cent share of total agricultural credit. 1. Difference in natural resources
2. The ratio of agricultural credit to 2. Size of population
agricultural GDP has steadily declined 3. Extent of foreign investment
since the economic reforms in 1990s. Select the correct answer using the code
Which of the statements given above is/are
given below.
correct?
(a) 1 and 2 only
(a) 1 only
(b) 1 and 3 only
(b) 2 only
(c) Both 1 and 2 (c) 2 and 3 only
(d) Neither 1 nor 2 (d) 1, 2 and 3

20 www.visionias.in ©Vision IAS

FREE BY KING R QUEEN P [ऋषभ राजपूत]


99. Which of the following disasdvantages is/are
associated with granting subsidies?
1. They encourage inefficiency and
wasteful use of resources.
2. They help indicate the market supply of
a good.
3. They enhance inequality between the
rich and poor.
Select the correct answer using the code
given below.
(a) 1 only
(b) 1 and 3 only
(c) 3 only
(d) 2 and 3 only

100. Which of the following is/are steps taken by


the government to improve 'Ease of Doing
Business' in India?
1. Establishment of Real Estate Investment
Trust (REITs)
2. Ratification of WTO's Trade Facilitation
Agreement
3. Implemention of Goods and Services
Tax (GST)
Select the correct answer using the code
given below.
(a) 1 and 3 only
(b) 1, 2 and 3
(c) 1 and 2 only
(d) 3 only

Copyright © by Vision IAS


All rights are reserved. No part of this document may be reproduced, stored in a retrieval system or
transmitted in any form or by any means, electronic, mechanical, photocopying, recording or otherwise,
without prior permission of Vision IAS.
21 www.visionias.in ©Vision IAS

FREE BY KING R QUEEN P [ऋषभ राजपूत]


VISIONIAS
www.visionias.in

Test Booklet Series

TEST BOOKLET

GENERAL STUDIES (P) 2024 – Test – 4148


C
Time Allowed: Two Hours Maximum Marks: 200

INSTRUCTIONS

1. IMMEDIATELY AFTER THE COMMENCEMENT OF THE EXAMINATION, YOU SHOULD CHECK THAT THIS BOOKLET
DOES NOT HAVE ANY UNPRINTED OR TURN OR MISSING PAGES OR ITEMS, ETC. IF SO, GET IT REPLACED BY A
COMPLETE TEST BOOKLET.

2. ENCODE CLEARLY THE TEST BOOKLET SERIES A, B, C OR D AS THE CASE MAY BE IN THE APPROPRIATE PLACE IN
THE ANSWER SHEET.

3. You have to enter your Roll Number on the Test Booklet in the Box
provided alongside. Do NOT write anything else on the Test Booklet.

4. This Test Booklet contains 100 items (Questions). Each item is printed in English. Each item comprises four
responses (answers). You will select the response which you want to mark on the Answer Sheet. In case you
feel that there is more than one correct response with you consider the best. In any case, choose ONLY ONE
response for each item.

5. You have to mark all your responses ONLY on the separate Answer Sheet provided. See direction in the
answers sheet.

6. All items carry equal marks. Attempt all items. Your total marks will depend only on the number of correct
responses marked by you in the answer sheet. For every incorrect response 1/3rdof the allotted marks will be
deducted.

7. Before you proceed to mark in the Answer sheet the response to various items in the Test booklet, you have to
fill in some particulars in the answer sheets as per instruction sent to you with your Admission Certificate.

8. After you have completed filling in all responses on the answer sheet and the examination has concluded, you
should hand over to Invigilator only the answer sheet. You are permitted to take away with you the Test
Booklet.

9. Sheet for rough work are appended in the Test Booklet at the end.

DO NOT OPEN THIS BOOKLET UNTIL YOU ARE ASKED TO DO SO


1 www.visionias.in ©Vision IAS

FREE BY KING R QUEEN P [ऋषभ राजपूत]


1. The lowest amount of water in the soil that a 5. Consider the following statements regarding
plant requires to maintain its turgidity is a catalyst in chemical reactions:
known as: 1. A catalyst is a substance that gets
(a) Field Capacity completely consumed in a reaction and
(b) Available Water Capacity increases the rate of the reaction.
(c) Permanent Wilting Point 2. Platinum is commonly used as a catalyst
(d) Total Soil Water Storage Capacity in chemical reactions.
Which of the statements given above is/are
2. Consider the following statements regarding correct?
bases: (a) 1 only
1. Bases often have a bitter taste. (b) 2 only
2. Bases turn red litmus paper blue. (c) Both 1 and 2
3. All bases are insouble in water. (d) Neither 1 nor 2
How many of the statements given above are
correct? 6. In the context of astronomy, what is the
(a) Only one phenomenon of gravitational lensing?
(b) Only two (a) The distortion of a planet's magnetic
(c) All three field
(d) None (b) The bending of light due to gravity,
caused by massive objects
3. What is the primary application of (c) The rotation of a galaxy around its
electrochemical machining (ECM) in
center
industry?
(d) The interaction of charged particles in a
(a) Coating surfaces with a thin layer of
nebula
metal
(b) Etching patterns on metal surfaces
7. Consider the following statements with
(c) Removing metal from surfaces through
respect to viruses:
controlled dissolution
1. A virus cannot reproduce by itself and
(d) Producing electrical insulation on
needs to enter an organism for its
conductive materials
reproduction.
2. The genetic material inside the virus
4. Arrange the melting points of the following
consists of RNA only.
metals in decreasing order:
3. A virus which infects and replicates
1. Tungsten
within a bacteria is called a viroid.
2. Titanium
4. The diseases of Herpes and Hepatitis are
3. Gold
caused by viruses.
4. Zinc
How many of the above statements are
Select the correct answer using the code
correct?
given below.
(a) Only one
(a) 1-2-3-4
(b) Only two
(b) 2-1-3-4
(c) Only three
(c) 3-1-2-4
(d) All four
(d) 1-2-4-3
2 www.visionias.in ©Vision IAS

FREE BY KING R QUEEN P [ऋषभ राजपूत]


8. Which one of the following statements best 11. Consider the following pairs with respect to
describes the purpose of 'Site Directed plant growth regulators and their functions:
Nuclease (SDN)'? Growth Function
(a) It represents a mitochondrial regulator
replacement therapy where male and 1. Auxin : Promotes cell elongation
female pronuclei are removed from each 2. Gibberlin : Induces dormancy of
fertilized egg prior to their fusing. seeds
(b) It denotes the DNA cutting enzymes 3. Cytokinins : Prevent aging of plant
used for the generation of the targeted parts
DNA breaks. 4. Ethylene : Promotes senescence
(c) It represents the latest eco-friendly and abscission of leaf
technology developed for the How many of the above pairs are correctly
remediation of oil sludge and oil spills. matched?
(d) It is a technology to efficiently extract (a) Only one
nuclear fuel from the uranium ores. (b) Only two
(c) Only three
(d) All four
9. Consider the following statements with
respect to plant physiology:
12. Which of the following applications/
1. C3 plants are twice as efficient as
phenomena are based on the Doppler effect?
C4 plants in terms of fixing carbon.
1. Echolocation by bats.
2. The direction of movement in the xylem
2. Sonic boom
is always unidirectional.
3. Police radar for measuring the speed of
3. Mycorrhizae represent a symbiotic
vehicles
association of rhizobacteria and plant
Select the correct answer using the code
roots.
given below.
How many of the statements given above are
(a) 1 and 2 only
correct?
(b) 1, 2 and 3
(a) Only one
(c) 3 only
(b) Only two
(d) 1 and 3 only
(c) Only three
(d) None 13. Semiconductors are present in most of the
modern electronic devices that we use.
10. Consider the following statements with Which of the following statements is/are true
reference to Project Cheetah: regarding semiconductors?
1. Project Cheetah has led to translocation 1. The resistivity of semiconductors is
of 20 cheetahs from the countries of more than insulators but less than
Sahel Region of Africa in Kuno National conductors.
Park. 2. The most commonly used
2. It has been implemented by National semiconductor elements are Silicon and
Tiger conservation authority. Germanium.
Which of the statements given above is/are Select the correct answer using the codes
correct? given below.
(a) 1 only (a) 1 only
(b) 2 only (b) 2 only
(c) Both 1 and 2 (c) Both 1 and 2
(d) Neither 1 nor 2 (d) Neither 1 nor 2
3 www.visionias.in ©Vision IAS

FREE BY KING R QUEEN P [ऋषभ राजपूत]


14. Consider the following statements regarding 16. With respect to Free Space Optical (FSO)
the Indian Regional Navigation Satellite communication, which of the following
statements is/are correct?
System (IRNSS):
1. It is a communication technology in
1. The IRNSS space segment consists of 10
which data is transmitted through the air.
satellites.
2. It can be used for point-to-point line of
2. IRNSS can provide accurate position sight communication.
information service in India as well as Select the correct answer using the code
regions extending up to 1500 km around given below:
(a) 1 only
India.
(b) 2 only
3. It provides a navigation solution with
(c) Both 1 and 2
position accuracy better than 20m during
(d) Neither 1 nor 2
all weather conditions anywhere within
India. 17. The terms Branching Descent and Natural
How many of the statements given above are Selection are used in the context of
(a) placing satellites in the orbit of the
correct?
Earth.
(a) Only one
(b) the theory of biological evolution.
(b) Only two
(c) evidences supporting the expanding
(c) All three universe hypothesis.
(d) None (d) strategic methods of international
diplomacy.

15. Consider the following statements with


18. Consider the following statements:
respect to TB disease:
Statement I: In India, cervical cancer is the
1. TB is an infectious disease caused by
most common cancer in women.
bacteria. Statement II: To prevent rising incidence of
2. Multi-drug-resistant tuberculosis (MDR- cervical cancer, India has introduced Human
TB) is a form of TB which is resistant to Papillomavirus (HPV) Vaccine in the
Universal Immunization Programme (UIP).
at least four of the core anti-TB drugs.
Which one of the following is correct in
3. Under the Universal Immunisation
respect of the above statements?
Programme, the BCG vaccine against (a) Both Statement-I and Statement-II are
tuberculosis is given to a child correct and Statement-II is the correct
immediately after birth. explanation for Statement-I
Which of the statements given above is/are (b) Both Statement-I and Statement-II are
correct and Statement-II is not the
correct?
correct explanation for Statement-I
(a) 1 only
(c) Statement-I is correct but Statement-II is
(b) 2 and 3 only
incorrect
(c) 1 and 3 only (d) Statement-I is incorrect but Statement-II
(d) 1, 2 and 3 is correct
4 www.visionias.in ©Vision IAS

FREE BY KING R QUEEN P [ऋषभ राजपूत]


19. Consider the following statements regarding 22. Consider the following statements regarding
cryogenic engines: the Milky Way galaxy:
1. Cryogenic engines are highly efficient
1. It is spiral in shape.
propulsion systems used in the upper
2. There is a supermassive black hole at the
stages of rockets
2. Cryogenic engine use Liquid Oxygen center of the Milky Way galaxy.
(LOX) and Liquid Hydrogen (LH2) as Which of the statements given above is/are
propellants. correct?
Which of the statements given above is/are
(a) 1 only
correct?
(b) 2 only
(a) 1 only
(b) 2 only (c) Both 1 and 2
(c) Both 1 and 2 (d) Neither 1 nor 2
(d) Neither 1 nor 2
23. Consider the following statements regarding
20. Consider the following statements with
Geostationary orbit (GEO):
respect to RNA interference (RNAi):
1. RNAi is the process of gene silencing Statement I: Satellites in GEO appear to be
initiated by a double-stranded RNA stationary over a fixed position.
(dsRNA). Statement II: Satellites in GEO circle Earth
2. RNAi takes place in all eukaryotic above the equator from east to west.
organisms as a method of cellular
Which one of the following is correct in
defense.
respect of the above statements?
3. RNAi is extensively used in creating
pest-resistant plants. (a) Both Statement-I and Statement-II are
How many of the statements given above are correct and Statement-II is the correct
correct? explanation for Statement-I
(a) Only one
(b) Both Statement-I and Statement-II are
(b) Only two
correct and Statement-II is not the
(c) All three
(d) None correct explanation for Statement-I
(c) Statement-I is correct but Statement-II is
21. Consider the following statements regarding incorrect
Small Satellite Launch Vehicle:
(d) Statement-I is incorrect but Statement-II
1. Small Satellite Launch Vehicle (SSLV)
is correct
is a single stage Launch Vehicle.
2. SSLV is capable of launching 500kg
satellite in 500km planar orbit from 24. Nitrogen gas is used in the tyres of an
Satish Dhawan Space Centre. airplane because
Which of the statements given above is/are
(a) Nitrogen gas is inert unlike air
correct?
(b) It is heavier than air
(a) 1 only
(b) 2 only (c) It forms liquid only below a very low
(c) Both 1 and 2 temperature
(d) Neither 1 nor 2 (d) Both (a) and (c)
5 www.visionias.in ©Vision IAS

FREE BY KING R QUEEN P [ऋषभ राजपूत]


25. Consider the following statements with 28. Consider the following statements:
regard to radiation: 1. A ramjet is a form of air-breathing jet

1. Transfer of heat by radiation requires a engine that uses the vehicle’s forward
motion to compress incoming air for
medium.
combustion.
2. All hot bodies radiate infrared radiation.
2. A scramjet is a type of ramjet
Which of the statements given above is/are airbreathing jet engine where
correct? combustion occurs in supersonic airflow.
(a) 1 only 3. Ramjets work most efficiently at
(b) 2 only hypersonic speeds.

(c) Both 1 and 2 How many of the statements given above are
correct?
(d) Neither 1 nor 2
(a) Only one
(b) Only two
26. What does the term "annealing" refer to in (c) All three
the context of glass manufacturing? (d) None
(a) Adding colorants to the glass
(b) Shaping the glass at high temperatures 29. Which of the following is/are white blood
cells?
(c) Rapid cooling of the glass to increase
1. Neutrophils
strength
2. Lymphocytes
(d) Slow cooling to relieve internal stresses
3. Basophils
4. Erythrocytes
27. Consider the following statements regarding Select the correct answer using the code
Hypersonic missiles: given below.
1. A hypersonic missile is a weapon system (a) 1 and 3 only
(b) 2 only
which flies at least at the speed of Mach
(c) 1, 2 and 3 only
2.
(d) 1, 2, 3 and 4
2. Hypersonic missiles cannot be
manoeuvred to the intended target. 30. The color of the sun appears reddish during
3. Conventional hypersonic weapons use sunset. Which of the following is the reason
only kinetic energy to destroy behind this phenomenon?
unhardened targets. (a) Absorption of light rays by the
atmosphere
How many of the statements given above are
(b) Human eyes are not sensitive to other
correct?
colors during that part of the day
(a) Only one
(c) Scattering of light rays by atmospheric
(b) Only two particles
(c) All three (d) The sun emits different wavelengths of
(d) None light during different times of the day
6 www.visionias.in ©Vision IAS

FREE BY KING R QUEEN P [ऋषभ राजपूत]


31. With reference to Passive Immunity, 34. Consider the following statements with
consider the following statements: reference to biocredits:
1. It is developed in a person after 1. Biocredits are similar to carbon credits
and they are designed to offset actions
receiving antibodies which are produced
with negative impacts on biodiversity.
by the immune system of another
2. To promote biocredits, the Biodiversity
individual.
Credit Alliance was launched at CoP15
2. It may be induced naturally or
of UNFCCC.
artificially. Which of the statements given above is/are
3. It provides immediate protection from a correct?
specific disease. (a) 1 only
4. It is more short-lived than active (b) 2 only
immunity. (c) Both 1 and 2

Which of the statements given above are (d) Neither 1 nor 2

correct?
35. Consider the following statements regarding
(a) 1 and 4 only
the galvanization process:
(b) 2, 3 and 4 only
1. Its primary purpose is to prevent
(c) 1, 2 and 3 only corrosion of metal surfaces.
(d) 1, 2, 3 and 4 2. Zinc is commonly used as a coating in
the Galvanization process.
32. In a nuclear fission reactor, what is the Which of the statements given above is/are
purpose of the control rods? correct?

(a) To absorb neutrons (a) 1 only


(b) 2 only
(b) To emit gamma rays
(c) Both 1 and 2
(c) To initiate fusion reactions
(d) Neither 1 nor 2
(d) To dispose radioactive waste

36. In the field of forensic science, which


33. Consider the following statements regarding chemical is commonly used to detect the
transgenic crops in India: presence of blood at crime scenes?
1. Cotton is the only transgenic crop that is (a) Phenolphthalein
being commercially cultivated in India. (b) Luminol

2. Cry2Ai is a gene found in cotton that is (c) Bromothymol blue


(d) Litmus
believed to make the crop resistant to the
pink bollworm.
37. How do electric eels generate electric shocks
Which of the statements given above is/are
for hunting and self-defense?
correct? (a) Through friction with other objects
(a) 1 only (b) By chemical reactions in their cells
(b) 2 only (c) By absorbing electrical energy from
(c) Both 1 and 2 other aquatic organisms
(d) Neither 1 nor 2 (d) Through lightning
7 www.visionias.in ©Vision IAS

FREE BY KING R QUEEN P [ऋषभ राजपूत]


38. Which polymer is used in making non-stick 43. “It is the evolution of different species in a
cookware, such as Teflon? given geographical area starting from a point
(a) Polyethylene and literally radiating to other areas of
(b) Polypropylene geography (habitats)”.
(c) Polyvinyl chloride (PVC)
Which of the following best describes the
(d) Polytetrafluoroethylene (PTFE)
above process?
(a) Adaptive Radiation
39. ISRO's Project NETRA aims to
(a) explore planets outside the solar system. (b) Convergent Evolution
(b) aid detection and mapping of ocean bed (c) Secondary Succession
resources. (d) Sequential Evolution
(c) provide early warning system in space to
detect debris and other hazards to Indian 44. With reference to phenomena of formation
satellites. of the rainbow, consider the following
(d) ease navigation in metropolitan cities. statements:
1. A rainbow is always formed in a
40. The terms 'Centimorgan' and 'Transposon'
direction opposite to that of the Sun.
are sometimes seen in the news in the
2. The sunlight undergoes refraction and
context of
dispersion in the water droplets to form
(a) Quantum physics
(b) Genetic engineering the colors of a rainbow.
(c) Artificial Intelligence Which of the statements given above is/are
(d) Nanotechnology correct?
(a) 1 only
41. How do transformers facilitate the long- (b) 2 only
distance transmission of electrical power (c) Both 1 and 2
without much loss? (d) Neither 1 nor 2
(a) By increasing voltage for efficient
transmission
45. Consider the following pairs with respect to
(b) By decreasing voltage for safety reasons
cell organelles and their functions:
(c) By increasing the electric current
(d) By converting AC to DC power Cell organelle Function
1. Ribosomes : Site for protein
42. Consider the following statements regarding synthesis
the Tea Board of India: 2. Plasma : Control movement of
1. It is a statutory body set up under Tea membrane ions and molecules
Act 1953. 3. Golgi : Modification and
2. It comes under the Ministry of apparatus sorting of proteins
Commerce and Industry. 4. Lysosomes : Cellular digestion
3. Its head office is located in the Delhi.
How many of the above pairs are correctly
Which of the statements given above is/are
matched?
correct?
(a) Only one
(a) 1 and 2 only
(b) 2 only (b) Only two
(c) 1 and 3 only (c) Only three
(d) 1, 2 and 3 (d) All four

8 www.visionias.in ©Vision IAS

FREE BY KING R QUEEN P [ऋषभ राजपूत]


46. Consider the following statements: 49. Consider the following statements regarding
1. She started the first indigenously-run various agricultural production systems:
school for girls in Pune in 1848. 1. Hydroponics is the method of growing
plants in water without soil.
2. She set up Balyata Pratibandak Gruha, a
2. Aquaponics is a food production system
childcare centre for the protection of
that combines aquaculture with
pregnant widows and rape victims. hydroponics.
3. Bavan Kashi Subodh Ratnakar ("The 3. Unlike hydroponics, nutrients come
Ocean of Pure Gems") was one of her from fish in aquaponics.
famous literary works. How many of the statements given above are
correct?
The above statements describes which of the
(a) Only one
following personalities?
(b) Only two
(a) Pandita Ramabai (c) All three
(b) Tarabai Shinde (d) None
(c) Savitribai Phule
(d) Usha Mehta 50. Consider the following statements with
reference to XPoSat mission:
1. XPoSat is the world’s first satellite-
47. Which of the following rivers flows through
based mission dedicated to making X-
Switzerland? ray polarimetry measurements.
1. Rhine 2. POLIX (Polarimeter Instrument in X-
2. Danube rays) which is the main payload will
3. Rhone serve as an X-ray Polarimeter.
3. It is placed in geostationary earth orbit
Select the correct answer using the code
which is approximately 36000 km above
given below.
the Earth equator.
(a) 1 and 3 only How many of the statements given above are
(b) 2 and 3 only correct?
(c) 1 and 2 only (a) Only one
(d) 1, 2 and 3 (b) Only two
(c) All three
(d) None
48. Consider the following statements:
1. Chloroplasts are green-colored plastids, 51. How many of the following are the benefits
which comprise pigments called of Direct Seeding of Rice (DSR)?
chlorophyll. 1. Savings on irrigation water
2. Chloroplasts are found in all green 2. Reduces cultivation time
3. Lower seed rates
plants and algae.
4. Lower GHG emissions
Which of the statements given above is/are
Select the correct answer using the code
correct? given below.
(a) 1 only (a) Only one
(b) 2 only (b) Only two
(c) Both 1 and 2 (c) Only three
(d) All four
(d) Neither 1 nor 2
9 www.visionias.in ©Vision IAS

FREE BY KING R QUEEN P [ऋषभ राजपूत]


52. Consider the following statements with 55. Consider the following statements regarding
reference to the Surrogacy (Regulation) Act, Gallium and Germanium, the rare earth
minerals, recently seen in news:
2021:
1. Both minerals can be found in Zinc ores.
1. Both altruistic and commercial
2. While Gallium is used in semiconductor
surrogacy is allowed in India. devices, Germanium finds application in
2. A woman can act as a surrogate only solar cells.
once in her lifetime. 3. India has no Gallium deposits but
Germanium is recovered as a byproduct
Which of the statements given above is/are
of alumina.
correct?
How many statements given above are
(a) 1 only correct?
(b) 2 only (a) Only one
(c) Both 1 and 2 (b) Only two
(c) All three
(d) Neither 1 nor 2
(d) None

53. What is the direction of conventional current 56. Consider the following pairs :
flow in an electrical circuit? Acid Source
(a) From positive to negative 1. Lactic acid : Curd
2. Vinegar : Methanoic acid
(b) From negative to positive
3. Tamarind : Tartaric acid
(c) It depends on the type of circuit
How many of the pairs given above are
(d) It does not have a specific direction correctly matched?
(a) Only one

54. With respect to lentils production in India, (b) Only two


(c) All three
consider the following statements:
(d) None
1. In India, Madhya Pradesh is the largest
producer of lentils. 57. Consider the following statements with
2. India is the largest producer of lentils in respect to cell division:

the world. 1. Mitotic cell division occurs in somatic


cells only and meiosis occurs in germ
3. India is the net exporter of lentils in the
cells only.
world because of surplus lentil 2. All mitotic progeny are genetically
production. similar whereas in meiosis, a new
How many of the statements given above are combination of genes is obtained.

correct? Which of the statements given above is/are


correct?
(a) Only one
(a) 1 only
(b) Only two (b) 2 only
(c) All three (c) Both 1 and 2
(d) None (d) Neither 1 nor 2
10 www.visionias.in ©Vision IAS

FREE BY KING R QUEEN P [ऋषभ राजपूत]


58. Consider the following: 61. Which of the following particles has the
1. Oxytocin least mass?
2. Serotonin
(a) Neutron
3. Endorphin
4. Dopamine (b) Proton
How many of the above are feel good (c) Electron
hormones that promote pleasure while (d) Alpha particle
reducing depression in human beings?
(a) Only one
(b) Only two 62. Consider the following statements with
(c) Only three reference to the Fast Breeder Reactor:
(d) All four
Statement-I: Fast Breeder reactors can help
extract up to 70 percent more energy than
59. Consider the following statements:
Statement-I: Maximum density of water is traditional reactors and are safer than
achieved at 4 degrees celsius. traditional reactors.
Statement-II: Specific heat of water is
Statement-II: The spent fuel from this
maximum at 4 degrees celsius.
reactor cannot be fed back into the reactor.
Which one of the following is correct in
respect of the above statements? Which one of the following is correct in
(a) Both Statement-I and Statement-II are respect of the above statements?
correct and Statement-II is the correct
(a) Both Statement-I and Statement-II are
explanation for Statement-I
(b) Both Statement-I and Statement-II are correct and Statement-II is the correct

correct and Statement-II is not the explanation for Statement-I


correct explanation for Statement-I (b) Both Statement-I and Statement-II are
(c) Statement-I is correct but Statement-II is
correct and Statement-II is not the
incorrect
(d) Statement-I is incorrect but Statement-II correct explanation for Statement-I
is correct (c) Statement-I is correct but Statement-II is
incorrect
60. Consider the following statements regarding
(d) Statement-I is incorrect but Statement-II
radiocarbon dating:
1. Radioactive isotope commonly used in is correct
radiocarbon dating is C-14.
2. It can measure age of fossils up to 100
63. Stripe Rust is a common disease of which
million years old.
plant?
Which of the statements given above is/are
correct? (a) Rice
(a) 1 only (b) Wheat
(b) 2 only
(c) Maize
(c) Both 1 and 2
(d) Potato
(d) Neither 1 nor 2
11 www.visionias.in ©Vision IAS

FREE BY KING R QUEEN P [ऋषभ राजपूत]


64. Daily, as the sun dawns on your landscape, 67. A person stands on loose sand and then lies
the crop management system takes charge. down on it. He goes deeper into the sand
Soil sensors embedded in the fields transmit
while standing than while lying down.
real-time data to the central hub. The
algorithm analyzed moisture, nutrient levels, Which one of the following is the reason?
and weather forecasts. Automated irrigation (a) The weight of the body is lesser while
systems are activated, precisely delivering standing up.
water. Drones soared, mapping crop health
(b) The density of the human body is lesser
and detecting pests. The system ensured
than sand
optimal conditions, maximizing yield and
minimizing environmental impact. (c) The pressure on the sand is higher while
Which of the following technologies is best standing up
described in the passage given above? (d) There is more friction between the body
(a) Machine Learning
and the sand while lying down
(b) Virtual private network
(c) Internet of Things
(d) Remote Sensing 68. Which carbon-based compound is
commonly used as an antifreeze in
65. Consider the following:
automobile radiators to prevent the coolant
1. Camphor
2. Cellulose from freezing?
3. Iodine (a) Acetone
How many of the above compounds undergo (b) Butanol
sublimation that is transitioning directly (c) Isopropyl alcohol
from the solid phase to the gas phase without
(d) Ethylene glycol
passing through the liquid phase?
(a) Only one
(b) Only two 69. Consider the following statements with
(c) All three respect to India - Maldives bilateral relation:
(d) None
1. Both India and Maldives are members of

66. Consider the following statements regarding the regional grouping SAARC and
Carbon Nanotubes (CNTs): BIMSTEC.
1. CNTs can be coated on cotton yarns of 2. The number of Indian tourists to
clothes to power wearable devices.
Maldives has doubled between 2018 to
2. CNTs can be useful in carbon capture
2022.
and storage technologies.
3. The easy uptake of CNTs in different 3. Great Male Connectivity Project, one of
cell types makes them significant in drug the largest infrastructure project of
and gene delivery. Maldives is funded by India.
4. CNTs have no toxic effects making them
Which of the statements given above is/are
the most sought in various industries.
How many statements were given above are correct?
correct? (a) 1 and 2 only
(a) Only one (b) 2 and 3 only
(b) Only two
(c) 3 only
(c) Only three
(d) All four (d) 1, 2 and 3

12 www.visionias.in ©Vision IAS

FREE BY KING R QUEEN P [ऋषभ राजपूत]


70. With reference to Havisure, consider the 73. Consider the following:
following statements:
1. 3D printing
1. It is India’s first indigenously developed
Hepatitis A vaccine. 2. Quantum computing
2. It has been developed by the Serum
3. Detection of contamination of food
Institute of India.
3. It is a single-dose vaccine. How many of the areas/fields mentioned
Which of the statements given above is/are
above is nanotechnology used?
correct?
(a) 1 and 2 only (a) Only one
(b) 2 and 3 only
(c) 1 and 3 only (b) Only two

(d) 1 only (c) All three

71. When a potted plant is over-watered for a (d) None


few days, it gradually dies because:
(a) plant is infected by soil microbes.
(b) roots do not receive oxygen for 74. With reference to the Critical Tiger Habitats
respiration.
(CTH), consider the following statements:
(c) roots are unable to absorb nutrients.
(d) water from soil cannot rise to aerial 1. Nagarjunsagar-Srisailam Tiger Reserve
parts.
has the largest area under Critical Tiger

72. Consider the following statements regarding Habitat.


nano fertilizers:
2. As of now, more than 50% of total tiger
Statement I: Nano Urea increases the
availability of nitrogen to crops by more reserves area is under Critical Tiger
than 80% resulting in higher Nutrient Use
Habitat.
efficiency.
Statement II: Compared to conventional urea 3. The customary and traditional forest
prill, nano Urea has less surface area.
rights under Forest Rights Act, 2006 are
Which one of the following is correct in
respect of the above statements? not recognised in Critical Tiger Habitat.
(a) Both Statement-I and Statement-II are
correct and Statement-II is the correct How many of the statements given above are

explanation for Statement-I correct?


(b) Both Statement-I and Statement-II are
correct and Statement-II is not the (a) Only one
correct explanation for Statement-I (b) Only two
(c) Statement-I is correct but Statement-II is
incorrect (c) All three
(d) Statement-I is incorrect but Statement-II
(d) None
is correct
13 www.visionias.in ©Vision IAS

FREE BY KING R QUEEN P [ऋषभ राजपूत]


75. In the context of gravitational force and 77. Consider the following statements regarding
electromagnetic force, consider the the Zeeman Effect :
1. It describes the change in frequency or
following statements:
wavelength of a wave in relation to an
1. While gravitational force is always
observer moving relative to the source of
attractive, electromagnetic force can be the wave.
both attractive and repulsive. 2. Its principles are used in the working of

2. While gravitational force is only Magnetic resonance imaging (MRI)


scanners.
dependent on mass, the electromagnetic
Which of the statements given above is/are
force is dependent on both mass and correct?
charge. (a) 1 only
3. While gravitational force is dependent (b) 2 only
(c) Both 1 and 2
upon the distance between the two
(d) Neither 1 nor 2
bodies, the electromagnetic force is
independent of it. 78. Isotopes have the same atomic number but
Which of the statements given above is/are different atomic masses. Which among the
correct? following are the uses of radioactive
isotopes?
(a) 1 only
1. Treatment of cancer
(b) 2 and 3 only
2. Fuel in nuclear reactors
(c) 1 and 3 only 3. Treatment of goitre
(d) 1, 2 and 3 Select the correct answer using the code
given below.
(a) 1 and 2 only
76. Consider the following statements regarding
(b) 1 and 3 only
the Photoelectric effect : (c) 2 and 3 only
1. It is a phenomenon where electrons are (d) 1, 2 and 3
absorbed when a material is exposed to
79. Arrange the reactivity of the following
light.
metals in decreasing order:
2. Solar panels utilize the photoelectric
1. Sodium
effect to convert sunlight into electrical 2. Zinc
energy. 3. Iron
Which of the statements given above is/are 4. Mercury
Select the correct answer using the code
correct?
given below.
(a) 1 only
(a) 1-2-3-4
(b) 2 only (b) 1-2-4-3
(c) Both 1 and 2 (c) 2-1-3-4

(d) Neither 1 nor 2 (d) 2-1-4-3


14 www.visionias.in ©Vision IAS

FREE BY KING R QUEEN P [ऋषभ राजपूत]


80. With reference to International Court of 82. Consider the following statements with
reference to indigenously manufactured
Justice, consider the following statements:
Unmanned Aerial Vehicle (UAV) 'Drishti
1. In order to be elected, a candidate must 10':
receive an absolute majority by the 1. It is developed by Hindustan
Aeronautics Limited.
voting done by the United Nations
2. It is entirely built with 100%
General Assembly and the Security indigenously made components.
3. It is a medium-altitude, low-endurance
Council.
UAV.
2. A member of the Court can be dismissed How many of the statements given above are
only with the unanimous opinion of the correct?
(a) Only one
other members.
(b) Only two
3. Both United Nations bodies and (c) All three
individual countries can take part in the (d) None

advisory proceedings of ICJ. 83. Consider the following:


Which of the statements given above is/are 1. Melting ice cream
2. Evaporation of water
correct?
3. Baking a cake
(a) 1 and 2 only In the context of chemistry, how many of the
(b) 2 only above are chemical changes?
(a) Only one
(c) 3 only (b) Only two
(d) 2 and 3 only (c) All three
(d) None

81. In the context of rising drone attack on 84. In the context of food preservation, salt
commercial vessel in Red Sea, consider the inhibits bacterial growth primarily by:
(a) increasing pH.
following statements regarding Red Sea: (b) decreasing pH.
1. The Red Sea lies in the tropical belt. (c) adding nutrients.
(d) absorbing water.
2. Israel through its Port of Eilat has access

to the Red Sea via Gulf of Aqaba. 85. Consider the following statements:
3. Gulf of Suez connects the Red Sea and 1. Hydrogenation increases the melting
point of fats.
Mediterranean Sea.
2. The hydrogenation process is used to
How many of the statements given above are prepare vegetable ghee from vegetable
oil.
correct?
Which of the statements given above is/are
(a) Only one correct?
(b) Only two (a) 1 only
(b) 2 only
(c) All three
(c) Both 1 and 2
(d) None (d) Neither 1 nor 2
15 www.visionias.in ©Vision IAS

FREE BY KING R QUEEN P [ऋषभ राजपूत]


86. In the context of Indian defence, consider the 88. DNA is the molecule that carries genetic
following pairs: information for the development and
functioning of an organism. Consider the
Missiles Description
following statements with respect to DNA:
1. Rudram : Long-range guided-bomb 1. DNA molecule is a double helix
2. Gaurav : Anti-radiation missile consisting of two strands of DNA linked
by ionic bonds.
3. BrahMos- : Supersonic cruise missile
2. In a DNA molecule, there are four
NG nucleotides present in equal amounts.
How many of the above pairs are correctly 3. Among the four bases in DNA, Adenine
always pairs with Guanine and Cytosine
matched?
always pairs with Thymine.
(a) Only one How many of the statements given above are
(b) Only two correct?
(c) All three (a) Only one
(b) Only two
(d) None
(c) Only three
(d) None
87. In the context of recent judgement of the
Supreme Court in Bilkis Bano case, consider 89. Vegetative Propagation, a form of asexual
reproduction can be achieved by which of
the following statements regarding the
the following parts of a plant?
Supreme Court's power to review its own 1. Roots
order or judgement. 2. Leaves

1. Article 142 of the Constitution of India 3. Stem


Select the correct answer using the code
gives Supreme Court the power to
given below.
review any of its own order or (a) 3 only
judgement. (b) 1 and 2 only
(c) 2 and 3 only
2. Only the parties to the case can seek a
(d) 1, 2 and 3
review of the judgement on it.
3. The review petition is heard only by a 90. Consider the following parts of a cell:
larger and distinct bench to uphold the 1. Centrosome
2. Plastids
principle of natural justice.
3. Cell wall
How many pairs given above are correctly
4. Vacuoles
matched? How many of the above given parts of a cell
(a) Only one are common to both plants and animal cells?
(a) Only one
(b) Only two
(b) Only two
(c) All three (c) Only three
(d) None (d) All four
16 www.visionias.in ©Vision IAS

FREE BY KING R QUEEN P [ऋषभ राजपूत]


91. Consider the following statements : 94. How does sunscreen protect your skin from
Statement-I : Longer surf boards are more the sun?
stable while maneuvering in water.
(a) By increasing perspiration
Statement-II : Longer surf boards have more
volume and surface area interacting with (b) Absorbing and scattering UV rays
water. (c) Increasing melanin production
Which one of the following is correct in
(d) Generating a cooling effect
respect of the above statements?
(a) Both Statement-I and Statement-II are
correct and Statement-II is the correct 95. With reference to the properties of the
explanation for Statement-I Graphene, consider the following
(b) Both Statement-I and Statement-II are
statements:
correct and Statement-II is not the
correct explanation for Statement-I 1. It is a two-dimensional form of carbon.
(c) Statement-I is correct but Statement-II is 2. It consists of a multi layers of carbon
incorrect
atoms arranged in a hexagonal lattice.
(d) Statement-I is incorrect but Statement-II
is correct 3. It is prepared by heating Graphite, Coal
tar and or Shellac.
92. Consider the following:
Which of the statements given above is/are
1. Detection of cancer
2. Reduction in occurrences of wildfire correct?
3. Writing poetry (a) 1, 2 and 3
4. Identification and classification of (b) 2 and 3 only
celestial objects
(c) 1 and 3 only
With the present state of development, how
many of the above-mentioned things are (d) 1 only
possible through Artificial Intelligence?
(a) Only one
96. Consider the following statements regarding
(b) Only two
(c) Only three the speed of sound in a different medium:
(d) All four 1. Among the three states solid, liquid and
gaseous, sound travels fastest in the
93. Consider the following statements regarding
gaseous state.
the Bharat Ratna Award:
1. It is India’s highest civilian award. 2. Sound travels faster in hot air than in
2. It is restricted to maximum of three in a cold air.
particular year.
Which of the statements given above is/are
3. It is a title under Article 18.
Which of the statements given above is/are correct?
correct? (a) 1 only
(a) 3 only (b) 2 only
(b) 2 and 3 only
(c) Both 1 and 2
(c) 1 and 2 only
(d) 1, 2 and 3 (d) Neither 1 nor 2

17 www.visionias.in ©Vision IAS

FREE BY KING R QUEEN P [ऋषभ राजपूत]


97. Consider the following statements regarding 99. In which of the following is the fission mode
polar bear: of reproduction observed?
1. Polar bear is both the largest bear and 1. Bacteria
the largest terrestrial carnivore in the 2. Virus
world. 3. Protozoa
2. The polar bear is primarily found in the Select the correct answer using the code
Arctic and Antarctic regions. given below.
3. The International Union for (a) 1 and 2 only
Conservation of Nature (IUCN) (b) 1 and 3 only
designates the polar bear as Endangered (c) 2 and 3 only
on its Red List. (d) 1, 2 and 3
How many of the statements given above are
correct? 100. Why does cutting an onion make one cry?
(a) Only one (a) Release of tear gas
(b) Only two (b) Release of sulfur compounds
(c) All three (c) Acidic reaction
(d) None (d) Water absorption

98. Arrange the above planets in decreasing


order of number of satellites:
1. Jupiter
2. Uranus
3. Mars
4. Neptune
Select the correct answer using the code
given below.
(a) 1-2-3-4
(b) 1-2-4-3
(c) 2-1-3-4
(d) 2-1-4-3

Copyright © by Vision IAS


All rights are reserved. No part of this document may be reproduced, stored in a retrieval system or
transmitted in any form or by any means, electronic, mechanical, photocopying, recording or otherwise,
without prior permission of Vision IAS.

18 www.visionias.in ©Vision IAS

FREE BY KING R QUEEN P [ऋषभ राजपूत]

You might also like